Download as pdf or txt
Download as pdf or txt
You are on page 1of 749

Study Guide of Surgery for

Medical Students
(module 4)
O.O. Bogomolets National Medical University
Surgery Department #2

Boris Bezrodny, Igor Kolosovych, Valeriy Teplyy, Roman Sydorenko,


Oleg Petrenko

Study Guide of Surgery


for Medical Students
(module 4)

Рекомендовано Центральним методичним кабінетом з вищої медичної освіти


МОЗ України як навчальний посібник для англомовних студентів вищих меди-
чних навчальних закладів ІV рівня акредитації (протокол № 2 від 26.10.2011 р.
засідання науково-методичної Комісії з медицини Міністерства освіти і науки,
молоді та спорту України)

Kyiv
2012
УДК 616-089.1(075.8)
ББK 54.5 я 73
S 90

Study Guide of Surgery for Medical Students (module 4)/ Boris Bezrodny, Igor
Kolosovych, Valeriy Teplyy, Roman Sydorenko, Oleg Petrenko/ Edited by Prof. Bo-
ris Bezrodny. - Kyiv, 2012. - 328 p.

The content of the textbook covers the new educational programme of surgery.
The textbook devotes to preparing to the 4-th module and includes the themes; gen-
eral questions of surgical aid in Ukraine, systemic surgical pathology, pathology of
the digestive, cardiovascular and respiratory systems. The structure of material is pre-
sented with attention on the basic part, terminology, signs, methods of diagnosis, tac-
tics, methods of conservative and surgical treatment, basic and additional literature,
tests for initial and final level of knowledge, tasks and materials for the self-study of
the students for each lesson. In the authors’ opinion, such structure of the textbook is
beneficial for mastering the subject.
For the foreign students of 6-th course of medical schools of III-IV accredita-
tion level.

Readers: F.Glumcher
Head of Anesthesiology and Intensive Therapy Department
Professor
O.O. Bogomolets National Medical University, Kyiv

L. Avrаkhova
Head of Foreign Languages Department
Associate Professor
O.O. Bogomolets National Medical University, Kyiv
Study guide #1.1
“Historical relationship between surgery and medicine.”

Overview
It remains a rhetorical question whether an understanding of surgical history is
important to the maturation and continued education and training of a surgeon. Con-
versely, it is hardly necessary to dwell on the heuristic value that an appreciation of
history provides in developing adjunctive humanistic, literary, and philosophic tastes.
Clearly, the study of medicine is a lifelong learning process that should be an enjoya-
ble and rewarding experience. For a surgeon, the study of surgical history can con-
tribute toward making this educational effort more pleasurable and can provide con-
stant invigoration. Tracing the evolution of what one does on a daily basis and under-
standing it from a historical perspective become enviable goals. In reality, there is no
way to separate present-day surgery and one's own clinical practice from the experi-
ence of all surgeons and all the years that have gone before. For budding surgeons, it
is a magnificent adventure to appreciate what they are currently learning within the
context of past and present cultural, economic, political, and social institutions. Ac-
tive practitioners will find that study of the profession, as it rightly must, with all as-
pects of the human condition-affords an excellent opportunity to approach current
clinical concepts in ways not previously appreciated.
In studying our profession's past, it is certainly easier to relate to the history of
so-called modern surgery over the past 100 or so years than to the seemingly primi-
tive practices of previous periods because the closer to the present, the more likely it
is that surgical practices will resemble those of nowadays. Nonetheless, writing the
history of modern surgery is in many respects more difficult than describing the de-
velopment of surgery before the late 19th century. One significant reason for this dif-
ficulty is the ever-increasing pace of scientific development in conjunction with unre-
lenting fragmentation (i.e., specialization and subspecialization) within the profes-
sion. The craft of surgery is in constant flux, and the more rapid the change, the more
difficult it is to obtain a satisfactory historical perspective. Only the lengthy passage
of time permits a truly valid historical analysis.
Educational aims:
Study of the basic stages of development of medicine and surgery in the world.
A student must know:
1. Human anatomy.
2. Pathophysiologic basis of surgical diseases
3. Methods of controlling hemorrhage and maintaining intraoperative hemostasis
4. The basic stages of development of anesthesiology, methods of anesthesia.
5. Methods of antiseptic and aseptic.
6. Stages of development of ccardiac surgery and organ transplantation
7. Ethics in surgery
Content:

Despite outward appearances, it was actually not until the latter decades of the
19th century that the surgeon truly emerged as a specialist within the whole of medi-
cine to become a recognized and respected clinical practitioner. Similarly, it was not
until the first decades of the 20th century that surgery could be considered to have
achieved the status of a bona fide profession. Before this time, the scope of surgery
remained quite limited. Surgeons, or at least those medical men who used the sobri-
quet surgeon, whether university educated or trained in private apprenticeships, at
best treated only simple fractures, dislocations, and abscesses and occasionally per-
formed amputations with dexterity but also with high mortality rates. They managed
to ligate major arteries for common and accessible aneurysms and made heroic at-
tempts to excise external tumors. Some individuals focused on the treatment of anal
fistulas, hernias, cataracts, and bladder stones. Inept attempts at reduction of incarcer-
ated and strangulated hernias were made, and hesitatingly, rather rudimentary colos-
tomies or ileostomies were created by simply incising the skin over an expanding in-
tra-abdominal mass, which represented the end stage of a long-standing intestinal ob-
struction. Compound fractures of the limbs with attendant sepsis remained mostly
unmanageable, with staggering morbidity being a likely surgical outcome. Although a
few bold surgeons endeavored to incise the abdomen in the hope of dividing obstruct-
ing bands and adhesions, abdominal and other intrabody surgery was virtually un-
known.

Despite it all, including an ignorance of anesthesia and antisepsis tempered with


the not uncommon result of the patient suffering from or succumbing to the effects of
a surgical operation (or both), surgery was long considered an important and medical-
ly valid therapy. This seeming paradox, in view of the terrifying nature of surgical in-
tervention, its limited technical scope, and its damning consequences before the de-
velopment of modern conditions, is explained by the simple fact that surgical proce-
dures were usually performed only for external difficulties that required an objective
anatomic diagnosis. Surgeons or followers of the surgical cause saw what needed to
be fixed (e.g., abscesses, broken bones, bulging tumors, cataracts, hernias) and would
treat the problem in as rational a manner as the times permitted. Conversely, the phy-
sician was forced to render subjective care for disease processes that were neither vis-
ible nor understood. After all, it is a difficult task to treat the symptoms of illnesses
such as arthritis, asthma, heart failure, and diabetes, to name but a few, if there is no
scientific understanding or internal knowledge of what constitutes their basic patho-
logic and physiologic underpinnings.

With the breathtaking advances made in pathologic anatomy and experimental


physiology during the 18th and the first part of the 19th centuries, physicians would
soon adopt a therapeutic viewpoint that had long been prevalent among surgeons. It
was no longer a question of just treating symptoms; the actual pathologic problem
could ultimately be understood. Internal disease processes that manifested themselves
through difficult-to-treat external signs and symptoms were finally described via
physiology-based experimentation or viewed pathologically through the lens of a mi-
croscope. Because this reorientation of internal medicine occurred within a relatively
short time and brought about such dramatic results in the classification, diagnosis,
and treatment of disease, the rapid ascent of mid-19th century internal medicine
might seem more impressive than the agonizingly slow, but steady advance of sur-
gery. In a seeming contradiction of mid-19th century scientific and social reality,
medicine appeared as the more progressive branch, with surgery lagging behind. The
art and craft of surgery, for all its practical possibilities, would be severely restricted
until the discovery of anesthesia in 1846 and an understanding and acceptance of the
need for surgical antisepsis and asepsis during the 1870s and 1880s. Still, surgeons
never needed a diagnostic and pathologic revolution in the manner of the physician.
Despite the imperfection of their scientific knowledge, the pre–modern era surgeon
did cure with some technical confidence.
That the gradual evolution of surgery was superseded in the 1880s and 1890s by
the rapid introduction of startling new technical advances was based on a simple cul-
minating axiom - the four fundamental clinical prerequisites that were required before
a surgical operation could ever be considered a truly viable therapeutic procedure had
finally been identified and understood:
1. Knowledge of human anatomy

2. Method of controlling hemorrhage and maintaining intraoperative hemostasis


3. Anesthesia to permit the performance of pain-free procedures
4. Explanation of the nature of infection along with the elaboration of methods
necessary to achieve an antiseptic and aseptic operating room environment

The first two prerequisites were essentially solved in the 16th century, but the
latter two would not be fully resolved until the ending decades of the 19th century. In
turn, the ascent of 20th century scientific surgery would unify the profession and al-
low what had always been an art and craft to become a learned vocation. Standard-
ized postgraduate surgical education and training programs could be established to
help produce a cadre of scientifically knowledgeable practitioners. Moreover, in a fi-
nal snub to an unscientific past, newly established basic surgical research laboratories
offered the means of proving or disproving the latest theories while providing a test-
ing ground for bold and exciting clinical breakthroughs.

KNOWLEDGE OF HUMAN ANATOMY

Few individuals have had an influence on the history of surgery as overwhelm-


ingly as that of the Brussels-born Andreas Vesalius (1514-1564). As professor of
anatomy and surgery in Padua, Italy, Vesalius taught that human anatomy could be
learned only through the study of structures revealed by human dissection. In particu-
lar, his great anatomic treatise De Humani Corporis Fabrica Libri Septem (1543)
provided fuller and more detailed descriptions of human anatomy than any of his il-
lustrious predecessors did. Most importantly, Vesalius corrected errors in traditional
anatomic teachings propagated 13 centuries earlier by Greek and Roman authorities,
whose findings were based on animal rather than human dissection. Even more radi-
cal was Vesalius blunt assertion that anatomic dissection must be completed by phy-
sician/surgeons themselves—a direct renunciation of the long-standing doctrine that
dissection was a grisly and loathsome task to be performed by a diener-like individual
(autopsy technician) while from on high the perched physician/surgeon lectured by
reading from an orthodox anatomic text. This principle of hands-on education would
remain Vesalius' most important and long-lasting contribution to the teaching of anat-
omy. Vesalius' Latin literae scriptae ensured its accessibility to the most well-known
physicians and scientists of the day. Latin was the language of the intelligentsia and
the Fabrica became instantly popular, so it was only natural that over the next 2 cen-
turies the work would go through numerous adaptations, editions, and revisions,
though always remaining an authoritative anatomic text.

METHOD OF CONTROLLING HEMORRHAGE

The position of Ambroise Paré (1510-1590) in the evolution of surgery remains


of supreme importance. He played the major role in reinvigorating and updating Re-
naissance surgery and represents severing of the final link between surgical thought
and techniques of the ancients and the push toward more modern eras. From 1536 un-
til just before his death, Paré was either engaged as an army surgeon, during which he
accompanied different French armies on their military expeditions, or performing
surgery in civilian practice in Paris. Although other surgeons made similar observa-
tions about the difficulties and nonsensical aspects of using boiling oil as a means of
cauterizing fresh gunshot wounds, Paré's use of a less irritating emollient of egg yolk,
rose oil, and turpentine brought him lasting fame and glory. His ability to articulate
such a finding in multiple textbooks, all written in the vernacular, allowed his writ-
ings to reach more than just the educated elite. Among Paré's important corollary ob-
servations was that when performing an amputation, it was more efficacious to ligate
individual blood vessels than to attempt to control hemorrhage by means of mass li-
gation of tissue or with hot oleum. Described in his Dix Livres de la Chirurgie avec le
Magasin des Instruments Necessaires à Icelle (1564), the free or cut end of a blood
vessel was doubly ligated and the ligature was allowed to remain undisturbed in situ
until, as a result of local suppuration, it was cast off. Paré humbly attributed his suc-
cess with patients to God, as noted in his famous motto, “Je le pansay. Dieu le
guérit,” that is, “I treated him. God cured him.”
PATHOPHYSIOLOGIC BASIS OF SURGICAL DISEASES

Although it would be another 3 centuries before the third desideratum, that of


anesthesia, was discovered, much of the scientific understanding concerning efforts to
relieve discomfort secondary to surgical operations was based on the 18th century
work of England's premier surgical scientist, John Hunter (1728-1793) Considered
one of the most influential surgeons of all time, his endeavors stand out because of
the prolificacy of his written word and the quality of his research, especially in using
experimental animal surgery as a way to understand the pathophysiologic basis of
surgical diseases. Most impressively, Hunter relied little on the theories of past au-
thorities but rather on personal observations, with his fundamental pathologic studies
first described in the renowned textbook A Treatise on the Blood, Inflammation, and
Gun-Shot Wounds (1794). Ultimately, his voluminous research and clinical work re-
sulted in a collection of more than 13,000 specimens, which became one of his most
important legacies to the world of surgery. It represented a unique warehousing of
separate organ systems, with comparisons of these systems, from the simplest animal
or plant to humans, demonstrating the interaction of structure and function. For dec-
ades, Hunter's collection, housed in England's Royal College of Surgeons, remained
the outstanding museum of comparative anatomy and pathology in the world. That
was until a World War II Nazi bombing attack of London created a conflagration that
destroyed most of Hunter's assemblage.

ANESTHESIA

Since time immemorial, the inability of surgeons to complete pain-free opera-


tions had been among the most terrifying of medical problems. In the preanesthetic
era, surgeons were forced to be more concerned about the speed with which an opera-
tion was completed than with the clinical efficacy of their dissection. In a similar
vein, patients refused or delayed surgical procedures for as long as possible to avoid
the personal horror of experiencing the surgeon's knife. Analgesic, narcotic, and sop-
orific agents such as hashish, mandrake, and opium had been put to use for thousands
of years. However, the systematic operative invasion of body cavities and the inevita-
ble progression of surgical history could not occur until an effective means of render-
ing a patient insensitive to pain was developed.
As anatomic knowledge and surgical techniques improved, the search for safe
methods to prevent pain became more pressing. By the early 1830s, chloroform,
ether, and nitrous oxide had been discovered and so-called laughing gas parties and
ether frolics were in vogue, especially in America. Young people were amusing
themselves with the pleasant side effects of these compounds as itinerant so-called
professors of chemistry travelled to hamlets, towns, and cities to lecture on and
demonstrate the exhilarating effects of these new gases. It soon became evident to
various physicians and dentists that the pain-relieving qualities of ether and nitrous
oxide could be applicable to surgical operations and tooth extraction. On October 16,
1846, William T. G. Morton (1819-1868), a Boston dentist, persuaded John Collins
Warren (1778-1856), professor of surgery at the Massachusetts General Hospital, to
let him administer sulfuric ether to a surgical patient from whom Warren went on to
painlessly remove a small, congenital vascular tumor of the neck. After the operation,
Warren, greatly impressed with the new discovery, uttered his famous words: “Gen-
tlemen, this is no humbug.”

Few medical discoveries have been so readily accepted as inhalational anesthe-


sia. News of the momentous event spread rapidly throughout the United States and
Europe, and a new era in the history of surgery had begun. Within a few months after
the first public demonstration in Boston, ether was used in hospitals throughout the
world. Yet no matter how much it contributed to the relief of pain during surgical op-
erations and decreased the surgeon's angst, the discovery did not immediately further
the scope of elective surgery. Such technical triumphs awaited the recognition and
acceptance of antisepsis and asepsis. Anesthesia helped make the illusion of surgical
cures more seductive, but it could not bring forth the final prerequisite: all-important
hygienic reforms.

Still, by the mid-19th century, both doctors and patients were coming to hold
surgery in relatively high regard for its pragmatic appeal, technologic virtuosity, and
unambiguously measurable results. After all, surgery appeared to some a mystical
craft. To be allowed to consensually cut into another human's body, to gaze at the
depth of that person's suffering, and to excise the demon of disease seemed an awe-
some responsibility. Yet it was this very mysticism, long associated with religious
overtones, that so fascinated the public and their own feared but inevitable date with a
surgeon's knife. Surgeons had finally begun to view themselves as combining art and
nature, essentially assisting nature in its continual process of destruction and rebuild-
ing. This regard for the natural would spring from the eventual, though preternatural-
ly slow, understanding and use of Joseph Lister's (1827-1912) techniques

ANTISEPSIS, ASEPSIS, AND UNDERSTANDING THE NATURE OF


INFECTION

In many respects, the recognition of antisepsis and asepsis was a more important
event in the evolution of surgical history than the advent of inhalational anesthesia
was. There was no arguing that deadening of pain permitted a surgical operation to be
conducted in a more efficacious manner. Haste was no longer of prime concern.
However, if anesthesia had never been conceived, a surgical procedure could still be
performed, albeit with much difficulty. Such was not the case with listerism. Without
antisepsis and asepsis, major surgical operations more than likely ended in death ra-
ther than just pain. Clearly, surgery needed both anesthesia and antisepsis, but in
terms of overall importance, antisepsis proved to be of greater singular impact.

In the long evolution of world surgery, the contributions of several individuals


stand out as being preeminent. Lister, an English surgeon, can be placed on such a se-
lect list because of his monumental efforts to introduce systematic, scientifically
based antisepsis in the treatment of wounds and the performance of surgical opera-
tions. He pragmatically applied others' research into fermentation and microorgan-
isms to the world of surgery by devising a means of preventing surgical infection and
securing its adoption by a skeptical profession.

It was evident to Lister that a method of destroying bacteria by excessive heat


could not be applied to a surgical patient. He turned, instead, to chemical antisepsis
and, after experimenting with zinc chloride and the sulfites, decided on carbolic acid.
By 1865, Lister was instilling pure carbolic acid into wounds and onto dressings. He
would eventually make numerous modifications in the technique of dressings, the
manner of applying and retaining them, and the choice of antiseptic solutions of vary-
ing concentrations. Although the carbolic acid spray remains the best remembered of
his many contributions, it was eventually abandoned in favor of other germicidal sub-
stances. Lister not only used carbolic acid in the wound and on dressings but also
went so far as to spray it in the atmosphere around the operative field and table. He
did not emphasize hand scrubbing but merely dipped his fingers into a solution of
phenol and corrosive sublimate. Lister was incorrectly convinced that scrubbing cre-
ated crevices in the palms of the hands where bacteria would proliferate. A second
important advance by Lister was the development of sterile absorbable sutures. He
believed that much of the deep suppuration found in wounds was created by previous-
ly contaminated silk ligatures. Lister evolved a carbolized catgut suture that was bet-
ter than any previously produced. He was able to cut the ends of the ligature short,
thereby closing the wound tightly, and eliminate the necessity of bringing the ends of
the suture out through the incision, a surgical practice that had persisted since the
days of Paré.

The acceptance of listerism was an uneven and distinctly slow process, for many
reasons. First, the various procedural changes that Lister made during the evolution of
his methodology created confusion. Second, listerism, as a technical exercise, was
complicated with the use of carbolic acid, an unpleasant and time-consuming nui-
sance. Third, various early attempts to use antisepsis in surgery had proved abject
failures, with many leading surgeons unable to replicate Lister's generally good re-
sults. Finally and most important, acceptance of listerism depended entirely on an un-
derstanding and ultimate recognition of the veracity of the germ theory, a hypothesis
that many practical-minded surgeons were loath to accept.

As a professional group, German-speaking surgeons would be the first to grasp


the importance of bacteriology and the germ theory. Consequently, they were among
the earliest to expand on Lister's message of antisepsis, with his spray being discarded
in favor of boiling and use of the autoclave. The availability of heat sterilization en-
gendered sterile aprons, drapes, instruments, and sutures. Similarly, the use of face-
masks, gloves, hats, and operating gowns also naturally evolved. By the mid-1890s,
less clumsy aseptic techniques had found their way into most European surgical am-
phitheaters and were approaching total acceptance by American surgeons. Any lin-
gering doubts about the validity and significance of the momentous concepts that Lis-
ter had put forth were eliminated on the battlefields of World War I. There, the im-
portance of just plain antisepsis became an invaluable lesson for scalpel bearers,
whereas the exigencies of the battlefield helped bring about the final maturation and
equitable standing of surgery and surgeons within the worldwide medical community.
In Russia, methods of aseptic and antiseptic were introduced by N. Pirogov
(1810-1871).
X-RAYS

Especially prominent among other late 19th century discoveries that had an
enormous impact on the evolution of surgery was research conducted by Pulyui Ivan
(Ukrainian scientist, 1845-1918) and Wilhelm Roentgen (1845-1923), which led to
his 1895 elucidation of x-rays. Having grown interested in the phosphorescence from
metallic salts that were exposed to light, Roentgen made a chance observation when
passing a current through a vacuum tube and noticed a greenish glow coming from a
screen on a shelf 9 feet away. This strange effect continued after the current was
turned off. He found that the screen had been painted with a phosphorescent sub-
stance. Proceeding with full experimental vigor, Roentgen soon realized that there
were invisible rays capable of passing through solid objects made of wood, metal, and
other materials. Most significant, these rays also penetrated the soft parts of the body
in such a manner that the more dense bones of his hand were able to be revealed on a
specially treated photographic plate. In a short time, numerous applications were de-
veloped as surgeons rapidly applied the new discovery to the diagnosis and location
of fractures and dislocations and the removal of foreign bodies.

TURN OF THE 20TH CENTURY

By the late 1890s, the interactions of political, scientific, socioeconomic, and


technical factors set the stage for what would become a spectacular showcasing of
surgery's newfound prestige and accomplishments. Surgeons were finally wearing an-
tiseptic-looking white coats. Patients and tables were draped in white, and basins for
bathing instruments in bichloride solution abounded. Suddenly all was clean and tidy,
with conduct of the surgical operation no longer a haphazard affair. This reformation
would be successful not because surgeons had fundamentally changed but because
medicine and its relationship to scientific inquiry had been irrevocably altered. Sec-
tarianism and quackery, the consequences of earlier medical dogmatism, would no
longer be tenable within the confines of scientific truth.

With all four fundamental clinical prerequisites in place by the turn of the centu-
ry and highlighted with the emerging clinical triumphs of various English surgeons,
including Robert Tait (1845-1899), William Macewen (1848-1924), and Frederick
Treves (1853-1923); German-speaking surgeons, among whom were Theodor Bill-
roth (1829-1894) ( Fig. 1-5 ), Theodor Kocher (1841-1917) ( Fig. 1-6 ), Friedrich
Trendelenburg (1844-1924), and Johann von Mikulicz-Radecki (1850-1905); French
surgeons, including Jules Peán (1830-1898), Just Lucas-Championière (1843-1913),
and Marin-Theodore Tuffiér (1857-1929); the Italians, most notably Eduardo Bassini
(1844-1924) and Antonio Ceci (1852-1920); American surgeons, exemplified by Wil-
liam Williams Keen (1837-1932), Nicholas Senn (1844-1908), and John Benjamin
Murphy (1857-1916) and Russian surgeons N. Pirogov (1810-1871), І. Bujalskyi
(1789-1864), J. Charukivsky (1798-1848), P. Shumljanskyi (1750-1824), М. Elinskyi
(1789-1834), V. Karavaev (1811-1892), M. Sklyfosovskyi (1836-1904), P. Peleh
(1842-1917), А. Pidriz (1852-1900), M. Volkovich (1858-1928), M. Trinkler (1859-
1925) scalpel wielders had essentially explored all cavities of the human body. None-
theless, surgeons retained a lingering sense of professional and social discomfort and
continued to be pejoratively described by nouveau scientific physicians as nonthink-
ers who worked in little more than an inferior and crude manual craft.

It was becoming increasingly evident that research models, theoretical concepts,


and valid clinical applications would be necessary to demonstrate the scientific basis
of surgery to a wary public. The effort to devise new operative methods called for an
even greater reliance on experimental surgery and absolute encouragement of it by all
concerned parties. Most importantly, a scientific basis for therapeutic surgical rec-
ommendations—consisting of empirical data, collected and analyzed according to na-
tionally and internationally accepted rules and set apart from individual authoritative
assumptions—would have to be developed. In contrast to previously unexplainable
doctrines, scientific research would triumph as the final arbiter between valid and in-
valid surgical therapies.

In turn, surgeons had no choice but to allay society's fear of the surgical un-
known by presenting surgery as an accepted part of a newly established medical ar-
mamentarium. This would not be an easy task. The immediate consequences of surgi-
cal operations, such as discomfort and associated complications, were often of more
concern to patients than was the positive knowledge that an operation could eliminate
potentially devastating disease processes. Accordingly, the most consequential
achievement by surgeons during the early 20th century was ensuring the social ac-
ceptability of surgery as a legitimate scientific endeavor and the surgical operation as
a therapeutic necessity.

ASCENT OF SCIENTIFIC SURGERY

William Stewart Halsted (1852-1922) more than any other surgeon, set the sci-
entific tone for this most important period in surgical history. He moved surgery from
the melodramatics of the 19th century operating theater to the starkness and sterility
of the modern operating room, commingled with the privacy and soberness of the re-
search laboratory. As professor of surgery at the newly opened Johns Hopkins Hospi-
tal and School of Medicine, Halsted proved to be a complex personality, but the im-
pact of this aloof and reticent man would become widespread. He introduced a new
surgery and showed that research based on anatomic, pathologic, and physiologic
principles and the use of animal experimentation made it possible to develop sophis-
ticated operative procedures and perform them clinically with outstanding results.
Halsted proved, to an often leery profession and public, that an unambiguous se-
quence could be constructed from the laboratory of basic surgical research to the clin-
ical operating room. Most importantly, for surgery's own self-respect, he demonstrat-
ed during this turn-of-the-century renaissance in medical education that departments
of surgery could command a faculty whose stature was equal in importance and pres-
tige to that of other more academic or research-oriented fields such as anatomy, bac-
teriology, biochemistry, internal medicine, pathology, and physiology.

As a single individual, Halsted developed and disseminated a different system of


surgery so characteristic that it was referred to as a school of surgery. More to the
point, Halsted's methods revolutionized the world of surgery and earned his work the
epithet halstedian principles, which remains a widely acknowledged and accepted
scientific imprimatur. Halsted subordinated technical brilliance and speed of dissec-
tion to a meticulous and safe, albeit sometimes slow performance. As a direct result,
Halsted's effort did much to bring about surgery's self-sustaining transformation from
therapeutic subservience to clinical necessity.

Despite his demeanor as a professional recluse, Halsted's clinical and research


achievements were overwhelming in number and scope. His residency system of
training surgeons was not merely the first such program of its kind; it was unique in
its primary purpose. Above all other concerns, Halsted desired to establish a school of
surgery that would eventually disseminate throughout the surgical world the princi-
ples and attributes that he considered sound and proper. His aim was to train able sur-
gical teachers, not merely competent operating surgeons. There is little doubt that
Halsted achieved his stated goal of producing “not only surgeons but surgeons of the
highest type, men who will stimulate the first youth of our country to study surgery
and to devote their energies and their lives to raising the standards of surgical sci-
ence.” So fundamental were his contributions that without them, surgery might never
have fully developed and could have remained mired in a quasi-professional state.

The heroic and dangerous nature of surgery seemed appealing in less scientifi-
cally sophisticated times, but now, surgeons were courted for personal attributes be-
yond their unmitigated technical boldness. A trend toward hospital-based surgery was
increasingly evident, owing in equal parts to new, technically demanding operations
and to modern hospital physical structures within which surgeons could work more
effectively. The increasing complexity and effectiveness of aseptic surgery, the diag-
nostic necessity of the x-ray and clinical laboratory, the convenience of 24-hour nurs-
ing, and the availability of capable surgical residents living within a hospital were
making the hospital operating room the most plausible and convenient place for a
surgical operation to be performed.

It was obvious to both hospital superintendents and the whole of medicine that
acute care institutions were becoming a necessity more for the surgeon than for the
physician. As a consequence, increasing numbers of hospitals went to great lengths to
supply their surgical staffs with the finest facilities in which to complete operations.
For centuries, surgical operations had been performed under the illumination of sun-
light or candles, or both. Now, however, electric lights installed in operating rooms
offered a far more reliable and unwavering source of illumination. Surgery became a
more proficient craft because surgical operations could be completed on stormy
summer mornings, as well as on wet winter afternoons.

INTERNATIONALIZATION, SURGICAL SOCIETIES, AND JOURNALS

As the sophistication of surgery grew, internationalization became one of its un-


derlying themes, with surgeons crossing the great oceans to visit and learn from one
another. Halsted and Hermann Küttner (1870-1932), director of the surgical clinic in
Breslau, Germany (now known as Wroclaw and located in southwestern Poland), in-
stituted the first known official exchange of surgical residents in 1914. This experi-
ment in surgical education was meant to underscore the true international spirit that
had engulfed surgery. Halsted firmly believed that young surgeons achieved greater
clinical maturity by observing the practice of surgery in other countries, as well as in
their own.

An inevitable formation of national and international surgical societies and the


emergence and development of periodicals devoted to surgical subjects proved to be
important adjuncts to the professionalization process of surgery. For the most part,
professional societies began as a method of providing mutual improvement via per-
sonal interaction with surgical peers and the publication of presented papers. Unlike
surgeons of earlier centuries, who were known to closely guard so-called trade se-
crets, members of these new organizations were emphatic about publishing transac-
tions of their meetings. In this way, not only would their surgical peers read of their
clinical accomplishments, but a written record was also established for circulation
throughout the world of medicine.

The first of these surgical societies was the Académie Royale de Chirurgie in
Paris, with its Mémoires appearing sporadically from 1743 through 1838. Of 19th
century associations, the most prominent published proceedings were the Mémoires
and Bulletins of the Société de Chirurgie of Paris (1847), the Verhandlungen of the
Deutsche Gesellschaft für Chirurgie (1872), and the Transactions of the American
Surgical Association (1883). No surgical association that published professional re-
ports existed in 19th century Great Britain, and the Royal Colleges of Surgeons of
England, Ireland, and Scotland never undertook such projects. Although textbooks,
monographs, and treatises had always been the mainstay of medical writing, the in-
troduction of monthly journals, including August Richter's (1742-1812) Chirurgische
Bibliothek (1771), Joseph Malgaigne's (1806-1865) Journal de Chirurgie (1843),
Bernard Langenbeck's (1810-1887) Archiv für Klinische Chirurgie (1860), and Lewis
Pilcher's (1844-1917) Annals of Surgery (1885), had a tremendous impact on updat-
ing and continuing the education of surgeons.

WORLD WAR I

Austria-Hungary, Germany and Russia continued as the dominating forces in


world surgery until World War I. However, results of the conflict proved disastrous to
the central powers (Austria-Hungary, Bulgaria, Germany, and the Ottoman Empire),
especially to German-speaking surgeons. Europe took on a new social and political
look, with the demise of Germany's status as the world leader in surgery a sad but
foregone conclusion. As with most armed conflicts, because of the massive human
toll, especially battlefield injuries, tremendous strides were made in multiple areas of
surgery. Undoubtedly, the greatest surgical achievement was in the treatment of
wound infection. Trench warfare in soil contaminated by decades of cultivation and
animal manure made every wounded soldier a potential carrier of any number of
pathogenic bacilli. On the battlefront, sepsis was inevitable. Most attempts to main-
tain aseptic technique proved inadequate, but the treatment of infected wounds by an-
tisepsis was becoming a pragmatic reality.

Surgeons experimented with numerous antiseptic solutions and various types of


surgical dressing. A principle of wound treatment entailing debridement and irriga-
tion eventually evolved. Henry Dakin (1880-1952), an English chemist, and Alexis
Carrel (1873-1944) the Nobel prize-winning French American surgeon, were the
principal protagonists in the development of this extensive system of wound man-
agement. In addition to successes in wound sterility, surgical advances were made in
the use of x-rays in the diagnosis of battlefield injuries, and remarkable operative in-
genuity was evident in reconstructive facial surgery and the treatment of fractures re-
sulting from gunshot wounds.
AMERICAN COLLEGE OF SURGEONS

For American surgeons, the years just before World War I were a time of active
coalescence into various social and educational organizations. The most important
and influential of these societies was the American College of Surgeons, founded by
Franklin Martin (1857-1935), a Chicago-based gynecologist, in 1913. Patterned after
the Royal Colleges of Surgeons of England, Ireland, and Scotland, the American Col-
lege of Surgeons established professional, ethical, and moral standards for every
graduate in medicine who practiced in surgery and conferred the designation Fellow
of the American College of Surgeons (FACS) on its members. From the outset, its
primary aim was the continuing education of surgical practitioners. Accordingly, the
requirements for fellowship were always related to the educational opportunities of
the period. In 1914, an applicant had to be a licensed graduate of medicine, receive
the backing of three fellows, and be endorsed by the local credentials committee.

In view of the stipulated peer recommendations, many practitioners, realistically


or not, viewed the American College of Surgeons as an elitist organization. With an
obvious so-called blackball system built into the membership requirements, there was
a difficult-to-deny belief that many surgeons who were immigrants, females, or
members of particular religious and racial minorities were granted fellowships spar-
ingly. Such inherent bias, in addition to questionable accusations of fee splitting
along with unbridled contempt of certain surgeons' business practices, resulted in
some very prominent American surgeons never being permitted the privilege of
membership.

The 1920s and beyond proved to be a prosperous time for American society and
its surgeons. After all, the history of world surgery in the 20th century is more a tale
of American triumphs than it ever was in the 18th or 19th centuries. Physicians' in-
comes dramatically increased and surgeons' prestige, aided by the ever-mounting suc-
cesses of medical science, became securely established in American culture. Still, a
noticeable lack of standards and regulations in surgical specialty practice became a
serious concern to leaders in the profession. The difficulties of World War I had
greatly accentuated this realistic need for specialty standards when many of the phy-
sicians who were self-proclaimed surgical specialists were found to be unqualified by
military examining boards. In ophthalmology, for example, more than 50% of tested
individuals were deemed unfit to treat diseases of the eye.

It was an unmistakable reality that there were no established criteria with which
to distinguish a well-qualified ophthalmologist from an upstart optometrist or to clari-
fy the differences in clinical expertise between a well-trained, full-time ophthalmo-
logic specialist and an inadequately trained, part-time general practition-
er/ophthalmologist. In recognition of the gravity of the situation, the self-patrolling
concept of a professional examining board, sponsored by leading voluntary ophthal-
mologic organizations, was proposed as a mechanism for certifying competency. In
1916, uniform standards and regulations were set forth in the form of minimal educa-
tional requirements and written and oral examinations, and the American Board for
Ophthalmic Examinations, the country's first, was formally incorporated. By 1940,
six additional surgical specialty boards were established, including orthopedic (1934),
colon and rectal (1934), urologic (1935), plastic (1937), surgical (1937), and neuro-
logic (1940).

As order was introduced into surgical specialty training and the process of certi-
fication matured, it was apparent that the continued growth of residency programs
carried important implications for the future structure of medical practice and the so-
cial relationship of medicine to overall society. Professional power had been consoli-
dated, and specialization, which had been evolving since the time of the Civil War,
was now recognized as an essential, if not integral part of modern medicine. Although
the creation of surgical specialty boards was justified under the broad imprimatur of
raising the educational status and evaluating the clinical competency of specialists,
board certification undeniably began to restrict entry into the specialties.

As the specialties evolved, the political influence and cultural authority enjoyed
by the profession of surgery were growing. This socioeconomic strength was most
prominently expressed in reform efforts directed toward the modernization and stand-
ardization of America's hospital system. Any vestiges of so-called kitchen surgery
had essentially disappeared, and other than numerous small private hospitals predom-
inantly constructed by surgeons for their personal use, the only facilities where major
surgery could be adequately conducted and postoperative patients appropriately cared
for were the well-equipped and physically impressive modern hospitals. For this rea-
son, the American College of Surgeons and its expanding list of fellows had a strong
motive to ensure that America's hospital system was as up to date and efficient as
possible.

On an international level, surgeons were confronted with the lack of any formal
organizational body. Not until the International College of Surgeons was founded in
1935 in Geneva would such a society exist. At its inception, this organization was in-
tended to serve as a liaison to the existing colleges and surgical societies in the vari-
ous countries of the world. However, its goals of elevating the art and science of sur-
gery, creating greater understanding among the surgeons of the world, and affording a
means of international postgraduate study never came to full fruition, in part because
the American College of Surgeons adamantly opposed the establishment—and con-
tinues to do so-of a viable American chapter of the International College of Surgeons.

MODERN ERA

Despite the global economic depression in the aftermath of World War I, the
1920s and 1930s signaled the ascent of American surgery to its current position of in-
ternational leadership. Highlighted by educational reforms in its medical schools,
Halsted's redefinition of surgical residency programs, and the growth of surgical spe-
cialties, the stage was set for the blossoming of scientific surgery. Basic surgical re-
search became an established reality as George Crile (1864-1943), Alfred Blalock
(1899-1964), Dallas Phemister (1882-1951), and Charles Huggins (1901-1997) be-
came world-renowned surgeon-scientists.

Much as the ascendancy of the surgeon-scientist brought about changes in the


way in which the public and the profession viewed surgical research, the introduction
of increasingly sophisticated technologies had an enormous impact on the practice of
surgery. Throughout the evolution of surgery, the practice of surgery—the art, the
craft, and finally, the science of working with one's hands—had largely been defined
by its tools. From the crude flint instruments of ancient peoples, through the simple
tonsillotomes and lithotrites of the 19th century, up to the increasingly complex sur-
gical instruments developed in the 20th century, new and improved instruments usu-
ally led to a better surgical result. Progress in surgical instrumentation and surgical
techniques went hand in hand.

Surgical techniques would, of course, become more sophisticated with the pas-
sage of time, but by the conclusion of World War II, essentially all organs and areas
of the body had been fully explored. In fact, within a short half-century the domain of
surgery had become so well established that the profession's foundation of basic op-
erative procedures was already completed. As a consequence, there were few tech-
nical surgical mysteries left. What surgery now needed to sustain its continued
growth was the ability to diagnose surgical diseases at earlier stages, to locate malig-
nant growths while they remained small, and to have more effective postoperative
treatment so that patients could survive ever more technically complex operations.
Such thinking was exemplified by the introduction in 1924 of cholecystography by
Evarts Graham (1883-1957) and Warren Cole (1898-1990). In this case, an emerging
scientific technology introduced new possibilities into surgical practice that were not
necessarily related solely to improvements in technique. To the surgeon, the discov-
ery and application of cholecystography proved most important, not only because it
brought about more accurate diagnoses of cholecystitis but also because it created an
influx of surgical patients where few had previously existed. If surgery was to grow,
large numbers of individuals with surgical diseases were needed.

It was an exciting era for surgeons, with important clinical advances being made
both in the operating room and in the basic science laboratory. Among the most nota-
ble highlights were the introduction in 1935 of pancreaticoduodenectomy for cancer
of the pancreas by Allen Oldfather Whipple (1881-1963) and a report in 1943 on va-
gotomy for operative treatment of peptic ulcer disease by Lester Dragstedt (1893-
1976). Frank Lahey (1880-1953) stressed the importance of identifying the recurrent
laryngeal nerve during the course of thyroid surgery; Owen Wangensteen (1898-
1981) successfully decompressed mechanical bowel obstructions by using a newly
devised suction apparatus in 1932; George Vaughan (1859-1948) successfully ligated
the abdominal aorta for aneurysmal disease in 1921; Max Peet (1885-1949) presented
his splanchnic resection for hypertension in 1935; Walter Dandy (1886-1946) per-
formed intracranial section of various cranial nerves in the 1920s; Walter Freeman
(1895-1972) described prefrontal lobotomy as a means of treating various mental ill-
nesses in 1936; Harvey Cushing (1869-1939) introduced electrocoagulation in neuro-
surgery in 1928; Marius Smith-Petersen (1886-1953) described a flanged nail for pin-
ning a fracture of the neck of the femur in 1931 and introduced Vitallium cup arthro-
plasty in 1939; Vilray Blair (1871-1955) and James Brown (1899-1971) popularized
the use of split-skin grafts to cover large areas of granulating wounds; Earl Padgett
(1893-1946) devised an operative dermatome that allowed calibration of the thickness
of skin grafts in 1939; Elliott Cutler (1888-1947) performed a successful section of
the mitral valve for relief of mitral stenosis in 1923; Evarts Graham completed the
first successful removal of an entire lung for cancer in 1933; Claude Beck (1894-
1971) implanted pectoral muscle into the pericardium and attached a pedicled
omental graft to the surface of the heart, thus providing collateral circulation to that
organ, in 1935; Robert Gross (1905-1988) reported the first successful ligation of a
patent arterial duct in 1939 and resection for coarctation of the aorta with direct anas-
tomosis of the remaining ends in 1945; and John Alexander (1891-1954) resected a
saccular aneurysm of the thoracic aorta in 1944.

With such a wide variety of technically complex surgical operations now possi-
ble, it had clearly become impossible for any single surgeon to master all the manual
skills as well as the pathophysiologic knowledge necessary to perform such cases.
Therefore, by the middle of the century, a consolidation of professional power inher-
ent in the movement toward specialization, with numerous individuals restricting
their surgical practice to one highly structured field, had become among the most sig-
nificant and dominating events in 20th century surgery. Ironically, the United States,
which had been much slower than European countries to recognize surgeons as a dis-
tinct group of clinicians separate from physicians, would now spearhead this move
toward surgical specialization with great alacrity. Clearly, the course of surgical
fragmentation into specialties and subspecialties was gathering tremendous speed as
the dark clouds of World War II settled over the globe. The socioeconomic and polit-
ical ramifications of this war would bring about a fundamental change in the way that
surgeons viewed themselves and their interactions with the society in which they
lived and worked.
LAST HALF OF THE 20TH CENTURY
The decades of economic expansion after World War II had a dramatic impact
on surgery's scale, particularly in the United States. It was as though being victorious
in battle permitted medicine to become big business overnight, with the single-
minded pursuit of health care rapidly transformed into society's largest growth indus-
try. Spacious hospital complexes were built that not only represented the scientific
advancement of the healing arts but also vividly demonstrated the strength of Ameri-
can's postwar socioeconomic boom. Society was willing to give surgical science un-
precedented recognition as a prized national asset.
The overwhelming impact of World War II on surgery was the sudden expan-
sion of the profession and the beginnings of an extensive distribution of surgeons
throughout the country. Many of these individuals, newly baptized to the rigors of
technically complex trauma operations, became leaders in the construction and im-
provement of hospitals, multispecialty clinics, and surgical facilities in their
hometowns. Large urban and community hospitals established surgical education and
training programs and found it a relatively easy matter to attract interns and residents.
For the first time, residency programs in general surgery were rivaled in growth and
educational sophistication by those in all the special fields of surgery. These changes
served as fodder for further increases in the number of students entering surgery. Not
only would surgeons command the highest salaries, but society was also enamored of
the drama of the operating room. Television series, movies, novels, and the more-
than-occasional live performance of a heart operation on network broadcast beckoned
the lay individual.
Despite lay approval, success and acceptability in the biomedical sciences are
sometimes difficult to determine, but one measure of both in recent times has been
awarding of the Nobel Prize in medicine and physiology. Society's continued appro-
bation of surgery's accomplishments is seen in the naming of nine surgeons as Nobel
laureates.
Surgeons Named Nobel Laureates in Medicine and Physiology
SURGEON COUNTRY FIELD (year of award)
Theodor Kocher (1841-1917) Switzerland Thyroid disease (1909)
Allvar Gullstrand (1862-1930) Sweden Ocular dioptrics (1911)
Alexis Carrel (1873-1944) France and United Vascular surgery (1912)
States
Robert Bárány (1876-1936) Austria Vestibular disease (1914)
Frederick Banting (1891-1941) Canada Insulin (1922)
Walter Hess (1881-1973) Switzerland Midbrain physiology (1949)
Werner Forssmann (1904- Germany Cardiac catheterization
1979) (1956)
Charles Huggins (1901-1997) United States Oncology (1966)
Joseph Murray (1919-) United States Organ transplantation (1990)

CARDIAC SURGERY AND ORGAN TRANSPLANTATION

Two clinical developments truly epitomized the magnificence of post–World


War II surgery and concurrently fascinated the public: the maturation of cardiac sur-
gery as a new surgical specialty and the emergence of organ transplantation. Togeth-
er, they would stand as signposts along the new surgical highway. Fascination with
the heart goes far beyond that of clinical medicine. From the historical perspective of
art, customs, literature, philosophy, religion, and science, the heart has represented
the seat of the soul and the wellspring of life itself. Such reverence also meant that
this noble organ was long considered a surgical untouchable. Whereas the late 19th
and 20th centuries witnessed a steady march of surgical triumphs in opening succes-
sive cavities of the body, the final achievement awaited the perfection of methods for
surgical operations in the thoracic space.

Such a scientific and technologic accomplishment can be traced back to the re-
pair of cardiac stab wounds by direct suture and the earliest attempts at fixing faulty
heart valves. As triumphant as Luther Hill's (1862-1946) first known successful su-
ture of a wound that penetrated a cardiac chamber was in 1902, it would not be until
the 1940s that the development of safe intrapleural surgery could be counted on as
something other than an occasional event. During World War II, Dwight Harken
(1910-1993) gained extensive battlefield experience in removing bullets and shrapnel
in or in relation to the heart and great vessels without a single fatality. Building on his
wartime experience, Harken and other pioneering surgeons, including Charles Bailey
(1910-1993) of Philadelphia and Russell Brock (1903-1980) of London, proceeded to
expand intracardiac surgery by developing operations for the relief of mitral valve
stenosis. The procedure was progressively refined and evolved into the open commis-
surotomy repair used today.

Despite mounting clinical successes, surgeons who operated on the heart had to
contend not only with the quagmire of blood flowing through an area where difficult
dissection was taking place but also with the unrelenting to-and-fro movement of a
beating heart. Technically complex cardiac repair procedures could not be developed
further until these problems were solved. John Gibbon (1903-1973) addressed this
enigma by devising a machine that would take on the work of the heart and lungs
while the patient was under anesthesia, in essence pumping oxygen-rich blood
through the circulatory system while bypassing the heart so that the organ could be
operated on at leisure. The first successful open heart operation in 1953, conducted
with the use of a heart-lung machine, was a momentous surgical contribution.
Through single-mindedness of purpose, Gibbon's research paved the way for all fu-
ture cardiac surgery, including procedures for correction of congenital heart defects,
repair of heart valves, and transplantation of the heart.

Since time immemorial, the focus of surgery was mostly on excision and repair.
However, beginning in the 20th century, the opposite end of the surgical spectrum—
reconstruction and transplantation—became realities. Nineteenth century experience
had shown that skin and bone tissues could be autotransplanted from one site to an-
other in the same patient. It would take the horrendous and mutilating injuries of
World War I to decisively advance skin transplants and legitimize the concept of sur-
gery as a method of reconstruction. With Harold Gillies (1882-1960) of England and
America's Vilray Blair establishing military-based plastic surgery units to deal with
complex maxillofacial injuries, a turning point in the way in which society viewed
surgery's raison d’être occurred. Now, not only would surgeons enhance nature's
healing powers, but they could also dramatically alter what had previously been little
more than one's physical foregone conclusion. For example, Hippolyte Morestin
(1869-1919) described a method of mammaplasty in 1902. John Staige Davis (1872-
1946) of Baltimore popularized a manner of splinting skin grafts and later wrote the
first comprehensive textbook on this new specialty, Plastic Surgery: Its Principles
and Practice (1919). Immediately after the war, Blair would go on to establish the
first separate plastic surgery service in a civilian institution at Barnes Hospital in St.
Louis. Vladimir Filatov (1875-1956) of Odessa, Russia, used a tubed pedicle flap in
1916, and in the following year, Gillies introduced a similar technique.

What about the replacement of damaged or diseased organs? After all, even at
the midpoint of the century, the very thought of successfully transplanting worn-out
or unhealthy body parts verged on scientific fantasy. At the beginning of the 20th
century, Alexis Carrel developed revolutionary new suturing techniques to anasto-
mose the smallest of blood vessels. Using his surgical élan on experimental animals,
Carrel began to transplant kidneys, hearts, and spleens. Technically, his research was
a success, but some unknown biologic process always led to rejection of the trans-
planted organ and death of the animal. By the middle of the century, medical re-
searchers had begun to clarify the presence of underlying defensive immune reactions
and the necessity of creating immunosuppression as a method to allow the host to ac-
cept the foreign transplant. Using high-powered immunosuppressant drugs and other
modern modalities, kidney transplantation soon blazed the way, and it was not long
before a slew of organs and

POLITICAL AND SOCIOECONOMIC INFLUENCES

Despite the 1950s and 1960s witnessing some of the most magnificent advances
in the history of surgery, by the 1970s, political and socioeconomic influences were
starting to overshadow many of the clinical triumphs. It was the beginning of a schiz-
ophrenic existence for surgeons in that complex and dramatic lifesaving operations
were completed to innumerable accolades, while concurrently, public criticism of the
economics of medicine, in particular, high-priced surgical practice, portrayed the
scalpel holder as an acquisitive, financially driven, selfish individual. This was in
stark contrast to the relatively selfless and sanctified image of the surgeon before the
growth of specialty work and the introduction of government involvement in health
care delivery.

Although they are philosophically inconsistent, the dramatic and theatrical fea-
tures of surgery that make surgeons heroes from one perspective and symbols of cor-
ruption, mendacity, and greed from the opposite point of view are the very reasons
why society demands so much of its surgeons. There is the precise and definitive na-
ture of surgical intervention, the expectation of success that surrounds an operation,
the short time frame in which outcomes are realized, the high income levels of most
surgeons, and the almost insatiable inquisitiveness of lay individuals concerning all
aspects of the act of consensually cutting into another human's flesh. These phenome-
na, ever more sensitized in an age of mass media and instantaneous telecommunica-
tion, make surgeons seem more accountable than their medical colleagues and, simul-
taneously, symbolic of the best and the worst in medicine. In ways previously unim-
aginable, this vast social transformation of surgery controls the fate of the individual
practitioner in the present era to a much greater extent than surgeons as a collective
force are able to control it by their attempts to direct their own profession.

20TH CENTURY SURGICAL HIGHLIGHTS

Among the difficulties in studying 20th century surgery is the abundance of fa-
mous names and important written contributions—so much so that it becomes a diffi-
cult and invidious task to attempt any rational selection of representative personalities
along with their significant journal or book-length writings. Although many justly
famous names might be missing, the following description of surgical advances is in-
tended to chronologically highlight some of the stunning clinical achievements of the
past century.

In 1900, the German surgeon Hermann Pfannenstiel (1862-1909) described his


technique for a suprapubic surgical incision. That same year, William Mayo (1861-
1939) presented his results on partial gastrectomy before the American Surgical As-
sociation. The treatment of breast cancer was radically altered when George Beatson
(1848-1933), professor of surgery in Glasgow, Scotland, proposed oophorectomy and
the administration of thyroid extract as a possible cure (1901). John Finney (1863-
1942) of The Johns Hopkins Hospital authored a paper on a new method of gastrodu-
odenostomy, or widened pyloroplasty (1903). In Germany, Fedor Krause (1856-
1937) was writing about total cystectomy and bilateral ureterosigmoidostomy. In
1905, Hugh Hampton Young (1870-1945) of Baltimore was presenting early studies
of his radical prostatectomy for carcinoma. William Handley (1872-1962) was sur-
geon of the Middlesex Hospital in London when he authored Cancer of the Breast
and Its Treatment (1906). In that work he advanced the theory that in breast cancer,
metastasis is due to extension along lymphatic vessels and not to dissemination via
the bloodstream. That same year, José Goyanes (1876-1964) of Madrid used vein
grafts to restore arterial flow. William Miles (1869-1947) of England first wrote
about his technique of abdominoperineal resection in 1908, the same year that Frie-
drich Trendelenburg (1844-1924) attempted pulmonary embolectomy. Three years
later, Martin Kirschner (1879-1942) of Germany described a wire for skeletal traction
and for stabilization of bone fragments or joint immobilization. Donald Balfour
(1882-1963) of the Mayo Clinic provided the initial account of his important opera-
tion for resection of the sigmoid colon, as did William Mayo for his radical operation
for carcinoma of the rectum in 1910.

In 1911, Fred Albee (1876-1945) of New York City began to use living bone
grafts as internal splints. Wilhelm Ramstedt (1867-1963), a German surgeon, de-
scribed a pyloromyotomy (1912) at the same time that Pierre Fredet (1870-1946) was
reporting a similar operation. In 1913, Henry Janeway (1873-1921) of New York City
developed a technique for gastrostomy in which he wrapped the anterior wall of the
stomach around a catheter and sutured it in place, thereby establishing a permanent
fistula. Hans Finsterer (1877-1955), professor of surgery in Vienna, improved on
Franz von Hofmeister's (1867-1926) description of a partial gastrectomy with closure
of a portion of the lesser curvature and retrocolic anastomosis of the remainder of the
stomach to the jejunum (1918). Thomas Dunhill (1876-1957) of London was a pio-
neer in thyroid surgery, especially in his operation for exophthalmic goiter (1919).
William Gallie (1882-1959) of Canada used sutures fashioned from the fascia lata in
herniorrhaphy (1923). Barney Brooks (1884-1952), professor of surgery at Vanderbilt
University in Nashville, Tennessee, initially introduced clinical angiography and
femoral arteriography in 1924. Five years later, Reynaldo dos Santos (1880-1970), a
Portuguese urologist, reported the first translumbar aortogram. Cecil Joll (1885-
1945), professor of surgery in London, fully described the treatment of thyrotoxicosis
by means of subtotal thyroidectomy in the 1930s.

In 1931, George Cheatle (1865-1951), professor of surgery in London, and Max


Cutler (1899-1984), a surgeon from New York City, published their important treatise
Tumours of the Breast. In that same year, Cutler detailed his systemic use of ovarian
hormone in the treatment of chronic mastitis. Around the same time, Ernst Sauer-
bruch (1875-1951) of Germany completed the first successful surgical intervention
for cardiac aneurysm, and his countryman Rudolph Nissen (1896-1981) removed an
entire bronchiectatic lung. Geoffrey Keynes (1887-1982) of St. Bartholomew's Hospi-
tal in England articulated the basis for the opposition to radical mastectomy and his
favoring of radium treatment in breast cancer (1932). The Irish surgeon Arnold Henry
(1886-1962) devised an operative approach for femoral hernia in 1936. Earl Should-
ice (1891-1965) of Toronto first began to experiment with a groin hernia repair based
on overlapping layers brought together by a continuous wire suture during the 1930s.
René Leriche (1879-1955) proposed an arteriectomy for arterial thrombosis in 1937
and, later, periarterial sympathectomy to improve arterial flow. Leriche also enunciat-
ed a syndrome of aortoiliac occlusive disease in 1940. In 1939, Edward Churchill
(1895-1972) of the Massachusetts General Hospital performed a segmental pneumo-
nectomy for bronchiectasis. Charles Huggins (1901-1997) a pioneer in endocrine
therapy for cancer, found that antiandrogenic treatment consisting of orchiectomy or
the administration of estrogens could produce long-term regression in patients with
advanced prostatic cancer. These observations formed the basis for the current treat-
ment of prostate and breast cancer by hormonal manipulation; Dr. Huggins was
awarded the Nobel Prize in 1966 for these monumental discoveries. Clarence
Crafoord (1899-1984) pioneered his surgical treatment of coarctation of the aorta in
1945. The following year, Willis Potts (1895-1968) completed an anastomosis of the
aorta to a pulmonary vein for certain types of congenital heart disease. Chester
McVay (1911-1987) popularized a repair of groin hernias based on the pectineal lig-
ament in 1948. Working at Georgetown University Medical Center in Washington,
DC, Charles Hufnagel (1916-1989) designed and inserted the first workable prosthet-
ic heart valve in a man (1951). That same year, Charles Dubost (1914-1991) of Paris
performed the first successful resection of an abdominal aortic aneurysm and inser-
tion of a homologous graft. Robert Zollinger (1903-1994) and Edwin Ellison (1918-
1970) first described their eponymic polyendocrine adenomatosis in 1955. The fol-
lowing year, Donald Murray (1894-1976) completed the first successful aortic valve
homograft. At the same time, John Merrill (1917-1986) was performing the world's
first successful homotransplantations of the human kidney between identical twin
brothers. Francis D. Moore (1913-2001) defined objectives of metabolism in surgical
patients and in 1959 published his widely quoted book Metabolic Care of the Surgi-
cal Patient. Moore was also a driving force in the field of transplantation and pio-
neered the technique of using radioactive isotopes to locate abscesses and tumors. In
the 1960s, Jonathan E. Rhoads (1907-2002 in collaboration with colleagues Harry
Vars and Stan Dudrick, described the technique of total parenteral nutrition, which
has become an important and lifesaving treatment in the management of a critically ill
patient who cannot tolerate standard enteral feedings. James D. Hardy (1918-2003),
at the University of Mississippi, performed the first lung (1963) and heart (1964)
transplants in a human.

The big contribution to development thoracic and cardiovascular surgery was


brought by Ukrainian surgeon N. Amosov, and in development abdominal and vascu-
lar surgery -A. Shalimov.

FUTURE TRENDS

Throughout most of its evolution, the practice of surgery has been largely de-
fined by its tools and the manual aspects of the craft. The last decades of the 20th
century saw unprecedented progress in the development of new instrumentation and
imaging techniques. These refinements have not come without noticeable social and
economic cost. Advancement will assuredly continue, for if the study of surgical his-
tory offers any lesson, it is that progress can always be expected, at least relative to
technology. There will be more sophisticated surgical operations with better results.
Eventually, automation may even robotize the surgeon's hand for certain procedures.
Still, the surgical sciences will always retain their historical roots as fundamentally a
manually based art and craft.

In many respects, the surgeon's most difficult future challenges are not in the
clinical realm but instead in better understanding the socioeconomic forces that affect
the practice of surgery and in learning how to effectively manage them. Many splen-
did schools of surgery now exist in virtually every major industrialized city, but none
can lay claim to dominance in all the disciplines that make up surgery. Likewise, the
presence of authoritative individual personalities who help guide surgery is more un-
usual today than in previous times. National aims and socioeconomic status have be-
come overwhelming factors in securing and shepherding the future growth of surgery
worldwide. In light of an understanding of the intricacies of surgical history, it seems
an unenviable and obviously impossible task to predict what will happen in the fu-
ture. In 1874, John Erichsen (1818-1896) of London wrote that “the abdomen, chest,
and brain will forever be closed to operations by a wise and humane surgeon.” A few
years later Theodor Billroth remarked, “A surgeon who tries to suture a heart wound
deserves to lose the esteem of his colleagues.” Obviously, the surgical crystal ball is a
cloudy one at best.

To study the fascinating history of our profession, with its many magnificent
personalities and outstanding scientific and social achievements, may not necessarily
help us predict the future of surgery. However, it does shed much light on the clinical
practices of our own time. To a certain extent, if surgeons in the future wish to be re-
garded as more than mere technicians, the profession needs to better appreciate the
value of its past experiences. Surgery has a distinguished heritage that is in danger of
being forgotten. Although the future of the art, craft, and science of surgery remains
unknown, it assuredly rests on a glorious past.

ETHICS IN SURGERY

Renewed public attention is being paid to ethics today. There are governmental
ethics commissions, research ethics boards, and corporate ethics committees. Some of
these institutional entities are little more than window dressing, whereas others are
investigative bodies called into being, for example, on suspicion that financial records
have been altered or data have been presented in a deceptive manner. However, many
of these groups do important work, and the fact that they have been established at all
suggests that we are not as certain as we once were, or thought we were, about where
the moral boundaries are and how we would know if we overstepped them. In search
of insight and guidance, we turn to ethics. In the professions, which are largely self-
regulating, and especially in the medical profession, whose primary purpose is to be
responsive to people in need, ethics is at the heart of the enterprise.
It is important to be clear at the outset about what ethics is and is not. Although
physicians are expected to uphold such standards of professionalism as reporting im-
paired colleagues, medical ethics is not primarily about keeping transgressors in line.
That is the domain of laws, courts, and boards of medical examiners. Ethics has to do
with discerning where the lines should be drawn in the first place and to what we
should aspire. It is about thinking through what we believe is good or bad or right or
wrong and why we think that way. The emphasis is on reflecting and deliberating.
Ethical reflection is especially useful in a social and cultural environment such as
ours in which values often conflict.

The ethical precepts of the medical profession have traditionally been summa-
rized in various oaths and codes. For example, it is still customary for students to re-
peat the Hippocratic Oath, or some contemporary adaptation of it, on graduation from
medical school. The American College of Surgeons' Statements on Principles con-
tains a fellowship pledge that includes a promise to maintain the College's historical
commitment to “the ethical practice of medicine.” The American College of Obstetri-
cians and Gynecologists (ACOG) subscribes to a code of ethics that governs the pa-
tient-physician relationship, physician conduct and practice, conflicts of interest, pro-
fessional relationships, and societal responsibilities. Moreover, ACOG's publication
Ethics in Obstetrics and Gynecology is exemplary in its comprehensiveness and spec-
ificity in discussing ethical issues ranging from reproductive choice to end-of-life
care. Several other surgical subspecialties, as well as anesthesiology, have also given
careful thought to ethical issues that arise in practice, research, education, and the in-
troduction of innovative surgical technologies and techniques.

Since 1847, the American Medical Association has promulgated a statement of


ethical principles. Although this code has evolved over time to accommodate changes
in society and medicine, it has always enunciated the ethical principles on which the
profession perceives itself to be grounded. The most recent version of this statement
of principles is more patient centered than ever before. It asserts that “a physician
must recognize responsibility to patients first and foremost” and spells out this re-
sponsibility as the provision of “competent medical care, with compassion and re-
spect for human dignity and rights.” Principle VIII states, “A physician shall, while
caring for a patient, regard responsibility to the patient as paramount.”

Responsibility to the patient in contemporary clinical ethics entails maximal pa-


tient participation, as permitted by the patient's condition, in decisions regarding the
course of care. For the surgeon, this means arriving at an accurate diagnosis of the pa-
tient's complaint, making a treatment recommendation based on the best knowledge
available, and then talking with the patient about the merits and drawbacks of the rec-
ommended course in light of the patient's life values. For the patient, maximal partic-
ipation in decision making means having a conversation with the surgeon about the
recommendation, why it seems reasonable and desirable, what the alternatives are, if
any, and what the probable risks are of accepting the recommendation or pursuing an
alternative course.

This view of ethically sound clinical care has evolved over the latter half of the
20th century from a doctor-knows-best ethic that worked reasonably well for both pa-
tients and physicians at a time when medical knowledge was limited and most of
what medicine could do for patients could be carried in the doctor's black bag or han-
dled in a small, uncluttered office or operating room.

The subsequent explosion of biomedical knowledge and the resulting prolifera-


tion of treatment options, many of them involving new technologic apparatus and in-
terventions, were accompanied by a growing dissatisfaction with medical paternal-
ism. As medicine grew more complex and doctors became more reliant on specialty
knowledge and instrumentation, physicians and patients became less familiar with
each other. Patients could no longer assume that they and their physicians shared a
common set of personal values sufficient to guide physicians in judging what was
best for their patients. For example, faced with a variety of treatment options, women
in whom breast cancer is diagnosed and men in whom prostate cancer is diagnosed
want to personally participate in decisions that will affect not only their bodies but
also their lives.

In response to these new complexities and following on the various rights


movements of the 1960s, some bioethicists began to advocate giving pride of place to
patient autonomy (respecting patients' right to decide by seeking their consent to
treatment) over physician beneficence (doing what, in the physician's judgment, is in
the patient's best interest) in the hierarchy of principles governing ethical medicine
(autonomy, nonmaleficence, beneficence, and justice).

Consent is permission, granted by the patient to the surgeon, to make a diagnos-


tic or therapeutic intervention on the patient's behalf. For consent to be valid, it must
be informed. The patient must be provided all relevant information. To be valid, it
must also be voluntary, that is, as free from coercion as possible while recognizing
that in extremes the patient's condition itself may be inherently coercive. The sur-
geon's ethical objective is to judiciously provide the patient sufficient information
with which to decide what course to follow. This entails selectively presenting all in-
formation pertinent to the patient's condition regarding benefits, risks, and alterna-
tives while avoiding overwhelming the patient with extraneous data. To walk the line
between what is pertinent and what is extraneous requires prudent judgment.

Informed consent has become a baseline best-practice ethical standard in modern


medical care. It is a necessary but insufficient condition for ethically sound patient
care. More moral work remains to be done if the physician-patient relationship is to
be more than a contractual arrangement for rendering services. The ultimate goal is to
achieve the best outcome, not only in terms of adherence to ethical principles of prac-
tice but also in keeping with patients' moral values, with what matters most to patients
in their relationships and their lives. Achieving this goal certainly entails the provi-
sion of information and the granting of consent, but this exchange must take place in
the context of a conversation about how the proposed intervention will affect a par-
ticular patient's life.

In 1984, Jay Katz foresaw the moral work that would be required to construct a
contemporary medical ethic capable of overcoming what he termed a prevailing si-
lence between doctors and patients. Katz was referring to the practice of physicians
deciding what was best for patients and patients abiding by the decision. He proposed
that this silence be supplanted by “meaningful conversation” based on “the humane-
ness of consensual understanding.”

Meaningful conversation requires conversation partners jointly committed to


treating the patient's ailment in a context of mutual respect and understanding. In ad-
dition to enhancing mutuality and promoting understanding, meaningful conversation
contributes to better health outcomes and to patients' satisfaction with their care. It
stands to reason that patients whose doctors are responsive to their questions are like-
ly to feel better. Is such attentiveness a luxury in today's time-conscious, monitored,
and managed environment? On the contrary, studies show that when doctors miss
clues to emotional and social matters that their patients cannot broach explicitly, vis-
its tend to be prolonged as the patient continues to try to elicit an acknowledgment
from the physician of concerns that may not seem immediately relevant to the pa-
tient's chief complaint.

Anticipating the need for physicians to cultivate the ability to engage patients in
meaningful conversation, the Accreditation Council for Graduate Medical Education
(ACGME) has included ethical and professional skills and behavior among the gen-
eral clinical competencies on which residency training programs are evaluated. Ac-
creditation criteria for programs include adherence to accepted ethical principles of
patient care, as well as respectful personal interactions with diverse patients, families,
and other professionals.

In the growing literature on diversity in clinical medicine it has become com-


monplace to use the shorthand concept of cultural competence. This may be a mis-
nomer in that competence denotes mastery of a body of knowledge whereas what is
wanted is improved cross-cultural interactions between patients and physicians. Cul-
ture is not a data set to be mastered and applied but a concept that is dynamic and
personal and interpersonal. Culture plays a significant role in influencing the way we
think about illness and health.

At its most basic, culture is a pattern of shared beliefs, values, and behavior.
Culture includes, but is not limited to, the language we use, shared customs and prac-
tices, and the way we think about relationships. A culture may be religious, social, or
professional (we speak, for example, of the culture of medicine), and it unquestiona-
bly affects interactions between patients and clinicians—often beneath the awareness
of either party to the relationship. At its best, studied attention to cultural sensitivity
in clinical medicine is aimed at raising physicians' awareness of the significance of
cultural factors in their practice. Some efforts to increase cultural awareness general-
ize about the so-called Hispanic, African American, or Asian American patient.
Knowledge of community values may be useful in caring for patients from different
communities, but the risk of stereotyping is great if insufficient attention is paid to the
individual patient from a particular community, whatever it is. Culturally competent
care is no substitute for patient-centered care. Practically speaking, when cultural dif-
ferences between patients and their physicians are not taken into account, patient dis-
satisfaction and poorer health outcomes may result.

In many cultures, patients traditionally are not told that they have cancer or other
life-threatening conditions. In some cultures, disclosure of a grave prognosis is be-
lieved to cause patients to suffer unnecessarily, whereas withholding such infor-
mation is believed to encourage hope. Being direct and explicit may be considered
insensitive. Families may try to protect their loved one by taking on decision-making
responsibility. It would be unfair to impose the standards of disclosure common in
one culture on patients from another culture who may not want to know. This is all
useful information that can contribute to culturally appropriate care—as long as one
keeps in mind the caveat mentioned before, that patients are not solely the products of
their culture and should therefore be related to as individuals who may share some of
their culture's attitudes and beliefs but not others. Joseph Betancourt describes the
case of an elderly Italian woman whose son asked her surgeon not to inform her that
she had metastatic colon cancer for fear that it would sap her will to live.[19] Decision-
making and truth-telling processes vary not only from culture to culture but also from
family to family. Exploring the reasons for and the consequences of a preference for
secrecy can lead to culturally sensitive and ethically appropriate care.

What practical steps can be taken by clinicians to evaluate patient attitudes and
behavior relative to the patient's cultural context so that the physician and patient to-
gether can reach mutually desired goals of care? Marjorie Kagawa-Singer and her
colleagues at the University of California, Los Angeles, developed a useful tool for
ascertaining patients' levels of cultural influence. It goes by the acronym RISK:

1. Resources: On what tangible resources can the patient draw, and how readily
available are they?
2. Individual identity and acculturation: What is the context of the patient's per-
sonal circumstances and her degree of integration within her community?
3. Skills: What skills are available to the patient that allows him to adapt to the
demands of the condition?
4. Knowledge: What can be discerned from a conversation with the patient about
the beliefs and customs prevalent in her community and relevant to illness and
health, including attitudes about decision making and other issues that may af-
fect the physician-patient relationship?
RISK, therefore, encompasses resources, identity, skills, and knowledge.
Nowhere are respectful personal interactions more in demand than in the care of
patients near the end of life.
In 1998 the American College of Surgeons adopted a Statement on Principles
Guiding Care at the End of Life, which includes the following principles:
1. Respect the dignity of both patient and caregivers.
2. Be sensitive to and respectful of the patient's and family's wishes.
3. Use the most appropriate measures that are consistent with the choices of the
patient or the patient's legal surrogate.
4. Ensure alleviation of pain and management of other physical symptoms.
5. Recognize, assess, and address psychological, social, and spiritual problems.
6. Ensure appropriate continuity of care by the patient's primary and/or special-
ist physician.
7. Provide access to therapies that may realistically be expected to improve the
patient's quality of life.
8. Provide access to appropriate palliative care and hospice care.
9. Respect the patient's right to refuse treatment.
10. Recognize the physician's responsibility to forgo treatments that are futile.

A Surgeons Palliative Care Workgroup was convened in 2001 to put these prin-
ciples into operation and to introduce the precepts and techniques of palliative care
into surgical practice and education by means of symposia, a palliative care website,
and focused contributions to the surgical literature.
In a paper introducing a monthly series from members of the work group written
for and by surgeons, Geoffrey P. Dunn and Robert A. Milch observe that caring for
patients near the end of life offers surgeons an “opportunity to rebalance decisiveness
with introspection, detachment with empathy,” and thereby “restore the integrity of
our relationships with our patients.” Other contributions to this series provide expert
discussions of such ethically difficult issues as decision making in palliative surgery;
chronic pain management and opioid tolerance ; withdrawing life support, including
tube feeding, hydration, and total parenteral nutrition ; management of dyspnea, de-
pression, and anxiety; and attending to dying patients' spiritual needs. Two themes
thread their way through these discussions. Patients in a surgeon's care near the end of
life stand not only to gain from the surgeon's cognitive and technical expertise as long
as rescue is an option but also to benefit from the surgeon's attentiveness and guid-
ance when what ails the patient cannot be remedied or reversed. Moreover, surgeons
themselves can derive satisfaction from staying the course with dying patients and
their families, responding to their trust, seeing them through difficult times, and car-
ing for them even when curative options are no longer indicated or available.

Among other responsibilities articulated in the American Medical Association's


Principles of Medical Ethics, two suggest a growing sense within the profession of
medicine's role as a public-spirited profession:

1 Contributing to betterment of the health of the community


2 Supporting access to medical care for everyone

Additional evidence of public-spiritedness is to be found in the association's


Declaration of Professional Responsibility, which was forged in response to the ter-
rorist attacks on New York and Washington in September 2001. Subtitled Medicine's
Social Contract with Humanity, this unprecedented oath contains the following decla-
ration:

We, the members of the world community of physicians, solemnly commit our-
selves to
1. Respect human life and the dignity of every individual.
2. Treat the sick and injured with competence and compassion and without preju-
dice.
3. Refrain from supporting or committing crimes against humanity and condemn
all such acts.
4. Apply our knowledge and skill when needed, although doing so may put us at
risk.
5. Protect the privacy and confidentiality of those for whom we care and breach
that confidence only when keeping it would seriously threaten their health and
safety and that of others.
6. Work freely with colleagues to discover, develop, and promote advances in
medicine and public health that ameliorate suffering and contribute to human
well-being.
7. Educate the public and polity about present and future threats to the health of
humanity.
8. Advocate for social, economic, educational, and political changes that amelio-
rate suffering and contribute to human well-being.
9. Teach and mentor those who follow us for they are the future of our caring pro-
fession.
We make these promises solemnly, freely, and on our personal and professional
honor.

Recognizing the social value of volunteerism, the Governors' Committee on So-


cioeconomic Issues of the American College of Surgeons created the Giving Back
project in the year 2000. Based on survey data from 500 fellows, the committee rec-
ommended that the college “Promote surgeon volunteerism as ‘The right thing to do’
and ‘Part of being a physician.’”
Taken together, these three documents, along with the emphasis on professional
values in the medical ethics literature and the ACGME General Competencies, indi-
cate a renewed commitment on the part of clinicians to competent, respectful, com-
passionate patient care and a growing awareness within the profession of the ethical
obligations of physicians in their various roles as clinicians, researchers, educators,
and citizens that arise from and extend beyond the traditional patient-physician rela-
tionship.
Contemporary clinical ethics is evolving toward a relational understanding of in-
teractions between doctors and patients. In the parlance of ethics, this means that eth-
ical principles are being supplemented by moral virtues. Adherence to principles
leads one to ask: What should I do? Attention to virtues prompts the question: What
kind of person or doctor should I be? How to conduct oneself with patients in an eco-
nomic and social environment that rewards haste, encourages narrow self-interest and
inattention to the patient as a person, and is increasingly inhospitable to underserved
populations is motivating a re-evaluation of medical professionalism not only at the
bedside but in society as well.
Basic literature:
1. Oxford Textbook of Surgery (3-Volume Set) 2nd edition (January 15, 2000):
by Peter J. Morris (Editor), William C. Wood (Editor) By Oxford Press
2. Sabiston Textbook of Surgery 17th edition by Courtney M. Townsend Jr.,
Kenneth L. Mattox, B. Mark, MD Evers, Kenneth L., MD Mattox, Courtney
Townsend, Daniel Beauchamp, B. Mark Evers, Kenneth Mattox W.B. Saun-
ders Company (June, 2004)
3. Schwartz´s Principles of Surgery 8th Edition F. Charles Brunicardi. Copyright
©2007 the McGraw-Hill Companies.
4. Hospital surgery/ Edited by L. Kovalchuk et al. - Ternopil: Ukrmedknyha,
2004. - 472 p.

Tests for initial level of knowledge:


1. What kind of fundamental clinical prerequisites were required for develop-
ment of surgery in 19-th century?
A. Knowledge of human anatomy
B. Knowledge of biochemistry
C. Knowledge in anaesthesiology
D. Knowledge of mathematics

2. The author of “Dix Livres de la Chirurgie avec le Magasin des Instruments


Necessaires à Icelle” was:
A. John Hunter
B. Andreas Vesalius
C. Theodor Kocher
D. Ambroise Paré

3. Very important event in the evolution of surgical history in 19-th century was:
A. Methods of controlling haemorrhage
B. Working out of techniques of surgical operations
C. Antisepsis, asepsis and understanding the nature of infection
D. Successes in oncology
4. Who introduced methods of aseptic and antiseptic in Russia?
A. N. Pirogov
B. Lister
C. T. Kocher
D. T. Billroth

5. Who among the outstanding surgeons of 19th century introduced fundamental


science achievements in practical activities of doctors of surgeons?
A. F. Trendelenburg
B. W. Halsted
C. J. Murphy
D. M. Sklyfosovskyi

6. In 1900, which surgeon presented his results on partial gastrectomy before the
American Surgical Association?
A. John Finney (1863-1942)
B. William Mayo (1861-1939)
C. Fedor Krause (1856-1937)
D. William Miles (1869-1947)

7. What prominent late 19th century discovery conducted by Pulyui Ivan and
Wilhelm Roentgen had an enormous impact on the evolution of surgery?
A. X-rays
B. Ultrasonography
C. Magnetic Resonance Imaging
D. Computer Tomography

8. Which surgeon is prominent for setting the scientific tone of surgery as a legit-
imate scientific endeavor in surgical history?
A. Jules Peán (1830-1898)
B. William Stewart Halsted (1852-1922)
C. Marin-Theodore Tuffiér (1857-1929)
D. Eduardo Bassini (1844-1924)

9. Which important Ukrainian surgeon contributed greatly to the development of


thoracic and cardiovascular surgery?
A. N. Pirogov
B. A. Shalimov
C. I. Pavlov
D. N. Amosov

10.Which two surgeons first described the eponymic polyendocrine adenomatosis


in 1955?
A. Willis Potts (1895-1968) and Charles Hufnagel (1916-1989)
B. Francis D. Moore (1913-2001) and Jonathan E. Rhoads (1907-2002)
C. James D. Hardy (1918-2003) and Rudolph Nissen (1896-1981)
D. Robert Zollinger (1903-1994) and Edwin Ellison (1918-1970)

Keys for initial tests

1 2 3 4 5 6 7 8 9 10
A,C D C A B B A B D D

Tests for final level of knowledge, keys for tests:

1. The first successful open heart operation in 1953, conducted with the use of a
heart-lung machine, was a momentous surgical contribution. Which of these sur-
geons devised the machine?
A. Russell Brock (1903-1980)
B. Luther Hill (1862-1946)
C. John Gibbon (1903-1973)
D. Charles Bailey (1910-1993)

2. In an exciting era for surgeons, what important surgical procedure did Allen
Oldfather Whipple (1881-1963) introduce in 1935?
A. pancreaticoduodenectomy for cancer of the pancreas
B. splanchnic resection for hypertension
C. vagotomy for operative treatment of peptic ulcer
D. intracranial section of various cranial nerves

3. At the beginning of the 20th century, Alexis Carrel developed a new revolu-
tionary technique. What was it?
A. new suturing technique to anastomose the smallest of blood vessels
B. new technique of creating immunosuppression as a method to allow the
host to accept the foreign transplant
C. new technique to suture a wound that penetrated a cardiac chamber
D. new technique for mammaplasty

4. Which English surgeon was outstanding for his efforts to introduce systematic,
scientifically based antisepsis in the treatment of wounds and the performance
of surgical operations?
A. Robert Tait (1845-1899)
B. William Macewen (1848-1924)
C. Frederick Treves (1853-1923)
D. Joseph Lister (1827-1912)
5. In 1998 the American College of Surgeons adopted a Statement on Principles
Guiding Care at the End of Life. Which of the following principles is
INCORRECT?
A. Use the most appropriate measures that are consistent with the choices of
the patient or the patient's legal surrogate.
B. Recognize the physician's responsibility to forgo treatments that are fu-
tile
C. Mystify the public and polity about present and future threats to the
health of humanity.
D. Recognize, assess, and address psychological, social, and spiritual prob-
lems

6. In 1900, which surgeon described his technique for a suprapubic surgical inci-
sion?
A. John Finney (1863-1942)
B. Hugh Hampton Young (1870-1945)
C. Friedrich Trendelenburg (1844-1924)
D. Hermann Pfannenstiel (1862-1909)

7. Which of the following surgeons reported the first successful ligation of a pa-
tent arterial duct in 1939 and resection for coarctation of the aorta with direct
anastomosis of the remaining ends in 1945?
A. Robert Gross (1905-1988)
B. John Alexander (1891-1954)
C. Claude Beck (1894-1971)
D. Elliott Cutler (1888-1947)

8. Which famous surgeon, notable for his method of controlling hemorrhage,


humbly attributed his success with patients to God, as noted in his famous mot-
to, “Je le pansay. Dieu le guérit,” that is, “I treated him. God cured him.”
A. Ambroise Paré (1510-1590)
B. John Hunter (1728-1793)
C. William T. G. Morton (1819-1868)
D. Theodor Kocher (1841-1917)

9. In response to the terrorist attacks on New York and Washington in September


2001, evidence of public-spiritedness is to be found in the Declaration of Pro-
fessional Responsibility, subtitled Medicine's Social Contract with Humanity.
Which of the following declaration of the oath is NOT correct?
A. Apply our knowledge and skill when needed, although doing so may put
us at risk.
B. Supporting access to medical care for only the privileged
C. Work freely with colleagues to discover, develop, and promote advances
in medicine and public health that ameliorate suffering and contribute to
human well-being.
D. Teach and mentor those who follow us for they are the future of our car-
ing profession.

10.Consent is permission, granted by the patient to the surgeon, to make a diag-


nostic or therapeutic intervention on the patient's behalf. All the following are
necessary for a consent to be valid EXCEPT:
A. it must be informed
B. it must also be voluntary
C. it must compel by force
D. it must be ethical

Keys for final tests

1 2 3 4 5 6 7 8 9 10

C A A D C D A A B C

Materials for the self-study of the students


Main tasks Notes (instructions)
Repeat:
1. Inflammation
2. Infection -To represent the methods of antiseptic
3. Pathophysiology of basic sur-
gical diseases.
Study:
1. The prominent discoveries in sur-
gery. -Last achievements in field of pathogen-
2. Development of the methods of esis of ulcer disease
diagnosis of surgical diseases.
3. Ethics in surgery
Study guide #1.2

“New technologies in surgery: microsurgical, endoscopic, endovascular, laser,


ultrasonic, welding of tissues, using of prostheses and implants.”

Medical technology is today a feature of hospitals, clinics and GP surgeries.


The practice of medicine has been revolutionised by computers, digitisation, new ma-
terials and good old-fashioned laboratory research. Yet this has been accompanied by
increased patient anxiety about the risks and consequences of medical intervention.
The introduction of new medical technology has not been so rapid in develop-
ing countries, but the sense of ambivalence is the same – technology represents both
progress and threat. In countries with multiple healing systems patients can choose
whom to consult, depending on their illness. In these countries a technological ap-
proach may be chosen as a first or last resort.
Educational aims:
1. To be acquainted with new technologies in surgery.
2. To determine the great importance of microsurgical, endoscopic, endovascular,
laser operations in surgery, welding of tissues, using of prostheses and im-
plants.
3. To determine the diagnostic significance of ultrasonic, endoscopic and others
new diagnostic procedures.
4. To determine the main advantages of new technologies in surgery.
5. To estimate efficiency of new technologies in surgery.
A student must know:
1. Foundations of microsurgery.
2. Foundations of endoscopic interventions.
3. Foundations of laser surgery.
4. Foundations of ultrasonic procedures.
5. Foundations of using of prostheses and implants
6. General information about welding of tissues. Principles of using
A student must be able to:
1. Correctly interpret the results of endoscopic, ultrasonic examinations and oth-
ers new diagnostic procedures.
2. Define indications for microsurgical, endoscopic, endovascular, laser opera-
tions.
3. Define contra-indications for microsurgical, endoscopic, endovascular, laser
operations.
4. Define the rational volume of laboratory and instrumental methods of research
for microsurgical, endoscopic, endovascular, laser operations.
5. Perform pre-operative preparation of patients.
6. Conduct post-operative care.
Terminology.
Term Definition
Is a medical procedure in endoscopy is combined with ul-
Endoscopic ultra-
trasound to obtain images of the internal organs in the
sound (EUS)
chest and abdomen.
Is imaging by sections or sectioning, through the use of
Tomography
any kind of penetrating wave.
is a medical imaging technique used in radiology to visu-
Magnetic resonance
alize detailed internal structures that uses of nuclear mag-
imaging (MRI)
netic resonance for imaging
Is a modern surgical technique in which operations in the
Laparoscopic sur- abdomen are performed through small incisions (usually
gery 0.5–1.5 cm) as compared to the larger incisions needed in
laparotomy.
is a general term for surgery requiring an operating
Microsurgery
microscope.
Replantation is the reattachment of a completely detached body part.
Is a form of minimally invasive surgery that was designed
Endovascular sur-
to access many regions of the body via major blood ves-
gery
sels.
Is joining damaged human tissues and restoration of the
functioning of human organs. It is used to dissect tissues
Welding
and to stop or prevent bleeding when tissue and blood
vessels are incised.

Content:
Endoscopic ultrasound (EUS) or echo-endoscopy is a medical procedure in
endoscopy (insertion of a probe into a hollow organ) is combined with ultrasound to
obtain images of the internal organs in the chest and abdomen. It can be used to visu-
alize the wall of these organs, or to look at adjacent structures. Combined with Dop-
pler imaging, nearby blood vessels can also be evaluated.
Endoscopic ultrasonography is most commonly used in the upper digestive
tract and in the respiratory system. The procedure is performed by gastroenterologists
or pulmonologists who have had extensive advanced training. For the patient, the
procedure feels almost identical to the endoscopic procedure without the ultrasound
part, unless ultrasound-guided biopsy of deeper structures is performed.
For endoscopic ultrasound of the upper digestive tract, a probe is inserted into
the oesophagus, stomach and duodenum during a procedure called esophagogastro-
duodenoscopy. Among other uses, it allows for screening for pancreatic cancer, oe-
sophageal cancer, and gastric cancer as well as benign tumours of the upper gastroin-
testinal tract. It also allows for characterization and biopsy of any focal lesions found
in the upper gastrointestinal tract. This is done by inserting a needle through the
stomach lining into the target.
Endoscopic ultrasound is performed with the patient sedated. The endoscope is
passed through the mouth and advanced to through the oesophagus to the suspicious
area. From various positions between the oesophagus and duodenum organs within
and outside the gastrointestinal tract can be imaged to see if they are abnormal and
they can be biopsied by a process called fine needle aspiration. Organs such as the
liver, pancreas and adrenal glands are easily biopsied as are any abnormal lymph
nodes. In addition the gastrointestinal wall itself can be imaged to see if it is abnor-
mally thick suggesting inflammation or malignancy.
The technique is highly sensitive for detection of Pancreatic Cancer (90-95%
sensitivity) particularly in patients who are suspected to have a mass or present with
jaundice. Its role in staging patients with pancreatic cancer is limited to local metasta-
ses; however, in combination with CT scan which provides information on regional
metastases, it provides an excellent imaging modality for diagnosis and staging of
pancreatic carcinoma.
Endoscopic ultrasound can also be used in conjunction with endoscopic retro-
grade cholangio pancreatography (ERCP). The ultrasound probe is used to locate gall
stones which may have migrated into the common bile duct. This occurrence may
cause obstruction of the drain shared by the liver and pancreas which may lead to
lower back pain, jaundice and pancreatitis.
Echo-endoscopy can also be used for imaging of the rectum and colon, alt-
hough these applications are lesser known. It is used primarily to stage newly diag-
nosed rectal or anal cancer. EUS guided fine needle aspiration may be used to sample
lymph nodes during this procedure. Evaluation of the integrity of the anal sphincters
may also be done during lower EUS procedures.
Respiratory tract
An endoscopic ultrasound probe placed in the oesophagus can also be used to
visualize lymph nodes in the chest surrounding the airways (bronchi), which is im-
portant for the staging of lung cancer. Ultrasound can also be performed with an en-
doscopic probe inside the bronchi themselves, a technique known as endobronchial
ultrasound.
The quality of the image produced is directly proportional to the frequency
used. Therefore a high frequency produces a better image. However, high frequency
ultrasound does not penetrate as well as lower frequency ultrasound so that the exam-
ination of the nearby organs may be more difficult.
Tomography is imaging by sections or sectioning, through the use of any kind
of penetrating wave. A device used in tomography is called a tomograph, while the
image produced is a tomogram. The method is used in radiology, archaeology, biolo-
gy, geophysics, oceanography, materials science, astrophysics and other sciences. In
most cases it is based on the mathematical procedure called tomographic reconstruc-
tion. The word was derived from the Greek word tomos which means "part" or "sec-
tion", representing the idea of "a section", "a slice" or "a cutting". A tomography of
several sections of the body is known as a polytomography.
The word "tomography" is derived from the Greek tomos (part) and graphein
(to write).
In conventional medical X-ray tomography, clinical staff makes a sectional im-
age through a body by moving an X-ray source and the film in opposite directions
during the exposure. Consequently, structures in the focal plane appear sharper, while
structures in other planes appear blurred. By modifying the direction and extent of the
movement, operators can select different focal planes which contain the structures of
interest. Before the advent of more modern computer-assisted techniques, this tech-
nique, ideated in the 1930s by the radiologist Alessandro Vallebona, proved useful in
reducing the problem of superimposition of structures in projection (shadow) radiog-
raphy.
Modern tomography
More modern variations of tomography involve gathering projection data from
multiple directions and feeding the data into a tomographic reconstruction software
algorithm processed by a computer. Different types of signal acquisition can be used
in similar calculation algorithms in order to create a tomographic image. With current
2005 technology, tomograms are derived using several different physical phenomena
listed in the following table.
Type of tomogram
• X-rays CT
• gamma rays SPECT
• radio-frequency waves MRI
• electron-positron annihilation PET
• electrons Electron tomography or 3D TEM
• ions atom probe
Some recent advances rely on using simultaneously integrated physical phe-
nomena, e.g. X-rays for both CT and angiography, combined CT/MRI and combined
CT/PET.
The term volume imaging might subsume these technologies more accurately
than the term tomography. However, in the majority of cases in clinical routine, staff
request output from these procedures as 2-D slice images. As more and more clinical
decisions come to depend on more advanced volume visualization techniques, the
terms tomography/tomogram may go out of fashion.
Many different reconstruction algorithms exist. Most algorithms fall into one of
two categories: filtered back projection (FBP) and iterative reconstruction (IR). These
procedures give inexact results: they represent a compromise between accuracy and
computation time required. FBP demands fewer computational resources, while IR
generally produces fewer artefacts (errors in the reconstruction) at a higher computing
cost.
Although MRI and ultrasound make cross sectional images they don't acquire
data from different directions. In MRI spatial information is obtained by using mag-
netic fields. In ultrasound, spatial information is obtained simply by focusing and
aiming a pulsed ultrasound beam.
Synchrotron X-ray tomographic microscopy
Recently a new technique called synchrotron X-ray tomographic microscopy
(SRXTM) allows for detailed three dimensional scanning of fossils.
Magnetic resonance imaging (MRI), nuclear magnetic resonance imaging
(NMRI), or magnetic resonance tomography (MRT) is a medical imaging technique
used in radiology to visualize detailed internal structures. The good contrast it pro-
vides between the different soft tissues of the body make it especially useful in brain,
muscles, heart, and cancer compared with other medical imaging techniques such as
computed tomography (CT) or X-rays.
Unlike CT scans or traditional X-rays MRI uses no ionizing radiation. Instead
it uses a powerful magnetic field to align the magnetization of some atoms in the
body, and then uses radio frequency fields to systematically alter the alignment of this
magnetization. This causes the nuclei to produce a rotating magnetic field detectable
by the scanner—and this information is recorded to construct an image of the scanned
area of the body.
The body is largely composed of water molecules. Each water molecule has
two hydrogen nuclei or protons. When a person goes inside the powerful magnetic
field of the scanner, the magnetic moments of some of these protons changes, and
aligns with the direction of the field.
In an MRI machine a radio frequency transmitter is briefly turned on, produc-
ing an electromagnetic field. The photons of this field have just the right energy,
known as the resonance frequency, to flip the spin of the aligned protons in the body.
As the intensity and duration of application of the field increase, more aligned spins
are affected. After the field is turned off, the protons decay to the original spin-down
state and the difference in energy between the two states is released as a photon. It is
these photons that produce the electromagnetic signal that the scanner detects. The
frequency at which the protons resonate depends on the strength of the magnetic
field. As a result of conservation of energy, this resonation frequency also dictates the
frequency of the released photons. The photons released when the field is removed
have energy — and therefore a frequency — due to the amount of energy the protons
absorbed while the field was active.
It is this relationship between field-strength and frequency that allows the use
of nuclear magnetic resonance for imaging. Additional magnetic fields are applied
during the scan to make the magnetic field strength depend on the position within the
patient, in turn making the frequency of the released photons dependent on position in
a predictable manner. Position information can then be recovered from the resulting
signal by the use of a Fourier transform. These fields are created by passing electric
currents through specially-wound solenoids, known as gradient coils. Since these
coils are within the bore of the scanner, there are large forces between them and the
main field coils, producing most of the noise that is heard during operation. Without
efforts to dampen this noise, it can approach 130 decibels (dB) with strong fields.
An image can be constructed because the protons in different tissues return to
their equilibrium state at different rates, which is a difference that can be detected.
Five different tissue variables — spin density, T1 and T2 relaxation times and flow
and spectral shifts can be used to construct images. By changing the parameters on
the scanner, this effect is used to create contrast between different types of body tis-
sue or between other properties.
Contrast agents may be injected intravenously to enhance the appearance of
blood vessels, tumours or inflammation. Contrast agents may also be directly injected
into a joint in the case of arthrograms, MRI images of joints. Unlike CT, MRI uses no
ionizing radiation and is generally a very safe procedure. Nonetheless the strong
magnetic fields and radio pulses can affect metal implants, including cochlear im-
plants and cardiac pacemakers. In the case of cochlear implants, the US FDA has ap-
proved some implants for MRI compatibility. In the case of cardiac pacemakers, the
results can sometimes be lethal, so patients with such implants are generally not eli-
gible for MRI.
MRI is used to image every part of the body, and is particularly useful for tis-
sues with many hydrogen nuclei and little density contrast, such as the brain, muscle,
connective tissue and most tumours.
In clinical practice, MRI is used to distinguish pathologic tissue (such as a
brain tumour) from normal tissue. One advantage of an MRI scan is that it is harmless
to the patient. It uses strong magnetic fields and non-ionizing radiation in the radio
frequency range.
While CT provides good spatial resolution (the ability to distinguish two sepa-
rate structures an arbitrarily small distance from each other), MRI provides compara-
ble resolution with far better contrast resolution (the ability to distinguish the differ-
ences between two arbitrarily similar but not identical tissues). The basis of this abil-
ity is the complex library of pulse sequences that the modern medical MRI scanner
includes, each of which is optimized to provide image contrast based on the chemical
sensitivity of MRI.
A number of features of MRI scanning can give rise to risks. These include:
Powerful magnetic fields, Cryogenic liquids, Claustrophobia
In addition, in cases where MRI contrast agents are used, these also typically
have associated risks.
Most forms of medical or bio stimulation implants are generally considered
contraindications for MRI scanning. These include pacemakers, vagus nerve stimula-
tors, implantable cardioverter-defibrillators, loop recorders, insulin pumps, cochlear
implants, deep brain stimulators.
Ferromagnetic foreign bodies such as shell fragments, or metallic implants
such as surgical prostheses and aneurysm clips are also potential risks. Interaction of
the magnetic and radio frequency fields with such objects can lead to trauma due to
movement of the object in the magnetic field or thermal injury from radio-frequency
induction heating of the object.
Titanium and its alloys are safe from movement from the magnetic field.
MR-Safe — the device or implant is completely non-magnetic, non-electrically
conductive, and non-RF reactive, eliminating all of the primary potential threats dur-
ing an MRI procedure.
Microsurgery is a general term for surgery requiring an operating microscope.
The most obvious developments have been procedures developed to allow
anastomosis of successively smaller blood vessels and nerves (typically 1 mm in
diameter) which have allowed transfer of tissue from one part of the body to another
and re-attachment of severed parts. Although microsurgery is used mostly in plastic
surgery, microsurgical techniques are utilized by all specialties today, especially
those involved in reconstructive surgery such as: general surgery, ophthalmology,
orthopedic surgery, gynecological surgery, otolaryngology, neurosurgery, oral and
maxillofacial surgery, and pediatric surgery.
Free tissue transfer
Free tissue transfer is a surgical reconstructive procedure using microsurgery.
A region of "donor" tissue is selected that can be isolated on a feeding artery and
vein; this tissue is usually a composite of several tissue types (e.g., skin, muscle, fat,
bone). Common donor regions include the rectus abdominis muscle, latissimus dorsi
muscle, fibula, and radial forearm bone and skin lateral arm skin. The composite
tissue is transferred (moved as a free flap of tissue) to the region on the patient
requiring reconstruction (e.g., mandible after oral cancer resection, breast after cancer
resection, traumatic tissue loss, congenital tissue absence). The vessels that supply the
free flap are anastomosed with microsurgery to matching vessels (artery and vein) in
the reconstructive site. The procedure was first done in the early 1970s and has
become a popular "one-stage" (single operation) procedure for many surgical
reconstructive applications.
Replantation
Replantation is the reattachment of a completely detached body part. Fingers
and thumbs are the most common but the ear, scalp, nose, face, arm and penis have
all been replanted. Generally replantation involves restoring blood flow through
arteries and veins, restoring the bony skeleton and connecting tendons and nerves as
required.
Initially, when the techniques were developed to make replantation possible,
success was defined in terms of a survival of the amputated part alone. However, as
more experience was gained in this field, surgeons specializing in replantation began
to understand that survival of the amputated piece was not enough to ensure success
of the replant. In this way, functional demands of the amputated specimen became
paramount in guiding which amputated pieces should and should not be replanted.
Additional concerns about the patients ability to tolerate the long rehabilitation
process that is necessary after replantation both on physical and psychological levels
also became important. So, when fingers are amputated, for instance, a replantation
surgeon must seriously consider the contribution of the finger to the overall function
of the hand. In this way, every attempt will be made to salvage an amputated thumb,
since a great deal of hand function is dependent on the thumb, while an index finger
or small finger may not be replanted, depending on the individual needs of the patient
and the ability of the patient to tolerate a long surgery and a long course of
rehabilitation.
However, if an amputated specimen is not able to be replanted to its original
location entirely, this does not mean that the specimen is unreplantable. In fact,
replantation surgeons have learned that only a piece or a portion may be necessary to
obtain a functional result, or especially in the case of multply amputated fingers, a
finger or fingers may be transposed to a more useful location to obtain a more
functional result. This concept is called "spare parts" surgery.

Transplantation
Microsurgical techniques have played a crucial role in the development of
transplantation immunological research because it allowed the use of rodents models,
which are more appropriate for transplantation research (there are more reagents,
monoclonal antibodies, knockout animals, and other immunological tools for mice
and rats than other species). Before it was introduced, transplant immunology was
studied in rodents using the skin transplantation model, which is limited by the fact it
is not vascularized. Thus, microsurgery represents the link between surgery and
transplant immunological research. The first microsurgical experiments (porto-caval
anastomosis in the rat) were performed by Dr. Sun Lee (pioneer of microsurgery) at
the University of Pittsburgh in 1958. After a short time, many models of organ
tranplants in rat and mice have been established. Today, virtually every rat or mouse
organ can be transplanted with relative high success rate. Microsurgery was also
important to develop new techniques of transplantation, that would be later performed
in humans. In addition, it allows reconstruction of small arteries in clinical organ
transplantation (e.g. accessory arteries in cadaver liver transplantation, polar arteries
in renal transplantation and in living liver donor transplantation).
Endovascular surgery is a form of minimally invasive surgery that was de-
signed to access many regions of the body via major blood vessels.
Endovascular techniques were originally designed for diagnostic purposes.
Basic techniques involve the introduction of a catheter percutaneously or through the
skin, into a large blood vessel. Typically the blood vessel chosen is the femoral artery
or vein found near the groin. Access to the femoral artery for example, is required for
coronary, carotid, and cerebral angiographic procedures. The catheter is injected with
a radio-opaque dye that can be seen on live X-ray or fluoroscopy. As the dye courses
through the blood vessels, characteristic images are seen by experienced viewers and
can assist in the diagnosis of diseases such as atherosclerosis, vascular trauma, or an-
eurysms.
In recent years, however, the development of intravascular balloons, stents and
coils have allowed for new therapies as alternatives to traditional surgeries such as
Coronary artery bypass surgery (CABG), carotid endarterectomy and aneurysm clip-
ping. Stents and coils are composed of fine wire materials such as platinum that can
be inserted through a thin catheter and expanded into a predetermined shape once
they are guided into place.
Endovascular surgery is performed by radiologists, neurologists, neurosur-
geons, cardiologists, and vascular surgeons. The field is rapidly growing as its mini-
mally invasive techniques offer an immediate advantage over more traditional, yet
highly invasive surgeries. However, the science of endovascular surgery and its de-
veloping techniques are so new that it is currently difficult to compare the long term
outcomes and complications of these patients. Several trials are underway, including
Carotid Revascularization Endarterectomy versus Stent Trial (CREST), and Interna-
tional Subarachnoid Aneurysm Trial (ISAT), among others.
Laser surgery is surgery using a laser to cut tissue instead of a scalpel. Exam-
ples include the use of a laser scalpel in otherwise conventional surgery, and soft tis-
sue laser surgery, in which the laser beam vaporizes soft tissue with high water con-
tent. Laser resurfacing is a technique in which molecular bonds of a material are dis-
solved by a laser.
Laser surgery is commonly used on the eye. Techniques used include LASIK,
which is used to correct near- and far-sightedness in vision, and photorefractive
keratectomy, a procedure which permanently reshapes the cornea using an excimer
laser to remove a small amount of tissue.
Green laser surgery is used for the treatment/reduction of enlarged prostates.
Laser surgery is much safer than normal surgery as it makes no physical contact so no
infections are spread. Laser surgery is used not only for eye surgery but also in many
other streams. In industries it is used for minute cutting of hard metals and also for
cutting diamonds. In military it is used for guiding missiles rockets and in rifles for
exact shots.
The laser is very superficial, which results in a much reduced recovery time for
the patient.
Welding technology is advancing victoriously on the ground, in the underwa-
ter world and in space. Welding is starting to advance in the medical field. It is used
with success for joining damaged human tissues and restoration of the functioning of
human organs. It is used to dissect tissues and to stop or prevent bleeding when tissue
and blood vessels are incised. After that the operated organs inevitably lose their
functions, which are not restored when the patient recovers. The basic phenomena,
which proceed in soft tissue welding, can be schematically described as follows. The
tissue layers being joined are brought into contact over their surface layers by means
of a welding tool. Then the surgeon clamps the tissue area to be welded by the elec-
trodes of the welding tool and switches on the welding current source. When the
welding process control program is completed and the power is turned off, the
clamped tissue is released, and then the process should be repeated until the wound is
closed completely.
What is Endoscopic Surgery?
Endoscopic surgery uses scopes going through small incisions or natural body
openings in order to diagnose and treat disease. Another popular term is minimally
invasive surgery (MIS), which emphasizes that diagnosis and treatments can be done
with reduced body cavity invasion.
Some terms refer to the body region being examined or treated:
• Laparoscopy (laparoscopic surgery)
• Arthroscopy (arthroscopic surgery)
• Thoracoscopy (thoracoscopic surgery)
• Rhinoscopy
• Otoscopy
• Cystoscopy
• Endosurgery
• Minimal access surgery
• Less invasive surgery
• Endoscopic microsurgery
• Video assisted surgery
• Videoendoscopic surgery
• Telescopic surgery
Laparoscopic surgery, also called minimally invasive surgery (MIS), keyhole
surgery is a modern surgical technique in which operations in the abdomen are per-
formed through small incisions (usually 0.5–1.5 cm) as compared to the larger inci-
sions needed in laparotomy.
Keyhole surgery uses images displayed on TV monitors for magnification of
the surgical elements.
Laparoscopic surgery includes operations within the abdominal or pelvic cavi-
ties, whereas keyhole surgery performed on the thoracic or chest cavity is called tho-
racoscopic surgery. Laparoscopic and thoracoscopic surgery belong to the broader
field of endoscopy.
There are a number of advantages to the patient with laparoscopic surgery ver-
sus an open procedure. These include reduced pain due to smaller incisions and hem-
orrhaging, and shorter recovery time.
The key element in laparoscopic surgery is the use of a laparoscope. There are
two types: (1) a telescopic rod lens system, that is usually connected to a video cam-
era (single chip or three chip), or (2) a digital laparoscope where the charge-coupled
device is placed at the end of the laparoscope, eliminating the rod lens system. Also
attached is a fiber optic cable system connected to a 'cold' light source (halogen or
xenon), to illuminate the operative field, inserted through a 5 mm or 10 mm cannula
or trocar to view the operative field. The abdomen is usually insufflated, or essential-
ly blown up like a balloon, with carbon dioxide gas. This elevates the abdominal wall
above the internal organs like a dome to create a working and viewing space. CO2 is
used because it is common to the human body and can be absorbed by tissue and re-
moved by the respiratory system. It is also non-flammable, which is important be-
cause electrosurgical devices are commonly used in laparoscopic procedures.
It is difficult to credit one individual with the pioneering of the laparoscopic
approach. In 1902 Georg Kelling, of Dresden, Saxony, performed the first laparo-
scopic procedure in dogs and in 1910, Hans Christian Jacobaeus of Sweden reported
the first laparoscopic operation in humans. In the ensuing several decades, numerous
individuals refined and popularized the approach further for laparoscopy. The start of
computer chip television camera was a seminal event in the field of laparoscopy. This
technological innovation provided the means to project a magnified view of the oper-
ative field onto a monitor and, at the same time, freed both the operating surgeon's
hands, thereby facilitating performance of complex laparoscopic procedures. Prior to
its conception, laparoscopy was a surgical approach with very limited application,
used mainly for purposes of diagnosis and performance of simple procedures in gy-
necologic applications.
The first publication on Diagnostic Laparoscopy by Raoul Palmer appeared in
the early 1950s, followed by the publication of Frangenheim and Semm. Hans Lin-
dermann and Kurt Semm practised CO2 hysteroscopy during the mid-1970s. In 1972,
Clarke invented, published, patented, presented, and recorded on film laparoscopic
surgery, with instruments marketed by the Ven Instrument Company of Buffalo, New
York, USA. In 1975, Tarasconi, from the Department of Ob-Gyn of the University of
Passo Fundo Medical School (Brazil), started his experience with organ resection by
Laparoscopy (Salpingectomy), first reported in the Third AAGL Meeting, Hyatt Re-
gency Atlanta, November 1976 and later published in The Journal of Reproductive
Medicine in 1981. This Laparoscopic Surgical Procedure was the first Laparoscopic
organ resection reported in the Medical Literature. In 1981, Semm, from the Univer-
sitats Frauenklinik, Kiel, Germany, performed the first Laparoscopic Appendectomy.
Following his lecture on Laparoscopic Appendectomy, the President of the German
Surgical Society wrote to the Board of Directors of the German Gynecological socie-
ty suggesting suspension of Semm from medical practice. Subsequently, Semm sub-
mitted a paper on Laparoscopic Appendectomy to the American Journal of Obstetrics
and Gynecology, at first rejected as unacceptable for publication on the ground that
the technique reported on was ' unethical,' but finally published in the Journal Endos-
copy. The Abstract of his paper on "Endoscopic Appendectomy" can be found at
here. Semm established several standard procedures that were regularly performed,
such as ovarian cyst enucleation, myomectomy, treatment of ectopic pregnancy and
finally laparoscopic-assisted vaginal hysterectomy (nowadays termed as cervical in-
tra-fascial Semm hysterectomy). He also developed a medical instrument company
Wisap in Munich, Germany, which still produces various endoscopic instruments of
high quality. In 1985, he constructed the pelvi-trainer - laparo-trainer, a practical sur-
gical model whereby colleagues could practice laparoscopic techniques. Semm pub-
lished over 1000 papers in various journals. He also produced over 30 endoscopic
films and more than 20,000 colored slides to teach and inform interested colleagues
about his technique. His first atlas, More Details on Pelviscopy and Hysteroscopy
was published in 1976, a slide atlas on pelviscopy, hysteroscopy, and fetoscopy in
1979, and his books on gynecological endoscopic surgery in German, English, and
many other languages in 1984, 1987, and 2002.
Prior to 1990, the only specialty performing laparoscopy on a widespread basis
was gynecology, mostly for relatively short, simple procedures such as a diagnostic
laparoscopy or tubal ligation. The introduction in 1990 of a laparoscopic clip applier
with twenty automatically advancing clips (rather than a single load clip applier that
would have to be taken out, reloaded and reintroduced for each clip application) made
general surgeons more comfortable with making the leap to laparoscopic cholecystec-
tomies (gall bladder removal). On the other hand, some surgeons continue to use the
single clip appliers as they save as much as $200 per case for the patient, detract noth-
ing from the quality of the clip ligation, and add only seconds to case lengths.
There are two different formats for laparoscopic surgery. Multiple incisions are
required for technology such as the "Da Vinci" system, which uses a console located
away from the patient, with the surgeon controlling a camera, vacuum pump, saline
cleansing solution, cutting tools, etc. each located within its own incision site, but ori-
ented toward the surgical objective. The surgeon uses two PlayStation type controls
to manipulate the devices.
In contrast, requiring only a single small incision, the "Bonati system" (invent-
ed by Dr. Alfred Bonati), uses a single 5-function control, so that a saline solution
and the vacuum pump operate together when the laser cutter is activated. A camera
and light provide feedback to the surgeon, who sees the enlarged surgical elements on
a TV monitor. The Bonati system was designed for spinal surgery and has been pro-
moted only for that purpose.
Laparoscopic cholecystectomy is the most common laparoscopic procedure
performed. In this procedure, 5-10mm diameter instruments (graspers, scissors, and
clip applier) can be introduced by the surgeon into the abdomen through trocars (hol-
low tubes with a seal to keep the CO2 from leaking. Rather than a minimum 20 cm
incision as in traditional (open) cholecystectomy, four incisions of 0.5–1.0 cm will be
sufficient to perform a laparoscopic removal of a gallbladder. Since the gall bladder is
similar to a small balloon that stores and releases bile, it can usually be removed from
the abdomen by suctioning out the bile and then removing the deflated gallbladder
through the 1 cm incision at the patient's navel. The length of postoperative stay in
the hospital is minimal, and same-day discharges are possible in cases of early morn-
ing procedures.
In certain advanced laparoscopic procedures where the size of the specimen being
removed would be too large to pull out through a trocar site, as would be done with a
gallbladder, an incision larger than 10mm must be made. The most common of these
procedures are removal of all or part of the colon (colectomy), or removal of the kid-
ney (nephrectomy). Some surgeons perform these procedures completely laparoscop-
ically, making the larger incision toward the end of the procedure for specimen re-
moval, or, in the case of a colectomy, to also prepare the remaining healthy bowel to
be reconnected (create an anastomosis). Many other surgeons feel that since they will
have to make a larger incision for specimen removal anyway, they might as well use
this incision to have their hand in the operative field during the procedure to aid as a
retractor, dissector, and to be able to feel differing tissue densities (palpate), as they
would in open surgery. This technique is called hand-assist laparoscopy. Since they
will still be working with scopes and other laparoscopic instruments, CO2 will have
to be maintained in the patient's abdomen, so a device known as a hand access port (a
sleeve with a seal that allows passage of the hand) must be used. Surgeons that
choose this hand-assist technique feel it reduces operative time significantly versus
the straight laparoscopic approach. It also gives them more options in dealing with
unexpected adverse events (i.e. uncontrolled bleeding) that may otherwise require
creating a much larger incision and converting to a fully open surgical procedure.
Advantages
There are a number of advantages to the patient with laparoscopic surgery
versus an open procedure. These include:
• Reduced hemorrhaging, which reduces the chance of needing a blood
transfusion.

• Smaller incision, which reduces pain and shortens recovery time, as well
as resulting in less post-operative scarring.

• Less pain, leading to less pain medication needed.

• Although procedure times are usually slightly longer, hospital stay is


less, and often with a same day discharge which leads to a faster return
to everyday living.

• Reduced exposure of internal organs to possible external contaminants


thereby reduced risk of acquiring infections.

Risks
The most significant risks are from trocar injuries to either blood vessels or
small or large bowel. The risk of such injuries is increased in patients who have be-
low average body mass index or have a history of prior abdominal surgery. The initial
trocar is typically inserted blindly. While these injuries are rare, significant complica-
tions can occur. Vascular injuries can result in hemorrhage that may be life threaten-
ing. Injuries to the bowel can cause a delayed peritonitis. It is very important that
these injuries be recognized as early as possible.
Some patients have sustained electrical burns unseen by surgeons who are
working with electrodes that leak current into surrounding tissue. The resulting inju-
ries can result in perforated organs and can also lead to peritonitis. This risk is elimi-
nated by utilizing active electrode monitoring.
Many patients with existing pulmonary disorders may not tolerate pneumoperi-
toneum (gas in the abdominal cavity), resulting in a need for conversion to open sur-
gery after the initial attempt at laparoscopic approach.
Not all of the CO2 introduced into the abdominal cavity is removed through the
incisions during surgery. Gas tends to rise, and when a pocket of CO2 rises in the ab-
domen, it pushes against the diaphragm (the muscle that separates the abdominal
from the thoracic cavities and facilitates breathing), and can exert pressure on the
phrenic nerve. This produces a sensation of pain that may extend to the patient's
shoulders. For an appendectomy, the right shoulder can be particularly painful. In
some cases this can also cause considerable pain when breathing. In all cases, howev-
er, the pain is transient, as the body tissues will absorb the CO2 and eliminate it
through respiration.
Coagulation disorders and dense adhesions (scar tissue) from previous ab-
dominal surgery may pose added risk for laparoscopic surgery and are considered rel-
ative contra-indications for this approach.
NEW ADVANCES IN LAPAROSCOPIC SURGERY
The major limitations of standard laparoscopic techniques have been the fol-
lowing:
The human hand is a wonderful structure and provides multitude of different
functions during open surgery. This function is absent during standard laparoscopic
surgery since the abdomen is closed and the procedure is performed with long surgi-
cal instruments inserted from the outside into the abdomen.
Two dimensional image of the laparoscope: The image transmitted by the lapa-
roscopic camera that surgeon utilizes as his eyes is a two dimensional image. For
some procedures this is a major limitation because of the poor depth perception that is
associated with two dimensional images.
Retraction of internal organs: During open surgery insertion of the hand into
the abdomen allows the surgeon to move the intestine and other organs away from the
site of the surgery. During standard laparoscopic surgery the hand is not introduced
into the abdomen and introducing long thin instruments into the abdomen performs
the surgery. Retraction of internal organs is often a major problem for some proce-
dures.
Limitation of instruments: the standard instruments in laparoscopic surgery are
long thin instruments. These instruments are poorly suited for many complex laparo-
scopic procedures.
New technologies for advance laparoscopic surgery
Two new technologies that are particularly promising are: hand access devices
and robotic surgery.
Hand access devices. The human hand performs many functions during surgery
that are difficult to reproduce with laparoscopic instruments. The loss of the ability to
place the hand into the abdomen during traditional laparoscopic surgery has limited
the use of laparoscopy for complex abdominal surgery on the pancreas, liver and bile
duct. New laparoscopic hand-access devices that allow the surgeon to place a hand
into the abdomen during laparoscopic surgery and perform many of the different
functions with the hand that was previously possible only during open surgery. We
have utilized this new device to develop a variety of laparoscopic pancreatic, liver
and biliary procedures such as the Whipple operation, distal pancreatectomy and liver
resection that were not possible previously by standard laparoscopic techniques.
Robot-assisted surgery utilizing the Da Vinci computer robot system. Da Vinci
is a computer-assisted robotic system that expands a surgeon's capability to operate
within the abdomen in a less invasive way during laparoscopic surgery. Da Vinci sys-
tem allows greater precision and better visualization compared to standard laparo-
scopic surgery.
The operations with the Da Vinci System are performed with no direct mechan-
ical connection between the surgeon and the patient. The surgeon is remote from the
patient, working a few feet from the operating table while seated at a computer con-
sole with a three-dimensional view of the operating field. The physician operates two
masters (similar to joysticks) that control the two mechanical arms on the robot. The
mechanical arms are armed with specialized instruments with hand-like movements
which carry out the surgery through tiny holes in the patient’s abdomen. The arms
eliminate any hand tremor by the surgeon and offer motion scaling – allowing ex-
tremely precise movements within the patient.
The Cyber Knife is a frameless robotic radiosurgery system invented by John R. Ad-
ler, a Stanford University Professor of Neurosurgery and Radiation Oncology, and
Peter and Russell Schonberg of Schonberg Research Corporation. The two main ele-
ments of the CyberKnife are (1) the radiation produced from a small linear particle
accelerator and (2) a robotic arm which allows the energy to be directed at any part of
the body from any direction.
The CyberKnife system is a method of delivering radiotherapy, with the inten-
tion of targeting treatment more accurately than standard radiotherapy. The Cyber-
Knife system is used for treating benign tumors, malignant tumors and other medical
conditions.
Several generations of the CyberKnife system have been developed since its in-
itial inception in 1990. There are two essential features of the CyberKnife system that
set it apart from other stereotactic therapy methods.
Robotic mounting
The first is that the radiation source is mounted on a general purpose industrial
robot. Mounted on the Robot is a compact X-band linac that produces 6MV X-ray ra-
diation. The linac is capable of delivering approximately 600 cGy of radiation each
minute - a new 800 cGy / minute. The radiation is collimated using fixed tungsten
collimators (also referred to as “cones”) which produce circular radiation fields. At
present the radiation field sizes are: 5, 7.5, 10, 12.5, 15, 20, 25, 30, 35, 40, 50 and 60
mm. Mounting the radiation source on the robot allows near-complete freedom to po-
sition the source within a space about the patient. The robotic mounting allows very
fast repositioning of the source, which enables the system to deliver radiation from
many different directions without the need to move both the patient and source as re-
quired by current gantry configurations.

Image guidance
The image guidance system is the other essential item in the CyberKnife sys-
tem. X-ray imaging cameras are located on supports around the patient allowing in-
stantaneous X-ray images to be obtained.
6D skull
The original (and still utilized) method is called 6D or skull based tracking. The
X-ray camera images are compared to a library of computer generated images of the
patient anatomy. Digitally Reconstructed Radiographs (or DRR's) and a computer al-
gorithm determines what motion corrections have to be given to the robot because of
patient movement. This imaging system allows the CyberKnife to deliver radiation
with an accuracy of 0.5mm without using mechanical clamps attached to the patient's
skull. The use of the image guided technique is referred to as frameless stereotactic
radiosurgery. This method is referred to as 6D because corrections are made for the 3
translational motions (X, Y and Z) and three rotational motions. It should be noted
that it is necessary to use some anatomical or artificial feature to orient the robot to
deliver X-ray radiation, since the tumor is never sufficiently well defined (if visible at
all) on the X-ray camera images.
6D Skull tracking
Additional image guidance methods are available for spinal tumors and for tu-
mors located in the lung. For a tumor located in the spine, a variant of the image
guidance called Xsight-Spine is used. The major difference here is that instead of tak-
ing images of the skull, images of the spinal processes are used. Whereas the skull is
effectively rigid and non-deforming, the spinal vertebrae can move relative to each
other, this means that image warping algorithms must be used to correct for the dis-
tortion of the X-ray camera images. A recent enhancement to Xsight is Xsight-Lung
which allows tracking of some lung tumors without the need to implant fiduciary
markers.

Fiducial
For soft tissue tumors, a method known as fiducial tracking can be utilized.
Small metal markers (fiducials) made out of gold for bio-compatibility and high den-
sity to give good contrast on X-ray images are surgically implanted in the patient.
This is carried out by an interventional radiologist, or neurosurgeon. The placement
of the fiducials is a critical step if the fiducial tracking is to be used. If the fiducials
are too far from the location of the tumor, or are not sufficiently spread out from each
other it will not be possible to accurately deliver the radiation. Once these markers
have been placed, they are located on a CT scan and the image guidance system is
programmed with their position. When X-ray camera images are taken, the location
of the tumor relative to the fiducials is determined, and the radiation can be delivered
to any part of the body. Thus the fiducial tracking does not require any bony anatomy
to position the radiation. Fiducials are known however to migrate and this can limit
the accuracy of the treatment if sufficient time is not allowed between implantation
and treatment for the fiducials to stabilize.
Synchrony
The final technology of image guidance that the CyberKnife system can use is
called the Synchrony system. The Synchrony system is utilized primarily for tumors
that are in motion while being treated, such as lung tumors and pancreatic tumors.
The synchrony system uses a combination of surgically placed internal fiducials, and
light emitting optical fibers (markers) mounted on the patient skin. Since the tumor is
moving continuously, to continuously image its location using X-ray cameras would
require prohibitive amounts of radiation to be delivered to the patient’s skin. The
Synchrony system overcomes this by periodically taking images of the internal fidu-
cials, and predicting their location at a future time using the motion of the markers
that are located on the patient's skin. The light from the markers can be tracked con-
tinuously using a CCD camera, and are placed so that their motion is correlated with
the motion of the tumor.
A computer algorithm creates a correlation model that represents how the in-
ternal fiducial markers are moving compared to the external markers. The Synchrony
system is therefore continuously predicting the motion of the internal fiducials, and
therefore the tumor, based on the motion of the markers. The correlation model can
be updated at any time if the patient breathing becomes in any way irregular. The ad-
vantage of the Synchrony system is that no assumptions about the regularity or repro-
ducibility of the patient breathing have to be made.
To function properly, the Synchrony system requires that for any given correla-
tion model there is a functional relationship between the markers and the internal fi-
ducials. The external marker placement is also important, and the markers are usually
placed on the patient abdomen so that their motion will reflect the internal motion of
the diaphragm and the lungs.
RoboCouch
A new robotic six degree of freedom patient treatment couch called Ro-
boCouch has been added to the CyberKnife which provides the capability for signifi-
cantly improving patient positioning options for treatment.
Frameless
The frameless nature of the CyberKnife also increases the clinical efficiency. In
conventional frame-based radiosurgery, the accuracy of treatment delivery is deter-
mined solely by connecting a rigid frame to the patient which is anchored to the pa-
tient’s skull with invasive aluminum or titanium screws. The CyberKnife is the only
radiosurgery device that does not require such a frame for precise targeting. Once the
frame is connected, the relative position of the patient anatomy must be determined
by making a CT or MRI scan. After the CT or MRI scan has been made, a radiation
oncologist must plan the delivery of the radiation using a dedicated computer pro-
gram, after which the treatment can be delivered, and the frame removed. The use of
the frame therefore requires a linear sequence of events that must be carried out se-
quentially before another patient can be treated. Staged CyberKnife radiosurgery is of
particular benefit to patients who have previously received large doses of convention-
al radiation therapy and patients with gliomas located near critical areas of the brain.
Unlike whole brain radiotherapy, which must be administered daily over several
weeks, radiosurgery treatment can usually be completed in 1-5 treatment sessions.
Radiosurgery can be used alone to treat brain metastases, or in conjunction with sur-
gery or whole brain radiotherapy, depending on the specific clinical circumstances.
By comparison, using a frameless system, a CT scan can be carried out on any
day prior to treatment that is convenient. The treatment planning can also be carried
out at any time prior to treatment. During the treatment the patient need only be posi-
tioned on a treatment table and the predetermined plan delivered. This allows the
clinical staff to plan many patients at the same time, devoting as much time as is nec-
essary for complicated cases without slowing down the treatment delivery. While a
patient is being treated, another clinician can be considering treatment options and
plans, and another can be conducting CT scans.
The delivery of a radiation treatment over several days or even weeks (referred
to as fractionation) can also be beneficial from a therapeutic point of view. Tumor
cells typically have poor repair mechanisms compared to healthy tissue, so by divid-
ing the radiation dose into fractions the healthy tissue has time to repair itself between
treatments. This can allow a larger dose to be delivered to the tumor compared to a
single treatment.
Gamma Knife
One of the most widely known stereotactic radiosurgery systems is the Gamma
Knife. The Gamma Knife was originally developed by Lars Leksell, remains the gold
standard method for delivery of stereotactic radiosurgery to the brain. The Gam-
maKnife system uses 201 Cobalt-60 sources located in a ring around a central treat-
ment point ("isocenter"). The Gamma Knife system is equipped with a series of 4 col-
limators of 4mm, 8mm, 12mm and 16mm diameter, and is capable of submillimeter
accuracies. The Gamma Knife system does however require a head frame to be bolted
onto the skull of the patient, and is only capable of treating cranial lesions. As a result
of frame placement, treatment with Gamma Knife does not require real time imaging
capability as the frame does not allow movement during treatment. This is the reason
that the Gamma Knife system is likely to be more accurate than Cyber Knife. The
Cyberknife Society and Accuray maintain that there are no peer-reviewed published
papers that establish Gamma Knife as being more accurate than CyberKnife.
Basic literature:
1. Oxford Textbook of Surgery (3-Volume Set) 2nd edition (January 15, 2000):
by Peter J. Morris (Editor), William C. Wood (Editor) By Oxford Press
2. Sabiston Textbook of Surgery 17th edition by Courtney M. Townsend Jr.,
Kenneth L. Mattox, B. Mark, MD Evers, Kenneth L., MD Mattox, Courtney
Townsend, Daniel Beauchamp, B. Mark Evers, Kenneth Mattox W.B. Saun-
ders Company (June, 2004)
3. Schwartz´s Principles of Surgery 8th Edition F. Charles Brunicardi. Copyright
©2007 the McGraw-Hill Companies.
4. Hospital surgery/ Edited by L. Kovalchuk et al. - Ternopil: Ukrmedknyha,
2004. - 472 p.
Additional literature:
A. Robbins and Cotran Pathological Basis of Disease, Kumar et al; 8th Edition,
Saunders Elsevier Publishing, 2010
B. Diseases of the Heart, Pericardium, and Pulmonary Vasculature Bed. In: Libby P,
Bonow RO, Mann DL, and Zipes DP, eds. Braunwald's Heart Disease: A Text-
book of Cardiovascular Medicine. 8th ed. St. Louis, Mo: WB Saunders; 2007:
Chap. 61.

Tests for initial level of knowledge, keys for tests:

i. Endovascular surgery is:


A.Is joining damaged human tissues and restoration of the functioning of human
organs. It is used to dissect tissues and to stop or prevent bleeding when tissue
and blood vessels are incised.
B. is a form of minimally invasive surgery that was designed to access many re-
gions of the body via major blood vessels
C. is the reattachment of a completely detached body part.
D.Is a modern surgical technique in which operations in the abdomen are per-
formed through small incisions (usually 0.5–1.5 cm) as compared to the larger
incisions needed in laparotomy.
E. is a general term for surgery requiring an operating microscope.
ii. Replantation:
A.Is joining damaged human tissues and restoration of the functioning of human
organs. It is used to dissect tissues and to stop or prevent bleeding when tissue
and blood vessels are incised.
B. is a form of minimally invasive surgery that was designed to access many re-
gions of the body via major blood vessels
C. is the reattachment of a completely detached body part.
D.Is a modern surgical technique in which operations in the abdomen are per-
formed through small incisions (usually 0.5–1.5 cm) as compared to the larger
incisions needed in laparotomy.
E. is a general term for surgery requiring an operating microscope.

iii. Microsurgery:
A.Is joining damaged human tissues and restoration of the functioning of human
organs. It is used to dissect tissues and to stop or prevent bleeding when tissue
and blood vessels are incised.
B. is a form of minimally invasive surgery that was designed to access many re-
gions of the body via major blood vessels
C. is the reattachment of a completely detached body part.
D.Is a modern surgical technique in which operations in the abdomen are per-
formed through small incisions (usually 0.5–1.5 cm) as compared to the larger
incisions needed in laparotomy.
E. is a general term for surgery requiring an operating microscope.

iv. Laparoscopic surgery is:


A.Is joining damaged human tissues and restoration of the functioning of human
organs. It is used to dissect tissues and to stop or prevent bleeding when tissue
and blood vessels are incised.
B. is a form of minimally invasive surgery that was designed to access many re-
gions of the body via major blood vessels
C. is the reattachment of a completely detached body part.
D.Is a modern surgical technique in which operations in the abdomen are per-
formed through small incisions as compared to the larger incisions needed in
laparotomy.
E. is a general term for surgery requiring an operating microscope.

v. Welding is:
A.Is joining damaged human tissues and restoration of the functioning of human
organs. It is used to dissect tissues and to stop or prevent bleeding when tissue
and blood vessels are incised.
B. is a form of minimally invasive surgery that was designed to access many re-
gions of the body via major blood vessels
C. is the reattachment of a completely detached body part.
D.Is a modern surgical technique in which operations in the abdomen are per-
formed through small incisions as compared to the larger incisions needed in
laparotomy.
E. is a general term for surgery requiring an operating microscope.

vi. What statement about GammaKnife is not correct?:


A. The GammaKnife system uses any kind of penetrating wave.
B. The GammaKnife system uses 201 Cobalt-60 sources located in a ring around a
central treatment point ("isocenter").
C. The Gamma Knife system is equipped with a series of 4 collimators of 4mm,
8mm, 12mm and 16mm diameter, and is capable of submillimeter accuracies.
D. The Gamma Knife system does however require a head frame to be bolted onto
the skull of the patient, and is only capable of treating cranial lesions.
E. Gamma Knife system is likely to be more accurate than Cyber Knife.

vii. What statement about Cyber Knife is not correct?:


A.The Cyber Knife is a frameless robotic radiosurgery system.
B. The element of the CyberKnife is the radiation produced from a small linear
particle accelerator
C. The element of the CyberKnife is a robotic arm which allows the energy to be
directed at any part of the body from any direction.
D.The CyberKnife system is a method of delivering radiotherapy, with the inten-
tion of targeting treatment more accurately than standard radiotherapy.
E. Treatment with CyberKnife system does not require real time imaging capabil-
ity as the frame does not allow movement during treatment.

viii. What statement about Robot-assisted surgery (Da Vinci) is not correct?:
A. Da Vinci is a computer-assisted robotic system that expands a surgeon's capabil-
ity to operate within the abdomen in a less invasive way during laparoscopic sur-
gery.
B. Da Vinci system allows greater precision and better visualization compared to
standard laparoscopic surgery.
C. The operations with the Da Vinci System are performed with direct mechanical
connection between the surgeon and the patient.
D. The surgeon is remote from the patient, working a few feet from the operating
table while seated at a computer console with a three-dimensional view of the
operating field.
E. The physician operates two masters (similar to joysticks) that control the two
mechanical arms on the robot. The mechanical arms are armed with specialized
instruments with hand-like movements which carry out the surgery through tiny
holes in the patient’s abdomen.

ix. What statement about Magnetic resonance imaging (MRI) is correct?


A. Is a medical procedure in endoscopy is combined with ultrasound to obtain im-
ages of the internal organs in the chest and abdomen.
B. Is imaging by sections or sectioning, through the use of any kind of penetrating
wave.
C. is a medical imaging technique used in radiology to visualize detailed internal
structures that uses of nuclear resonance for imaging
D. Magnetic resonance imaging provides good spatial resolution (the ability to
distinguish two separate structures an arbitrarily small distance from each oth-
er)
E. It uses ionizing radiation in the radio frequency range.

x. What statement about CT is not correct?


A. CT provides good spatial resolution (the ability to distinguish two separate
structures an arbitrarily small distance from each other)
B. CT provides comparable resolution with far better contrast resolution (the abil-
ity to distinguish the differences between two arbitrarily similar but not identi-
cal tissues).
C. It uses ionizing radiation
D. CT scanning is not contraindicated in case of cardioverter-defibrillator.
E. CT scanning is not contraindicated in case of insulin pumps.

Keys for tests


1 2 3 4 5 6 7 8 9 10
B C E D A A E C C B

Tests for final level of knowledge, keys for tests:


1. What statement about laparoscopic cholecystectomy is not correct?
A. Laparoscopic cholecystectomy is the most common laparoscopic procedure
performed.
B. In this procedure, 5-10mm diameter instruments (graspers, scissors, and clip
applier) can be introduced by the surgeon into the abdomen through trocars
(hollow tubes with a seal to keep the CO2 from leaking.
C. Rather than a minimum 20 cm incision as in traditional (open) cholecystecto-
my, four incisions of 0.5–1.0 cm will be sufficient to perform a laparoscopic
removal of a gallbladder.
D. Since the gall bladder can’t be removed from the abdomen by suctioning out
the bile through the 1 cm incision at the patient's navel.
E. The length of postoperative stay in the hospital is minimal, and same-day dis-
charges are possible in cases of early morning procedures.

2. What is contra-indication for laparoscopic cholecystectomy?


A. Choledocholithiasis
B. Cholangitis
C. Acute cholecystitis
D. Adhesions in upper part of abdomen
E. Large stone in gallbladder

3. Significant risks from laparoscopic surgery are all EXCEPT:


A.Trocar injuries of blood vessels
B. Trocar injuries of small or large bowel.
C. Electrical burns of surrounding tissues.
D.Pulmonary disorders in patients with pulmonary diseases
E. Eventration

4. What does increase the risk of trocar injuries of small or large bowel during
laparoscopic surgery?
A. Coagulation disorders
B. Adhesions (scar tissue) from previous abdominal surgery.
C. Obesity
D. Malnutrition
E. Two dimensional image of the laparoscope.

5. The major limitations of standard laparoscopic techniques are following


EXCEPT:
A. Two dimensional image of the laparoscope.
B. Retraction of internal organs.
C. Limitation of instruments movement.
D. Adhesions (scar tissue) from previous abdominal surgery.
E. Bipolar coagulator.

6. There are advantages to the patient with laparoscopic surgery versus an open
procedure EXCEPT:
A. Smaller incision, which reduces pain and shortens recovery time, as well as re-
sulting in less post-operative scarring.
B. Less pain, leading to less pain medication needed.
C. Hospital stay is less, and often with a same day discharge which leads to a faster
return to everyday living.
D. Reduced exposure of internal organs to possible external contaminants thereby
reduced risk of acquiring infections.
E. Laparoscopic surgery is more suitable for patients with pulmonary diseases.

7. What statement about Welding technology is not correct?


A. Welding technology is used with success for joining damaged human tissues.
B. Welding technology is used to restoration of the functioning of human organs.
C. Welding technology is used to dissect tissues.
D. Welding technology is used to stop or prevent bleeding when tissue and blood
vessels are incised.
E. Welding technology is used only for joining tissues.

8. What statement about Replantation is not correct?


A. Replantation is the reattachment of a completely detached body part.
B. Fingers and thumbs are the most common but the ear, scalp, nose, face, arm
and penis have all been replanted.
C. Generally replantation involves restoring blood flow through arteries and veins,
restoring the bony skeleton and connecting tendons and nerves as required.
D. Survival of the amputated piece was not enough to ensure success of the
replant.
E. If an amputated specimen is not able to be replanted to its original location
entirely, this is means that the specimen is unreplantable.
9. What statement about Microsurgery is not correct?
A. Microsurgery is a general term for surgery requiring an operating microscope.
B. Microsurgery allows regain smaller blood vessels and nerves (typically 1 mm in
diameter)
C. Microsurgery allows transfer of tissue from one part of the body to another and
re-attachment of severed parts.
D. Microsurgery is not used in general surgery.
E. Microsurgery is used in ophthalmology, orthopedic surgery, gynecological
surgery, otolaryngology, neurosurgery, oral and maxillofacial surgery, and
pediatric surgery.

10.What statement about free tissue transfer is not correct?


A. Free tissue transfer is a surgical reconstructive procedure using microsurgery.
B. A region of "donor" tissue is selected that can be isolated on a feeding artery
and vein; this tissue is usually a composite of several tissue types (e.g., skin,
muscle, fat, bone).
C. Afer free tissue transfer the patients need immunosuppression.
D. The composite tissue is transferred (moved as a free flap of tissue) to the region
on the patient requiring reconstruction (e.g., mandible after oral cancer
resection, breast after cancer resection, traumatic tissue loss, congenital tissue
absence).
E. The vessels that supply the free flap are anastomosed with microsurgery to
matching vessels (artery and vein) in the reconstructive site.

Keys for tests

1 2 3 4 5 6 7 8 9 10
D D E B E E E E D C

Tasks for final level of knowledge


1. The patient 42 year-old after the attack of pain in right cubcostal area, yellowness
of skin and mucous membranes. Ultrasound investigation shows calculus in the
gall bladder, but common billiary duct and head of pancreas are not clearly visual-
izes. Endoscopic retrograde cholangiography is not made because patient has deep
parapapillar diverticula. Which the investigation should be used to make correct
diagnosis?
The answer is endoscopic ultrasonography
2. The patient 68-years-old is complaining on yellowness of skin and sclera, grey
stool, dark of the urine. Body temperature is increased to 38, 5 degrees. Jaundice
appeared a day after attack of pain in the right subcostal area. Abdomen soft, pain-
ful in the right subcostal area, liver is not increased. Symptom Murphy is positive.
Bilirubin - 88 mkmol/L (direct - 68, indirect - 20). Ultrasound investigation
shows: calculus in the increased gall bladder, common billiary duct is extended to
1, 2 sm. What is the most probable diagnosis? What is the most appropriate man-
agement?
The answer is асute cholecystitis, cholangitis, obstructive jaundice. The
most appropriate management is Endoscopic retrograde cholangiography + papillo-
sphincterotomy + lithoextraction, next – laparoscopic cholecystectomy.
3. The patient 48 years-old was complaining on yellowness of skin and sclera, grey
stool. Laparoscopic cholecystectomy was made two years. Postcholecystectomy
syndrome is diagnosed. To make correct diagnosis the patient was directed on
magnetic resonance imaging. But he was refused in investigation. What is reason?
The answer is existence of metal foreign bodies that was left after laparo-
scopic cholecystectomy is contraindication for magnetic resonance imaging.
4. 48-years-old woman with ischemic cardiomyopathy have progressive attacks of
stenocardia. On coronary angiography: interventricular coronary artery is stenosed
approximately 70% during 0, 7 sm. Other arteries without hemodynamically
changes. It is indication for:

The answer is endovascular balloon angioplasty


5. Patient 53- years-old, with ischemic heart disease is disturbed by stenocardia
attacks which are accompanied by pain in the area of heart that irradiate in the left
arm, sew and jaw. At selective coronography there is a narrowing of 50% road clear-
ance of coronal arteries. Which method is the best in this case?

The answer is endovascular stenting of coronal arteries

Materials on the independent teaching of students


Main tasks Notes (instructions)
Repeat:
1. Principles of using of ultrasonic
- To represent consecution and value of
waves
the diagnostic methods for abdominal
2. Principles of using of radiological
pathology as a table
waves
- To represent merits and demerits of the
3. Principles of using of endoscopic
Laparoscopic methods as a table
techniques
4. Principles of transplantation
-To determine the best radiological
methods depending on types of patholo-
Study:
gy
a. Types of radiological waves.
-To select more suitable mini invasive
b. Types of endoscopic techniques.
methods depending on types of patholo-
gy
Study guide #2
“Systemic inflammatory response syndrome (SIRS) in surgical patients. Patho-
genesis and significance in different diseases and traumas. The choice of diag-
nostic and curative tactics. Algorithm of intensive therapy implementation. Sur-
gical sepsis, septic shock. Antibacterial therapy of surgical infection. Surgical
aspects of AIDS.”

Overview.

The inflammatory response to injury and activation of cellular processes are


inherently designed to restore tissue function and eradicate invading microorganisms.
Local injuries of limited duration are usually followed by functional restoration with
minimal intervention. By contrast, major insults to the host are associated with an
overwhelming inflammatory response that, without appropriate and timely interven-
tion, can lead to multiple organ failure and adversely impact patient survival. There-
fore understanding how the inflammatory response is mobilized and ultimately con-
trolled provides a functional framework upon which interventions and therapeutics
are formulated for the surgical patient. The maturation of minimally invasive tech-
niques for major surgery during the last decade has brought complementary perspec-
tives to the injury response paradigm, and the immunologic benefits for these surgical
approaches are undergoing validation. Furthermore, the sequencing of the human ge-
nome and available technology such as deoxyribonucleic acid (DNA) microarray
analysis potentially affords surgeons additional tools to profile the genetic mecha-
nisms governing the host response to injury.

Infections with HIV leading to acquired immune deficiency syndrome (AIDS)


have been the focus of considerable academic attention during the past 20 years. This
infection is an indolent process that potentially requires a full decade, even in the un-
treated patient, from the time of acute infection before immunosuppression becomes
clinically significant. Surgical problems in the HIV patient include abnormal presen-
tation of common illnesses but also include unique problems that are consequences of
the acquired immunosuppression.

Educational aims:
1. Interrogation and clinical inspection of patients with SIRS, sepsis.
2. To determine the etiologic and pathogenic factors of SIRS, sepsis.
3. To develop a plan of examination of the patients with SIRS, sepsis.
4. To estimate results of laboratory data.
5. To make and formulate a diagnosis of SIRS, sepsis.
6. To estimate efficiency of treatment and prognosis of disease.
7. The stages of AIDS, clinical picture, diagnosis, surgical aspects.
A student must know:
8. Etiology and pathogenesis of SIRS, sepsis.
9. Manifestation of SIRS, sepsis.
10. Modern methods of diagnosis of SIRS, sepsis.
11. Significance in different diseases and traumas
12. The choice of diagnostic and curative tactics.
13. Algorithm of intensive therapy implementation.
14. Surgical aspects of AIDS.
A student must be able to:
1. Collect and estimate anamnesis of the patients with SIRS, sepsis.
2. Use methods of examination patients with SIRS, sepsis.
3. Set the algorithm of examination patient with SIRS, sepsis.
4. Estimate the results of laboratory, instrumental inspection of patients.
5. Determine optimum medical tactics for a concrete patient (conservative, surgi-
cal), and in the case of SIRS, sepsis, with AIDS complicated surgical pathology.
6. To prove necessity of pre-operation preparation of patient and to define the op-
eration of choice depending on localization, age, sex.
Terminology.
Term Definition
the pathologic reaction whereby fluid and circulating leuko-
Infection cytes accumulate in extravascular tissues in response
to injury or infection. Identifiable source of microbial insult
are extremely complex processes involving numerous differ-
ent cell types as well as hundreds of different humoral media-
tors
Two or more of following criteria
SIRS • Temperature ≥38°C or ≤36°C
• Heart rate ≥90 beats/min
• Respiratory rate ≥20 breaths/min or PaCO2 ≤32 mm
Hg or mechanical ventilation
• White blood cell count ≥12,000/L or ≤4000/L or ≥10%
band forms

Sepsis SIRS caused by infection (identifiable source of infection +


SIRS)
Severe sepsis is characterized as sepsis (defined above) combined with the
presence of new-onset organ failure.
is a state of acute circulatory failure identified by the presence
of persistent arterial hypotension (systolic blood pressure <90
Septic shock mm Hg) despite adequate fluid resuscitation, without other
identifiable causes (sepsis + cardiovascular collapse (requir-
ing vasopressor support))
is a blood-borne infection that is transmitted to the suscepti-
HIV ble host after percutaneous or a mucus membrane exposure to
infected blood or body fluids.
AIDS is infections with HIV leading to acquired immune deficiency
syndrome

Content:

Conceptually, the systemic response to injury can be broadly compartmental-


ized into two phases: (1) a proinflammatory phase characterized by activation of cel-
lular processes designed to restore tissue function and eradicate invading microorgan-
isms, and (2) an anti-inflammatory or counter regulatory phase that is important for
preventing excessive proinflammatory activities as well as restoring homeostasis in
the individual. While the terminologies that describe the various facets of systemic
inflammation are often used interchangeably, there are distinct criteria for each term.

Systemic phase of infection: sepsis

If local circumscription of infection is impossible either by removing the


bacteria or by abscess formation, micro-organisms eventually invade the bloodstream
and may reach distant organs. The presence of bacteria in the bloodstream
(bacteremia) occurs transiently in healthy individuals too. In patients, bacteria often
will be found on the intravascular portion of catheters. Nontoxin-producing, mostly
nonmultiplying bacteria can sometimes be isolated by blood culture, but these cause
no or only mild systemic symptoms. Bacteremia may, however, progress to systemic
disease, especially in immunocompromised and postoperative patients. If the
condition persists and is associated with multiplication of bacteria in the bloodstream,
and large numbers of bacteria die as they are attacked by host defense mechanisms,
then large quantities of bacterial cell-wall structures (endotoxins) are liberated and a
serious state of infection termed sepsis ensues.

Sepsis is characterized not only by invasion and multiplication in the


bloodstream of large numbers of bacteria but also by the potential for subsequent
sudden overload of the host with endotoxins and cytokines, leading to septic shock.
Key to this process is a small molecule – nitric oxide – that when excessively
produced will block mitochondrial energy generation by inhibiting Krebs' cycle
enzymes and cells are running out of fuel for life. Clinically we then observe
sequential organ dysfunction. Sepsis is the clinical, symptomatic state resulting from
the host response to bacteremia. Liberated bacterial exo- and endotoxins are
deleterious to many organ functions; equally, cytokine mediators of host defenses are
potentially damaging if their downregulation fails. If not treated successfully, the
patient may die immediately of septic shock or later following multisystem organ
failure: 1 ng endotoxin/kg body wt results in irreversible shock and death within 2 h.
Clinical symptoms of sepsis include the following. Fever is usually is high, spiking,
and accompanied by chills. Tachycardia accompanies or precedes the fever and is
proportional to it. The total leukocyte count may not be particularly abnormal in
sepsis and may even be low, due to the consumption of polymorphs. The differential
count is more reliable: there is always a shift to the left. Petechial lesions may be seen
in the skin or conjunctiva of patients suffering from sepsis caused by streptococci,
meningococci, or pseudomonads. Anemia secondary to hemolysis may appear rapidly
when sepsis is due to staphylococci, pseudomonads, coliforms, or clostridia. During
the initial, hyperdynamic phase of septic shock the peripheral vasodilation is
explained by a circulatory response that aims to compensate for the inability of cells
to use oxygen. The last stage of septic shock is hypodynamic, due to cell death.
Shock is common in sepsis caused by Gram-negative organisms, but occurs relatively
less often with Gram-positive infections. Metastatic abscesses, especially of the bone,
brain, or spleen, are not unusual after a septic episode: any injured tissue is easily
infected during sepsis. Diagnosis is aided by a high index of suspicion.

Thermolabile exotoxins are released by living bacteria, particularly the Gram-


positive; thermostable endotoxins are released by all bacteria after death. Endotoxins
are complex moieties of high molecular weight consisting of phospholipids,
polysaccharides, and proteins derived from the outer cell wall, particularly of Gram-
negative rods such as Escherichia coli. Clinically measurable effects of endotoxin
include fever, consumptive coagulopathy, increased vagotonus, hyperglycemia
followed by hypoglycemia, leucopenia or leukocytosis, increased plasma lipids,
release of hepatic enzymes, thrombocytopenia, and a reduced serum iron. Low doses
of endotoxin primarily affect the reticuloendothelial system. Animal studies have
shown a marked reduction in the clearance of particulates such as colloidal carbon
during endotoxemia. Mediators such as collagenases, pyrogenic prostaglandins, and
coagulation factors are released from macrophages; after 7 days, antibodies against
endotoxin are produced. Endotoxins act directly on the hypothalamic temperature-
regulation center to cause fever, reinforcing the activity of pyrogenic substances
released from dying neutrophils. Erythropoiesis is shifted from the bone marrow to
the spleen, resulting in leucopenia followed by leukocytosis after 2 to 6 h. In small
doses, endotoxins increase phagocytic activity and bacterial killing.
Thrombocytopenia, accompanied by aggregation and lysis of thrombocytes, results in
the release of ADP, vasoactive amines, histamine, serotonin, and platelet factor III,
which in turn may lead to consumptive coagulopathy. In the extrinsic coagulation
system, endotoxins cause release of a tissue factor derived from macrophages, as well
as platelet factors and thromboplastins. In the intrinsic system, factor XII (Hageman
factor) is activated, leading to disseminated intravascular coagulation. Endotoxin has
a profound effect on metabolism. Initially it induces hyperglycemia, which is
followed after several hours by hypoglycemia. Hyperlipidemia results from altered
metabolism of free fatty acids, cholesterol, phospholipids, and triglycerides. Protein
synthesis by the liver is stimulated; lactate dehydrogenase, transaminases, and
phosphokinases are released, increasing their serum concentrations. Release of
adrenocorticotrophic hormone, cortisone, and growth hormone is increased;
thyrotropin and luteinizing hormone are not affected. Plasma iron and total iron-
binding capacity are reduced. A vagotonic effect results in loss of thirst and appetite,
stomach emptying is delayed, and diarrhea may occur.

Pathogenesis of inflammation

Central Nervous System Regulation of Inflammation. The central nervous


system, operating through autonomic signaling, has an integral role in regulating the
inflammatory response that is primarily involuntary. Classically, the autonomic sys-
tem regulates heart rate, blood pressure, respiratory rate, gastrointestinal motility, and
body temperature. An additional role of the autonomic nervous system is to regulate
inflammation in a reflex manner, much like the patellar tendon reflex. Inflammation
originating from a specific location sends afferent signals to the hypothalamus, which
in turn rapidly relays opposing anti-inflammatory messages to the site of inflamma-
tion to reduce inflammatory mediator release by immunocytes. Tracey and colleagues
have further linked reflex inhibition of inflammation to the parasympathetic signaling
pathway whereby acetylcholine, the primary neurotransmitter of the parasympathetic
system, reduces tissue macrophage activation. Furthermore, cholinergic stimulation
directly reduces tissue macrophage release of the proinflammatory mediators TNF-α,
IL-1, IL-18, and high mobility group protein (HMG-1), but not the anti-inflammatory
cytokine IL-10. The attenuated inflammatory response induced by cholinergic stimuli
was further validated by the identification of acetylcholine (nicotinic) receptors on
tissue macrophages. Vagal stimulation reduces heart rate, increases gut motility, di-
lates arterioles, and causes pupil constriction, as well as regulates inflammation. Un-
like the humoral anti-inflammatory mediators that are released into the circulation
and allowed to travel to a site of injury, signals discharged from the vagus nerve are
precisely targeted at the site of injury or infection. Moreover, this cholinergic signal-
ing occurs rapidly in real time. From the available preclinical studies, it can be pro-
posed that impaired cholinergic activity from the vagus nerve portends a greater pro-
inflammatory response in patients who are critically ill.

Hormonal Response to Injury. Hormones are chemically classified as polypep-


tides (e.g., cytokines, glucagon, and insulin), amino acids (e.g., epinephrine, seroto-
nin, and histamine), or fatty acids (e.g., glucocorticoids, prostaglandins, and leukotri-
enes). Most hormone receptors generate signals by one of three major pathways,
which overlap. Specifically, these receptor pathways are (1) receptor kinases such as
insulin and insulin-like growth factor receptors, (2) guanine nucleotide-binding or G-
protein receptors such as neurotransmitter and prostaglandin receptors, and (3) lig-
and-gated ion channels which permit ion transport when activated. Upon activation
of membrane receptors, secondary signaling pathways are often utilized to amplify
the initial stimuli. Hormone signals are further mediated by intracellular receptors
with binding affinities for both the hormone itself, as well as for the targeted gene se-
quence on the DNA. These intracellular receptors may be located within the cytosol
or may already be localized in the nucleus, bound to the DNA. The classic example of
a cytosolic hormonal receptor is the glucocorticoid (GC) receptor. Intracellular GC
receptors are maintained in the cytosol by linking to the stress-induced protein, heat
shock protein (HSP). When the glucocorticoid ligand binds to the GC receptor, the
dissociation of HSP from the receptor activates the receptor-ligand complex and is
transported to the nucleus. Virtually every hormone of the hypothalamic-pituitary-
adrenal (HPA) axis influences the physiologic response to injury and stress, but some
with direct influence on the inflammatory response or immediate clinical impact will
be highlighted.

Mediators of Inflammation

Cytokines appear to be the most potent mediators of the inflammatory re-


sponse. When functioning locally at the site of injury or infection, cytokines eradicate
invading microorganisms and promote wound healing. However, overwhelming pro-
duction of proinflammatory cytokines in response to injury can cause hemodynamic
instability (i.e., septic shock) or metabolic derangements (i.e., muscle wasting). If un-
controlled, the outcome of these exaggerated responses is end-organ failure and
death. The production of anti-inflammatory cytokines as part of the inflammation
cascade serves to oppose the excessive actions of proinflammatory cytokines. How-
ever, inappropriate anti-inflammatory mediator release may render the patient im-
munocompromised and susceptible to overwhelming infections.

Heat Shock Proteins. Stimuli such as hypoxia, trauma, heavy metals, local
trauma, and hemorrhage all induce the production of intracellular heat shock proteins
(HSPs). HSPs are intracellular protein modifiers and transporters that are presumed to
protect cells from the deleterious effects of traumatic stress. The formation of HSPs
requires gene induction by the heat shock transcription factor.

Reactive oxygen metabolites are short-lived, highly reactive molecular oxygen


species with an unpaired outer orbit. They cause tissue injury by oxidation of unsatu-
rated fatty acids within cell membranes. Oxygen radicals are produced by complex
processes that involve anaerobic glucose oxidation coupled with the reduction of ox-
ygen to superoxide anion. Superoxide anion is an oxygen metabolite that is further
metabolized to other reactive species such as hydrogen peroxide and hydroxyl radi-
cals. Activated leukocytes are potent generators of reactive oxygen metabolites. Cells
are not immune to damage by their own reactive oxygen metabolites, but are general-
ly protected by oxygen scavengers that include glutathione and catalases.

Kallikrein-Kinin System. Bradykinins are potent vasodilators that are produced


through kininogen degradation by the serine protease kallikrein. Kallikrein exists in
blood and tissues as inactive prekallikrein that is activated by various factors such as
Hageman factor, trypsin, plasmin, factor XI, glass surfaces, kaolin, and collagen. Ki-
nins increase capillary permeability and tissue edema, evoke pain, inhibit gluconeo-
genesis, and increase bronchoconstriction. They also increase renal vasodilation and
consequently reduce renal perfusion pressure. The resulting increase in renin for-
mation activates sodium and water retention via the renin-angiotensin system.

The neurotransmitter serotonin (5-hydroxytryptamine, 5-HT) is a tryptophan


derivative that is found in chromaffin cells of the intestine and in platelets. Patients
with midgut carcinoid tumors often secrete 5-HT in excess. This neurotransmitter
stimulates vasoconstriction, bronchoconstriction, and platelet aggregation. Serotonin
is also a myocardial chronotrope and inotrope. Although serotonin is clearly released
at sites of injury, its role in the inflammatory response is unclear.

Histamine is derived from histidine and stored in neurons, skin, gastric muco-
sa, mast cells, basophils, and platelets. Histamine release is activated by increased
calcium levels. There are two receptor types for histamine binding. H1 binding stimu-
lates bronchoconstriction, intestinal motility, and myocardial contractility. H2 binding
inhibits histamine release. Both H1 and H2 receptor activation induce hypotension, pe-
ripheral pooling of blood, increased capillary permeability, decreased venous return,
and myocardial failure. The rise in histamine levels has been documented in hemor-
rhagic shock, trauma, thermal injury, endotoxemia, and sepsis.

Pathogens in surgical infections

This discussion of pathogens commonly responsible for surgical infections is


not intended to be a complete review. Rather, it focuses on some broad distinctions
and classifications that help organize the vast body of data concerning the usual bac-
terial flora of different surgical infections and the antibiotic susceptibility patterns of
these pathogens. Bacteria important in surgical infections are broadly divided into
aerobic and facultative bacteria in one group and anaerobic bacteria in the other; into
gram-positive and gram-negative bacteria; and into bacilli (rods) and cocci. Most in-
fections presenting in surgical patients are caused by endogenous bacteria. Specific
bacteria are found in specific parts of the body, and the exposed anatomic areas dur-
ing a surgical procedure are usually the source of microorganisms that cause infec-
tion. It is helpful to know the normal microbial flora of the body, since this helps di-
rect prophylactic antibiotics, start intelligent empiric therapy, and suspect the origin
of an unknown source of infection in patients with positive blood cultures. It is also
helpful to be familiar with the different classifications of bacteria since it can take up
to 72 hours for a final culture to give the result as a specific bacteria; however, Gram
stain and biochemical tests can help in providing earlier guidance regarding which
group of bacteria may be responsible for an infection.

Gram-positive cocci of importance to surgeons include staphylococci and


streptococci. Staphylococci are divided into coagulase-positive and coagulase-
negative strains. Coagulase-positive staphylococci are S. aureus and are the most
common pathogen associated with infections in wounds and incisions not subject to
endogenous contamination. Coagulase-positive staphylococci should be assumed re-
sistant to penicillin and require treatment by a penicillinase-resistant antibiotic. Ex-
tensive use of penicillinase-resistant β-lactam antibiotics in the past has encouraged
the emergence of MRSA. These organisms do not seem to have intrinsic pathogenici-
ty greater than that of other staphylococci, but they are more difficult to treat because
of antibiotic resistance. The prevalence of MRSA varies considerably by geographic
region but has been increasing during the past 2 decades. MRSA initially was seen
primarily in hospitalized patients but is now seen in an increasing number of commu-
nity-acquired infections. The incidence of MRSA recovery is increased in patients
coming from long-term care facilities, previously hospitalized or treated with antibi-
otics, and those with diabetes or on dialysis. MRSA must be treated with vancomy-
cin, quinupristin/dalfopristin, or linezolid. Recent years have seen the introduction of
S.aureus strains with decreased susceptibility to vancomycin, and, more recently, of
S. aureus strains with high-level resistance to vancomycin. If the history of other
pathogens and antimicrobial agents repeats itself, the number of such strains will in-
crease in the future.

For many years, coagulase-negative staphylococci were considered contami-


nants and skin flora incapable of causing serious disease. However, in the correct
clinical setting, coagulase-negative staphylococci can cause serious disease. This is
most common in patients who have been compromised by trauma, extensive surgery,
or metabolic disease and who have invasive vascular devices in place. Coagulase-
negative staphylococci are the most common organisms recovered in nosocomial bac-
teremia and are frequently associated with clinically significant infections of intra-
vascular devices. Coagulase-negative staphylococci are also found in endocarditis,
prosthetic joint infections, vascular graft infections, and postsurgical mediastinitis.
Most coagulase-negative staphylococci are methicillin resistant. Although most of the
infections associated with intravascular devices are cured simply by removing the de-
vice, if empiric antibiotic therapy is indicated, vancomycin, quinupristin/dalfopristin,
or linezolid should be chosen. The streptococcal species include β-hemolytic strepto-
cocci (especially group A or S.pyogenes), S.pneumoniae, and other α-hemolytic
streptococci. These species initially were uniformly sensitive to penicillin G and al-
most all other β-lactam antibiotics. Penicillin-resistant S.pneumoniae is now found in
most urban communities. The β-hemolytic streptococci alone, although not common-
ly recovered from soft tissue wounds, can cause life threatening infections. The other
α-hemolytic streptococci or viridans streptococci rarely are significant pathogens in a
surgical setting. They are commonly found on mucous membranes and skin and may
be recovered from the peritoneal cavity after upper gastrointestinal perforations but
are almost never found as the sole cause of significant surgical infections. The precise
significance of enterococci (group D streptococci) in surgical infections is controver-
sial. Enterococci are commonly recovered as part of a mixed flora in intra-abdominal
infections. It is rare to recover enterococci alone from a surgical infection. Enterococ-
cal bacteremia in association with a surgical infection carries a grave prognosis. The
stimulus for discussing the pathogenic significance of enterococci derives from the
relative resistance of these species to antibiotic therapy. No single antibiotic is relia-
bly effective for eradicating deep-seated infections or bacteremia. The most effective
antibiotic combination for treating enterococcal infections is gentamicin combined
with either ampicillin (or another advanced-generation penicillin) or vancomycin.

Aerobic and Facultative Gram-Negative Rods. A great variety of gram-


negative rods are associated with surgical infections. Most fall into the family Enter-
obacteriaceae. These are all facultative anaerobic bacteria and include the familiar
genera Escherichia, Proteus, and Klebsiella. These three genera (easy gram-negative
rods) are considered together because they are relatively common in mixed surgical
infections and because they are relatively sensitive to a broad variety of antibiotics,
especially second-generation cephalosporins. Other genera within the Enterobacteri-
aceae that are also common in surgical infections include Enterobacter, Morganella,
Providencia, and Serratia. These genera (difficult gram-negative rods) commonly ex-
hibit greater intrinsic antimicrobial resistance. Empiric antibiotic therapy directed at
these organisms requires a third-generation cephalosporin, one of the expanded-
spectrum penicillin, a monobactam, carbapenems, quinolone, or aminoglycoside. In
many locales these organisms have acquired extended-spectrum β-lactamase enzymes
that are capable of inactivating even third-generation cephalosporins. These organ-
isms are more common in hospital-acquired and postoperative surgical infections.
Gram negative rods recovered from infections originating in the community, such as
uncomplicated appendicitis or diverticulitis, are less likely to involve antibiotic-
resistant strains. Obligate aerobic gram-negative rods that can be found in surgical in-
fections include Pseudomonas and Acinetobacter species. These organisms are most
commonly found in hospital-associated pneumonias in surgical patients but may also
be recovered from the peritoneal cavity or severe soft tissue infections. These species
are often antibiotic resistant and require treatment with specific antipseudomonal an-
tibiotics such as ceftazidime, cefepime, aztreonam, imipenem/cilastatin, meropenem,
ciprofloxacin, an acylureido-penicillin, or an aminoglycoside. Acinetobacter species
are resistant to aztreonam. A significant proportion of these species exhibit strains re-
sistant even to the most effective antibiotics, and patients with such pathogens are
probably best treated empirically with two antibiotics until in vitro susceptibility test-
ing becomes available. Even after susceptibility data are known, critically ill patients
may benefit from treatment with two effective agents. Bacteria from both of these
genera have a tendency to develop resistance to antibiotics during therapy. Although
using two agents may not reduce this process, it does leave the patient with at least
one effective drug when it occurs. Stenotrophomonas maltophilia (previously Pseu-
domonas or Xanthomonas maltophilia) is uniformly resistant to imipenem and mero-
penem and is most commonly encountered as an emerging organism when one of
these carbapenems is used for empiric treatment of a serious infection.

Anaerobic bacteria are the most numerous inhabitants of the normal gastroin-
testinal tract, including the mouth. The most common anaerobic isolate from surgical
infections is Bacteroides fragilis. B.fragilis and Bacteroides thetaiotaomicron are two
common anaerobic species with significant resistance to many β-lactam antibiotics.
The most effective antibiotics against these species are metronidazole, clindamycin,
chloramphenicol, imipenem, meropenem, and ertapenem and the combinations of a
penicillin and a β-lactamase inhibitor (ticarcillin/clavulanate, ampicillin/sulbactam,
and piperacillin/tazobactam). Other anaerobic species commonly recovered from sur-
gical infections but with less significant bacterial resistance patterns include Bac-
teroides melaninogenicus and most of the anaerobic cocci. The other important genus
of anaerobic bacteria found in surgical infections is Clostridium, previously men-
tioned in the discussion of necrotizing soft tissue infections. Although they can sur-
vive for variable periods while exposed to oxygen, they require an anaerobic envi-
ronment for growth and invasion and for elaboration of the toxins that account for
their dramatic virulence in soft tissue infections. The Clostridium species are all
gram-positive, spore-forming rods. However, when present in human infections, they
do not form spores, so Gram-stained material from a soft tissue infection shows
gram-positive rods without spores. Clostridium difficile belongs to this family, and
Clostridium tetani is responsible for tetanus. The prevention of tetanus is accom-
plished solely through active and passive immunization, not through antibiotic admin-
istration. Anaerobic bacteria have a special importance in relation to surgical infec-
tions. These strains grow only in settings with a low oxidation-reduction potential,
which is incompatible with the survival of mammalian tissue. Thus, the recovery of
anaerobes from a soft tissue infection or even from the blood implies their growth and
multiplication in a focus of dead tissue. The predominant source of anaerobic bacteria
is the gastrointestinal tract; thus, an anaerobic infection implies a defect in the ana-
tomic integrity of the gastrointestinal tract. Both of these conditions (dead tissue and
a defect in the gastrointestinal tract) require surgical correction, so most anaerobic in-
fections (other than lung abscess) require surgical intervention. Certainly an anaero-
bic bacteremia should always prompt a search for an abscess or for an enteric lesion
that requires surgical intervention.

Diagnosis

History and physical examination The early accurate diagnosis of surgical


infections is essential: delayed treatment can result in dissemination, verwhelming
sepsis, and multisystem organ failure. The history and physical examination are the
surgeon's most important diagnostic tools. The classic signs of tumor, rubor, calor,
dolor and functio laesa are indicative of localized surgical infections. Clinical
symptoms of systemic sepsis include disturbed sensorium, tachypnea, tachycardia,
hypotension, fever, oliguria, and high-output heart failure. In postoperative patients,
the sudden appearance of tachypnea and hypotension suggests Gram-negative sepsis.
This condition has a potential mortality of 30 to 50 per cent, but early diagnosis and
treatment markedly improves the chances of survival. The entire body must be
examined; all dressings should be removed. Inspection and palpation of a suspicious
area may reveal the first three of the classical signs of infection. Removal of the
dressing around an intravenous cannula may reveal purulent drainage or
thrombophlebitis. Rectal examination may show tenderness and induration as signs of
a developing pelvic abscess. Auscultation of the chest may reveal the presence of
pneumonia before it is evident on a chest radiograph. The patient should be examined
for clues to the source of the infection, such as pain or redness in the surgical wound
or at an intravenous infusion site, or purulent sputum, cough, pleuritic pain, rales, or
dullness in the chest, diarrhea, dysuria, or flank pain. A foul-smelling odor may lead
to the site of an anaerobic infection. Pain in the shoulder and an immobile diaphragm
suggest a subphrenic abscess. A pelvic or prostatic mass on rectal examination may
indicate an abscess, and headache or nuchal rigidity may indicate an infection of the
central nervous system.

Hematology, urinalysis, and radiologyMost bacterial infections produce


leukocytosis and, more importantly, a shift to the left in the differential count or a
relative lymphopenia. This increase in the proportion of the more immature forms of
polymorphonuclear leukocytes may signal infection before an abnormal total
leukocyte count is evident. The differential count may also reveal lymphocytosis in
viral infections, monocytosis in tuberculosis, eosinophilia in parasitic infections or
hypersensitivity reactions (drug allergy), and toxic degranulation of leukocytes in
acute bacterial infection. A low white count or a leukemoid response (a total white
count of over 25 000 cells/mm ) may be seen in sepsis in general, and in
pneumococcal pneumonia, liver abscess or cholangitis, infected pancreatic necrosis,
necrotic bowel, or retroperitoneal phlegmon in particular. Leukopenia is a sign of
overwhelming bacterial infection and carries a bad prognosis. Viral infection, typhoid
perforation of the bowel, or tuberculosis may also present with leukopenia. Anemia
may be associated with infection caused by bacteria, such as Clostridium perfringens,
group A streptococci, or coagulase-positive staphylococci, that produce hemolytic
enzymes. Routine chest films may reveal generalized or focal atelactasis, or may
indicate intra-abdominal infection through signs of gastrointestinal leakage or free air
identified under the diaphragm. In the investigation of patients with suspected intra-
abdominal infection, flat, upright, and decubitus films may reveal a localized air–
fluid level, suggesting an intra-abdominal abscess, or a spreading air-bubble pattern
suggestive of infection with a gas-producing organism. Specialized radiologic
procedures may be helpful in confirming the diagnosis of intra-abdominal abscess.
These studies include ultrasonography and computed tomography (CT). Although a
gallium scintiscan may be helpful in special circumstances, this examination is
subject to appreciable error and is difficult to interpret in a patient who has had a
recent operation.
Bacteriology Observation of exudates and secretions such as wound drainage,
urine, and sputum for odor, color, and consistency may be useful in diagnosis. Grape-
like odors occur with pseudomonal infections, urea-like odors with Proteus
infections, and feculent odors with anaerobic organisms such as Bacteroides,
fusobacteria, clostridia, and peptostreptococci. A Gram stain offers the earliest clue to
the cause of an infection, particularly when a specific monobacterial infection is
suspected. Since surgical infections are mostly due to multiple infecting organisms
that are obligate or facultative anaerobes, the Gram stain usually shows a variety of
pathogenic bacteria. Note should be taken of the numbers of polymorphonuclear
leukocytes on the slide (few, many, loaded) and whether organisms can be seen inside
them. Acid-fast and fungal stains can be used if such infections are likely.

Technique of obtaining the specimen Purulent material from the deepest


aspect of the wound should be aspirated into a syringe and any air evacuated. Pus is
the best medium in which to preserve bacteria for transport to the laboratory. The
capped syringe is sent for aerobic and anaerobic culture, and for assay of antibiotic
sensitivity. Alternatively, a moist swab can be used to obtain bacteria from a site of
suspected infection. Ideally, anaerobic specimens should be transported immediately
in a CO -filled tube and plated within 1 h of 2 sampling; fastidious organisms may
otherwise die, resulting in a false-negative culture result. If the specimen is held
overnight, it should be placed in an anaerobic sterile vial or tube; under no
circumstances should an anaerobic specimen be refrigerated. Generally speaking, E.
coli (aerobe) and Bacteroides fragilis (anaerobe) are the usual causes of wound
infection following gastrointestinal or gynecologic operations, while streptococci,
staphylococci, and peptostreptococci are the usual causative organisms when intra-
abdominal viscera have not been resected or opened. Blood cultures are helpful in
guiding the specific antibiotic therapy of serious infections if the empirically started,
initial antibiotics fail. Following careful disinfection of the venepuncture site with an
iodophor preparation, blood samples should be obtained for aerobic and anaerobic
culture. Blood should not ordinarily be drawn for culture through an existing
intravenous needle or catheter. It is important to obtain a number of blood cultures
from different sites and at different times. Once the patient chills and a fever spike is
observed, most bacteria have already been killed by host defense mechanisms and
blood cultures will be negative. It is possible, however, to predict the time of the next
bacteremic episode, because fever spikes occur intermittently. Drawing four blood
samples at hourly intervals before the next peak will increase the likelihood of a
positive culture. If the patient is receiving treatment with antimicrobial drugs, a drug-
removing device is helpful in obviating antimicrobial action during culture.

Sensitivity tests need to be interpreted appropriately and with caution since


they are not always reproducible and are an oversimplification of the complex
foundations upon which antimicrobial chemotherapy is based. Disc diffusion tests are
highly sensitive to small technical and environmental changes. Their results may not
correlate well with the actual minimal inhibitory or bactericidal concentration of an
antibiotic, or with the concentration of antibiotic achieved at the site of infection with
the chosen dosage. While important for epidemiologic purposes, routine disc
sensitivity tests are generally of little value in guiding an individual patient's
antibiotic therapy. The minimal inhibitory concentration (MIC), or the minimal
bactericidal or fungicidal concentrations, are more useful clinically, because
antimicrobial dosing can be adjusted to achieve and sustain antibiotic concentrations
at the focus that are three to four times in excess of the concentration required to kill
bacteria in the test-tube.

Therapy of surgical infections

All wounds, whether made at the operating table or resulting from trauma, ex-
pose normally sterile tissue and provide an environment for bacterial growth. Infec-
tions can be minimized if wound management follows these principles.

1. Tissue should be handled gently, and operative trauma kept at a minimum.

2. Further contamination should be minimized by use of aseptic techniques.

3. Devitalized tissue, debris, and traumatic foreign bodies should be removed.

4. Complete hemostasis should be achieved.

5. Blood supply is essential for healing and should not be impaired.


6. Formation of dead space should be avoided during closure.

7. The wound should be closed by layer-to-layer approximation without ten-


sion.

8. Operative time should be kept to a minimum to reduce the numbers of bacte-


ria entering the wound.

9. The wound may be irrigated with liberal amounts of sterile saline/Ringer's


lactate solution prior to closure.

Basic understanding of how the body defends itself against infection is essen-
tial to a rational application of surgical and other therapeutic principles to the control
of infection.

General principles of therapy

Whichever antibiotics are employed, the goal of therapy is to achieve levels of


antibiotic at the site of infection that exceed the minimum inhibitory concentration for
the pathogens present. For mild infections, including most that can be handled on an
outpatient basis, this may be achievable with oral antibiotics when appropriate choic-
es are available. For severe surgical infections, however, the systemic response to in-
fection may make gastrointestinal absorption of antibiotics unpredictable and thus an-
tibiotic levels unreliable. In addition, for intra-abdominal infections, gastrointestinal
function is often directly impaired. For this reason, most initial antibiotic therapy for
surgical infections is begun intravenously. Each patient with a serious infection
should be evaluated daily or more frequently to assess response to treatment. If obvi-
ous improvement is not seen within 2 to 3 days, one often hears the question, “Which
antibiotic should we add [switch] to?” That question is appropriate, however, only af-
ter the following question has been addressed: Why is the patient failing to improve?
Likely answers include the following:

1. The initial operative procedure was not adequate.

2. The initial procedure was adequate but a complication has occurred.


3. A superinfection has developed at a new site.

4. The drug choice is correct, but not enough is being given.

5. Another or a different drug is needed.

The choice of antibiotics is not the most common cause for failure unless the
original choice was clearly inappropriate, such as failing to provide coverage for an-
aerobes in an intra-abdominal infection. As the patient improves, one must decide
when to stop antibiotic therapy. For most surgical infections there is not a specific du-
ration of antibiotics known to be ideal. Antibiotics generally support local host de-
fenses until the local responses are sufficient to limit further infection. When an ab-
scess is drained, the antibiotics prevent invasive bacterial infection in the fresh tissue
planes opened in the course of drainage. After 3 to 5 days, the local responses of new
capillary formation and inflammatory infiltrate provide a competent local defense.
For deep-seated or poorly localized infections, longer treatment may be needed. A re-
liable guideline is to continue antibiotics until the patient has shown an obvious clini-
cal improvement based on clinical examination and has had a normal temperature for
48 hours or more.

Signs of improvement include improved mental status, return of bowel func-


tion, resolution of tachycardia, and spontaneous diuresis. A shorter course of antibiot-
ics may be sufficient, but data supporting a specific duration are not available. The
recent availability of potent systemic antibiotics that can be given orally has led to
some studies demonstrating that patients with intra-abdominal and other serious in-
fections can be treated initially with parenteral antibiotics and then switched to oral
antibiotics to complete their antibiotic course. This has the potential to reduce overall
costs of antibiotic treatment, but it also has the risk to increase unnecessarily the dura-
tion of antibiotic treatment. Some physicians have succumbed to the temptation to
send home patients with antibiotics by mouth because it is easy when previously the
same patient would have been sent home without any antibiotics at all. This tempta-
tion should be resisted.

The white blood cell count may not have returned to normal when antibiotics
are stopped. If the white blood cell count is normal, the likelihood of further infec-
tious problems is small. If the white blood cell count is elevated, further infections
may be detected but in most cases they will not be prevented by continuing antibiot-
ics. Rather, a new infection requires drainage or different antibiotics for a new, re-
sistant pathogen in a different location. In this case, the best approach is to stop the
existing drugs and observe the patient closely for subsequent developments. When
choosing an antibiotic for empiric treatment, the following guidelines should be fol-
lowed:

1. Coverage of the presumed microorganisms involved should be ensured. This


usually means starting broad-spectrum antibiotics that can then be tailored and nar-
rowed to the specific microorganism isolated. Anaerobic spectrum antibiotics should
be avoided when possible since this group of bacteria plays an important role in
maintaining the gastrointestinal tract microenvironment.

2. The antibiotic chosen should be able to reach the site of the infection. Specif-
ically for UTI and for cholangitis, antibiotics with high renal and biliary concentra-
tions, respectively, should be chosen. Skin, lungs, and central nervous system tissue
concentration should also be considered for infections at these sites.

3. Toxicity should be considered, particularly in critically ill patients in whom


bioavailability and therapeutic and toxic level range are harder to predict. Once an an-
timicrobial with significant toxic side effects is started, blood levels and organ func-
tion should be closely monitored.

4. Whenever an infection that will need antibiotics is identified, these should be


dosed aggressively. The volume of redistribution of these patients is unpredictable
since they usually have aggressive fluid replacements as part of their support or re-
suscitation.

5. Whenever an antibiotic regimen is started, set a time limit for the period for
which the antibiotic will be given.

A superinfection is a new infection that develops during antibiotic treatment


for the original infection. Whenever antibiotics are used, they exert a selective pres-
sure on the endogenous flora of the patient and on exogenous bacteria that colonize
sites at risk. Bacteria that remains are resistant to the antibiotics being used and be-
come the pathogens in superinfection. Respiratory tract infections are common super-
infections that occur during the treatment of intra-abdominal infection. The greater
the severity of the abdominal infection and the greater the risk of poor outcome, the
greater the risk of pneumonia as well. Careful surveillance of hospitalized patients re-
veals superinfections in 2% to 10% of antibiotic-treated patients, depending on the
underlying risk factors. The best preventive action is to limit the dose and duration of
antibiotic treatment to what is obviously required and to be alert to the possibility of
superinfections. The use of

increasingly powerful and broad-spectrum antibiotics during the past 2 decades have
also led to an increasing incidence of fungal superinfections. Antibiotic-associated
colitis is another significant superinfection that can occur in hospitalized patients with
mild to serious illness. This entity is caused by the enteric pathogen C. difficile and
has been reported after treatment with every antibiotic except vancomycin. C. dif-
ficile colitis can vary from a mild, self-limited disease to a rapidly progressive septic
process culminating in death. The most important step in treating this disease is to
suspect it. Diagnosis is best accomplished by detecting C.difficile toxin in the stool.
In severe cases, endoscopy, revealing the typical mucosal changes with inflammation,
ulceration, and plaque formation can make a more rapid diagnosis of the severe form
of the disease, pseudomembranous colitis. Treatment is supportive with fluid and
electrolytes, withdrawal of the offending antibiotic if possible, and oral metronidazole
to treat the superinfection. Vancomycin should be reserved for metronidazole fail-
ures. In rare instances when an overwhelming colitis does not respond to medical
management, emergency colectomy may be required.

Antibiotic resistance is an escalating problem presenting particularly in pa-


tients in ICUs. Its implications include increased length of stay, increased costs of
care, and, more importantly, an increased morbidity and mortality derived from infec-
tions treated unsuccessfully. Resistance has been broadly divided into two forms: (1)
intrinsic resistance, in which a specific species is inherently resistant to a specific an-
tibiotic (e.g., gram-negative bacteria to vancomycin) and (2) acquired resistance, in
which a change of the genetic composition of the bacteria occurs. This acquired re-
sistance can be the result of intrinsic changes within the native genetic material of the
pathogen or can be transferred from another species. The molecular mechanisms by
which bacteria acquire resistance to antibiotics can be broadly classified into the fol-
lowing four categories:
1. Decreased intracellular concentration of antibiotic, either by decreased in-
flux or increased efflux— Most antibiotics are susceptible to this mechanism (Pseu-
domonas/Enterobacteriaceae to β-lactams).

2. Neutralization by inactivating enzymes—This is the most common mecha-


nism of antibiotic resistance and affects all β-lactam antibiotics (e.g., β-lactamases
from gram-positive and gram-negative bacteria).

3. Alteration of the target at which the antibiotic will act—It affects all antibi-
otics and is the main resistance mechanism for some specific bacteria (Pneumococcus
to penicillin or MSRA to all β-lactam antibiotics).

4. Complete elimination of the target at which the antibiotic will act—Some


specific bacteria develop the ability to create new metabolic pathways and completely
eliminate a specific target (e.g., VRE).

Antibiotic resistance is usually achieved by the combination of these different


mechanisms. However, the presence of one of them may confer resistance to one or
more different groups of antibiotics. The bacterial genome is divided into chromoso-
mal DNA, which gives specific characteristics and metabolic pathways to the bacte-
ria, and smaller, circular, and independent DNA elements (plasmids) that encode in-
formation for supplemental bacterial activities such as virulence factors and re-
sistance mechanisms. Most resistance mechanisms are plasmid mediated, although
they can interchange with chromosomal information (with the aid of transposons
[mobile DNA elements]), conferring more fixed mechanisms that will be transmitted
vertically. However, plasmids can also be transmitted horizontally through conjuga-
tion, transduction, and transformation processes in which different bacteria are ex-
posed to a specific plasmid. Risk factors for antibiotic resistance in a specific patient
include use of antibiotics, prolonged hospital stays, use of broad-spectrum antibiotics,
use of invasive devices (e.g., endotracheal tubes, central lines, Foley catheters) and
the presence of outbreaks that may reflect ineffective infection control policies. The
populations at highest risk are ICU patients in which the potential absence of effec-
tive antibiotic treatments correlates with higher mortality rates. Prevention strategies
have been studied, and although it is difficult to establish a clear relation between
their practice and decreased resistance, they should be part of a discipline that not on-
ly reduces the incidence of antibiotic resistance but also follows a logical practice for
infection control and use of antibiotics. Some of these strategies include guidelines
for use of antibiotics (hospital formulary restriction, use of narrow-spectrum antibiot-
ics, antibiotic cycling, use of new antibiotics), assessment of infection risk and quan-
titative cultures, infectious disease specialists, and area-specific use of antibiotics
(e.g., outpatients vs. nosocomial, hospital to hospital difference). Nonantibiotic strat-
egies include prevention of nosocomial infections (general and specific measures)
and prevention of hospital transmission (hand washing, contact precautions). The bat-
tle against antibiotic resistance is definitely multidisciplinary and involves the devel-
opment of new antibiotics as well as strategies in the everyday care of patients from
all the health care personnel.

A number of studies have demonstrated the importance of empiric antimicrobi-


al agent therapy in patients who develop severe sepsis syndrome and subsequently are
found to have developed bacteremia, concurrent with fluid resuscitation, metabolic
support, and control of any site-specific source of infection, leading to secondary bac-
teremic events. Use of institutional and unit-specific sensitivity patterns and
knowledge of likely pathogens are critical in selecting an appropriate agent for the
treatment of presumed bacteremia. Retrospective reviews demonstrate that appropri-
ate therapy is associated with a two- to threefold reduction in mortality. A number of
new therapies for treatment of patients with severe sepsis have recently been demon-
strated to be of significant benefit in patients with severe sepsis or septic shock. A
number of investigators have revisited the issue of corticosteroids for the treatment of
septic shock. High-dose corticosteroid therapy had been previously investigated in the
late 1980s and early 1990s with no evidence of benefit for septic patients. Recent in-
terest has arisen for the use of corticosteroids in patients presenting to the ICU in sep-
tic shock subsequent to the observation that many patients in this state harbor adrenal
insufficiency. A number of randomized, controlled trials have demonstrated the bene-
fit of replacement doses of corticosteroids in patients with severe shock states. In pa-
tients who develop septic shock, currently we initiate low-dose hydrocortisone (100
mg/8 h) after performing a corticotropin stimulation test (baseline cortisol level, cor-
ticotropin 250 μg intravenously, cortisol level 1 hour later). Adrenal insufficiency is
identified if the baseline cortisol level is less than 30 μg/dL, or if an increase of less
than 9 μg/dL occurs after corticotropin stimulation. Low-dose steroid therapy should
be discontinued in patients with normal adrenal function.
Nutrition in the surgical patient

The goal of nutritional support in the surgical patient is to prevent or reverse


the catabolic effects of disease or injury. While several important biologic parameters
have been used to measure the efficacy of nutrition regimens, the ultimate validation
for nutritional support in surgical patients should be improvement in clinical outcome
and restoration of function.

Rationale for Parenteral Nutrition The principal indications for parenteral nu-
trition are found in seriously ill patients suffering from malnutrition, sepsis, or surgi-
cal or accidental trauma, when use of the gastrointestinal tract for feedings is not pos-
sible. In some instances, intravenous nutrition may be used to supplement inadequate
oral intake. The safe and successful use of parenteral nutrition requires proper selec-
tion of patients with specific nutritional needs, experience with the technique, and an
awareness of the associated complications. As with enteral nutrition, the fundamental
goals are to provide sufficient calories and nitrogen substrate to promote tissue repair
and to maintain the integrity or growth of lean tissue mass. Listed below are situa-
tions in which parenteral nutrition has been used in an effort to achieve these goals:

1. Newborn infants with catastrophic gastrointestinal anomalies, such as trache-


oesophageal fistula, gastroschisis, omphalocele, or massive intestinal atresia.
2. Infants who fail to thrive due to gastrointestinal insufficiency associated with
short bowel syndrome, malabsorption, enzyme deficiency, meconium ileus, or
idiopathic diarrhea.
3. Adult patients with short bowel syndrome secondary to massive small bowel re-
section (<100 cm without colon or ileocecal valve, or <50 cm with intact ileoce-
cal valve and colon).
4. Enteroenteric, enterocolic, enterovesical, or high-output enterocutaneous fistulas
(>500 mL/d).
5. Surgical patients with prolonged paralytic ileus following major operations (>7
to 10 days), multiple injuries, blunt or open abdominal trauma, or patients with
reflex ileus complicating various medical diseases.
6. Patients with normal bowel length but with malabsorption secondary to sprue,
hypoproteinemia, enzyme or pancreatic insufficiency, regional enteritis, or ul-
cerative colitis.
7. Adult patients with functional gastrointestinal disorders such as esophageal dys-
kinesia following cerebrovascular accident, idiopathic diarrhea, psychogenic
vomiting, or anorexia nervosa.
8. Patients with granulomatous colitis, ulcerative colitis, and tuberculous enteritis,
in which major portions of the absorptive mucosa are diseased.
9. Patients with malignancy, with or without cachexia, in whom malnutrition might
jeopardize successful delivery of a therapeutic option.
10. Failed attempts to provide adequate calories by enteral tube feedings or high
residuals.
11. Critically ill patients who are hypermetabolic for more than 5 days or when en-
teral nutrition is not feasible.

Conditions contraindicating hyperalimentation include the following:

1. Lack of a specific goal for patient management, or in cases in which instead of


extending a meaningful life, inevitable dying is delayed.
2. Periods of hemodynamic instability or severe metabolic derangement (e.g., se-
vere hyperglycemia, azotemia, encephalopathy, hyperosmolality, and fluid-
electrolyte disturbances) requiring control or correction before attempting hyper-
tonic intravenous feeding.
3. Feasible gastrointestinal tract feeding; in the vast majority of instances, this is
the best route by which to provide nutrition.
4. Patients with good nutritional status.
5. Infants with less than 8 cm of small bowel, since virtually all have been unable
to adapt sufficiently despite prolonged periods of parenteral nutrition.
6. Patients who are irreversibly decerebrate or otherwise dehumanized

Rationale for Enteral Nutrition Enteral nutrition generally is preferred over


parenteral nutrition based on reduced cost and associated risks of the intravenous
route. Laboratory models have long demonstrated that luminal nutrient contact reduc-
es intestinal mucosal atrophy when compared with parenteral or no nutritional sup-
port. Studies comparing postoperative enteral and parenteral nutrition in patients un-
dergoing gastrointestinal surgery have demonstrated reduced infection complications
and acute phase protein production when fed by the enteral route. Yet, prospectively
randomized studies for patients with adequate nutritional status (albumin = 4 g/dL)
undergoing gastrointestinal surgery demonstrate no differences in outcome and com-
plications when administered enteral nutrition compared to maintenance intravenous
fluids alone in the initial days following surgery. Recent meta-analysis for critically
ill patients demonstrates a 44% reduction in infectious complications in those receiv-
ing enteral nutritional support over those receiving parenteral nutrition. Most prospec-
tively randomized studies for severe abdominal and thoracic trauma demonstrate sig-
nificant reductions in infectious complications for patients given early enteral nutri-
tion when compared with those who are unfed or receiving parenteral nutrition. The
exception has been in studies for patients with closed-head injury, because no signifi-
cant differences in outcome are demonstrated between early jejunal feeding compared
with other nutritional support modalities. Recommendations for instituting early en-
teral nutrition to surgical patients with moderate malnutrition (albumin = 2.9 to 3.5
g/dL) can only be made by inferences due to a lack of data directly pertaining to this
population. For these patients, it is prudent to offer enteral nutrition based on meas-
ured energy expenditure of the recovering patient, or if complications arise that may
alter the anticipated course of recovery (e.g., anastomotic leaks, return to surgery,
sepsis, or failure to wean from the ventilator). Other clinical scenarios with substanti-
ated benefits from enteral nutritional support include permanent neurologic impair-
ment, oropharyngeal dysfunction, short bowel syndrome, and bone marrow transplan-
tation patients. Collectively, the data support the use of early enteral nutritional sup-
port following major trauma and in patients who are anticipated to have prolonged
recovery after surgery. Healthy patients without malnutrition undergoing uncompli-
cated surgery can tolerate 10 days of partial starvation (i.e., maintenance intravenous
fluids only) before any significant protein catabolism occurs. Earlier intervention is
likely indicated in patients with poorer preoperative nutritional status. Initiation of en-
teral nutrition should occur immediately after adequate resuscitation, most readily de-
termined by adequate urine output. Presence of bowel sounds and the passage of fla-
tus or stool are not absolute requisites for initiating enteral nutrition, but feedings in
the setting of gastroparesis should be administered distal to the pylorus. Gastric resid-
uals of 200 mL or more in a 4- to 6-hour period or abdominal distention will require
cessation of feeding and adjustment of the infusion rate. Concomitant gastric decom-
pression with distal small bowel feedings may be appropriate in certain patients such
as closed-head injury patients with gastroparesis. There is no evidence to support
withholding enteric feedings for patients following bowel resection, or in those with
low-output enterocutaneous fistulas of less than 500 mL/d, but low-residue formula-
tions may be preferred. Enteral feeding should also be offered to patients with short-
bowel syndrome or clinical malabsorption, but caloric needs, essential minerals, and
vitamins should be supplemented with parenteral modalities.

AIDS

HIV is a blood-borne infection that is transmitted to the susceptible host after


percutaneous or a mucus membrane exposure to infected blood or body fluids. The
virus attaches to specific receptors on the host CD4 lymphocytes. The virus is inter-
nalized with release of the viral RNA. The unique enzyme, reverse transcriptase, of
the virus then results in the synthesis of complementary copies of DNA to the RNA
template of the virus (cDNA). This cDNA then migrates into the nucleus of the in-
fected cell, is incorporated into the chromosomal configuration of the host cell, and
then initiates the synthesis of new viral particles. The viral burden within the infected
cell reaches a critical level with lysis of the infected cell and release of viral particles
to infect other cells. The result of this process over time is the systematic depletion of
CD4 depressor cells with dominance of the CD8 cells and subsequent immunosup-
pression of the host. The natural history of HIV infection passes through four phases.
First there is the acute viral infection, which includes fever, malaise, pharyngitis, and
other symptoms that would be nonspecific features of many viral infections. Second,
there is a sustained period of asymptomatic disease. It is during this asymptomatic
disease period that active viral replication is occurring in a slow but progressive fash-
ion, which slowly progresses to a state with significant reduction in CD4 cell counts.
This indolent second phase is highly variable in different patients and for many pa-
tients evolves over a decade or longer. A third phase, which was formerly referred to
as AIDS-related complex, represents the first evidence of symptomatic AIDS. The
patients present with evidence of regional adenopathy. During this early symptomatic
period, the viral load in the patient is increasing and the CD4 count is progressively
declining. Clinical AIDS is considered to exist when the patient has an indicator con-
dition or has a CD4 count lower than 200. The indicator conditions may present as
problems requiring surgical care, or conventional illnesses may present with an ob-
scure presentation because of the patient’s immunosuppressed state, or conventional
illnesses may be mistaken for nonsurgical illnesses associated with the primary HIV
infection.

Acute Abdomen. The AIDS patient has an increased frequency of the clinical
acute abdominal pain syndrome than does the age-matched non-AIDS population.
AIDS patients undergo abdominal exploration for a host of different reasons. It is
likely that AIDS patients actually have an increased rate of emergency abdominal
procedures because they have the anticipated rates of operation for commonly seen
indications (e.g., appendicitis) but have indications in addition to those that are spe-
cific for this disease. An increased probability of abdominal operation but also in-
creased nonsurgical causes for abdominal pain means that a discriminating evaluation
of these patients is always necessary. Acute appendicitis in the AIDS patient occurs
due to the conventional occlusion of the appendiceal orifice by a fecalith but also due
to occlusion of the orifice by Kaposi’s sarcoma lesions and acute CMV infections.
Accumulated appendicitis cases in aged patients indicate that 30% are caused by
complications of AIDS-related conditions. Clinical presentation for the AIDS patient
with appendicitis is with characteristic right lower pain but is commonly associated
with normal WBC counts in most patients. Most have fever, but fever and nonspecific
abdominal pain alone are common findings among AIDS patients without surgical
illness. Although there is no clear definition in the published literature, there appears
to be an increased rate of perforation, gangrenous appendicitis, and initial appendiceal
abscess among AIDS patients. Delay in patient presentation because of frequent ab-
dominal pain and fever, and delay by the physician because of the numerous nonsur-
gical causes of abdominal pain, may account for this apparent observation.

Perforation of the gastrointestinal tract not related to appendicitis is certainly


increased in the AIDS patient. The mean age of the clinical onset of AIDS is in the
late 30s. Other than appendicitis, this age group infrequently has a perforated viscus.
AIDS patients have perforation of the gastroduodenum, small bowel, and colon due
to CMV infection in particular. The terminal ileum and colon are most common sites
for CMV perforations. The diagnosis of CMV perforations is confirmed by identifica-
tion of the intranuclear inclusion bodies on biopsy specimens from sites of perfora-
tion. Appropriate surgical management of the site of perforation requires suture plica-
tion of gastroduodenal perforations, resection and anastomosis of small bowel perfo-
rations, and colostomy for colonic perforations. Acute antiviral chemotherapy is initi-
ated. CMV perforation is an indicator of advanced HIV disease and carries a grave
prognosis owing to death from peritonitis or other AIDS-related complications. Ka-
posi’s sarcoma, gastrointestinal lymphoma, and severe ileocolitis from Mycobacte-
rium avium intracellulare are additional causes of AIDS-related perforations of the
gastrointestinal tract. Biopsies of the site of perforation at operation are necessary to
establish causation. Management of the perforated site is the same as for any other
perforation due to infectious causes.
Gastrointestinal obstruction is seen secondary to AIDS-related disease.
Causes include gastric outlet obstruction secondary to lymphoma, small bowel ob-
struction due to mycobacterial disease, intussusceptions secondary to Kaposi’s sar-
coma, and an Ogilvie-like syndrome progressing to toxic megacolon due to CMV in-
fection. The diagnosis of the AIDS-related events needs to be differentiated for more
conventional causes of obstruction. In the usual age group for AIDS patients, particu-
larly when prior abdominal operation and the risk of adhesions are not present, most
intestinal obstruction events are AIDS related.

Gastrointestinal bleeding is similarly seen by the same array of disease pro-


cesses that are responsible for perforation and obstruction. When bleeding arises from
the gastroduodenum or colon, endoscopy procedures will assist diagnosis. Operation
is required only when medical measures to control hemorrhage have failed.

Hepatobiliary disease is common in the HIV-infected patient. Chronic hepatitis


B and C infections share common routes of transmission with HIV disease. Persistent
elevation of hepatic enzymes from chronic hepatitis is common with cirrhosis as a
frequent result. Once clinical AIDS has evolved, infection of the liver parenchyma
from Candida albicans and M. avium intracellulare result in small hepatic abscesses,
which may require liver biopsy for diagnosis. Although infections with Entamoeba
histolytica among the male homosexual population with AIDS are common, amebic
abscess is much less common.

A particularly interesting but infrequent infectious problem is AIDS-associated


cholangiopathy. This appears to be the consequence of infection of the actual bile
ducts themselves with opportunistic pathogens including Cryptosporidium species,
CMV, and Microsporidia. Inflammatory changes secondary to invasion of the ducts
result in a sclerosis-like picture. The patients have new-onset right upper quadrant
pain, fever, alkaline phosphatase elevations, but rarely jaundice, at the time of initial
presentation. Jaundice becomes more of a feature of the disease as the process ad-
vances over time. Diagnosis is suggested by ultrasound demonstration of thickened
ducts. Endoscopic retrograde cholangiopancreatography is used to culture the bile or
obtain biopsies of the bile ducts. Specific antimicrobial chemotherapy is used for
treatment. Surgical care for these patients is limited. An occasional patient may de-
velop acute cholecystitis secondary to cystic duct occlusion. Radionuclide scans are
used for diagnosis, and cholecystectomy may be necessary in the patient with acute
cholecystitis.

Splenomegaly is a common finding among AIDS patients but may be the result
of multiple causes. Patients may have portal hypertension from severe liver disease
or portal fibrosis. Parenchymal infection of the spleen may be secondary to CMV,
Microbacterium, Pneumocystis carinii, and other pathogens. Splenic enlargement
may be secondary to lymphoma or Kaposi’s sarcoma. The patients commonly have
left upper quadrant pain and the spleen is palpable and quite tender on physical exam-
ination. Splenectomy may infrequently be necessary secondary to spontaneous rup-
ture or to rupture from incidental trauma.

Vascular infections are reported among AIDS patients. Some infected pseudo-
aneurysms are seen among the intravenous drug abuse population with common bac-
teria (e.g., S. aureus). These infections among the AIDS population are difficult to
eradicate. Perhaps more interesting and somewhat unique to the AIDS patient is Sal-
monella arteritis. AIDS patients have a high incidence of Salmonella infection. Ap-
parently Salmonella has a particular affinity for atherosclerotic plaque. Adherence of
the microbe to an atheroma of the distal aorta or iliac arteries can result in invasive
infection, pseudoaneurysm formation, and potential rupture. Surgical management
prior to rupture is desired. Reconstruction of these patients following resection pro-
ceeds along guidelines for management of any mycotic aneurysm infection.

Neoplasms. B-cell lymphoma occurs commonly among AIDS patients. These


malignancies are commonly undifferentiated and aggressive. Operative intervention
for the lymphoma patient is for the purpose of diagnosis of the disease (e.g., needle
biopsy). More commonly surgical intervention is for the management of complica-
tions secondary to bleeding, obstruction, or perforation of the gastrointestinal tract.
Primary management of the lymphoma disease is medical. Kaposi’s sarcoma is a ne-
oplasm of the skin that was uncommon until the AIDS epidemic. Kaposi’s sarcoma is
the result of the patient having chronic infection with human herpesvirus-8, but clini-
cal disease occurs only when the patient’s clinical immunosuppression reaches an ad-
vanced stage. Kaposi’s sarcoma in the AIDS patients occurs at numerous different
sites, including skin, gastrointestinal tract, lung, liver, and even the heart. Surgical
involvement is primarily for diagnosis and the management of complications, particu-
larly in the gastrointestinal tract. As noted earlier, perforation, bleeding, and small
bowel intussusceptions are noted gastrointestinal complications from Kaposi’s sar-
coma. Radiation and chemotherapy are the primary treatment modalities for this neo-
plasm.

Anorectal Disease. The immunosuppressed AIDS patient is at increased risk


for human papillomavirus infection. Large condylomata acuminata are the result.
Very large condylomata commonly need to be surgically reduced to be followed by
local topical therapy. Squamous cell carcinoma of the anus occurs with increased fre-
quency presumably due to the role of papillomavirus in causing this disease.

Occupational Risk of Infection. A major surgical concern about the HIV-


infected patient was the potential of infection being occupationally acquired during
the course of providing surgical care for these patients. Surgical exposure to patient
blood during the performance of operative procedures has been well documented.
With the knowledge that HIV is a blood-borne pathogen and that other blood-borne
pathogens (e.g., hepatitis B) have been documented to be transmitted in the operating
room, many surgeons have been concerned about this risk.

At this time it can be said that the risk of occupational transmission of HIV dis-
ease is low, but it is not zero. As of the last available Centers for Disease Control and
Prevention report, 57 documented cases of occupational transmission of HIV have
occurred and 138 cases of probable transmission among health care workers have
been identified. No documented cases have been seen in surgeons. Most occupation-
al infections have come from major percutaneous injuries from hollow needles. Solid-
needle injuries have not been documented to occur in the United States. Current rates
of transmission from hollow needles are about 0.2% to 0.3%. Surgeons should feel
comfortable in providing care for HIV-infected patients but should use appropriate
and standardized safeguards to prevent blood exposure in the care of all patients.

Basic literature:
5. Oxford Textbook of Surgery (3-Volume Set) 2nd edition (January 15, 2000):
by Peter J. Morris (Editor), William C. Wood (Editor) By Oxford Press
6. Sabiston Textbook of Surgery 17th edition by Courtney M. Townsend Jr.,
Kenneth L. Mattox, B. Mark, MD Evers, Kenneth L., MD Mattox, Courtney
Townsend, Daniel Beauchamp, B. Mark Evers, Kenneth Mattox W.B. Saun-
ders Company (June, 2004)
7. Schwartz´s Principles of Surgery 8th Edition F. Charles Brunicardi. Copyright
©2007 the McGraw-Hill Companies.
8. Hospital surgery/ Edited by L. Kovalchuk et al. - Ternopil: Ukrmedknyha,
2004. - 472 p.
Additional literature:
1. Bachetti T, Pasini E, Suzuki H, et al: Species-specific modulation of the
nitric oxide pathway after acute experimentally induced endotoxemia. Crit Care
Med 31:1509, 2003.
2. Briegel J, Jochum M, Gippner-Steppert C, et al: Immunomodulation in
septic shock: Hydrocortisone differentially regulates cytokine responses. J Am Soc
Nephrol 12:S70, 2001.
3. Healy DP: New and emerging therapies for sepsis. Ann Pharmacother
36:648, 2002. Raeburn CD, Sheppard F, Barsness KA, et al: Cytokines for surgeons.
Am J Surg 183:268, 2002.
4. Turnidge J: Impact of antibiotic resistance on the treatment of sepsis.
Scand J Infect Dis 35:677, 2003.

Tests for initial level of knowledge, keys for tests:


1. A 65 y.o. man who has problems with urination as a result of benign prostate
gland adenoma developed fever and chill, hypotension, sinus tachycardia. Skin is
warm and dry. Clinical blood analysis revealed absolute granulocytopenia. These
hemodynamic changes are most likely to be caused by:
A Endotoxemia with activation of complement system
B Secondary reflex vasodilatation as a result of lowered cardiac output
C Secondary circulation insufficiency with retained systolic function as
a result of peripheral vasoconstriction
D Reflex vagus stimulation with lowered cardiac output
E Secondary endothelial changes as a result of bacterial lesion

2. A 24-year-old law student is brought to the emergency room complaining of


severe abdominal pain of 6-8 hours duration. He had been to a party the night
before. The pain is in the epigastrium radiating to the back and is accompanied
by nausea. The patient had vomited twice prior to coming to the emergency room.
Clinical examination revealed that the young man was anxious, with acute condition,
with a regular pulse rate of 100/min, blood pressure of 100/68 mm Hg, and body
temperature of 38,1oC. The most likely diagnosis is:
A Acute cholecystitis
B Acute pancreatitis
C Acute appendicitis
D Acute diverticulitis
E Mesenteric adenitis

3. A 33 y.o. patient was admitted to the reception room of the Central District
Hospital. He complains of a severely painful swelling localized on posterior
neck, fever up to 38,40C and general weakness. In anamnesis: diabetes mellitus with-
in 5 years. On physical examination on the posterior neck surface there is an infil-
trate elevated above surrounding skin. The tissues affected by swelling are tense and
blue reddish discoloration in central area. There are also several purulent necrotic
pustules which are connected with each other and form a large skin necrosis. A
thinned necrotic skin of this swelling has holes looking like sieve, pus discharges
throughout. What disease should a doctor consider first of all?
A Acute skin cellulitis
B Furuncle
C Carbuncle
D Carbuncle associated with anthrax
E Skin abscess

4. A patient complains of an extremely intense pain in epigastrium. He has pep-


tic ulcer disease of duodenum for 10 years. The patient is in the forced position be-
ing on the right side with legs abducted to stomach. Abdomen has acute tenderness
in the epigastrium. Guarding contraction of the abdominal wall muscles is ob-
served. What is the preliminary diagnosis?
A Penetration of ulcer into pancreas
B Acute pancreatitis
C Acute condition of peptic ulcer disease
D Perforation of ulcer
E Thrombosis of mesenteric vessels

5. A 30-year-old patient complains of pain, hyperemia along subcutaneous veins, rise


in body temperature. While examining the large shin subcutaneous vein, there is hy-
peremia, pain by pressing. Homanss and Luses's symptoms are negative. What
is the preliminary diagnosis?
A Thrombosis of aorta
B Acute ileofemoral phlebothrombosis
C Lymphostasis
D Embolism of aorta
E Acute thrombophlebitis of subcutaneous veins

6. On the 4th day after recovering from a cold a patient was hospitalized with
complaints of solitary spittings of mucoid sputum. On the 2nd day there was a single
discharge of about 250 ml of purulent blood-streaked sputum. Objectively: the pa-
tient's condition is moderately severe. Respiratory rate - 28-30/min, Ps- 96 bpm, AP-
110/70 mm Hg. Respiration above the left lung is vesicular, weak above the right
lung. There are moist rales of different types above the lower lobe and amphoric
breath near the angle of scapula. What is the most likely diagnosis?
A Acute pulmonary abscess
B Exudative pleuritis
C Acute focal pneumonia
D Pleural empyema
E Pyopneumothorax
7. A 24 y.o. woman consulted a doctor about continued fever, night sweating. She
lost 7 kg within the last 3 months. She had casual sexual contacts. Objectively: en-
largement of all lymph nodes, hepatolienal syndrome. Blood count: leukocytes -
2,2*109/L. What disease can be suspected?
A Lymphogranulomatosis
B HIV-infection
C Tuberculosis
D Infectious mononucleosis
E Chroniosepsis

8. A 20 y.o. patient suddenly felt ill 12 hours ago. There was pain in epigas-
tric area, nausea, sporadic vomiting. He had taken alcohol before. In few hours the
pain localized in the right iliac area. On examination: positive rebound tenderness
symptoms. WBC- 12,2·109/L. What is the most probable diagnosis?
A Perforated ulcer
B Acute pancreatitis
C Acute appendicitis
D Right side kidney colic
E Acute cholecystitis

9. A 40-year-old patient underwent an operation for a lumbar phlegmon.


Body temperature rose again up to 38oC, he got intoxication symptoms, there was an
increase of leukocyte number in blood. The wound that was nearly free from necrot-
ic tissues and full of granulations started to discharge pus, the granulations turned
pale. What complication developed in this patient?
A Allergic reaction
B Putrid phlegmon
C Erysipelas
D Sepsis
E Erysipeloid

10. A 52 year old patient complains about pain in the right part of her chest, dyspnea,
and cough with a lot of foul-smelling albuminoid sputum in form of "meat slops".
Objectively: the patient's condition is grave, cyanosis is present, breathing rate is
31/min, percussion sound above the right lung is shortened, auscultation revealed dif-
ferent rales. What is the most probable diagnosis?
A Chronic pneumonia
B Lung abscess
C Pleura empyema
D Multiple bronchiectasis
E Lung gangrene

Keys for tests


1 2 3 4 5 6 7 8 9 10
A B C D E A B C D E

Tests for final level of knowledge, keys for tests:


1. Which of the following surgical interventions is least likely to provide ac-
ceptable prolongation of life for patients with AIDS?
A. Splenectomy for AIDS-related idiopathic thrombocytopenic purpura
B. Colonic resection for perforation secondary secondary to cytomegalovirus in-
fection
C. Cholecystectomy for acalculous cholecystitis
D. Tracheostomy for ventilator dependent patients with respiratory failure
E. Gastric resection for a bleeding gastric lymphoma or Kaposi’s sarcoma

2. Human immunodeficiency virus (HIV) has been isolated from many body flu-
ids. Which of the following is a major source of transmission?
A. Tears
B. Sweat
C. Semen
D. Urine
E. Breast milk

3. Signs and symptoms associated with early sepsis include


A. Respiratory acidosis
B. Decreased cardiac output
C. Hypoglycemia
D. Increased arteriovenous oxygen difference
E. Cutaneous vasodilation

4. A 42-year-old man has a calculated resting energy expenditure of 1800


kcal/day (basal energy expenditure plus 10%). Match the following clinical situa-
tions with the appropriate daily energy requirement. In case of sepsis energy ex-
penditure is
A. 1600
B. 2300
C. 2800
D. 3600
E. 4500

5. Major alterations in pulmonary function associated with adult respiratory


distress syndrome (ARDS) include
A. Hypoxemia
B. Increased pulmonary compliance
C. Increased resting lung volume
D. Increased functional residual capacity
E. Decreased dead space ventilation

6. Which statement regarding transmission of viral illness through homologous


blood transfusion is true?
A. The most common viral agent transmitted via blood transfusion in the United
States is human immune deficiency virus (HIV)
B. Blood is routinely tested for cytomegalovirus (CMV) because CMV infection
is often fatal
C. The most frequent infectious complication of blood transfusion continues to
be viral meningitis
D. Up to 10% of those who develop post transfusion hepatitis will develop cir-
rhosis or hepatoma or both
E. The etiologic agent in post transfusion hepatitis remains undiscovered

7. Central venous pressure (CVP) may be decreased by


A. Pulmonary embolism
D. Hypervolemia
C. Positive-pressure ventilation
D. Pneumothorax
E. Gram-negative sepsis

8. The appropriate antibiotic to prescribe while awaiting specific culture verifi-


cation is
A. Penicillin
B. Erythromycin
C. Tetracycline
D. Azathioprine
E. Cloxacillin

9. Tissue injury or infection results in the release of tumor necrosis factor


(TNF) by which of the following cells?
A. Fibroblasts
B. Damaged vascular endothelial cells
C. Monocytes/macrophages
D. Activated T lymphocytes
E. Activated killer lymphocytes

10. Which statement about transmission of HIV in the health care setting is true?
A. A freshly prepared solution of dilute chlorine bleach will not adequately de-
contaminate clothing
B. All needles should be capped immediately after use
C. Cuts and other open skin wounds are believed to act as portals of entry for
HIV
D. Double gloving reduces the risk of intraoperative needle sticks
E. The risk of seroconversion following a needle stick with a contaminated nee-
dle is greater for HIV than for hepatitis B

Keys for tests


1 2 3 4 5 6 7 8 9 10
D C E C A D E A C C

Tasks for final level of knowledge


1. A 62-year-old man presents with chills, fever, dysuria, urinary fre-
quency, diffuse low back pain, and an exquisitely tender prostate on rectal exam.

What is it? SIRS. Acute bacterial prostatitis.The treatment for this man is
intuitive: he needs IV antibiotics—but what should not be done are any
more rectal exams or any vigorous prostatic massage. Doing so could lead
to septic shock.

2. An adult woman relates that 5 days ago she began to notice frequent,
painful urination, with small volumes of cloudy and malodorous urine. For the
first 3 days she had no fever, but for the past 2 days she has been having chills,
high fever, nausea, and vomiting. Also in the past 2 days she has had pain in the
right flank. She has had no treatment whatsoever up to this time.

What is it? SIRS. Pyelonephritis.This woman needs hospitalization, TV anti-


biotics, and at least a sonogram to make sure that there is no concomitant
obstruction.

3. A 56-year-old alcoholic man is admitted with a clinical clinical pic-


ture of acute-upper abdominal pain. The pain is constant, radiates straight through
the back, and is extremely severe. He has a serum amylase of 800, a hematocrit
of 40%, WBC count of 18,000, blood glucose of 150 mg/dL and serum calcium
of 6.5. He is given IV fluids and kept NPO with NG suction. By the next morn-
ing, his hematocrit has dropped to 30%, the serum calcium has remained below 7
despite calcium administration, his blood urea nitrogen (BUN) has gone up to 32,
and he has developed metabolic acidosis and a low arterial P02.

What is it? Sepsis, hemorrhagic pancreatitis (with at least eight of Ranson's cri-
teria predicting 80 to 100% mortality.)

4. A 43-year-old obese mother of six children has severe right upper


quadrant abdominal pain that began 6 hours ago. The pain was colicky at first, radi-
ated to the right shoulder and around toward the back, and was accompanied by nau-
sea and vomiting. For the past 2 hours the pain has been constant. She
has tenderness to deep palpation, muscle guarding, and rebound in the right
upper quadrant. Her temperature is 101 °F, and she has a WBC count of 16,000.
She has had similar episodes of pain in the past brought about by ingestion of
fatty food, but they all had been of brief duration and relented spontaneously
or with anticholinergic medications.

What is it? SIRS. Acute cholecystitis.

5. Forty-five minutes after completion of a cystoscopy, a patient devel-


ops chills and a fever spike of 104°F.

What is it? This is bacteremia. Take blood cultures times 3, and start em-
piric antibiotic therapy.
Materials for the self-study of the students
Main tasks Notes (instructions)
Repeat:
-To represent the methods of HIV diag-
1. Inflammation
nosis
2. Pathophysiology of inflamma-
-To make the flow diagram of mecha-
tion
nisms of sepsis.
3. Stages of AIDS.

Study:
1. Stages of sepsis. -To conduct differential diagnosis differ-
2. Methods of diagnosis. ent tapes of SIRS
3. Antibiotic resistance
Study guide #3
“Critical conditions in surgical clinic: shock, circulatory collapse, acute respira-
tory distress syndrome, abdominal compartment syndrome. Causes, diagnosis
and medical tactics.”

Overview.
Shock remains one of the surgeon's most formidable foes. Sixty years after
publication of Alfred Blalock's classic textbook, Principles of Surgical Care: Shock
and Other Problems, the diagnosis of shock and the management of shock resuscita-
tion continue to challenge the clinician and investigator. The objective of this guide is
to provide the reader with practical methods for recognizing and resuscitating patients
who are in shock.
Educational aims:
1. Interrogation and clinical inspection of patients with shock.
2. To determine the etiologic and pathogenic factors of different forms of shock, ab-
dominal compartment syndrome and acute respiratory distress syndrome.
3. To find out the clinical features of the intraabdominal hypertension.
4. To develop a plan of examination and management of the patients with shock.
5. To estimate results of physical examination, laboratory tests, ECG, ultrasonogra-
phy and X-ray examination of patients in critical conditions.
6. To make a differential diagnosis between different forms of shock.
7. To determine the criteria for the diagnosis of acute respiratory distress syndrome.
8. To compare the different approaches to the treatment of shock.

A student must know:


1. Classification of shock.
2. Pathogenesis of different kinds of shock.
3. Risks of resuscitation of the patient with shock
4. The algorithms of management in case of sudden circulatory collapse of a patient
in the operating room, immediate, intermediate and late postoperative period.
5. Factors influencing intraabdominal pressure
6. Mechanisms of the development of the intraabdominal hypertension and ACS.
7. Different causes of circulatory collapse.
8. Pathogenesis of ARDS.
9. The optimal ventilator strategy for ARDS
10. Predisposing conditions to ACS
11. Grading of ACS
A student must be able to:
1. Prescribe the investigations to the patients in critical conditions.
2. Define the rational quantity of laboratory and instrumental research methods.
3. Be able to examine patients with shock, ARDS and ACS.
4. Define the indications to resuscitation of the patients with shock.
5. To prescribe the treatment to the patients with ARDS.
6. To recognize the clinical signs of ACS.
7. To differentiate the types of ACS and prescribe appropriate treatment.
Terminology
Term Definition
ARDS Acute Respiratory Distress Syndrome
PAWP Pulmonary artery wedge pressure
An index of arterial oxygenation efficiency that corre-
PaO2 : FiO2
sponds to ratio of partial pressure of arterial O2 to the frac-
tion of inspired O2
is defined as a general or specific failure of the circulation,
Circulatory collapse
either cardiac or peripheral in nature.
is the end result of a progressive, unchecked increase in in-
Abdominal Compart-
tra-abdominal pressure from a myriad of disorders that
ment Syndrome
eventually lead to multiple organ dysfunction.

Content:
Shock
Shock is a circumstance in which homeostasis is disrupted. A universal physio-
logic threat to the patient in shock is deficient oxygen delivery to the mitochondria of
cells. As a consequence, aerobic metabolism cannot be sustained at the rate needed to
maintain cell function. The cell cannot recover from sustained interruption of aerobic
metabolism. As cells die, organ failure ensues. A wide range of mechanisms cause
shock. Surgeons treat many of these patients by focusing therapy on restoring cardio-
vascular function, by treating either impaired cardiac contractility, a decline in sys-
temic vascular resistance, or depleted intravascular volume. But an emphasis on ther-
apy that measurably influences whole-organ function should not deflect the apprecia-
tion that patient survival will ultimately be determined by events within cells. Pro-
found hemorrhage leads to a rapidly lethal form of shock; a sustained period of a
modestly reduced oxygen delivery leading to irreversible intracellular dysfunction is
just as lethal.
Descriptions of the shock syndrome aggregate pathogenesis, manifestation, and
physiologic responses in ways that facilitate clinical recognition but conveniently ob-
scure causal relationships. Despite decades of research, the causes of shock (or, more
important, the causes of irreversibility and the sequelae of shock) remain enigmatic.
The problem with current descriptions, many of which focus on a mismatch between
metabolic supply and demand and its consequences, is that they do not capture either
the self-sustaining nature of shock or the importance of timeliness in recognition and
management. Whereas most authors cite these latter characteristics as “features” of
shock and of shock therapy, the self-sustaining characteristic and the effectiveness of
early intervention may well be fundamental to the physiologic derangement and res-
cue, respectively. Because “shock” is commonly described in terms of metabolic
shortfall (metabolic demand exceeding supply of essential nutrients), “not shock” or
the basal physiologic state must also be examined through the lens of energetics. All
life forms, from the prokaryotes to complex mammals, share three imperatives: to ex-
tract energy from the environment to hold entropy at bay; to adapt to (the variable)
external environment to maintain constant the internal environment; and to replicate.
We focus first on energy extraction. Organisms and their constituent cells are ther-
modynamically open systems. Ingested carbohydrate, fat, and protein are biochemi-
cally degraded into primitive units. The currency of biochemical energy, high-energy
phosphates (including adenosine triphosphate, guanosine triphosphate, creatine phos-
phate, and others), can be generated directly from the primitives or, alternatively, the
primitives can be stored. Most humans have several months' worth of stored fat and a
day's worth of stored carbohydrate (hepatic and muscle glycogen). There is no storage
form of protein: all known proteins are structural or catalytic. The primitives—
glucose and fatty acids—generate high-energy phosphates through two biochemical
pathways, one that requires molecular oxygen (oxidative phosphorylation) and one
that does not (anaerobic glycolysis). The yield of high-energy phosphates through the
oxidative pathway is sufficient to sustain life, whereas the yield through the anaerobic
pathway is not. Proof of this distinction is clinical: carbon monoxide poisoning
(which prevents oxygen transport on hemoglobin) and cyanide poisoning (which un-
couples oxidative phosphorylation) are lethal. These represent two unusual causes of
shock that the surgeon occasionally encounters. There is no storage form of oxygen.
Arterial hemoglobin is normally 95% to 98% saturated with oxygen, whereas mixed
venous blood is normally 70% to 75% saturated, suggesting that approximately one
fourth of the available oxygen is removed during each circuit through vital organs and
tissues. Human blood volume normally circulates approximately once each minute.
These facts suggest that even if every oxygen molecule could be unloaded from he-
moglobin to cells, unreplenished oxygen delivery will be exhausted in approximately
4 minutes. This is important for three reasons. It points to oxygen as the critical nutri-
ent; it points to the importance of efficient resuscitation; and it points to restoration of
oxygen delivery as the imperative in resuscitation from shock. If resuscitation is un-
timely or incomplete, the consequences are predictable and often lethal. Cells initially
switch from oxidative phosphorylation to the more anaerobic metabolic pathways.
End products of anaerobic metabolism, notably lactic acid, accumulate. More im-
portant, the electrochemical gradients across cytoplasmic and subcellular membranes
that are normally maintained by a constant supply of high-energy phosphates start
failing. As gradients fail, water and salt on either side equilibrate, disrupting the
three-dimensional organization of proteins. Disrupted proteins cannot be repaired be-
cause the repair mechanisms require high-energy phosphates. Disrupted proteins can-
not be recycled because the recycling mechanisms require high-energy phosphates.
Beyond a salvage threshold of failed gradients and disrupted proteins, the affected
cell becomes necrotic. Unfortunately, clean-up of necrotic tissue also requires energy.
The result is a collective, accelerating spiral of deteriorating function of cells, tissues,
and organs. Decades of research offer no better therapy than the prompt restoration of
oxygen delivery. The adequacy of oxygen delivery is properly local, but oxygen de-
livery itself can be estimated from global measures. Thus, oxygen delivery is the ge-
ometric product of arterial oxygen saturation, hemoglobin concentration, stroke vol-
ume, and heart rate. The product of stroke volume and heart rate is cardiac output, or,
equivalently, the amount of venous blood returning to the heart. The focus of shock
resuscitation is the optimization of these parameters, and it is therefore worthwhile
restating the definition of oxygen delivery in the form of a relation:
DO2 = SaO2 x [Hgb] x heart rate x stroke volume
However, optimization of these four parameters is only the second most important
clinical imperative. The most important task in shock intervention is early recognition
of the shock syndrome. The syndrome is composed not only of the metabolic de-
rangements directly attributable to inadequate perfusion but also of the reflex re-
sponses teleologically aimed at mitigating the inadequate perfusion. The reflex re-
sponses are clinically appreciable far earlier than the derangements themselves.
These reflex responses are mediated by the neuroendocrine system, which secretes a
series of hormones to sustain delivery of nutrients to cells and promote diffusion and
transport of nutrients into cells. Several classes of hormones are released during the
initial response to shock, the catecholamines, the renin-angiotensin-aldosterone axis,
as well as antidiuretic hormone. The catecholamines, epinephrine and norepinephrine,
are full agonists for both a-adrenergic (vasoconstrictor) and b 1-adrenergic (increased
heart rate, increased heart contractility, increased heart conduction velocity) recep-
tors. (Each is a partial agonist for b 2-adrenergic receptors, which mediate vasodilata-
tion). The catecholamines cause three early events. First, the heart rate accelerates,
second, peripheral arterial beds and splanchnic beds empty into the systemic circula-
tion, and third, potassium is shifted to intracellular compartments. These events are
appreciated as tachycardia; as delayed capillary refill and a slight rise in diastolic
blood pressure; and as mild hypokalemia. Catecholamine secretion is prominent in all
forms of shock and the effects of catecholamines are nearly always the first physical
signs of shock. The observed response to catecholamines is less effective when the
specific cause of shock renders target cells refractory to catecholamines, namely, sep-
tic shock. The response is also less effective when catecholamine responsiveness has
been altered with drugs such as b-adrenergic blockers. Renin is released from the
kidneys in response to hypovolemia, and the release is potentiated by epinephrine.
The release catalyzes the conversion of angiotensinogen to the angiotensins. The sud-
den rise in circulating angiotensins contributes substantially to overall splanchnic
vasoconstriction. Such constriction can mobilize up to 30% of the total blood volume,
compensating for but also masking the loss of blood from the systemic circulation.
The combination of catecholamines, renin, and antidiuretic hormone released early in
response to shock causes the kidneys to retain water and sodium and decreases
splanchnic perfusion. Urine output is therefore modulated relatively early in the re-
sponse to shock. Other hormones secreted somewhat later in response to shock in-
clude glucagon, cortisol, and growth hormone. Collectively, they alter physiology to
create a state similar to diabetes, including mild hyperglycemia and insulin resistance.
Both muscle protein and fat stores are mobilized during recovery from shock to aug-
ment plasma glucose through gluconeogenesis. Except to correct demonstrable defi-
ciencies, administration of these hormones has not been shown to improve outcome
from shock.
THE PATHWAYS TO SHOCK
Once shock is recognized, the surgeon must simultaneously identify and re-
verse the underlying cause while performing resuscitation. The former is more diffi-
cult than the latter. It is helpful to remember that there are three fundamental path-
ways to shock. These pathways reflect problems with the “three P's”:
• the perfusate (intravascular volume);
• the pump [problems with the heart or getting blood into the heart (obstruction)];
• the pipes (distributive problems that allow blood to pool into the periphery and to
pass by starving tissues without unloading nutrients).
This classification is simple enough to commit to memory and can guide decision
making for the first several minutes of resuscitation.
The surgeon most commonly encounters shock through the perfusate pathway.
The associated clinical syndromes are hypovolemic shock due to dehydration and
hemorrhagic shock due to acute loss of blood volume. Mild perfusate loss is common
and does not cause clinical symptoms. For example, voluntary blood donation corre-
sponds to acute loss of approximately 10% of the circulating blood volume. The skin
and skeletal muscle vasculature experience a slight decrease in perfusion. However,
such a small acute loss is well tolerated because the intravascular volume can be
quickly recruited from interstitial and intracellular reserves. Beyond 10% loss, how-
ever, the neuroendocrine response to shock becomes clinically apparent. The adrenal
medulla increases its blood flow, ensuring both adequate oxygen delivery to its own
tissues as well as swift delivery of catecholamines into the systemic circulation. As
occupancy of the peripheral adrenergic receptors increases, heart rate and diastolic
blood pressure rise, even while blood is squeezed out of the splanchnic bed. This
compensatory redistribution fails at approximately 30% volume loss, a failure clini-
cally manifested as the onset of systolic and diastolic hypotension. The decrease in
urine flow in the early stages of volume loss is not due to early failure of renal blood
flow, but rather to (a) a fall in glomerular filtration rate, (b) the sympathetically in-
duced increases in resorption of sodium from the proximal tubules, and (c) the effects
of antidiuretic hormone on retention of free water and that of aldosterone on distal
tubular sodium resorption. Once hypotension occurs, further blood flow redistribution
occurs in favor of the brain, but at the expense of the heart and the kidneys. A 40% to
50% volume loss exhausts all compensatory mechanisms. The need to restore perfu-
sion and eliminate the cause of shock is evident.
The pump pathway to shock has two important entrances: primary pump failure
and inability of the pump to accept the perfusate. The latter is commonly termed ob-
structive shock and is considered separately. The causes of pump failure, or cardio-
genic shock, are familiar: acute failure of the cardiac muscle or a cardiac valve, and
acute dysrhythmias. Specific diagnoses include myocardial infarction, rupture of a
papillary muscle, and fracture of the chordae tendineae (the latter processes, thankful-
ly rare, lead to acute regurgitation and failure of the left heart). The obstructive path-
way to shock, the inability of the pump to accept the perfusate, is frequently traversed
by injured patients. The specific diagnoses causing obstruction in the acutely injured
are tension pneumothorax and pericardial tamponade. These diagnoses share a patho-
physiologic process that transmits pressure to the external wall of the atria, thereby
preventing blood flow into the cardiac chambers. Decompression (of the pleural
space or of the pericardial space) is lifesaving. Acute embolism of a blood clot from
the systemic veins into the heart (pulmonary embolism) is a common cause of ob-
structive shock among surgical patients. Therapy is focused on relief of the intralu-
minal obstruction. A less common but deadly cause of obstructive shock is air embo-
lism consequent to inadequately filled systemic veins brought into contact with the
atmosphere, either during surgery or by a central venous catheter. A large air embolus
obstructs the right ventricular outflow tract, whereas slow entrainment of air causes
distal pulmonary arteries to become obstructed with acute right ventricular dysfunc-
tion. Therapy of a large right ventricular air lock requires relief of the obstruction
through positioning (placing the patient in the right side down, head down position to
try to move the embolus to the apex of the right ventricle), aspirating the right heart
through a preexisting central venous catheter, cardiac massage, or direct puncture of
the right heart to aspirate the air. These maneuvers are usually performed in the se-
quence listed until one is successful.
Discrimination between “perfusate” problems and “pump” problems is critical
because the therapies are distinct. Unfortunately, the neuroendocrine response to
pump shock is clinically indistinguishable to the response to perfusate shock: the skin
is poorly perfused, moist, and cool; the pulse is weak; the heartbeat and respiratory
rates are rapid; and the urine flow is reduced. Bedside discrimination between pump
and perfusate shock is based directly on mechanism: in pump shock, the capacitance
(venous) vascular beds are full because the pump cannot or will not accept inflow.
Thus, in pump shock, the neck veins are distended, the patient has an elevated central
venous pressure, and abnormal heart sounds may be present. In perfusate shock, the
neck veins are collapsed and the central venous pressure is low. This difference can-
not be overemphasized: given a patient with the classic presentation of shock (ashen
facies, diaphoresis, tachycardia, tachypnea, and hypotension), attention should be
immediately directed at the neck veins to discriminate pump from perfusate path-
ways.
“Pipe” shock (formally, distributive shock and neurogenic shock) follows fail-
ure of mechanisms that regulate tissue-specific resistance and capacitance. There are
two routes to this form of shock. The first is through interruption of the sympathetic
nervous system, the consequence of either spinal cord injury or neuraxial instillation
of local anesthetic agents (spinal or epidural anesthesia). The second route is through
attenuation of the sympathetic effects in the periphery, most commonly in the context
of sepsis. Unlike the patient with perfusate or pump shock, patients with pipe shock
fail to vasoconstrict in the periphery and therefore usually have warm skin. Tachycar-
dia may be absent and bradycardia is often observed in spinal shock, particularly
when the level of the spinal cord injury is at or above T-4. Importantly, the distribu-
tive cause of the shock also underlies the early failure of redistributive compensatory
mechanisms. Neck veins are typically flat and the central venous pressure remains
low.
TYPES OF SHOCK
Hypovolemic Shock
The most common shock state encountered by the surgeon is hypovolemic
shock. Acute hypovolemia causes a parallel left shift of the venous return curve. The
intersection with the normal cardiac function curve also shifts down and to the left.
The neuroendocrine response, by releasing catecholamines into the circulation, ro-
tates the cardiac function curve up and to the left, increasing cardiac flow, but only
marginally. Clinically, tachycardia, tachypnea, and oliguria are reliable guides to the
depth of the hypovolemia.
Cardiogenic Shock
Cardiogenic shock causes a pivot, rotating the cardiac function curve down and
to the right. The equilibrium intersection between the venous return curve and the de-
pressed cardiac function curve causes the low cardiac output. The physiologic com-
pensatory response is to increase P ms, thus shifting the venous return curve up and to
the right in parallel to the original venous return curve. This is a good time to consid-
er and compare the vascular effects of dopamine (which has a predominantly vaso-
constrictor effect in high doses) with the effects of dobutamine (which has a more
vasodilatory profile). The two drugs have similar inotropic effects, so that administra-
tion of either drug partially restores cardiac function, pivoting the cardiac function
curve up and to the left. The increase in afterload associated with dopamine may at-
tenuate the left pivot. The effects on the venous return curve are quite different, how-
ever. Dopamine further increases P ms, shifting the venous return curve to the right in
parallel with the other venous return curves. Dobutamine functions quite differently,
keeping P ms roughly constant and pivoting the curve up and to the right as vascular
resistance falls. These effects help to explain why dobutamine is usually preferred
over dopamine in cardiogenic shock.
Septic Shock
Septic shock is the most common form of distributive shock encountered by the
surgeon. Absent medical intervention, the venodilatation of sepsis causes not only a
decrease in venous resistance but a fall in P ms. Volume resuscitation restores P ms to
its normal value, but now with a markedly decreased venous resistance. The compet-
ing cardiac effects of sepsis are readily modeled with appropriate shifts in the cardiac
function curves: whereas afterload reduction tends to increase cardiac performance,
direct myocardial depression overwhelms the advantage of this afterload reduction in
late, uncompensated sepsis, or in sepsis with preexisting cardiac disease. Bone and
colleagues convened a consensus conference to define criteria for categorization of
sepsis-related inflammatory response. The results of their deliberations were pub-
lished in 1992, and these definitions have been widely used in subsequent studies to
categorize inflammatory responses to infection. These authors defined four categories
of clinical disease that represented successive levels of escalating severity of inflam-
matory response. The core concept of Bone and colleagues was that as the burden of
bacterial toxins increases and the extent of endogenous inflammatory response inten-
sifies, the clinical manifestations of the severity of illness become exaggerated and
the risk of death increases. According to this classification severe sepsis is when pa-
tients are not responsive to intravenous fluid infusion for resuscitation or require ino-
tropic or vasopressor agents to maintain systolic blood pressure.
Obstructive Shock
Obstruction to venous return is a surgical emergency. The two common causes
encountered by general surgeons are pericardial tamponade and tension pneumotho-
rax; obstetricians encounter a similar physiologic effect when the gravid uterus press-
es on the inferior vena cava. All abdominal surgeons occasionally cause transient ob-
structive shock by pressing on the inferior vena cava during surgery. Pulmonary em-
bolism and air embolism are the other two major causes of obstructive shock. The ve-
nous return curve is markedly distorted because the pleural pressure exceeds the right
atrial pressure. Venous return no longer depends on the arithmetic difference between
P ms and right atrial pressure, but on the difference between P ms and (the very posi-
tive) pleural pressure. The cardiac function curve is also adversely affected by two
mechanisms. The rightward shift occurs because the transmural filling pressure is ze-
ro when the right atrial pressure falls to the (now positive) value of the pleural pres-
sure. The downward pivot of the cardiac function curve is caused by a reflex increase
in pulmonary vascular resistance. Although there is an endogenous catecholamine
surge, it is apparent from the analysis that neither a volume load nor administration of
exogenous catecholamines will have a significant effect on circulation. The only ef-
fective therapy is immediately to reduce pleural pressure by relieving the tension
pneumothorax. Pericardial tamponade provides a nearly identical analysis, except that
the limitation on transmural pressure is not pleural pressure but pericardial pressure.
Neurogenic Shock
Surgeons encounter neurogenic shock in two arenas: the trauma resuscitation
bay and the operating room. Traumatic spinal injury occurs when the cord is severed
at a level within or above the sympathetic chain, whereas neurogenic shock encoun-
tered in the operating room is the consequence of a neuraxial anesthetic that has ex-
tended beyond its intended effect. Bearing in mind that the heart also receives sympa-
thetic input, there is an important functional distinction between an injury above T-4
and one below T-4. The former depresses cardiac function in addition to affecting ve-
nous return, whereas the latter leaves cardiac performance unaffected. When cardiac
performance is unaffected, limited volume resuscitation and treatment with a pure a-
agonist such as phenylephrine is sufficient therapy. However, if the cardiac sympa-
thetic innervation is compromised, vagal parasympathetic innervation may predomi-
nate and administration of phenylephrine may aggravate reflex bradycardia. To pre-
clude this undesirable effect of therapy, a mixed inotrope and chronotrope such as
dopamine or norepinephrine is used. In extreme cases, temporary cardiac pacing may
be lifesaving. Volume restoration is also required.
Shock recognition and resuscitation: practical aspects
Resuscitation from shock must begin immediately on recognition. Restoration
of oxygen delivery is the imperative. The simple ABC approach is effective: establish
and maintain an airway, ensure breathing with 100% oxygen, and restore the circula-
tion. Supplemental oxygen must be administered by nonrebreather face mask, Ambu-
bag, or tracheal intubation. Chronic obstructive airway disease is not a contraindica-
tion to the administration of oxygen. Ventilation should be confirmed by auscultation
(axillae and stomach) and by demonstration of end-tidal carbon dioxide if the trachea
has been intubated. The neck veins should be inspected to discriminate pump shock
from perfusate shock. If the neck veins are distended, the axillae should be reauscul-
tated to exclude a tension pneumothorax and consideration should be given to the
possibility of pericardial tamponade. The heart should be auscultated to determine
whether heart sounds are audible and, if so, whether abnormal heart sounds such as
pathologic murmurs and third and fourth heart sounds are present. A pulse should be
sought. The absence of central pulses (femoral, carotid) mandates cardiac life support
including cardiopulmonary resuscitation, determination of rhythm, and cardioversion-
defibrillation. More commonly, a faint pulse is palpable. The carotid pulse ordinarily
is present in adults with systolic blood pressures greater than 60 mm Hg. A short,
wide-bore intravenous catheter should be inserted into a peripheral vein and an ali-
quot of blood taken for analysis. Given the frequency of hypovolemic blood loss in
the surgical population, one of the most important steps is an immediate crossmatch-
ing of blood. A hemoglobin determination is also desirable. The venous catheter then
serves as the conduit for rapid infusion of a balanced salt solution such as lactated
Ringer's solution; 20 mL/kg should be administered as rapidly as practicable (within
5 minutes). The fluid bolus serves to increase preload, diminish venous resistance,
and possibly decrease arterial afterload, all of which augment cardiac performance.
Stroke volume improves by this mechanism even in patients who have sustained an
acute myocardial infarction or have pericardial tamponade, as seen in the Guyton dia-
grams. Fluid bolus should therefore be withheld only when there is incontrovertible
evidence that the cause is cardiogenic and associated with frothy pink pulmonary
edema. The fluid bolus can be repeated immediately if the shock is not immediately
responsive. While the fluid is being administered, electrophysiologic monitoring
should begin. An electrocardiogram (ECG) should be obtained and the rhythm moni-
tored continuously. The systemic blood pressure and heart rate should be determined
at regular, frequent (e.g., every 2 to 5 minutes) intervals and recorded on a purpose-
specific form. A pulse oximeter should be applied to determine the oxygen saturation
of capillary blood. The signal may be difficult to obtain because of intense vasocon-
striction and application of the probe to the earlobe or nose may be helpful. The
stomach should be decompressed to prevent the complication of aspiration. None of
the aforementioned methods is especially effective in determining the quality of or-
gan perfusion or the adequacy of shock reversal. The best first proxy for the adequacy
of organ perfusion appears to be urine output, which should be measured every 30
minutes by an indwelling bladder (Foley) catheter. The best first proxy for the ade-
quacy of shock reversal is the pH on arterial blood gas analysis. During shock, the pH
falls into the acid range as a consequence of anaerobic metabolism with obligatory
accumulation of lactic acid. Once resuscitation is adequate, the lactate should be me-
tabolized and the anion gap acidosis should normalize. Persistent anion gap acidosis
suggests inadequate resuscitation or frankly nonviable tissue. A non-anion gap acido-
sis is less worrisome and may follow resuscitation with normal saline. To avoid con-
fusion, Ringer's solution is often used as the balanced salt solution. To reiterate, heart
rate and urine output are often the best indicators of the current depth of shock but do
not indicate adequacy of resuscitation. Because shock is operationally defined as
metabolic shortfall, the adequacy of resuscitation is reflected in tissue perfusion. The
most immediately available measures of this perfusion include arterial blood gas
analysis for pH and the by-product of anaerobic metabolism, lactate. Neither of these
proxies directly assesses oxygen delivery to tissues most vulnerable to nutrient depri-
vation, such as neural tissue. However, owing to the sensitivity of the splanchnic bed
to even mild shock states, perfusion of abdominal viscera can be used as an interme-
diate surrogate. The mucosal lumen of the gastrointestinal (GI) tract is accessible
though the mouth and the rectum, and there appears to be a tight correlation between
the adequacy of mucosal perfusion and outcome. The adequacy of mucosal perfusion
can be assessed by tonometry, a technique that indirectly measures the accumulated
acids in mucosal cells. However, it is disputed whether titration of care to a predeter-
mined tonometric value improves outcome. Given the metabolic imperatives, it is im-
portant frequently to reassess the determinants of oxygen delivery. Oxygen saturation
should be maintained above 95%. After resuscitation from shock, metabolic demands
can be substantial. The ideal hemoglobin concentration in unstable patients is un-
known. Although stable patients can often tolerate hemoglobin levels as low as 7
g/dL, the unstable patient may be better served by targeting a slightly higher level un-
til the shock is fully reversed. The heart rate should be maintained in a physiologic
range, usually between 80 to 120 beats/min. The upper constraint on heart rate is lift-
ed for children, who regulate cardiac output almost exclusively by heart rate. The
stroke volume should be optimized by augmenting preload. If (and usually, only if)
there is uncertainty about the magnitude of the cardiac output, it should be measured.
This can be done by the thermodilution technique [after insertion of a pulmonary ar-
tery catheter (PAC)], by the less invasive dye clearance technique, or by aortic flow
assessment with an esophageal ultrasound transducer. Resuscitation from shock states
ordinarily requires a minimum indexed cardiac output (cardiac index) of 2.5
L/min/m2 of body surface area, and often considerably more, to deliver sufficient ox-
ygen. Although the definition of “sufficient” oxygen delivery is still contested, all
agree that a subnormal oxygen delivery is never adequate. Thus “sufficient” is at least
“normal” oxygen delivery, which is approximately 500 mL/min/m2. Because fully
saturated oxyhemoglobin at a normal hemoglobin concentration of 15 g carries ap-
proximately 20 mL oxygen per 100 mL blood, the minimum cardiac index of 2.5
L/min/m2 is adequate only if there is a normal hemoglobin concentration that is fully
saturated with oxygen. Decreases in either hemoglobin concentration or oxygen satu-
ration require compensatory proportionate increments in cardiac output to maintain
oxygen delivery at a minimum value of 500 mL/min/m2.
Risks of resuscitation
Perhaps the most vexing aspect of shock is that resuscitation is not synonymous
with reversal, much less a guarantee of recovery. Despite timely intervention and ag-
gressive management, a significant number of patients with shock sustain secondary
injuries attributable to reperfusion and inflammation associated with resuscitation.
The biology of reperfusion and inflammation is complex and incompletely under-
stood.
Pulmonary artery catheter in shock
The indications for use of the PAC (Swan-Ganz catheter) are in flux. This device is
nontherapeutic and performance depends heavily on the skill of the operator and the
expertise of the interpreter. Conclusions regarding its value need to be considered in
the context of its use. With this in mind, the PAC appears to be overused. Acknowl-
edging the paucity of data from randomized, prospective studies, it is suggested that
surgeons managing myocardial infarction with progressive cardiogenic shock are jus-
tified in prompt placement of a PAC. The value of the PAC in other forms of shock is
indeterminate, but in any case its use should follow routine management with fluids
and pressors guided by central venous and systemic arterial pressure monitoring.
Failure to respond to routine management or uncertainty concerning the response are
adequate reasons to use a PAC with its attendant risks.
Problem of secondary sepsis
Once a patient has been resuscitated and moved to the intensive care unit
(ICU), the risks of sepsis are increased. The ICU has an indigenous microflora of vir-
ulent organisms available to attack immunocompromised, resuscitated patients whose
defenses are further weakened by invasive devices. Prevention of secondary sepsis
remains an important but very elusive goal for surgical critical care. The nosocomial
component of ICU sepsis is widely acknowledged but poorly controlled. Transmis-
sion of disease from patient to provider to patient has been documented in many stud-
ies, yet compliance with hand washing and barrier (isolation) directives is poor. The
indigenous flora is different from the endogenous flora, most likely as a consequence
of widespread (ab) use of potent antimicrobials. As in the initial shock, recognition
that the resuscitated patient is becoming secondarily septic is difficult. Once the diag-
nosis is suspected, selection of appropriate antibiotic therapy is relatively simple.
Shock in the Operating Room
Although anesthetics and operations have become progressively safer, most
surgeons are eventually confronted with sudden circulatory collapse of a patient in
the operating room. Such situations can be salvaged if the surgeon and anesthesiolo-
gist work rapidly to analyze and correct the problem. Should the anesthesiologist an-
nounce that the patient is in extremis, the most important next step is to determine
whether there is ventilation and circulation. Presence of carbon dioxide in the end-
tidal gas confirms that both are present. Conversely, absence of end-tidal carbon di-
oxide means that either ventilation or circulation, or both, has failed. Such failure re-
quires immediate confirmation that the endotracheal tube is in the airway, immediate
ventilation, and initiation of cardiac compression while the underlying cause of the
arrest is sought. Open cardiac massage is more effective than closed massage, and
there should be no hesitation in performing a sternotomy or thoracotomy if closed
massage is not immediately effective. The cardiac rhythm should be inspected on the
monitor, and the anesthesiologist asked about any changes in morphology (suggestive
of myocardial ischemia or infarction) before the collapse. If a life-threatening ar-
rhythmia is noted, it should be treated using advanced cardiac life support guidelines.
If ventilation and circulation are present but there is circulatory collapse in the con-
text of a reasonably normal cardiac rhythm, the next step is to look at the operative
field while asking the anesthesiologist about the airway pressures. The surgeon must
look for excessive bleeding and at the shape of the diaphragms. If significant bleeding
is observed, isolation and control become the next priority. The reason to inspect the
diaphragms while asking about airway pressures is that pneumothoraces are not only
common, but quickly become tension pneumothoraces under positive-pressure venti-
lation. The diaphragm on the affected side billows into the abdomen and remains
relatively distended throughout the ventilatory cycle. The airway pressures are higher
than previously observed. If such a billowing diaphragm is observed, it should be
immediately incised (1 to 2 cm) to convert the tension pneumothorax into an open
pneumothorax. While the surgeon is inspecting the diaphragms, the anesthesiologist
should be listening for breath sounds and heart sounds. The reason for listening to the
heart sounds is to exclude a rarer cause of obstructive shock, air embolism, a cause
that should be suspected in any patient who either has a central venous catheter in
place or who has had a large vein open in the operative field. Diagnosis is based en-
tirely on suspicion, but the central venous catheters should be inspected and the heart
should be auscultated for a continuous murmur. If air embolism is thought likely, an
attempt should be made to aspirate air back through the central catheter while the pa-
tient is placed in Trendelenburg position. Management of this complication in the op-
erating room is typically operative, aspirating the right ventricular outflow tract by
direct puncture if cardiac massage proves insufficient immediately to break up the air
lock. If breath sounds and heart sounds are normal and bleeding is not a problem, it
should be ascertained whether there was a drop in end-tidal carbon dioxide just before
the circulatory collapse. When such a drop has occurred, it suggests acute pulmonary
embolism. Refractory shock caused by acute pulmonary embolism can occasionally
be reversed by direct cardiac massage (breaking up the large embolus into smaller
pieces) or, if appropriate personnel and equipment are immediately available, surgical
retrieval of the clot. Finally, the possibilities of anaphylaxis to a recently administered
drug and of a major transfusion reaction need to be considered. In the operating room
and ICU, a view of the heart and aorta in real time can provide helpful information
about cardiac performance. Personnel skilled in transesophageal echocardiography
who can avail themselves of the necessary equipment can rapidly obtain information
about cardiac performance, exclude pericardial tamponade, and make inferences
about whether the venous system is sufficiently filled within a minute or two. Resus-
citation should not be interrupted while the views are being obtained.
Shock in the Immediate (0- to 4-Hour) Postoperative Period
Shock in the immediate postoperative period is attributed to bleeding until
proven otherwise. Plans should be made to return the patient to the operating room
while an alternative cause is sought. Alternative causes are common and include
acute myocardial dysfunction and delayed presentation of a pneumothorax after posi-
tive-pressure ventilation. The value of an immediate ECG and chest radiograph can-
not be overemphasized. More often than not, bleeding is either an obvious cause of
the shock state or is suggested by a lower-than-expected hematocrit. Although explo-
ration of the surgical site is mandatory, the cause of the bleeding is not always surgi-
cal, and appropriate coagulation studies should be ordered along with blood products
as soon as immediate postoperative shock is recognized.
Shock in the Intermediate (4- to 24-Hour) Postoperative Period
As anesthetics and pain medications wear off, patients often experience signifi-
cant pain and respond with a catecholamine surge. The associated increase in heart
rate can cause or mask an evolving myocardial infarction in patients at cardiovascular
risk. Surgical site pain can extinguish anginal pain, and an ECG along with chemical
tests for myocardial damage should be obtained promptly. Bleeding should be no
lower than number two on the differential diagnosis of shock, and resuscitation
should proceed even while plans are made to return the patient to the operating room.
During this interval, serious surgical site infections can cause shock. These site infec-
tions, typically streptococcal, cause a brawny cellulitis (sometimes associated with
brown edema fluid) that masks a necrotizing myofascial infection. For this reason,
shock appearing during the intermediate postoperative period mandates at least an in-
spection of the wound. If cellulitis is present, the wound should be promptly explored
in the operating room, where radical debridement is undertaken. Aggressive antibiotic
therapy, an adjunct to surgical debridement (not a substitute), may be lifesaving.
Shock in the Late (>24 Hours) Postoperative Period
There are four common causes of unexplained shock in the late postoperative
period. Sepsis is by far the most common, including surgical site infections, blood-
stream (catheter-associated) infections, urinary tract infections, and pneumonias. My-
ocardial infarction is also common and can occur without significant pain during the
first few postoperative days. Pulmonary embolism in the setting of occult deep ve-
nous thrombosis tends to occur somewhat later because the operation and consequent
immobility are usually the cause of the deep venous thrombosis, and pulmonary em-
bolism must follow its formation. Shock and unexplained hypoxemia should suggest
pulmonary embolism. Finally, occult GI bleeding causes painless hypovolemic shock
that is unexplained until the oral or rectal passage of blood. Above all, the surgeon
should be aware of time. Regardless of the cause of the shock, prompt recognition of
the shock state, correction of the underlying problem, and immediate resuscitation
appear to be the best guarantors of a favorable outcome.
Acute Lung Injury/Acute Respiratory Distress Syndrome
Acute lung injury and ARDS are clinical syndromes of pulmonary dysfunction
that may result from any number of infectious, inflammatory, or tissue injury or cellu-
lar shock conditions.
Criteria for the diagnosis of ARDS include:
• acute onset,
• bilateral pulmonary infiltrates on chest radiography,
• the absence of cardiogenic pulmonary edema (i.e., pulmonary artery wedge pres-
sure < 18 mm Hg), and
• hypoxemia (PaO2 :FIO2 < 200).
On the same continuum, acute lung injury is a milder form, with PaO2 :FIO2 =
201–300.
The mortality of ARDS approaches 40% to 50%, with most deaths attributed to
MOF.
The pathogenesis of ARDS involves three stages. The first stage, coinciding
with the acute onset of respiratory failure, is known as the exudative phase. Disrup-
tion of the alveolar epithelium results in the influx of protein-rich edema fluid and a
leukocytic infiltrate. Destruction of type II pneumocytes disrupts normal alveolar flu-
id transport and surfactant production, leading to alveolar flooding and collapse. Mac-
rophages release proinflammatory cytokines that attract and activate neutrophils, pro-
voking tissue injury. Some patients have an uncomplicated course with resolution of
the process, but others progress to the fibroproliferative phase. Mesenchymal cells fill
the alveolar space and initiate fibrosis, with collagen and fibronectin accumulating in
the lung. In the resolution phase, alveolar edema is resolved as type II pneumocytes
repopulate the epithelium; protein is cleared; and there is gradual remodeling of gran-
ulation tissue and fibrosis.
The treatment of ARDS is primarily supportive.
1. The underlying cause should be identified and treated.
2. Nutritional support should be provided.
3. Appropriate prophylactic measures against venous thromboembolism and stress
gastritis.
Adequate oxygenation and ventilation must be provided; this generally requires
intubation and mechanical ventilation. A number of novel adjunctive therapies have
been studied in ARDS. Preliminary clinical studies suggest that fluid management
aimed at lowering filling pressures may decrease pulmonary edema; whether this im-
proves outcome remains to be seen. Surfactant-replacement therapy has been suc-
cessful in neonates but not yet proven beneficial in adults with ARDS. Despite en-
couraging results in observational studies, nitric oxide has not proven beneficial in
PRCTs; the same goes for other vasodilators. Corticosteroids were never found to be
beneficial when administered early in ARDS. However, as the pathophysiology be-
came better understood, the therapy was applied to the fibroproliferative phase. En-
couraging results were reported in observational studies as well as in a small PRCT.
Corticosteroids warrant consideration as salvage therapy for severe ARDS that is not
resolving but must be used with caution because they predispose patients to the risk
of infection.
The optimal ventilatory strategy for ARDS patients remains elusive. A number
of methods have been employed, including extracorporeal membrane oxygenation
(ECMO); extracorporeal carbon dioxide removal; high-frequency jet ventilation;
high-frequency oscillatory ventilation; liquid ventilation; permissive hypercapnia; and
inverse-ratio ventilation. None of these has been associated with a mortality reduc-
tion. Prone positioning has been proposed as a means to improve oxygenation by in-
creasing end-expiratory lung volume, improving ventilation and perfusion matching,
and changing chest wall mechanics. In a multicenter PRCT, prone positioning im-
proved oxygenation but not survival. Although this intervention may be useful in
treating severe hypoxemia for short periods, care must be exercised to minimize
complications such as pressure ulceration, accidental extubation, and loss of vascular
catheters and feeding/drainage tubes. Low tidal volume (VT) ventilation has been the
focus of a number of PRCTs. The National Institutes of Health ARDS Network study
group performed a multicenter PRCT in which patients were randomized to a VT of
12 mL/kg vs 6 mL/kg, with plateau pressures maintained at less than 50 versus less
than 30 cm H2O, respectively. After enrolling 861 patients, the trial was stopped be-
cause in-hospital mortality was reduced from 40% to 31% in the low VT group. The
results of this study were discrepant from earlier, smaller trials. Whether the benefit
was attributable solely to lower VT is unclear; nevertheless, this approach has gained
widespread support. Positive end-expiratory pressure (PEEP) can improve oxygena-
tion by recruiting collapsed alveoli and reducing functional residual capacity. “Con-
ventional” ventilation generally calls for the minimal PEEP necessary to provide ac-
ceptable oxygenation. However, in the setting of ARDS, there may be benefit to in-
creasing PEEP to improve oxygenation as well as to protect the lung by preventing
repetitive recruitment/de-recruitment of alveoli, reducing cyclic reopening and stretch
during mechanical breaths. The optimal level of PEEP may be determined by incre-
mentally increasing PEEP to maximize the PaO2 :FIO2 ratio; however, some argue
that this ignores lung mechanics. A lung pressure-volume curve may be generated for
a given patient, and the lower inflection point (PFLEX)—the point at which the slope
increases in steepness, representing a pressure at which the majority of alveolar units
are open—identified. Alternatively, the PEEP may be titrated to maximal compliance,
which may be easier to measure at the bedside. A “lung-protective” strategy em-
ployed in a PRCT included a VT less than 6 mL/kg, PEEP above PFLEX, driving
pressures less than 20 cm H2O above the PEEP level, pressure-limited ventilation,
and permissive hypercapnia. Compared with conventional ventilation, there was im-
proved 28-day survival, less barotrauma, and a higher rate of weaning from mechani-
cal ventilation. This trial was small and had a higher than expected mortality in the
conventional ventilation group, but it has stimulated further study into the use of
higher PEEP levels.
A circulatory collapse is defined as a general or specific failure of the circula-
tion, either cardiac or peripheral in nature. A common cause of this could be shock or
trauma from injury or surgery. A "general failure" is one that occurs across a wide
range of locations in the body, such as systemic shock after the loss of a large amount
of blood collapsing all the circulatory systems in the legs. A specific failure can be
traced to a particular point, such as a clot.
Intra-Abdominal Hypertension (IAH) and Abdominal Compartment Syn-
drome (ACS)
As John Hunt stated abdominal Compartment Syndrome is “.the end result of a
progressive, unchecked increase in intra-abdominal pressure from a myriad of disor-
ders that eventually leads to multiple organ dysfunction.”
Korn and associate first used the term ACS in 1980s. It is only in the past dec-
ade, that the pathophysiological repercussions of the increased intraabdominal pres-
sure (IAP) and ACS have been recognised in a wide spectrum of surgical patients and
treated aggressively. Emerson first noted the cardiovascular morbidity and mortality
associated with elevated intra-abdominal pressure in 1911. However, the recognition
of abdomen as a compartment and the concept of intraabdominal hypertension (IAH)
resulting in ACS have only recently received attention.
The abdomen is a closed cavity. The pressure values inside follow the hydro-
static laws. The IAP is the steady state pressure within the abdominal cavity and
changes with respiration.
Factors influencing IAP:
• The movements of diaphragm
• The costal arch shifts
• The contractions of the abdominal wall
• The volume of the intestines, which may be empty or over distended
• The presence of any additional content in the abdominal cavity
Existing points of view concerning normal values of IAP:
• Normally, mean intra-abdominal pressures are zero or less (A.A. Meyer).
• The normal values of IAP are 0-5 mm Hg (M. Malbrain, 2002).
• Mean intraabdominal pressure is 6.5 mm Hg (range 0.2-16.2 mm Hg) (N.C.
Sanchez et al., 2001).
Traditionally, ACS was considered a traumatic surgical disease. Nowadays it is
proved that ACS is a problem in many critically ill patients who have suffered no
trauma, especially those suffering systemic inflammatory response syndromes
(SIRS). In untreated cases mortality is about 100%.
Causes of Intra-abdominal Pressure (IAP) Elevation
• Retroperitoneal: pancreatitis, retroperitoneal or pelvic bleeding, contained AAA
rupture, aortic surgery, abscess, visceral edema
• Intraperitoneal: intraperitoneal bleeding, acute gastric dilatation, bowel obstruction,
ileus, mesenteric venous obstruction, pneumoperitoneum, abdominal packing, ab-
scess, visceral edema secondary to resuscitation (SIRS)
• Abdominal Wall: burn eschar, repair of gastroschisis or omphalocele, reduction of
large hernias, pneumatic anti-shock garments, lap closure under tension, abdominal
binders
• Chronic: central obesity, ascites, large abdominal tumors, peritoneal dialysis, preg-
nancy.
The following predisposing conditions are well recognised:
• hypothermia
• acidosis
• polytransfusion
• dilutional coagulopathy or disseminated intravascular coagulation
• sepsis with capillary leakage
• vasculopathy, or liver dysfunction
The combination of acidosis, hypothermia and coagulopathy has been proposed as a
triad, leading to ACS.
Grading of ACS (J.M. Burch and colleagues, 1996)
• Grade I = 10 to 15 mm of Hg
• Grade II = 15 to 25 mm of Hg
• Grade III = 25 to 35 mm of Hg
• Grade IV >35 mm of Hg
Morris definition of ACS:
1) A pathologic state caused by an acute increase in IAP above 20-25 mm Hg (or
27.2-34 cmH2O)
2) End-organ dysfunction or serious wound complications and
3) Improvement by abdominal decompression
Types of ACS
1. PRIMARY ACS is essentially organ dysfunction and IAH in the presence of
direct injury to the abdominal contents. The examples are trauma, peritonitis, ileus,
and haemorrhage etc.
2. SECONDARY ACS consists of elevated pressure and organ dysfunction
caused by third space oedema and resuscitation. The examples are resuscitation of
haemorrhagic shock patients, burns etc.
3. RECURRENT ACS in which the patient has recovered from the ACS once
but because of secondary insults the cycle begins again. This verity is associated with
very high mortality rate.
Physiologic sequelae of high intraabdominal pressure
1. Cardiovascular:
Increased intra-abdominal pressures causes:
• Decrease in the preload
• Increase in the afterload due to an elevation in systemic vascular resistance
(SVR)
• Impairment of cardiac contractility
End result: Dramatic reduction in venous return to the heart (preload). Right
coronary artery blood flow drop with resultant RV subendocardial ischemia and
worsening cardiac dysfunction. Elevated SVR leads to reduced blood flow to organs
already suffering from ischemia and venous engorgement. They are now more is-
chemic and the capillary leak worsens, further exacerbating the syndrome.
Elevated intra-thoracic pressure directly impacts traditional pressure-based car-
diac filling measurements such as CVP and PAOP (wedge). These pressure meas-
urements are erroneously elevated and do not reflect actual fluid resuscitation end-
points. Failure to understand this, and reliance on pressure-based cardiac indices will
lead to inadequate fluid resuscitation, persistent global organ ischemia and higher in-
stances of MOF and death. Volumetric indices such as RVEDVI and GEDVI accu-
rately reflect fluid volume status in the face of elevated IAP and ITP. Focusing vol-
ume resuscitation end points on a volume-based index will result in improved cardiac
function and reduced organ failure.
2. Pulmonary:
Increased intra-abdominal pressures causes:
• Elevation of the diaphragms with reduction in lung volumes
The result:
• Elevated intrathoracic pressure (which further reduces venous return to heart, ex-
acerbating cardiac problems)
• Increased peak pressures
• Reduced tidal volumes
• Barotrauma, atelectasis, hypoxia, hypercarbia
3. Gastrointestinal:
Increased intra-abdominal pressures causes:
• Compression of mesenteric arteries
• Congestion of mesenteric veins and capillaries
• Reduced cardiac output to the gut
The result:
• Decreased gut perfusion, increased gut edema and leak
• Ischemia, necrosis, cytokine release
• Bacterial translocation
• Development and perpetuation of SIRS
• Further increases in intra-abdominal pressure
4. Renal:
Elevated intra-abdominal pressure causes:
• Compression of renal veins and arteries
• Reduced cardiac output to kidneys
The Result:
• Decreased renal artery and vein flow
• Renal congestion and edema
• Decreased glomerular filtration rate (GFR)
• Acute tubular necrosis (ATN)
• Renal failure, oliguria/anuria
5. Neuro:
Elevated intra-abdominal pressure causes:
• Increases in intrathoracic pressure
• Increases in superior vena cava (SVC) pressure with reduction in drainage of
SVC into the thorax
The Result:
• Increased central venous pressure and IJ pressure
• Increased intracranial pressure
• Decreased cerebral perfusion pressure
• Cerebral edema, brain anoxia, brain injury
6. Miscellaneous
Elevated intra-abdominal pressure causes:
• Reduces perfusion of surgical and
• traumatic wounds
• Reduced blood flow to critical organs and tissues
The Result:
• Poor wound healing and dehiscence
• Coagulopathy
• Immunosuppression
Classic signs of ACS (It normally develops 12 to 24 hours after the first
operation) are:
• decreased PO2
• very highly elevated PCO2
• high peak inspiratory pressure
• lack of urinary output
• a massively distended abdomen.
A better way of diagnosing this condition, however, is through continuous intra-
abdominal pressure monitoring in the intensive care unit (ICU) in all critically ill pa-
tients at high risk for these complications.
The principle ways of assessing pressure are:
• inferior vena cava catheter
• urinary bladder catheter
• peritoneal catheter
• vaginal catheter
• gastric balloon
Intraabdominal pressure is most commonly measured indirectly by monitoring
bladder pressures. Bladder pressure monitoring through the Foley catheter is compa-
rable to direct intraperitoneal pressure measurements, but is non-invasive, more relia-
ble and reproducible than clinical judgment, allows early detection of intra-abdominal
hypertension, allowing intervention before ACS develops.
IAH/ACS Management
• Fluids – adequate fluid resuscitation
o normovolemic in patients with Grade I IAH
o hypervolemic in patients with Grade II-IV
• Abdominal perfusion pressure-optimize fluids first then add vasopressors. Shoot
for a on perfusion pressure > 60 mm Hg
• Paralytics
• Cathartics / enema to clear bowel?
• Paracentesis
o need significant free fluid on US
o can place temporary catheter
• Decompressive laparotomy – in patients with Grade III–IV is obligatory, in some
patients with Grade II also can be recommended,
o can be performed bedside for unstable patients
o delay in abdominal decompression may lead to intestinal ischemia
Post-laparotomy ACS – Same problem the same is the treatment
• Be aware that ACS can recur following a decompression laparotomy
• Score or replace dressing to treat recurrence
Indications for IAP monitoring:
1. Sepsis/SIRS/Ischemia-reperfusion
a. Sepsis and resuscitation with > 6 l crystalloid/colloid or > 4 units of blood in 8
hours
b. Pancreatitis
c. Peritonitis
d. Ileus/bowel obstruction
e. Mesenteric ischemia/necrosis
2. Visceral compression/reduction
a. Large ascites/peritoneal dialysis
b. Retroperitoneal/abdominal tumor
c. Laparotomy closed under tension
d. Gastroschisis/omphalocele
3. Surgical
a. Intra-operative fluid balance > 6 l
b. Abdominal aortic aneurysm repair
4. Trauma
a. Shock requiring resuscitation (ischemia-reperfusion)
b. Damage control laparotomy
c. Multiple trauma with or without abdominal trauma requiring resuscitation with
> 6 l crystalloid/colloid or > 4 units blood in 8 hours
d. Major burns (> 25%)
Basic literatures:

1. Townsend: Sabiston Textbook of Surgery, 18th ed.


2. Essentials of Surgery: Scientific Principles and Practice 2nd edition / Green-
field L. J., Mulholland M. W., Oldham K. T., Zelenock G. B., Lillimoe K. D.,
Oldham K. – 1997. - Lippincott Williams & Wilkins Publishers.
3. Oxford Textbook of Surgery (3-Volume Set) 2nd edition / Morris P. J., Wood
W. C. – 2000. - Oxford Press
4. Schwartz’s manual of surgery (8th edition) / Brunicardi F. - 2006 C.
McGRAW-HILL Medical Publishing Division New York, Chicago, San Fran-
cisco, Lisbon, London, Madrid, Mexico City, Milan, New Delhi, San Juan,
Seoul, Singapore, Sydney, Toronto.
Additional literatures:
1. Zhang M. et al. Rapid detection of pneumothorax by ultrasonography in pa-
tients with multiple trauma. Critical care. – 2006. – 10;R112.
2. Udeani J. Hemorrhagic shock. – 2011. – available at
http://emedicine.medscape.com
3. Fairichild E. Study examines sepsis and septic shock after surgery. –
Arch.Surg. 2010. – vol.145, №7. – P.695-700.

Tests for initial level of knowledge, keys for tests:


1. Signs and symptoms associated with early sepsis include
A. Respiratory acidosis
B. Decreased cardiac output
C. Hypoglycemia
D. Increased arteriovenous oxygen difference
E. Cutaneous vasodilation
2. Major alterations in pulmonary function associated with adult respiratory distress
syndrome (ARDS) include
A. Hypoxemia
B. Increased pulmonary compliance
C. Increased resting lung volume
D. Increased functional residual capacity
E. Decreased dead space ventilation

3. A conscious person in shock should be given:


A. coffee or tea.
B. orange juice.
C. clear liquids.
D. nothing by mouth
E. alcohol

4. The response to shock includes which of the following metabolic effects?


A. Increase in sodium and water excretion
B. Increase in renal perfusion
C. Decrease in cortisol levels
D. Hyperkalemia
E. Hypoglycemia

5. Animal and clinical studies have shown that administration of lactated Ringer’s so-
lution to patients with hypovolemic shock may
A. Increase serum lactate concentration
B. Impair liver function
C. Improve hemodynamics by alleviating the deficit in the interstitial fluid compart-
ment
D. Increase metabolic acidosis
E. Increase the need for blood transfusion

6. Which of the following signs and symptoms suggest possible hypovolemic shock?
A. Weak, rapid pulse; cold, clammy skin; pallor; shallow breathing
B. Slow, strong pulse; dizziness; cold perspiration; nausea
C. Blank expression; cold extremities; regular breathing
D. Blank expression; chills; unconsciousness; dry skin

7. Which of the following statements regarding stress ulceration is true?


A. It is true ulceration, extending into and through the muscularis mucosa
B. It classically involves the antrum
C. Increased secretion of gastric acid has been shown to play a causative role
D. It frequently involves multiple sites
E. It is seen following shock or sepsis, but for some unknown reason does not occur
following major surgery, trauma, or burns

8. The principle ways of assessing intraabdominal pressure are the following


EXCEPT:
A. superior vena cava catheter
B. urinary bladder catheter
C. peritoneal catheter
D. vaginal catheter
E. gastric balloon

9. Predisposing factors for the development of intraabdominal hypertension include


the following EXCEPT:
A. hypothermia
B. acidosis
C. polytransfusion
D. dilutional coagulopathy or disseminated intravascular coagulation
E. spinal cord injury

10. The clinical signs of ACS don’t include:


A. decreased PO2
B. very highly elevated PCO2
C. high peak expiratory pressure
D. lack of urinary output
E. a massively distended abdomen.

Keys for tests

1 2 3 4 5 6 7 8 9 10
E A D D C A D A E C

Tests for final level of knowledge, keys for tests:

1. Which of these hernias can most likely cause acute respiratory distress in infants?
A. Rupture of the diaphragm
B. Paraesophageal hiatal hernia
C. Sliding hiatal hernia
D. Foramen of Bochdalek hernia
E. Foramen of Morgagni hernia

2. A full-term male newborn experiences respiratory distress immediately after birth.


A prenatal sonogram had been read as normal. An emergency radiograph is shown
below. The patient was intubated and placed on 100% O2. The arterial blood gas re-
vealed pH 7.24; PO2 60 kPa; PCO2 52 kPa. The baby has sternal retractions and a
scaphoid abdomen. Which of the following statements correctly refers to this condi-
tion?
A. The most likely cause of this problem is in utero traumatic rupture of the dia-
phragm
B. The most important aspect in management would be immediate exploration and
repair of the defect
C. The size of the defect directly correlates with severity of the disease
D. The defect is usually anteromedial in location
E. Any abdominal organ can be involved
3. The use of oxygen in treatment of shock in considered to be:
A. a routine part of treatment.
B. of no particular benefit to the patient.
C. a last resort measure if all other treatment fails.
D. useful only if the patient has difficulty breathing.
E. of a benefit in patients with AIRDS

4. When operating to repair civilian colon injuries (choose the correct statement)
A. A colostomy should be performed for colonic injury in the presence of gross fecal
contamination
B. The presence of shock on admission or more than two associated intraabdominal
injuries is an absolute contraindication to primary colonic repair
C. Distal sigmoidal injuries should not be repaired primarily
D. Right-sided colonic wounds should not be repaired primarily
E. Administration of intravenous antibiotics with aerobic and anaerobic coverage has
not been shown to decrease the incidence of wound infections after repair of colonic
injuries

5. Choose the correct statement concerning the use of the pneumatic antishock gar-
ment (PASG)
A. Elevates blood pressure by an “autotransfusion” effect, with augmentation of ve-
nous return and cardiac output
B. Is not recommended for control of persistent bleeding in the setting of severe pel-
vic fracture
C. Increases peripheral vascular resistance
D. Expedites assessment of lower body injuries in the trauma patient
E. Should be terminated by means of prompt deflation as soon as the trauma patient
reaches the emergency department

6. Which of the following parts of the body deteriorates the fastest without constant
perfusion?
A. Heart
B. Liver
C. Kidneys
D. Skeletal muscle
E. Skin

7. Cardiogenic shock develops when:


A. too much blood is lost.
B. sympathetic vascular tone is lost.
C. the lungs no longer take in oxygen.
D. the heart no longer pumps effectively.
E. the volume of vascular fluid is reduced

8. Treatment of a patient in anaphylactic shock should include:


A. immersion in an ice bath.
B. administration of oxygen.
C. vigorous massage of the extremities.
D. intramuscular injection of nitroglycerin
E. intramuscular injection of sympathomimetics

9. Psychogenic shock can be caused by:


A. loss of blood or body fluid.
B. damage to the cervical spine.
C. an allergic reaction to insect bites.
D. temporary, generalized vascular dilation.
E. intramuscular injection of sympathomimetics

10. Which of the following terms best describes the failure of the circulatory system
to provide sufficient circulation so that each body part can perform it functions?
A. Shock
B. Epistaxis
C. Perfusion
D. Oxygenation
E. Distress syndrome

Keys for tests

1 2 3 4 5 6 7 8 9 10
D E A B C A D B D A

Tasks for the final level of knowledge.

1. A 68-year-old hypertensive man undergoes successful repair of a ruptured ab-


dominal aortic aneurysm. He receives 9 L Ringer’s lactate solution and 4 units of
whole blood during the operation. Two hours after transfer to the surgical intensive
care unit, the following hemodynamic parameters are obtained:
• Systemic blood pressure (BP): 90/60 mm Hg
• Pulse rate: 110 beats/min
• Central venous pressure (CVP): 7 mm Hg
• Pulmonary artery pressure: 28/10 mm Hg
• Pulmonary capillary wedge pres-sure: 8 mm Hg
• Cardiac output: 1.9 L/min
• Systemic vascular resistance: 35 Woods units (normal is 24–30 Woods units)
•PaO2 : 140 kPa (FiO2 : 0.45)
• Urine output: 15 mL/h (specific gravity: 1.029)
• Hematocrit: 35%
What is proper management?

The answer is administration of a fluid challenge to increase urine output

2. The patient after the repair of a ruptured abdominal aortic aneurysm has an im-
provement in all hemodynamic parameters. However, 6 h later he develops ST seg-
ment depression, and a 12-lead cardiogram shows anterolateral ischemia. New hemo-
dynamic parameters are obtained:
• Systemic BP: 70/40 mm Hg
• Pulse rate: 100 beats/min
• Central venous pressure (CVP): 18 cm H2O
• Pulmonary capillary wedge pressure (PCWP): 25 mm Hg
• Cardiac output: 1.5 L/min
• Systemic vascular resistance: 25 Woods units
What would be the single best pharmacologic intervention?

The answer is dobutamine.

3. A 31-year-old man is brought to the emergency room following an automobile ac-


cident in which his chest struck the steering wheel. Examination reveals stable vital
signs, but the patient exhibits multiple palpable rib fractures and paradoxical move-
ment of the right side of the chest. Chest x-ray shows no evidence of pneumothorax
or hemothorax, but a large pulmonary contusion is developing. What would be the
proper treatment?

The answer is tracheostomy, mechanical ventilation, and positive end-expiratory


pressure

4. A 42-year-old man sustained gunshot wound to the abdomen and is in shock. Mul-
tiple units of packed red blood cells are transfused in an effort to resuscitate him. He
complains of numbness around his mouth and displays carbopetal spasm and a posi-
tive Chvostek’s sign. An electrocardiogram demonstrates a prolonged QT interval.
Which of the following is the most appropriate treatment?

The answer is Intravenous calcium

5. You are called for a 56-year-old man whose wife is having difficulty keeping him
awake. He is displaying signs of shock, including tachycardia and diaphoresis. The
patient's wife tells you that her husband had an episode of severe chest pain about 2
days ago. What additional sign would tend to accompany the particular type of shock
that this patient is experiencing?

The answer is fluid in the lungs

Materials for self-education

Main tasks Notes(instructions)

Repeat: 1. Make a scheme of blood circula-


1. Anatomy of the vascular system tion, formula of an average blood
2. Physiology of blood circulation pressure calculation
3. Physiology of breathing 2. Formula of peripheral vascular re-
sistance assessment
Study:
1. Surgical approaches to the heart to per-
1. Make the algorithm of actions in
form direct heart massage
case of circulatory collapse in opera-
2. BSI technique
tive room
3. Methods of evaluation of abdominal
2. Make the algorithms of actions in
compliance
cases of postoperative shock
4. The optimal ventilatory strategy for
ARDS patients
Study guide #4
“Abdominal pain, estimation features of pain syndrome and differential diagno-
sis of abdominal viscose diseases. The syndrome of dysphagia, vomiting and vio-
lation of defecation, their values in diagnostics and differential diagnosis.”

Overview.

Abdominal pain accounts for 5% to 10% of all emergency department visits or


5 to 10 million patient encounters in the United States annually. Another study
demonstrates that 25% of patients presenting to the emergency department com-
plained of abdominal pain. Diagnoses vary according to age group: pediatric, geriat-
ric, and everyone else who deals with abdominal pain in children. Appendicitis is
more common in children, whereas biliary disease, colonic diverticulitis, and intesti-
nal infarction occur more commonly in the elderly. Hospitalized patients may devel-
op abdominal pain during the course of their illness, making diagnosis and treatment
more difficult.
Educational aims:
1. To collect the anamnesis and to make clinical inspection on patients with ab-
dominal pain.
2. To find the etiology and pathogenesis of abdominal pain.
3. To understand the clinical pictures of abdominal pain.
4. The complications of abdominal pain.
5. To make plan of inspection on patients with abdominal pain.
6. To carry out the analysis of data received from laboratory and tool methods of
inspection on patients with abdominal pain.
7. To define indications for surgical treatment on patients with abdominal pain or
to appoint conservative treatment.
8. To treat patients with abdominal pain.
9. To estimate work capacity for patients with abdominal pain.

A student must know:

1, Anatomical-physiological information about organs of the abdominal cavity.


2. Classification of acute abdominal pain.
3, Clinical picture of acute abdominal pain and acute abdomen.
4. Methods of observation patients with abdominal pain
5. Giving first aid for abdominal pain.
6. Medical program (conservative and surgical treatment) for patients with ab-
dominal pain.
7. Question of rehabilitation patients with acute abdomen.
A student must be able to:

1. Anatomical-physiological features of innervations of organs of abdominal cavi-


ty
2. Etiology and pathogenesis of acute abdominal pain.
3. Methods of inspection of the abdominal cavity organs.
4. Clinical examinations and tests.
5. Ultrasonography, computered tomography, peritoneal lavage and endoscopic ul-
trasound to establish the correct diagnosis.
6. Clinical symptomatic of acute abdomen and its complicated forms.
7. Differential diagnostic of acute abdominal pain.
8. Testimony and contra-indication to surgical and conservative methods of treat-
ment.
9. Methods of surgical treatment of different forms of acute abdomen.

Terminology.

Term Definition
Pain The focal issue in the evaluation of the patient suspected of
having an acute abdomen.
Parietal pain Associated with intra-abdominal disorders may be more in-
tense and precisely localized.
Referred pain Perceived at a site distant from the source of stimulus.
Visceral pain dull and poorly localized, usually in the epigastrium peri-
umbilical region, or suprapubic region
Colicky pain this pain is assumed as crescendo-decrescendo character
Guarding the detection of increased abdominal muscle tone during
palpation
Content:

ANATOMY AND PHYSIOLOGY


Developmental Anatomy
The developmental anatomy of the abdominal cavity and of its viscera deter-
mines normal structure and influences the pathogenesis and clinical manifestations of
most abdominal diseases. Peritoneal attachments and visceral sensory innervations
are particularly important to the evaluation of acute abdominal disease. After the 3rd
week of fetal development, the primitive gut divides into foregut, midgut, and hind-
gut. The superior mesenteric artery supplies the midgut (the fourth portion of the du-
odenum to the midtransverse colon). The foregut includes the pharynx, the esopha-
gus, the stomach, and the proximal duodenum, whereas the hindgut comprises the
distal colon and the rectum. The afferent fibers accompanying the vascular supply
provide sensory innervations to the bowel and associated visceral peritoneum.
Thus, disease in the proximal duodenum (foregut) stimulates celiac axis affer-
ents to produce epigastric pain. Stimuli in the cecum or appendix (midgut) activate
afferent nerves accompanying the superior mesenteric artery to cause periumbilical
pain, and distal colon disease induces inferior mesenteric artery afferent fibers to
cause suprapubic pain. The phrenic nerve and afferent fibers in C3, C4, and C5 der-
matomes accompanying the phrenic arteries innervate the diaphragmatic musculature
and the peritoneum on its undersurface. Stimuli to the diaphragm therefore cause re-
ferred shoulder pain. The parietal peritoneum, abdominal wall, and retroperitoneal
soft tissue receive somatic innervations corresponding to the segmental nerve roots.
The richly innervated parietal peritoneum is particularly sensitive. Parietal peritoneal
surfaces sharply localize painful stimuli to the site of the stimulus. When visceral in-
flammation irritates the parietal peritoneal surface, localization of pain occurs. Ma-
neuvers that exacerbate this irritation then intensify the pain. The many “peritoneal
signs” useful in the clinical diagnosis of the acute abdomen originate in this fashion.
Dual-sensory innervation of the abdominal cavity by both visceral afferents and so-
matic nerves produces clinical pain patterns that aid in diagnosis. For example, the
pain of acute appendicitis originates with poorly localized periumbilical pain pro-
gressing to sharply localized right lower quadrant pain when the inflammation in-
volves the parietal peritoneal surface.
Peripheral nerves mediate sharp, sudden, well-localized pain. Sensory afferents
involved with intraperitoneal abdominal pain transmit dull, sickening, poorly local-
ized pain of more gradual onset and protracted duration. The vagus nerve does not
transmit pain from the gut. Small, unnamed sympathetic afferent nerves transmit pain
from the esophagus to the spinal cord. Afferent nerves from the liver capsule, the he-
patic ligaments, and the central portion of the diaphragm, the splenic capsule, and the
pericardium enter the central nervous system from C3 to C5. The spinal cord from T6
to T9 receives pain fibers from the periphery of the diaphragm, the gallbladder and
the stomach, the pancreas, and the small intestine. Pain fibers from the colon, appen-
dix, and pelvis viscera enter the central nervous system at the 10th and 11th thoracic
segments. The sigmoid colon, rectum, renal pelvis and capsule, ureter, and testes pain
fibers enter the central nervous system at T11 and L1. The bladder and the rectosig-
moid colon send afferent nerves to the spinal cord from S2 to S4.
Cutting, tearing, crushing, or burning usually do not produce pain in the ab-
dominal viscera. However, stretching or distention of the peritoneum produces pain.
Bacterial or chemical peritoneal inflammation produces visceral pain, so does ische-
mia. Cancer can cause intra-abdominal pain by invading sensory nerves. Abdominal
pain may be visceral, parietal, or referred. Visceral pain is dull and poorly localized,
usually in the epigastrium periumbilical region, or suprapubic region, and it usually
does not lateralize well. Patients with visceral pain may also experience sweating,
restlessness, and nausea.
The parietal or somatic pain associated with intra-abdominal disorders may be
more intense and precisely localized. Referred pain is perceived at a site distant from
the source of stimulus. For example, irritation of the diaphragm may produce pain in
the shoulder. Disease in the bile duct or gallbladder may produce shoulder pain.
Distention of the small bowel can produce pain referred to the back. During the
5th week of fetal development, the bowel outgrows the peritoneal cavity, protrudes
through the base of the umbilical cord, and undergoes a 180-degree counterclockwise
rotation. During this process, the bowel remains outside the peritoneal cavity until
approximately the 10th week, when it returns to the abdomen, and an additional 90-
degree counterclockwise rotation occurs. This embryologic rotation places the viscera
in their adult positions, and subsequent fusion of the portions of the colonic and duo-
denal mesenteries with the mesothelium of the posterior abdomen forms the normal
ultimate peritoneal attachments. Knowledge of these attachments is clinically im-
portant during the evaluation of patients with the acute abdomen because of variation
in the exact position of the viscera (e.g., pelvic or retrocecal appendix) and the com-
partmentalization of the abdomen by mesenteric attachments. The latter, for example,
may channel duodenal or gastric contents from the site of a perforated ulcer to the
right lower quadrant.
Peritoneal Pathophysiology
Mesothelial cells cover the visceral and parietal peritoneal surfaces. Openings
into radials arranged lymphatics penetrate the diaphragmatic peritoneal surface.
Introduction of bacteria into the peritoneal cavity can cause an outpouring of
fluid from the peritoneal membrane. This loss of fluid from the circulation may lead
to dehydration and may produce the clinical signs of resting or orthostatic hypoten-
sion and tachycardia. Diaphragmatic lymphatics are the major route for the clearance
of bacteria and cellular debris from the abdominal cavity. This process leads to an in-
traperitoneal circulation of fluid toward the subdiaphragmatic regions bilaterally. Flu-
id not cleared in this fashion tends to accumulate in the deep end of the pelvis. Thus,
subdiaphragmatic, subhepatic, paracolic, or pelvic fluid collections can accompany
visceral perforation. The peritoneal surfaces localize bacteria and the products of in-
flammation. The peritoneum responds to inflammation by increased blood flow, in-
creased permeability, and the formation of a fibrinous exudate on its surface. The
bowel also responds to inflammation with localized or generalized paralysis. The fi-
brinous surface thus created, aided by decreased intestinal movement, causes adher-
ence between bowel and omentum and effectively walls off inflammation. An abscess
may produce sharply localized pain with normal bowel sounds and gastrointestinal
function, whereas a disseminated process, such as a perforated ulcer, produces gener-
alized abdominal pain with a quiet abdomen. Peritonitis may affect the entire ab-
dominal cavity or a portion of the visceral or parietal peritoneum.
Transudation can produce an increase in the peritoneal fluid, which is rich in
protein and leukocytes that facilitate the formation of fibrin on peritoneal surfaces.
Peritonitis denotes peritoneal inflammation from any cause. Primary or spontaneous
peritonitis can occur as a diffuse bacterial infection without an obvious intra-
abdominal source of contamination. Primary peritonitis, most commonly caused by
Pneumococcus or hemolytic Streptococcus, occurs more commonly in children than
in adults. However, adults with ascites and cirrhosis are susceptible to spontaneous
peritonitis resulting from Escherichia coli and Klebsiella.
The more common secondary peritonitis results from perforation, infection, or
gangrene of an intra-abdominal organ, usually of the gastrointestinal tract.
Gastrointestinal secretions, pancreatic secretions, bile, blood, urine, and meco-
nium cause chemical peritonitis when in contact with the peritoneum. A common
form of chemical peritonitis follows perforation of a peptic ulcer. Bile peritonitis may
result from perforation of the gallbladder or leakage from the bile ducts. Ordinarily,
slow bleeding into the abdominal cavity produces relatively few signs of inflamma-
tion; the addition of bacteria to blood produces suppuration. The sickest postoperative
patients may have tertiary peritonitis that kills 30% to 64% of affected patients. The
syndrome of poorly localized intra-abdominal infection, an altered microbial flora,
progressive organ dysfunction, and high mortality define tertiary peritonitis.
Peritonitis causes abdominal pain, either generalized or localized, depending on
the disease. Appendicitis usually causes localized pain. Perforated peptic ulcer usual-
ly produces generalized abdominal pain. Acute cholecystitis causes right upper quad-
rant pain referred to the right scapula or shoulder. Physical findings of patients with
peritonitis are abdominal tenderness, guarding, and rebound tenderness.
Site of abdominal pain in relation to suspected pathology
Whole abdomen Generation peritonitis and mesenteric infraction
Right upper quadrant Acute cholecystitis
Cholangitis
Hepatitis
Peptic ulceration
Left upper quadrant Peptic ulceration
Pancreatitis
Splenic infarct
Right lower quadrant Appendicitis
Ovarian cyst
Ectopic pregnancy
Pelvic inflammatory disease
Meckels diverticulum
Mesenteric adenitis
Ureteric colic
Rectus sheath haematoma
Right-sided lobar pneumonia
Left lower quadrant Sigmoid diverticular disease
Ovarian cyst
Ectopic pregnancy
Pelvic inflammatory disease
Ureteric colic
Rectus sheath haematoma
Left-sided lobar pneumonia
Radiating pain Peptic ulcer
Pancreatitis
Back
Aortic aneurysm
Dissecting aneurysm
Groin
Ureteric colic
Testicular torsion

CLINICAL DIAGNOSIS
History and Present Illness
Pain is the focal issue in the evaluation of the patient suspected of having an
acute abdomen. The history should therefore characterize and document the pain as
precisely as possible. The duration of the pain is important, but the location, mode of
onset, and character of the pain help in making a diagnosis. Abdominal pain that per-
sists for 6 hours or more with severe intensity increases the likelihood that surgical
operation will be required. If the pain ebbs after a few hours, however, the probability
of surgical disease decreases, but not to zero. Visceral pain caused by distention, in-
flammation, or ischemia usually feels dull and poorly localized in the midabdomen.
Depending on the organ involved, the pain may be felt in the epigastrium, the peri-
umbilical area, or the lower abdomen. Diseases of the kidneys or ureters produce pain
in the flanks. Parietal pain, however, is sharper and better localized. Localized parie-
tal peritonitis can produce pain confined to one of the four quadrants of the abdomen.
In an evaluation of the location of the pain, the concept of referred pain be-
comes important. Subdiaphragmatic disorders can produce pain referred to the shoul-
der.
Blood or pus beneath the left diaphragm can cause left shoulder pain. Biliary
disease can cause referred pain in the right shoulder or the back. Diseases above the
diaphragm such as basal pneumonia can cause pain referred to the neck or shoulder in
the C4 distribution. Upper abdominal pain suggests peptic ulcer, acute
cholecystitis, or pancreatitis. Conversely, ovarian cysts, diverticulitis, and ruptured
tubo-ovarian abscesses produce lower abdominal pain. Small bowel obstruction usu-
ally causes midabdominal pain sometimes referred to the back.
Migratory pain shifting from one place to another can give insight into the di-
agnosis. For example, pain that moves from the epigastrium to the periumbilical area
to the right lower quadrant suggests acute appendicitis. Distention and inflammation
of the appendix produce visceral pain perceived in the periumbilical area. When the
inflammation spreads and produces parietal peritonitis, the pain localizes in the right
lower quadrant of the abdomen. Another example of moving or migratory pain occurs
with perforated duodenal ulcer. The leakage of duodenal contents from a perforated
ulcer causes intense and localized epigastric pain. However, if the leaked duo-
denal content gravitates down the right paracolic gutter into the right lower quadrant,
the patient may also experience right lower quadrant pain. Although the location of
abdominal pain may be helpful, particularly early in the course of the disease, it may
not be typical in all patients. Late in many cases, the pain may become generalized
because of diffuse peritonitis.
The initial manifestations of the acute abdomen and the evolution of the pain
syndrome may give some insight into the cause of the pain. The pain can start sud-
denly or instantly with no prior symptoms. Sudden or explosive onset of severe ab-
dominal pain suggests free perforation of a viscous such as the duodenum or acute
intestinal ischemia from a visceral artery embolus. This type of pain onset can awak-
en patients from sleep or can incapacitate them during work or play. Sudden, general-
ized, excruciating pain suggests an intra-abdominal catastrophe that may produce
shock requiring resuscitation and prompt operation. In other conditions, the pain
comes on with progressively increasing intensity over 1 to 2 hours. This progressive
pain represents the usual manifestation of the diseases that commonly produce the
acute abdomen such as acute cholecystitis, acute pancreatitis, and proximal small
bowel obstruction. Some illness begins with vague general abdominal discomfort that
progress to abdominal pain over a few hours. The pain becomes more intense and
subsequently localizes. This group of illnesses generally includes acute appendicitis,
incarcerated hernia, distal small bowel obstruction, colon obstruction, diverticulitis,
and contained or walled-off visceral perforation. The quality, severity, and periodicity
of the pain may provide clues to the diagnosis. Steady, sharp pain accompanies perfo-
rated duodenal ulcer or perforated appendix.
The early pain of small bowel obstruction is vague and deep seated. This pain
then assumes a crescendo-decrescendo character described as colicky pain.
However, if obstruction produces intestinal infarction, then the pain becomes
dull and constant. The pain of ureteral obstruction is extremely severe and intense.
Patients with kidney stones appear restless, agitated, or hyperactive and tend to
move about, in contrast to patients with peritoneal inflammation, who prefer to lie
quietly and remain undisturbed. Sudden, excruciating pain in the upper abdomen or
the lower chest or interscapular region suggests aortic dissection.
Radiation of pain or referral of pain may help in diagnosis. Radiation of pain
around the right costal margin to the right shoulder and scapula suggests acute chole-
cystitis. Pancreatitis usually produces epigastric pain that may radiate along the costal
margins to the back or straight through to the back. Kidney stones may cause pain ra-
diating to the groin or the perineal area.
Vomiting may occur from the severity of the pain or because of disease in the
gastrointestinal tract. Generally, patients with abdominal pain requiring surgical
treatment experience the pain before vomiting occurs. Vomiting frequently precedes
the pain in patients with medical conditions. Patients with appendicitis usually have
pain and anorexia for a while before vomiting, and patients with gastroenteritis expe-
rience vomiting before abdominal pain. Vomiting frequently occurs in patients with
acute cholecystitis, acute gastritis, acute pancreatitis, and bowel obstruction. Proximal
small bowel obstruction produces more vomiting than distal small bowel obstruction.
Vomiting occurs uncommonly in patients with colon obstruction. Small bowel
obstruction of longer duration can cause feculent vomiting. Obstruction distal to the
ampulla of Vater causes bile-stained vomitus, whereas obstruction proximal to the
ampulla causes clear vomitus. Most patients with acute abdominal pain have no de-
sire to eat. Anorexia may precede the pain of acute appendicitis.
Bowel function, including a history of constipation, diarrhea, or a recent
change in bowel habits, can be important. Watery diarrhea associated with abdominal
pain suggests gastroenteritis. Immunosuppressed patients can contract cytomegalovi-
rus (CMV) infection, salmonellosis, or cryptosporidiosis, which may produce diar-
rhea.
A past history of diarrhea raises the suspicion of inflammatory bowel disease,
either Crohn’s disease or ulcerative colitis. Failure to pass gas or bowel movements
suggests mechanical intestinal obstruction. A history of jaundice, hematemesis,
hematochezia, or hematuria is important in the evaluation of acute abdominal pain.
A careful menstrual history is important in women with abdominal pain. Ovu-
lation can produce significant abdominal pain. Furthermore, abdominal pain in a
woman with a missed menstrual period or irregular menstrual periods can be related
to complications of an undiagnosed pregnancy or an ectopic pregnancy.
The drug history is important in managing patients with acute abdominal pain.
Corticosteroids predispose to gastroduodenal ulceration and the possibility of perfora-
tion.
Corticosteroids also immunosuppress patients and obscure the manifestations
of acute intra-abdominal disease. Furthermore, patients who have taken steroids for
long periods require perioperative steroid supplementation. Patients who take diuret-
ics need evaluation of their fluid and electrolyte status. Anticoagulants can cause in-
tra-abdominal, intestinal, and mesenteric bleeding. The effects of anticoagulants must
be reversed preoperatively. Cocaine can cause abdominal pain. Of course, many pa-
tients developing acute abdominal pain are taking cardiovascular drugs, hormones,
tranquilizers, diuretics, and numerous other classes of agents that must be managed in
the perioperative period.
Past history becomes important, especially regarding prior surgery. For exam-
ple, if a patient has had an appendectomy, cholecystectomy, and so forth, it has a sig-
nificant impact on the differential diagnosis of acute abdominal pain. Past history can
also give clues to the diagnosis of the present illness. In addition, past history\ may
reveal significant co-morbid conditions requiring careful management during the
perioperative period. Systemic illnesses or cardiac or pulmonary disease must be ex-
cluded as possible causes of the abdominal pain syndrome.
Physical Examination
The physical examination usually provides important information that helps in
the diagnosis and management of patients with acute abdominal pain. The patient’s
overall appearance, ability to communicate, habitus, and signs of pain should be not-
ed. Does the patient lie quietly in bed or actively move about? Does the patient lie on
his or her side with knees and hips flexed? Does the patient appear dehydrated with
dry mucous membranes? An apprehensive patient lying quietly in bed, avoiding mo-
tion, and complaining of abdominal pain probably has serious intra-abdominal dis-
ease. The physical examination continues with the evaluation of the vital signs.
Low fever often accompanies diverticulitis, appendicitis, and acute cholecysti-
tis. High fever more often occurs in pneumonia, urinary tract infection, septic cholan-
gitis, or gynecologic infection. Rapid heart rate and hypotension may mean advanced
complicated disease with peritonitis. Peritonitis causes hypovolemia as plasma vol-
ume leaves the intravascular space. The general appearance of the patient and the vi-
tal signs determine the urgency of the diagnostic work-up and implementation of
therapy.
Examination of the abdomen always begins with inspection, with particular at-
tention to scars, hernias, masses, or abdominal wall defects. Hernias incarcerated in
the groin, umbilicus, or incisions of obese patients can be difficult to detect. The ex-
aminer should observe whether the contour of the abdomen appears scaphoid, flat, or
distended. Abdominal distention can mean intestinal obstruction, ileus, or fluid in-
cluding ascites, blood, or bile.
Palpation is a crucial step in evaluating the patient with acute abdominal pain.
For this examination, the patient and the examiner should be positioned comfortably
to conduct gentle palpation. The examiner should assess the patient’s facial expres-
sion for signs of pain or discomfort during the examination. Careful palpation for
tenderness is important. This must be done gently to avoid hurting the patient and
should begin in an area away from the pain site if possible. The finding and the de-
scription of tenderness are the most important steps in palpation of the abdomen of
patients with acute abdominal pain. Localized tenderness over the McBurney point
suggests appendicitis. Tenderness in the right upper quadrant suggests an inflamed
gallbladder. Diverticulitis produces tenderness in the left lower quadrant. Tenderness
throughout the abdomen may reflect diffuse peritonitis.
The detection of increased abdominal muscle tone during palpation is called
guarding. Guarding may be voluntary, involuntary, localized, or generalized. To de-
tect guarding, the examiner should press gently but slowly and firmly on the patient’s
abdomen. Using two hands works best. The detection of muscle spasm denotes
guarding. If, after asking the patient to relax and breathe deeply, the patient’s muscles
relax, it denotes voluntary guarding. If the muscles remain rigid or tense, it indicates
involuntary guarding, which means underlying peritonitis. Guarding may be localized
or generalized. Generalized intense guarding produces the board-like abdomen char-
acteristic of perforated duodenal ulcer. Careful deep palpation can detect abdominal
masses. Acute cholecystitis, acute pancreatitis, abdominal aortic aneurysm, and diver-
ticulitis can produce abdominal masses. Severe guarding can interfere with the detec-
tion of abdominal masses by palpation.
Rebound tenderness is also a sign of peritonitis. To detect rebound tenderness,
the examiner presses deep into the patient’s abdomen with flattened fingers. Sudden
withdrawal of that hand may cause an increase in the abdominal pain, and this symp-
tom indicates peritonitis. Rebound tenderness can be elicited directly over the site of
the abdominal pain. Pressing and releasing the abdomen away from the site of pain
can exacerbate the pain at the original site. Careful, deep palpation can detect ab-
dominal masses. Severe guarding can interfere with the detection of abdominal mass-
es by palpation. In acute cholecystitis, palpation in the right subcostal area during
deep inspiration by the patient may elicit pain. This finding is called a positive Mur-
phy’s sign. This sign can be detected either with the patient sitting or supine.
The gallbladder may be palpated during this maneuver. Direct compression by
the probe may cause pain during ultrasound examination.
Auscultation of the abdomen should give information about the presence or ab-
sence of bowel sounds. A quiet abdomen indicates ileus. Hyperactive bowel sounds
may occur in gastroenteritis. Periods of quiet interrupted by the onset of high-pitched
hyperactive bowel sounds characterize the peristaltic rushes of mechanical small
bowel obstruction. Evaluation of bowel sounds requires careful auscultation for sev-
eral minutes. During auscultation of the abdomen, the examiner can effectively eval-
uate tenderness and guarding further by palpating gently with the stethoscope. The
examiner should also note the presence or absence of bruits in the abdomen.
Percussion is an important part of the abdominal examination. When percus-
sion elicits tenderness, it indicates inflammation and has the same implication as re-
bound tenderness. Hyper-resonance or tympanic sound is heard during percussion of
the abdomen means gaseous distention of the intestine or stomach. Resonance to per-
cussion over the liver suggests free intra-abdominal gas.
Other tests or maneuvers can aid in the assessment of patients with abdominal pain.
Pain during gentle tapping of a fist or deep palpation at the costovertebral angles may
suggest pyelonephritis. An inflamed retrocecal appendix or a psoas abscess can pro-
duce pain or tenderness on motion of the psoas muscle. If passively extending the hip
or actively flexing the hip against resistance causes pain, this is called a positive ili-
opsoas sign. If internal or external rotation of the flexed hip causes pain, it is referred
to as a positive obturator sign.
During the bimanual pelvic examination, the physician should seek evidence of
uterine or adnexal masses or tenderness. Acute salpingitis, tubo-ovarian abscess, or
torsion of an ovarian cyst can cause acute abdominal pain. The speculum examination
allows inspection of the cervix for discharge. Rectal examination should include tests
for occult blood, and the examiner should note the presence of masses or tenderness.
An inflamed pelvic appendix or a pelvic abscess can cause tenderness detected by
rectal examination.
Laboratory Testing
Laboratory investigation of most patients with acute abdominal pain usually in-
cludes a complete blood count. Intra-abdominal inflammation can produce elevation
in the white blood cell count, although this is not always true. One study demonstrat-
ed a poor correlation between the white blood cell count and the degree of intra-
abdominal inflammation in patients operated on because of acute abdominal pain. If a
patient with unequivocal and persistent abdominal pain has a normal or low white
blood cell count, a differential count may disclose a marked left shift, which can be
more significant than finding an elevation in the white blood cell count. If patients
have obvious dehydration, a history of vomiting or diarrhea, or if they have been tak-
ing medications such as diuretics that may influence their serum electrolyte values,
one should measure the concentrations of serum sodium, potassium, blood urea nitro-
gen, creatinine, glucose, chloride, and carbon dioxide. In addition, these laboratory
tests enable one to detect diabetes, renal failure, or other systemic diseases. Meas-
urements of serum amylase and lipase may help in the evaluation of upper abdominal
pain by giving evidence of pancreatitis. Although elevated serum amylase accompa-
nies pancreatitis, other diseases such as perforated duodenal ulcer and small bowel
infarction can also cause increased serum amylase concentrations. Patients with right
upper quadrant abdominal pain should have measurements of serum bilirubin, alka-
line phosphatase, and serum transaminase because of the possibility of obstructive
jaundice or acute hepatitis. Urinalysis can detect evidence of urinary tract infection,
hematuria, proteinuria, or hemoconcentration. Women of childbearing age who have
acute abdominal pain or hypotension should have measurement of the serum or urine
ß-human chorionic gonadotropin concentration.
Diagnostic Imaging
History and physical examination are the most important and useful steps in the
evaluation of patients with abdominal pain. However, advances in imaging of the ab-
domen have improved the diagnostic accuracy and the overall management of pa-
tients experiencing acute abdominal pain. Before the widespread availability of ultra-
sonography and computed tomography (CT), surgeons performed a careful history
and physical examination, obtained laboratory tests, and reviewed plain films of the
abdomen and chest. With that information, a decision to operate or not was made
usually on the basis that the patient probably had some disease best treated surgically.
The laparotomy was considered diagnostic as well as therapeutic. Historically, before
modern imaging tests, as many as 20% of patients operated on for acute appendicitis
did not have it.
Plain films still have usefulness in several circumstances. A radiograph cen-
tered on the diaphragm detects pneumoperitoneum better than other radiographic
techniques. An upright chest radiograph can detect under the diaphragm as little as 1
mL of air injected into the peritoneal cavity. For the occasional patient who cannot
stand up, a lateral decubitus radiograph of the abdomen can also detect pneumoperi-
toneum effectively. A cross-table lateral radiograph with the patient in the left lateral
position can detect 5 to 10 mL of gas under the lateral abdominal wall. Free air in the
peritoneal cavity indicates a perforation of the gastrointestinal tract.
Perforated duodenal ulcers usually allow small amounts of air to escape into
the peritoneal cavity. About 75% of patients with perforated duodenal ulcers have
radiographically detectable pneumoperitoneum. Perforations of the stomach and the
colon can cause extensive pneumoperitoneum. The amount of pneumoperitoneum can
also depend on the duration of the leak from the perforation. Plain films of the abdo-
men can show extensive pneumoperitoneum. If the film defines both the serous and
the related mucosal walls of the bowel, it means free air is at that serous surface. In
addition, free air can delineate the falciform ligament on plain abdominal films.
An extensive hydropneumoperitoneum appears as an extremely long air-fluid
level on an upright film. A supine film can show a large air collection beneath the ab-
dominal wall that does not conform to any bowel loop. Plain films show abnormal
calcifications. About 10% of gallstones and 90% of kidney stones contain sufficient
calcium to be radiopaque. Appendicoliths can calcify and appear radio-graphically in
5% of patients with appendicitis. Pancreatic calcifications characteristic of chronic
pancreatitis show on plain films, and vascular calcifications can aid in the evaluation
of abdominal aortic aneurysms, visceral artery aneurysms, and atherosclerosis of vis-
ceral vessels.
Supine and erect plain films of the abdomen show gastric outlet obstruction;
proximal, mid, and distal small bowel obstruction; and colon obstruction. The charac-
teristics of small bowel obstruction include multiple air-fluid levels in dilated, cen-
trally located loops of intestine with visible valvulae conniventes and an absence or
paucity of colon gas. Obstructed colon usually appears as peripherally located dis-
tended bowel with haustral markings. If the ileocecal valve is incompetent, colon ob-
struction will cause distention of the distal small bowel.
Some patients with an acute abdomen have plain abdominal films that show a
bowel pattern suggesting mechanical obstruction when no obstruction exists. Paralyt-
ic ileus can produce distended bowel with multiple air-fluid levels. Plain radiographs
show paralytic ileus resulting from intra-abdominal or retroperitoneal inflammation.
The radiographic findings of paralytic ileus include excessive distention and
fluid with gas distributed from stomach to rectum. Plain films of the abdomen may
also detect gas in the portal or mesenteric venous system, intramural gas in the gastro-
intestinal tract, gas in the biliary ducts or gallbladder, and gas in the urinary tract or
retroperitoneal areas. When plain films show gas in the portal or mesenteric veins, it
usually means advanced and serious disease. CT can show small amounts of gas in
veins and also may delineate the cause of the abnormality. If the patient’s history
suggests renal colic, an intravenous pyelogram may confirm the diagnosis of a kidney
stone.
CT scanning has provided definite improvements in diagnostic accuracy in
evaluating patients with abdominal pain and also reveals anatomic and pathologic de-
tail not possible with plain radiographs. Therefore, CT and ultrasonography now oc-
cupy the central imaging role in this situation. Although history and physical exami-
nation provide essential information in evaluating patients with the acute abdomen,
modern imaging techniques, including ultrasound and CT, can lead to an anatomic
diagnosis in most cases. One prospective study of 40 patients with acute abdominal
pain revealed that CT significantly improved the diagnostic accuracy of clinical eval-
uation plus plain radiographs. Clinical examination and plain films were 50% correct,
but CT scanning was 95% correct. CT scans accurately detected the specific anatomic
lesion in 57.5% of cases compared with 17.5% with clinical examination and plain
films. This study included no patients with appendicitis, the most common cause of
the acute abdomen, because the surgeons did not refer any cases of suspected appen-
dicitis for inclusion in the study. However, other investigators evaluated the role of
CT in the diagnosis of acute appendicitis in 100 consecutive patients studied prospec-
tively. The CT interpretation had 98% sensitivity, 98% specificity, 98% positive
predictive value, 98% negative predictive value, and 98% overall accuracy for diag-
nosing or ruling out appendicitis. According to the authors’ calculations, these 100
CT scans produced a net savings of $44,731 in the care of the study patients because
of improved diagnostic accuracy. CT scans can add important value to the diagnosis
of acute appendicitis. However, focused specialists using excellent equipment in an
environment of inquiry conducted this study, and the results may not be reproducible
in all hospitals. Other workers questioned the value of CT scanning in the diagnosis
of acute appendicitis.
Ultrasonography is useful for patients with acute abdominal pain because it
provides rapid, safe, low-cost evaluation of the liver, gallbladder, bile ducts, spleen,
pancreas, appendix, kidneys, ovaries, adnexa, and uterus. Transabdominal and in-
travaginal ultrasonography can aid in the evaluation of the ovaries, adnexa, and uter-
us. Ultrasonography also detects and characterizes the distribution of intra-abdominal
fluid. Color-Doppler ultrasonography allows evaluation of the intra-abdominal and
retroperitoneal blood vessels. Aortic and visceral artery aneurysms, venous throm-
bosis, arteriovenous fistulas, and vascular anomalies can be evaluated with ultrasound
. Unfortunately, patients with acute abdominal disease frequently have excessive ab-
dominal gas that interferes with careful and detailed sonographic evaluation of the
abdominal organs, but overlying gas, bone, and Appendicitis, the most common cause
of the acute surgical abdomen in North America, can be difficult to diagnose. Plain
films and barium enema studies generally add little to the diagnosis. However, in pa-
tients with uncomplicated appendicitis, ultrasonography can detect appendicoliths,
demonstrate a distended or thick-walled appendix, or detect periappendiceal and pe-
ricecal inflammatory changes. Ultrasound is reliable and sensitive for the detection of
appendicoliths and the demonstration of an abnormally distended or thick-walled ap-
pendix. Conversely, CT detects acute appendicitis and defines the changes of compli-
cated appendicitis. CT scans can enable the examiner to differentiate diffuse per-
iappendiceal inflammation from an abscess. In addition, CT scans detect many of the
diseases included in the differential diagnosis of acute appendicitis. CT detects blood
and other fluids in the abdominal cavity. Intramural intestinal hemorrhage is readily
detected by CT. CT scans accurately reveal mesenteric venous thrombosis. CT scans
can delineate diverticulitis and its complications, such as abscess and even pyelo-
phlebitis. CT is especially helpful in evaluating pancreatitis by revealing minimal
edema, extensive edema, fluid collections, hemorrhage, and necrosis; in addition, it
effectively evaluates the complications of pancreatitis such as abscess or pseudocyst.
CT scans show the signs of advanced peritonitis. With this technique, one can also
evaluate the complications of colon perforation and of small bowel disease such as
intussusception. Although history and physical examination provide essential infor-
mation in evaluating patients with the acute abdomen, modern imaging techniques,
including ultrasound and CT, can lead to an anatomic diagnosis in the majority of
cases.

CLINICAL MANAGEMENT
Differential Diagnosis
Information from the patient’s history, physical examination, laboratory tests, and
imaging studies usually permits a diagnosis, but uncertainty can still remain. Because
appendicitis is a common disease, it must remain in the differential diagnosis of any
patient with persistent abdominal pain, particularly right lower quadrant pain. The di-
agnosis of appendicitis is easy to miss, and perforation substantially increases mor-
bidity and mortality from the disease. Delay in diagnosis is the principal reason for
unfavorable outcomes in appendicitis. Appendicitis is the most common cause of the
acute abdomen in childhood; however, in older patients, acute cholecystitis, bowel
obstruction, cancer, and acute vascular conditions assume importance in addition to
appendicitis. The differential diagnosis in young women can be difficult because they
can have salpingitis, dysmenorrhea, ovarian lesions, and urinary tract infections as
well as complications of pregnancy, which can confound the evaluation of abdominal
pain. Of course, the medical causes of abdominal pain must be considered, but pa-
tients with medical disease generally lack specific localized tenderness and guarding.
The other problem is that about one third of patients who present with acute ab-
dominal pain have nonspecific abdominal pain, and no clear diagnosis is ever estab-
lished.
Decision to Operate
These difficulties notwithstanding, the surgeon must make a decision to operate
or not. Certain indications for surgical treatment exist. For example, definite signs of
peritonitis such as tenderness, guarding, and rebound tenderness support the decision
to operate. Likewise, severe or increasing localized abdominal tenderness should
prompt an operation. Patients with abdominal pain and signs of sepsis that cannot be
explained by any other finding should undergo operation.
Those patients suspected of having acute intestinal ischemia should be operated
on after complete evaluation. Certain radiographic findings confidently predict the
need for operation. These findings include pneumoperitoneum and radiologic evi-
dence of gastrointestinal perforation. Patients who present with abdominal pain and
free intra-abdominal gas seen on radiograph are warrant for operation with limited
exceptions. Observation with serial examinations may be appropriate for a patient
with free gas after a colonoscopy. Intra-abdominal gas can persist for a day or two
following celiotomy. Imaging tests can reveal signs of vascular occlusion requiring
operation.
After careful examination and evaluation, diagnostic uncertainty can remain.
Some patients may have equivocal physical findings. When this occurs and the diag-
nosis is unclear and the patient’s wellness is unclear, it may be advisable to defer op-
eration and to re-examine the patient carefully after several ours. This is best done in
a short-stay unit in the hospital, in a special unit in the emergency department, or if
necessary, by regular hospital admission. In a period of hours, vague pain with mini-
mal physical findings may proceed to definite localized pain with tenderness, guard-
ing, and rebound tenderness; if that occurs, operation should follow. After several
hours, the patient’s symptoms and signs may also resolve. When that happens, the pa-
tient can be dismissed, although the patient should have a follow-up appointment
scheduled within a day or so to permit re-examination to be certain that an important
diagnosis was not missed. Certain patients are difficult to evaluate because of special
characteristics. For example, patients who are neurologically impaired as a result of a
stroke or a spinal cord injury may be difficult to evaluate. Patients who are under the
influence of drugs or alcohol may require special or subsequent examination. Patients
who take steroids or are otherwise immunosuppressed deserve special mention be-
cause steroids and immunosuppressant mask the intensity of abdominal pain and the
physical findings of severe, life-threatening intra-abdominal disease. Patients in this
category who have persistent, unequivocal abdominal pain and even minimal findings
should be considered for surgical operation.
Some patients with clear findings of the acute abdomen may be treated without
surgical operation. For example, patients with perforated duodenal ulcer who seek at-
tention late in the course of their disease after they have been sick for several days
may be treated best by careful supportive care including nasogastric suction, intrave-
nous fluids, and pain relief. Certain patients with empyema of the gallbladder, espe-
cially those with other serious concomitant illnesses, can be treated by percutaneous
drainage of the infected gallbladder and careful supportive care rather than with chol-
ecystectomy. Some patients who have acute appendicitis may not seek attention until
several days into the course of the illness, at which time they may have walled off the
perforation and may have an appendiceal abscess. These patients have right lower
quadrant pain, tenderness, and perhaps guarding, but if they have an appendiceal ab-
scess, this is usually best managed by percutaneous drainage of the abscess and
avoidance of appendectomy at that time. Acute diverticulitis is usually best managed
nonsurgically. If a patient with diverticular disease has a diverticular abscess, percu-
taneous drainage and supportive care will suffice, and the diverticular disease may be
removed electively later. Most patients with acute pancreatitis should be managed
without operation unless they have a specific indication for operation. Indications for
operation include the development of an abscess.
Preoperative Preparation
In a stable, otherwise healthy patient, preoperative procedures should include
insertion of a nasogastric tube, establishment of access for intravenous fluid admin-
istration, insertion of a Foley catheter in the urinary bladder to record urinary output,
and pain relief. Once a decision has been made to perform an operation, the patient
should be given a narcotic or other suitable analgesic unless he or she is being taken
immediately to the operating room. Most patients with an acute abdomen requiring an
operation have conditions in which infection is either present or likely. For that rea-
son, antibiotics should be administered preoperatively.
Unstable patients must have more careful evaluation and resuscitation before
one proceeds to surgical intervention. These patients also require a nasogastric tube, a
Foley catheter, intravenous fluids, and antibiotics, but if patients have hypotension,
tachycardia, or oliguria and evidence of dehydration, they will need a period of sup-
portive care and intravenous fluids before they undergo general anesthesia and opera-
tion. After evaluation of their fluid and electrolyte status and degree of dehydration,
these patients should receive sufficient quantities of intravenous fluid to establish uri-
nary output of 0.5 mL/kg per hour. Preoperative blood pressure should be greater than
100 mm Hg systolic, and the pulse should fall to less than 100 beats/min. Patients tak-
ing steroids should have supplemental doses administered before and after the opera-
tion, including their maintenance dose. Diabetic patients should have attention to con-
trol their hyperglycemia and acid-base balance. Cardiovascular function must be
monitored in patients with a history of heart disease, and they should have preopera-
tive and postoperative management of their current drugs.
Patients with an acute abdomen should be operated on when they become he-
modynamically stable and have satisfactory urinary output. Patients who are hypoka-
lemic should have potassium infusion after the establishment of urine flow.
Operation
After concluding that a patient with abdominal pain needs an operation, the
surgeon must plan the surgical approach. General inhalation anesthesia administered
through an endotracheal tube should be used in most cases. Then the surgeon must
choose whether to employ laparotomy or laparoscopy. That choice depends on the
surgeon’s experience and the probable diagnosis. Some factors such as multiple prior
laparotomies, hemodynamic instability, or advanced abdominal distention preclude
laparoscopy. For open operation, the surgeon must choose an incision. In cases of
probable appendicitis, a right lower quadrant muscle-splitting incision works well. If
acute cholecystitis is nearly certain, a right subcostal incision should be used. An in-
carcerated groin hernia should be approached through a groin incision.
When the diagnosis is uncertain, a midline incision works well. The use of lap-
aroscopy has become more common and more effective in the management of acute
abdominal pain. In 1975, Sugarbaker and associates demonstrated the utility of lapa-
roscopy in the management of patients with acute abdominal pain. In this study, 56
patients required hospitalization because of acute abdominal pain. Twenty-seven of
these patients had a “definite” clinical diagnosis and underwent laparotomy. Six, or
22%, of these patients had a negative laparotomy, whereas 21 patients had diseases
managed best by laparotomy. Twenty-nine patients without an “exact” diagnosis un-
derwent laparoscopy. Eighteen of those patients had, at laparoscopy, a definitive di-
agnosis of a disease that did not require laparotomy, and 11 patients required laparot-
omy after laparoscopy. Laparoscopy required 20 minutes on average and incurred no
complications.
The patients in the laparoscopy group had shorter hospital stays and lower hos-
pital charges. Since 1975, of course, laparoscopic surgical skills and technology have
improved dramatically, and the usefulness of laparoscopy in managing patients with
acute abdominal pain is generally recognized and accepted.
Laparoscopy has become an important technique in the management of patients
with acute abdominal pain. In a study of 255 patients with acute abdomen, Laparos-
copy proved helpful. In this set of patients, laparoscopy provided a definitive diagno-
sis in 93%, and the remaining 7% required laparotomy for diagnosis. The treatment of
the acute abdominal pain was exclusively laparoscopic in 73% of the patients, where-
as 23% were treated by conventional surgery. Four percent had a combined procedure
of conventional surgery assisted by laparoscopy. Eight patients died from the natural
course of their disease, five from nonresected intestinal infarctions, and three from
disseminated peritoneal malignant disease. Excluding these patients, the operative
mortality was 2%, that is, 5 of 247 cases. One 80-year-old patient had a fatal stroke,
an 89-year-old patient who was operated on for a large intestinal obstruction had mul-
tiple organ failure, an 82-year-old patient had an intraoperative complication resulting
in massive blood loss and died on the 48th postoperative day, and an 89-year-old pa-
tient died of a thoracic empyema.
More recently, several authors reported favorable experiences using laparosco-
py in the diagnosis and treatment of patients with acute abdominal pain.
The diagnostic accuracy of laparoscopy varied from 93% to 100%. Laparoscopic
techniques accomplished definitive treatment of the underlying disease in 44% to
73% of cases. From 10% to 38% of patients required laparotomy for definitive treat-
ment. In 20% to 38% of patients, laparoscopy revealed either no abnormality or dis-
covered a disease requiring no surgery for proper treatment. The morbidity rates
ranged from 0 to 20%, and the mortality rates ranged from 0 to 5%.
Diagnostic and therapeutic laparoscopic techniques have an important place in the
management of patients with acute abdominal pain. The diagnostic accuracy spares
many patients an unnecessary laparotomy and also allows definitive laparoscopic
therapy that prevents additional patients from undergoing unnecessary laparotomy.
Evidence suggests that diagnostic laparoscopy reduces the cost of managing patients
with acute abdominal pain. Whether diagnostic laparoscopy and therapeutic laparos-
copy reduce the cost remains unclear. Most patients with acute abdominal pain should
be suitable candidates for laparoscopy. Laparoscopy should be avoided in hemody-
namically unstable patients and in patients with extensive gaseous distention of the
abdomen. Whether pregnant women with the acute abdomen should undergo laparos-
copy is a practical question. One study suggested that laparoscopy in this setting was
safe and effective.
Outcomes
It is difficult to know the mortality rate for patients with the acute abdomen. A
study from the United Kingdom of patients hospitalized with abdominal pain revealed
a mortality rate for all patients of 3.0% and an operative mortality of 7.7%. Another
study of 300 consecutive patients undergoing laparotomy within 6 hours of consulta-
tion for gastrointestinal perforation, intestinal infarction, or hemorrhage demonstrated
a mortality rate of 20%. This study included mostly critically ill patients. Other stud-
ies revealed a 16% to 40% mortality rate for emergency in older patients.

ACUTE VISCERAL ISCHEMIA


Although patients experiencing acute visceral ischemia account for a small per-
centage of the population seeking medical attention for acute abdominal pain, this
topic deserves special attention because of extreme difficulty in establishing a correct
and timely diagnosis and because the condition has a high mortality rate. Acute
arterial disease may be either occlusive or nonocclusive, and venous disease can also
produce the syndrome. Arterial occlusion may be either embolic or thrombotic.
Generally, acute superior mesenteric artery embolism causes a sudden onset of
extremely severe abdominal pain. This ischemic pain persists for a long time before
the development of intestinal necrosis. Because the pain results from ischemia and
not from peritonitis, these patients have no abdominal tenderness, guarding, or re-
bound. Therefore, abdominal pain out of proportion to the abdominal physical find-
ings should raise a question about this diagnosis. Because ischemia stops bowel mo-
tility promptly, the abdomen may be quiet to auscultation, depending on the amount
of ischemic bowel. The heart is the most likely source of a superior mesenteric artery
embolus. Therefore, any patient with cardiac arrhythmias, particularly atrial fibrilla-
tion, a known mural thrombus, or a recent myocardial infarction who develops acute
abdominal pain should have acute superior mesenteric artery embolism high in the
differential diagnosis. Patients with atherosclerosis can develop thrombosis at a supe-
rior mesenteric artery stenosis. Patients with acute visceral ischemia usually have
marked leukocytosis and acidosis. Because cardiovascular disease is important in the
development of acute visceral ischemia, most patients with that condition are persons
who are middle aged or older.
Conversely, venous thrombosis can cause visceral ischemia, and those patients
can be younger. Birth control pills have been implicated in venous thrombosis in
young women. Patients suspected of having acute visceral ischemia should undergo
arteriography. Although duplex scanning can provide information about the visceral
circulation, arteriography provides better images for planning arterial reconstruction
or embolectomy. However, arteriography may not help in venous disease. CT scans
or magnetic resonance imaging studies can reveal and delineate clots in visceral
veins. Most patients with acute visceral ischemia should undergo laparotomy.
Some patients develop visceral ischemia because of poor perfusion resulting
from decreased cardiac output. Patients usually develop nonocclusive visceral ische-
mia while they are in the hospital, particularly in an intensive care setting. Improving
cardiac output to restore intestinal perfusion is an important step in managing this
problem. Arteriography may be required for complete evaluation and allows direct
infusion of vasodilators for therapy.
Basic literature:
9. Oxford Textbook of Surgery (3-Volume Set) 2nd edition (January 15, 2000):
by Peter J. Morris (Editor), William C. Wood (Editor) By Oxford Press
10. Sabiston Textbook of Surgery 17th edition by Courtney M. Townsend Jr.,
Kenneth L. Mattox, B. Mark, MD Evers, Kenneth L., MD Mattox, Courtney Town-
send, Daniel Beauchamp, B. Mark Evers, Kenneth Mattox W.B. Saunders Company
(June, 2004)
11. Schwartz´s Principles of Surgery 8th Edition F.Charles Brunicardi. Copyright
©2007 The McGraw-Hill Companies.
12. Hospital surgery/ Edited by L.Kovalchuk et al.- Ternopil: Ukrmedknyha,
2004.- 472 p.

Additional literature:
1. Jones PF: Suspected acute appendicitis: Trends in management over 30 years. Br J
Surg 88:1570, 200

2. Marco CA et al: Abdominal pain in geriatric emergency patients: Variables associ-


ated with adverse outcome. Acad Emerg Med 5:1163, 1998

3. Scott HJ, Rosin RD: The influence of diagnostic and therapeutic laparoscopy on
patients presenting with an acute abdomen. J R Soc Med 86:699, 1993
4. Tait IS et al: Do patients with abdominal pain wait unduly long for analgesia? J R
Coll Surg Edinb 44:181, 1999

5. Taourel P et al: Acute abdomen of unknown origin: Impact of CT on diagnosis and


management. Gastrointest Radiol 17:287, 1992

6. Weyant MJ et al: Interpretation of computed tomography does not correlate with


laboratory or pathologic findings in surgically confirmed acute appendicitis. Sur-
gery 128:145, 2000

7. Yu J et al: Helical CT evaluation of acute right lower quadrant pain: Part I, com-
mon mimics of appendicitis. AJR Am J Roentgenol 184:1136, 2005

Tests for initial level of knowledge, keys for tests:


1. A white, obese 40-year-old mother of five children gives a history of repeated
episodes of right upper quadrant abdominal pain brought about by the ingestion of
fatty foods, and relieved by the administration of anticholinergic medications. The
pain is colicky, radiates to the right shoulder and around to the back, and is accom-
panied by nausea and occasional vomiting. Physical examination is unremarkable.
What is it?
A. Acute appendicitis
B. Caecal carcinoma
C. Hematoma of the rectus sheath
D. Torsion of an ovarian cyst
E. Gallstones, with biliary colic

2. A 48-year-old woman develops pain of the right lower quadrant while playing
tennis. The pain progresses and the patient presents to the emergency room later that
day with a low-grade fever, a white blood count of 13,000, and complaints of anorex-
ia and nausea as well as persistent, sharp pain of the right lower quadrant. On exami-
nation she is tender in the right lower quadrant with muscular spasm and there is a
suggestion of a mass effect. An ultrasound is ordered and shows an apparent mass in
the abdominal wall. Which of the following is the most likely diagnosis?
A. Acute appendicitis
B. Caecal carcinoma
C. Hematoma of the rectus sheath
D. Torsion of an ovarian cyst
E. Cholecystitis

3. A 54-year-old man has had colicky abdominal pain and protracted vomiting for
several days. He has developed progressive moderate abdominal distention,
and has not had a bowel movement or passed any gas for 5 days. He has high-
pitched, loud bowel sounds that coincide with the colicky pain, and x-rays
show distended loops of small bowel and air-fluid levels. Five years ago he had
an exploratory laparotomy for a gunshot wound of the abdomen. What is it?
A. Acute appendicitis
B. Cecal carcinoma
C. Hematoma of the rectus sheath
D. Torsion of an ovarian cyst
E. Mechanical intestinal obstruction, caused by adhesion

4. A 54-year-old man has had colicky abdominal pain and protracted vomiting for
several days. He has developed progressive moderate abdominal distention,
and has not had a bowel movement or passed any gas for 5 days. He has high-
pitched, loud bowel sounds that coincide with the colicky pain, and x-rays
show distended loops of small bowel and air-fluid levels. Five years ago he had
an exploratory laparotomy for a gunshot wound of the abdomen. Six hours
after being hospitalized and placed on NG suction and IV fluids, he develops
fever, leukocytosis, abdominal tenderness, and rebound tenderness. What is it?
A. Acute appendicitis
B. Cecal carcinoma
C. Strangulated obstruction
D. Torsion of an ovarian cyst
E. Acute pancreatitis

5. A 22-year-old man develops anorexia followed by vague periumbilical pain


that several hours later becomes sharp, severe, constant, and well localized to
the right lower quadrant of the abdomen. He has abdominal tenderness, guard-
ing, and rebound to the right and below the umbilicus, temperature of 37,8°C,
and white blood cell (WBC) count is 12,500, with neutrophilia and immature
forms. What is it?
A. Acute appendicitis
B. Acute diverticulitis
C. Strangulated obstruction
D. Acute cholecystitis
E. Acute pancreatitis

6. A 27-year-old man is recovering from an appendectomy for gangrenous acute


appendicitis with perforation and periappendicular abscess. He has been receiv-
ing Clindamycin and Tobramycin for 7 days. Eight hours ago he developed wa-
tery diarrhea, crampy abdominal pain, fever, and leukocytosis. What is it?
A. Obstructive jaundice
B. Cancer of sigmoid
C. Strangulated obstruction
D. Torsion of an ovarian cyst
E. Pseudomembranous colitis

7. A 59-year-old woman has a history of three prior episodes of left lower


quadrant abdominal pain for which she was briefly hospitalized and treated
with antibiotics. She began to feel discomfort 12 hours ago, and now she has
constant left lower quadrant pain, tenderness, and a vaguely palpable mass.
She has fever and leukocytosis. What is it?
A. Acute appendicitis
B. Cecal carcinoma
C. Strangulated obstruction
D. Acute diverticulitis
E. Torsion of an ovarian cyst

8. A 24-year-old woman develops moderate, generalized abdominal pain of sudden


onset, and shortly thereafter faints. At the time of evaluation in the ER she is pale,
tachycardic, and hypotensive. The abdomen is mildly distended and tender, and
she has hemoglobin of 70 g/l. There is no history of trauma. On inquiring as to
whether she might be pregnant, she denies the possibility because she has been on
birth control pills since she was 14, and has never missed taking them. What is it?
A. Acute cholecystitis
B. Obstructive jaundice
C. Bleeding from a ruptured hepatic adenoma
D. Torsion of an ovarian cyst
E. Acute pancreatitis

9. A 52-year-old man has right flank colicky pain of sudden onset that radiates
to the inner thigh and scrotum. There is microscopic hematuria. What is it?
A. Acute appendicitis
B. Ureteral colic
C. Strangulated obstruction
D. Torsion of an ovarian cyst
E. Rupture of abdominal aorta

10. An 82-year-old man develops severe abdominal distension, nausea, vomiting, and
colicky abdominal pain. He has not passed any gas or stool for the past 12 hours. He
has a tympanic abdomen with hyperactive bowel sounds. X-ray shows distended
loops of small and large bowel, and a very large gas shadow that is located in the
right upper quadrant and tapers toward the left lower quadrant with the shape of a
parrot's beak. What is it?
A. Acute appendicitis
B. Cecal carcinoma
C. Volvulus of the sigmoid
D. Torsion of an ovarian cyst
E. Gallstones, with biliary colic

Keys for tests

1 2 3 4 5 6 7 8 9 10
E C E C A E D C B C
Tests for final level of knowledge, keys for tests:
1. A 44-year-old alcoholic man presents with severe epigastric pain that began shortly
after a heavy bout of alcoholic intake, and reached maximum intensity over a period
of 2 hours. The pain is constant, radiates straight through to the back, and is accom-
panied by nausea, vomiting, and retching. He had a similar episode 2 years ago, for
which he required hospitalization. The best tactics option in this patient would be:
A. Peritoneal lavage
B. X-ray abdominal cavity
C. Endoscopic retrograde cholangiopancreatography
D. Sonogram, elective cholecystectomy will follow
E. Diagnostic laparotomy

2. . A 52-year-old obese mother of three children presents with progressive jaundice,


which she first noticed 6 weeks ago. She has a total bilirubin of 107, with 98 direct
and 9 indirect, and minimally elevated SGOT. The alkaline phosphatase is about six
times the upper limit of normal. She gives a history of multiple episodes of colicky
right upper quadrant abdominal pain, brought about by ingestion of fatty food. The
initial maneuver should be to:
A. Peritoneal lavage
B. X-ray examination of the abdominal cavity
C. Endoscopic retrograde cholangiopancreatography
D. Sonogram, elective cholecystectomy will follow
E. Diagnostic laparotomy

3. A 44-year-old man shows up in the ER at 11 PM with exquisite perianal


pain. He cannot sit down, reports that bowel movements are very painful,
and has been having chills and fever. Physical examination shows a tender,
red, fluctuant mass between the anus and the ischial tuberosity.
A. Acute appendicitis
B. Ureteral colic
C. Strangulated obstruction
D. Torsion of an ovarian cyst
E. Ischiorectal abscess

4. A 54-year-old obese man gives a history of burning retrosternal pain and


heartburn that is brought about by bending over, wearing tight clothing, or ly-
ing flat in bed at night. He gets symptomatic relief from antacids but has nev-
er been formally treated. The problem has been present for many years, and
seems to be progressing. What is it?
A. Gastro esophageal reflux disease
B. Gastric peptic ulcer
C. Acute cholecystitis
D. Acute mesenteric ischemia
E. Acute pancreatitis
5. A 66-year-old man has an upper Gl endoscopy done as an outpatient to
check on the progress of medical therapy for gastric ulcer. Six hours after the
procedure, he returns complaining of severe, constant retrosternal pain that
began shortly after he went home. He looks prostrate and very ill, is diapho-
retic, has a fever of 38°C, and a respiratory rate of 30. There is a hint of sub-
cutaneous emphysema at the base of the neck. What is correct diagnosis?
A. Acute mesenteric thrombosis
B. Right-side lobar pneumonia
C. Perforated duodenum ulcer
D. Instrumental perforation of the esophagus
E. Acute pancreatitis

6. A 56-year-old man presents with progressive jaundice, which he first no-


ticed 6 weeks ago. He has a total bilirubin of 225, with 165 direct and 60 indi-
rect, and minimally elevated SGOT. The alkaline phosphatase is about eight
times the upper limit of normal. He has lost 5 kg over the past 2 months, and
has a persistent, nagging mild pain deep into his epigastrium and in the upper
back. His sister died at age 44 from a cancer of the pancreas. A sonogram
shows dilated intrahepatic ducts, dilated extrahepatic ducts, and a much dis-
tended, thin-walled gallbladder.
A. Cancer of the head of the pancreas
B. Acute edematous pancreatitis
C. Obstructive jaundice
D. Acute ascending cholangitis
E. Hepatocellular jaundice

7. A 73-year-old obese mother of six children has severe right upper quadrant
abdominal pain that began 3 days ago. The pain was colicky at first but has
been constant for the past 2.5 days. She has tenderness to deep palpation, mus-
cle guarding, and rebound in the right upper quadrant. She has temperature
spikes of 38,4 and 38,6°C, with chills. Her WBC count is 22,000, with a shift
to the left. Her bilirubin is 39µmol/l, and she has an alkaline phosphatase of
2,000 (about 20 times more than normal).
A. Acute pancreatitis
B. Perforated duodenum ulcer
C. Torsion of an ovarian cyst
D. Hepatocellular jaundice
E. Acute ascending cholangitis

8. A 28-year-old man is brought to the office by his mother. Beginning 4 months


ago he has had three operations, done elsewhere, for a perianal fistula, but after
each one the area has not healed, and in fact the surgical wounds have become
bigger. He now has multiple non healing ulcers, fissures, and fistulas all around
the anus, with purulent discharge. There are no palpable masses. What is it?
A. Crohn’s disease
B. Ureteral colic
C. Strangulated obstruction
D. Torsion of an ovarian cyst
E. Ischiorectal abscess

9. A 33-year-old alcoholic man shows up in the ER with epigastric and


midabdominal pain that began 12 hours ago shortly after the ingestion of a
large meal. The pain is constant and very severe, and radiates straight through
to the back. He vomited twice early on, but since then has continued to have
retching. He has tenderness and some muscle guarding in the upper abdomen,
is afebrile, and has mild tachycardia. Serum amylase is 6,5, and his hematocrit
is 52%. What is it?

A. Obstructive jaundice
B. Cancer of sigmoid
C. Strangulated obstruction
D. Torsion of an ovarian cyst
E. Acute edematous pancreatitis

10. A 23-year-old woman describes exquisite pain with defecation and blood
streaks on the outside of the stools. Because of the pain she avoids having
bowel movements and when she finally does, the stools are hard and even
more painful. Physical examination cannot be done, as she refuses to allow
anyone to even "spread her cheeks" to look at the anus for fear of precipitating
the pain.
A. Crohn’s disease
B. Ureteral colic
C. Strangulated obstruction
D. Anal fissure
E. Ischiorectal abscess

Keys for tests


1 2 3 4 5 6 7 8 9 10
D C E A D A E A E D
Tasks for the final level of knowledge.

1. A 40-year-old obese mother of five children presents with progressive jaundice,


which she first noticed 4 weeks ago. She has a total bilirubin of 22, with 16 direct and
6 indirect, and minimally elevated SGOT. The alkaline phosphatase is about six times
the upper limit of normal. She gives a history of multiple episodes of colicky right
upper quadrant abdominal pain, brought about by ingestion of fatty food. What is it?

The answer is obstructive jaundice caused by stones.

2. A 66-year-old man presents with progressive jaundice, which he first noticed 6


weeks ago. He has total bilirubin of 22, with 16 direct and 6 indirect, and minimally
elevated SGOT. The alkaline phosphatase is about six times the upper limit of nor-
mal. He has lost 10 pounds over the past 2 months, but is otherwise asymptomatic. A
sonogram shows dilated intrahepatic ducts, dilated extrahepatic ducts, and a much
distended, thin-walled gallbladder. What is it?

The answer is malignant obstructive jaundice

3. A 44-year-old alcoholic man presents with severe epigastric pain that began shortly
after a heavy bout of alcoholic intake, and reached maximum intensity over a period
of 2 hours. The pain is constant, radiates straight through to the back, and is accom-
panied by nausea, vomiting, and retching. He had a similar episode 2 years ago, for
which he required hospitalization. What is it?

The answer is acute pancreatitis

4. A 73-year-old obese mother of six children has severe right upper quadrant ab-
dominal pain that began 3 days ago. The pain was colicky at first but has been con-
stant for the past 2.5 days. She has tenderness to deep palpation, muscle guarding,
and rebound in the right upper quadrant. She has temperature spikes of 38,5and
38,8°C, with chills. Her WBC count is 22,000, with a shift to the left. Her bilirubin
is 105 µmol/land she has an alkaline phosphatase of 2,000 (about 20 times the nor-
mal) What is it?

The answer is acute ascending cholangitis.

5. A 54-year-old obese man gives a history of burning retrosternal pain and


heartburn that is brought about by bending over, wearing tight clothing, or
lying flat in bed at night. He gets symptomatic relief from antacids but has
never been formally treated. The problem has been present for many years,
and seems to be progressing. What is it?
The answer is classic for gastro esophageal reflux disease
Materials for the independent teaching of students
Main tasks Notes (instructions)
Repeat:
1. Anatomy of organs of ab-
dominal cavity, structure of the liver.
Anatomy and features of blood circu-
lation of the spleen.
2. Physiology peritoneum of ab-
dominal cavity and pelvic peritoneum -To represent the methods of diagnostics
3. Pathogenesis of abdominal dis- of diseases of abdominal cavity as a table
ease and innervations organs of ab- -To make the flow diagram of mecha-
dominal cavity. nisms pains in abdomen
4. Morphological changes in the
organs of abdominal cavity at disease
5. Clinical displays and methods
of diagnostics of peritonitis
Study guide #5

“Syndromes of intestinal dysphagia and violations of defecation, acute intestinal


obstruction, inflammatory diseases of colon.”

Overview.
The description of patients presenting with small bowel obstruction dates back
to the third or fourth century, when Praxagoras created an enterocutaneous fistula to
relieve a bowel obstruction. Despite this success with operative therapy, the nonoper-
ative management of these patients with attempted reduction of hernias, laxatives, in-
gestion of heavy metals (e.g., lead or mercury), and leeches to remove toxic agents
from the blood was the rule until the late 1800s, when antisepsis and aseptic surgical
techniques made operative intervention safer and more acceptable. A better under-
standing of the pathophysiology of bowel obstruction and the use of isotonic fluid re-
suscitation, intestinal tube decompression, and antibiotics have greatly reduced the
mortality rate for patients with mechanical bowel obstruction. However, patients with
a bowel obstruction still represent.
Intestinal obstruction may be defined as failure of propulsion of intestinal con-
tents aborally. The condition occurs in many forms in both the small and large intes-
tine, due to either mechanical obstruction or a motility problem caused by neuromus-
cular failure or ischemia.
Neuromuscular failure is frequently associated with inflammation in the perito-
neal cavity or in the retroperitoneum. This type of intestinal obstruction, where the
intestinal lumen is not compromised, is also known as adynamic ileus. Pain originat-
ing from distension of the intestine, as occurs in bowel obstruction, is initially re-
ferred to the embryologic dermatome supplied by the same somatic nerve.
Inflammatory bowel disease (IBD) is an idiopathic disease, probably involv-
ing an immune reaction of the body to its own intestinal tract. The 2 major types of
IBD are ulcerative colitis (UC) and Crohn’s disease (CD). As the name suggests, ul-
cerative colitis is limited to the colon; Crohn’s disease can involve any segment of the
gastrointestinal tract from the mouth to the anus.
Educational aims:
6. Interrogation and clinical inspection of patients with bowel obstruction,
inflammatory bowel disease.
7. To determine the etiologic and pathogenic factors of bowel obstruction,
inflammatory bowel disease.
8. To find out the types of bowel obstruction, the clinical features, different
variants of manifestation and complications.
9. To develop a plan of examination of the patients with bowel obstruction,
inflammatory bowel disease.
10. To estimate laboratory data, results of X-ray, ultrasound examination,
computed tomography (CT), magnetic resonance imaging (MRI) of the intestines.
11. To draw a differential diagnosis, substantiate and formulate a diagnosis
of the bowel obstruction.
12. To prescribe the treatment for patients with the bowel obstruction, in-
flammatory bowel disease.
13. To determine the indications for operative treatment of patients with the
bowel obstruction, inflammatory bowel disease.
14. To cure of the patients with bowel obstruction, inflammatory bowel dis-
ease after operations.
15. To estimate efficiency of treatment and prognosis of disease.
A student must know:
7. Аnatomo-physiological information about of the intestines.
8. Classifications of bowel obstruction.
9. Mechanisms of development of bowel obstruction.
10.Clinical picture of bowel obstruction, inflammatory bowel disease.
11.Methods of diagnostics of bowel obstruction, inflammatory bowel disease.
12.Principles of treatment of bowel obstruction, inflammatory bowel disease.
13.Features of surgical interventions for the different types of bowel obstruction.
A student must be able to:
7. Collect and estimate the complaints of patient with the bowel obstruction, in-
flammatory bowel disease, information of anamnesis, to conduct physical re-
search and correctly interpret the results obtained.
8. Define the rational volume of laboratory and instrumental methods of research.
9. Correctly interpret the results of clinical analyses, X-ray. Ultrasound diagnos-
tics, computed tomography, magnetic resonance imaging.
10.Define indications for operation and other methods of treatment of patients.
11.Perform pre-operative preparations of patients.
12.Conduct post-operative care.
Terminology.
Term Definition
Mechanical obstruc- means that luminal contents cannot pass through the gut
tion tube because the lumen is blocked.
Luminal contents fail to pass because of disturbances in
Functional obstruc-
gut motility that prevents coordinated peristalsis from one
tion
region of the gut to the next.
Blood flow to the obstructed segment is compromised,
and tissue necrosis and gangrene are imminent. Strangula-
Strangulated ob-
tion usually implies that the obstruction is complete, but
struction
some forms of partial obstruction can also be complicated
by strangulation.
The Vala's sign Is the elastic sausage-shaped deformity of the bowel.
Sklarov's sign Is the sound of intestinal splash.
Kywul's sign Is the sound on percussion above the exaggerated bowel.
Spasokukotsky's sign Is "sound of falling drop".
incompletely closed anus in combination with balloon
Hochenegg's sign
expansion of ampulla of rectum
is a term generally used to denote two diseases of un-
Inflammatory bowel
known etiology with similar general characteristics: ul-
disease (IBD)
cerative colitis and Crohn's disease

Content:
BOWEL OBSTRUCTION
Etiology
Small Intestine Colon
Gallstone ileus Fecal impaction
Luminal Foreign body Foreign body
Worms
Tumor Tumor
Bowel wall Strictures Strictures
lesions Intussusceptions Diverticular disease
Radiation enteritis
Extrinsic Adhesions Volvulus
compression Hernias Extrinsic tumor
Volvulus Hernias
Extrinsic tumor
Extrinsic
inflammation

Classification (by D.P.Chuhrienko, 1958)


Acute intestinal obstruction is divided:
I. According to morphofunctional signs.
1. Dynamic intestinal obstruction:
a) paralytic;
b) spastic;
c) Inadequate blood supply of intestines(embolism, thromboembolism).
2. Mechanical intestinal obstruction:
a) strangulation( volvulus, node formation)
b) obturation (closing of bowel lumen in side or compression from outside)
c) Mixed (invagination, adhesive [intestinal] obstruction).

II. According to clinical features.


1. Acute.
2. Chronic.
III. According to the level of obstruction.
1. Small - bowel obstruction.
2. Large -bowel obstruction.
IV. According to the passing of intestinal contents.
1.Complete.
2.Partial.
V. According to the origin.
A. Innate.
B. Acquired.
VI. According to development of pathological process.
1. Stage of acute violation of intestinal motility.
2. Stage of hemodynamic disorders of bowel wall and its mesentery.
3. Stage of peritonitis.
Pathophysiology
With onset of obstruction, gas and fluid accumulate within the intestinal lumen
proximal to the site of obstruction. With ongoing gas and fluid accumulation, the
bowel distends and intraluminal and intramural pressures rise. If the intramural pres-
sure becomes high enough, microvascular perfusion to the intestine is impaired, lead-
ing to intestinal ischemia, and, ultimately to necrosis.
With partial bowel obstruction, only a portion of the intestinal lumen is occlud-
ed, allowing passage of some gas and fluid. The progression of pathophysiologic
events described above tends to occur more slowly than with complete bowel ob-
struction, and development of strangulation is less likely. In contrast, progression to
strangulation occurs especially rapidly with closed loop obstruction in which a seg-
ment of intestine is obstructed both proximally and distally (e.g., with volvulus).

Clinical Presentation
The symptoms of bowel obstruction are abdominal pain, nausea, vomiting, dis-
tention, and obstipation. Continued passage of flatus and/or stool beyond 6–12 h after
onset of symptoms is characteristic of partial rather than complete obstruction.
Strangulated intestinal obstruction. Blood supply disturbance and ische-
mia(compression of mesenteric vessels) is the characteristic feature of this form of
intestinal obstruction. It determines the pathomorphologic changes and clinical signs
of disease. Acute pain syndrome and ischemic disorders in the wall of bowel cause
necrotic changes in area of bowel affected by the disease. It is accompanied by the
progressive worsening of the patient condition and origin of endotoxicosis. Features
of strangulated obstruction include tachycardia, localized abdominal tenderness, fe-
ver, marked leukocytosis, and acidosis. Strangulation obstruction of the small or large
intestine is accompanied by symptoms and signs that suggest peritonitis. Large fluid
shifts and systemic toxicity are imminent or have already occurred. These signs in-
clude abdominal tenderness or involuntary guarding localized to the area of the stran-
gulated loop of bowel, decreased urine output, fever, and tachycardia.
Obturation intestinal obstruction, unlike strangulated, does not progress quick-
ly.
HIGH SMALL BOWEL OBSTRUCTION. The presenting symptoms of high
small bowel obstruction are colicky upper abdominal pain and profuse, bilious vomit-
ing. The onset of vomiting is close to the onset of pain, and the vomitus is nonfecu-
lent. On physical examination, vital signs are normal except late in the course, when
dehydration is present. Distension, if present, is not prominent. Abdominal tenderness
is absent, and bowel sounds are hyperactive, the crescendos of which coincide with
attacks of colicky pain.
LOW SMALL BOWEL OBSTRUCTION Colicky, midabdominal pain, vomit-
ing, and abdominal distension are the presenting symptoms of low small bowel ob-
struction. The interval between onset of pain and vomiting lengthens as the site of ob-
struction is more distal. Vomiting may be feculent. No gas or feces will have been
passed through the rectum for variable periods of time. Abdominal tenderness is min-
imal or absent, and rectal examination is normal.
LARGE BOWEL OBSTRUCTION
Colon cancer and diverticulitis account for nearly 90% of cases of large intesti-
nal obstruction, with colon cancer alone responsible for 65% to 70% of these. The
most frequent site of obstruction from either disease is the sigmoid. Other causes of
colon obstruction include inflammatory bowel disease, postanastomotic strictures,
benign tumors, and fecal impaction.
Symptoms and signs include those of mechanical obstruction as well as those
of underlying disease producing the obstruction. The pain is crampy and suprapubic.
Vomiting is a late symptom, and the vomitus is typically feculent. Constipation and
obstipation are constant features. Abdominal distension can be prominent, especially
in sigmoid volvulus. Bowel sounds are hyperactive and high pitched. Superimposed
on these symptoms and signs are those of the underlying disease producing the ob-
struction. Patients with colon cancer may complain of a change in bowel habits and
rectal bleeding. Patients with diverticulitis may have a history of alternating diarrhea
and constipation and usually have signs of the inflammatory process: fever, tachycar-
dia, and tenderness or mass in the left lower quadrant.
Sigmoid Volvulus Three conditions promote sigmoid volvulus: a redundant
sigmoid, long sigmoid mesentery with a narrow base, and fecal loading due to chron-
ic constipation. The condition tends to occur in the elderly, in those who are bedrid-
den, and in those receiving psychotropic medication for a psychiatric disorder. The
bowel twists counterclockwise about its long mesentery. A complete twist of 360°
leads to occlusion of not only the bowel lumen but also of the vascular pedicle in the
mesentery. If the obstruction is not reversed promptly, sigmoid gangrene and perfora-
tion ensue. Usually, abdominal distension is very prominent, and colicky suprapubic
pain develops. Pain is usually not an important symptom, but abdominal distension
can be massive and lead to cecal perforation. Cecal Volvulus Cecal volvulus is much
less common than sigmoid volvulus, occurring with half the frequency. A predispos-
ing condition is incomplete embryologic fixation of the cecum, which results in hy-
permobility of the organ. Distension and colicky pain develop in the midabdomen.
The patient often has a previous medical history of similar but milder attacks.
Diagnosis
The diagnostic evaluation should focus on the following goals: distinguishing
mechanical obstruction from adynamic obstruction; determining the etiology of the
obstruction; discriminating partial from complete obstruction; and discriminating
simple(obturation) from strangulating obstruction. Important elements to obtain on
history include prior abdominal operations(suggesting the presence of adhesions) and
the presence of abdominal disorders (e.g., intra abdominal cancer or inflammatory
bowel disease) that may provide insights into the etiology of obstruction. On exami-
nation, a meticulous search for hernias (particularly in the inguinal and femoral re-
gions) should be conducted. The stool should be checked for gross or occult blood,
the presence of which is suggestive of intestinal strangulation.
The diagnosis of bowel obstruction is usually confirmed with radiographic ex-
amination. The abdominal series consists of a radiograph of the abdomen with the pa-
tient in a supine position, a radiograph of the abdomen with the patient in an upright
position, and a radiograph of the chest with the patient in an upright position. The
findings which are most specific for bowel obstruction are: dilated bowel loop, air–
fluid levels seen on upright films. False negative findings on radiographs can result
when the site of obstruction is located in the proximal small bowel and when the
bowel lumen is filled with fluid but no gas, thereby preventing visualization of air–
fluid levels or bowel distention.
Computed tomography (CT) scan findings of bowel obstruction include a dis-
crete transition zone with dilation of bowel proximally, decompression of bowel dis-
tally, intraluminal contrast that does not pass beyond the transition zone, and in case
of small-bowel obstruction a colon containing little gas or fluid. Strangulation is sug-
gested by thickening of the bowel wall, pneumatosis intestinalis (air in the bowel
wall), portal venous gas, mesenteric haziness, and poor uptake of intravenous contrast
into the wall of the affected bowel. CT scanning also offers a global evaluation of the
abdomen and may therefore reveal the etiology of obstruction. A limitation of CT
scanning is its low sensitivity in the detection of low-grade or partial bowel obstruc-
tion. In such cases, contrast examinations of the bowels can be helpful. Contrast is
swallowed or instilled into the stomach through a nasogastric tube.
Abdominal radiographs are then taken serially as the contrast travels distally in
the intestine. Although barium can be used, water-soluble contrast agents, such as
Gastrografin, should be used if the possibility of intestinal perforation exists. For en-
teroclysis, 200– 250 mL of barium followed by 1–2 L of a solution of methylcellulose
in water is instilled into the proximal jejunum via a long nasoenteric catheter.
In obstruction, laboratory studies do not play a direct role in diagnosis, but are
helpful in evaluating complications such as dehydration, strangulation, and sepsis.
Laboratory findings reflect intravascular volume depletion and consist of hemocon-
centration and electrolyte abnormalities. Mild leukocytosis is common.
Other abdominal conditions, such as appendicitis, diverticulitis, perforated pep-
tic ulcer, cholecystitis, or choledocholithiasis, can usually be distinguished from
bowel obstruction by clinical examination and basic laboratory data. Bowel obstruc-
tion can complicate any of these abdominal conditions. Thus, the presence of another
abdominal process does not exclude the complication of bowel obstruction, and the
symptoms of bowel obstruction do not exclude other conditions.
Therapy
Bowel obstruction is usually associated with a marked depletion of intravascu-
lar volume caused by decreased oral intake, vomiting, and sequestration of fluid in
bowel lumen and wall. Therefore, fluid resuscitation is integral to treatment. An in-
dwelling bladder catheter is placed to monitor urine output. Central venous or pulmo-
nary artery catheter monitoring may be necessary, particularly in patients with under-
lying cardiac disease. Broad-spectrum antibiotics are commonly administered be-
cause of concerns that bacterial translocation may occur in the setting of small-bowel
obstruction. The stomach should be continuously evacuated of air and fluid, using a
nasogastric (NG) tube, to decrease nausea, distention, and the risk of vomiting and
aspiration.
The operative procedure performed varies according to the etiology of the ob-
struction. For example, adhesions are lysed, tumors are resected, and hernias are re-
duced and repaired. Regardless of the etiology, the affected intestine should be exam-
ined, and nonviable bowel resected. Criteria suggesting viability are normal color,
peristalsis, and marginal arterial pulsations. Usually visual inspection alone is ade-
quate in judging viability. In borderline cases, a Doppler probe may be used to check
for pulsatile flow to the bowel, and arterial perfusion can be verified by visualizing
intravenously administered fluorescein dye in the bowel wall under ultraviolet illumi-
nation. In general, if the patient is hemodynamically stable, short lengths of bowel of
questionable viability should be resected and primary anastomosis of the remaining
intestine performed. However, if the viability of a large proportion of the intestine is
in question, a concerted effort to preserve intestinal tissue should be made. In such
situations, the bowel of uncertain viability should be left intact and the patient reex-
plored in 24–48 h in a “second-look” operation. At that time, definitive resection of
nonviable bowel is completed.
Exceptions to the recommendation for expeditious surgery for intestinal ob-
struction include partial bowel obstruction, obstruction occurring in the early postop-
erative period, intestinal obstruction as a consequence of Crohn’s disease, and carci-
nomatosis. Progression to strangulation is less likely to occur with partial bowel ob-
struction, and an attempt at nonoperative resolution is warranted. However, most pa-
tients with partial small obstruction whose symptoms do not improve within 48 h af-
ter initiation of nonoperative therapy should undergo surgery. Patients undergoing
nonoperative therapy should be closely monitored for signs suggestive of peritonitis,
the development of which would mandate urgent surgery.
Obstruction that occurs in the early postoperative period is usually partial and
only rarely is associated with strangulation. Therefore, a period of extended nonoper-
ative therapy consisting of bowel rest, hydration, and total parenteral nutrition (TPN)
administration is usually warranted. However, if complete obstruction is demonstrat-
ed or if signs suggestive of peritonitis are detected, expeditious reoperation should be
undertaken without delay.
Intestinal obstruction in patients with Crohn’s disease often responds to medi-
cal therapy and is discussed in more detail later under “Crohn’s Disease.” Twenty-
five to 33 percent of patients with a history of cancer who present with small-bowel
obstruction have adhesions as the etiology of their obstruction and therefore should
not be denied appropriate therapy. Even in cases in which the obstruction is related to
recurrent malignancy, palliative resection or bypass can be considered. Patients with
obvious carcinomatosis pose a difficult challenge, given their limited prognosis.
Management must be tailored to an individual patient’s prognosis and desires.
Gallstone Ileus
As a result of intense inflammation surrounding a gallstone, a fistula may de-
velop between the biliary tree and the small or large intestine. Most fistulae develop
between the gallbladder fundus and duodenum. If the stone is more than 2.5 cm in di-
ameter, it can lodge in the narrowest portion of the terminal ileum, which is just prox-
imal to the ileocecal valve. This complication is rare, accounting for fewer than 6 in
1,000 cases of cholelithiasis and no more than 3% of cases of intestinal obstruction.
Typically, the patient is elderly and presents with intermittent symptoms over several
days, as the stone tumbles distally toward the ileum. The classic findings on plain ra-
diographs include intestinal obstruction, a stone lying outside the right upper quad-
rant, and air in the biliary tree .
Treatment includes removal of the stone and resection of the obstructed bowel
segment if there is evidence of tissue necrosis. The difficult decisions in management
relate to the biliary tract. Arguments in favor of resecting the biliary fistula and re-
moving the gallbladder include the possibility of recurrence of gallstone ileus and the
risk of cholangitis because of reflux of intestinal content into the biliary tree. When
surgery on the biliary fistula is performed, the mortality rate doubles relative to that
of simple removal of the gallstone. The long-term incidence of biliary tract infections
has not been high enough to warrant an aggressive approach at the initial operation.
Some clinicians have advocated cholecystectomy at a second operation, especially if
the patient is young and fit. Except in highly selected patients, cholecystectomy
should not be performed at the initial operation for gallstone ileus. The entire intes-
tine should be carefully searched to exclude the possibility of additional large stones.
The risk of a recurrent gallstone ileus is approximately 5% to 10%. Recurrences typi-
cally occur within 30 days of the initial episode and are usually caused by stones in
the small intestine that were missed at the original operation.
ILEUS AND OTHER DISORDERS OF INTESTINAL MOTILITY
Epidemiology
Dynamic ileus and intestinal pseudo-obstruction designate clinical syndromes
caused by impaired intestinal motility and are characterized by symptoms and signs
of intestinal obstruction in the absence of a lesion-causing mechanical obstruction.
Ileus is temporary and generally reversible if the inciting factor can be corrected. In
contrast, chronic intestinal pseudo-obstruction comprises a spectrum of specific dis-
orders associated with irreversible intestinal dysmotility.
Pathophysiology
The most frequently encountered factors causing ileus are abdominal opera-
tions, infection and inflammation, electrolyte abnormalities, and drugs. Among the
proposed mechanisms responsible for postoperative ileus are surgical stress-induced
sympathetic reflexes, inflammatory response-mediator release, and anesthet-
ic/analgesic effects, each of which can inhibit intestinal motility. Chronic intestinal
pseudo-obstruction can be caused by a large number of specific abnormalities affect-
ing intestinal smooth muscle, the myenteric plexus, or the extra intestinal nervous
system. Both sporadic and familial forms of visceral myopathies and neuropathies ex-
ist. Systemic disorders involving the smooth muscle such as progressive systemic
sclerosis and progressive muscular dystrophy, and neurologic diseases such as Par-
kinson disease also can be complicated by chronic intestinal pseudo-obstruction. Ad-
ditionally, viral infections, such as those associated with cytomegalovirus and Ep-
stein- Barr virus can cause intestinal pseudo-obstruction.
Clinical Presentation
The clinical presentation of ileus resembles that of small-bowel obstruction. In-
ability to tolerate liquids and solids by mouth, nausea, and lack of flatus or bowel
movements are the most common symptoms. Vomiting and abdominal distention
may occur. Bowel sounds are characteristically diminished or absent, in contrast to
the hyperactive bowel sounds that usually accompany mechanical small-bowel ob-
struction. The clinical manifestations of chronic intestinal pseudo-obstruction include
variable degrees of nausea, vomiting, abdominal pain, and distention.
Diagnosis
Routine postoperative ileus should be expected and requires no diagnostic
evaluation. If ileus persists beyond 3–5 days postoperatively or occurs in the absence
of abdominal surgery, diagnostic evaluation to detect specific underlying factors ca-
pable of inciting ileus and to rule out the presence of mechanical obstruction is war-
ranted.
Patient medication lists should be reviewed for the presence of drugs known to
be associated with impaired intestinal motility. Measurement of serum electrolytes
may demonstrate hypokalemia, hypocalcemia, hypomagnesemia, hypermagnesemia,
or other electrolyte abnormalities commonly associated with ileus. Abdominal radio-
graphs are often obtained, but the distinction between ileus and mechanical obstruc-
tion may be difficult based on this test alone. In the postoperative setting, CT scan-
ning is the test of choice because it can demonstrate the presence of an intra ab-
dominal abscess or other evidence of peritoneal sepsis that may be causing ileus and
can exclude the presence of complete mechanical obstruction.
The diagnosis of chronic pseudo-obstruction is suggested by clinical features
and confirmed by radiographic and manometric studies. Diagnostic laparotomy or
laparoscopy with full-thickness biopsy of the small intestine may be required to es-
tablish the specific underlying cause.
Management
The management of ileus consists of limiting oral intake and correcting the un-
derlying inciting factor. If vomiting or abdominal distention are prominent, the stom-
ach should be decompressed using a nasogastric tube. Fluid and electrolytes should
be administered intravenously until ileus resolves. If the duration of ileus is pro-
longed, TPN may be required. The therapy of patients with chronic intestinal pseudo-
obstruction focuses on palliation of symptoms and fluid, electrolyte, and nutritional
management.
Surgery should be avoided if at all possible. Prokinetic agents, such as meto-
clopramide and erythromycin, are associated with poor efficacy. Cisapride has been
associated with palliation of symptoms; however, because of cardiac toxicity and re-
ported deaths, this agent is restricted to compassionate use.
Colonic Pseudo obstruction
Etiologic Factors Acute pseudo obstruction of the colon, also known as
Ogilvie's syndrome, is an often painless paralytic ileus of the large bowel character-
ized by rapidly progressive abdominal distention. Plain radiographs of the abdomen
may reveal air in the small bowel and distention of discrete segments of the colon
(cecum or transverse colon) or of the entire abdominal colon. Although the distention
of the colon is not caused by mechanical obstruction, the wall of the bowel, particu-
larly that of the cecum, can become sufficiently distended so that its blood supply is
compromised. Gangrene, perforation, peritonitis, and shock can follow. Major risk
factors for the development of Ogilvie's syndrome include severe blunt trauma, or-
thopedic trauma or procedures, acute cardiac events or coronary bypass surgery, acute
neurologic events or neurosurgical procedures, and acute metabolic derangements.
Only 5% of cases occur in the absence of other conditions. Several lines of evidence
suggest that Ogilvie's syndrome is related, at least partly, to sympathetic nervous
overactivity or interference with sacral parasympathetic efferents.
Diagnosis
The diagnosis is usually apparent from plain films. In doubtful cases, and when
bowel necrosis is not a significant worry, a gentle Hypaque contrast enema can estab-
lish the nonmechanical nature of the dilatation. Colonoscopy can be both therapeutic
and diagnostic. Features suggesting the complication of bowel ischemia include local-
ized tenderness, leukocytosis, metabolic acidosis, evidence of sepsis, and a rapidly
deteriorating clinical course.

Management
Initial management includes resuscitation and correction of underlying meta-
bolic or electrolyte imbalances. A nasogastric tube is helpful if the patient is vomiting
and can prevent swallowed air from passing distally. When bowel ischemia is sus-
pected, surgery is indicated. If bowel necrosis is found, the affected segment is re-
sected and an ileostomy or colostomy established. If the bowel is viable, a cecostomy
is placed to vent the colon and prevent distention.
If distention is painless and the patient shows no signs of toxicity or bowel is-
chemia, expectant management is successful in approximately 50% of cases. If the
distention worsens so that the cecal diameter increases beyond 10 to 12 cm, or if it
persists for more than 48 hours, colonoscopy is recommended. Endoscopic decom-
pression is successful in 60% to 90% of cases but colonic distention can recur in up to
40%. Rectal tubes are ineffective in managing distention of the proximal colon. Such
tubes can be useful in promoting passage of air and feces after colonoscopy, but
should not be used as temporizing measures to avoid colonoscopic decompression. In
anecdotal reports, prokinetic agents such as cisapride and erythromycin have been
used to treat Ogilvie's syndrome with success. Successful resolution of pseudo ob-
struction has been reported with sympatholytic agents or spinal sympathetic block.
The efficacies of these modalities have not been systematically evaluated.
In the most recent studies, the sympatholytic agent, neostigmine, has been ad-
vocated if a 24-hour interval of conservative measures (nasogastric suction, intrave-
nous fluids, nothing by mouth) has failed to improve symptoms. Serious cardiovascu-
lar complications can occur and patients require telemetry. In addition, underlying
factors (sepsis, electrolyte abnormalities, and ileus-promoting medications) should be
addressed to obtain the earliest and maximum benefit.
MESENTERIC ISCHEMIA
Mesenteric ischemia can present as one of two distinct clinical syndromes:
acute mesenteric ischemia and chronic mesenteric ischemia.
Pathophysiology
Four distinct pathophysiologic mechanisms can lead to acute mesenteric is-
chemia: arterial embolus, arterial thrombosis, vasospasm (also known as nonocclu-
sive mesenteric ischemia, or NOMI), and venous thrombosis. Regardless of the path-
ophysiologic mechanism, acute mesenteric ischemia can lead to intestinal mucosal
sloughing within 3 h of onset and full-thickness intestinal infarction by 6 h.
In contrast, chronic mesenteric ischemia develops insidiously, allowing for de-
velopment of collateral circulation, and, therefore, rarely leads to intestinal infarction.
Chronic mesenteric arterial ischemia results from atherosclerotic lesions in the main
splanchnic arteries (celiac, superior mesenteric, and inferior mesenteric arteries). In
most patients with symptoms attributable to chronic mesenteric ischemia, at least two
of these arteries are either occluded or severely stenosed. A chronic form of mesenter-
ic venous thrombosis can involve the portal or splenic veins and may lead to portal
hypertension, with resulting esophagogastric varices, splenomegaly, and hypersplen-
ism.
Clinical Presentation
Abdominal pain for which the severity is out of proportion to the degree of ten-
derness on examination is the hallmark of acute mesenteric ischemia. Associated
symptoms can include nausea, vomiting, and diarrhea. Physical findings are charac-
teristically absent early in the course of ischemia. With the onset of bowel infarction,
abdominal distention, peritonitis, and passage of bloody stools occur. With chronic
mesenteric ischemia, postprandial abdominal pain is the most prevalent symptom,
producing a characteristic aversion to food (“food-fear”) and weight loss. Most pa-
tients with chronic mesenteric venous thrombosis are asymptomatic because of the
presence of extensive collateral venous drainage routes. However, some patients with
chronic mesenteric venous thrombosis present with bleeding from esophagogastric
varices.
Diagnosis
It is important to consider and pursue the diagnosis of acute mesenteric ische-
mia in any patient who has the classic early finding of severe abdominal pain out of
proportion to physical findings. Laboratory test abnormalities, such as leukocytosis
and acidosis are late findings; no laboratory tests have clinically useful sensitivity for
the detection of acute mesenteric ischemia prior to the onset of intestinal infarction.
Patients suspected of having acute mesenteric ischemia and who have physical
findings suggestive of peritonitis should undergo emergent laparotomy. In the ab-
sence of such findings, diagnostic imaging should be performed. Although angi-
ography is the most reliable method for diagnosing acute mesenteric arterial occlu-
sion, it is invasive, time-consuming, and costly. Therefore, most patients suspected of
having acute mesenteric ischemia should undergo CT scanning as the initial imaging
test. CT scans should be evaluated for (1) disorders other than acute mesenteric is-
chemia that might account for abdominal pain; (2) evidence of ischemia in the intes-
tine and the mesentery; and (3) evidence of occlusion or stenosis of the mesenteric
vasculature. CT scanning is also the test of choice for diagnosing acute mesenteric
venous thrombosis. Angiography should be performed if CT scanning reveals no evi-
dence of mesenteric ischemia or other conditions that could account for acute ab-
dominal pain and a high clinical suspicion for the presence of mesenteric ischemia
remains. Additionally, because the CT findings of NOMI are nonspecific, patients at
risk and suspected of having NOMI should undergo angiography without delay. The
angiographic findings of NOMI include diffuse narrowing of mesenteric vessels in
the absence of obstructing lesions and reduced opacification of bowel parenchyma.
The gold standard for the diagnosis of chronic arterial mesenteric ischemia is
angiography, although CT angiography with three-dimensional reconstruction is non-
invasive and offers good resolution. CT findings suggestive of chronic mesenteric is-
chemia include the presence of atherosclerotic calcified plaques at or near the origins
of proximal splanchnic arteries and obvious focal stenosis of proximal mesenteric
vessels with prominent collateral development.
Therapy
For embolus or thrombus-induced acute mesenteric ischemia, the standard
treatment is surgical revascularization (embolectomy/thrombectomy/mesenteric by-
pass). These procedures are not indicated if most of the bowel supplied by the affect-
ed artery has already become infarcted, or if the patient is too unstable to undergo ad-
ditional surgery beyond resection of infarcted intestine. For patients diagnosed with
embolus or thrombus-induced acute mesenteric ischemia who do not have signs of
peritonitis, thrombolysis, using agents such as streptokinase, urokinase, or recombi-
nant tissue plasminogen activator, is an alternative therapeutic option.
The standard treatment of NOMI is selective infusion of a vasodilator, most
commonly papaverine hydrochloride, into the superior mesenteric artery. If signs of
peritonitis develop, emergent laparotomy should be performed and infarcted intestine
resected. The standard treatment of acute mesenteric venous thrombosis is anticoagu-
lation. Heparin administration is associated with reductions in mortality and recur-
rence rates, and should be initiated as soon as the diagnosis is made. As for mesenter-
ic ischemia of arterial origin, signs of peritonitis mandate laparotomy, and infarcted
bowel should be resected. Most patients should be maintained on warfarin to achieve
chronic anticoagulation for 6–12 months.
The standard therapy for chronic arterial mesenteric ischemia is surgical revas-
cularization using aortomesenteric bypass grafting and mesenteric endarterectomy
procedures. An alternative therapy is percutaneous transluminal mesenteric angio-
plasty alone or with stent insertion. The durability of these procedures in inducing re-
lief of symptoms appears to be less than that associated with surgical revasculariza-
tion.
Patients with chronic venous mesenteric thrombosis who have an underlying
thrombophilia identified should be treated with chronic anticoagulation.
Additional therapy is indicated to control or prevent recurrent bleeding caused
by esophagogastric varices. Pharmacologic agents such as propanol and endoscopic
therapy are the first-line modalities. Surgical portosystemic shunts are indicated in
patients whose bleeding cannot be controlled by conservative measures and who have
a suitable vein for portosystemic venous anastomosis.
INFLAMMATORY BOWEL DISEASE
Inflammatory bowel disease (IBD) is a term generally used to denote two dis-
eases of unknown etiology with similar general characteristics: ulcerative colitis and
Crohn's disease. The distinction between the two entities can usually be established
based on clinical and pathologic criteria, including history and physical examination,
radiologic and endoscopic studies, gross appearance, and histology. However, in
about 10% to 15% of patients with inflammatory disease confined to the colon, a
clear distinction cannot be made, and the disease is labeled indeterminate colitis.
The main differences between Crohn's disease and UC are:
• Location and nature of the inflammatory changes. Crohn's can affect any part
of the gastrointestinal tract, from mouth to anus (skip lesions), although a ma-
jority of the cases start in the terminal ileum. Ulcerative colitis, in contrast, is
restricted to the colon and the rectum.
• Microscopically, ulcerative colitis is restricted to the mucosa (epithelial lining
of the gut), while Crohn's disease affects the whole bowel wall.
• Crohn's disease and ulcerative colitis present with extra-intestinal manifesta-
tions (such as liver problems, arthritis, skin manifestations and eye problems)
in different proportions.
Epidemiology
An estimated 1-2 million people in the United States have ulcerative colitis or
Crohn’s disease. Before 1960, the incidence of ulcerative colitis was several times
higher than that of Crohn’s disease. The latest data suggest that the current incidence
of Crohn’s disease is approaching that of ulcerative colitis, although this change may
reflect improved recognition and diagnosis of Crohn’s disease. The incidence of ul-
cerative colitis is 7.3 cases per 100,000 people per year and the prevalence is 116 cas-
es per 100,000 people; the incidence of Crohn’s disease is 5.8 cases per 100,000 peo-
ple per year and the prevalence is 133 cases per 100,000 people.
• The geographic distribution of ulcerative colitis and Crohn's disease is similar
worldwide, with highest incidences in the United States, Canada, the United
Kingdom, and Scandinavia. Higher incidences are seen in northern locations
compared to southern locations in Europe and the United States.
• As with Crohn's disease, ulcerative colitis is thought to occur more commonly
among Ashkenazi Jewish people than non-Jewish people.
Etiology and pathogenesis.
The pathophysiology of IBD is under active investigation. The common end
pathway is inflammation of the mucosal lining of the intestinal tract, causing ulcera-
tion, edema, bleeding, and fluid and electrolyte loss.
Persons with IBD have a genetic predisposition (or perhaps susceptibility) for
the disease. The triggering event for the activation of the immune response has yet to
be identified. Possible factors related to this event include a pathogenic organism (as
yet unidentified), an immune response to an intraluminal antigen (e.g., protein from
cow milk), or an autoimmune process whereby an appropriate immune response to an
intraluminal antigen and an inappropriate response to a similar antigen is present on
intestinal epithelial cells (i.e., alteration in barrier function).
A great deal of research has been performed to discover potential genes linked
to IBD. One of the early linkages discovered was on chromosome 16 (IBD1 gene),
which led to the identification of the NOD2 gene (now called CARD15) as the first
gene clearly associated with IBD (as a susceptibility gene for Crohn’s disease). Stud-
ies have also provided strong support for IBD susceptibility genes on chromosomes 5
(5q31) and 6 (6p21 and 19p). NOD2/CARD15 is a polymorphic gene involved in the
innate immune system. The gene has more than 60 variations. Three of these varia-
tions play a role in 27% of patients with Crohn’s disease, primarily in patients with
ileal disease. One important point to note with all of these potential genes is that they
appear to be permissive (i.e., allow IBD to occur) but not causative (i.e., just because
the gene is present does not necessarily mean the disease will develop).
Ulcerative Colitis
Ulcerative colitis is a disease in which the major pathologic process involves
the mucosa and submucosa of the colon, with sparing of the muscularis. The typical
gross appearance of ulcerative colitis is hyperemic mucosa. Friable and granular mu-
cosa is common in more severe cases, and ulceration may not be readily evident, es-
pecially early in the course of the disease. However, ulceration may appear and vary
widely, from small superficial erosions to patchy ulceration of the full thickness of
the mucosa. The rectum is invariably involved with the inflammatory process. In fact,
rectal involvement (proctitis) is the sine qua non of the disease, and the diagnosis
should be seriously questioned if the rectal mucosa is not affected. The mucosal in-
flammation extends in a continuous fashion for a variable distance into the more
proximal colon. Pseudopolyps, or inflammatory polyps, represent regeneration of in-
flamed mucosa and are composed of a variable mixture of non-neoplastic colonic
mucosa and inflamed lamina propria.
A key feature of UC is:
• continuous involvement of rectum and colon
• does not involve ileum.
Classification of the disease.
1. Extent of involvement:
• Proctitis / proctosigmoiditis
• Left-sided colitis
• Subtotal
• Total involvement (pancolitis)
2. Endoscopic grades of inflammation activity:
• Loss of the vascular appearance of the colon
• Erythema (or redness of the mucosa) and friability of the mucosa (contact
bleeding)
• Superficial ulceration, which may be confluent
• Pseudopolyps.
3. Severity of disease:
• Mild disease correlates with fewer than four stools daily, with or without
blood, no systemic signs of toxicity, and a normal erythrocyte sedimentation
rate (ESR). There may be mild abdominal pain or cramping.
• Moderate disease correlates with more than four stools daily, but with minimal
signs of toxicity. Patients may display anemia (not requiring transfusions),
moderate abdominal pain, and low grade fever, 38 to 39 °C (99.5 to 102.2 °F).
• Severe disease, correlates with more than six bloody stools a day, and evidence
of toxicity as demonstrated by fever, tachycardia, anemia or an elevated ESR.
4. Local (colonic) complications:
• Toxic megacolon
• Perforation, peritonitis
• Profuse bleeding
• Flat dysplasia
• Malignancy
5. Extraintestinal features
• Aphthous ulcers of the mouth
• Ophthalmic (involving the eyes):
– Iritis or uveitis, which is inflammation of the iris
– Episcleritis
• Musculoskeletal:
– Seronegative arthritis, which can be a large-joint oligoarthritis (affecting
one or two joints), or may affect many small joints of the hands and feet
– Ankylosing spondylitis, arthritis of the spine
– Sacroiliitis, arthritis of the lower spine
• Cutaneous (related to the skin):
– Erythema nodosum,
– Pyoderma gangrenosum,
• Deep venous thrombosis and pulmonary embolism
• Autoimmune hemolytic anemia
• Clubbing, a deformity of the ends of the fingers
• Primary sclerosing cholangitis, or inflammation of the bile ducts
• Malnutrition and growth retardation in pediatric patients
The clinical presentation of ulcerative colitis depends on the extent of the dis-
ease process. Patients usually present with diarrhea mixed with blood and mucus, of
gradual onset.
They also may have signs of weight loss, and blood on rectal examination. The
disease is usually accompanied with different degrees of abdominal pain, from mild
discomfort to severely painful cramps.
Ulcerative colitis characteristically runs an intermittent course of relapse and
remission, although some patients may have a chronic continuous variant. In some
cases, the initial attack is fulminant, and toxic dilatation with exacerbation of ab-
dominal and systemic symptoms may occur at any time. Diarrhea with the passage of
mucus and blood is typical of relapse.
The following conditions may present in a similar manner as ulcerative colitis,
and should be excluded:
• Crohn's disease
• Infectious colitis, which is typically detected on stool cultures
• Pseudomembranous colitis, or Clostridium difficile-associated colitis, bacterial
upsets often seen following administration of antibiotics
• Ischemic colitis, inadequate blood supply to the intestine, which typically af-
fects the elderly
• Radiation colitis in patients with previous pelvic radiotherapy
• Chemical colitis resulting from introduction of harsh chemicals into the colon
from an enema or other procedure.
Diagnostic Studies
Expert colonoscopy is the mainstay of diagnosis and assessment of disease se-
verity and extent. In the early stages of disease, the only sign on sigmoidoscopy may
be loss of the rectal mucosal vessels. As the disease progresses, severe ulceration
leads to fulminant colitis, the complications of which include dramatic nutritional de-
pletion, toxic dilatation, perforation and severe bleeding. The biopsy specimens ob-
tained at colonoscopy are used for making a definitive diagnosis of ulcerative colitis
or Crohn’s disease, and for mucosa displasia grade evaluation, as well as for malig-
nancy detection.
Barium enema may also help in the assessment of the extent of disease but is
contraindicated in patients with fulminant colitis and those with toxic dilatation be-
cause of the risk of precipitating perforation. Typical changes include loss of haustra-
tions, fluffy granularity of the mucosa, and pseudopolyps. Undermining ulcers may
create a double contour to the edge of the colon. Widening of the retrorectal space,
due to perirectal inflammation and reduced distensibility of the rectum, is common.
In an acute attack, plain films of the abdomen may reveal a dilated gas-filled
colon in which pseudopolyps are evident. When toxic dilatation is suspected, daily
plain X-rays are essential to monitor progress.
CT scanning of the abdomen and pelvis has limited use in the diagnosis of
IBD, but findings may be very suggestive of IBD. Wall thickening on CT scans is
nonspecific and may occur from smooth muscle contraction alone, especially in the
absence of other extraintestinal inflammatory changes; however, the presence of in-
flammatory changes significantly increases the predictive value of the CT scan. CT
scanning is the ideal study to determine if the patient has abscesses, and it can be used
to guide percutaneous drainage of these abscesses. Fistulae also may be detected on
CT scans.
General workup
• A complete blood count is done to check for anemia; thrombocytosis, a high
platelet count, is occasionally seen
• Electrolyte studies and renal function tests are done, as chronic diarrhea may be
associated with hypokalemia, hypomagnesemia and pre-renal failure.
• Liver function tests are performed to screen for bile duct involvement: primary
sclerosing cholangitis.
• Urinalysis
• Stool culture, to rule out parasites and infectious causes.
• Erythrocyte sedimentation rate can be measured, with an elevated sedimenta-
tion rate indicating that an inflammatory process is present.
• C-reactive protein can be measured, with an elevated level being another indi-
cation of inflammation.
Medical Management
• Aminosalicylates
Brand name formulations include
– Mesalazine, Asacol, Pentasa, Mezavant, Lialda, and Salofalk.
– Sulfasalazine, also known as Azulfidine.
– Balsalazide - Disodium, also known as Colazal.
– Olsalazine, also known as Dipentum.
• Corticosteroids
– Cortisone, Prednisone, Prednisolone, Hydrocortisone
– Methylprednisolone
– Budesonide - under the brand name of Entocort, Budenofalk
• Immunosuppressive drugs
– Mercaptopurine,
– Azathioprine, ( Imuran, Azasan or Azamun, which metabolises to 6-
MP).
– Methotrexate, which inhibits folic acid
– Tacrolimus

• Biological treatment
– anti TNF drugs: Infliximab (5 mg/kg body weight), Visilizumab
– Helminthic Therapy (There are currently two closely related treatments
available, either inoculation with Necator americanus, commonly known
as hookworms, or Trichuris Suis Ova, (TSO) commonly known as Pig
Whipworm Eggs.)
Indications for surgery in ulcerative colitis
Elective
• Symptomatic disease unresponsive to or poorly controlled by medical man-
agement
• Chronic relapsing disease on discontinuation of medical management and ster-
oid dependency
• Complications of medical management
• Concerns about long-term immunosuppression, risk of malignancy and vi-
ral/atypical infections
• Severe dysplasia on surveillance biopsies
• Onset of colorectal adenocarcinoma
• Rarely, control of debilitating extra-colonic manifestations such as iritis and
sacroiliitis.
Emergency
• Fulminant colitis unresponsive to maximal medical management
• Toxic megacolon
• Free perforation
• Life-threatening haemorrhage
• Acute complications of medical management

Modern surgical practice aims to preserve continence following removal of the


entire diseased large bowel. Restorative proctocolectomy with retention of the anal
sphincters and reconstruction by formation of an ileal pouch anastomosed to the up-
per anal canal are now standard. This approach has the benefits of removing all but a
tiny cuff of rectal mucosa in the upper anal canal, and also maintaining the ability of
the patient's bowel to move normally. A temporary ileostomy may be required. Faecal
urgency is eliminated and the overall quality of life is excellent, despite the passage
of 4-6 liquid motions per day.
A Koch's pouch-ileostomy may be considered following panproctocolectomy
as an alternative to permanent ileostomy.
On emergency the two-stage procedure is recommended (obstructive colecto-
my first, and 2-nd stage procrectomy with ileal pouch –anal anastomosing restora-
tion).
Crohn’s Disease
Although originally described as a disease affecting the terminal ileum, it is
now clear that any part of the gastrointestinal tract can be involved. At the time of ini-
tial clinical presentation, the features of Crohn's disease (CD) may be indistinguisha-
ble from those of ulcerative colitis. Indeed, in cases of colonic Crohn's disease, it may
be difficult to differentiate the two conditions, even after resection and histological
assessment.
Leonard-Jones algorithm for CD differentiation
1) Segmental involvement of the colon;
2) Macroscopically non-involved mucosa surrounding affected segments;
3) Terminal ileitis
4) Deep snake-like) ulcers
5) Fistulas
6) Anal- / Peri-anal involvement
7) Granulomas
Diagnosis “ Crohn’s Disease” is possible in case: of any 3 criteria’s (1-6)
or in case of 7-th + any one (1-6).

There are four typical clinical forms of CD:


– In case of terminal ileum involvement – like appendicitis
– In case of colon involvement (> 20% ) – like UC
– Perianal fistulas, abscess
– Extraintestinal features
W.R. Best Crohn’s Disease Activity Index (CDAI)
• Remission (CDAI<150);
• Mild (150<CDAI>250);
• Moderate (250<CDAI>350);
• Severe (CDAI>350).
Diagnostic Studies
The general assessment includes evaluation of nutritional status and the detec-
tion of anaemia, which may be due to iron deficiency from chronic blood loss.
Normocytic anaemia of chronic disease or macrocytic anaemia due to vitamin B12 or
foliate malabsorption may result.
Elevated erythrocyte sedimentation rate (ESR) and acute-phase proteins such
as C-reactive protein are useful in monitoring disease, but are not specific for diag-
nostic purposes.
The standard diagnostic investigation is barium follow-through (or small bowel
enema). A typical appearance of small bowel contrast enema is rose-thorn ulcers and
a long irregular stricture of the terminal ileum at the site of previous ileocaecal resec-
tion may be seen. Active disease produces radiological evidence of thickening, lu-
minal narrowing and separation of loops, and is often associated with ulceration,
spike-like fissures and a cobblestone appearance. Skip lesions and fistula formation
may be apparent.
Proctoscopy, sigmoidoscopy and colonoscopy are used to determine the pres-
ence and extent of large bowel disease. Biopsy of macroscopically normal rectum at
sigmoidoscopy or colonoscopy may reveal occult large bowel involvement, particu-
larly if there has been troublesome perianal involvement.
Double-contrast barium enema may be useful in assessing the extent of disease
and may delineate fistula formation.
Ultrasound and CT scanning might be helpful to determine the abdominal (pel-
vic) abscesses, fistulas, strictures.
Medical Management
There are no drugs that can guarantee freedom from relapse, but Crohn's dis-
ease may respond to 5-aminosalicylic acid (5-ASA) agents, such as mesalazine and
olsalazine. These agents may be useful in colonic disease and, in some cases, for re-
lapsing terminal ileal disease, and there is limited evidence that maintenance therapy
may marginally reduce the risk of relapse.
Steroids can be used in the acute phase (prednisolone 30-60 mg daily).
Immunosuppression using azathioprine (3 mg/kg daily) or 6-mercaptopurine
can be used in resistant cases to induce remission and also to maintain it. However,
there are concerns about complications of long-term immunosuppression and the
agents are not generally continued beyond 2 years without review; they are seldom
used beyond 4 years.
Monoclonal antibodies to tumour necrosis factor-α (TNF-α) are now used fre-
quently in specialist centres and have a place in patients with fistulating Crohn's dis-
ease.
Newer agents such as anti-integrins are now being tested in clinical trials
Surgical management
Almost all patients with Crohn's disease require surgery at some stage. Unin-
volved bowel should be preserved and the residual small bowel length documented.
Stricturoplasty is a useful technique that involves longitudinal division of the stric-
tured small bowel, with closure of the defect transversely to widen the intestinal lu-
men. Radical surgery is contraindicated, as the risk of recurrence is determined by the
natural history of the disease rather than the extent of surgery. The recurrence rate
following small bowel resection is around 30%, as opposed to less than 20% in colon-
ic disease. In the initial phases of colonic Crohn's disease, segmental resection is pre-
ferred to bypass of affected segments.
Indications for surgery in Crohn's disease
Elective
• Chronic subacute obstruction due to fibrotic strictures, adhesions or refractory
disease
• Symptomatic disease unresponsive to or poorly controlled by medical man-
agement
• Chronic relapsing disease on discontinuation of medical management and ster-
oid dependency
• Complications of medical management (e.g. osteoporosis)
• Concerns about long-term immunosuppression, risk of malignancy and vi-
ral/atypical infections
• Perianal sepsis and fistula
• Enterocutaneous fistula
• Onset of malignancy, including colorectal adenocarcinoma and small bowel
lymphoma
• Rarely, control of debilitating extra-colonic manifestations such as iritis and
sacroiliitis
Emergency
• Fulminant colitis or acute small bowel relapse unresponsive to medical
management
• Acute bowel obstruction
• Life-threatening haemorrhage
• Abscess or free perforation
• Perianal abscess
Prevention
• No known dietary or lifestyle changes prevent IBD.
• Dietary manipulation may help symptoms in persons with ulcerative colitis,
and it actually may help reduce inflammation in persons with Crohn’s disease
(see Diet). However, no evidence indicates that consuming or avoiding any par-
ticular food item causes or avoids flares of IBD.
• Smoking cessation is the only lifestyle change that may benefit patients with
Crohn’s disease. Smoking has been linked to increases in the number and severity
of flares of Crohn’s disease. Smoking cessation is occasionally sufficient to
achieve remission in a patient with refractory Crohn’s disease.
Basic literature:
13. Oxford Textbook of Surgery (3-Volume Set) 2nd edition (January 15,
2000): by Peter J. Morris (Editor), William C. Wood (Editor) By Oxford Press
14. Sabiston Textbook of Surgery 17th edition by Courtney M. Townsend
Jr., Kenneth L. Mattox, B. Mark, MD Evers, Kenneth L., MD Mattox, Courtney
Townsend, Daniel Beauchamp, B. Mark Evers, Kenneth Mattox W.B. Saunders
Company (June, 2004)
15. Schwartz´s Principles of Surgery 8th Edition F. Charles Brunicardi. Cop-
yright ©2007 the McGraw-Hill Companies.
16. Hospital surgery/ Edited by L. Kovalchuk et al. - Ternopil: Ukrmedkny-
ha, 2004. - 472 p.

Additional literature:
A. Brandt LJ, Boley SJ: AGA technical review on intestinal ischemia. Gastroen-
terol 118:954, 2000.
B. Cohen JL, Strong SA, Hyman NH et al. Practice parameters for the surgical
treatment of ulcerative colitis // Dis Colon Rectum – 2005. - Nov;48(11) –
P.1997-2009.
C. Bass KN, Jones B, Bulkley GB. Current management of small-bowel ob-
struction. Adv Surg 1997;31:1–34.

Tests for initial level of knowledge, keys for tests:


i. Which of the following regarding inflammatory bowel disease is true?
A. Bloody diarrhea is more common in Crohn’s disease than ulcerative colitis.
B. Associated extraintestinal disease is less common in ulcerative colitis than in
Crohn’s disease.
C. Crohn’s disease frequently involves the entire colon.
D. Crohn’s is more commonly associated with primary sclerosing cholangitis
than is ulcerative colitis.
E. Surgical intervention for ulcerative colitis is limited to management of com-
plications of the primary disease process.

ii. Which of the following is generally not an acceptable surgical option for man-
agement of ulcerative colitis?
A. Lifelong conservative observation with biannual colonoscopy
B. Total proctocolectomy with ileostomy
C. Total proctocolectomy with continent ileostomy (Kock pouch)
D. Total abdominal colectomy with ileo-rectal anastomosis
E. Total proctocolectomy with “J”-pouch anastomosis

iii. Which of the following is not an indication for urgent surgery in patients with
ulcerative colitis?
A. Ongoing hemorrhage
B. Toxic megacolon
C. Failure of maximal medical management
D. Colonic perforation
E. All are indications for urgent surgery

D. Comparing the anorectal manifestation of Crohn's disease and ulcerative coli-


tis, which of the following is incorrect?
A. Perianal disease is common in Crohn's disease.
B. Perirectal fistulas occur frequently in ulcerative colitis.
C. Ulcerative colitis attacks always involve the rectum.
D. Bloody stools are less common in Crohn's disease than in ulcerative colitis.
E. Granulomas are a common finding on rectal biopsy in Crohn's disease.

17.Which of the following is not an indication for surgical intervention in ul-


cerative colitis?
A. Intractable bloody diarrhea
B. Perforation
C. Toxic colitis
D. Diagnosis of ulcerative colitis for more than 5 years
E. Poorly controlled extraintestinal manifestations

18. The symptoms of bowel obstruction are all EXEPT


A. Abdominal pain
B. Vomiting
C. Distention
D. Obstipation.
E. Violet spots on face and trunk

19. All of statements about high small bowel obstruction are correct EXEPT
A. Colicky upper abdominal pain
B. Profuse, bilious vomiting
C. Vomitus is nonfeculent
D. Distension is prominent
E. Dehydration is present

20. All of statements about large bowel obstruction are correct EXEPT
A. Colicky abdominal pain
B. Profuse, bilious vomiting
C. Vomitus is feculent
D. Distension is prominent
E. Dehydration is present

21.The main reason of large bowel obstruction is


A. Adhesions
B. Hernias
C. Volvulus
D. Tumor
E. Extrinsic inflammation

22.The main reason of small bowel obstruction is


A. Adhesions
B. Hernias
C. Volvulus
D. Extrinsic tumor
E. Extrinsic inflammation

Keys for tests


1 2 3 4 5 6 7 8 9 10
B D C B D E D B D A

Tests for final level of knowledge, keys for tests:


1. The most informative investigation in acute bowel obstruction is
A. X-ray of abdomen
B. Ultrasound
C. Colonoscopy
D. Anoscopy
E. Barium enema

2. All of statements about Cloiberg's cup are correct EXEPT


A. Sign of acute bowel obstruction
B. Has a form of the inverted bowl
C. Can be observed after an enema
D. Is formed by the level of liquid and air
E. Cloiberg's cup is free air in an abdomen

3. Cloiberg's cup in small bowel obstruction is


A. Wide and tall
B. Narrow and low
C. Wide and low
D. Narrow and tall
E. Is located on each side of abdomen

4. Cloiberg's cup in large bowel obstruction is


A. Wide and tall
B. Narrow and low
C. Wide and low
D. Narrow and tall
E. Narrow or wide and tall

5. Which of the following would be expected to stimulate intestinal


motility?
A. Fear
B. Gastrin
C. Secretin
D. Acetylcholine
E. Prolactin

6. For a symptomatic partial duodenal obstruction secondary to an


annular pancreas, the operative treatment of choice is
A. A Whipple procedure
B. Gastrojejunostomy
C. Vagotomy and gastrojejunostomy
D. Partial resection of the annular pancreas
E. Duodenojejunostomy

7. Operative planning and preoperative counseling for a patient with


a rectal carcinoma can be best provided if the patient is staged before sur-
gery by
A. Rigid proctoscopy
B. Barium enema
C. MRI of the pelvis
D. CT scanning of the pelvis
E. Rectal endosonography

8. A 65-year-old man who is hospitalized with pancreatic carcinoma


develops abdominal distention and obstipation. The following abdominal
radiograph is obtained. Appropriate management would best be achieved by
A. Urgent colostomy or cecostomy.
B. Discontinuation of anticholinergic medications and narcotics and
correction of metabolic disorders.
C. Digital disimpaction of a fecal mass in the rectum
D. Diagnostic and therapeutic colonoscopy
E. Detorsion of the volvulus and colopexy or resection

9. Criteria suggesting viability of intestine are all EXEPT:


A. Normal color
B. Temperature of intestine
C. Peristalsis
D. Marginal arterial pulsations
E. Absent of veins obstructed by clots.

10. All of statements about obstruction are correct EXEPT


A. The operative procedure performed varies according to the etiolo-
gy of the obstruction.
B. Adhesions are lysed
C. Tumors are resected
D. The lesion is much more common in the jejunum than in the ileum
at middle age
E. Nonviable bowel should be resected.

Keys for tests


1 2 3 4 5 6 7 8 9 10
A E C D D E E D E D

Tasks for final level of knowledge


1. An 82-year-old nursing home patient presents to the emergency room
with vomiting, abdominal pain, and distention. A radiograph is obtained and
demonstrates a grossly dilated loop of intestine overlying the sacrum in the shape
of an upside down U. Select previous diagnosis

The answer is bowel obstruction

2. A 36-years-old patient complaining of colicky upper abdominal pain, bil-


ious vomiting and an increasingly distended abdomen. The presenting symptoms
of high small bowel obstruction are colicky upper abdominal pain and profuse, bil-
ious vomiting. On physical examination dehydration is present, abdominal tender-
ness is present, and bowel sounds are hyperactive. A radiograph demonstrates a
grossly dilated bowel loop , air–fluid levels seen on upright films. Make diagnosis
and correct treatment.

The answer is acute bowel obstruction. Treatment includes rehydra-


tion and surgical operation. Type of operation depends on kind of obstruc-
tion and condition of intestines.

3. An 80-year-old man is admitted to the hospital complaining of nausea,


abdominal pain, distention, and diarrhea. A cautiously performed transanal con-
trast study reveals an “apple core” configuration in the rectosigmoid. Make diag-
nosis. What is appropriate management at this time would be?

The answer is large bowel obstruction. Treatment is colon resection


and proximal colostomy

4. A 70-year-old woman has nausea, vomiting, abdominal distention, and


episodic, crampy midabdominal pain. She has no history of previous surgery but
has a long history of cholelithiasis for which she has refused surgery. Her ab-
dominal radiograph reveals a spherical density in the right lower quadrant. Make
diagnosis and correct treatment.

The answer is Gallstone Ileus. Treatment includes removal of the


stone and resection of the obstructed bowel segment if there is evidence of
tissue necrosis, resecting the biliary fistula and removing the gallbladder.

5. A 30-year-old female patient who presents with bleeding per rectum is found at
colonoscopy to have colitis confined to the transverse and descending colon. A bi-
opsy was performed. What diagnosis you expect to confirm by histological study?

The answer is: Crohn’s disease.


Materials for independent teaching of students

Main tasks Notes (instructions)


Repeat:
1. Surgical anatomy of the intestines -To represent the methods of diagnostics of
2. Physiology of intestines acute bowel obstruction as a table.
3. Pathogenesis of development of -To make the flow diagram of mechanisms of
complications of bowel obstruction. damage of intestines in acute bowel obstruction.
4. Morphological changes in the intes- - Make the scheme of T-cells response in nor-
tines in bowel obstruction. mal mucosa, pro-inflammatory and anti-
5. Clinical manifestations and diagnos- inflammatory mediators
tic methods in acute bowel obstruction.
Study:
1. Types of acute bowel obstruction. -To conduct differential diagnosis between
2. Types of accesses to the intestines. acute diseases of abdominal organs and acute
3. Tactics at arterio-mesenteric duode- bowel obstruction
nal obstruction -To conduct differential diagnosis of patholo-
4. Tactics in chronic bowel obstruction gies of the intestines
5. Endoscopic signs of IBD, diverticulosis, -Write the main endoscopic symptoms of the
colorectal polyps mentioned diseases.
6. Diagnostic value of barium enema, US, - List radiological, US, and CT-scan symptoms
and spiral CT in verifying the colorectal of the mentioned colorectal diseases.
diseases.
Study guide #6
“Syndrome of acute abdomen. Diagnosis, differential diagnosis and medi-
cal tactics. Differential diagnosis of acute abdominal viscous diseases.
Pseudoacute abdomen.”

Overview

Acute disease within the abdomen is common and many patients with ab-
dominal symptoms present every day to doctors working in the community. Within a
Western population of half-a-million people, between 5 and 10 patients are admitted
to a surgical ward each day with acute abdominal pain. One or two more will com-
plain of acute abdominal symptoms after an accident. By definition the illness starts
suddenly and most patients present to a hospital within 7 or 10 days of the onset of
symptoms. In the majority of patients, symptoms arise from disease within the ab-
dominal cavity itself, but occasionally they originate elsewhere in the body. The
range of disease extends from the relatively trivial to the immediately life threatening,
and attempts to reach a diagnosis must sometimes be curtailed in the interests of im-
mediate treatment. More commonly there is time to take a history, to examine the pa-
tient, and to organize the investigations that will be helpful in establishing a diagnosis
and planning treatment. Accurate recording of the relevant facts is vital, and a clear
understanding of the anatomy and pathophysiology of intra-abdominal disease is nec-
essary for both diagnosis and treatment. These patients are therefore ideal for training
junior members of a surgical team. Some patients require early surgery. This in itself
varies from a simple, straightforward procedure to a highly complex operation that
stretches the ability and skill of even the most experienced surgeon. The immediate
feedback that an emergency operation provides on the accuracy and the adequacy of
the preoperative assessment and preparation is another reason why the patient with an
acute abdomen is an important part of surgical training.
Educational aims
10. To collect the anamnesis and to spend clinical inspection sick with acute ab-
domen.
11. To know an etiology and pathogenesis of acute abdomen.
12. To know a clinical picture of acute abdomen.
13. The complication nobility of acute abdomen.
14. To make the plan of inspection sick of acute abdomen.
15. To carry out the analysis of data the received at laboratory and tool methods of
inspection sick with acute abdomen.
16. To define indications to surgical treatment acute abdomen or to appoint con-
servative treatment.
17. To spend care sick with acute abdomen.
18. To estimate work capacity sick with acute abdomen.
A student must know:
1. Surgical anatomy of the organs of abdominal cavity.
2. Pathophysiology and classification of acute abdomen.
3. Uncomplicated and complicated of acute abdomen.
4. Methods of examination of patients with the acute abdomen.
5 Symptoms and signs of the acute abdomen.
6. Clinical examination and tests.
7. Comparison of diagnostic methods.
8. Conservative treatment and operative treatment.
9. Indications and contraindications to the surgical interference and choice of the
method of the operation of acute abdomen.
A student must be able to:
1. Take anamnesis carefully.
2. Make diagnosis correctly.
3. Order additional examination.
4. To establish pathogenetic factors of occurrence of acute abdomen.
5. To diagnose complications of acute abdomen.
6. To make the plan of inspection sick with acute abdomen.
7. To estimate data of tool methods of inspection at patients with acute abdomen
(ultrasonic research, computer tomography, diagnostic laparoscopy).
8. To carry out differential diagnostics acute abdomen and pseudoacute abdomen.
9. To establish indications for surgical treatment and conservative treatment.
10.To estimate efficiency of treatment and patients and work capacity of patients.
Terminology.
Term Definition
Acute abdomen Designates symptoms and signs of intra-abdominal disease
usually treated best by surgical operation
Peritonism implying thereby irritation of the peritoneum without in-
flammation
Septic shock circulation bacterial endotoxins give rise to widespread pe-
ripheral vasodilatation and increased capillary permeabil-
ity with fluid leakage into the tissue
Signs of peritonitis are tenderness, guarding, rigidity and rebound tenderness
Pseudo acute abdomen is abdominal symptoms causes pathologic processes out-
sides the abdominal cavity

Content:

Anatomy
A good knowledge of normal and abnormal abdominal anatomy, and particu-
larly surface anatomy, is essential. Variations within and between individuals are ob-
vious, but normal anatomy also changes with age, posture, respiration, disease, and
previous surgery. Nevertheless, with experience most surgeons carry a remarkably
accurate mental picture of the expected internal position of any particular organ in
any particular patient. The embryological development of the abdomen is relevant in
two respects. The intestine and all its associated organs such as the liver and the pan-
creas develop initially as midline structures. Thus visceral pain is usually felt along
the midline of the abdomen. The gut also has a segmental origin so that the division
into foregut, midget, and hindgut exactly correlates with the vascular supply, and,
correspondingly, pain is felt in the epigastria, the umbilical area, and the hypogastria.
Certain congenital abnormalities can predispose to acute abdominal complications. In
contrast to the visceral peritoneum, the parietal peritoneum is innervated by somatic
nerves. Pain is therefore accurately localized to the site of irritation of the abdominal
wall and is accompanied by a reflex contraction of its muscles. This applies both to
the anterior and the posterior abdominal walls. Poses spasm from acute appendicitis
and a scoliosis concave to the side of intra-abdominal inflammation are two good ex-
amples. Inflammation confined to the pelvis may not, however, be accompanied by
spasm of anterior abdominal muscles and this may cause clinical confusion. This is
because the somatic nerves that supply the organs in the pelvis do not supply the
muscles of the anterior abdominal wall. When describing the findings of abdominal
examination the surface is best divided into six areas by a transverse line going
through the umbilicus and longitudinal lines running through the tip of the ninth rib
on each side. Thus there are epigastria and hypo gastric areas in the middle, and an
iliac fosse and hypochondria laterally. It is often also useful to describe the periumbil-
ical area. How-ever, it is important to realize that none of these divisions has a true
anatomical basis.
Physiology and pathology
Normal physiology is rapidly disrupted by the onset of acute intra-abdominal
disease. Many patients vomit, and gastrointestinal secretion, absorption and motility
all change in the presence of obstruction, luminal infection, or peritonitis. Urine is re-
duced in volume and altered in content, usually secondary to redistribution of fluid in
the body compartments but sometimes because of a direct toxic effect on the kidneys.
The mediation of abdominal pain is not well understood. It is perfectly possible to
handle the intra-abdominal organs and even divide the bowel of a conscious patient
without causing any pain. However, distension or stretching of the bowel wall is ac-
companied by reflex contraction of the smooth muscle in the wall, which is immedi-
ately painful. This may be due to transient ischemia of the muscle. The pain fibers run
with the splanchnic sympathetic nerves to the spinal column, where they are distrib-
uted segment ally. The pain is localized to the abdominal cavity but not to the precise
segment of bowel that is being stretched. Other pathways within the spinal column
are also stimulated, and vomiting, which is a common accompaniment of severe pain,
can also be centrally mediated.
The gastrointestinal tract is a significant source of a wide variety of hormones.
These change in response to acute disturbances of function but whether this is a pri-
mary or a secondary effect is not yet clear. Inflammation is the most common cause
of acute pathology within the abdomen, followed by obstruction, hemorrhage, trau-
ma, and ischemia. Bacteria, viruses, fungi, parasites, and chemicals can all cause in-
flammation: bacteria from the bowel, such as Escherichia coli, Streptococcus facials,
and various anaerobes, are by far the most important. Other bacteria that cause acute
abdominal pain are Salmonella and Shigella spp., Yersinia, and Campylobacter.
Acute inflammation normally develops into clinical significance over hours rather
than minutes or days, and progression either to suppuration or resolution also takes
time. Perforation and ischemia develop in minutes and cause very acute symptoms.
Resolution, whatever the underlying pathology, always takes longer than develop-
ment. Neoplasia, neurogenic, and metabolic disorders occur less commonly but they
are all well-recognized causes of acute abdominal pain. Some of these pathological
processes are closely interlinked. There are a number of causes of intestinal obstruc-
tion, of which neoplasia is one. Peritonitis from perforation of the bowel into the po-
tential peritoneal space usually arises from local ischemia, but this may in turn be
caused by inflammation or obstruction that has progressed to strangulation. The clini-
cal presentation and the physiological consequences of obstruction or peritonitis may
be similar whatever the cause, but a careful history and examination should enable
the underlying pathology to be discerned.
Clinical diagnosis
Most patients with an acute abdomen can be managed using simple clinical
skills. An accurate history and a thorough examination are often sufficient to make a
diagnosis and recommend treatment; modern investigations can help and may reas-
sure the anesthetist that the patient is fit for an operation. The primary objective when
the patient and the doctor first meet is, therefore, to elicit the symptoms and the signs
necessary to make a rapid and accurate diagnosis. It is sometimes obvious that the pa-
tient is in severe pain or seriously ill. The necessary immediate treatment must then
take precedence over making a diagnosis.
Unfortunately, even the most experienced clinicians only make a correct clini-
cal diagnosis of acute abdominal pain on four occasions out of five; younger doctors
and those who practice in the community are only right half the time. Many attempts
have been made to improve on these results and one method that has attracted much
attention is computer-assisted diagnosis. By a curious coincidence this has simply
taught us once again that taking an accurate history and examining the patient careful-
ly are still the most important factors in making a correct diagnosis.
History
Many patients will make their own diagnosis as one listens to their story: the
art of taking a history is to induce every patient to do so. Doctor and patient have not
usually met before, and the style and the approach of the doctor really do matter. A
relaxed, confident manner and a smile always help, and you must make it absolutely
plain to the patient that they have your complete attention and that you have plenty of
time to listen, even if this is not so. You should discourage interruptions by other
members of staff or requests to answer the telephone. Patients like to be treated as in-
dividuals. Go and sit by their bedside knowing their name and introduce yourself
clearly with your own name. Some patients will immediately start to describe their
symptoms and must be left to continue. Others look for a cue from the doctor. Sim-
ple, non-specific questions such as ‘what has happened?' or ‘why have you come to
hospital?' are best. Some will then give their history spontaneously; others reply in
only a few words and then need prompting again. It is occasionally better initially to
engage the patient in conversation about something entirely unrelated, such as their
job or their family, and then when they are relaxed lead the discussion back to the
acute problem. This is particularly useful with very anxious patients. The most diffi-
cult patient is the one who is garrulous about everything but the reason they have
come for help. Often there is nothing for it but to stop the flow of words deliberately
and redirect the patient to the current problem. It is difficult to do this without appear-
ing rude or disinterested: beware of the temptation to assume that there is little wrong
with these patients. They are sometimes simply frightened.
Most patients come to the end of their story spontaneously, and sometimes they
have told you everything you need to know in perfect order. Never intervene to clari-
fy a point of detail but do stop the `patient glen the information they offer becomes
irrelevant: it is important not to overload the brain with too many facts. When the pa-
tient has finished there will usually be some points that need amplifying or some fur-
ther information that is essential. This is best obtained by asking direct, but not lead-
ing, questions. It is very easy indeed to suggest the answer you want either by the
words you use, your facial expression, or the manner in which you speak or behave.
If you do this the answers will be unreliable. Asking questions is also an art that re-
quires tact and skill. Short, specific questions are best, and they must be phrased
clearly without using jargon and in language the patient understands. Some patients,
like most politicians, do not answer the question they are asked. You should insist,
politely, on a specific answer if one is possible. No two doctors ever obtain exactly
identical histories: a young surgeon may be amazed to hear a patient give a totally
contradictory reply to an apparently identical question from a senior colleague. It is
also surprising how often it is the very last thing the patient says that clinches the di-
agnosis. Not everyone can give a history themselves. Most children are shy or fright-
ened, although others, even the very young, sometimes tell a perfect story. The con-
fused and the mentally handicapped are often unreliable as regards facts, while the
memory of an elderly patient who is ill is often faulty. A relative or a friend must then
relate the history, but the clinician should remember that his or her personality then
intervenes. This is a particular problem if the patient is foreign and the history has to
be taken through an interpreter.
Complete attention to the patient and absolute concentration on everything he
or she says and how it is said is essential. Observation of the patient is slightly differ-
ent from inspection during the examination. It encompasses demeanor as well as an
assessment of personality, mood, and reaction to the illness. Movement, particularly
expressive movement of the hands, is always useful. Patients with peritonitis lie quite
still and look ill, patients with colic really do roll around, and patients with cholelithi-
asis often describe the pain radiating round into the flanks with their hands, for exam-
ple. Obvious and significant physical signs such as gross abdominal distension with
audible borborygmi, jaundice, or the smell of Melina should not be ignored: they all
point to a specific pathology that may be confirmed by specific questions. Allowing
the patient to talk freely does not prevent recording the facts in a systematic fashion.
In most hospitals this has to be done freehand but there are advantages in specially
designed forms. The information is recorded systematically, and omissions are obvi-
ous and can be corrected at once. Such forms also require the clinician to be specific
about the features of certain symptoms.
Pain
Most patients admitted with an acute abdominal problem complain of ab-
dominal pain. Cope, in his classic book, observed that acute pain lasting for more
than 6 h in a previously fit patient usually has a surgical cause. It is also a most im-
portant symptom: detailed enquiry about the nature of the pain will often indicate the
correct diagnosis.
Site
The first thing to establish is the precise site of the pain that the patient has
now. Some patients are extraordinarily obtuse about this, partly because they have
difficulty in answering and partly because they often do not understand why you want
to know. It is best to ask the patient to point with one finger to where the pain is worst
and to record this site in the notes. Those who wave a hand vaguely everywhere
probably do not have too much wrong with them. Pain often moves during the course
of an illness and it is then worthwhile asking where the pain was situated at the be-
ginning.

Radiation
Radiation of the pain to other parts of the body is often diagnostic. Radiations
of the pain to the testicle in urethras colic, to the shoulders in acute cholecystitis, and
to the knee with an obstructed obturator hernia are specific and typical examples.
Sometimes patients volunteer that a pain radiates elsewhere but more commonly it is
necessary to ask directly.
Onset
Some patients can say exactly when the pain started. They may be able to give
a time or say what they were doing. This always suggests a significant cause and an
acute pathological process, such as perforation or strangulation. Pain that wakes the
patient up at night is also significant, although it is not often possible to describe the
acuteness of onset. Sometimes pain is not the first symptom the patient noticed and
this may suggest a medical cause, as with the vomiting from gastroenteritis or the
marked anorexia of hepatitis. The duration of the illness gives some idea how far any
pathology may have progressed and this can be correlated with the findings on clini-
cal examination. Some patients relate the onset of their pain to an injury. Apparently
mild trauma is occasionally followed by serious intra-abdominal injury; on the other
hand it is more common for patients, after the onset of the symptoms, to try and relate
them to an injury. This can be dangerously misleading, as with acute testicular torsion
for example.
Frequency
There are two aspects to frequency. Alterations in the pain since this episode
started are useful pointers to the immediate diagnosis, whereas pains that have come
and gone in a similar way in the previous weeks or months suggest a longer term and
more chronic disease process. Variations in intensity in the short term can be classi-
fied into two types. Either the pain is constant or it comes and goes. If it comes and
goes with some degree of regularity, it is colic. Constant pain is associated with in-
flammatory conditions and colicky pain with distension of smooth muscle, as de-
scribed below.
Aggravation and alleviation
Any movement makes the pain of peritonitis worse, while lying still makes it
better. Acute exacerbation of the pain on walking, breathing, coughing, or going over
a bump in the road on the ride in to the hospital is equivalent to rebound tenderness
on examination. Pain in the shoulder on lying down comes from diaphragmatic stimu-
lation by an irritant fluid. The fluid is often blood from an intra-abdominal injury or
an ectopic pregnancy. Analgesics usually make the pain better; this can be deceiving.
Sometimes vomiting temporarily relieves the pain of obstruction.
Severity and type
Pain is a very subjective symptom and people's reaction to it varies widely. Ac-
companying signs such as sweating and tachycardia give the observer some idea of
severity, but this only establishes that there is something wrong with the patient,
which is often perfectly obvious anyway. Most patients find it very difficult to de-
scribe the nature of their pain and require prompting. No particular diagnoses are
suggested by such descriptions as boring, dragging, sharp or dull, and they are best
avoided.
Nausea and vomiting
These are two quite separate symptoms and both are useful in diagnosis. Nau-
sea may precede vomiting but it need not do so and neither does vomiting always fol-
low nausea. Nausea by itself is a less specific symptom, although it is a common ac-
companiment of gallstone disease. Anorexia is a separate and somewhat non-specific
symptom since most people, and particularly children, lose their appetite when they
are unwell. Pain normally precedes vomiting in surgical disease of the abdomen
whereas the reverse is often the case in medical conditions.
Vomiting is a classic symptom of intestinal obstruction and it usually accom-
panies colic. Vomiting often occurs after a bout of pain in obstruction and the shorter
the interval between the two the higher the obstruction. The vomit itself is initially
green in color but turns yellow and then frankly fecal as the obstruction persists.
Retching without vomiting suggests acute torsion of an intra-abdominal structure.
Vomiting does not often accompany perforation of a peptic ulcer or intra-abdominal
hemorrhage, and it is a late event in distal obstruction of the large bowel if it occurs at
all. Nausea and anorexia are more common than vomiting in appendicitis.
Bowel function
Diarrhea and constipation are two potentially confusing symptoms because
they mean different things to different people. It is important first to establish the pa-
tient's normal bowel habit and the normal consistency of the stool, and then to decide
if there have been any recent changes. Diarrhea to some people simply means fre-
quent defecation of normal fecal material, whereas repeated loose watery stools are of
greater interest to the surgeon. When true diarrhea is present it is important to estab-
lish whether other members of the household are afflicted. The presence of blood,
slime, or the black, tarry stools of Melina are all of obvious diagnostic value. If intes-
tinal obstruction is suspected, then failure to pass wind as well as stool is important.
Gynecological symptoms
Symptoms arising from the uterus, fallopian tubes, and ovaries are a common
reason for admission to hospital with acute abdominal pain. Furthermore, the ‘nega-
tive laparotomy' rate is highest in young women. Questions about normal and abnor-
mal menstrual function, vaginal discharge, and the risk of pregnancy are therefore es-
sential. Tact and sensitivity are required, but the answers really do matter: a ruptured
ectopic pregnancy is a potentially lethal condition.
Urinary symptoms
Alterations in the pattern of maturation suggest disease of the urinary tract.
Frequency is linked with inflammation, while anuria is most commonly caused by
acute retention in elderly men. Pain on passing urine must be separated into two clas-
ses. Abdominal pain exacerbated by maturation suggests irritation of the peritoneal
surface of the bladder, while stinging pain in the urethra on urination is characteristic
of infection. Patients should also be asked about the color of the urine and the pres-
ence of blood or pus. Dysuria is a symptom that means different things to different
doctors, and the term should not be used without specifying what is meant.
Past history
Any previous medical problem may be relevant to the cause of an acute admis-
sion for abdominal pain and it will certainly be relevant to the management. Chronic
indigestion can be a useful pointer to a possible cause of peritonitis. A past history of
abdominal surgery is important because adhesions have now overtaken hernias as the
most common cause of intestinal obstruction. Patients often report previous episodes
of abdominal pain and it is useful to establish whether this episode is identical. If it is,
then chronic surgical diseases that flare up intermittently must be considered. Recur-
rent acute pancreatitis would be a good example.
Drugs
Many people take therapeutic drugs. Most patients, when asked, think only of
those prescribed by the doctor but in many countries in the world, including the Unit-
ed Kingdom, it is possible to buy drugs without a doctor's prescription and these may
be relevant too. Diuretics and sympathomimetic drugs may be implicated in the onset
of acute retention; digoxin overdose classically causes vomiting followed by ab-
dominal pain, and many drugs cause cholestatic jaundice. Not all patients know what
drugs they are taking and pills may be transferred from bottle to bottle so that the la-
bels are unreliable. Ultimately, a direct enquiry to the doctor or the pharmacist who
wrote or supplied the prescription may be necessary.
Examination
No experienced doctor completely separates examination from taking the histo-
ry. Observation begins the moment the doctor meets the patient and does not end until
they part company. Most clinicians rapidly assimilate, almost unconsciously, many
features of a new patient, and not all of them can easily be described in words. Atti-
tude, alertness, mood, agitation, sweating, respiration, movement, the eyes, the color,
the facial expression, the pulse, the handshake, and many other factors are all put to-
gether to give an instant impression of the severity of the illness and sometimes the
diagnosis. The restlessness of a patient with colic is in marked contrast to the immo-
bility of peritonitis. The gaunt patient with sunken eyes, a weak, thready pulse, and
little respiratory or abdominal movement looks the same today as did patients’ two-
and-a-half thousand years ago when Hippocrates first described the fancies of severe
peritonitis. First impressions can, of course, be false and they are not a substitute for a
systematic examination. Some would say that examination does not add much to a
well taken history but more evidence to help unravel a diagnosis is usually welcome.
As with the history, examination of the whole patient is relevant in the overall man-
agement, although here we are concerned with the signs that are important in the di-
agnosis of the acute abdomen.
Vital signs
Pulse rate, respiratory rate, temperature, and blood pressure are all essential ob-
servations. The initial values on admission may be misleading because of the hustle
and bustle of the journey to hospital, but subsequent measurements are important in
any patient whose condition is observed following admission. The charts may give a
general clue as to the diagnosis. An increase in respiratory rate suggests pulmonary
pathology rather than abdominal disease. An isolated rise in temperature certainly in-
dicates disease but it does not specify where, nor does it necessarily signify infection.
The height and the course of a fever in an adult may point to a diagnosis; in children,
fever is an unreliable guide as it is notoriously labile. Consistent changes in these four
vital signs over time are useful indicators of progressive pathology. A persistent rise
in the pulse with an accompanying fall in the blood pressure is sure evidence that a
peptic ulcer is still bleeding; increasing fever means that an empyema of the gallblad-
der needs draining. Changes in pulse and blood pressure following abdominal trauma
are useful, although they usually indicate the need for active treatment rather than
specifying the underlying diagnosis.
General features
There are many signs found elsewhere in the examination that indicate disease
within the abdomen. General features of the patient, such as anemia, jaundice, and fa-
cial flushing, all have a direct relevance to abdominal diagnosis. The pallor of fear
must not be confused with the pallor of anemia, and cyanosis often accompanies an
acute intra-abdominal catastrophe. In children, acute inflammation of the upper res-
piratory tract can present with abdominal pain and examination is not complete until
the tonsils and the ear-drums have been inspected. Here, however, we are primarily
concerned with the abdominal signs.
Examination of the abdomen
Physical examination of the abdomen follows the time-honored sequence of in-
spection, palpation, percussion, and auscultation. Many signs can be seen and few pa-
tients, even young children, object to simple observation. Palpation can be painful
and it is certainly unusual. Explaining what you are doing helps a patient to relax and
so does distraction with conversation. Sometimes palpation with a stethoscope is use-
ful. Percussion and auscultation are less useful in the abdomen than in the chest. Dif-
ferent doctors obtain different histories and variations in the interpretation of physical
signs are even more marked. Natural variation is compounded by the lack of univer-
sal agreement on the definition of some physical signs. Despite this the basic findings
should be recorded in the notes. Eponymous signs are best avoided. In practice they
are rarely absolutely path gnomonic of one condition.
Inspection
Inspection of the abdomen is a subtle art. First and foremost both the patient
and the examiner should be comfortable. The patient must lie as flat and as straight as
possible with the head on a single pillow. Daylight and warmth are desirable and ade-
quate exposure of the abdomen essential, although it is kind to keep the genitalia cov-
ered until they are actually examined. Time should then be spent simply looking but
looking in an intelligent and thoughtful way. Most important physical signs can often
be seen. The history will have given some clues as to possible diagnoses, and there
will be specific signs to look for while remembering that negative findings are equal-
ly important. Previous abdominal operations will have been noted in the history and
the scars can be examined. Their only importance now is that there may be an inci-
sional hernia or underlying intraperitoneal adhesions. Obvious discoloration is always
important. Bruising from a seat-belt injury or the blue-grey discoloration in the flanks
or around the umbilicus from hemorrhagic pancreatitis are both good examples.
Shape
The first thing to decide is whether the shape, symmetry, and contour of the
abdominal wall are normal. Generalized distension is usually obvious except in obese
patients, when it can be very difficult to decide if the abdomen is simply fat. The
most common cause of generalized distension is a fetus. Excess fluid and air in the
gut and ascites are the common pathological causes of distension; this is usually
symmetrical. Asymmetrical distension is best judged from the end of the bed and is
caused either by a mass within the abdominal cavity or a lump in the abdominal wall.
The two can be differentiated because the latter must always move with the ab-
dominal wall whereas intraperitoneal lumps do not necessarily do so.
Movement
The abdominal wall normally moves with respiration. With the patient laying
on his or her back the abdominal wall rises up on inspiration as the diaphragm de-
scends and falls back on expiration. If this respiratory movement hurts, then the pa-
tient will try to reduce or eliminate any movement by keeping the abdominal wall
over the painful area still. This can often be seen and the effect can be enhanced by
asking the patient to take a deep breath. Another common technique, but one that is
less useful in the author's experience, is to ask the patient to blow their tummy out
and to suck it in. Patients with peritonitis find this painful, as they do when asked to
cough. Sometimes, in thin patients, it is possible to see the muscles of the abdominal
wall contract spontaneously in response to the painful stimulus. This is visible guard-
ing. Sometimes movement within the abdominal cavity can be seen on the surface.
Aortic pulsation and fetal movements are both normal, and so, occasionally, in elder-
ly individuals or those with gastroenteritis, is visible peristalsis. It is, however, a clas-
sic sign of intestinal obstruction. Distended loops of bowel can be seen through the
abdominal wall and peristaltic contractions can often also be seen. These contractions
are sometimes accompanied by borborygmi audible with or without stethoscope. Pa-
tience is needed, and sometimes peristalsis can be stimulated by palpation of the ab-
domen.
Palpation
Palpation of the abdomen requires warm hands, short fingernails, and care. By
convention the doctor sits on the patient's right with the right hand flat on the abdo-
men in a comfortable position. Students, however, should learn to be ambidextrous
because sometimes only the left side of the patient is accessible and some organs,
such as the gallbladder, are occasionally easier to feel from the left. Superficial palpa-
tion should consist of gentle movements of the whole hand. Deep palpation is
achieved by gentle pressure and by flexion of the metacarpophalangeal joints whilst
keeping the fingers extended. It is best to begin by asking the patient where the ab-
domen hurts and then to start palpating in the opposite corner. Work towards the
painful area but do take care. Once hurt, few patients will relax. The signs then be-
come difficult to interpret and are sometimes actually misleading. The abnormalities
of importance in the acute abdomen separate into three groups. There are the signs
associated with peritonitis, those that accompany a mass or enlargement of one of the
solid organs, and finally those that differentiate the causes of abdominal distension.
Signs of peritonitis The four signs of peritonitis are tenderness, guarding, rigid-
ity, and rebound tenderness. Eliciting these signs is painful and it is better to see than
to hear the pain. A flicker of the eyelids or a facial grimace is quite sufficient to es-
tablish the presence of pain, although guarding and rigidity are usually felt.
Tenderness This is present when any palpation of the abdominal wall causes
pain. It is either present or absent, although it is also possible to establish the extent of
the tenderness over the abdominal wall. It is not easy to assess severity because pa-
tients vary so much in their reaction to pain. It is useful to establish where in an indi-
vidual patient the pain is worst. Pain arising from the parietal peritoneum is accurate-
ly localized and patients can often point to the site of most intense pain. The examiner
can also ask the patient to compare the intensity of pain by direct pressure in the four
quadrants of the abdomen.
Guarding There are different opinions about the physical signs of guarding and
rigidity, so the examiner must be specific about what s/he actually means. In the au-
thor's opinion, guarding is present when there is reflex contraction of the muscles of
the abdominal wall when the examining hand palpates it and thus causes slight pain.
This may be seen but is more commonly felt.
Rigidity Again there is no generally accepted definition of this sign, but the
most useful description is of an involuntary increase in the resting tone of the muscles
of the abdominal wall. It may be localized or generalized. It is felt as an increased re-
sistance of the abdominal wall to palpation. The intensity varies from minor increases
in tension right up to the typical generalized, board-like rigidity classically associated
with perforation of a peptic ulcer.
Rebound tenderness This is the most important physical sign of the four. It can
be a difficult sign to elicit but when present it establishes the presence of peritonitis.
It occurs when inflamed visceral peritoneum moves across and irritates the parietal
peritoneum, and is best detected by percussion. This produces small movements of
the underlying tissues, causes least pain, and can even localize the sign to specific ar-
eas within the abdomen. The classical method of detecting rebound tenderness by
gross depression of the abdominal wall with the hand and then sudden release (hence
the term ‘release tenderness') is both crude and unkind, and while sometimes useful
should generally be abandoned. Rebound tenderness is also a symptom. Movement
such as walking or the jolting of a vehicle may exacerbate the abdominal pain and it
always worth enquiring about this whilst taking the history.
Abdominal swellings It is essential to establish the size of all the solid intra-
abdominal organs during palpation and equally important to identify any abnormal
masses. When the liver, spleen, and kidneys are enlarged there are certain specific
signs that must be sought. When an abnormal mass is felt either within or separately
from the solid organs, then all the usual rules relating to the examination of lumps
apply, although it may be impossible to assess swellings that lie deep within the ab-
dominal cavity. Particular attention should be paid to the anatomical origin of the
lump. Here mobility, and movement with respiration and pulsation, is useful. It is al-
ways helpful to establish that a swelling is cystic. Sometimes tenderness and the other
signs of peritonitis coexist with an abdominal swelling.
Abdominal distension Abnormal abdominal distension may be caused by an
abdominal swelling but flatus, fluid, and feces are more common. Pregnancy is gen-
erally obvious and feces are easily discovered on rectal examination. Excess gas or
fluid within the abdominal cavity is easy to demonstrate, but establishing the
presence of free intraperitoneal air or ascites can be difficult.
Groins and genitalia No abdominal examination is complete without examina-
tion of the groins and the genitalia, particularly in men. Hernias are common but not
always obvious. A small femoral hernia in a large woman is easily missed. If the her-
nia is the cause of an obstruction it will also be tense, tender, and irreducible, but it
may not be very large. Scrotal abnormalities such as testicular torsion and epididymo-
orchitis can present with abdominal pain, but there are always abnormal scrotal signs
on examination.
Rectal and vaginal examination No patient likes a rectal or a vaginal examina-
tion but they are essential. Again, the examination needs to be conducted with
thought.
Consider all the anatomical structures in the pelvis, including the prostate and
the cervix, and look at the glove for blood or pus when the examination is finished.
Rectal tenderness on the patient's right side may be the only sign of pelvic appendici-
tis. A swelling in a fallopian tube on vaginal examination may be the only sign of an
ectopic pregnancy.
Percussion
Percussion of the abdomen has three specific uses. First, it is the best method of
eliciting rebound tenderness. Second, it is the most sensitive method for detecting en-
largement of the bladder. Third, shifting dullness determines the presence or absence
of ascites. It has a subsidiary role in confirming the size of the liver and spleen, and
may sometimes be useful in outlining an intra-abdominal mass.
Auscultation
Auscultation of the abdomen is not very helpful but the presence or absence of
bowel sounds is a useful physical sign. Qualitative observations are less reliable.
Nevertheless, an increase in the magnitude and the frequency of bowel sounds ac-
companies mechanical intestinal obstruction whilst a succession splash, which can
sometimes be heard without a stethoscope, is a sure sign of obstruction. Bowel
sounds that definitely disappear during observation of a patient with abdominal pain
and tenderness indicate the onset of peritonitis and the need for a laparotomy.
Investigations
Although investigations are more or less routinely requested in most patients
with acute abdominal pain, very few of the tests are actually valuable in making a di-
agnosis. In a few patients no investigations are necessary because the diagnosis is
clinically obvious. In the majority the cause of the pain is initially uncertain and it is
hoped that tests will help. Older and more experienced surgeons maintain that it is
preferable to wait and see in these circumstances. They argue that significant disease
is usually progressive and when the patient is re-examined after an interval the physi-
cal signs are more marked and the diagnosis easier. Younger surgeons think that the
delay gives time for complications to develop, with a consequent increase in postop-
erative complications that diagnostic investigations might avoid. However, their en-
thusiasm for investigation can also delay a necessary operation if the tests take too
long to perform. In a few patients an accurate diagnosis that is essential for correct
treatment can only be made with the help of special investigations. We are most con-
cerned here with tests that will help in the diagnosis and the management of the pa-
tient within the first 24 h of admission. After that the number of tests that can some-
times be useful is vast and they are considered in the individual subject chapters.
Analysis of venous blood and various radiographs are the most popular immediate
investigations, with the addition of ultrasonographic examination and computed to-
mography. They can be divided into two groups. Those tests that help in diagnosis
and those that help in management.
Tests useful in diagnosis
Testing the urine
Simple clinical inspection of the urine should still be regarded as an essential
part of examination of the abdomen. Urine containing tiny amounts of blood looks
smoky. Sugar and ketones can both be smelt. Infected urine may be cloudy or blood-
stained, smells unpleasant, and contains nitrites and white blood cells. Confirmation
of all these findings using biochemical sticks is convenient and easy. When an infec-
tion of the urinary tract is suspected, then a carefully collected urine specimen should
also be sent immediately to the laboratory for analysis. It is not easy for any patient to
provide a true mid-stream urine specimen, and they must be both helped and super-
vised. Even then, contamination can be a problem and there are occasions, particular-
ly in women and children, when a catheter specimen should be collected. Urethral
catheterization is usually appropriate, but suprapubic puncture of the bladder provides
the least contaminated specimen and carries the least risk of introducing an infection.
Even though the culture result will not be available for a few days the sample must be
sent immediately otherwise the opportunity to identify the organism responsible may
be lost, as most patients with a urinary-tract infection presenting with acute ab-
dominal pain will need immediate treatment with antibiotics.
Blood tests
White blood-cell count Many significant cause of acute abdominal pain are as-
sociated with some degree of inflammation. As a consequence, an increase both in the
absolute numbers of white cells and in the proportion of neutrophils might be ex-
pected. The reverse observation is also true: an increase in the white-cell count indi-
cates the existence of inflammation. It is always necessary to interpret the result in the
clinical context, for the inflammation may not necessarily be within the abdomen. A
value within the normal range does not exclude intra-abdominal inflammation. This
very simple way of looking at the white-cell count is not the most useful. It is more
helpful to interpret the result in a statistical sense. In other words, the probability of a
patient with a normal white-cell count having acute appendicitis, for example, is low,
whilst the chances with a raised count are higher. The same observations may be
made about an excess of neutrophils in the differential white-cell count. Indeed the
results of all such tests used to establish a diagnosis should ideally be analyzed in this
way. In practice a normal white-cell count is often used to reassure the surgeon who
wants to wait and see, while an increased count supports a decision to operate. The
surgeon should realize, however, that the test is then being used to help in a manage-
ment decision and not to make a diagnosis.
Serum amylase Acute pancreatitis usually presents with the symptoms and
signs of peritonitis, and normally patients with peritonitis warrant an immediate lapa-
rotomy. Surgery is, however, best avoided in patients with acute pancreatitis. The rise
in serum amylase that usually accompanies pancreatitis allows the correct diagnosis
to be made and a laparotomy is thus averted. Because the result is so important for
both diagnosis and treatment it is essential to appreciate the limitations of the test.
Other intra-abdominal catastrophes, such as a perforated peptic ulcer, a ruptured aor-
tic aneurysm, or dead gut, can cause a modest rise in the serum amylase, while if the
blood sample is taken too long after the onset of the pancreatitis the enzyme's concen-
tration may have reverted to normal and so give a false-negative result. Again, a sta-
tistical approach can be adopted. A low serum amylase carries a low chance that the
patient has acute pancreatitis whilst a high concentration implies a high chance (but
not a certainty) that pancreatitis is indeed the diagnosis.
Radiological investigations
Plain abdominal radiographs Controversy surrounds the use of plain ab-
dominal radiology. Sometimes the films confirm the clinical diagnosis, add further
detail, and modify the management of an individual patient. At other times the films
are simply misleading, although occasionally they suggest a diagnosis that the clini-
cian has not considered. One thing is certain. Not every patient with acute abdominal
pain needs an abdominal radiograph. When one is requested the doctor should be
clear what information s/he hopes to gain and s/he must have the skills to interpret the
films if no radiologist is available. Traditionally two films are taken, one with the pa-
tient lying supine and the other with the patient sitting or standing erect. Modern pro-
tagonists of a single supine film point out that little additional information is derived
from the erect film and add that not every patient with acute abdominal pain can safe-
ly or comfortably sit or stand. Some radiologists prefer, as an alternative to the erect
film, to lay the patient on their right side and then take a lateral radiograph (the lateral
decubitus view). In the author's opinion an erect view does, on occasion, add useful
information whereas the lateral decubitus view usually does not. It provides only a
limited view of the abdominal cavity and free intraperitoneal gas is better seen on a
chest radiograph.
Abdominal films are more use in some circumstances than in others. None of
the radiological signs of acute appendicitis is truly helpful, but radiological examina-
tion should be performed in patients with suspected intestinal obstruction and those
who have suffered abdominal trauma. Stones in the kidney, the ureter, or the
gallbladder are sometimes confirmed on a plain film, and calcification of the wall of
an abdominal aortic aneurysm may be the only clue to its presence. Radiology of the
abdomen is more useful in older patients, who tend to have more significant patholo-
gy and thus more abnormalities on such films. It is important to remember that the
presence of abnormalities on any abdominal radiograph is valuable but their absence
is meaningless.
Chest radiography A good-quality, erect chest radiograph is the best film with
which to confirm the presence of free intraperitoneal air. This can be seen as a black
crescent, sometimes with an air/fluid level, underneath one or both diaphragms. Prox-
imal perforations of the bowel tend to lead to larger amounts of free air than distal
ones; if the perforation has occurred some time before presentation, as can happen in
patients with diverticular disease, the margin of the pneumoperitoneum on the radio-
graph is often rather hazy and irregular. There may also be a small pleural reaction
above the diaphragm. In very old and very young patients, pneumonia and pleurisy
present with referred abdominal pain. Fractures of lower ribs may indicate a ruptured
spleen or lacerated liver.
Intravenous urography Renal colic is usually an easy clinical diagnosis to
make because of the characteristic distribution of the pain. An emergency ultraso-
nographic examination may show a hydronephrosis on the affected side and will oc-
casionally identify a calculus. An urgent intravenous urogram adds additional infor-
mation and will indicate the site of an obstruction, and outline the degree of dilatation
of the urinary tract and the size of an offending stone.
Occasionally, emergency urography is useful. Delayed excretion of contrast on
the side where the patient complains of pain confirms the diagnosis. A normal
urogram effectively excludes the diagnosis, provided the examination is done within
a short time of the last episode of pain. Other causes of the abdominal pain can then
be considered. Intravenous urography is also useful in trauma to the urinary tract.
Most such patients have haematuria. The degree and the site of any damage may be
displayed and the presence of a normally functioning kidney on the unaffected side
can be confirmed.
Ultrasound examination
Ultrasound is widely used in the diagnosis of acute abdominal pain. Its place in
elective diagnosis of conditions affecting the upper abdomen, the pelvis, and the retro
peritoneum is already established; it is also useful in the emergency patient. Gall-
stones and an aortic aneurysm are easy to see, as are the edematous gallbladder wall
and a tear in the aneurysm. Transabdominal and vaginal ultrasound are both useful in
the pelvis to identify swellings of the uterus, ovaries or fallopian tubes. The ultra-
sound probe can also be used, like the examining hand, to identify the specific struc-
ture that hurts.
Ultrasound is less useful in examining the bowel because of the presence of
gas. However, the inflamed appendix often lies behind the caecum and contains little
air. Certainly the ultrasound probe can localize the tenderness to this specific area and
sometimes it can also demonstrate an edematous tubular structure at the site where a
retrocaecal appendix should lie.
Doppler ultrasound, which demonstrates flow in vessels, can help decide the
cause of acute testicular pain. The hyperemia of epididymo-orchitis is in marked con-
trast to the ischemia of torsion. Following trauma, ultrasound can demonstrate the
presence of free intraperitoneal fluid and look for damage to the liver, spleen, kidneys
and pancreas. It cannot identify blood clot very well and it is of no practical use in
looking for injury to the gut. Ultrasound is more readily available and can be brought
to the patient's bedside but computed tomography is more accurate.
Computed tomography (CT)
CT has become an essential tool in the diagnosis and the treatment of the acute
abdomen. Swelling of the pancreas and per pancreatic edema will confirm a diagnosis
of acute pancreatitis. Lack of perfusion of parts of the pancreas on a contrast study
implies potential pancreatic necrosis and identifies a patient with a high risk of com-
plications. CT with contrast is the best method of confirming a diagnosis of acute di-
verticulitis. It can also identify a per colic abscess, which may best be treated by CT-
guided drainage. CT is of occasional use in the diagnosis of an abdominal aortic an-
eurysm. Many patients with abdominal trauma require an immediate laparotomy.
Other patients are less acutely injured and CT is helpful in identifying the nature and
the extent of any intra-abdominal injury.
Tests useful in management
Many of the tests that are useful in diagnosis also have a role in management.
A progressive reduction in the white-cell count or an improvement in the radiological
signs of obstruction after treatment both indicates resolution of the pathology. A large
number of other tests also help in the treatment of a patient, many of which are under-
taken soon after the patient is admitted to hospital. Some of them also play a part in
diagnosis as well.
Blood tests
Hemoglobin concentration and packed cell volume The clinical diagnosis of
anemia is not always reliable, and in any patient who may possibly have an anesthetic
it is clearly important to know the oxygen-carrying capacity of the blood. The initial
hemoglobin value does not indicate the volume of blood lost in patients with overt
evidence of acute hemorrhage, but sequential measurements can give a rough guide,
provided any blood transfused is taken into account. Occasionally, the discovery of
an unexpectedly low hemoglobin can help in diagnosis: carcinoma of the caecum as a
cause for intestinal obstruction with anemia is a classic example. Packed cell volume
accurately reflects the severity of fluid loss in a dehydrated patient and it is a good
guide to the adequacy of rehydration.
Creatinine and electrolytes Most patients with significant intra-abdominal pa-
thology should have their creatinine and electrolyte concentrations measured on
admission. The initial values must be interpreted in the clinical context, particularly if
the patient is dehydrated; in most circumstances it is the serum potassium that is the
most important because of its role in cardiac function. Serial values are vital for prop-
er postoperative fluid management.
Liver function tests Most patients with an acute abdomen due to liver and bili-
ary disease are jaundiced. The depth of the jaundice reflects the severity of the pa-
thology, and it is rare to need to measure the liver function acutely. It is, however, es-
sential to obtain a blood sample on admission for later analysis because
subsequent biochemical deterioration, which may not be clinically obvious, will de-
mand further action. This particularly applies to elderly patients in whom the signs
and symptoms of biliary disease are often obscure. The diagnosis is sometimes not
even considered until abnormal results of liver function tests are discovered.
Calcium concentration This is only of immediate value in patients with acute
pancreatitis. Depleted values are an indirect guide to the diagnosis and are used in
some severity-scoring systems. When low calcium threatens to induce tetany, intra-
venous calcium supplements will be needed. Calcium is always measured in patients
with renal colic, in whom evidence of hyperparathyroidism is sought, but hypercal-
caemia is rarely found.
Blood gas analysis An arterial blood sample should be analyzed in a patient
who is severely ill with an acute abdomen from whatever cause. Many such patients
are covertly hypoxic, and the result of blood gas analysis may indicate the need for
immediate ventilatory support. More commonly, patients will need ventilation after
an emergency operation; preoperative values are then a useful measure of the patient's
progress. Blood gas analysis is also a component of many scoring systems to assess
the severity of acute pancreatitis.
Radiology
If a chest radiograph is not necessary for diagnosis it is unlikely that it will be
needed in the management of the patient. Nevertheless, there are times when, alt-
hough a clinical diagnosis can be made, a chest radiograph should be obtained simply
to provide a baseline. It is often useful to know that postoperative changes in a num-
ber of investigations, particularly the chest radiograph, were not present before sur-
gery.
Contrast radiology Conventional contrast radiology is rarely needed as an
emergency, although an urgent barium or air enema is important in a child with sus-
pected intussusceptions. In adults with obstruction of the large bowel a limited bari-
um enema examination is sometimes useful to establish the presence of a mechanical
rather than pseudo-obstruction. In patients with obstruction of the small bowel where
the cause is obscure or resolution is not occurring as fast as expected, then a small-
bowel enema is always helpful.
Electrocardiography
Anyone over the age of 40 years, who presents with acute abdominal pain, par-
ticularly if the diagnosis is not straightforward, should have an electrocardiograph.
Very occasionally a myocardial infarct will present with abdominal pain and recovery
is unlikely to be helped by an unnecessary laparotomy.

Endoscopy and arteriography


Emergency gastro copy and colonoscopy, occasionally performed on the operating
table, are helpful in patients who present with acute gastrointestinal hemorrhage. Pre-
cise localization of the bleeding point is essential for effective treatment. Mesenteric
angiography may be needed as well. In both instances, treatment as well as diagnosis
may be possible. Endoscopy has no part to play in the diagnosis of a perforated peptic
ulcer or diverticulitis. It may make matters worse by blowing air into the peritoneal
cavity through the perforation.
Peritoneal lavage
This is a useful investigation in patients with abdominal trauma, particularly if
they are unconscious or are otherwise unable to cooperate in an abdominal examina-
tion. The presence of significant amounts of blood or intestinal contents in the wash-
out fluid is a clear indication for an urgent laparotomy. Excess neutrophils in fluid as-
pirated from, or washed out of, the peritoneum is a reliable indicator of the presence
of peritonitis. The test is rarely used because it gives no indication of the site or the
cause of the inflammation.
Laparoscopy
Laparoscopy is invaluable in the diagnosis and the treatment of patients with
acute abdominal pain, although it does involve a general anesthetic. All the organs
responsible for the common causes of acute abdominal pain, such as the appendix, the
gallbladder, the fallopian tubes, the ovaries, most of the small bowel, the stomach and
the sigmoid colon, are easily seen through the laparoscope. In many cases it will be
appropriate to treat the problem as well. The technique of laparoscopic appendictomy
is now well established. In any event, it is usually possible to use a better-placed and
often smaller incision once the diagnosis is established even if a laparotomy is need-
ed. Laparoscopy has a limited role in the victim of abdominal trauma. If blood is
found, then the endoscopist must look for the source and decide if the severity is suf-
ficient to justify a laparotomy. If intestinal contents are found, a laparotomy is obliga-
tory.
Making a diagnosis
No one really understands how a doctor makes a diagnosis, although the pro-
cess has been analyzed many times. In theory it is simply a matter of collecting all the
relevant facts and analyzing them correctly. The contrast between the junior clinician
who takes time and trouble over the patient and yet makes the wrong diagnosis half
the time and the senior colleague who asks a few questions, performs a limited exam-
ination, and is right eight times out of ten shows that this is not the whole story. Very
few patients present with all the symptoms and signs of their disease and only experi-
ence can teach the clinician which few questions to ask, how to ask them, and how to
interpret the answers correctly in the context of the individual patient. The last skill is
particularly important when some of the facts conflict. Experience and constant prac-
tice are certainly essential for maximum accuracy. In actual clinical practice, several
methods are used to make a diagnosis. Some involve purely practical considerations
whilst others look at the same data in different ways. Most clinicians use all the
methods at one time or another, often together, and usually without giving the matter
a second thought.

Basic literature:
23.Oxford Textbook of Surgery (3-Volume Set) 2nd edition (January 15, 2000):
by Peter J. Morris (Editor), William C. Wood (Editor) By Oxford Press
24.Sabiston Textbook of Surgery 17th edition by Courtney M. Townsend Jr.,
Kenneth L. Mattox, B. Mark, MD Evers, Kenneth L., MD Mattox, Courtney
Townsend, Daniel Beauchamp, B. Mark Evers, Kenneth Mattox W.B. Saun-
ders Company (June, 2004)
25.Schwartz´s Principles of Surgery 8th Edition F. Charles Brunicardi. Copyright
©2007 the McGraw-Hill Companies.
26.Hospital surgery/ Edited by L. Kovalchuk et al. - Ternopil: Ukrmedknyha,
2004. - 472 p.
Additional literature:
1. Sherck JP, McCort JJ, Oakes DD. Computed tomography in thoraco-abdominal
trauma. Journal of Trauma 1984; 24: 1015–21. [Thoracoabdominal CT scan-
ning of stable trauma patients accurately identifies injury to the liver, spleen,
kidney, and the retro peritoneum.]
2. Siddiqui MN, Abid Q, Qaseem T, Hameed S, Ahmed M. ‘Spontaneous' rectus
sheath hematoma: a rare cause of abdominal pain. Journal of the Royal Society
of Medicine 1992; 85: 420–1. [CT is useful in the diagnosis of rectus-sheath
hematoma.]
3. Dombal FT. Diagnosis of acute abdominal pain, 2nd edn. Churchill Living-
stone, Edinburgh, 1991. [A compilation of scientific studies of abdominal pain
completed over 20 years.]
4. Adams ID et al. Computer aided diagnosis of acute abdominal pain: a multi-
centre study. British Medical Journal 1986; 293: 800–4. [A study of 16 737 pa-
tients confirms that computer aided diagnosis improves diagnostic accuracy,
decision-making and patient outcome.]
5. Jones PF. Emergency abdominal surgery, 2nd edn. Blackwell Scientific Publi-
cations, Oxford, 1987. [A comprehensive text with additional information
about the management of children with abdominal pain.
6. Williamson RCN, Cooper MJ, ed. Emergency abdominal surgery. Vol.17,
Clinical surgery international. Churchill Livingstone, Edinburgh, 1990. [Multi-
ple chapters that focus on the surgical treatment of the acute abdomen.]

Tests for initial level of knowledge, keys for tests:

1. Which of the following statements regarding the etiology of obstructive jaundice is


true?
A. A markedly elevated SGOT and SGPT are usually associated with
obstructive jaundice
B. When extra hepatic biliary obstruction is suspected, the first test
should be endoscopic ultrasonography (EUS)
C. A Klatskin tumor will result in extra hepatic ductal dilation only
D. A liver-spleen scan will add significantly to the diagnostic workup for
obstructive jaundice
E. Carcinoma of the head of the pancreas can cause deep epigastria or
back pain in as many as 80% of patients

2. A previously healthy 15-yearold boy is brought to the emergency room with com-
plaints of about 12 h of progressive anorexia, nausea, and pain of the right lower
quadrant. On physical examination, he is found to have a rectal temperature of 38.8C
(and has direct and rebound abdominal tenderness localizing to McBurney’s point as
well as involuntary guarding in the right lower quadrant. At operation through a
McBurney-type incision, the appendix and caecum are found to be normal, but the
surgeon is impressed with the marked edema of the terminal ileum, which also has an
overlying fibrin purulent exudates. The correct procedure is to
A. Close the abdomen after culturing the exudates
B. Perform a standard appendectomy
C. Resects the involved terminal ileum
D. Perform the ileocolic resection
E. Perform an ileocolostomy to bypass the involved terminal ileum

3. A 55-year-old man who is extremely obese reports weakness, sweating, tachycar-


dia, confusion, and headache whenever he fasts for more than a few hours. He has
prompt relief of symptoms when he eats. These symptoms are most suggestive of
which of the following disorders?
A. Diabetes mellitus
B. Insulinoma
C. Zollinger-Ellison syndrome
D. Carcinoid syndrome
E. Multiple endocrine neoplasia

4. An 80-year-old man is admitted to the hospital complaining of nausea, abdominal


pain, distention, and diarrhea. A cautiously performed transanal contrast study reveals
an “apple core” configuration in the recto sigmoid. Appropriate management at this
time would include
A. Colonoscopic decompression and rectal tube placement
B. Saline enemas and digital disimpaction of fecal matter from the rec-
tum
C. Colon resection and proximal colostomy
D. Oral administration of metronidazole and checking a Clostridium dif-
ficile titer
E. Evaluation of an electrocardiogram and obtaining an angiogram to
evaluate for colonic mesenteric ischemia

5. Which of the following statements regarding appendicitis during pregnancy is cor-


rect?
A. Appendicitis is the most prevalent extra uterine indication for celi-
otomy during pregnancy
B. Appendicitis occurs more commonly in pregnant women than in non-
pregnant women of comparable age
C. Suspected appendicitis in a pregnant woman should be managed with
a period of observation of due to the risks of laparotomy to the fetus
D. Noncomplicated appendicitis results in a 20% fetal mortality and
premature labor rate
E. The severity of appendicitis correlates with increased gestational age
of the fetus

6. A 32-year-old woman presents to the hospital with a 24-h history of abdominal


pain of the right lower quadrant. She undergoes an uncomplicated appendectomy for
acute appendicitis and is discharged home on the fourth postoperative day. The
pathologist notes the presence of a carcinoid tumor (1.2 cm) in the tip of the appen-
dix. Which of the following statements is true?
A. The patient should be advised to undergo ileocolectomy
B. The most common location of carcinoids is in the appendix
C. The carcinoid syndrome occurs in more than half the patients with
carcinoid tumors
D. The tumor is an apudoma
E. Carcinoid syndrome is seen only when the tumor is drained by the
portal venous system

7. Which of the following statements regarding direct inguinal hernias is true?


A. They are the most common inguinal hernias in women
B. They protrude medially to the inferior epigastria vessels
C. They should be opened and ligated at the internal ring
D. They commonly protrude into the scrotal sac in men
E. They incarcerate more commonly than indirect hernias

8. An 88-year-old man with a history of end-stage renal failure, severe coronary ar-
tery disease, and brain metastases from lung cancer presents with acute cholecystitis.
His family wants “everything done.” The best management option in this patient
would be:
A. Tube cholecystostomy
B. Open cholecystectomy
C. Laparoscopic cholecystectomy
D. Intravenous antibiotics followed by elective cholecystectomy
E. Lithotripsy followed by long-term bile acid therapy

9. Which statement concerning cholangitis is correct?


A. The most common infecting organism is Staphylococcus aureus
B. The diagnosis is suggested by the Charcot triad
C. The disease occurs primarily in young, immunocompromised patients
D. Cholecystostomy is the procedure of choice in affected patients
E. Surgery is indicated once the diagnosis of cholangitis is made

10. A 28-year-old previously healthy woman arrives in the emergency room com-
plaining of 24 h of anorexia and nausea and lower abdominal pain that is more in-
tense in the right lower quadrant than elsewhere. On examination she has peritoneal
signs of the right lower quadrant and a rectal temperature of 38.38C. At
exploration through incision of the right lower quadrant, she is found to have a small,
contained perforation of a cecal diverticulum. Which of the following statements re-
garding this situation is true?
A. Cecal diverticula are acquired disorders
B. Cecal diverticula are usually multiple
C. Cecal diverticula are mucosal herniations through the muscularis pro-
pria
C. Diverticulectomy, closure of the cecal defect, and appendectomy may
be indicated
E. An ileocolectomy is indicated even with well-localized inflammation

Keys for tests


1 2 3 4 5 6 7 8 9 10
B B B C A D B A D A

Tests for final level of knowledge, keys for tests:

1. A 59-year-old man arrives in the ER at 2 AM, accompanied by his wife who


is wearing curlers on her hair and a robe over her nightgown. He has abdominal pain
that began suddenly about 1 hour ago, and is now generalized, constant, and extreme-
ly severe. He lies motionless on the stretcher, is diaphoretic, and has shallow, rapid
breathing. His abdomen is rigid, very tender to deep palpation, and has guarding and
rebound tenderness in all quadrants.
A. Definitely an acute abdomen.
B. Right-side lobar pneumonia
C. Cirrhosis of the liver
D. Peritonism
E. Kidney colic

2. A 62-year-old man with cirrhosis of the liver and ascites presents with generalized
abdominal pain that started 12 hours ago. He now has moderate tenderness over the
entire abdomen, with some guarding and equivocal rebound. He has mild fever and
leukocytosis. What is it?
A. Acute mesenteric ischemia
B. Perforated duodenal ulcer
C. Peritonitis in the cirrhotic with ascites
D. Acute pancreatitis
E. Acute appendicitis

3. A 43-year-old man develops excruciating abdominal pain at 8:18 PM. When


seen in the ER at 8:50 PM, he has a rigid abdomen, lies motionless on the
examining table, has no bowel sounds, and is obviously in great pain, which
he describes as constant. X-ray shows free air under the diaphragm. What is it?

A. Acute cholangitis
B. Perforated duodenal ulcer
C. Acute diverticulitis
D. Acute appendicitis
E. Acute pancreatitis

4. A white, obese 40-year-old mother of five children gives a history of repeated epi-
sodes of right upper quadrant abdominal pain brought about by the ingestion of fatty
foods, and relieved by the administration of anticholinergic medications. The pain is
colicky, radiates to the right shoulder and around to the back, and is accompanied by
nausea and occasional vomiting. Physical examination is unremarkable. What is it?

A. Perforated duodenal ulcer B.


Acute pancreatitis C.
Acute mesenteric ischemia D.
Gallstones, with biliary colic E. Cir-
rhosis of the liver

5. A 43-year-old obese mother of six children has severe right upper quadrant
abdominal pain that began 6 hours ago. The pain was colicky at first, radiated
to the right shoulder and around toward the back, and was accompanied by
nausea and vomiting. For the past 2 hours the pain has been constant. She
has tenderness to deep palpation, muscle guarding, and rebound in the right
upper quadrant. Her temperature is 101 °F, and she has a WBC count of 16,000.
She has had similar episodes of pain in the past brought about by ingestion of
fatty food, but they all had been of brief duration and relented spontaneously
or with anticholinergic medications. What is it?

A. Acute pancreatitis B.
Acute cholecystitis C.
Acute appendicitis D. Per-
forated duodenal ulcer E. Acute
mesenteric ischemia
6. A 59-year-old man is referred for evaluation because he has been fainting at his
job where he operates heavy machinery. He is pale and gaunt, but otherwise his phys-
ical examination is remarkable only for 4+ occult blood in the stool. Lab shows a he-
moglobin_of_50_g/l. What is it?
A. Acute cholecystitis B.
Acute pancreatitis
C. Acute appendicitis D.
Perforated duodenal ulcer E.
Cancer of the right colon

7. A 59-year-old woman has a history of three prior episodes of left lower quadrant
abdominal pain for which she was briefly hospitalized and treated with antibiotics.
She began to feel discomfort 12 hours ago, and now she has constant left lower
quadrant pain, tenderness, and a vaguely palpable mass. She has fever and leukocy-
tosis. What is it?
A. Acute cholecystitis B.
Perforated duodenal ulcer C.
Acute diverticulitis D.
Acute mesenteric ischemia E.
Acute appendicitis

8. A 79-year-old man with atrial fibrillation develops an acute abdomen. He has a


silent abdomen, with diffuse tenderness and mild rebound. There is a trace of blood
in the rectal exam. He has acidosis and looks quite sick. X-rays show distended
small bowel and distended colon up to the middle of the transverse colon. What is
it?
A. Acute pancreatitis
B. Perforated duodenal ulcer
C. Cancer of the right colon
D. Embolic occlusion of the mesenteric vessels
E. Hepatocellular jaundice

9. A 19-year-old college student returns from a trip to Cancun, and 2 weeks later de-
velops malaise, weakness, and anorexia. A week later he notices jaundice. When he
presents for evaluation his total bilirubin are 120, with 70 indirect and 50
direct. His alkaline phosphatase is mildly elevated, and the SGOT and SGPT
(transaminases) are very high. What is it?

A. Hepatocellular jaundice B.
Acute cholecystitis
C. Perforated duodenal ulcer
D. Acute edematous pancreatitis
E. Acute mesenteric ischemia

10 A 33-year-old alcoholic man shows up in the ER with epigastria and


midabdominal pain that began 12 hours ago shortly after the ingestion of a large
meal. The pain is constant and very severe, and radiates straight through to the back.
He vomited twice early on, but since then has continued to have retching. He has
tenderness and some muscle guarding in the upper abdomen, is afebrile, and has
mild tachycardia. Serum amylase is 6, 5, and his hematocrit is 52%. What is it?

A. Acute cholecystitis B.
Acute mesenteric ischemia
C. Perforated duodenal ulcer
D. Gallstones, with biliary colic
E. Acute edematous pancreatitis

Keys for tests

1 2 3 4 5 6 7 8 9 10
A C B D B E C D A E

Tasks for the final level of knowledge.

1. A 57-year-old alcoholic man is being treated for acute hemorrhagic


pancreatitis. He was in the ICU for 1 week, required chest tubes for pleural
effusion, and was on a respirator for several days, but eventually improved
enough to be transferred to the floor. Two weeks after the onset of the disease,
he begins to spike fever and to demonstrate leukocytosis. What is it?

The answer is pancreatic abscess


2. Four days after exploratory laparotomy for blunt abdominal trauma with resec-
tion and reanastomosis of damaged small bowel, a patient has abdominal disten-
tion, without abdominal pain. He has no bowel sounds and has not passed flatus,
and his abdominal x-rays show dilated loops of small bowel without air fluid lev-
els. What is it?
The answer is paralytic ileus

3. An elderly gentleman with Alzheimer's disease who lived in a nursing home is op-
erated on for a fractured femoral neck. On the fifth postoperative day it is noted that
his abdomen is grossly distended and tense, but not tender. He has occasional bowel
sounds. X-rays show a much distended colon, and a few distended loops of small
bowel. What is it?

The answer is massive colonic dilatation

4. A 56-year-old man has bloody bowel movements. The blood coats the outside of
the stool, and has been present on and off for several weeks. For the past 2 months he
has been constipated, and his stools have become of narrow caliber. What is it?

The answer is cancer of the distal, left side of the colon.

5. A 42-year-old man has suffered from chronic ulcerative colitis for 20 years. He
weighs 90 pounds and has had at least 40 hospital admissions for exacerbations of the
disease. Because of a recent relapse, he has been placed on high-dose steroids and
Imuran. For the past 12 hours he has had severe abdominal pain, temperature of
104°F, and leukocytosis. Helooksilland"toxic." His abdomen is tender, particularly in
the epigastria area, and he has muscle guarding and rebound. X-rays show a massive-
ly distended transverse colon, and there is gas within the wall of the colon. What is it?

The answer is toxic megacolon.

Materials on the independent teaching of students

Main tasks Notes (instructions)


Repeat:
1. Anatomy of organs of abdominal
cavity, sectoral structure of liver. Anatomy
and features of blood circulation of the
spleen. -To represent the methods of diagnostics of dis-
2. Physiology peritoneum of abdominal eases of abdominal cavity as a table
cavity and pelvic peritoneum -To make the flow diagram of mechanisms of
3. Pathogenesis of abdominal disease damage of abdomen
4. Morphological changes in the organs
of abdominal cavity at disease
5. Clinical displays and methods of di-
agnostics of peritonitis
Study guide #6.1
“Local and widespread inflammatory diseases of abdominal cavity organs and
peritoneum. Primary, secondary and tertiary peritonitis. Etiology, pathogenesis,
clinical picture, diagnosis and treatment.”

Overview.
Intra-abdominal bacterial infection usually presents as one of three syndromes:
(a) spontaneous bacterial peritonitis, which typically occurs in patients with preexist-
ing ascites; (b) secondary peritonitis resulting from an identifiable preceding cause
such as a perforated organ; or (c) intra-abdominal abscess, commonly a postoperative
complication. Polymicrobial infection with both Gram-negative aerobic enteric bacilli
(e.g., Escherichia coli) and anaerobes (e.g., Bacteroides fragilis) is regularly present
except in spontaneous bacterial peritonitis, which is most often caused by a single
isolate such as E. coli or Streptococcus pneumoniae. Although the diagnosis of sec-
ondary peritonitis or intra-abdominal abscess is usually apparent clinically, occasion-
ally paracentesis or laparotomy may be initially required to confirm the diagnosis.
The diagnosis of intra-abdominal abscess has been facilitated by ultrasonography,
computed tomography, and radionuclide scanning. Percutaneous guided-needle aspi-
ration can confirm the diagnosis and simultaneously provide successful drainage in
selected cases. Treatment consists of prompt and adequate drainage combined with
effective antimicrobial therapy.
Educational aims:
9. Interrogation and clinical inspection of patients with the peritonitis.
10. To determine the etiologic and pathogenic factors of different forms of peritonitis.
11. To find out the clinical features of the intra abdominal sepsis.
12. To develop a plan of examination of the patients with the abdominal infection.
13. To estimate results of physical examination, laboratory tests, ultrasonography and
X-ray examination in patients with abdominal sepsis.
14. To make a differential diagnosis between peritonitis and other forms of acute ab-
domen.
15.To determine the indications for treatment of patients with the peritonitis
16.To compare the different surgical approaches to the treatment of diffuse and local-
ized peritonitis.
17.To provide the postoperative care after surgery.
A student must know:
12. Аnatomy of the abdominal cavity.
13. Classification of peritonitis.
14. Mechanisms of the development of the intra abdominal infection and inflamma-
tion of the peritoneum.
15. Clinical presentations of different form of peritonitis.
16. Methods of diagnostics of abdominal infection.
17. Differential diagnosis of the peritonitis with other urgent abdominal conditions.
18. Surgical approaches to the management of the peritonitis.
A student must be able to:
8. Collect and estimate the complaints of patient with abdominal infection, gather
anamnesis, conduct physical research and interpret the received results.
9. Define the rational quantity of laboratory and instrumental research methods.
10. Be able to examine patients with peritonitis.
11. Define the indications to surgical interventions and choose the appropriate surgi-
cal method.
12. To prescribe preoperative preparation depending on patient’s state.

Terminology
Term Definition
Is an inflammatory state affecting the whole body, fre-
quently in response of the immune system to infection, but
Systemic inflammatory
not necessarily so; it is related to sepsis, a condition in
response syndrome
which individuals both meet criteria for SIRS and have a
known or highly suspected infection.
is spontaneous bacterial peritonitis (SBP) due to chronic
Primary peritonitis
liver disease and ascites
is caused by perforation or necrosis (transmural infection)
Secondary peritonitis of a hollow visceral organ with bacterial inoculation of the
peritoneal cavity
represents the persistence or recurrence of peritoneal infec-
Tertiary peritonitis tion following apparently adequate therapy of SBP or SP,
often without the original visceral organ pathology
Chemical (sterile) peri-
may be caused by irritant substances
tonitis

Content:
History of the Procedure
Untreated cases of acute peritonitis may be fatal. In 1926, the fundamental
role of operative therapy in the treatment of peritonitis was documented. Kirschner
(1926) reported that the mortality rate from intra-abdominal infections decreased
from more than 90% to less than 40% during the period from 1890-1924 with the in-
troduction of operative management as an effective therapeutic modality.
Current treatment of peritonitis and peritoneal abscesses consists of a multi-
modality approach directed at correction of the underlying cause, administration of
systemic antibiotics, and supportive therapy to prevent or limit secondary complica-
tions due to organ system failure.
Inflammation and/or infection of the peritoneal cavity are commonly encoun-
tered problems in the practice of clinical medicine today. In general, the term perito-
nitis refers to a constellation of signs and symptoms, which includes abdominal pain
and tenderness on palpation, abdominal wall muscle rigidity, and systemic signs of
inflammation. Patients may present with an acute or insidious onset of symptoms,
limited and mild disease, or systemic and severe disease with septic shock.
The peritoneum reacts to a variety of pathologic stimuli with a fairly uniform in-
flammatory response. Depending on the underlying pathology, the resultant peritoni-
tis may be infectious or sterile (i.e., chemical or mechanical).
Peritoneal infections are classified as primary (i.e., spontaneous), secondary (i.e., re-
lated to a pathologic process in a visceral organ), or tertiary (i.e., persistent or recur-
rent infection after adequate initial therapy). The intra-abdominal infections are usual-
ly divided into generalized (peritonitis) and localized (intra-abdominal abscess).

Frequency
The overall incidence of peritoneal infections and abscess formation is diffi-
cult to establish and varies with underlying abdominal disease processes.
The most common etiology of primary peritonitis is spontaneous bacterial
peritonitis (SBP) due to chronic liver disease. Approximately 10-30% of all patients
with liver cirrhosis who have ascites develop bacterial peritonitis over time.
The common etiologic entities of secondary peritonitis (SP) include perforat-
ed appendicitis; perforated gastric and duodenal ulcer disease; perforated (sigmoid)
colon caused by diverticulitis, volvulus, or cancer; and strangulation of the small
bowel.
Iatrogenic trauma to the upper GI tract, including the pancreas and biliary
tract and colon, often results from endoscopic procedures; anastomotic dehiscence
and inadvertent bowel injury (e.g., mechanical, thermal) are common causes of leak
in the postoperative period. After elective abdominal operations for noninfectious eti-
ologies, the incidence of SP (caused by anastomotic disruption, breakdown of enter-
otomy closures, or inadvertent bowel injury) should be less than 2%. Operations for
inflammatory disease (i.e., appendicitis, diverticulitis, and cholecystitis) without per-
foration carry a risk of less than 10% for the development of SP and peritoneal ab-
scess. This risk may rise to greater than 50% in gangrenous bowel disease and viscer-
al perforation. After operations for penetrating abdominal trauma, SP and abscess
formation is observed in a small number of patients. Duodenal and pancreatic in-
volvement, as well as colon perforation, gross peritoneal contamination, perioperative
shock, and massive transfusion, are factors that increase the risk of infection in these
cases. Anaerobic microorganisms are found in less than 5% of cases, and multiple
isolates are found in less than 10%.
SP is, by far, the most common form of peritonitis encountered in clinical
practice today. It is caused by perforation or necrosis (transmural infection) of a hol-
low visceral organ with bacterial inoculation of the peritoneal cavity. The spectrum of
pathogens depends to some degree on the site of the original disease. Gram-positive
organisms predominate in the upper GI tract; however, a shift toward gram-negative
organisms may be noticed in patients on long-term gastric acid suppressive therapy.
Contamination from a distal small bowel or colon source initially may result in the
release of several hundred bacterial species (and fungi); host defenses quickly elimi-
nate most of these organisms. The resulting peritonitis is almost always polymicrobi-
al, containing a mixture of aerobic and anaerobic bacteria with a predominance of
gram-negative organisms. As many as 15% of patients who have cirrhosis with asci-
tes who were initially presumed to have SBP have secondary peritonitis. In many of
these patients, clinical signs and symptoms alone are not sensitive or specific enough
to reliably differentiate the 2 entities. A thorough history, evaluation of the peritoneal
fluid, and additional diagnostic tests are needed to establish the correct diagnosis and
treatment in these patients. Peritoneal abscess describes the formation of an infected
fluid collection encapsulated by fibrinous exudate, omentum, and/or adjacent visceral
organs. The overwhelming majority of abscesses occurs subsequent to SP. Approxi-
mately half of patients develop a simple abscess without loculation (the presence of
numerous small spaces or cavities), whereas the other half of patients develop com-
plex abscesses secondary to fibrinous septation and organization of the abscess mate-
rial. Abscess formation occurs most frequently in the subhepatic area, the pelvis, and
the paracolic gutters, but it may also occur in the perisplenic area, the lesser sac, and
between small bowel loops and their mesentery. In general, the incidence of abscess
formation after abdominal surgery is less than 1-2%, even when the operation is per-
formed for an acute inflammatory process. This incidence increases with preoperative
perforation of the hollow viscus, significant fecal contamination of the peritoneal cav-
ity, bowel ischemia, delayed diagnosis and therapy of the initial peritonitis, the need
for reoperation, and in the setting of immunosuppression. In these instances, the risk
of abscess formation may be as high as 10-30%. Overall, abscess formation is the
leading cause of persistent infection and development of tertiary peritonitis. Tertiary
peritonitis represents the persistence or recurrence of peritoneal infection following
apparently adequate therapy of SBP or SP, often without the original visceral organ
pathology. Patients with tertiary peritonitis usually present with an abscess, or phleg-
mon, with or without fistula. Tertiary peritonitis develops more frequently in patients
with significant preexisting comorbid conditions and in patients who are immuno-
compromised. Although rarely observed in uncomplicated peritoneal infections, the
incidence of tertiary peritonitis in patients requiring ICU admission for severe ab-
dominal infections may be as high as 50-74%.
Patients who develop tertiary peritonitis demonstrate significantly longer lengths of
stay in the ICU and hospital, higher organ dysfunction scores, and higher mortality
rates (50-70%). Resistant and unusual organisms (e.g., Enterococcus, Candida,
Staphylococcus, Enterobacter, and Pseudomonas species) are found in a significant
proportion of cases of tertiary peritonitis. Most patients with tertiary peritonitis de-
velop complex abscesses or poorly localized peritoneal infections that are not amena-
ble to percutaneous drainage. Antibiotic therapy appears less effective compared to
all other forms of peritonitis.
Tuberculous peritonitis (TP) is a rare disease in the United States (<2% of all
causes of peritonitis), but it continues to be a significant problem in underdeveloped
countries and among patients with HIV disease. The presenting symptoms are often
nonspecific and insidious in onset (e.g., low-grade fever, anorexia, weight loss).
More than 95% of patients have evidence of ascites on imaging studies, and more
than half of these patients have clinically apparent ascites. Most patients have evi-
dence of cirrhosis, and the diagnosis of TP may be unsuspected. Chest radiograph
findings are abnormal in most patients, but active pulmonary disease is present in
fewer than 30% of patients. Results on Gram stain of ascitic fluid are rarely positive,
and culture results may be falsely negative in up to 80% of patients. A peritoneal flu-
id protein level greater than 2.5 g/dL, lactate dehydrogenase (LDH) level greater than
90 U/mL, or predominantly mononuclear cell count greater than 500 cells/µL should
raise suspicion but has limited specificity for the diagnosis. Laparoscopy and visuali-
zation of granulomas on peritoneal biopsy specimens, as well as positive results on
cultures (requires 4-6 wk) may be needed for the definitive diagnosis; however, em-
piric therapy should begin immediately.
Chemical (sterile) peritonitis may be caused by irritant substances such as
bile, blood, barium, and other substances or by transmural inflammatory processes of
visceral organs (e.g., Crohn’s disease) without bacterial inoculation of the peritoneal
cavity. Clinical signs and symptoms are indistinguishable from those of SP or perito-
neal abscess, and the diagnostic and therapeutic approach should be the same.
Relevant Anatomy
The peritoneum is the largest and most complex serous membrane in the
body. It forms a closed sac (i.e., coelom) by lining the interior surfaces of the ab-
dominal wall (anterior and lateral), by forming the boundary to the retroperitoneum
(posterior), by covering the extraperitoneal structures in the pelvis (inferior), and by
covering the undersurface of the diaphragm (superior). This parietal layer of the peri-
toneum reflects onto the abdominal visceral organs to form the visceral peritoneum. It
thereby creates a potential space between the 2 layers (i.e., the peritoneal cavity). The
peritoneum consists of a single layer of flattened mesothelial cells over loose areolar
tissue. The loose connective tissue layer contains a rich network of vascular and lym-
phatic capillaries, nerve endings, and immune-competent cells, particularly lympho-
cytes and macrophages. The peritoneal surface cells are joined by junctional com-
plexes, thus forming a dialyzing membrane that allows passage of fluid and certain
small solutes. Pinocytotic activity of the mesothelial cells and phagocytosis by mac-
rophages allow for clearance of macromolecules. Normally, the amount of peritoneal
fluid present is less than 50 mL, and only small volumes are transferred across the
considerable surface area in a steady state each day. The peritoneal fluid represents a
plasma ultrafiltrate, with electrolyte and solute concentrations similar to that of
neighboring interstitial spaces and a protein content of less than 30 g/L, mainly albu-
min. In addition, peritoneal fluid contains small numbers of desquamated mesothelial
cells and various numbers and morphologies of migrating immune cells (reference
range is <300 cells/µL, predominantly of mononuclear morphology). The peritoneal
cavity is divided incompletely into compartments by the mesenteric attachments and
secondary retroperitonealization of certain visceral organs. A large peritoneal fold,
the greater omentum, extends from the greater curvature of the stomach and the infe-
rior aspect of the proximal duodenum downward over a variable distance to fold upon
itself (with fusion of the adjacent layers) and ascends back to the taenia omentalis of
the transverse colon. This peritoneal fold demonstrates a slightly different microscop-
ic anatomy, with fenestrated surface epithelium and a large number of adipocytes,
lymphocytes, and macrophages, and it functions as a fat storage location and a mobile
immune organ.
The compartmentalization of the peritoneal cavity, in conjunction with the greater
omentum, influences the localization and spread of peritoneal inflammation and in-
fections.
Pathophysiology
Peritonitis causes a reduction in the intra-abdominal fibrinolytic activity (in-
creased plasminogen activator inhibitor activity) and fibrin sequestration with subse-
quent adhesion formation. The production of fibrinous exudates is considered an im-
portant part of the host defense, but large numbers of bacteria may be sequestered
within the fibrin matrix. This may lead to retardation of spread and systemic dissemi-
nation and may decrease early mortality rates from sepsis, but it also is integral to the
development of residual infection and abscess formation. As the fibrin matrix ma-
tures, the bacteria within are protected from host clearance mechanisms. The ultimate
effect (containment vs. persistent infection) of fibrin may be related to the degree of
peritoneal bacterial contamination. In animal studies of mixed bacterial peritonitis ex-
amining the effects of systemic defibrinogenization and those of abdominal fibrin
therapy, heavy peritoneal contamination uniformly led to severe peritonitis with early
death ( <48 h) because of overwhelming sepsis. Abscess formation has been viewed
as a host defense strategy to contain the spread of infection; however, this process can
lead to persistent infection and life-threatening sepsis. The initiation of abscess for-
mation involves the release of bacteria and an abscess-potentiating agent into a nor-
mally sterile environment. The host defense is unable to eliminate the infecting agent
and attempts to control the spread by compartmentalization. This process is aided by
a combination of factors that share a common feature, i.e., impairment of phagocytot-
ic killing. Some studies suggest that the number of bacteria present at the onset of ab-
dominal infections is much higher than originally believed (approximately 2 X 108
CFU/mL, much higher than the routinely used 5 X 105 CFU/mL inoculated for in
vitro susceptibility testing). This bacterial load may locally overwhelm the host de-
fense. In minimal contamination, bacterial clearance was complete in nearly 100% of
cases, and no differences in outcome were observed among fibrin-depleted, normal,
and fibrin-treated groups. With moderate contamination, fibrin-treated animals
demonstrated a significantly reduced early mortality rate but developed more ab-
dominal abscesses. Finally, studies with adhesion-reducing devices (i.e., bioresorba-
ble membranes) increased the incidence of peritonitis and peritoneal infections in ex-
perimental peritonitis models.
Transient bacterial peritoneal contamination (caused by primary visceral disease and
intentional or unintentional violation of the gut) is common. The resultant exposure to
bacterial antigens has been shown to alter subsequent immune responses to recurrent
peritoneal inoculation. This may lead to an increased incidence of later abscess for-
mation, alteration of the bacterial content, and increased late mortality rates. More re-
cent studies have shown that nosocomial infections at other sites (e.g., pneumonia,
line sepsis, wound infections) also increase the likelihood of subsequent abdominal
abscess formation.
Bacterial virulence factors that interfere with phagocytosis and neutrophil-
mediated bacterial killing are important mediators leading to persistence of infections
and abscess formation. Among these factors are capsule formation, facultative anaer-
obic growth, adhesion capabilities, and succinic acid production. Synergy between
certain bacterial and fungal organisms may also play an important role in impairing
the host's defense. One such synergy may exist between B fragilis and gram-negative
bacteria, particularly E coli, where co-inoculation significantly increases bacterial
proliferation and abscess formation. Enterococci may be important in enhancing the
severity and persistence of peritoneal infections. In animal models of peritonitis with
E coli and B fragilis, the systemic manifestations of the peritoneal infection and bac-
teremia rates were increased, as were bacterial concentrations in the peritoneal fluid
and rate of abscess formation. This is more important in light of the difficulties in
eradicating Enterococcus faecalis with conventional antimicrobial therapy. The role
of Enterococcus organisms in uncomplicated intra-abdominal infections remains un-
clear. Antibiotics that lack specific activity against Enterococcus organisms are often
used successfully in the therapy of peritonitis, and the organism is recovered uncom-
monly as a blood-borne pathogen in intra-abdominal sepsis. The role of fungi in the
formation of intra-abdominal abscesses is not fully understood. Abdominal infections,
particularly with Candida species, are becoming increasingly common in critically ill
patients. Studies suggest that the microbiology of intra-abdominal infections may be
inherently different in severely ill patients. Candida albicans was the organism most
commonly isolated from the peritoneum in critically ill patients with culture-proven
intra-abdominal infections and preoperative APACHE II (acute physiology and
chronic health evaluation) scores greater than or equal to 15, with an associated mor-
tality rate of 52%. Additional common peritoneal organisms in this patient population
were Enterococcus and Enterobacter species and Staphylococcus epidermidis. These
data suggest that broader antimicrobial, and possibly antifungal, coverage may be
warranted in patients with severe abdominal sepsis. Some authors suggest that bacte-
ria and fungi exist as nonsynergistic parallel infections with incomplete competition,
allowing the survival of all organisms. In this setting, treatment of the bacterial infec-
tion alone may lead to an overgrowth of fungi, which may contribute to increased
morbidity. Predisposing factors for the development of abdominal candidiasis include
prolonged use of broad-spectrum antibiotics, gastric acid suppressive therapy, central
venous catheters and intravenous hyperalimentation, malnutrition, diabetes, and ster-
oids and other forms of immunosuppression. Most animal and human studies suggest
that abscess formation occurs only in the presence of abscess-potentiating agents.
Although the nature and spectrum of these factors has not been studied exhaustively,
certain fiber analogues (e.g., bran) and the contents of autoclaved stool have been
identified as such abscess-potentiating agents. In animal models, these factors inhibit-
ed opsonization and phagocytotic killing by interference with complement activation.
The role of cytokines in mediation of the body's immune response and their role in
the development of the systemic inflammatory response syndrome (SIRS) and multi-
ple organ failure (MOF) have been a major focus of research over the past decade.
Comparatively little data exist about the magnitude of the intraperitoneal/abscess cy-
tokine response and implications for the host. Existing data suggest that bacterial
peritonitis is associated with an immense intraperitoneal compartmentalized cytokine
response. Higher levels of certain cytokines (i.e., tumor necrosis factor-alpha [TNF-
alpha], interleukin [IL]-6) have been associated with worse outcomes, as well as sec-
ondary (uncontrolled) activation of the systemic inflammatory cascade.
Presentation
The diagnosis of peritonitis is usually clinical. Essentially, all patients present
with some degree of abdominal pain. This pain may be acute or more insidious in on-
set. Initially, the pain is often dull and poorly localized (visceral peritoneum) and then
progresses to steady, severe, and more localized pain (parietal peritoneum). If the in-
fectious process is not contained, the pain becomes diffuse. In certain disease entities
(e.g., gastric perforation, severe acute pancreatitis, intestinal ischemia), the abdominal
pain may be generalized from the beginning.
Anorexia and nausea are frequently present and may precede the development
of abdominal pain. Vomiting may occur because of the underlying visceral organ pa-
thology (i.e., obstruction) or secondary to the peritoneal irritation.
On physical examination, patients with peritonitis most often appear unwell
and in acute distress. Fever with temperatures that can exceed 38°C is usually present,
but patients with severe sepsis may present with hypothermia. Tachycardia is caused
by the release of inflammatory mediators and intravascular hypovolemia caused by
anorexia and vomiting, fever, and third-space losses into the peritoneal cavity. With
progressive dehydration, patients may become hypotensive, they may demonstrate
decreased urine output, and, with severe peritonitis. They may present in overt septic
shock.
On abdominal examination, essentially all patients demonstrate tenderness to
palpation. (When examining the abdomen of a patient with peritonitis, the patient
should be supine. A roll or pillows underneath the patient's knees may allow for better
relaxation of the abdominal wall.) In most patients (even with generalized peritonitis
and severe diffuse abdominal pain), the point of maximal tenderness or referred re-
bound tenderness roughly overlies the pathologic process (i.e., the site of maximal
peritoneal irritation).
Nearly all patients demonstrate increased abdominal wall rigidity. The in-
crease in abdominal wall muscular tone may be voluntary in response to or in antici-
pation of the abdominal examination or involuntary because of the peritoneal irrita-
tion. Patients with severe peritonitis often avoid all motion and keep their hips flexed
to relieve the abdominal wall tension. The abdomen is often distended, with hypoac-
tive-to-absent bowel sounds. This finding reflects a generalized ileus and may not be
present if the infection is well localized. Occasionally, the abdominal examination re-
veals an inflammatory mass.
Rectal examination often elicits increased abdominal pain, particularly with
inflammation of the pelvic organs but rarely indicates a specific diagnosis. A tender
inflammatory mass toward the right may indicate appendicitis, and anterior fullness
and fluctuation may indicate a cul de sac abscess.
In female patients, vaginal and bimanual examination may lead to the differ-
ential diagnosis of pelvic inflammatory disease (e.g., endometritis, salpingo-
oophoritis, tubo-ovarian abscess), but the findings are often difficult to interpret in
severe peritonitis.
When evaluating the patient with suspected peritoneal infection, performing a
complete physical examination is important. Thoracic processes with diaphragmatic
irritation (e.g., empyema), extraperitoneal processes (e.g., pyelonephritis, cystitis,
acute urinary retention), and abdominal wall processes (e.g., infection, rectus hema-
toma) may mimic certain signs and symptoms of peritonitis. Always examine the pa-
tient carefully for the presence of external hernias to rule out intestinal incarceration.
Remember that the presentation and the findings on clinical examination may be en-
tirely inconclusive or unreliable in patients with significant immunosuppression (e.g.,
severe diabetes, steroid use, post transplant status, HIV), in patients with altered men-
tal state (e.g., head injury, toxic encephalopathy, septic shock, analgesic agents), in
patients with paraplegia, and in patients of advanced age. With localized deep perito-
neal infections, fever and/or an elevated WBC count may be the only signs present.
As many as 20% of patients with SBP demonstrate very subtle signs and symptoms.
New onset or deterioration of existing encephalopathy may be the only sign of the in-
fection at the initial presentation. Most patients with TP demonstrate only vague
symptoms and may be afebrile.
Laboratory Studies
• CBC with differential - Most patients will have leukocytosis (>11,000
cells/µL), with a shift to the immature forms on the differential cell count. Pa-
tients with severe sepsis, who are immunocompromised, or who have certain
types of infections (e.g., fungal, cytomegalovirus) may demonstrate absence of
leukocytosis or leukopenia. In cases of suspected SBP, hypersplenism may re-
duce the polymorphonuclear leukocyte count.
• Blood chemistry - May reveal dehydration and acidosis
• PT, PTT, and INR
• Liver function tests - If clinically indicated
• Amylase and lipase - If pancreatitis is suspected
• Urinalysis (UA) - To rule out urinary tract diseases (e.g., pyelonephritis, renal
stone disease); however, patients with lower abdominal and pelvic infections
often demonstrate WBCs in the urine and microhematuria.
• Stool sample - In patients with diarrhea, evaluate a stool sample — employing
a Clostridium difficile toxin assay, a WBC count, and a specific culture (i.e.,
Salmonella, Shigella, cytomegalovirus [CMV]) — if the patient's history sug-
gests infectious enterocolitis.
• Aerobic and anaerobic blood cultures
• Peritoneal fluid (i.e., paracentesis, aspiration of abdominal fluid collections, in-
traoperative peritoneal fluid cultures)
o Diagnostic peritoneal lavage (DPL) may be helpful in patients who do
not have conclusive signs on physical examination or who cannot pro-
vide an adequate history. A DPL with more than 500 leukocytes/mL is
considered positive and suggests peritonitis.
o Evaluate the sample for pH, glucose, protein, lactate dehydrogenase
(LDH), cell count, Gram stain, and aerobic and anaerobic cultures.
o Include analysis if pancreatitis or pancreatic leak is suspected.
o Test for bilirubin when a biliary leak is suspected and for fluid creatinine
level when a urinary leak is suspected.
o Compare the peritoneal levels to the respective serum levels.
The fluid in bacterial peritonitis generally demonstrates low pH and glucose as well
as elevated protein and LDH levels. A fluid pH lower than 7.1 (and partial pressure of
oxygen [PO2] <49 mm Hg) has demonstrated positive and negative predictive values
of greater than 98% in some studies (median pH of 6.75 versus 7.49 for elective sur-
gery, with PO2 28 versus 144 mm Hg). The drop in peritoneal fluid pH (and PO2) is
more pronounced in mixed infections and severe bacterial contamination, with in-
creased numbers of anaerobic bacteria in these circumstances.
In SBP, a peritoneal fluid WBC count of more than 250 cells/µL (>500 in some stud-
ies), with more than 50% polymorphonuclear leukocytes (PMNs) is an indication to
begin antibiotic therapy. Although up to 30% of culture findings remain negative in
these patients, most of these patients are presumed to have bacterial peritonitis; they
should be treated. A significantly decreased peritoneal fluid glucose level (<50
mg/dL), a peritoneal fluid LDH level much greater than the serum LDH, a peritoneal
fluid WBC count greater than 10,000 cells/µL, a pH lower than 7.0, high amylase
levels, multiple organisms on Gram stain, or recovery of anaerobes from the culture
raises the suspicion of SP in these patients. Some authors recommend repeating the
paracentesis in 48-72 hours to monitor treatment success (decrease in neutrophil
count to <50% of the original value). In TP, the fluid Gram stain and acid-fast stain
results are rarely positive, and routine culture findings are falsely negative in as many
as 80% of cases. A peritoneal fluid protein level greater than 2.5 g/dL, LDH level
greater than 90 U/mL, and predominantly mononuclear cell count of more than 500
cells/µL should raise the suspicion of TP, but specificity for the diagnosis is limited.
Laparoscopy with visualization of granulomas on peritoneal biopsy and specific cul-
ture (requires 4-6 wk) may be needed for definitive diagnosis. Routine intraoperative
peritoneal fluid cultures in defined acute disease entities (i.e., gastric or duodenal ul-
cer perforation, appendicitis, diverticulitis or perforation of the colon caused by ob-
struction or ischemia) are controversial. Several studies found no significant differ-
ence in patients with appendicitis, diverticulitis, and other common etiologies for bac-
terial peritonitis with regard to postoperative complication rates or overall outcomes.
The antibiotic regimen was altered only 8-10% of the time based on operative culture
data. In patients who had previous abdominal operations or instrumentation (e.g., per-
itoneal dialysis catheter, percutaneous stents) and patients with prolonged antibiotic
therapy, critical illness, and/or hospitalization, these cultures may reveal resistant or
unusual organisms that should prompt alteration of the antibiotic strategy.
Imaging Studies
Radiographs
o Plain films of the abdomen (e.g., supine, upright, and lateral decubitus positions)
are often the first imaging studies obtained in patients presenting with peritonitis.
Their value in reaching a specific diagnosis is limited.
o Free air is present in most cases of anterior gastric and duodenal perforation but is
much less frequent with perforations of the small bowel and colon and is unusual
with appendiceal perforation. Upright films are useful for identifying free air un-
der the diaphragm (most often on the right) as an indication of a perforated viscus.
Remember that the presence of free air is not mandatory with visceral perforation
and that small amounts of free air are missed easily on plain films.
Ultrasonography
o Abdominal ultrasonography may be helpful in the evaluation of right upper quad-
rant (e.g., perihepatic abscess, cholecystitis, biloma, pancreatitis, pancreatic pseu-
docyst), right lower quadrant, and pelvic pathology (e.g., appendicitis, tubo-
ovarian abscess, Douglas pouch abscess), but the examination is sometimes lim-
ited because of patient discomfort, abdominal distension, and bowel gas interfer-
ence.
o Ultrasonography may detect increased amounts of peritoneal fluid (ascites), but
its ability to detect quantities of less than 100 mL is limited. The central (perimes-
enteric) peritoneal cavity is not visualized well with transabdominal ultrasonogra-
phy. Examination from the flank or back may improve the diagnostic yield, and
providing the ultrasound technician with specific information of the patient's con-
dition and the suspected diagnosis before the examination is important. With an
experienced ultrasound technician, a diagnostic accuracy of greater than 85% has
been reported in several series.
o Ultrasonographically guided aspiration and placement of drains has evolved into a
valuable tool in the diagnosis and treatment of abdominal fluid collections.
CT scanning
o If the diagnosis of peritonitis is made clinically, a CT scan is not necessary and
generally delays surgical intervention without offering clinical advantage. CT
scans of the abdomen and pelvis remain the diagnostic study of choice for perito-
neal abscess and related visceral pathology. CT scanning is indicated in all cases
in which the diagnosis cannot be established on clinical grounds and findings on
abdominal plain films. Whenever possible, the CT scan should be performed with
enteral and intravenous contrast. CT scans can detect small quantities of fluid, ar-
eas of inflammation, and other GI tract pathology, with sensitivities that approach
100%.
Treatment
Medical Therapy
The general principles guiding the treatment of intra-abdominal infections are 4-fold:
(1) to control the infectious source, (2) to eliminate bacteria and toxins, (3) to main-
tain organ system function, and (4) to control the inflammatory process. The treat-
ment of peritonitis is multidisciplinary, with complimentary application of medical,
operative and nonoperative interventions included in the therapy. Medical support in-
cludes (1) systemic antibiotic therapy; (2) intensive care with hemodynamic, pulmo-
nary, and renal support; (3) nutrition and metabolic support; and (4) inflammatory re-
sponse modulation therapy. Early control of the septic source is mandatory and can
be achieved by operative and nonoperative means. Nonoperative interventional thera-
pies include percutaneous drainage of abscesses and percutaneous and endoscopic
stent placements. Treatment of peritonitis and intra-abdominal sepsis always begins
with volume resuscitation, correction of potential electrolyte and coagulation abnor-
malities, and empiric broad-spectrum parenteral antibiotic coverage.

Antibiotic therapy
A. Spontaneous bacterial peritonitis
Untreated SBP has a mortality rate of up to 50%, but with prompt diagnosis
and treatment of the condition, this figure may be reduced to 20%. Empiric therapy
with a third-generation cephalosporin must begin promptly and can subsequently be
narrowed according to the culture results. Avoid aminoglycosides in patients with
liver disease, because these patients are at an increased risk for nephrotoxicity. The
optimal duration of therapy is not known; traditionally, a course of 10 days is recom-
mended, although studies have suggested that 5 days of therapy (with documentation
of a decrease of peritoneal fluid WBC count to <250 cells/μ L) may be sufficient in
most cases. The patient with SBP is also likely to require attention to changes in he-
modynamic function related to inflammatory pathways, as well as resultant renal
function impairment, although a discussion of this is beyond the scope of this chapter.
There is a high risk of relapse after SBP (40-70% in 12 months); a variety of prophy-
lactic antibiotic regimens are available. A preliminary study of Norfloxacin for pri-
mary prophylaxis of SBP was positive.

B. Secondary and tertiary peritonitis


In secondary and tertiary peritonitis, systemic antibiotic therapy is the second
mainstay of treatment. Several studies suggest that antibiotic therapy is not as effec-
tive in the infection's later stages and that early (preoperative) systemic antibiotic
therapy can significantly reduce the concentration and growth rates of viable bacteria
in the peritoneal fluid. Antibiotic therapy begins with empiric coverage (effective
against common gram negative and anaerobic pathogens) and should be initiated as
soon as possible, with a transition made to narrower spectrum agents as culture re-
sults become available. Perforations of upper GI tract organs are associated with
gram-positive bacteria, whereas the distal small bowel and colon perforations involve
polymicrobial aerobic and anaerobic species. Culture results may be especially im-
portant in tertiary peritonitis, which is more likely to involve gram-positive bacteria
(enterococci); antibiotic-resistant, gram-negative bacteria; and yeast. In community-
acquired infections, a second- or third-generation cephalosporin or a quinolone with
or without metronidazole provides adequate coverage, as do broad-spectrum penicil-
lins with anaerobic activity (i.e., ampicillin/sulbactam) and newer quinolones (i.e.,
trovafloxacin, clinafloxacin). Most studies suggest that single-drug therapy is as ef-
fective as dual or triple combination therapy in mild to moderate abdominal infec-
tions. For peritoneal dialysis – associated infections, Cochrane reviews of all pub-
lished randomized, controlled trials have not found significant differences between
antimicrobial agents or combinations, with similar response and relapse rates for gly-
copeptide regimens and first-generation cephalosporins. Intraperitoneal antibiotics
had a lower failure rate than intravenous regimens. Risk for early peritonitis is re-
duced with perioperative intravenous antibiotics; other prophylactic approaches are
not yet proven. In severe and hospital-acquired intra-abdominal infections, imipenem,
piperacillin/tazobactam, and a combination of aminoglycosides and metronidazole are
often effective. A study of nearly 400 patients documented that ertapenem, a novel
carbapenem with a half-life that allows once-a-day dosing, was effective (86.7% suc-
cess rate) compared to piperacillin/tazobactam (81.2% success rate) in the treatment
of complicated intra-abdominal infection and was well tolerated. Additional clinical
antimicrobial studies are underway investigating the efficacy of new quinolones in
the treatment of intra-abdominal infection. With persistence of the infection (i.e., ter-
tiary peritonitis) and prolonged critical illness, obtaining peritoneal fluid and/or ab-
scess cultures with sensitivities at operation or drainage is important to properly treat
unusual (e.g., gram-positive organisms, fungi) and resistant organisms (e.g., Entero-
coccus, Staphylococcus, Pseudomonas, resistant Bacteroides, and Candida species).
Certain preexisting conditions, immunocompromised, gastric acid suppression thera-
py, and recent antibiotic use may also influence the spectrum of microorganisms.
Consultation with infectious disease specialists is warranted in these cases. The opti-
mal duration of antibiotic therapy must be individualized and depends on the underly-
ing pathology, severity of infection, speed and effectiveness of source control, and
patient response to therapy. In uncomplicated peritonitis in which there is early, ade-
quate source control, a course of 5-7 days of antibiotic therapy is adequate in most
cases. Mild cases (e.g., early appendicitis, cholecystitis) may not need more than 24-
72 hours of postoperative therapy. Inadequate initial therapy has been linked to worse
outcomes, and these outcomes could not be significantly changed by later specific or
prolonged therapy. Antimicrobial therapy should continue until signs of infection
(e.g., fever, leukocytosis) have resolved; when signs of infection continue, persistent
infection or the presence of a nosocomial infection should be investigated. Some pa-
tients demonstrate persistent signs of inflammation without a defined infectious fo-
cus. In these patients, continued broad-spectrum antibiotic therapy may be more
harmful than beneficial (e.g., emergence of resistant organisms, C difficile colitis),
and a trial of antibiotic therapy cessation under close surveillance may be warranted.
Complicated persistent infections and infections in patients who are immunocom-
promised may warrant a prolonged course of antibiotic therapy. In these cases, con-
tinuously seeking and aggressively treating all new extraperitoneal and new or persis-
tent intra-abdominal sources is important. The length of the individual course of
treatment is variable and is often linked to signs of resolution of the inflammatory
process (e.g., lack of fever for >24-48 h, return of the WBC count to reference range
levels). Of note, antibiotics alone are seldom sufficient to treat intra-abdominal ab-
scesses, and adequate drainage of the abscess is of paramount importance. For most
of the commonly used antibiotics, abscess fluid antibiotic levels are generally below
the minimum inhibitory concentration-90 (MIC90) for B fragilis and E coli, and re-
peated dosing or high-dose therapy does not improve penetration significantly.

Nonoperative drainage
CT scan – and ultrasound guided percutaneous drainage are well established
as effective source controls and may in some cases decrease the need for surgical
therapy. In some instances, success also includes the ability to delay surgery until the
acute process and sepsis are resolved and a definitive procedure can be performed
under elective circumstances. For primary percutaneous management of intra-
abdominal abscesses, the etiology, location, and morphology of the abscess must be
defined; evaluate for the presence of an ongoing enteric leak or fistula formation.
With proper indication, most studies have reported success rates of greater than 80%
(range 33-100%) for drainage of localized nonloculated abscesses; however, the suc-
cess rates depend to some degree on the underlying pathology. In these studies, no
significant differences were found between operative and primary nonoperative man-
agement with regard to the overall morbidity or length of hospital stay (mean duration
of drainage 8.5 d). Common reasons for failure of primary nonoperative management
include enteric fistula (e.g., anastomotic dehiscence), pancreatic involvement, infect-
ed clot, and multiple or multiloculated abscesses. Procedure-related significant com-
plications are reported to occur in less than 10% of cases (range 5-27%), with less
than a 1% attributable mortality rate with experienced physicians. In peritoneal ab-
scess formation caused by subacute bowel perforation (e.g., diverticulitis, Crohn’s
disease, appendicitis), primary percutaneous management with percutaneous drainage
was successful in most patients. Patients with Crohn’s disease whose abscesses were
drained percutaneously had significantly fewer associated fistulae. Failure in these
patients was related to preexisting fistulization and extensive stricture formation.
Concerns regarding the transgression of small or large bowel with drainage catheters
in deep abscesses or ileus have been addressed in animal studies, which have found
no increase in abscess formation, independent of whether catheters remained for 5
days or longer. Similar data are not available for human patients. In summary, percu-
taneous and surgical drainage should not be considered competitive but rather com-
plementary. If an abscess is accessible to percutaneous drainage and the underlying
visceral organ pathology does not clearly require an operative approach, percutaneous
drainage can be used safely and effectively as the primary treatment modality. In
these cases, patients must be closely monitored, and improvement should be observed
in less than 24-48 hours. With lack of improvement, patients must be reevaluated ag-
gressively (e.g., repeat CT scan) and the therapeutic strategy should be altered ac-
cordingly.
Surgical Therapy
Surgery remains a cornerstone of treating peritonitis. Any operation should
address the first 2 principles of the treatment of intra-peritoneal infections: early and
definitive source control and elimination of bacteria and toxins from the abdominal
cavity. The issue of timing and adequacy of surgical source control is paramount be-
cause an improper, untimely, or incorrect operation may have an overwhelmingly
negative effect on outcome (compared to medical therapy). The operative approach is
directed by the underlying disease process and the type and severity of the intra-
abdominal infection. In many cases, the indication for operative intervention will be
clear, as in cases of peritonitis caused by ischemic colitis, a ruptured appendix, or co-
lonic diverticula. The surgeon should always strive to arrive at a specific diagnosis
and delineate the intra-abdominal anatomy as accurately as possible prior to the oper-
ation. However, in severe abdominal sepsis, delays in operative management may
lead to a significantly higher need for reoperations and to worse outcomes overall;
early exploration (i.e., prior to completion of diagnostic studies) may be indicated.
Surgical intervention may include resection of a perforated viscus with re-
anastomosis or creation of a fistula. To reduce the bacterial load, a lavage of the ab-
dominal cavity is performed, with particular attention to areas prone to abscess for-
mation (e.g., paracolic gutters, subphrenic area). Among the causes of peritonitis,
pancreatitis is unique in several ways. Patients may present with significant ab-
dominal symptoms and a severe systemic inflammatory response, yet they may have
no clear organ-specific indications for emergent exploration. Not all cases of severe
(i.e., necrotizing) pancreatitis and peripancreatic fluid collections are associated with
a superinfection. These patients may best be served by a period of 12-24 hours of ob-
servation and intensive medical support. Deterioration of the patient's clinical status
or development of organ-specific indications (e.g., intra-abdominal bleed, gas-
forming infection of the pancreas) should lead to prompt operation. Percutaneous
treatment is reserved for the management of defined peripancreatic fluid collections
in stable patients. Pancreatic abscess or infected pancreatic necrosis generally should
be treated with surgical debridement and repeated exploration. If an anastomotic de-
hiscence is suspected, percutaneous drainage is of limited value, and the patient
should be treated surgically.
Open-abdomen technique and scheduled reoperation
In certain situations, staging the operative approach to intraperitoneal infec-
tions is appropriate. Staging may be performed as a scheduled second-look operation
or through open management, with or without temporary closure (e.g., mesh, VAC
technique). Second-look operations may be used in a damage control fashion. In these
cases, the patient at initial operation is severely ill and unstable from septic shock or
coagulopathy (e.g., mediator liberation, disseminated intravascular coagulation). The
goal of the initial operation is to provide preliminary drainage and to remove obvious-
ly necrotic tissue. Then, the patient is resuscitated and stabilized in an ICU setting for
24-36 hours and returned to the operating room for a more definitive drainage and
source control. In conditions related to bowel ischemia, the initial operation aims to
remove all frankly devitalized bowel. The second-look operation serves to reevaluate
for further demarcation and decision-making regarding reanastomosis or diversion. In
severe peritonitis, particularly with extensive retroperitoneal involvement (e.g., ne-
crotizing pancreatitis), open treatment with repeat reexploration, debridement, and in-
traperitoneal lavage has been shown to be effective. Temporary closure of the abdo-
men to prevent herniation and contamination from the outside of the abdominal con-
tents can be achieved using gauze and large, impermeable, self-adhesive membrane
dressings, mesh (e.g., Vicryl, Dexon), nonabsorbable mesh (e.g., GORE-TEX, poly-
propylene) with or without zipper or Velcro-like closure devices, and vacuum-
assisted closure (VAC) devices (see Table 3). Advantages of this management strate-
gy include avoidance of abdominal compartment syndrome (ACS) and easy access
for reexploration. The disadvantages include significant disruption of respiratory me-
chanics and potential contamination of the abdomen with nosocomial pathogens. For
delayed primary closure (permanent), our experience with the use of human acellular
dermis (commercially known as AlloDerm) has been satisfactory, although this op-
tion has the disadvantage of being more expensive than others. The decision to per-
form a series of reexplorations may be made during the initial surgery if additional
debridement and lavage is needed beyond that which can be achieved in the first pro-
cedure. Indications for planned relaparotomy may include failure to achieve adequate
source control, diffuse fecal peritonitis, hemodynamic instability, and intra-abdominal
hypertension. Multiple reoperations may be associated with significant risks, includ-
ing from a substantial inflammatory response, fluid and electrolyte shifts, and hypo-
tension; however, these must be balanced against the risks of persistent necrotic or
infectious abdominal foci. The open-abdomen technique allows for thorough drainage
of the intra-abdominal infection, but the specific indications are not clearly defined.
Many trials lack control groups or use historical controls; outcome variables (e.g.,
mortality) are often not specific enough, and data on resource use are limited. To
date, no conclusive data suggest a clear advantage for the open-abdomen versus the
closed-abdomen technique in the treatment of severe abdominal sepsis; however, in
the author's experience, bowel edema and subsequent inflammatory changes limit the
use of the closed-abdomen technique. Secondary abdominal compartment syndrome
(secondary ACS) may ensue if abdominal closure is performed before the inflamma-
tory process has resolved. In some cases, staged operative interventions will be
planned. In other cases, patients may present continued peritonitis or abscess for-
mation requiring "on demand" relaparotomy. A 2004 study suggested that the mor-
tality rate of on-demand laparotomy is higher for those patients receiving intervention
more than 48 hours after their index operation.

Laparoscopy
Laparoscopy is gaining wider acceptance in the diagnosis and treatment of
abdominal infections. As with all indications for laparoscopic surgery, outcomes vary
depending on the skill and experience of the laparoscopic surgeon. Initial laparoscop-
ic examination of the abdomen can assist in determination of the etiology of peritoni-
tis (e.g., right lower quadrant pathology in female patients). Laparoscopic surgery is
commonly used in the treatment of uncomplicated appendicitis, although in prelimi-
nary studies, outcomes for complicated appendicitis have generally been positive. For
complicated and uncomplicated appendicitis, the laparoscopic approach is associated
with a shorter length of stay and fewer wound infections than the open approach;
however laparoscopic surgery may be associated with a higher rate of intra-
abdominal abscess. Laparoscopic diagnosis and peritoneal lavage in patients with per-
itonitis secondary to diverticulitis has been shown to be safe and has helped to avoid
the need for colostomy in many patients in small clinical trials. In a prospective study
comparing laparoscopic peritoneal lavage to an open Hartmann’s procedure for perfo-
rated diverticulitis with generalized peritonitis, peritoneal lavage without operative
intervention was found to be feasible, with a comparable mortality rate and a low risk
of short-term recurrence. Successful laparoscopic repair of perforated gastric and du-
odenal ulcers has also been reported. No definitive guidelines have been established
regarding the optimal selection of patients for successful laparoscopic repair. Studies
have been investigating scoring systems (e.g., APACHE II, Boey score) for patient
risk stratification to better select appropriate patients for laparoscopic repair. The
treatment of perihepatic infections via laparoscopic approach has been well estab-
lished in acute cholecystitis, where laparoscopic cholecystectomy has become the
mainstay of therapy. More recently, primary treatment of subphrenic abscesses and
laparoscopic, ultrasound assisted drainage of pyogenic liver abscesses have been per-
formed successfully. Individual reports also describe successful drainage of peripan-
creatic fluid collections and complicated intra-abdominal abscesses that are not ame-
nable to CT scan – or ultrasound guided percutaneous drainage. As minimally inva-
sive procedures continue to advance technologically, use of these approaches is likely
to increase, reducing the need for the open surgical approach for peritoneal abscess
drainage.
Preoperative Details
Volume resuscitation and prevention of secondary organ system dysfunction
are of utmost importance in the treatment of patients with intra-abdominal infections.
Depending on the severity of the disease, these patients should have Foley catheters
placed to monitor urine output. Use invasive hemodynamic monitoring in severely ill
patients to guide volume resuscitation and inotropic support. Correct existing serum
electrolyte disturbances and coagulation abnormalities as best as possible before any
intervention. Begin empiric broad-spectrum systemic antibiotic therapy as soon as the
diagnosis of intra-abdominal infection is suspected and tailor therapy according to the
underlying disease process and culture results. Remember that patients with peritoni-
tis often have severe abdominal pain. Provide adequate analgesia with parenteral nar-
cotic agents as soon as possible. In the setting of significant nausea, vomiting, or ab-
dominal distension caused by obstruction or ileus, institute nasogastric decompres-
sion as soon as possible. Consider intubation and ventilator support early in patients
with evidence of septic shock or altered mental status to prevent further decompensa-
tion. Even if patients do not appear critically ill initially, arranging for postoperative
intensive care support before the operation is often wise, particularly in patients of
advanced age and those with significant comorbidities. In patients with severe infec-
tions and certain disease processes (e.g., necrotizing pancreatitis, bowel ischemia),
informed consent should include the potential need for several reoperations and enter-
ic diversion. The involved physicians and surgeon should not downplay the signifi-
cant morbidities associated with abdominal sepsis when discussing these issues with
the patient and/or family.
Intraoperative Details
A discussion of the specific details of the operative treatment of all the poten-
tial etiologies of intraperitoneal infections is beyond the scope of this article. Certain
principles always apply when performing celiotomies in patients with peritonitis. The
goals of operative treatment of peritonitis are to eliminate the source of contamina-
tion, to reduce the bacterial inoculum, and to prevent recurrent or persistent sepsis. A
vertical midline incision is the incision of choice in most patients with generalized
peritonitis because it allows access to the entire peritoneal cavity. In patients with lo-
calized peritonitis (e.g., acute appendicitis, cholecystitis), an incision directly over the
site of pathology (e.g., right lower quadrant, right subcostal) is usually adequate. In
patients with an unclear etiology of the peritonitis, initial diagnostic laparoscopy may
be useful. The intra-abdominal anatomy may be significantly distorted because of in-
flammatory masses and adhesions. Normal tissue planes and boundaries may be
obliterated. The inflamed organs are often very friable, and the surgeon must exercise
great caution when exploring the patient with peritoneal infection. Hemodynamic in-
stability may occur at any time during treatment because of bacteremia and cytokine
release. Patients often demonstrate significant fluid shifts with third spacing. Swelling
of the bowel, retroperitoneum, and abdominal wall may preclude safe abdominal clo-
sure after prolonged cases in patients who are severely ill. Inflammation causes re-
gional hyperemia, and sepsis may cause coagulation deficits and platelet dysfunction,
leading to increased bleeding. Careful dissection and meticulous hemostasis are of
utmost importance. When faced with extensive abdominal inflammatory disease and
septic shock, draining the infection temporarily, controlling the visceral leak quickly
(e.g., oversewing, enteric diversion), and deferring any definitive repair until after the
patient has recovered from the initial insult (i.e., damage control operation) may be
better. One of the critical decisions in the surgical treatment of patients with severe
peritonitis is regarding whether to use a closed-abdomen or open-abdomen technique.
The goal of the closed-abdomen technique is to provide definitive surgical treatment
at the initial operation; perform primary fascial closure and perform repeat laparoto-
my only when clinically indicated. The goal of the open-abdomen technique is to
provide easy direct access to the affected area. Source control is achieved through re-
peated reoperations or open packing of the abdomen. This technique may be well
suited for initial damage control in extensive peritonitis. Also consider patients who
are at high risk for development of abdominal compartment syndrome (e.g., intestinal
distension, extensive abdominal wall and intra-abdominal organ edema) for this tech-
nique because attempts to perform primary fascial closure under significant tension in
these circumstances are associated with an increased incidence of MOF (e.g., renal,
respiratory), necrotizing abdominal wall infections, and mortality.
Postoperative Details
Postoperatively, monitor all patients closely in the appropriate clinical setting
for adequacy of volume resuscitation, resolution or persistence of sepsis, and the de-
velopment of organ system failure. Appropriate systemic broad-spectrum antibiotic
coverage must be continued without interruption for the appropriate time. The pa-
tient's overall condition should improve significantly and progressively within 24-72
hours of the initial treatment (i.e., resolution of the signs and symptoms of infection,
mobilization of interstitial fluid). This time course may be prolonged in patients who
are critically ill with significant multiple organ system dysfunction. A lack of im-
provement should prompt an aggressive search for a persistent or recurrent intraperi-
toneal or new extraperitoneal infectious focus. Patients requiring surgical intervention
for peritonitis demonstrate a significantly increased risk for surgical site infections
and wound healing failure; monitor patients closely for this potential complication.
All patients who are critically ill and patients receiving prolonged antibiotic therapy
are at increased risk for developing secondary opportunistic infections (e.g., C dif-
ficile colitis, fungal infections, central venous catheter infections, ventilator-
associated pneumonia); monitor patients closely for signs and symptoms of these
complications.
Nutrition
In general, patients with peritonitis develop some degree of gut dysfunction
(e.g., ileus) after exploration. Consider establishing some form of nutritional support
early in the course of treatment because most patients have an insufficient enteral in-
take for a variable amount of time preoperatively. The existing data support that en-
teral nutrition is superior to parenteral hyperalimentation. If enteral feeding is contra-
indicated or not tolerated, parenteral nutrition should be instituted.
Follow-up
After resolution of peritonitis and peritoneal abscesses, follow-up care is di-
rected mostly by specifics of the underlying disease process and the presence or ab-
sence of chronic complications (e.g., enterocutaneous fistulae). Patients with simple
peritoneal infections after appendicitis or cholecystitis are usually cured and do not
require long-term follow-up care. Patients with peritoneal operations for perforated
peptic ulcer disease, Crohn’s disease, pancreatitis, and others often require lifelong
medical therapy and treatment of recurrent complications.
Complications
Surgical site infection/dehiscence
The incidence of surgical site infection increases with the degree of contami-
nation; therefore, surgical site infection occurs at much higher rates after operations
for peritonitis and peritoneal abscess (i.e., 5-15% compared to <5% for elective ab-
dominal operations for noninfectious etiologies). Surgical site infection may be ex-
pected if the wound is closed in the setting of gross abdominal contamination. Peri-
operative systemic antibiotics, the use of wound protector devices, and lavage of the
wound at the end of therapy do not reliably prevent this complication. These wounds
should be left open and be treated with wet-to-dry dressing changes several times a
day or VAC dressing should be applied.
Impaired wound healing
The same factors that impair clearance of the abdominal infection contribute
to increased problems related to wound healing (e.g., malnutrition, severe sepsis,
multiple organ system dysfunctions, advanced age, immunosuppression) and should
be addressed aggressively. Patients with severe abdominal infections demonstrate
higher incidences of fascial dehiscence and incisional hernia development, requiring
later reoperation.
Complications related to percutaneous drainage
Percutaneous drainage procedures carry a risk of related significant complica-
tions of less than 10% (range 5-27%) depending on the underlying pathology and ab-
scess location. These complications include bleeding, injury, erosion, transgression of
small and large bowel, fistula formation, and others. Strategies to prevent these prob-
lems include correction of coagulation problems and determination of the exact etiol-
ogy, location, and anatomic relationships of the abscess. Indication for percutaneous
treatment of complex abscesses and patients with a persistent enteric leak should be
reviewed critically, and operative treatment should not be delayed with lack of ade-
quate patient improvement.
Tertiary peritonitis
Persistence of intra-abdominal infection (i.e., tertiary peritonitis) is a compli-
cation that may occur following the treatment of primary or secondary peritonitis and
peritoneal abscess.
Complications related to the open-abdomen technique
One of the complications related to treatment of severe intra-abdominal infections
with the open-abdomen technique and multiple reoperations is the development of en-
terocutaneous fistulae. A study of trauma patients found that morbidity due to wound
complications (wound infections, abscess, and/or fistula) from the open abdomen re-
mained high at 25%. Enterocutaneous fistulae can lead to ongoing (potentially large)
volume, protein, and electrolyte losses; inability to use the gut for nutritional support;
and associated long-term complications of intravenous alimentation. Patients with
small, low-output, and distal fistulae often can be fed enterally with elemental diets.
A proportion of these fistulae close spontaneously as the patient's overall status and
nutritional status improve. High-output and proximal fistulae often require a delayed
surgical repair. Optimal timing of this repair is critical. Initial inflammatory adhesions
and dense scar formation may make safe reexploration impossible. Maturation of the
scar tissue occurs over 6-12 months. Close observation of the patient's overall condi-
tion and nutritional status is important during that time. Deterioration of the patient's
condition may force an earlier reoperation. For an extended time after operations for
intra-abdominal infections, patients are at a several-fold increased risk of developing
bowel obstruction related to intra-abdominal scar formation. While in some patients
this obstruction may be partial and reversible and may improve with cessation of en-
teral intake and gastric decompression, most patients require reoperation over time.
Complications related to abdominal compartment syndrome
ACS is a well-recognized disease entity related to acutely increased ab-
dominal pressure (i.e., intra-abdominal hypertension [IAH]) and is associated with the
development of multiple organ dysfunction. Elevated intra-abdominal pressure ad-
versely impacts pulmonary, cardiovascular, renal, splanchnic, musculoskeletal, integ-
umentary, and central nervous system physiology. The combination of IAH and dis-
ordered physiology results in a clinical syndrome with significant morbidity and mor-
tality. ACS can occur in a variety of surgical conditions, particularly those with major
life-threatening hemorrhage, massive volume resuscitation, prolonged operation
times, and coagulopathy. In patients who are severely traumatized, the incidence of
ACS is reported to be as high as 15% after damage control laparotomies. The exact
incidence of ACS in patients undergoing surgery for intra-abdominal infections and
peritonitis is unknown. However, closure of the abdomen under tension at completion
of the laparotomy is clearly associated with a much higher risk for ACS postopera-
tively (visceral edema and accumulation of peritoneal fluid). The intra-abdominal
pressure can easily be assessed by measuring the urine bladder pressure, which corre-
lates well with the actual intra-abdominal pressure. Increasing bladder pressure meas-
urements suggest increased risk for ACS and warrant more aggressive intervention. A
study determined that abdominal perfusion pressure, defined as mean arterial pressure
minus intra-abdominal pressure, was statistically superior to mean arterial pressure
and intravesicular pressure in predicting patient survival from IAH and ACS. Multi-
ple regression analysis demonstrated that abdominal perfusion pressure was also su-
perior to other common resuscitation endpoints, including arterial pH, base deficit,
arterial lactate, and hourly urinary output. The onset of ACS requires prompt recogni-
tion and appropriately timed and staged intervention to optimize outcome. Surgical
decompression of the abdomen by means of a laparotomy is the treatment of choice
for ACS. Urgent laparotomy can be lifesaving in some cases. However, no single
threshold of abdominal pressure can be applied universally. The best therapeutic op-
tion is decompression of the abdomen surgically if the intravesical pressure is 25 mm
Hg or higher in patients with refractory hypotension, acute renal failure, or respirato-
ry failure caused by abdominal distension. Studies also suggest that abdominal de-
compression for ACS can be accomplished with laparoscopy in patients with in-
creased intra-abdominal pressure postoperatively that is related to the accumulation
of tense ascites and not intraperitoneal hemorrhage.
Complications related to enteric insufficiency
Extensive initial (gastrointestinal) disease, chronic recurrent infections, and
associated reoperations may lead to enteric insufficiency because of short gut, pan-
creatic insufficiency, or hepatic dysfunction. Treatment of these problems can be
quite challenging and can require a multispecialty approach to optimize gastrointesti-
nal function and nutritional status.
Outcome and prognosis
Spontaneous bacterial peritonitis
The overall mortality rate of patients with SBP may exceed 30% if diagnosis and
treatment are delayed, but the mortality rate is less than 10% in fairly well-
compensated patients with early therapy. As many as 70% of patients who survive an
episode of SBP have a recurrent episode within 1 year, and, for these patients, the
mortality rate approaches 50%. Some studies suggest that the recurrence rate of SBP
may be decreased to less than 20% with long-term antibiotic prophylaxis (e.g., quin-
olones, trimethoprim-sulfamethoxazole); however, whether this improves long-term
survival without liver transplantation is unclear.
Secondary peritonitis and peritoneal abscess
Treatment success of peritoneal infections is defined as adequate source con-
trol with resolution of sepsis and clearance of all residual intra-abdominal infection.
With percutaneous treatment, the definition of success includes the avoidance of fur-
ther operative intervention and, in some cases, the delay of surgery until after resolu-
tion of the initial sepsis. Over the past decade, the combination of better antibiotic
therapy, more aggressive intensive care, and earlier diagnosis and therapy with a
combination of operative and percutaneous techniques have led to a significant reduc-
tion in morbidity and mortality related to intra-abdominal sepsis. Uncomplicated SP
and simple abscesses carry a mortality rate of less than 5%, but this rate may increase
to greater than 30-50% in severe infections. The overall mortality rate related to intra-
abdominal abscess formation is less than 10-20%. Factors that independently predict
worse outcomes include advanced age, malnutrition, presence of cancer, a high
APACHE II score on presentation, preoperative organ dysfunction, the presence of
complex abscesses, and failure to improve in less than 24-72 hours after adequate
therapy. In severe intra-abdominal infections and peritonitis, the mortality rate may
increase to greater than 30-50%. The concurrent development of sepsis, SIRS, and
MOF can increase the mortality rate to greater than 70%, and, in these patients, more
than 80% of deaths occur with an active infection present. Several scoring systems
(e.g., APACHE II, SIRS, multiple organ dysfunction syndrome [MODS], and Mann-
heim peritonitis index) have been developed to assess the clinical prognosis of pa-
tients with peritonitis. Most of these scores rely on certain host criteria, systemic
signs of sepsis, and complications related to organ failure. Although valuable for
comparing patient cohorts and institutions, these scores have limited value in the spe-
cific day-to-day clinical decision-making process for any given patient. In general,
the mortality rate is less than 5% with an APACHE II of less than 15 and rises to
greater than 40% with scores above 15. Rising APACHE II scores on days 3 and 7
are associated with an increase of mortality rates to greater than 90%, whereas falling
scores predict mortality rates of less than 20%. The mortality rate without organ fail-
ure generally is less than 5% but may rise to greater than 90% with quadruple organ
failure. A delay of more than 2-4 days of either medical therapy or surgical therapy
has been clearly associated with increased complication rates, the development of ter-
tiary peritonitis, the need for reoperation, multiple organ system dysfunction, and
death. Outcomes are worse in patients requiring emergent reoperations for persistent
or recurrent infections (30-50% increase in the mortality rate); however, patients un-
dergoing early planned second-look operations do not demonstrate this trend. Persis-
tent infection, recovery of enterococci, and multidrug-resistant gram-negative organ-
isms, as well as fungal infection, are related to worse outcomes and recurrent compli-
cations. Patients older than 65 years have a 3-fold increased risk of developing gener-
alized peritonitis and sepsis from gangrenous or perforated appendicitis and perforat-
ed diverticulitis than younger patients and are 3 times more likely to die from these
disease processes. Older patients with perforated diverticulitis are 3 times more likely
than younger patients to have generalized rather than localized (i.e., pericolic, pelvic)
peritonitis. These findings are consistent with the hypothesis that the biologic features
of peritonitis differ in elderly persons, who are more likely to present with an ad-
vanced or more severe process than younger patients with peritonitis. Overall, studies
suggest that host-related factors are more significant than the type and source of in-
fection with regard to the prognosis in intra-abdominal infections.
Basic literatures:

1. Townsend: Sabiston Textbook of Surgery, 18th ed.


5. Essentials of Surgery: Scientific Principles and Practice 2nd edition / Green-
field L. J., Mulholland M. W., Oldham K. T., Zelenock G. B., Lillimoe K. D.,
Oldham K. – 1997. - Lippincott Williams & Wilkins Publishers.
6. Oxford Textbook of Surgery (3-Volume Set) 2nd edition / Morris P. J., Wood
W. C. – 2000. - Oxford Press
7. Schwartz’s manual of surgery (8th edition) / Brunicardi F. - 2006 C.
McGRAW-HILL Medical Publishing Division New York, Chicago, San Fran-
cisco, Lisbon, London, Madrid, Mexico City, Milan, New Delhi, San Juan,
Seoul, Singapore, Sydney, Toronto.

Additional literatures:

Peralta R. Peritonitis and Abdominal Sepsis / 2010. - Available at:


http://emedicine.medscape.com/article/192329-followup

Tests for initial level of knowledge, keys for tests:

3. All of the following statements concerning intraabdominal abscesses are


correct EXCEPT:
F. A common cause is perforation of hollow viscus
G. Treatment usually includes surgical exploration with drainage of the ab-
scess
H. A high index of suspicious is essential for the diagnosis because there
may be no physical signs of infection
I. Ultrasonography, CT scan, or MRI can be used to direct surgical drain-
age
J. These infections are usually due to staphylococcal organism

4. Postoperatively, the patient requires an indwelling bladder catheter for 5


days to treat urinary retention. He does well until the tenth postoperative day, at
which point he develops a fever of 39 C0, right lower quadrant pain, and an ileus. The
midline wound is not inflamed. The most likely development is:
F. Blind loop syndrome
G. Pyelonephritis
H. Recurrent Crohn’s disease
I. Intraabdominal abscess
J. Pseudomembranous enterocolitis

5. Choose the incorrect statement


A. Peritonitis causes a reduction in the intra-abdominal fibrinolytic activity
B. Large numbers of bacteria may be sequestered within the fibrin matrix
C. Fibrinolytics improve the outcome
D. Production of fibrinous exudates is considered an important part of the
host defense
E. As the fibrin matrix matures, the bacteria within are protected from host
clearance mechanisms

6. Following factors lead to persistence of infections and abscess formation


EXCEPT:
A. Capsule formation
B. Facultative anaerobic growth
C. Adhesion capabilities
D. Succinic acid production
E. Increase in the intraabdominal fibrinolytic activity
7. Predisposing factors for the development of abdominal candidiasis in-
clude the following EXCEPT:
A. Prolonged use of broad-spectrum antibiotics
B. Gastric acid suppressive therapy
C. Central venous catheters and intravenous hyperalimentation
D. Laparostomy
E. Steroids and other forms of immunosuppression

8. Choose cytokine which role in the development of the systemic inflam-


matory response syndrome and multiple organ failure is proved:
A. TNF-α
B. IL-10
C. IL-2
D. CRP
E. leptin

9. The clinical signs of peritonitis does not include:


A. Abdominal pain
B. Anorexia and nausea
C. Jugular veins distension
D. Increased abdominal wall rigidity
E. Tenderness to palpation of the abdomen

10. The general principles guiding the treatment of intra-abdominal infec-


tions include the following EXCEPT:
A. To control the infectious source
B. To eliminate bacteria and toxins
C. To maintain organ system function
D. To control the inflammatory process
E. To prescribe corticosteroids

11. While treating SBP should be avoided:


A. Aminoglycosides
B. Norfloxacin
C. Quinolone
D. Cephalosporins
E. Metronidazole

12. Temporary closure of the abdomen to prevent herniation and contamina-


tion from the outside of the abdominal contents can be achieved using:
A. Self-adhesive membrane dressings
B. Mesh
C. Velcro-like closure devices
D. Adhesive tape
E. Vacuum-assisted closure (VAC) devices

Keys for tests


1 2 3 4 5 6 7 8 9 10
E D C D D E C E A E

Tests for final level of knowledge, keys for tests:

1. The 19-year-old female is brought to the emergency room after being in a


single-car accident just 20 min earlier in which she lost control of her car on black ice
and hit a retaining column of an overpass at about 45 miles per hour. She was the on-
ly occupant and was wearing a seat belt but looks pale, has tachycardia and positional
hypotension, is extremely nauseated, and is lying in the fetal position due to severe
abdominal pain. She does not appear to have any broken bones and a cranial nerve
test appears normal. You order an abdominal CT because you suspect which of the
following?
A. That she is pregnant
B. That she has peritonitis from a ruptured spleen
C. That she has peritonitis from a ruptured gallbladder
D. That she has diverticulitis
E. That she has hemorrhoids

2. A 20-year-old man has undergone appendectomy for perforated appendi-


citis with generalized peritonitis. Seven days postoperatively, his temperature contin-
ues to spike to 38, 8 0 C despite antibiotic therapy with ampicillin, gentamicin, and
metronidazole. A CN scan reveals a large pelvic abscess. Soon afterward, he has
bleeding from the surgical wound and all intravenous puncture sites. It is due to:
A. Anaphylactoid reaction to intravenous dye
B. Disseminated intravascular coagulation
C. Antibiotic-induced coagulopathy
D. Liver failure
E. Congenital bleeding disorder

3. A 38-year-old man with a history of fever associated with abdominal


pain of 3 week’s duration presents now with a sudden onset of explosive abdominal
pain and vomiting. Plain abdominal x-ray reveals air under a diaphragm. A CT scan
shows mesenteric lymphadenopathy and splenomegaly is found. Laparotomy is per-
formed and 3 feet of ileum resected. The luminal aspect of the resected bowel shows
marked ulceration of Peyer’s patches. What is the most likely diagnosis?
A. Typhoid enteritis
B. Tuberculosis enteritis
C. Crohn’s disease
D. Primary peritonitis
E. Ulcerative colitis

4. Laparoscopy in abdominal trauma may be indicated in which of the fol-


lowing?
A. To exclude diaphragmatic injury
B. In patients with multiple previous abdominal operations
C. If there is limited cardiovascular reserve
D. If severe diffuse peritonitis exist
E. In hemodynamically unstable patients
5. A 30-year-old man with a history of Crohn’s disease develops an entero-
cutaneous fistula and is placed on total parenteral nutrition through a right subclavian
central venous catheter. After five days, the patient develops a fever and leukocytosis;
CT scan of the abdomen reveals no intraabdominal abscess. The subclavian catheter
insertion site is inspected and noted to be erythematous and painful. Blood cultures
are positive. Which of the following organisms is the most likely cause of his fever?
A. Coagulase-positive staphylococcus
B. Coagulase-negative staphylococci
C. Group A streptococcus
D. Enterococcus
E. Escherichia coli

6. Which of the following statements regarding appendicitis during preg-


nancy is correct?
A. Appendicitis is the most prevalent extrauterine indication for celiotomy during
pregnancy
B. Appendicitis occurs more commonly in pregnant women than in nonpregnant
women of comparable age
C. Suspected appendicitis in a pregnant woman should be managed with a period of
observation of due to the risks of laparotomy to the fetus
D. Noncomplicated appendicitis results in a 20% fetal mortality and premature labor
rate
E. The severity of appendicitis correlates with increased gestational age of the fetus

7. A 70-year-old man with aortic and mitral valvular regurgitation under-


goes an emergency sigmoid colectomy and end colostomy for perforated diverticuli-
tis. His postoperative course is complicated by a myocardial infarction and atrial fi-
brillation. Four weeks later, he has improved and requests elective colostomy closure.
You would recommend:
A. Discontinuation of antiarrhythmic and antihypertensive medications on the morn-
ing of surgery
B. Discontinuation of beta-blocking medications on the day prior to surgery
C. Control of congestive heart failure with diuretics and digitalis in severe cases
D. Administration of prophylactic antibiotics, other than ampicillin and gentamicin,
for patients with valvular heart disease who are undergoing gastrointestinal proce-
dures
E. Postponement of elective surgery for 6–8 wk after a subendocardial myocardial
infarction

8. An elderly diabetic woman with chronic steroid-dependent bron-


chospasm has an ileocolectomy for a perforated cecum. She is taken to the ICU intu-
bated and is maintained on broad-spectrum antibiotics, renal-dose dopamine, and a
rapid steroid taper. On postoperative day 2 she develops a fever of 39.2°C (102.5°F),
hypotension, lethargy, and laboratory values remarkable for hypoglycemia and hy-
perkalemia. The most likely diagnosis of this acute event is:
A. Sepsis
B. Hypovolemia
C. Adrenal insufficiency
D. Acute tubular necrosis
E. Diabetic ketoacidosis

9. Which one of the following cases is considered a clean-contaminated


wound?
A. Open cholecystectomy for cholelithiasis
B. Herniorrhaphy with mesh repair
C. Lumpectomy with axillary node dissection
D. Appendectomy with walled-off abscess
E. Gunshot wound to the abdomen with injuries to the small bowel and sigmoid co-
lon

10. A 45-year-old woman is explored for a perforated duodenal ulcer 6 h af-


ter onset of symptoms. She has a history of chronic peptic ulcer disease treated medi-
cally with minimal symptoms. The procedure of choice is:
A. Simple closure with omental patch
B. Excision of the ulcer, duodenoplasty
C. Antrectomy and truncal vagotomy
D. Highly selective vagotomy
E. Hemigastrectomy

Keys for tests

1 2 3 4 5 6 7 8 9 10
B C C E B A C C A B

Tasks for the final level of knowledge.

1. An 82-year-old nursing home patient presents to the emergency room with vomit-
ing, abdominal pain, and distention. A radiograph is obtained and demonstrates a
grossly dilated loop of intestine overlying the sacrum in the shape of an upside down
U. Your diagnosis?

The answer is sigmoid volvulus.

2. A patient with a non obstructing carcinoma of the sigmoid colon is being prepared
for elective resection. To minimize the risk of postoperative infectious complications,
your planning should include

The answer is postoperative administration for 2–4 days of parenteral an-


tibiotics effective against aerobes and anaerobes.
3. A 32-year-old man undergoes a distal pancreatectomy, splenectomy, and partial
colectomy for a gunshot wound to the left upper quadrant of the abdomen. One week
later he develops a shaking chill in conjunction with a temperature spike to 39.44°C
(103°F). His blood pressure is 70/40 mm Hg with a pulse of 140 beats/min and his
respiratory rate is 45 breaths/min. He is transferred to the ICU where he is intubated
and a Swan-Ganz catheter is placed. Your diagnosis and what should initial therapy
include?

The answer is the case presented is most consistent with septic shock from a post-
operative intraabdominal abscess. Initial therapy is aimed at resuscitation and stabi-
lization. This includes fluid replacement and vasopressors as well as antibiotic thera-
py aimed particularly at gram-negative rods and anaerobes for patients with presumed
intraabdominal collections, especially after bowel surgery.

4. A 59-year-old woman undergoes an exploratory laparotomy for peritonitis and is


found to have perforated diverticulitis. She undergoes a sigmoid resection with an end
colostomy. She is administered a third-generation cephalosporin with 1 h prior to the
incision and the antibiotic is continued postoperatively. One week later, she develops
in intraabdominal abscess, which is percutaneously drained. Bacteroides fragilis is
isolated from the cultures.
Why has this happened?

The answer is third-generation cephalosporins provide good coverage against


most gram-negative bacteria, but have poor activity against anaerobic bacteria.
Clindamicin or metronidazole can be used in addition to a third-generation cephalo-
sporin to provide anaerobic coverage. Alternatives include a combination of a penicil-
lin/β-lactamase inhibitor or a cefoxitin or cefotetan both of which second-generation
cephalosporins.

5. A 43-year-old man with signs and symptoms of peritonitis in the right lower quad-
rant. The clinical impression and supportive data suggest acute appendicitis. At ex-
ploration, however, a tumor is found; frozen section suggests carcinoid features.
What is the most appropriate surgical procedure?

The answer is right hemicolectomy.


Study guide #6.2
“Acute appendicitis, the most frequent reason of syndrome of acute
abdomen.”

Overview
In 2000 in the United States, acute appendicitis accounted for over one million
in patient hospital days, at a cost of three billion dollars. Acute appendicitis remains
one of the most common diseases treated by the general surgeon. Appendectomy is
the most commonly performed emergency surgery in industrialized countries. Inter-
estingly, the incidence of acute appendicitis is much lower in areas of Africa, Asia,
and South America. Differences in diet, nutritional status, and alterations in colonic
flora only partly explain this difference.
The pathophysiology of acute appendicitis has long been thought to be the re-
sult of luminal obstruction by a fecalith, hyperplastic lymphoid tissue, parasitic infes-
tation, or tumour, with subsequent localized venous ischemia resulting in mucosal
disruption followed by invasive bacterial infection; viral ulceration may also be the
cause of mucosal ulceration in certain patients. Infection limited to the appendix itself
results in localized inflammation and simple, or suppurative, appendicitis. Progres-
sion to full thickness necrosis and gangrene of the appendix wall may result in com-
plications of appendicitis, e.g., free perforation, abscess formation if the process is
contained by adjacent structures, or even fistula formation if the
inflammatory process continues unabated. Mortality associated with acute appendici-
tis has decreased steadily during the 20th century, and in the United States, has been
recently reported as 0.2 deaths per 100,000 cases. This most likely is attributable to
improvements in preoperative resuscitation and monitoring, and antibiotic use. The
majority of mortalities occur in the elderly, who have a much greater risk ratio for
death.

Educational aims
19. To collect the anamnesis and to conduct clinical examination on patient with
acute appendicitis.
20. To know the aetiology and pathogenesis of acute appendicitis.
21. To know the clinical picture of acute appendicitis.
22. To know the complications of acute appendicitis.
23. To make the plan of inspection of acute appendicitis.
24. To carry out the analysis of laboratory data and instrumental methods of
inspection in patients with acute appendicitis.
25. To define indications to surgical treatment in acute appendicitis.
26. To provide care for patients with acute appendicitis.
27. To estimate the work capacity of patients with acute appendicitis.
A student must know:
1. Surgical anatomy of the appendix and colon.
2. Pathophysiology and classification of acute appendicitis.
3. Uncomplicated and complicated acute appendicitis.
4. Aetiology and pathogenesis of acute appendicitis.
5. Methods of examination of patients with acute appendicitis.
6. Physical examination, laboratory findings, ultrasonography and computed to-
mography in patients with acute appendicitis.
7. Comparison of diagnostic methods.
8. The differential diagnosis.
9. Operative treatment of acute appendicitis.
10.Indications to surgical treatment.
11.Open appendectomy and laparoscopic appendectomy.
12.Perforated appendicitis and appendiceal abscess.
13.Acute appendicitis in the elderly, acute appendicitis during pregnancy.
A student must be able to:
11.Take anamnesis carefully.
12.Make diagnosis correctly.
13.Order additional examination.
14.Establish pathogenesis factors of occurrence of acute appendicitis
15.Diagnose complications in patients with acute appendicitis.
16.Make the plan of inspection for patients with acute appendicitis.
17.Estimate data of instrumental methods of inspection in patients with acute appen-
dicitis (ultrasonic research, computed tomography).
18.Carry out differential diagnostics.
19.Establish indications for surgical treatment.
20.Estimate efficiency of treatment and work capacity of patients.

Terminology
Term Definition
Acute Is an inflammation of vermiform appendix caused by festering
Appendicitis microflora.
Pain on extension of right thigh (retroperitoneal retrocecal ap-
Psoas sign
pendix
Obturator sign Pain on internal rotation of right thigh (pelvic appendix)
Rovsing's sign Pain in right lower quadrant with palpation of left lower quadrant
Dunphy's sign Increased pain with coughing
Rozdolskyy's
At percussion there is painfulness in the right iliac area
sign
With the patient lying on the back, the most painful place of the
Obraztsov's right iliac area is pressed with the index and middle fingers and
sign the patient is asked to lift up the straightened right leg. At appen-
dicitis, the pain increases acutely
Bartomier's It is the increase of pain intensity during the palpation in right ili-
sign ac area of patient lying on the left side.
Pull down the shirt of patient with the left hand and fix it on pu-
bis. By the taps of 2-4 fingers of right hand epigastric area is
Voskresenkyy
pressed and during exhalation of patient quickly and evenly the
sign
hand slides in the direction of right iliac area, without taking the
hand away. Thus there is an acute increase of pain.
After gradual pressing by fingers on anterior abdominal wall
from the place of pain quickly, but not acutely, the hand is taken
Blumberg's
away. Strengthening of pain is considered as a positive symptom
sign
in that place. Obligatory here is tension of muscles of anterior
abdominal wall.

Content:
The diagnosis of appendicitis can be difficult, occasionally taxing the skills of
even the most experienced surgeon. Likewise, decisions on the management of pa-
tients with appendiceal inflammation or abscess can be difficult. The patient with ap-
pendicitis must first recognize that he or she has an episode of pain that is unique, and
then present to a physician who recognizes the condition. Delays in
diagnosis arises from errors on the part of either patient or physician, and all delays
complicate the illness.

Anatomy and Function


The appendix first becomes visible in the eighth week of embryologic
development as a protuberance off the terminal portion of the cecum. During both an-
tenatal and postnatal development, the growth rate of the cecum exceeds that of the
appendix, displacing the appendix medially toward the ileocecal valve. The relation-
ship of the base of the appendix to the cecum remains constant, whereas the tip can be
found in a retrocecal, pelvic, subcecal, preileal, or right pericolic position. These ana-
tomic considerations have significant clinical importance in the context of acute ap-
pendicitis. The three taenia coli converge at the junction of the cecum with the ap-
pendix and can be a useful landmark to identify the appendix. The appendix can vary
in length from less than 1 cm to greater than 30 cm; most appendices are 6 to 9 cm in
length. Appendiceal absence, duplication, and diverticula have all been described.
For many years, the appendix was erroneously viewed as a vestigial organ with
no known function. It is now well recognized that the appendix is an immunologic
organ that actively participates in the secretion of immunoglobulins, particularly im-
munoglobulin A (IgA). Although the appendix is an integral component of the gut-
associated lymphoid tissue (GALT) system, its function is not essential and appen-
dectomy is not associated with any predisposition to sepsis or any other manifestation
of immune compromise. Lymphoid tissue first appears in the appendix approximately
2 weeks after birth. The amount of lymphoid tissue increases throughout puberty, re-
mains steady for the next decade, and then begins a steady decrease with age. After
the age of 60 years, virtually no lymphoid tissue remains within the appendix, and
complete obliteration of the appendiceal lumen is common.

Etiology and pathogenesis

Most frequent causes of acute appendicitis are festering microbes: intestinal ba-
cilli, streptococcus, and staphylococcus. Moreover, microflora can be present in the
cavity of appendix or range by hematogenic route, and for women - by lymphogenic
one. Factors which promote the origin of appendicitis are the following: a) change of
reactivity of organism; b) constipation and atony of intestine; c) twisting or bends of
appendix; d) excrement stone in its cavity; e) thrombosis of vessels of appendix and
gangrene of the wall as a part of inflammatory process (special cases).

Bacteriology
The bacteriology of the normal appendix is similar to that of the normal colon.
The appendiceal flora remains constant throughout life with the exception of Porphy-
romonas gingivalis. This bacterium is seen only in adults. The bacteria cultured in
cases of appendicitis are therefore similar to those seen in other colonic infections
such as diverticulitis. The principal organisms seen in the normal appendix, in acute
appendicitis, and in perforated appendicitis are Escherichia coli and Bacteroides fra-
gilis. However, a wide variety of both facultative and anaerobic bacteria and myco-
bacteria may be present (Table. 1). Appendicitis is a polymicrobial infection, with
some series reporting up to different organisms cultured in patients with perforation.

Table 1. Common Organisms Seen in Patients with Acute Appendicitis


Aerobic and Facultative Anaerobic
Gram-negative bacilli Gram-negative bacilli
E. coli Bacteroides fragilis
Pseudomonas aeruginosa Bacteroides species
Klebsiella species Fusobacterium species
Gram-positive cocci Gram-positive cocci
Streptococcus anginosus Peptostreptococcus anginosus
Streptococcus species Gram-positive bacilli
Enterococcus species Clostridium species

Pathomorphology

Simple (superficial) and destructive (phlegmonous, primary and secondary gan-


grenous) appendicitis which are morphological stages of acute inflammation that is
completed by necrosis and can be distinguished. In simple appendicitis the changes
are observed, mainly, in the distal part of appendix. There is stasis in capillaries and
venules, edema and hemorrhage. Focus of festering inflammation of mucus mem-
brane with the defect of the epithelium is seen in 1-2 hours (primary affect of
Ashoff). This characterizes acute superficial appendicitis. The phlegmon of appendix
develops by the end of the day. The organ increases, it serous layer becomes dimmed,
sanguineous, stratifications of fibrin appear on its surface, and there is pus in its cavi-
ty. In gangrenous appendicitis the appendix is thickened, its serous tunic is covered
by dimmed fibrinogenous, differentiating the layer structure through destruction is
not succeeded.

Classification (by V.I. Kolesov)

1. Appendiceal colic.
2. Simple superficial appendicitis.
3. Destructive appendicitis:
a) phlegmonous;
b) gangrenous;
c) perforated.
4. Complicated appendicitis:
a) appendiceal infiltrate;
b) appendiceal abscess;
c) Poured festering peritonitis.
4. Other complications of acute appendicitis (pylephlebitis, sepsis, retroperitoneal
phlegmon, local abscesses of abdominal cavity).

Symptoms and clinical staging


Clinically, four phases are distinguished according to clinical features of acute
appendicitis: 1) epigastric; 2) local symptoms; 3) calming down; 4) complications.
The disease begins with a sudden pain in the abdomen. It is localized in a right
iliac area, has moderate intensity, permanent character and does not irradiate. In 70 %
of patients the pain arises in the epigastric area - it is an epigastric phase of acute ap-
pendicitis. In 2-4 hours it shifts to the area of appendix (the Kocher's symptom). On
coughing there is marked increase of pain in the right iliac area - it is a positive cough
symptom. Together with it, nausea and vomiting that have reflex character can dis-
turb the patient. Often there is a delay of gases. The temperature of body of most pa-
tient rises, but high temperature can occur rarely and, mainly, it is a low grade fever.
The general condition of patients gets worse only in case of growth of destructive
changes in appendix.
During the examination it is possible to mark, that the right half of stomach re-
tracts during breathing, and the patient wants to lie down on the right side with legs
bent. Tenderness and pain are the basic and decisive signs of acute appendicitis dur-
ing the examination by palpation in the right iliac area, tension of muscle of ab-
dominal wall, positive symptoms of peritoneal irritation. About 100 pain symptoms
characteristic of acute appendicitis are known, however only some of them have real
practical value.
The Blumberg's sign. After gradual pressing by fingers on anterior abdominal
wall from the place of pain quickly, but not acutely, the hand is taken away. Strength-
ening of pain is considered as a positive symptom in that place. Obligatory here is
tension of muscles of anterior abdominal wall.
The Voskresenkyy Sign. By a left hand the shirt of patient is drawn downward
and fixed on pubis. By the taps of 2-4 fingers of right hand epigastric area is pressed
and during exhalation of patient quickly and evenly the hand slides in the direction of
right iliac area, without taking the hand away. Thus there is an acute increase of pain.
The Bartomier's sign is the increase of pain intensity during the palpation in
right iliac area of patient in position on the left side. In such position the omentum
and loops of small intestine is displaced to the left, and the appendix becomes acces-
sible for the palpation sense the pain.
The Sitkovsky's sign. A patient, who lies on the left, feels the pain which arises
or increases in a right iliac area. The mechanism of intensification of pain is ex-
plained by displacement of cecum to the left, by drawing of mesentery of the in-
flamed appendix.
The Rovsing's sign. By a left hand a sigmoid bowel is pressed to the back wall of
stomach. By a right hand by balloting palpation a descending bowel is pressed. Ap-
pearance of pain in a right iliac area is considered as a sign characteristic of appendi-
citis.
The Obraztsov sign. With the position of patient on the back by index and mid-
dle fingers the right iliac area of most painful place is pressed and the patient is asked
to raise the straightened right leg. At appendicitis pain increases acutely.
The Rozdolskyy's sign. At percussion there is painfulness in a right iliac area.

Laboratory Findings
Mild leukocytosis, ranging from 10,000 to 18,000/mm3, is usually present in
patients with acute, uncomplicated appendicitis and is often accompanied by a mod-
erate polymorphonuclear predominance. However, white blood cell counts are varia-
ble. It is unusual for the white blood cell count to be greater than 18,000/mm3 in un-
complicated appendicitis. White blood cell counts above this level raise the possibil-
ity of a perforated appendix with or without an abscess. Urinalysis can be useful to
rule out the urinary tract as the source of infection. Although several white or red
blood cells can be present from urethral or bladder irritation as a result of an inflamed
appendix, bacteriuria in catheterized urine specimen is not generally seen with acute
appendicitis

Imaging Studies
Plain films of the abdomen, although frequently obtained as part of the general
evaluation of a patient with acute abdomen, are rarely helpful in diagnosing acute ap-
pendicitis. However, plain radiographs can be of significant benefit in ruling out other
pathology. In patients with acute appendicitis, one often sees an abnormal bowel gas
pattern, which is a nonspecific finding. The presence of a fecalith is rarely noted on
plain films, but if present, is highly suggestive of the diagnosis. A chest x-ray is
sometimes indicated to rule out referred pain from a right lower lobe pneumonic pro-
cess. Additional radiographic techniques include barium enema and radioactive-label
leukocyte scans. If the appendix fills on barium enema, appendicitis is excluded. On
the other hand, if the appendix does not fill, no determination can be made. To date,
there has not been enough experience with radionuclide scans to assess their utility.
Graded compression sonography has been suggested as an accurate way to es-
tablish the diagnosis of appendicitis. The technique is inexpensive, can be performed
rapidly, does not require contrast, and can be used even in pregnant patients. So-
nographically, the appendix is identified as a blind-ending, nonperistaltic bowel loop
originating from the cecum. With maximal compression, the diameter of the appendix
is measured in the anteroposterior dimension. A scan is considered positive if a
noncompressible appendix 6 mm or greater in the anteroposterior direction is demon-
strated. The presence of an appendiceal establishes the diagnosis. The presence of
thickening of the appendiceal wall and periappendiceal fluid is highly suggestive. So-
nographic demonstration of a normal appendix, which is an easily compressible
blind-ending tubular structure measuring 5 mm or less in diameter, excludes the diag-
nosis of acute appendicitis. The study is considered inconclusive if the appendix is
not visualized and there is no pericecal fluid or mass. When the diagnosis of acute
appendicitis is excluded by sonography, a brief survey of the remainder of the ab-
dominal cavity should be performed to establish an
alternative diagnosis. In females of child-bearing age, the pelvic organs must be ade-
quately visualized either by transabdominal or endovaginal ultrasonography in order
to exclude gynaecologic pathology as a cause of acute abdominal pain. The so-
nographic diagnosis of acute appendicitis has a reported sensitivity of 55 to 96% and
a specificity of 85 to 98%. Sonography is similarly effective in children and pregnant
women, although its application is somewhat limited in late pregnancy. Although so-
nography can easily identify abscesses in cases of perforation, the technique has limi-
tations and results are user-dependent. A false-positive scan can occur in the presence
of periappendicitis from surrounding inflammation, a dilated fallopian tube can be
mistaken for an inflamed appendix, inspissated stool can mimic an appendicolith,
and, in obese patients, the appendix may not be compressible because of overlying
fat. False-negative sonograms can occur if appendicitis is confined to the appendiceal
tip, the appendix is retrocecal in location, the appendix is markedly enlarged and mis-
taken for small bowel, or if the appendix is perforated and therefore compressible.
Some studies have reported that graded compression sonography improved the diag-
nosis of appendicitis over clinical exam, specifically decreasing the percentage of
negative explorations for appendectomies from 37 to 13%. Sonography also decreas-
es the time before operation. Sonography identified appendicitis in 10% of patients
who were believed to have a low likelihood of the disease on physical examination.
The positive and negative predictive values of ultrasonography have impressively
been reported as 91 or 92%, respectively. However, in a recent prospective multicen-
ter study, routine
ultrasonography did not improve the diagnostic accuracy or rates of negative
appendectomy or perforation when compared to clinical assessment.
High-resolution, helical, computer tomography also has been used to diagnose
appendicitis. On CT scan, the inflamed appendix appears dilated (greater than 5 cm)
and the wall is thickened. There is usually evidence of inflammation, with "dirty fat,"
thickened mesoappendix, and even an obvious phlegmon. Fecaliths can be easily vis-
ualized, but their presence is not necessarily pathognomonic of appendicitis. An im-
portant suggestive abnormality is the arrowhead sign. This is caused by thickening of
the cecum, which funnels contrast toward the orifice of the inflamed appendix. CT
scanning is also an excellent technique for identifying other inflammatory processes
masquerading as appendicitis. Several CT techniques have been used, including fo-
cused and nonfocused CT scans and enhanced and nonenhanced helical CT scanning.
The nonenhanced helical CT scan is important because one of the disadvantages of
using CT scanning in the evaluation of right lower quadrant pain is dye allergy. Sur-
prisingly, all these techniques have yielded essentially identical rates of diagnostic
accuracy, i.e., 92 to 97% sensitivity, 85 to 94% specificity, 90 to 98% accuracy, and
75 to 95% positive and 95 to 99% negative predictive values. The additional use of
rectal contrast did not improve the results of CT scanning. A number of studies have
documented improvement in diagnostic accuracy with the liberal use of CT scanning
in the workup of suspected appendicitis. Computed tomography lowered the rate of
negative appendectomies from 19 to 12% in one study, and the incidence of negative
appendectomies in women from 24 to 5% in another. The use of this imaging study
altered the care of 24% of patients studied and provided alternative diagnoses in half
of the patients with normal appendices on CT scan. Despite the potential usefulness
of this technique, there are significant disadvantages. CT scanning is expensive, ex-
poses the patients to significant radiation, and cannot be used during pregnancy. Al-
lergy contraindicates the application of intravenous contrast in some patients, and
others cannot tolerate the oral ingestion of luminal dye, particularly in the presence of
nausea and vomiting. Finally, not all studies have documented the utility of CT scan-
ning in all patients with right lower quadrant pain.
The rational approach is the selective use of CT scanning. Selective CT scanning
based on the likelihood of appendicitis takes advantage of the clinical skill of the ex-
perienced surgeon and, when indicated, adds the expertise of the radiologist and his
or her imaging study.
Comparison of Ultrasound and Appendiceal CT Evaluation
of Suspected Appendicitis
Comparison graded Appendiceal computed
ultrasound tomographic scan
Sensitivity 85% 90 to 100%
Specificity 92% 95 to 97%
Use Evaluate patients with equivocal Evaluate patients with equivo-
diagnosis of appendicitis cal diagnosis of appendicitis
Advantages Safe Relatively inexpensive. Can More accurate. Better identifies
rule out pelvic disease in fe- phlegmon and abscess. Better
males. Better for children identifies normal appendix
Disadvantages Operator dependent Technically Cost Ionizing radiation Contrast
inadequate studies due to gas
Pain

Laparoscopy can serve as both a diagnostic and therapeutic maneuver for pa-
tients with acute abdominal pain and suspected acute appendicitis. Laparoscopy is
probably most useful in the evaluation of females with lower abdominal complaints
because appendectomy is performed on a normal appendix in as many as 30 to 40%
of these patients. Differentiating acute gynecologic pathology from acute appendicitis
can be effectively accomplished by using the laparoscope.

Clinical variants

Acute appendicitis in children. In infants acute appendicitis can be seen infre-


quently, but, quite often carries atypical character. All this is characterized, mainly,
by the features of anatomy of appendix, insufficient of plastic properties of the peri-
toneum, short omentum and high reactivity of child's organism. The inflammatory
process in the appendix of children rapidly progresses in the first half of the day,
there can be destruction, even perforation. The child, more frequently than an adult,
suffer from vomiting. General condition gets worse quickly, and already the positive
symptoms of irritation of peritoneum can show up during the first hours of a disease.
The temperature reaction is also expressed considerably worse. In the blood test there
is high leukocytosis. It is necessary to remember, that during the examination of agi-
tated children it is expedient to use a chloral hydrate enema.

Acute appendicitis in people of declining and old ages are not seen with often,
as in persons of middle ages and youth. This group of patients is hospitalized rather
late usually than: in 2-3 days from the beginning of the disease. Because of the in-
creased threshold of pain sensitivity, the intensity of pain in such patients is small;
therefore they almost do not pay attention to the epigastric phase of appendicitis. Fre-
quently nausea and vomiting is present, and the temperature reaction is expressed
poorly. Tension of muscles of abdominal wall is absent or is insignificant due to old-
age relaxation of muscles. But the symptoms of irritation of peritoneum keep the di-
agnostic value in this group of patients. Thus, the sclerosis of vessels of appendix re-
sults in rapid numbness, initially- gangrenous appendicitis develops. Because of such
reasons the destructive forms of appendicitis prevail, often there is appendiceal infil-
trate.

In pregnant women both the bend of appendix and violation of its blood flow
are causes of the origin of appendicitis. Increase in uterus size causes such changes.
Especially in the second half of pregnancy, the growing uterus displaces the cecum
together with the appendix upwards, and an over distension abdominal wall does not
create adequate tension. It is needed also to remember, that pregnant women periodi-
cally can have a moderate pain in the abdomen and changes in the blood test. Togeth-
er with that, psoas-symptom and the Bartomier's sign have a diagnostic value in
pregnant women.
Clinical passes of acute appendicitis in abnormally located appendix (not in a
right iliac area) will differ from the classic vermiform appendix. Appendicitis at ret-
rocecal and retroperitoneal location of appendix can be seen with 8-20 % patients.
Thus an appendix can be placed both in a free abdominal cavity and the retroperito-
neum. An atypical clinical picture presents, as a rule, at the retroperitoneal location.
The patients complain of pain in lumbar region or above the wing of right ileum.
There is marked tenderness on palpation. Sometimes the pain irradiates to the pelvis
and into the right thigh. The positive sign of Rozanov - tenderness on palpation in the
right Pti triangle is characteristic. In transition of inflammatory process to the ureter
and kidney; in the urines analysis red corpuscles can be found.
Appendicitis at the pelvic location of appendix can be seen in 11-30 % cases.
In such patients the pain is localized above the right Poupart's ligament and above
pubis. In a lowly placed appendix in the beginning of disease the reaction of muscles
of anterior abdominal wall to inflammatory process can be absent. With transition of
inflammation to the urinary bladder or rectum patients’ present
dysuria or diarrhea, mucus appears in stool excrement. Distribution of process on in-
ternal genital organs provokes signs characteristic of their inflammation.
Appendicitis in medially located appendix. The appendix in patients with such
pathology is located between the loops of intestine, which is in a large field of suction
and causing irritation of peritoneum. With anatomic features mesentery is pulled in
the inflammatory process, acute dynamic of the intestinal obstruction develops in
such patients. The pain in the abdomen is intensive, widespread, there is increased
tension of muscles of abdominal wall, together with symptoms of the
irritation of peritoneum, specify the substantial threat of peritonitis development.
For the subhepatic location of appendix the pain is characteristic in right
hypochondrium. During palpation, tenderness and tension of muscles can be marked.
Left-sided appendicitis appears infrequently and, as a rule are seen, in cases of
the reverse location of all organs, however it can occur at a mobile blind gut. In this
situation all signs which characterize acute appendicitis will be exposed not on the
right, but usually, on the left side.

Appendicitis in Patients with HIV (AIDS) Infection


The incidence of acute appendicitis in HIV (AIDS) -infected patients is reported
to be 0,5%. This is higher than the 0,1 to 0,2% incidence reported for the general
population. The presentation of acute appendicitis in HIV(AIDS) -infected patients is
similar to that of noninfected patients. The majority of HIV(AIDS) -infected patients
with appendicitis will have fever, periumbilical pain radiating to the right lower quad-
rant (91%), right lower quadrant tenderness (91%), and rebound tenderness (74%).
HIV(AIDS) -infected patients will not manifest an absolute leukocytosis; however, if
a baseline leukocyte count is available, nearly all HIV(AIDS)-infected patients with
appendicitis will demonstrate a relative leukocytosis. There appears to be an in-
creased risk of appendiceal rupture in HIV(AIDS) -infected patients. In one large se-
ries of HIV(AIDS) -infected patients who underwent appendectomy for presumed
appendicitis, 43% of patients were found to have perforated appendicitis at laparoto-
my. The increased risk of appendiceal rupture may be related to the delay in presenta-
tion seen in this patient population. The mean duration of symptoms prior to arrival in
the emergency room has been reported to be increased in HIV(AIDS)-infected pa-
tients, with more than 60% of patients reporting the duration of symptoms to be long-
er than 24 hours. In early series, significant hospital delay also may have contributed
to high rates of rupture. However, with increased understanding of abdominal pain in
HIV-infected patients, hospital delay has become less prevalent. A low CD4 count is
also associated with an increase in appendiceal rupture.
The differential diagnosis of right lower quadrant pain is expanded in
HIV(AIDS)-infected patients when compared to the general population. Opportunis-
tic infections should be considered as a possible cause of right lower quadrant pain.
These opportunistic infections include cytomegalovirus (CMV), Kaposi's sarcoma,
tuberculosis, lymphoma, and other causes of infectious colitis.
In the HIV(AIDS)-infected patient with classic signs and symptoms of appendi-
citis, immediate appendectomy is indicated. In those patients with diarrhea as a prom-
inent symptom, colonoscopy may be warranted. In patients with equivocal findings,
CT scan is usually helpful. The majority of pathologic findings identified in
HIV(AIDS) -infected patients who undergo appendectomy for presumed appendicitis
are typical. The negative appendectomy rate is 5 to 10%. However, up to 25% of pa-
tients will have AIDS-related entities in the operative specimens, including CMV,
Kaposi's sarcoma, and M.avium intracellulare.
Morbidity rates for HIV(AIDS)-infected patients with nonperforated appendici-
tis are similar to those seen in the general population. Postoperative morbidity rates
appear to be higher in HIV(AIDS)-infected patients with perforated appendicitis. Ad-
ditionally, the length of hospital stay for HIV(AIDS)-infected patients undergoing
appendectomy is twice that of the general population. No series has been reported to
date that addresses the role of laparoscopic appendectomy in the HIV (AIDS)-
infected population.
The complications of acute appendicitis
Among the complications of acute appendicitis most important are appendiceal
infiltrates and abscesses.
Appendiceal infiltrate is the conglomerate of organs and tissue not densely ac-
crete round the inflamed vermiform appendix. It develops, certainly, on 3rd -5th day
from the beginning of disease. Acute pain in the stomach decreases, the general con-
dition of the patient gets better. Dense, not mobile, painful, with unclear contours,
mass is palpated in the right iliac area. There are different sizes of infiltrate; some-
times it occupies all right iliac region. The stomach round infiltrate during palpation
is soft and non tender. At reverse development of infiltrate (when resorption comes)
the general condition of the patient gets better, sleep and appetite improves, activity
grows, the temperature of body and indexes of blood is normalized. Pain in the right
iliac area decreases, infiltrate diminishes in size. In this phase of infiltrate physiother-
apeutic procedure is appointed, warmth on the iliac area. Two months after resorption
of infiltrate appendectomy is conducted. In abscess formation the condition of the pa-
tient gets worse, the symptoms of acute appendicitis become more expressed, the
temperature of body, rises, the fever appears. Next to that, pain in the right iliac area
increases. Tender mass is palpable in the right iliac region. Blood test shows leukocy-
tosis with the acutely expressed change of leukocyte formula to the left.
Local abscesses of abdominal cavity, develops mainly in cases of the atypical
location of appendix or due to suppuration. Pelvic abscesses are seen more frequent-
ly, thus a patient is disturbed by pain below the abcupula, and there is dysuria, diar-
rhea and tenesmus. The temperature of body rises to 38, 0-39, 0°C, and rectal tem-
perature is considerably higher. In the blood test leukocytosis, shift of formula of
blood is shifted to the left. During the rectal examination the weakened sphincter of
anus is observed. The anterior wall of rectum at first is only painful, and then its
overhanging is observed as dense painful infiltrate.
A subdiaphragmatic abscess develops with highly located appendix. The pain
in the lower parts of thorax and in a upper quarter of abcupula often to the right that
increases with deep inspiration except for the signs of intoxication is characteristic of
it. A patient, generally, occupies semi sitting position. Swelling in the epigastric area
is observed in complicated cases, smoothing and painful intercostals intervals. The
abcupula often during palpation is soft, although tension in the area of right hypo-
chondrium is possible. Tenderness on pressure on lower ribs is the early and perma-
nent sign of subdiaphragmatic abscess (the Krukov's sign). Roentgenologically the
right half of diaphragm can fall behind from left one while breathing and reactive ex-
udates to present right pleura cavity. A gas bubble is considered to be roentgenologic
sign of subdiaphragmatic abscess with the horizontal level of liquid, which is placed
under the diaphragm.
Interloop abscesses are not frequent complications of acute appendicitis. Like
all abscesses of the abdominal cavity, they pass the period of infiltrate and abscess
formation with the recreation of the proper clinic.
Peritonitis develops as a result of the timely unoperated appendicitis. Diagno-
sis of this pathology does not cause difficulties.
Pylephlebitis is a complication of both appendicitis and post appendectomy pe-
riod. The cause of this pathology is acute retrocecal appendicitis. There's thrombo-
phlebitis of veins of appendix, which passes to the veins of bowel mesentery, and
then to the portal vein. Patients complain of general weakness, pain in right hypo-
chondrium, high temperature of body, fever and increased sweating. Patients are ady-
namic, with slightly icteric of the scleras. During palpation painfulness is observed in
the right half of abcupula often and the symptoms of irritation of peritoneum are not
acutely expressed. In case of rapid progress of disease, icterus appears, the liver is in-
creased, hepato-venous insufficiency progresses, and patients die in 7-10 days after
the onset of disease. At gradual subacute development of pathology the liver and
spleen is increased in size, and after the septic state of organism ascites arises.
Diagnosis

1. Anamnesis.
2. Information of objective examination.
3. General analysis of blood and urine.
4. Vaginal examination for women.
5. Rectal examination for men.
Differential Diagnosis
The differential diagnosis of acute appendicitis depends on four major factors:
the anatomic location of the inflamed appendix; the stage of the process (i.e., simple
or ruptured); the patient's age; and the patient's sex.
Acute mesenteric adenitis is the disease most often confused with acute appen-
dicitis in children. Almost invariably, an upper respiratory infection is present or has
recently subsided. The pain is usually diffuse, and tenderness is not as sharply local-
ized as in appendicitis. Voluntary guarding is sometimes present, but true rigidity is
rare. Generalized lymphadenopathy may be noted. Laboratory procedures are of little
help in making the correct diagnosis, although a relative lymphocytosis, when pre-
sent, suggests mesenteric adenitis. Observation for several hours is in order if the di-
agnosis of mesenteric adenitis seems likely, because mesenteric adenitis is a self-
limited disease. However, if the differentiation remains in doubt, immediate explora-
tion is the safest course of action.
Acute gastroenteritis is common in childhood but can usually be easily differen-
tiated from appendicitis. Viral gastroenteritis, an acute self-limited infection of di-
verse causes, is characterized by profuse watery diarrhea, nausea, and vomiting. Hy-
perperistaltic abdominal cramps precede the watery stools. The abdomen is relaxed
between cramps, and there are no localizing signs. Laboratory values are normal.
Salmonella gastroenteritis results from ingestion of contaminated food. Abdominal
findings are usually similar to those in viral gastroenteritis, but in some cases, the ab-
dominal pain is intense, localized, and associated with rebound tenderness. Chills and
fever are common. The leukocyte count is usually normal. The causative organisms
can be isolated from nearly 100% of patients. However, cultures may take too long to
assist the clinician in making a timely differential diagnosis. Similar attacks in other
persons eating the same food as the patient greatly strengthen the presumptive diag-
nosis of salmonella gastroenteritis. Because typhoid fever is now a rare disease, its
diagnosis is frequently missed. The onset is less acute than in appendicitis, with a
prodrome of several days. Differentiation is usually possible because of prostration,
maculopapular rash, inappropriate bradycardia, and leukopenia. Diagnosis is con-
firmed by culture of Salmonella typhosa from stool or blood. Intestinal perforation,
usually in the lower ileum, develops in 1% of cases and requires immediate surgical
therapy.
Diseases of the male urogenital system must be considered in differential diag-
nosis of appendicitis, including torsion of the testis and acute epididymitis, because
epigastric pain may overshadow local symptoms early in these diseases. Seminal ve-
siculitis may also mimic appendicitis, but can be diagnosed by palpating the enlarged,
tender seminal vesicle on rectal examination.
Meckel's diverticulitis causes a clinical picture similar to that of acute appendi-
citis. The Meckel's diverticulum is located within the distal 2 feet of the ileum. Meck-
el's diverticulitis is associated with the same complications as appendicitis and re-
quires the same treatment—prompt surgical intervention. Resection of the segment of
ileum bearing the diverticulum with end-to-end anastomosis can nearly always be
done through a McBurney incision, extended if necessary, as well as laparoscopical-
ly.
In contrast to Meckel's diverticulitis, it is extremely important to differentiate in-
tussusception from acute appendicitis as the treatment is different. Patient age is im-
portant: appendicitis is very uncommon in children younger than age 2 years, whereas
nearly all idiopathic intussusceptions occur in children younger than age 2 years. In-
tussusception occurs typically in a well-nourished infant who is suddenly doubled up
by apparent colicky pain. Between attacks of pain, the infant appears well. After sev-
eral hours, the patient usually passes a bloody mucoid stool. A sausage-shaped mass
may be palpable in the right lower quadrant. As the intussusception progresses distal-
ly, the right lower quadrant feels abnormally empty. The preferred treatment of intus-
susception, if seen before signs of peritonitis supervene, is reduction by barium ene-
ma, but treatment of acute appendicitis by barium enema may be catastrophic.
Crohn's Enteritis The manifestations of acute regional enteritis—fever; right
lower quadrant pain and tenderness, and leukocytosis often simulate acute appendici-
tis. Diarrhea and the infrequency of anorexia, nausea, and vomiting favour a diagno-
sis of enteritis but are not sufficient to exclude acute appendicitis. In an appreciable
percentage of patients with chronic regional enteritis, the diagnosis has been first
made at the time of operation for presumed acute appendicitis. In the presence of an
acutely inflamed distal ileum with no cecal involvement, and a normal appendix, ap-
pendectomy is indicated. Progression to chronic Crohn's ileitis is uncommon.
Perforated peptic ulcer closely simulates appendicitis if the spilled gastroduo-
denal contents gravitate down the right gutter to the cecal area and if the perforation
spontaneously seals, minimizing upper abdominal findings.
Colonic lesions Diverticulitis or perforating carcinoma of the cecum, or of that
portion of the sigmoid that lies on the right side, may be impossible to distinguish
from appendicitis. These entities should be considered in older patients. CT scanning
is often helpful in making a diagnosis in older patients with right lower quadrant pain
and atypical clinical presentations.
Epiploic appendagitis probably results from infarction of the colonic append-
age(s) secondary to torsion. Symptoms may be minimal, or there may be continuous
abdominal pain in an area corresponding to the contour of the colon, lasting several
days. Pain shift is unusual, and there is no diagnostic sequence of symptoms. The pa-
tient does not look ill, nausea and vomiting are unusual, and appetite is commonly
unaffected. Localized tenderness over the site is usual and is often associated with
marked rebound tenderness without rigidity. In 25% of reported cases, pain has per-
sisted or recurred until the infarcted epiploic appendage was removed.
Urinary Tract Infection Acute pyelonephritis, on the right side particularly, may
mimic a retroileal acute appendicitis. Chills, right costovertebral angle tenderness,
pyuria, and bacteriuria are usually sufficient to make the diagnosis.
Urethral Stone If the calculus is lodged near the appendix, it may simulate ret-
rocecal appendicitis. Pain referred to the labia, scrotum, or penis; hematuria; and/or
absence of fever or leukocytosis suggest the presence of a urethral stone. Pyelography
and CT scanning without oral contrast usually confirm the diagnosis.
Primary peritonitis occurs most often in patients with nephrotic syndrome, cir-
rhosis, and endogenous or exogenous immunosuppression. It rarely mimics simple
acute appendicitis, but presents a picture similar to diffuse peritonitis secondary to a
ruptured appendix. The diagnosis is made by peritoneal aspiration. If only gram-
positive cocci are seen on the Gram-stained smear, peritonitis is primary and treated
with antibiotics; if the flora are mixed or gram-negative rods, secondary peritonitis
should be suspected.
Henoch-Schönlein Purpura This syndrome usually occurs 2 to 3 weeks after a
streptococcal infection. Abdominal pain may be prominent, but joint pains, purpura,
and nephritis are also frequently present.
Yersiniosis Human infection with Yersinia enterocolitica or Y. pseudotuberculo-
sis is transmitted through food contaminated by feces or urine. Yersinia infections
cause a variety of clinical syndromes, including mesenteric adenitis, ileitis, colitis,
and acute appendicitis. Many of the infections are mild and self-limited, but some
lead to a systemic septic course with a high fatality rate if untreated. The organisms
are usually sensitive to tetracyclines, streptomycin, ampicillin, and kanamycin. A
preoperative suspicion of the diagnosis should not delay operative intervention be-
cause appendicitis caused by Yersinia cannot be clinically distinguished from appen-
dicitis from other causes. Approximately 6% of cases of mesenteric adenitis and 5%
of cases of acute appendicitis are caused by Yersinia infection.
Gynecologic Disorders The rate of false-negative appendectomies is highest in
young adult females. The finding of a normal appendix is seen in 32 to 45% of ap-
pendectomies performed in women 15 to 45 years of age. 15 Diseases of the female
internal reproductive organs that may be erroneously diagnosed as appendicitis are, in
approximate descending order of frequency, pelvic inflammatory disease, ruptured
graafian follicle, twisted ovarian cyst or tumor, endometriosis, and ruptured ectopic
pregnancy.
Pelvic Inflammatory Disease The infection is usually bilateral, but if confined
to the right tube, may mimic acute appendicitis. Nausea and vomiting often are pre-
sent in patients with appendicitis, but only in approximately 50% of those with pelvic
inflammatory disease. The greatest value of these symptoms for establishing a diag-
nosis of pelvic inflammatory disease is their absence. Pain and tenderness are usually
lower, and motion of the cervix is exquisitely painful. Intracellular diplococci may be
demonstrable on smear of the purulent vaginal discharge. The ratio of appendicitis to
pelvic inflammatory disease is low in the early phase of the menstrual cycle and high
during the luteal phase. The clinical use of all the above-mentioned distinctions has
resulted in a reduction of the incidence of negative findings on laparotomy in young
women to 15%.
Ruptured Graafian Follicle Ovulation commonly results in the spillage of suffi-
cient amounts of blood and follicular fluid to produce brief, mild, lower abdominal
pain. If the amount of fluid is unusually copious and is from the right ovary, appendi-
citis may be simulated. Pain and tenderness are rather diffuse. Leukocytosis and fever
are minimal or absent. Because this pain occurs at the midpoint of the menstrual cy-
cle, it is often called mittelschmerz.
Ruptured Ectopic Pregnancy Pregnancies may implant in the fallopian tube
(usually the ampullary portion), ovary, and, rarely, the peritoneum. Rupture of right
tubal or ovarian pregnancies can mimic appendicitis. Patients usually give a history of
abnormal menses; either missing one or two periods or noting only slight vaginal
bleeding. Unfortunately, patients do not always realize they are pregnant. The devel-
opment of right lower quadrant or pelvic pain may be the first symptom. The diagno-
sis of ruptured ectopic pregnancy should be relatively easy. The presence of a pelvic
mass and elevated level of chorionic gonadotropin is characteristic. While the leuko-
cyte count rises slightly (to approximately 14,000), the hematocrit level falls as a con-
sequence of the intra-abdominal hemorrhage. Vaginal examination reveals cervical
motion and adnexal tenderness, and a more definitive diagnosis can be established by
culdocentesis. The presence of blood and particularly decidual tissue is pathogno-
monic. The treatment of ruptured ectopic pregnancy is emergency surgery.
Twisted Ovarian Cyst Serous cysts of the ovary are common and generally re-
main asymptomatic. When right-sided cysts rupture or undergo torsion, the manifes-
tations are similar to those of appendicitis. Patients develop right lower quadrant pain,
tenderness, rebound, fever, and leukocytosis. If the mass is palpable on vaginal exam,
the diagnosis can be made easily. Both transvaginal ultrasonography and CT scanning
can be diagnostic if a mass is not palpable. Torsion requires emergent operative
treatment. If the torsion is complete or long-standing, the pedicle undergoes throm-
bosis, and the ovary and tube become gangrenous and require resection. However,
leakage of ovarian cysts resolves spontaneously and is best treated nonoperatively.
Treatment
Despite the advent of more sophisticated diagnostic modalities, the importance
of early operative intervention should not be minimized. Once the decision to operate
for presumed acute appendicitis has been made, the patient should be prepared for the
operating room. Adequate hydration should be ensured; electrolyte abnormalities cor-
rected; and pre-existing cardiac, pulmonary, and renal conditions should be ad-
dressed. Many trials have demonstrated the efficacy of preoperative antibiotics in
lowering the infectious complications in appendicitis. Most surgeons routinely ad-
minister antibiotics to all patients with suspected appendicitis. If simple acute appen-
dicitis is encountered, there is no benefit in extending antibiotic coverage beyond 24
hours. If perforated or gangrenous appendicitis is found, antibiotics are continued un-
til the patient is afebrile and has a normal white blood cell count. For intra-abdominal
infections of gastrointestinal tract origin of mild to moderate severity, the Surgical In-
fection Society has recommended single-agent therapy with cefoxitin, cefotetan, or
ticarcillin-clavulanic acid. For more severe infections, single-agent therapy with car-
bapenems or combination therapy with a third-generation cephalosporin, monobac-
tam, or aminoglycoside plus anaerobic coverage with clindamycin or metronidazole
is indicated.
Open Appendectomy Most surgeons employ either a McBurney (oblique) or
Rocky-Davis (transverse) right lower quadrant muscle-splitting incision in patients
with suspected appendicitis. The incision should be centered over either the point of
maximal tenderness or a palpable mass. If an abscess is suspected, a laterally placed
incision is imperative to allow retroperitoneal drainage and to avoid generalized con-
tamination of the peritoneal cavity. If the diagnosis is in doubt, a lower midline inci-
sion is recommended to allow a more extensive examination of the peritoneal cavity.
This is especially relevant in older patients with possible malignancy or diverticulitis.
Several techniques can be used to locate the appendix. Because the cecum is
usually visible within the incision, the convergence of the taeniae can be followed to
the base of the appendix. A sweeping lateral to medial motion can aid in delivering
the appendiceal tip into the operative field. Occasionally, limited mobilization of the
cecum is needed to aid in adequate visualization. Once identified, the appendix is
mobilized by dividing the mesoappendix, taking care to ligate the appendiceal artery
securely.
The appendiceal stump can be managed by simple ligation or by ligation and in-
version with either a purse-string or Z stitch. As long as the stump is clearly viable
and the base of the cecum not involved with the inflammatory process, the stump can
be safely ligated with a nonabsorbable suture. The mucosa is frequently obliterated to
avoid the development of mucocele. The peritoneal cavity is irrigated and the wound
closed in layers. If perforation or gangrene is found in adults, the skin and subcutane-
ous tissue should be left open and allowed to heal by secondary intention or closed in
4 to 5 days as a delayed primary closure. In children, who generally have little subcu-
taneous fat, primary wound closure has not led to an increased incidence of wound
infection.
If appendicitis is not found, a methodical search for an alternative diagnosis
must be performed. The cecum and mesentery should first be inspected. Next, the
small bowel is examined in a retrograde fashion beginning at the ileocecal valve and
extending at least 2 feet. In females, special attention should be paid to the pelvic or-
gans. An attempt is also made to examine the upper abdominal contents. Peritoneal
fluid should be sent for Gram's stain and culture. If purulent fluid is encountered, it is
imperative that the source be identified. A medial extension of the incision (Fowler-
Weir), with division of the anterior and posterior rectus sheath, is acceptable if further
evaluation of the lower abdomen is indicated. If upper abdominal pathology is en-
countered, the right lower quadrant incision is closed and an appropriate upper mid-
line incision performed.
Laparoscopy Semm first reported successful laparoscopic appendectomy in
1983, several years before the first laparoscopic cholecystectomy. However, the
widespread use of the laparoscopic approach to appendectomy did not occur until af-
ter the success of laparoscopic cholecystectomy. This may be due to the fact that ap-
pendectomy, by virtue of its small incision, is already a form of minimal-access sur-
gery. Laparoscopic appendectomy is performed under general anaesthesia. A naso-
gastric tube and a urinary catheter are placed prior to obtaining a pneumoperitoneum.
Laparoscopic appendectomy usually requires the use of three ports. Four ports may
occasionally be necessary to mobilize a retrocecal appendix. The surgeon usually
stands to the patient's left. One assistant is required to operate the camera. One trocar
is placed in the umbilicus (10 mm), with a second trocar placed in the suprapubic po-
sition. Some surgeons will place this second port in the left lower quadrant. The su-
prapubic trocar is either 10 or 12 mm, depending on whether a linear stapler will be
used. The placement of the third trocar (5 mm) is variable and is usually either in the
left lower quadrant, epigastrium, or right upper quadrant. Placement is based on loca-
tion of the appendix and surgeon preference. Initially, the abdomen is thoroughly ex-
plored to exclude other pathology. The appendix is identified by following the anteri-
or taeniae to its base. Dissection at the base of the appendix enables the surgeon to
create a window between the mesentery and base of the appendix. The mesentery and
base of the appendix are then secured and divided separately. When the mesoappen-
dix is involved with the inflammatory process, it is often best to divide the appendix
first with a linear stapler, and then to divide the mesoappendix immediately adjacent
to the appendix with clips, electrocautery, Harmonic Scalpel, or staples. The base of
the appendix is not inverted. The appendix is removed from the abdominal cavity
through a trocar site or within a retrieval bag. The base of the appendix and the
mesoappendix should be evaluated for haemostasis. The right lower quadrant should
be irrigated. Trocars are removed under direct vision.
Patients with appendiceal infiltrate are managed conservatively; bed rest, re-
stricted diet, cold compress on the area of infiltrate and antibiotic therapy. According
to resorption of infiltrate, within or after two months, elective appendectomy is per-
formed.
Treatment of appendiceal abscess must be only operative. Opening and drain-
age of the abscess, through retroperitoneal route (incision), is performed. In this
case removal of the appendix is not necessary, because of danger of bleeding, peri-
tonitis and intestinal fistula.

Antibiotic Therapy
Patients with acute appendicitis should be treated with perioperative broad-
spectrum antibiotics directed against colonic flora, including gram-positive, gram-
negative, and anaerobic organisms. Peritoneal cultures are generally not clinically
helpful in the selection of the type of antibiotics. The optimal length of antibiotic
therapy is not known. In simple cases of acute appendicitis, 24 hours or less is gener-
ally sufficient. However, in cases of perforated appendicitis, longer courses of antibi-
otics are generally used, approximately 5 to 7 days.

Outcomes
The mortality rate after appendectomy is less than 1%. The morbidity of perfo-
rated appendicitis is higher than that of nonperforated cases and is related to increased
rates of wound infection, intra-abdominal abscess formation, increased hospital stay,
and delayed return to full activity. Surgical site infections are the most common com-
plications seen after appendectomy. About 5% of patients with uncomplicated appen-
dicitis develop wound infections after open appendectomy. Laparoscopic appendec-
tomy is associated with a lower incidence of wound infections; this difference is
magnified among groups of patients with perforated appendicitis (14% versus 26%).
Patients with a fever and leukocytosis and a normal-appearing wound after appendec-
tomy undergo CT or ultrasonography to exclude an intra-abdominal abscess. Similar-
ly, if pus emanates from a fascial opening during wound inspection, an imaging study
is obtained to identify any undrained intra-abdominal fluid collections. In this situa-
tion, we place a percutaneous drain into the collection to divert the infected material
away from the fascia and facilitate wound healing. For pelvic abscesses that are lo-
cated in proximity to the rectum or vagina, we prefer ultrasound-guided transrectal or
transvaginal drainage, thereby avoiding the discomfort of a percutaneous perineal
drain. Small bowel obstruction occurs in less than 1% of patients after appendectomy
for uncomplicated appendicitis and in 3% of patients with perforated appendicitis
who are followed for 30 years. About one half of these patients present with bowel
obstruction during the first year. The risk for infertility following appendectomy in
childhood appears to be small. A history of either simple or perforated appendicitis
was sought in a large cohort of infertile patients and compared with the frequency of
appendicitis in pregnant women; no significant differences were found. There are rare
reports of appendicocutaneous or appendicovesical fistulas after appendectomy, typi-
cally for perforated appendicitis. Fistulas to the skin generally close after any local
infection is treated. Fistulas to the bladder have been successfully diagnosed and
treated laparoscopically in recent years.

Basic literature:
27.Oxford Textbook of Surgery (3-Volume Set) 2nd edition (January 15, 2000):
by Peter J. Morris (Editor), William C. Wood (Editor) By Oxford Press
28.Sabiston Textbook of Surgery 17th edition by Courtney M. Townsend Jr.,
Kenneth L. Mattox, B. Mark, MD Evers, Kenneth L., MD Mattox, Courtney
Townsend, Daniel Beauchamp, B. Mark Evers, Kenneth Mattox W.B. Saun-
ders Company (June, 2004)
29.Schwartz´s Principles of Surgery 8th Edition F.Charles Brunicardi. Copyright
©2007 The McGraw-Hill Companies.
30.Hospital surgery/ Edited by L.Kovalchuk et al.- Ternopil: Ukrmedknyha,
2004.- 472 p.
Additional literature:
1. Greenfield's Surgery: Scientific principles and practice, 4th Edition, Editors:
Mulholland, Michael W.; Lillemoe, e.a., 2006, Lippincott Williams & Wilkins.
2. Fischer, Josef E., Mastery of Surgery, 5th Edition, 2007, Lippincott Williams
& Wilkins.

Tests for initial level of knowledge, keys for tests:


1. What is the general mortality rate from acute appendicitis?
A. In the general population, it is 4/10,000
B. After rupture, appendicitis is 4–5%
C. For nonruptured appendicitis, it is 2%
D. It is 80% if an abscess has formed
E. It has increased in the past 40 years

2. Clinical findings in patient with acute appendicitis:


A. Symptoms and signs – vague colicky periumbilical or epigastric pain
B. Laboratory – moderate leukocytosis (10,000-20,000 WBC)
C. Bronchoscopy
D. Imaging – not necessary with typical appendicitis

3. Differential diagnosis of acute appendicitis includes all of the following


EXCEPT:
A. Gastroenteritis (typically with NVD)
B. Gynaecological disorders
C. Diverticulitis
D. Perforated colonic cancer
E. Budd Chiari’s disease

4. Clinical findings associated with acute appendicitis include all of the following
EXCEPT
E. Focal tenderness at Mc Burney's point
F. A positive Rovsing’s sign
G. Pyuria
H. A positive Dunphy’s sign
I. Leukocytosis

5. Correct statements concerning acute appendicitis include all of the following


EXCEPT
A. Obstruction of the lumen is believed to be the major cause of acute ap-
pendicitis
B. When not treated appropriately, appendicitis remains a potentially lethal
condition.
C. Right lower quadrant pain and tenderness are typical symptoms of acute
appendicitis
D. Approximately 1% of all appendectomy specimens will contain a neo-
plasm
E. Heparin infusion is the treatment of choice

6. The appendix receives blood supply from


A.Artery colica media
B. Artery mesenterica superior
C. Artery femoralis
D.Artery ileocolica
E. Artery tibialis

7. In case of antibiotic therapy after appendectomy you must do:


A. To test on sensitivity to antibiotics
B. Probe Foharti
C. Pelvic CT scan
D. Probe Rovsing

8. Indications for antibiotic therapy after appendectomy is:


A. Peritonitis
B. Tachycardia
C. Symptom of Blumberg
D.Laparoscopic appendectomy
E. General narcosis

9. For a acute appendicitis at the elderly it is characteristic


A. Jaundice
B. Patients have decreased findings of peritonitis on examination, and may have a
delayed leukocytosis
C. Increased pain in abdomen with coughing or other movement
D. Anorexia, nausea, and/or vomiting
E. Early perforation and abscess formation

10. Instrumental methods of diagnostics of an acute appendicitis are:


A. US, CT scan
B. Barium enema
C. Colonoscopy
D. Abdominal radiographs
E. Contrast venography

Keys for tests

1 2 3 4 5 6 7 8 9 10
A C E C E D A A E A

Tests for final level of knowledge, keys for tests:


1. Pylephlebitis is:
A. Septic thrombosis of branches of vena portae
B. Syndrome Budd-Chiari
C. Septic thrombosis of cava vein
D. Thrombosis of upper mesenteric vein
2. Appendiceal infiltrate is result of:
A. Simple superficial appendicitis.
B. Phlegmonous appendicitis
C. Gangrenous appendicitis.
D. Gangrenous perforated appendicitis.
E. All are correct
3. Retrocecal appendicitis can be diagnosed by means of following symptoms:
A. Rovsing's sign
B. Obraztsov's sign
C. Sitkovsky's sign
D. Blumberg's sign

4. Dissection of abscess around appendix vermiform is made from a cut on:


A. By Pirogov
B. By Fedorov
C. By Mc Burney.
D. By Volkoviche-Dyakonove

5. For an acute appendicitis at pregnant women it is characteristic


A. Hematuria
B. Pyuria
C. Localisation of pain over pu-
bis
D. Dysuria
E. Localisation of pains above and lateral, than in a typical
case

6. The contraindications for surgical treatment of acute appendicitis are all of the fol-
lowing EXCEPT?
A. Obstruction of deep veins
B. Phlegmonous an appendicitis
C. Appendiceal infiltrate
D. Obesity of III degree
E. Acute myocardium infarction

7. For empirical antibacterial therapy after appendectomy is rational to use


A. Penicillins
B. Quinolones
C. Sulphanilamides
D. Cephalosporines
E. Macrolides
8. Abscess of space of Douglas is diagnosed with the help of:
A. Digital rectal examination
B. Ultrasonography, pelvic CT-scan
C. Researches per vaginum
D. Auscultation an abdomen
E. Colonoscopy

9. After appendectomy concerning destructive appendicitis an abdominal cavity must


be drained by:
A. Rubber strip
B. Tubular drainage
C. Not drain
D. Gauze tampon
E. Tubular drainage blanket of silicon

10. Appendectomy in patients, sufferings obesity III degree must be executed with
the help of:
A. Classic method
B. Laparoscopy method
C. Not to do
D. Combined method

Keys for tests

1 2 3 4 5 6 7 8 9 10
A C B A E C D B E B

Tasks for final level of knowledge.

1. A 23-year-old man presents to the emergency room complaining of pain in


right iliac area. The pain arise in epigastric area 4 hours earlier and dislocated in the
right iliac area, appeared nausea, and the patient wants to lie down on the right side
with legs failed. On physical examination patient has temperature 37,5C0, palpation in
the right iliac area revealed local pain, tension of muscle of abdominal wall, positive
Rovsing's symptom and symptom of peritoneal irritation, Leukocytosis reached 9,000
mm3. What is the patient's problem?

The answer is acute appendicitis.

2. For the woman of 32 years, after dissection of abdominal region during an op-
eration concerning sharp appendicitis, the presence of blood is exposed in an ab-
dominal region. What diagnosis is possible? What actions should the surgeon take?

The answer is extra-uterine pregnancy or apoplexy of ovary. It is needed to


execute laparotomy, revision of abdominal region and to execute the pathogenet-
ic grounded operation.
3. A 79-year-old man has had abdominal pain for 4 days. An operation is per-
formed, and a gangrenous appendix is removed. The stump is inverted. Why does
acute appendicitis in elderly patients and in children have a worse prognosis?

The answer is the omentum and peritoneal cavity appears to be less efficient
in localizing the disease in these age groups.

4. A 17-year-old female model presents to the emergency room with a 1-day


history of lower abdominal pain. On examination she is most tender in the right low-
er quadrant (RLQ) and also has pelvic tenderness. White blood cell (WBC) count is
13,000 and temperature is 37.6°C. A provisional diagnosis of uncomplicated appen-
dicitis is made and laparoscopic appendectomy is offered. Regarding laparoscopic
appendectomy which of the following is TRUE?

The answer is procedure cost is less than with open technique.

5. A 29-year-old woman presents to her physician’s office with pain in the right
iliac fossa. Examination reveals tenderness in this region. Her last menstrual cycle
was 2 weeks previously and findings on gynaecologic examination and leukocyte
count are normal. A provisional diagnosis of acute appendicitis is made. She should
be informed that operations to treat this condition reveal acute appendicitis in what
percentage of cases?

The answer is a small percentage of cases


Materials for the self-study of the students

Main tasks Notes (instructions)


Repeat: 1. Represent methods of diagnostics
1. Normal Anatomy of the abdo- of an acute abdomen.
men’s cavity 2. Causes of peritonitis.
2. Normal Physiology of intestinal 3. Surgical acute abdominal condi-
tract. tions.
3. Pathophysiology and etiology of 4. Choose and write methods of sur-
the acute abdominal disease. gical and conservative treatment of
4. Special investigations in patients acute appendicitis.
with gastrointestinal disease.
5. 5Causes of peritonitis
Study: 1. Represent methods of diagnostics
1. Clinical symptoms and signs of acute of acute appendicitis.
appendicitis. 2. To make a differential diagnos-
2. Clinical examination and special in- tics of acute appendicitis.
vestigations in patients with acute 3. Surgical methods and technique
appendicitis. in treatment of acute appendicitis.
3. Principles of surgical treatment of
patients with complication's forms of
acute appendicitis.

Study guide #7.1


“Acute cholecystitis. Anatomo-phisiological features of biliary ducts. Etiology
and pathogenethis.Classification. Manifestation, diagnostics, differential diag-
nostics. Surgical tactics. Features of clinical picture of acute cholecystitis, diag-
nostic and medical tactics in elderly patients with concomitant pathology. Com-
plication of acute cholecystitis and cholecystectomy.”

Overview:
In spite of great success of surgical treatment the problem of acute cholecysti-
tis during last decades is still very important, because of spreading of this disease (9-
10 per 1000 of population), and in the connection of big amount of discussing ques-
tions about clinical tactic, terms of surgical intervention, the degree of risk for old
people, pathogenesis and classification of the disease.
Acute inflammation gallbladder now is admitted by all doctors as a surgical
disease. However only nearly 30% of sick people with such diagnose are operated.
And in this case also cholecystectomia (as the most widespread operation at acute
cholecystitis) takes the second place after appendectomy. Implementation of difficult
surgical interferences on bilious ways by many surgeons who not always have a nec-
essary experience, and sometimes technical providing, that is necessary for the ob-
servation of extrahepatic bile duct during an operation, considerably increased the
percent of complications, growth of frequency so-called to the postcholecysistomic
syndrome. The difficult topographoanatomic mutual relations of liver, extrahepatic
bile duct and vessels, lots of anomalies require from the surgeon deep knowledge of
anatomy of hepatopancreatoduodenalis area.

Educational aims:

28. To collect the anamnesis data and evaluate clinical inspection of


gallbladder illnesses.
29. To know an etiology and pathogenesis of gallbladder illnesses.
30. To know clinical pictures of gallbladder illnesses.
31. To determine mobility complications of gallbladder illnesses.
32. To make the plan for inspection of gallbladder illnesses.
33. To carry out the received analysis data at laboratory and instrumental
methods of inspection in gallbladder illness.
34. To define indications for surgical treatment gallbladder illness or to ap-
point conservative treatment.
35. To take care of gallbladder illness.
36. To estimate work capacity in gallbladder illness.
A student must know:

1. Anatomo-physiological features of liver and extrahepatic bile duct, most fre-


quent anomalies of development.
2. Etiology and pathogeny of acute cholecystitis.
3. Methods of inspection of the functional state of liver and extrahepatic bile duct.
4. Clinical examination and tests.
5. Ultrasonography and Endoscopic Retrograde Cholangiography and Endoscop-
ic Ultrasound.
6. Clinical symptomatic of acute cholecystitis and its complicated forms.
7. Differential diagnostic of acute cholecystitis.
8. Contra-indication for surgical and conservative methods of treatment.
9. Methods of surgical treatment in different forms of acute cholecystitis.

A student should be able to:

1. To collect anamnesis data and complaints of patient with acute cholecystitis, to


underline importance of them.
2. To exemine a patient to define the volume of the special, instrumental, functional
and biochemical researches.
3. To estimate the received information.
4. To define tactic and volume of future treatment.
5. To carry out differential diagnostics of acute cholecystitis.
6. To make the plan of inspection acute cholecystitis.
7. To establish indications for surgical treatment and conservative treatment.
8. To prepare patients for the operation, to assist during the time of operation, to
give indications for a patient in post-operation period.
9. To predict possible complications during the treatment of patients with acute
cholecystitis, to give suggestions in relation to prevent these complications.
Terminology

Acute cholecystitis acute inflammation of gallbladder


Cholecystectomy is an operation removing of gallbladder
in this operative treatment gall-bladder is exposed
Atypical cholecystectomy along its longitudinal axis, its contents are then re-
moved and with the control of the finger, position of
neck is determined, and then deleted
Laparoscopic complex of special apparatus is required for laparos-
cholecystectomy copy: operating laparoscope with a video camera and
colored video monitor

Cholecystostomy is an operation, that is based on creation of external


fistula of gallbladder
Laparoscopic cholecys- is imposition of external fistula on gallbladder under
tostomy the control of laparoscope.
Choledochoscopy is a method of endoscope examination of biliary
ducts by choledoscope during choledochostomy

Content

ANATOMY

The gallbladder

The gallbladder is a pear-shaped sac, about 7 to 10 cm long with an average


capacity of 30 to 50 ml. When obstructed, the gallbladder can distend markedly and
contain up to 300 ml. the gallbladder is located in a fossa on the inferior surface of
the liver that is in line with the anatomic division of the liver into right and left liver
lobes. The gallbladder is divided into four anatomic areas: the fundus, the corpus
(body), the infundibulum, and the neck. The fundus is the rounded, blind end that
normally extends 1 to 2 cm beyond the liver's margin. It contains most of the smooth
muscles of the organ, in contrast to the body, which is the main storage area and con-
tains most of the elastic tissue. The body extends from the fundus and tapers into the
neck, a funnel-shaped area that connects with the cystic duct. The neck usually fol-
lows a gentle curve, the convexity of which may be enlarged to form the infundibu-
lum or Hartman's pouch. The neck lies in the deepest part of the gallbladder fossa and
extends into the free portion of the hepatoduodenal ligament.

The same peritoneal lining that covers the liver covers the fundus and the infe-
rior surface of the gallbladder. Occasionally the gallbladder has a complete peritoneal
covering, and is suspended in a mesentery off the inferior surface of the liver, and
rarely is it embedded deep inside the liver parenchyma (an intrahepatic gallbladder).
The gallbladder is lined by a single, highly-folded, tall columnar epithelium
that contains cholesterol and fat globules. The mucus secreted into the gallbladder
originates in the tubuloalveolar glands found in the mucosa lining the infundibulum
and neck of the gallbladder, but are absent from the body and fundus. The epithelial
lining of the gallbladder is supported by a lamina propria. The muscle layer has circu-
lar longitudinal and oblique fibers, but without well-developed layers. The perimus-
cular subserosa contains connective tissue, nerves, vessels, lymphatics, and adipo-
cytes. It is covered by the serosa except where the gallbladder is embedded in the liv-
er. The gallbladder differs histologically from the rest of the gastrointestinal tract in
that it lacks a muscularis mucosa and submucosa. The cystic artery that supplies the
gallbladder is usually a branch of the right hepatic artery (>90% of the time). The
course of the cystic artery may vary, but it nearly always is found within the hepato-
cystic triangle, the area bound by the cystic duct, common hepatic duct, and the liver
margin (triangle of Calot). When the cystic artery reaches the neck of the gallbladder,
it divides into anterior and posterior divisions. Venous return is carried either through
small veins that enter directly into the liver, or rarely to a large cystic vein that carries
blood back to the portal vein. Gallbladder lymphatics drain into nodes at the neck of
the gallbladder. Frequently, a visible lymph node overlies the insertion of the cystic
artery into the gallbladder wall. The nerves of the gallbladder arise from the vagus
and from sympathetic branches that pass through the celiac plexus.The preganglionic
sympathetic level are T8 and T9. Impulses from the liver, gallbladder, and the bile
ducts pass by means of sympathetic afferent fibers through the splanchnic nerves and
mediate the pain of biliary colic.The hepatic branch of the vagus nerve supplies cho-
linergic fibers to the gallbladder, bile ducts, and the liver. The vagal branches also
have peptide-containing nerves containing agents such as substance P, somatostatin,
enkephalins, and vasoactive intestinal polypeptide (VIP).

The Bile Ducts

The extrahepatic bile ducts consist of the right and left hepatic ducts, the com-
mon hepatic duct, the cystic duct, and the common bile duct or choledochus. The
common bile duct enters the second portion of the duodenum through a muscular
structure, the sphincter of Oddi.
The left hepatic duct is longer than the right and has a greater propensity for di-
latation as a consequence of distal obstruction. The two ducts join to form a common
hepatic duct, close to their emergence from the liver. The common hepatic duct is 1 to
4 cm in length and has a diameter of approximately 4 mm. It lies in front of the portal
vein and to the right of the hepatic artery. The common hepatic duct is joined at an
acute angle by the cystic duct to form the common bile duct.

The length of the cystic duct is quite variable. It may be short or absent and
have a high union with the hepatic duct, or long and run parallel, behind, or spiral to
the main hepatic duct before joining it, sometimes as far as at the duodenum. Varia-
tions of the cystic duct and its point of union with the common hepatic duct are surgi-
cally important. The segment of the cystic duct adjacent to the gallbladder neck bears
a variable number of mucosal folds called the spiral valves of Heister. They do not
have any valvular function, but may make cannulation of the cystic duct difficult.

The common bile duct is about 7 to 11 cm in length and 5 to 10 mm in diame-


ter. The upper third (supraduodenal portion) passes downward in the free edge of the
hepatoduodenal ligament, to the right of the hepatic artery and anterior to the portal
vein. The middle third (retroduodenal portion) of the common bile duct curves behind
the first portion of the duodenum and diverges laterally from the portal vein and the
hepatic arteries. The lower third (pancreatic portion) curves behind the head of the
pancreas in a groove, or traverses through it and enters the second part of the duode-
num. There the pancreatic duct frequently joins it. The common bile duct runs
obliquely downward within the wall of the duodenum for 1 to 2 cm before opening on
a papilla of mucous membrane (ampulla of Vater), about 10 cm distal to the pylorus.
The union of the common bile duct and the main pancreatic duct follows one of three
configurations. In about 70% of people these ducts unite outside the duodenal wall
and traverse the duodenal wall as a single duct. In about 20%, they join within the
duodenal wall and have a short or no common duct, but open through the same open-
ing into the duodenum. In about 10%, they exit via separate openings into the duode-
num. The sphincter of Oddi, a thick coat of circular smooth muscle, surrounds the
common bile duct at the ampulla of Vater. It controls the flow of bile, and in some
cases pancreatic juice, into the duodenum.
The extrahepatic bile ducts are lined by a columnar mucosa with numerous
mucous glands in the common bile duct. A fibroareolar tissue containing scant
smooth muscle cells surrounds the mucosa. A distinct muscle layer is not present in
the human common bile duct. The arterial supply to the bile ducts is derived from the
gastroduodenal and the right hepatic arteries, with major trunks running along the
medial and lateral walls of the common duct (sometimes referred to as 3 o'clock and 9
o'clock). These arteries anastomose freely within the duct walls. The density of nerve
fibers and ganglia increase near the sphincter of Oddi, but the nerve supply to the
common bile duct and the sphincter of Oddi is the same as for the gallbladder.

Diagnostic Studies

A variety of diagnostic modalities are available for the patient with suspected
disease of the gallbladder and the bile ducts. In 1924 the diagnosis of gallstones was
improved significantly by the introduction of oral cholecystography by Graham and
Cole. For decades it was the mainstay of investigation for gallstones. In the 1950 bili-
ary scintigraphy was developed and later transhepatic and endoscopic retrograde
cholangiography, allowing imaging of the biliary tract. Later ultrasonography, com-
puted tomography (CT), and magnetic resonance imaging (MRI), vastly improved the
ability to image the biliary tract.

Blood Tests When patients with suspected diseases of the gallbladder or the ex-
trahepatic biliary tree are evaluated, a complete blood count (CBC) and liver function
tests are routinely requested. An elevated white blood cell (WBC) count may indicate
or raise suspicion of cholecystitis. If associated with an elevation of bilirubin, alkaline
phosphatase, and aminotransferase, cholangitis should be suspected. Cholestasis, an
obstruction to bile flow, is characterized by an elevation of bilirubin (i.e., the conju-
gated form), and a rise in alkaline phosphatase. Serum aminotransferases may be
normal or mildly elevated. In patients with biliary colic, blood tests will typically be
normal.

Ultrasonography An ultrasound is the initial investigation of any patient sus-


pected of disease of the biliary tree. It is noninvasive, painless, does not submit the
patient to radiation, and can be performed on critically ill patients. It is dependent up-
on the skills and the experience of the operator and it is dynamic (i.e., static images
do not give the same information as those obtained during the ultrasound investiga-
tion itself). Adjacent organs can frequently be examined at the same time. Obese pa-
tients, patients with ascites, and patients with distended bowel may be difficult to ex-
amine satisfactorily with an ultrasound.An ultrasound will show stones in the
gallbladder with sensitivity and specificity of over 90%. Stones are acoustically dense
and reflect the ultrasound waves back to the ultrasonic transducer. Because stones
block the passage of sound waves to the region behind them, they also produce an
acoustic shadow (Fig. 31-6). Stones also move with changes in position. Polyps may
be calcified and reflect shadows, but do not move with change in posture. Some
stones form a layer in the gallbladder; others a sediment or sludge. A thickened
gallbladder wall and local tenderness indicate cholecystitis. The patient has acute
cholecystitis if a layer of edema is seen within the wall of the gallbladder or between
the gallbladder and the liver. When a stone obstructs the neck of the gallbladder, the
gallbladder may become very large, but thin walled. A contracted, thick-walled
gallbladder indicates chronic cholecystitis.

The extrahepatic bile ducts are also well visualized by ultrasound, except for
the retroduodenal portion. Dilation of the ducts in a patient with jaundice establishes
an extrahepatic obstruction as a cause for the jaundice. Frequently the site, and some-
times the cause of obstruction, can be determined by ultrasound. Small stones in the
common bile duct frequently get lodged at the distal end of it, behind the duodenum,
and are therefore difficult to detect. A dilated common bile duct on ultrasound, small
stones in the gallbladder, and the clinical presentation allow one to assume that a
stone or stones are causing the obstruction. Periampullary tumors can be difficult to
diagnose on ultrasound, but beyond the retroduodenal portion, the level of obstruction
and the cause may be visualized quite well. Ultrasound can be helpful in evaluating
tumor invasion and flow in the portal vein, an important guideline for resectability of
periampullary tumors.

Oral Cholecystography Once the diagnostic procedure of choice for gallstones,


oral cholecystography has largely been replaced by ultrasonography. It involves oral
administration of a radiopaque compound that is absorbed, excreted by the liver, and
passed into the gallbladder. Stones are noted on a film as filling defects in a visual-
ized, opacified gallbladder. Oral cholecystography is of no value in patients with in-
testinal malabsorption, vomiting, obstructive jaundice, and hepatic failure.

Biliary Radionuclide Scanning (HIDA Scan) provides a noninvasive evalua-


tion of the liver, gallbladder, bile ducts, and duodenum with both anatomic and func-
tional information. 99m-Technetium-labeled derivatives of dimethyl iminodiacetic acid
(HIDA) are injected intravenously, cleared by the Kupffer cells in the liver, and ex-
creted in the bile. Uptake by the liver is detected within 10 minutes, and the gallblad-
der, the bile ducts, and the duodenum are visualized within 60 minutes in fasting sub-
jects. The primary use of biliary scintigraphy is in the diagnosis of acute cholecystitis,
which appears as a nonvisualized gallbladder, with prompt filling of the common bile
duct and duodenum. Evidence of cystic duct obstruction on biliary scintigraphy is
highly diagnostic for acute cholecystitis. The sensitivity and specificity for the diag-
nosis is about 95% each. False-positive results are increased in patients with gallblad-
der stasis, as in critically ill patients and in patients receiving parenteral nutrition.
Filling of the gallbladder and common bile duct with delayed or absent filling of the
duodenum indicates an obstruction at the ampulla. Biliary leaks as a complication of
surgery of the gallbladder or the biliary tree can be confirmed and frequently local-
ized by biliary scintigraphy.

Computed Tomography Abdominal CT scans are inferior to ultrasonography


in diagnosing gallstones. The major application of CT scans is to define the course
and status of the extrahepatic biliary tree and adjacent structures. It is the test of
choice in evaluating the patient with suspected malignancy of the gallbladder, the ex-
trahepatic biliary system, or nearby organs, in particular the head of the pancreas. Use
of CT scan is an integral part of the differential diagnosis of obstructive jaundice.
Spiral CT scanning provides additional staging information, including vascular in-
volvement in patients with periampullary tumors.

Percutaneous Transhepatic Cholangiography An intrahepatic bile duct is ac-


cessed percutaneously with a small needle under fluoroscopic guidance. Once the po-
sition in a bile duct has been confirmed, a guidewire is passed and subsequently a
catheter passed over the wire (Fig. 31-8). Through the catheter, a cholangiogram can
be performed and therapeutic interventions done, such as biliary drain insertions and
stent placements. Percutaneous transhepatic cholangiography (PTC) has little role in
the management of patients with uncomplicated gallstone disease, but is particularly
useful in patients with bile duct strictures and tumors, as it defines the anatomy of the
biliary tree proximal to the affected segment. As with any invasive procedure, there
are potential risks. For PTC these are mainly bleeding, cholangitis, bile leak, and oth-
er catheter-related problems.

Magnetic Resonance Imaging Available since the mid-1990, MRI provides


anatomic details of the liver, gallbladder, and pancreas similar to those obtained from
CT. Using MRI with newer techniques and contrast materials, accurate anatomic im-
ages can be obtained of the bile ducts and the pancreatic duct. It has a sensitivity and
specificity of 95 and 89%, respectively, at detecting choledocholithiasis. If available,
MRI with magnetic resonance cholangiopancreatography (MRCP) offers a single
noninvasive test for the diagnosis of biliary tract and pancreatic disease.

Endoscopic Retrograde Cholangiography and Endoscopic Ultrasound Using


a side-viewing endoscope, the common bile duct can be cannulated and a cholangio-
gram performed using fluoroscopy. The procedure requires intravenous sedation for
the patient. The advantages of endoscopic retrograde cholangiography (ERC) include
direct visualization of the ampullary region and direct access to the distal common
bile duct, with the possibility of therapeutic intervention. The test is rarely needed for
uncomplicated gallstone disease, but for stones in the common bile duct, in particular
when associated with obstructive jaundice, cholangitis, or gallstone pancreatitis, ERC
is the diagnostic and often therapeutic procedure of choice. Once the endoscopic
cholangiogram has shown ductal stones, sphincterotomy and stone extraction can be
performed, and the common bile duct cleared of stones. In the hands of experts, the
success rate of common bile duct cannulation and cholangiography is more than 90%.
Complications of diagnostic ERC include pancreatitis and cholangitis, and occur in
up to 5% of patients. An endoscopic ultrasound requires a special endoscope with an
ultrasound transducer at its tip. The results are operator dependent, but offer noninva-
sive imaging of the bile ducts and adjacent structures. It is of particular value in the
evaluation of tumors and their resectability. The ultrasound endoscope has a biopsy
channel, allowing needle biopsies of a tumor under ultrasonic guidance. Endoscopic
ultrasound also has been used to identify bile duct stones, and although it is less sen-
sitive than ERC, the technique is less invasive.

ACUTE CHOLECYSTITIS

Acute cholecystitis is inflammation of gall-bladder. In frequency of includence


it is second only to appendicitis and comprises about 10 % of all acute surgical dis-
eases of organs of abdominal cavity.

Etiology and pathogenesis

In etiology of cholecystitis major factors considered are the following: infec-


tion, incoordination in passage of bile and metabolic disturbance. All of them prede-
termine formation of calculus.

On bacteriological examination of secretions of gall-bladder colon bacillus,


staphylococcus and enterococcus are seen. Rarerly there is streptococcus and other
microorganisms.

Considerably more frequently (70-80 %) it is seen in women, in who during


pregnancy the passage of bile in duodenum is always violated. It is promoted by life-
style with lack of mobility and other types of hypodynamia.

Pathomorphology

The catarrhal develops at acute cholecystitis fibrinogenous and Suppuration in-


flammation. The wall of gall-bladder is thickened, edematous, and hyperemic with
stratifications of fibrin and pus. Progress of process can lead to necrosis (gangrene) of
the gall-bladder.

Classification

Acute cholecystitis is divided into:


I. Acute calculous cholecystitis
II. Acute Acalculous cholecystitis
1. Catarrhal.
2.Phlegmonous.
3. Gangrenous.
4. Perforated.
5.Complicated:
a) Hydropsy;
b) Empyema;
c) Pancreatitis;
d) Icterus;
e) Hepatitis;
f) Cholangitis;
g) Infiltrate;
h) Abscess;
i) Hepato-renal insufficiency;
j) Peritonitis (local, diffuse, general).

Clinical management

The disease, as a rule, begins after violation of diet: intake of plenty of rich,
meat food, especially in combination with strong drinks.

Pain syndrome. Characteristic for it is severe pain in right hypochon-drium and


epigastric area with radiation to right supraclavicular area and right shoulder. If pain
syndrome is strongly expressed and is cramp-like in character, it is named hepatic
colic.

Dyspepsia syndrome. Frequent symptoms which disturb a patient, are nausea,


frequent vomiting, initially by gastric contents, and later - by bile. Later feeling of
fullness of abdomen, delay of emptying and gases often follow.

Examination. During examination in almost all patients slight icterus of sclera


even with normal secretion of bile can be observed. Tongue, as a rule, is white-grey
in colour. Patients complain of dryness of mouth. In difficult cases the tongue is usu-
ally dry, assessed by white stratification with a yellow spot in the center.
Increase of temperature of body is brief and insignificant (on the average to
37,2 С) but in catarrhal cholecystitis the temperature is within the limits of 38 С at
its destructive forms.

Tachycardia to a certain extent testifies the degree of intoxication. In the initial


phase of disease the pulse, usually, is relevant to the tempera ture, and with progress
of the disease, especially with development of peritonitis, it becomes rapid and low
volume.

During palpation tenderness in the place of crossing of right costal margin with
the external edge of direct muscle of stomach can be observed (the Kehr's point). By
superficial and deep palpation of right hypo-chondrium, as a rule, tenderness, in-
creased size of gall-bladder is exposed, that can be important as a symptom.

The following characteristic signs are elicited in acute cholecystitis:


- Murphy's sign is a delay of breathing during palpation of gall-bladder on inspira-
tion.
- Kehr's sign is an increase of pain at pressure on the area of gall-bladder, especially
on deep inspiration.
- Ortner's sign - tenderness on light percussion on right costal margin by the edge of
the palm.
- Mussy's sign - tenderness on palpation between the two of right sternocleidomastoid
muscle.
- Blumberg's sign is the increase of pain on rapidly removing fingers which palpates
the abdominal wall. This symptom is not pathognomic for cholecystitis but matters
very much in diagnosis of peritonitis.
It is also needed to mark the importance of grading of symptoms: acutely posi-
tive, poorly positive, and doubtful, absent.
Symptoms of acute cholecystitis can increase during 2-3 hours, and then with-
out any treatment, by use of hot-water bottle or with the start of conservative treat-
ment, quickly subside and disappear completely. It always means that the cause of
acute inflammation is liquidated.
Destructive cholecystitis presents with the most difficult clinical picture. Thus,
gangrenous cholecystitis as a rule is associated with the acutely expressed phenome-
na of intoxication and is accompanied by the features of biliary peritonitis. The perfo-
ration can complicate phlegmonous or gangrenous cholecystitis and then there is
sudden worsening of the patient's condition. Initially it starts with the sudden increase
of pain and rapid progress of peritonitis. But it is needed to remember that such clini-
cal picture can develop only in case of perforation with leakage of gall-bladder con-
tents in free abdominal cavity.
Laboratory analysis. Leukocytosis within the limits of 10xl0 9 /L and more,
shift of leukocytic formula to the left, lymphopenia and increased ESR.
Sonographic examination of gall-bladder can reveal the increase in its sizes,
bulge of walls, development of perivesical abscesses, presence or absence of sludge
and their sizes.
Roentgenoscopy examination of organs of abdominal cavity allows establish-
ing the presence X-ray photography-positive of stones with contents of calcium in the
projection of gall-bladder.
Clinical variants and complications

Clinical features of acute cholecystitis is various and depends on the many


causative factors, among which degree of violation of transfer of bile by the cystic
duct and choledochus, virulence of infection, presence or absence of pancreato-cystic
reflux by pancreatic juice are the most important. In addition the preceding anatomic
and functional changes of gall-bladder and adjoining organs, and also the state of pro-
tective and regulator mechanisms of patient also play a role.

Features of acute cholecystitis in the older age of the patients. In them there is
increased frequency of development of destructive forms of cholecystitis and compli-
cation by peritonitis are characteristic. Thus, it is needed to state that such changes in
gall-bladder can develop within the first hour of onset of peritonitis as a result of per-
foration of gall-bladder.

Hydrops of gallbladder is aseptic inflammation of the gall-bladder that arises as


a result of blockade of cystic duct by stones or sludge. The bile from a gallbladder is
sucked in, and on replacement transparent exudation accumulates over it. During pal-
pation gall-bladder is palpable due to the increase in size.
Empyema of gallbladder is a consequence of untimely correction of hydrops
that with repeated infection is transformed into a new form. Gall-bladder in such pa-
tients is palpated as a dense, moderately painful mass; however, the symptoms of irri-
tation of peritoneum, as a rule, are absent. In blood leucocytosis with the shift of for-
mula of blood to the left is present.

Biliary pancreatitis. Worsening of the patient's condition, appearance of pain,


frequent vomiting, signs of cardio-vascular insufficiency, increase amylase in urine,
presence of infiltrate in epigastric area and positive Voskresensky's and Mayo-
Robson's symptoms are its basic signs.

Icterus arises up with violation of passage of bile in duodenum as a result of


obliteration choledochus by stone, by sludge or due to edema of head of pancreas.
Thus icteric sclera, bilirubinemia, dark urine and light: color stool are present.

Cholangitis. The Sharko triad is characteristic for the patient with this patholo-
gy. Next to pain syndrome and icterus, the temperature of body rises to 38-39 С,
there is a fever, high leucocytosis and abnormality of functional tests of liver is ob-
served.

Hepatitis presents by icterus, growth of the phenomena of general weakness,


increase in the blood of indexes of alanine aminotransferase and asparaginase and al-
kaline phosphatase. Liver in this pathology tender on palpation with well defined
borders.

Infiltrate is a complication that can arise on 3-4 days after the attack of acute
cholecystitis. Dull pain presence of dense mass with unclear contours in right hypo-
chondrium, increase of temperature of body to 37,5-38 °C, negative symptoms of irri-
tation of peritoneum are characteristic for it.

Abscess. Patients with this pathology complain about high temperature, pain in
the right superior quadrant of abdomen, where painful mass is palpated, there is fever,
general weakness, absence of appetite, icterus and sometimes vomiting. Roentgeno-
logic^ in right hypochondrium the horizontal level of fluid and gas is observed above
it High leucocytosis with the shift of leukocytic formula to the left is present in blood.
Hepato-renal insufficiency often arise in the most severe forms of cholecystitis.
The general condition of patient is bad, acutely expressed intoxication, hallucinations,
delirium, oliguria and anuria are observed.

Peritonitis is the most frequent complication during the perforation of gall-


bladder into the free abdominal cavity. Patients skin is cold, clammy, pale, arterial
pressure falls, pulse is rapid. During the inspection the tension of muscles of anterior
abdominal wall is marked, positive guarding sign in the right half of abdomen or
along it is present.

Diagnosis program

1. History and physical methods of inspection.


2. Roentgenoscopy of organs of abdominal cavity.
3. Sonography.
4. General analysis of blood and urine.
5. Urinary diastase.
6. Biochemical blood test (bilirubin, amylase, alanine aminotransferase, asparaginase,
alkaline phosphatase, remaining nitrogen, creatinine).
7. Coagulogram.

Differential diagnosis

Perforated ulcer. For this disease the Mondor's triad (stabbing pain, tension of
muscles of anterior abdominal wall and history of ulcer) and positive Spizharskyy's
symptom are characteristic (disappearance of hepatic dullness). During roentgenos-
copy examination of organs of abdominal cavity in a patient, pneumoperitoneum is
visible as sickle-shaped strip under the right or left dome of diaphragm.

Renal colic. Pain of right renal colic also can be located in right hypochondri-
um. However, it is always accompanied by disorders of urination, and at cholecysti-
tis, as a rule, is not present. Renal pain always radiates downward along the ureter, to
genital region. Except, for this pathology micro- or macrohematuria, presence of
stone, exposed at sonography and on survey urogram, absence of function of kidney
during chromocystoscopy can be characteristic.
Acute appendicitis. It is need always to remember that the subhepatic location
of the pathologically changed appendix may also present with pain in right hypo-
chondrium. However, for patients with acute appendicitis beginning of pain in epigas-
tric area, absence of history of hepatic diseases, expressed dyspeptic phenomena, in-
flammatory changes on the gall-bladder at sonography are inherent.

Myocardial infarction. The so called cholecysto-cardiac syndrome which quite


often imitates pain of stenocardia. Electrocardiography examination is decisive in es-
tablishment of diagnosis. However, laparoscopy is performed in doubtful cases.

Pancreatitis. Acute pancreatitis is accompanied by the expressed pain in the


epigastric area of belt-like character. On palpation in left costovertebral angle patients
feel pain (the Mayo-Robson's symptom), and it is not at cholecystitis.

Techniques and choice of treatment method

Conservative treatment of the patients with destructive or complicated chole-


cystitis must include:
1. Bed rest.
2. Hunger orally for 1-2 days, in the following table № 5 by Peuzner.
3. Application of cold on right hypochondrium.
4. Spasmolytics (sulfate of atropine, platyphyllin, papaverin, no-shparum, baralgin).
5. Antibacterial therapy:
a) semisynthetic penicillin (ampicillin, oxacilline, ampiox);
b) cephalosporin (kefzol, klaforan);
c) nitrofurans (nitrofurantoin, furazolidon);
d) sulfanilamides (biseptol, ethazole, norsulfazole).
6. Inhibitors of protease (contrical, trasilol, gordox, antagosan).
7. Desensitizing preparation (dimedrole, pipolphen, tavegile).
8. Disintoxication therapy (neohemodes, reopolyglucine).
9. Vitamins (C, Bl, B6, B12 vitamins).
Intensive pain in patients is controlled by morphin (1 % - 1,0) together with in-
troduction of atropin (0,1 % _
1,0). Less acute pain controlled with baralgin,
platyphyllin and papaverin. If the patient's condition does not get better, it is neces-
sary to offer urgent surgical interference.
Indication to surgical treatment. All forms of acute calculous cholecystitis, de-
structive and complicated forms of noncalculous cholecystitis (except for infiltrative),
and also acute catarrhal cholecystitis for which conservative treatment was ineffec-
tive, are subjected to surgical treatment.
Overhead-middle laparotomy is considered the best choice. However, many
surgeons give advantage to pararectal laparotomy, oblique (by Koher) and angular
(by Fedorov) accesses.
Methods of operative treatment. Most rational of operations in this pathology is
cholecystectomy from the neck (retrograde). In this process the cystic duct and artery
are ligated, and then gall-bladder is cut, suturing up its bed. It is applicable is pres-
ence of shallow calculus in gall-bladder.
Cholecystectomy from the bottom (antegrade) is applied in case of technical
difficulties during the selection of elements of neck of gallbladder. It consists in de-
leting of gall-bladder from the bottom to the neck with the next ligation of cystic
duct, to the artery and suturing of bed of gall-bladder.
Atipical cholecystectomy. In this operative treatment gall-bladder is exposed
along its longitudinal axis, its contents are then removed and with the control of the
finger, position of neck is determined, and then deleted. Operation is executed in case
of widespread infiltrate and when accretions surround not only the gall-bladder but
also neck, cystic and biliary ducts. The surgeon must remember thus, that the explora-
tions of cystic duct and artery in such terms can be dangerous.
Laparoscopic cholecystectomy. Complex of special apparatus is required for
laparoscopy: operating laparoscope with a video camera and coloured video monitor.
After creation of pneumoperitoneum, introduction to the abdominal cavity of laparo-
scope and through other separate punctures of abdominal wall instruments, manipula-
tors are entered. With their help under the visual control in accordance with the image
on video monitor, gall-bladder is deleted. And on the stump of cystic duct and on the
artery metallic clips are imposed.
Ideal cholecystolytotomy is operation that includes opening of gallbladder, re-
moval of stone and its suturing.
Cholecystostomy is an operation, which is based on creation of external fistula
of gall-bladder. During this operation the lower part of thegall-bladder is taken into
the wound so that it is isolated from abdominal cavity.
Laparoscopic cholecystostomy is imposition of external fistula on gallbladder
under the control of laparoscope. It is the only operation, executed under the control
of laparoscope.
Cholecystostomy is performed, mainly, as the first stage of operation in the
very sick patients for removing the infected contents from the gallbladder and ducts.
This operation is palliative and often in the future requires repeated operation for the
removal of biliary fistula - cholecystectomy.

It is necessary to remember that during cholecystectomy it is important to con-


duct the intraoperative revision of biliary tract, which must include the examination,
palpation and determination of di- ameter of hepatico-choledochus (norm is to 8
mm), and also instrumental meth- ods of examination of bile ducts.
Intraoperative cholangiography is contrasting of biliary ducts by iodine prep-
arations (bilignost, cardiotrust. verigraphine in the concentration 30-33%) through
the stump of cystic duct. Cholangiography enables to define the width of ducts, pres-
ence or absence of calculus in them, and also cone-shaped narrowing of terminal part
of choledochus characteristic of stenosis). Cholangiomanometry is a method, which
allows measuring the degree of biliary hypertension in ducts by the water manometer
of the Valdmann device. Normal pressure is within the limits of 80-120 mm of waters
item (0,78-1,17 kPa).
Debitomanometry is a method of determination of amount of liquid perfusion
through the Vater's papilla under permanent pressure for time unit (1 min). In patients
with normal of biliary ducts the size to the debit of fluid at pressure 150 mm water
col. (1, 47 kPa) ranges from 5 to 8 ml/ min. In similar terms, at their obliteration, this
debit diminishes, and at insufficiency of sphincteric device - is increased. However,
much cholangio- and debitomanometry are applied rarely in operations. These meth-
ods of examination are used for chronic cholecystitis mainly. Choledochoscopy is a
method of endoscopy examination of biliary ducts by choledochoscope during chole-
dochostomy.
External draining of biliary ducts can be done by such methods:
1) By Pickovskyy - polyethylene catheter which is entered through stump of cyst-
ic duct;
2) By Kehr - by T- shaped latex drainage;

3) By Vyshnevsky - drainage to the gate of liver;


4) By Holsted – drainage to the duodenum through of choledochostomy.
Indication to external draining:
1) After diagnostic choledochustomy;
2) After choledocholithotomy;
3) With accompanying cholangitis and pancreatitis.
Internal drainage of biliary ducts is performed by transduodenal sphinctero-
plasty or choledochoduodenostomy. In acute cholecystitis these operations are exe-
cuted under absolute indications.
The best method of renewal of normal outflow of bile in stenosis with fixity of
calculus of large duodenal papilla, transduodenal dissection is considered. This meth-
od of internal drainage allows correcting the cause of defective transfer in ducts.
For choledochoduodenoanastomosis widely used are: methods by Jurash,
Flerken, Finsterer (and Kirschner's methods. The principle difference between them
consists of correlations of direction of incision on choledochus and duodenum (longi-
tudinal, oblique and transverse). The width of anastomosis must be not less than 2,5-3
cm. It is thus needed to remember, that imposition of choledochoduodenoanastomosis
in the condition with presence of inflammatory process-in abdominal cavity, always
no needs to be supplemented with external drainage of choledochus by Pikovskyy
(double draining).
Basic literatures:
31. Oxford Textbook of Surgery (3-Volume Set) 2nd edition (January 15,
2000): by Peter J. Morris (Editor), William C. Wood (Editor) By Oxford Press
32. Sabiston Textbook of Surgery 17th edition by Courtney M. Townsend
Jr., Kenneth L. Mattox, B. Mark, MD Evers, Kenneth L., MD Mattox, Courtney
Townsend, Daniel Beauchamp, B. Mark Evers, Kenneth Mattox W.B. Saunders
Company (June, 2004)
33. Schwartz´s Principles of Surgery 8th Edition F.Charles Brunicardi. Copy-
right ©2007 the McGraw-Hill Companies.
34. Hospital surgery/ Edited by L.Kovalchuk et al. - Ternopil: Ukrmedkny-
ha,2004.- 472 p.
Additional Literatures

1. Ahrendt SA: Biliary tract surgery. Curr Gastroenterol Rep 1:107, 1999.
[PMID: 10980936]
2. Klein AS, Lillemoe KD, Yeo CJ, et al: Liver, biliary tract, and pancreas,
in O'Leary JP (ed): Physiologic Basis of Surgery. Baltimore: Williams & Wilkins,
1996, p 441.
3. Scott-Conner CEH, Dawson DL: Operative Anatomy. Philadelphia: JB
Lippincott Company, 1993, p 388.
4. Washington M, Ghazi A: Complications of ERCP, in Scott-Conner
CEH (ed): The SAGES Manual. New York: Springer-Verlag, 1999, p 516.
5. Halpin VJ, Dunnegan D, Soper NJ: Laparoscopic intracorporeal ultra-
sound versus fluoroscopic intraoperative cholangiography: After the learning curve.
Surg Endosc 16:336, 2002. [PMID: 11967692]

Tests for initial level of knowledge, keys for tests:


1.The female patient complains about a pain in an epigastric area, in right sub-
costal with irradiation in a right shoulder-blade. There was vomit which did not bring
a relief. Temperature of body is 37,6 º С. A stomach is moderately bloated, there is
tense and pain in epigastric and right subcostal. Dense, sickly formation papates here.
Moderate tension of muscles of abdominal wall is marked in right subcostal.
Sypmtom Ortnera is positive. Formulate a preliminary diagnosis:
A. Acute cholecystitis.
B. Acute appendicitis.
C. Acute pancreatitis.
D. Acute iliac passion.
E. Perforated stomach ulcer.

2. Patient male, 50 years old, in an urgent order was hospitalized in a surgical clinic,
inspected, diagnosis was set: cholelithiasis, acute cholecystitis. After conservative
treatment pains diminished considerably, but an icterus appeared. What is most prob-
able reason of icterus at sick?
A. Migration of concretion from gallbladder to choledoch.
B. Benign hyperbilirubinemia.
C. Acute viral hepatitis А.
D. Viral hepatitis С.
E. Liver echinococcus.

3. The patient complains about icteric of skin, skler, white excrement, itch of skin.
She has been ill for three weeks. Does not mark pain. A stomach is painless. Biliru-
binemiya of 155 mcmole/l, direct – 105 mcmole/l, ASAT 2,3, ALAT of 3,1
mcmole/l. At sonographia a gall-bladder with a wall 0,4 sm, enlarged. Hepaticohole-
doh 1,8 sm in a diameter. A main pancreatic ductus is 0,5 sm in a diameter, coiled.
What is the most probable prior diagnosis?
A. Cancer of a head of pancreas, obturational jaundice.
B. Acute cholecystitis, obturational jaundice.
C. Choledocholidias, obturational jaundice.
D. Viral hepatitis.
E. Papillitis, obturational jaundice.

4. Patient B. 45 years, after eating “fat” food was disturbed by acute pain in a right
intercostal area with an irradiation in a right shoulder-blade, by womit, dryness and
bitter taste in a mouth. Рs-92 per 1 min., rhythmic. AT-135/85 mm Hg. A tongue is
dryish, assessed white and yellow stratification. A stomach “breathes”, is
moderately tensed and acute sickly in right subcostal. The symptoms of irritation of
peritoneum are absent. Positive symptom of Grekov-Ortner. What is the most
probable diagnosis?
A. Acute cholecystitis.
B. Acute appendicitis.
C. Acute pancreatitis.
D. Hepatitis.
E. Perforated ulcer.

5. What are the anatomical components of the triangle Calot, which in biliary tract
surgery called "key" for operations in the biliary system?
A. Common bile duct, cystic. Duct, v. portae.
B. Cystic duct, cystic artery, common bile duct.
C. V. portae, cystic.duct, common hepatic artery.
D. Cystic artery, common hepatic artery, v. portae.
E. Common hepatic artery, common bile duct, v. portae.
6. Symptom Mussy - Georgyevsky - is:
A. Cutting painful percussion on the right subcostal arch.
B. Painful palpation between two bases of right
Sternocleidomastoideus muscle.
C. Painful palpation projection gallbladder.
D. Painful pressing near the navel.
E. Painful palpation on the left subcostal arch.

7. Bilirubin normal blood parameters:


A. 30, 0-36.2 micmol \ l
B. 25, 8-32, 6 micmol \ l
C. 22.5 - 28.0 micmol \ l
D. 40, 0-50, 0 micmol \ l
E. 8, 5-20 micmol \ l

8. Surgical intervention in acute calculous cholecystitis shown:


A. All the patients in urgent procedure.
B. In the absence of the effect of conservative therapy within 48-72
hours.
C. At onset of peritonitis.
D. With increasing temperature to 37.5 c
E. Wait a mechanical jaundice.
9. Name the operation performed by impressions life very difficult patients in acute
cholecystitis.
A. Choletsystostomy .
B. Cholecystectomy.
C. Choledochoduodenoanastomos.
D. Laparoscopic cholecyctectomy.
E. Laparoscopic cholecystostomy.

10. A 25-year old patient after receiving oily food 3 days ago appeared to severe pain
in the right subcostal arch which decreased after taking “baralgin”. Body temperature
is 37, 7-38, 0º C. Abdomen soft. In the right costal arch at palpation painful infiltrate.
Leukocytosis 14.5 · 109/l. About the disease which primarily should think?
A. Acute appendicitis.
B. Acute pancreatitis.
C. Perfored ulcer of stomach.
D. Acute cholecystitis.
E. Acute pyelonephritis.

Keys for tests


1 2 3 4 5 6 7 8 9 10
A A B A B B E B A D

Tests for final level of knowledge, keys for tests:

1. A 66-year old parient complaining of severe acute pain in the right half of the
stomach, which irradiations in the right supraclavicular area, increase in body tem-
perature, dryness and bitterness in the mouth. There have been repeated vomiting,
which is not brought relief. The emergence of pain linked with consumption of fatty
and fried foods. The patient lies on the right side. Pale. Tachycardia. Tongue dry. Ab-
domen painful palpation in the right half of the abdomen and somewhat strained in
the right subcostal area. Probable diagnosis?
A. Acute cholecystitis.
B. Perforated ulcer.
C. Acute appendicitis.
D. Right side renal colic.
E. Acute intestinal obstruction.

2. A 46-year old patient, female, sick for two weeks when he noticed yellow sclera,
noted slight pain in the right subcostal area. After two days of onset, appeared gray
stools, sallow skin which gradually appreciated. When the department in general con-
dition is satisfactory. When ultrasonography revealed enlargement intrahepatic and
extrahepatic bile ducts of the main pancreatic duct. What is the most likely diagnosis?
A. Major duodenal papilla tumor.
B. Choledocholithiasis,
C. Norm cholestatic hepatitis.
D. Cirrhosis.
E. Total liver tumor

3. A 54-year patient, female, operated on of chronic calculous cholecystitis, Chole-


docholithiasis. Three years after surgery in patients with recurrence choledocholithia-
sis it. Which of the listed causes of non-core, leading to relapse in extrahepatic bile
duct calculus after surgery cholecystectomy?
A. Long-cystic duct stump
B. Availability of bile outflow
C. Narrowing of the terminal and associated cholangitis choledoch
D. Failure to comply with the recommendations of diet
E. It seams that threads of formation choledoch still not resolved
4. A 52-year old patient, female, operated of on acute cholecystitis. Operations are
performed without complications, gallbladder removed subserous from neck to bot-
tom with a ligation of cystic artery and cystic duct. A gallbladder bed is peritonised.
Choledochal is not extended. How do you stitch up the abdominal cavity?
A. Abdominal cavity tightly sutured without drainage of abdominal
B. Wear-provizorni stitches on a wound.
C. Abdominal cavity sutured tightly with drainage of abdominal cavity
through the bottom corner of wounds.
D. Ask gauze pads and drainage to the cystic duct stump, operating tight-
ly wound sutured.
E. Platforms tightly wound sutured with drainage under liver space

5. A 56-year old patient, after the seizure of pain in the right subcostal area noted
yellowness of skin and mucous membranes. When ultrasound calculus found in the
gallbladder, choledochal extended to 1.5 cm. The choledochal content is not clearly
visualizes. Which test will confirm choledocholithiasis?
A. Fibrogastroduodenoscopy.
B. Re-examination after ultrasound training.
C. Endoscopic retrograde cholangiography.
D. Computer tomography.
E. Blood tests for bilirubin, aminotransferase.

6. A 46-year old patient came to the surgery department complaining of pain in the
right subcostal area, that radiate to the right shoulder, nausea, vomiting, fever 37.80
C. His belly is swollen, stiff determined by palpation, tenderness in the right subcos-
tal, determined in the same dense, painful movements. Positive Symptoms by Ortner,
Mussee-Georgievsky are positive. In the blood - leukocytes are 12.6·109/l. Formulate
a preliminary diagnosis?
A. Acute pancreatitis
B. Acute cholecystitis
C. Perforated ulcer
D. Acute intestinal obstruction
C. Right side renal colic

7. Female, 58-year old patient complaining of periodic nagging pain in the right sub-
costal area, bitter taste in the mouth, increased pain after eating fatty, spicy food. Ul-
trasound results: gallbladder wall thicken in formation calculus, choledochal is 0.8
cm. What type of surgery the best in this case?

A. Laparoscopic cholecystectomy.
B. Open cholecystectomy.
C.Endoscopic-papilosphincterotomy.
D.Cholecystectomy, Choledochoduodenoanastomosis.
E. Cholecystectomy, external drainage of choledochal

8. A 41-years old patient with acute destructive cholecystitis. The presence of puru-
lent cholangitis. While sonography found no choledochal calculus and duodenal ste-
nosis symptom. Cholecystectomy has done. How to end the operation?
A. External drainage of choledochal.
B. Supraduodenal Choledochoduodenoanastomosis.
C. Transduodenal choledochoduodenostomy.
D. Choledochoduodenostomy.
E. Drainage of the abdominal cavity.

9. Female 53-years old was operated cholecystectomy for acute calculous cholecysti-
tis three years ago. After operation four times already had attacks of pain in the right
subcostal area, which lasted 6-8 hours and passed after antispasmodic. After the sec-
ond attack marked yellowing of the skin and scleras. What research will be most in-
formative for putting the diagnosis?
A. Ultrasonography.
B. Stomach X-ray examination.
C. Fibrocolonoscopy.
D. Oral-cholangiography.
E. Thermography.

10. The 43 years old patient operated with the probable diagnosis choledocholithiasis,
obstructive jaundice. In the operation of biliary tract disease and gall bladder were not
found. It increased the liver; it has a reddish-brown color. Diagnosed hepatitis. What
therapeutic activities appropriate?
A. Stitches the wound.
B. Cholecystectomy, drainage abdomen cavity.
C. Drainage of the free space.
D. Cholecystostomy.
E. External drainage of bile-duct
Keys for tests

1 2 3 4 5 6 7 8 9 10
A A D E C B A A A A

Tasks for the final level of knowledge.

1. Patient, 72 years old, plump woman during 20 years suffers from


cholelithiasis and cholecystitis and also by heavy form of diabetes; acute pain
appeared in right subcostal and irradiate in a right shoulder and shoulder-blade,
vomit, temperature is 38’С. Pain has been stable for already three days. A tongue is
dry, assessed with a white raid. Stomach is a little subinflated.Acute pain in the right
subcostal and considerable muscular tension. Other parts of stomach are painless. The
symptom of Schotkin-Blyumberg is located in right subcostal. There are leucocytes in
blood -12 Kg/l, sugar of blood of 8,5 g/d. What is the tactic of treatment?

The answer is surgical intervention

2. The patient of 48 years old. During an operation concerning acute


cholecystitis it was discovered largenesses tense phlegmonic changed gall-bladder.It
was removed. General bilious channel is extended to 3 sm in a diameter. At
choledohotomy 2 stones(1х1 sm) and plenty of dark mass with the presence of great
number of small stone and sand were remoted. How must a surgeon finish the
operation?

The answer is application of choledochoduodenostomia

3. An 88-year-old man with a history of end-stage renal failure, severe coro-


nary artery disease, and brain metastases from lung cancer presents with acute chole-
cystitis. His family wants “everything done.” The best management option in this pa-
tient would be

The answer is tube cholecystostomy

4. A 43-year-old obese mother of six children has severe right upper quadrant
abdominal pain that began 6 hours ago. The pain was colicky at first, radiated to the
right shoulder and around toward the back, and was accompanied by nausea and
vomiting. For the past 2 hours the pain has been constant. She
has tenderness to deep palpation, muscle guarding, and rebound in the right
upper quadrant. Her temperature is 101 °F, and she has a WBC count of 16,000.
She has had similar episodes of pain in the past brought about by ingestion of
fatty food, but they all had been of brief duration and relented spontaneously
or with anticholinergic medications.

The answer is acute cholecystitis

5. A 43-year-old obese mother of six children has severe right upper quad-
rant abdominal pain that began 6 hours ago. The pain was colicky at first, radiat-
ed to the right shoulder and around toward the back, and was accompanied by
nausea and vomiting. For the past 2 hours the pain has been constant. She has ten-
derness to deep palpation, muscle guarding, and rebound in the right upper
quadrant. Her temperature is 38,6 °C, and she has a WBC count of 12,000. Liver
function tests are normal.

Answer: acute cholecystitis

Materials for self-education

Main tasks Notes(instructions)

To repeat:
4. Anatomy of a gall bladder and hepatoduodenal - Represent methods of acute cholecystitis
ligament. diagnostics using scheme.
5. Pathophysiology of liver and pathophysiology -Make up a block diagram of symptoms of
extrahepatic biliary tract. Pathogenesis of acute acute cholecystitis.
inflammation of gall-bladder and complica-
tions.
6.
To study:
5. Age-specific features of acute cholecystitis -To do a differential diagnostic of ab-
course. dominal painful syndrome.
6. Tactic at complications of acute cholecystitis. -To do a differential diagnostic of acute
stomach.
Study guide #7.2
“Mechanical jaundice and its classification, pathogenesis, differential diagnosis,
tactics and treatment. Acute hepatic insufficiency in case of surgical diseases,
methods of treatment and prophylaxis.”

Overview.
Jaundice is a clinical symptom of acute infectious and chronic diseases of liver,
bile duct obturation sign and pathology of erythrocytes. Jaundice with any etiology
accompanied by pronounced intoxication, dysfunction of liver, kidneys and other
systems, is a real threat to the life of the patient. The main cause of obstructive
jaundice is choledocholithiasis (70%). In a number of cases of jaundice after the
elimination of the changes occurring in the liver parenchyma, leading to the chronic
organ disease (cirrhosis). Obstructive jaundice requires surgical treatment.
Calculous biliary disease - disease that is characterized by the formation of
stones in the biliary system. In Europe, the prevalence of cholelithiasis is 10-12%.
Postcholecystectomical syndrome - covers various types of diseases that differ
as a cause and clinical manifestations. In development postcholecystectomical
syndrome leading role is forgotten during cholecystectomy (35-40%) in common
bile calculus duct.
Educational aims:

37. To know anatomy and physiology liver and biliary tract.


38. To collect the anamnesis data and evaluate clinical inspection of mechanical
jaundice, calculous biliary disease and post-cholecystectomy syndrome (PCES).
39. To know the etiology and pathogenesis of mechanical jaundice, calculous bili-
ary disease and PCES.
40. To know clinical pictures of mechanical jaundice, calculous biliary disease and
PCES.
41. To determine mobility complications of mechanical jaundice, calculous biliary
disease and PCES.
42. To make the plan for inspection of mechanical jaundice, calculous biliary dis-
ease and PCES.
43. To carry out the received data at laboratory analysis and instrumental methods
of inspection in mechanical jaundice, calculous biliary disease and PCES.
44. To define indications for surgical treatment of mechanical jaundice, calculous
biliary disease and PCES or to appoint conservative treatment.
45. To treat mechanical jaundice, calculous biliary disease and PCES.
46. To estimate work capacity in of mechanical jaundice, calculous biliary disease
and PCES.
A student must know:
1. Anatomo-physiological features of liver and extrahepatic bile duct, most frequent
anomalies of development, gallstone formation.
2. Etiology and pathogenesis of mechanical jaundice, calculous biliary disease and
PCES.
3. Methods of inspection of the functional state of liver and extrahepatic bile duct.
4. Clinical examination and tests.
5. Ultrasonography and Endoscopic Retrograde Cholangiography and Endoscopic Ul-
trasound.
6- Clinical symptomatic of mechanical jaundice, calculous biliary disease and PCES.
7. Differential diagnostic of mechanical jaundice, calculous biliary disease and PCES.
8. Contra-indication for surgical and conservative methods of treatment.
9. Methods of surgical treatment in different forms of mechanical jaundice, calculous
biliary disease and PCES.
A student should be able to:
1. To collect anamnesis data and complaints of patient with of mechanical jaundice,
calculous biliary disease and PCES, to underline importance of them.
2. To examine a patient to define the volume of the special, instrumental, functional
and biochemical researches.
3. To estimate the received information.
4. To define tactic and volume of further treatment.
5. To carry out differential diagnostics of mechanical jaundice, calculous biliary dis-
ease and PCES.
6. To make the plan of inspection of mechanical jaundice, calculous biliary disease
and PCES.
7. To establish indications for surgical treatment and conservative treatment.
8. To prepare patients for the operation, to assist during the time of operation, to give
indications for a patient in post-operation period.
9. To predict possible complications during the treatment of patients with of mechan-
ical jaundice, calculous biliary disease and PCES, to give suggestions in relation to
prevent these complications.
Terminology

Jaundice is a pathologic process which characterized by yellow


discolouration of skin owing to excess bile acids in the
blood
Obstructive jaundice Is the type of jaundice which is seen as a result of viola-
tion of patency of biliary tract due to obliteration by inter-
nal or external compression, or cicatrical narrowing.

Calculous biliary dis- is the formation of stones into gallbladder, hepatic and ex-
ease trahepatic bile duct in consequence of violation of func-
tion of gallbladder and metabolism of bile acids
Choledocholithasis is the complication of gallstone disease and migrated
stones through the cystic duct into the common bile duct
Postcholecystectomy is abdominal pain or other symptoms originally attributed
syndrome to the gallbladder, may persist or recur months or years
following cholecystectomy

Content

ANATOMY
Extrahepatic Biliary Tract
The extrahepatic biliary tract consists of the bifurcation of the left and right hepat-
ic ducts, the common hepatic duct and common bile duct, and the cystic duct and
gallbladder. The left hepatic duct is formed by the ducts draining segments II, III, and
IV of the liver, courses horizontally along the base of segment IV, and has an extra-
hepatic length of 2 cm or more. The right hepatic duct is formed by the right posterior
(segments VI and VII) and right anterior (segments V and VIII) hepatic ducts and has
a short extrahepatic length. The hepatic duct bifurcation is usually extrahepatic and
anterior to the portal vein bifurcation. The common hepatic duct lies anteriorly in the
hepatoduodenal ligament and joins the cystic duct to form the common bile duct. The
common bile duct extends from the cystic duct, into the common hepatic duct junc-
tion inferiorly, to the papilla of Vater, where it empties into the duodenum. The
common bile duct varies in length from 5 to 9 cm depending on its junction with the
cystic duct and is divided into three segments: supraduodenal, retroduodenal, and in-
trapancreatic. The distal common bile duct and pancreatic duct may join outside the
duodenal wall to form a long common channel, within the duodenal wall to form a
short common channel, or they may enter the duodenum through two distinct ostia.
The gallbladder is a pear-shaped reservoir in continuity with the common hepatic
and common bile ducts via the cystic duct. The gallbladder lies on the inferior surface
of the liver partially enveloped in a layer of peritoneum. The gallbladder is anatomi-
cally divided into the fundus, body, infundibulum, and neck, which empties into the
cystic duct. Both the gallbladder neck and the cystic duct contain spirally oriented
mucosal folds known as the valves of Heister. The cystic duct varies in length from
1to 4 cm usually joining the common hepatic duct at an acute angle.

BILIARY PHYSIOLOGY

The bile ducts, gallbladder, and sphincter of Oddi act together to modify, store,
and regulate the flow of bile. During its passage through the bile ductules and hepatic
duct, canalicular bile is modified by the absorption and secretion of electrolytes and
water. The gastrointestinal hormone (secretin) increases bile flow primarily by in-
creasing the active secretion of chloride-rich fluid by the bile ducts and ductules. Bile
duct secretion is also stimulated by other hormones such as cholecystokinin (CCK)
and gastrin. The bile duct epithelium is also capable of water and electrolyte absorp-
tion, which may be of primary importance in the storage of bile during fasting in pa-
tients who have previously undergone cholecystectomy.
Gallbladder
The main functions of the gallbladder are to concentrate and store hepatic bile
during the fasting state and deliver bile into the duodenum in response to a meal. The
usual capacity of the human gallbladder is only about 40 to 50 mL. Only a small frac-
tion of the 600 mL of bile produced each day would be stored was it not for its re-
markable absorptive capacity. The gallbladder mucosa has the greatest absorptive ca-
pacity per unit area of any structure in the body. Bile is usually concentrated 5-fold to
10-fold by the absorption of water and electrolytes leading to a marked change in bile
composition.
Active NaCl transport by the gallbladder epithelium is the driving force for the
concentration of bile. Water is passively absorbed in response to the osmotic force
generated by solute absorption. The concentration of bile may affect the solubility of
two important components of gallstones: calcium and cholesterol. Although the
gallbladder mucosa does absorb calcium, this process is not nearly as efficient as for
sodium or water, leading to greater relative increase in calcium concentration. As the
gallbladder bile becomes concentrated, several changes occur in the capacity of bile
to solubilize cholesterol. The solubility in the micellar fraction is increased, but the
stability of phospholipid-cholesterol vesicles is greatly decreased. Because cholester-
ol crystal precipitation occurs preferentially by vesicular rather than micellar mecha-
nisms, the net effect of concentrating bile is an increased tendency to nucleate choles-
terol (see Gallstone Pathogenesis).
The gallbladder epithelial cell secretes at least two important products into the
gallbladder lumen: glycoproteins and hydrogen ions. Secretion of mucus glycopro-
teins occurs primarily from the glands of the gallbladder neck and cystic duct. The
resultant mucin gel is believed to constitute an important part of the unstirred layer
(diffusion-resistant barrier) that separates the gallbladder cell membrane from the lu-
minal bile. This mucus barrier may be very important in protecting the gallbladder
epithelium from the strong detergent effect of the highly concentrated bile salts found
in the gallbladder. However, considerable evidence also suggests that mucin glyco-
proteins play a role as a pronucleating agent for cholesterol crystallization. The
transport of hydrogen ions by the gallbladder epithelium leads to a decrease in
gallbladder bile pH through a sodium-exchange mechanism. Acidification of bile
promotes calcium solubility, thereby preventing its precipitation as calcium salts. The
gallbladder’s normal acidification process lowers the pH of entering hepatic bile from
7.5 to 7.8 down to 7.1 to 7.3.

Biliary Motility
Gallbladder
Gallbladder filling is facilitated by tonic contraction of the ampullary sphincter,
which maintains a constant pressure in the common bile duct (10 to 15 mm Hg). The
gallbladder does not, however, simply fill passively and continuously during fasting.
Rather, periods of filling are punctuated by brief periods of partial emptying (10% to
15% of its volume) of concentrated gallbladder bile that are coordinated with each
passage through the duodenum of phase III of the migrating myoelectric complex
(MMC). This process is mediated, at least in part, by the hormone motilin. Following
a meal, the release of stored bile from the gallbladder requires a coordinated motor
response of gallbladder contraction and sphincter of Oddi relaxation. One of the main
stimuli to gallbladder emptying is the hormone CCK, which is released from the duo-
denal mucosa in response to a meal. When stimulated by eating, the gallbladder emp-
ties 50% to 70% of its contents within 30 to 40 minutes. Gallbladder refilling then
occurs gradually over the next 60 to 90 minutes. Many other hormonal and neural
pathways are also necessary for the coordinated action of the gallbladder and sphinc-
ter of Oddi. Defects in gallbladder motility, which increase the time of bile staying in
the gallbladder, play a central role in the pathogenesis of gallstones.

Sphincter of Oddi
The human sphincter of Oddi is a complex structure that is functionally independ-
ent from the duodenal musculature. Endoscopic manometric studies have demonstrat-
ed that the human sphincter of Oddi creates a high-pressure zone between the bile
duct and the duodenum. The sphincter regulates the flow of bile and pancreatic juice
into the duodenum, prevents the regurgitation of duodenal contents into the biliary
tract, and also diverts bile into the gallbladder. This latter function is achieved by
keeping pressure within the bile and pancreatic ducts higher than duodenal pressure.
The sphincter of Oddi also has very high-pressure phasic contractions. The exact
functions of these phasic waves in humans are unknown, but they may play a role in
preventing the regurgitation of duodenal contents into the biliary tract.
Both neural and hormonal factors influence the sphincter of Oddi. In humans,
sphincter of Oddi pressure and phasic wave activity diminish in response to CCK.
Thus, sphincter pressure relaxes after a meal, allowing the passive flow of bile into
the duodenum. During fasting, high-pressure phasic contractions of the sphincter of
Oddi persist through all phases of the MMC. Recent animal studies suggest, however,
that sphincter of Oddi phasic waves do vary to some degree in relation with the
MMC.
Thus, sphincter of Oddi activity is undoubtedly coordinated with the partial
gallbladder emptying and increases in bile flow that occur during phase III of the
MMC. This activity may be a preventative mechanism against the accumulation of
biliary crystals during fasting. Neurally mediated reflexes link the sphincter of Oddi
with the gallbladder and stomach to coordinate the flow of bile and pancreatic juice
into the duodenum. The cholecystosphincter of Oddi reflex allows the human sphinc-
ter to relax as the gallbladder contracts. Similarly, antral distention causes both
gallbladder contraction and sphincter relaxation.

CALCULOUS BILIARY DISEASE

Gallstone Pathogenesis
Bile facilitates the intestinal absorption of lipids and fat-soluble vitamins and rep-
resents the route of excretion for certain organic solids, such as bilirubin and choles-
terol. The major organic solutes in bile are bilirubin, bile salts, phospholipids, and
cholesterol. Bilirubin is the breakdown product of spent red blood cells and is conju-
gated with glucuronic acid prior to being excreted. Bile salts solubilize lipids and fa-
cilitate their absorption. Phospholipids are synthesized in the liver in conjunction with
bile salt synthesis. The final major solute of bile is cholesterol, which is also pro-
duced primarily by the liver with little contribution from dietary sources. Cholesterol
is highly non-polar and insoluble in water and, thus, in bile. The normal volume of
bile secreted daily by the liver is 500 to 1000 mL.
Gallstones represent a failure to maintain certain biliary solutes, primarily choles-
terol and calcium salts, in a solubilized state. Gallstones are classified by their choles-
terol content as either cholesterol or pigment stones. Pigment stones are further classi-
fied as either black or brown. Pure cholesterol gallstones are uncommon (10%), with
most cholesterol stones containing calcium salts in their center, or nidus. In most
American populations, 70% to 80% of gallstones are cholesterol, and black pigment
stones account for most of the remaining 20% to 30%.
An important biliary precipitate in gallstone pathogenesis is biliary “sludge,”
which refers to a mixture of cholesterol crystals, calcium bilirubinate granules, and a
mucin gel matrix. Biliary sludge has been observed clinically in prolonged fasting
states or with the use of long-term total parenteral nutrition. Both of these conditions
are also associated with gallstone formation. The finding of macromolecular com-
plexes of mucin and bilirubin, similar to biliary sludge in the central core of most
cholesterol gallstones, suggests that sludge may serve as the nidus for gallstone
growth.

Cholesterol Gallstones
The pathogenesis of cholesterol gallstones is clearly multifactorial but essentially
involves three stages: cholesterol supersaturation in bile, crystal nucleation, and stone
growth. For many years, gallstones were thought to result primarily from a defect in
the hepatic secretion of biliary lipids. More recently, it has become increasingly clear
that gallbladder mucosal and motor function also play key roles in gallstone for-
mation. The key to maintaining cholesterol in solution is the formation of micelles, a
bile salt-phospholipid-cholesterol complex, and cholesterol-phospholipid vesicles.
Present theory suggests that in states of excess cholesterol production, these large
vesicles may also exceed their capability to transport cholesterol, and crystal precipi-
tation may occur. Cholesterol solubility depends on the relative concentration of cho-
lesterol, bile salts, and phospholipid. By plotting the percentages of each component
on triangular coordinates, the micellar zone in which cholesterol is completely solu-
ble can be demonstrated. In the area above the curve, bile is supersaturated with cho-
lesterol, and precipitation of cholesterol crystals can occur.
Cholesterol super saturation is present in many normal humans without gallstones,
and a significant overlap exists in cholesterol saturation in patients with and without
gallstones. Thus, cholesterol super saturation results in a metastable state in which
cholesterol precipitation may or may not take place and additional factors in bile must
be present, therefore, to either enhance or inhibit the nucleation of cholesterol leading
to the next stage in gallstone formation.
Nucleation refers to the process in which solid cholesterol monohydrate crystals
form and conglomerate. Nucleation occurs more rapidly in gallbladder bile of patients
with cholesterol stones than in individuals with cholesterol-saturated bile without
stones. As bile is concentrated in the gallbladder, a net transfer of phospholipids and
cholesterol from vesicles to micelles occurs. The phospholipids are transferred more
efficiently than cholesterol, leading to cholesterol enrichment of the remaining vesi-
cles. These cholesterol-rich vesicles aggregate to form large multi-lamellar liquid ves-
icles that then precipitate cholesterol monohydrate crystals. Several pronucleating
factors including mucin glycoproteins, immunoglobulins, and transferrin accelerate
the precipitation of cholesterol in bile.
For gallstones to cause clinical symptoms, they must obtain a size sufficient to
produce mechanical injury to the gallbladder or obstruction of the biliary tree. Growth
of stones may occur in two ways: progressive enlargement of individual crystals or
stones by deposition of additional insoluble precipitate at the bile-stone interface or
fusion of individual crystals or stones to form a larger conglomerate. In addition, de-
fects in gallbladder motility increase the time of bile stay in the gallbladder, thereby
playing a role in stone formation. Gallstone formation occurs in clinical states with
gallbladder stasis, as seen with prolonged fasting, the use of long-term parenteral nu-
trition, after vagotomy, and in patients with somatostatin-producing tumors or in
those receiving long-term somatostatin therapies.
Pigment Gallstones
With the recognition that calcium salts are present in most, if not all, cholesterol
gallstones, renewed interest has developed in the events leading to the precipitation of
calcium with the anions, bilirubin, carbonate, phosphate, or palmitate. Precipitation of
these anions as insoluble calcium salts serves as a nidus for cholesterol stone for-
mation. Furthermore, calcium bilirubinate and calcium palmitate also form major
components of pigment gallstones.
Pigment gallstones are classified as either black or brown pigment stones. Black
pigment stones are typically tarry and are associated frequently with hemolytic condi-
tions or cirrhosis. In hemolytic states, the bilirubin load and concentration of uncon-
jugated bilirubin increases. These stones are usually not associated with infected bile
and are located almost exclusively in the gallbladder. In contrast, brown pigment
stones are earthy in texture and are typically found in the bile ducts, especially in
Asian populations. Brown stones often contain more cholesterol and calcium palmi-
tate and occur as primary common duct stones in Western patients with disorders of
biliary motility and associated bacterial infection. In these settings, bacteria produc-
ing slime and those containing the enzyme-glucuronidase cause enzymatic hydrolysis
of soluble conjugated bilirubin glucuronide to form free bilirubin, which then precipi-
tates with calcium.

CHRONIC CALCULOUS CHOLECYSTITIS


Inflammation of gall-bladder, that gained prolonged chronic character, is consid-
ered chronic cholecystitis.
Etiology and pathogenesis
Everything mentioned above about etiology and pathogenesis of acute cholecys-
titis to a certain extent is responsible for chronic cholecystitis, but with the proper
amendments, related to expression and duration of inflammatory process, degree of
violation of passage of bile and individual features of patient. Chronic cholecystitis is
calculous. Its non-calculous forms are met infrequently. Salmonella is considered to
be a leading cause of special clinical form of non-calculous chronic cholecystitis.
Pathomorphology
Gall-bladder is sclerosed, deformed, with connecting-tissue accretions. Histolog-
ical atrophy of mucosa, sclerosis, often petrification, possible formations of adenomas
(pre-cancer) and obliteration of lumen are observed.
Classification
Chronic cholecystitis is divided into:
1) Chronic calculous.
2) Chronic non-calculous.
Division of chronic cholecystitis on primary one, recurrent and complicated has
practical value.
Primary cholecystitis is that, which arises without any previous acute attack, re-
current - when in history there are one or more attacks, complicated - if the chronic
cholecystitis is associated with:
1) violation of patency of biliary ducts;
2) septic cholangitis;
3) obliterating cholangitis;
4) hydropsy of gall-bladder;
5) pancreatitis;
6) hepatitis;
7) sclerosis of gall-bladder;
8) paravesicular chronic abscess;
9) Internal fistula.
Clinical management
Dull, aching pain is considered the main symptom of chronic cholecystitis in
right hypochondrium that radiates to right subclavian area, shoulder-blade or shoul-
der. Belt-like character of pain testifies the involvement of the pancreas. Violation of
diet causes, usually, increases of pain and can provoke the attack of acute cholecysti-
tis.
Dyspeptic syndrome. Feeling of fullness, feeling of swelling in right hypochon-
drium, distension of abdomen after the intake of meal, bitter taste in the mouth, belch-
ing, nausea, vomiting, which brings some relief, heartburn and violation of emptying,
are the frequent signs of this syndrome (delay, more frequent diarrhea). It is important
to pay special attention to these symptoms, because they can be the first signs of dis-
ease of chronic cholecystitis.
On examination skin and sclera there is often slight icterus. Prolonged history, in
a thin, exhausted patient, rather yellowish color of the person should lead to the sus-
picion of the possibility of cirrhosis of liver, and when there is increase of tempera-
ture of body - one should think about acute attack of the disease or presence of chol-
angitis. Sometimes in these patients it is possible to expose brown spots in right hy-
pochondrium -acquired by application of hot-water bottle. Activity of patients often
causes pain in the right hypochondrium.
During palpation of abdomen it is possible to expose insignificant rigidity of
muscles in right hypochondrium and tenderness in the projection of gall-bladder. On
palpation gall-bladder is often tender. Symptoms of cholecystitis in such patients, as a
rule, are poorly expressed. Murphy's and Mussy-Georgievsky's signs are almost al-
ways "positive". The signs of Orthner’s and Kehr’s signs become positive with ex-
pressed inflammatory process.
Sonography examination reveals the size of gall-bladder, thickness of its walls,
presence or absence of calculus and their sizes.
Roentgenoscopy survey of right hypochondrium in 10 % of patients exposes
roentgen opaque calculus of salts of calcium.
With contrast per oral or intravenous cholecystogram is confirms: the multiple
defects of filling of gall-bladder due to calculus. Special attention is to be paid to the
state of ducts. Dilation of general bile duct to the diameter larger than 10 mm speci-
fies the violation of passage of bile and requires correction during operation. Negative
cholecystogram (when a gall-bladder is filled) is suggests of blockade of cystic duct.
By duodenal intubation the inflammatory changes are exposed in bile from duo-
denum (portion A), from gall-bladder (portion B) and from hepatic passing (the С
portion). Absence of the bile "B" grounds suspects impossibility of cystic duct or vio-
lation of function of gallbladder.
Clinical variants and complications
Clinical features of chronic cholecystitis are characterized by the periods of acute
attacks and remission. After the second acute attacks with increase of pain, remission
comes in a few weeks that can sometimes last to a few months. Violation of diet
again causes worsening of the state - attack of acute cholecystitis. The atypical course
of disease is met with infrequently.
Most complications of chronic cholecystitis are the same as at acute cholecystitis:
hydropsy, cholangitis, pancreatitis, icterus, hepatitis, abscess and hepato-renal insuf-
ficiency. As a result of connection of gallbladder or ducts with the hollow organs
there can also be internal biliary fistula. Thus, with the formation of connections be-
tween gall-bladder and stomach there is vomiting of bile, sometimes there can be cal-
culus in the vomitus. The perforation of contents of gall-bladder into the small intes-
tine often causes enteritis.

Diagnosis program
1. History and physical examination.
2. Survey roentgenoscopy of organs of abdominal cavity.
3. Oral and intravenous cholangiography.
4. Sonography.
5. General analysis of blood and urine.
6. Analysis of urine for diastasis.
7. Biochemical blood test (bilirubin, amylase, hepatic tests).
8. Coagulogram.
9. Duodenal intubation.
10.Endoscopy.
Techniques and choice of treatment method
Conservative treatment is the basis for treating patients with non-calculous
chronic cholecystitis It includes:
- diet: table № 5 by Pevsner;
- choleic preparations (alohol, holagol, holenzyme, holosas, olimetyn);
- cholekinetics (sulfate of magnesium, cholecystokinin, pituitrin);
- spasmolytics (sulfate of atropine, platyphyllin, methacin, amino-phylline);
- duodenal intubation;
- Antibacterial preparations (during acute attacks).
For the relief of pain syndrome parenteral spasmolytic preparations are given:
no-shpa (2 ml 2 % solution), papaverine (2 ml 2 % solution), metoclopramide, reglan,
cerucal (for 2 ml), atropine (1 ml 0,1 % solution), methacin (1 ml 0,1 % solution).
With considerably expressed pain syndrome analgin (2 ml 50 % solution) or prome-
dol is simultaneously given (1 ml 2 % solution). Baralgine is often used (5 ml intra-
venously or intramuscular). On occasion talamonal is applied (2-4 ml intramuscular).
All these preparations are given 3-4 times per day, till pain does not disappear. With
the purpose of removal of inflammatory process antibiotics are given: erythromycin
(0, 25 g 6 times per days), doxycycline (0, 5-1, 0 g 2 times per days), ampicillin (0, 5
g 4-6 times per days), biseptol (2 tabs. 2 times per day after meal), furazolidone (0, 05
g 4 times per days). Course of treatment: 8-10 days. In severe intoxication there is the
intravenous infusion of neohemodes (200-400 ml) or polydes (250-450 ml). Calcu-
lous, as well as chronic recurrent cholecystitis is treated by operative means only.
All forms of chronic calculous cholecystitis and chronic non-calculous cholecys-
titis with proof of bacterial infection of gall-bladder and biliary ducts which does not
respond to therapeutic treatment are subject to surgical treatment.
Operative treatment must provide deletion of hearth of inflammatory process
(cholecystectomy), and in case of violation of passage of bile by the ducts it is re-
paired. It is necessary to remember, that earlier the operation is performed, the better
are the immediate and late results of surgery.
OBSTRUCTIVE JUANDICE
Obstructive jaundice is the type of jaundice which is seen as a result of violation
of patency of biliary tract due to obliteration by internal or external compression, or
cicatrical narrowing.
Etiology and pathogenesis
Obliteration of external biliary ducts can arise due to various causes: as a result
of inflammatory process of surrounding organs (pancreas, paracholedochus lymph
nodes), damage of walls of ducts during operations. However, in most cases it is a
gallstone disease, choledocholithiasis and related to them, scars of large papilla of
duodenum. The second cause of obstructive jaundice is due to tumors of the cancer of
head of pancreas and large papilla of duodenum.
Disregarding the fact that each of the adopted diseases has special clinical fea-
tures, obstruction of biliary ducts causes the changes which have general character.
With complete blockade of external biliary ducts and increase of pressure higher than
300 mm of waters or 2,94 kPa (after a norm - not higher 150 mm or 1,47 kPa), excre-
tions of bile into biliary capillaries is stopped. It is characterized by the fact that the
secretory mechanism of hepatic cells (hepatocyte) can not overcome such resistance.
Thus bile enters into the lymphatic vessels and vein of liver and from there enters
blood, causing the syndrome of mechanical jaundice.

Classification (by O.O. Shalimov, 1993)


Obstructive jaundices are divided into:
I. According to the level of barrier:
1) Obstruction of distal parts of common bile duct;
2) Obstruction of supraduodenal part of common bile duct;
3) Obstruction of initial part of general hepatic duct and bifurcation of hepatic ducts.
II. According to the etiologic factor:
1) Characterized obstruction by calculus, foreign bodies, blood during hemobilia,
parasites, iatrogenic influence during operation;
2) Obstruction due to diseases of the wall of biliary tract - innate anomalies (hypo-
plasia, cysts and atresia), inflammatory diseases (obstructing papillitis and cholan-
gitis), scar strictures (posttraumatic and inflammatory), biliary tract tumors;
3) Obstruction caused by diseases of other organs lying near the biliary tract, which
drags them into the process (tubular stenosis of common bile duct of pancreatic
origin, ulcer disease of duodenum, paracholedocheal lymphadenitis, adhesions).
Except that, according to the duration the disease is distinguished into:
1) Acute obstructive jaundice, which lasts for 10 days;
2) Prolonged, that lasts from 10 to 30 days;
3) Chronic, that lasts for more than a month.
Clinical management
The clinical picture of obstructive jaundice is mainly due to the violation of out-
flow of bile.
The pain syndrome is a characteristic accompaniment of gallstone disease and
choledocholithiasis which appears with the attacks of hepatic colic. However, pain
syndrome in these pathologies is not often expressed or is usually absent. Pain is of-
ten observed in stricture of biliary ducts, but it is not quite typical of patients with the
cancer of bile ducts.
Icterus is an important sign of obstruction of biliary tract, speed of origin and in-
tensity of which depend on the amount of passage of bile into the intestine.
The itching of body is a frequent accompaniment of icterus, which rises up as a
result of action of biliary acids. During examination the yellowness of sclera, mucosa
and skin are observed. At the same time patients complains of high colored urine and
discolorations of stools darkening ("argil"). The increase of temperature of body testi-
fies the development of cholangitis, metastasis of tumors in liver is rarer.
In the right hypochondrium in emaciated patients it is sometimes possible to see
a mass that moves during breathing; probably it is the gall-bladder. If it is elastic, is
not painful and it is accompanied by icterus (the Courvoisier's symptom), patient is
then having cancer of head of pancreas or distal parts of common bile ducts.
Laboratory analysis. For obstructive jaundice a cholestatic syndrome with high
bilirubinemia mainly due to direct faction of bilirubin and bilirubinuria is characteris-
tic, by absence of urobilin in urine and stercobilin in feces, by high activity of alka-
line phosphatase at the insignificantly promoted transaminase activity and negative
thymol test.
With increase of hyperbilirubinemia this intercommunication changes to the side
of increase of direct bilirubin. In general analysis of blood unsteady changes are
which depend on the degree of intoxication or occult bleeding (in cancer patients).
Sonography examination allows defining the sizes of liver, gallbladder, state of
internal and external hepatic ducts, presence and degree of dilatation or narrowing,
presence or absence of calculus and new formation in hepatic parenchyma.
Duodenography in the conditions of artificial low blood pressure apply for the
exposure of pathology of organs of pancreatoduodenal area.
Retrograde cholangiopancreatography enables to examine stomach, duodenum
by endoscope, to conduct biopsy, to extract bile and pancreatic juice for examination,
to get the roentgenologic image of ducts: external and internal hepatic ducts and duct
of pancreas and in a number of cases in the presence of calculus to conduct endoscop-
ic papillotomy and extraction calculus through papillotomic access.
Clinical variants and complications
Clinical features of obstructive jaundice almost always depend on the causes of
obstruction of biliary duct. In patients with tumours icterus gradually progresses and
is seen in complete and permanent obstruction, along with that, in presence of calcu-
lus in biliary ducts intensity of icterus can vary. Its temporal, transitional character
takes place in choledocholithiasis, acute cholecystitis or pancreatitis.
On this background, cholangitis, together with abscess formation in liver, sepsis
develops. In other case there can be the bleeding (more frequent gastroduodenal) or
hepato-renal insufficiency.
In some patients internal biliary fistula, which clinically is a proof of cholangitis,
appears as a result of inflammatory and necrotic processes. On the skiagram survey of
organs of abdominal cavity in such cases it is possible to see air in the hepatic ducts,
the so called "aerocholia".
Diagnosis programs
1. History and physical methods of examination.
2. General analysis of blood and urine.
3. Analysis of urine on diastasis.
4. Biochemical blood test (bilirubin, urea, albumin-globulin coefficient, blood on an
Australian antigen, amylase, alanine aminotransferase, asparaginase, alkaline
phosphatase).
5. Coagulogram.
6. Sonography.
7. Endoscopy.
8. Retrograde cholangiopancreatography.
9. Laparoscopy with biopsy.
10.Percutaneous transhepatic cholangioduodenography.
11.Computer tomography.
Techniques and choice of treatment method
Final diagnosis, that maximally represents the character of obstructive jaundice,
as a rule, is set only during the intraoperative revision of the organs. In determination
of medical techniques and choice of method of surgical treatment of such jaundice it
is needed also to objectively estimate severity and general condition of patients. For
this purpose it is necessary to take into account the character of icterus, stage of he-
patic insufficiency keeping in mind the duration and intensity of cholestasis, presence
and character of cholangitis, severity and expression of accompanying pathology.
Medical measures in preoperative period must be directed in correction of viola-
tions of homeostasis, hemocoagulation (aminocapronic acid, vicasol, 10 % solution of
chlorous calcium, fresh-frozen plasma, inhibitor of protease), improvement of micro-
circulation in liver (10 % solution of glucose with insulin, reopolyglucine, hepatopro-
tectors), desintoxication organism (neohemodes, enterosorbent), biliary decompres-
sion (percutaneous transhepatic cholangiostomy or cholecystostomy), antibacterial
therapy for cholangitis taking into account the character of microflora and its sensi-
tiveness to the antibiotics and vitamins.
In case of the gallstone disease complicated by choledocholithiasis and mechani-
cal icterus, the volume of surgical interference must include: cholecystectomy, chole-
docholithotomy and external or internal drainage of common bile duct. In presence of
the special apparatus in case of choledocholithiasis complicated by mechanical icter-
us, two-stage tactics of treatment is the method of choice - endoscopic papillosphinc-
terotomy with subsequent extraction of calculus and their lithotripsy on the first stage
and cholecystectomy - on the second. Endoscopic papillosphincterotomy is the meth-
od of choice for treatment of remaining (after cholecystectomy) choledocholithiasis.
For older patients with severe pathology combination of extracorporeal shock-
wave lithotripsy with endoscopic sanitation of hepaticocholedochus is an effective
method. For some of them with high risk of operative treatment and small calculus of
common bile duct (by the diameter of to 10 mm) endoscopic papillotomy can be ef-
fective.
In malignant formations of biliary tract with obstructive jaundice, depending on
distribution of tumor process, radical or palliative operative treatments are executed.
For patients with the tumor of head of pancreas, large papilla of duodenum and
terminal part of choledochus, that is exposed to radical operation, a pancreatoduode-
nal resection is performed, thus in the case of severe jaundice on the first stage of
treatment decompression of the hepatobiliary system is performed (percutaneous he-
paticocholangiostomy, forming of biliary-enteric anastomosis). A pancreatoduodenal
resection is executed on the second stage, in 30-35 days after imposition of bile-
excreting anastomosis and correction of icterus.
Palliative bile-excreting operations in neglected tumours are mainly directed on
correction of obstruction of biliary ducts. In such patients, as a rule, roundabout bili-
ary-enteric anastomosis is imposed: cholecystoentero-or hepaticojejunostomy. If
through technical difficulties and bad general condition of patients it is impossible to
execute this, external drainage of biliary tract is applied.
Acute hepatic insufficiency in case of surgical diseases
Introduction
Postoperative liver dysfunction is a common problem. Although the incidence
after elective abdominal surgery is less than 1 per cent, much higher rates occur after
major surgery, multiple trauma, and prolonged intervention. The majorities of cases is
mild, transient, and resolve spontaneously, but occasionally the liver injury may be
severe and result in fulminant liver failure and/or chronic liver disease. There are
many etiological factors, and in any one patient the pathogenesis is often multifacto-
rial.
The surgical patient with normal preoperative liver function
Introduction
Postoperative hepatic dysfunction in surgical patients with normal liver func-
tion can be classified into three groups: those due to (1) overproduction of bilirubin;
(2) hepatocellular dysfunction; and (3) extrahepatic biliary obstruction.
Overproduction of bilirubin
In a healthy individual the liver conjugates up to 500 μmol of bilirubin per day
as a result of the breakdown of red blood cells. The liver is capable of handling sever-
al times this quantity without the occurrence of hyperbilirubinaemia and only if hae-
molysis is severe or occurs in conjunction with hepatocellular insufficiency does
jaundice develops. Unconjugated bilirubin comprising 90 per cent of the total is sug-
gestive of haemolysis. When the level of unconjugated bilirubin is excessively high
there appears to be a concomitant rise in the conjugated fraction. The cause of signifi-
cant haemolysis may be haemolytic anaemia, blood transfusion, resorption of hema-
tomas, sepsis, or open-heart surgery.
Haemolytic anemia
Congenital and acquired haemolytic anemia can be associated with postopera-
tive jaundice. In surgical patients with sickle-cell disease there are increased risks as
acute haemolysis and severe pain can be precipitated by infection, dehydration, aci-
dosis, and hypoxia. In the postoperative period the earliest signs of infection must be
treated promptly, especially as these patients have splenic hypofunction and are sus-
ceptible to bacterial infection. Patients from the African continent, parts of Asia, the
Arabian Peninsula, and southern Europe should be screened for sickle-cell disease.
Patients with hereditary spherocytosis may also experience a haemolytic crisis fol-
lowing infection, and in the postoperative period this can cause an unconjugated hy-
perbilirubinaemia. The diagnosis is suggested by the family history or the presence of
a raised mean concentration of corpuscular haemoglobin, with more than 1 to 2 per
cent of spherocytes on the blood film.
Surgery, infection, acidosis, and many drugs, including antibiotics and analge-
sics may precipitate haemolysis in patients with glucose 6 –phosphate dehydrogenase
deficiency. At least 10 million people worldwide have this red cell enzyme deficien-
cy, and thus patients from the Mediterranean, South-East Asia, the
Middle East and West Africa should be screened preoperatively. The cresyl blue de-
coloration test or the methaemoglobin reduction test can be used in screening, and the
diagnosis made by enzyme assay. Pyruvate kinase deficiency is another red cell en-
zymopathy in which infection can precipitate haemolysis. Patients should be aware of
their diagnosis but macrocytosis and an abnormal enzyme assay will confirm the di-
agnosis. Causes of non-immune acquired haemolytic anaemia include disseminated
intravascular coagulation; vasculitis; pneumococcal, meningococcal, and Gram-
negative sepsis; Clostridium perfringens (was C. welchii) infection; burns; drowning;
and some drugs. These are covered in other parts of this section.
Blood transfusion
Immediate and delayed haemolytic reactions may occur following blood trans-
fusion. Within 24 h of the transfusion of one unit of stored blood at least 10 per cent
undergoes haemolysis. Transfusion of two units of blood should not result in an in-
crease in the serum bilirubin. If transfusion is rapid, massive, or occurs in a patient
with impaired liver function, the capacity of the liver to conjugate bilirubin may be
exceeded. Jaundice in this situation occurs 10 to 12 h after transfusion. Incompatibil-
ity of transfused blood may result in a severe immediate haemolytic reaction, which
may occur if there are antibodies to the donated blood in the recipient's plasma. Jaun-
dice appears at 12 h after commencing transfusion, peaks at between 24 and 36 h, and
lasts for a total of 4 or 5 days. Delayed haemolytic transfusion reactions are seen be-
tween 3 days and 3 weeks post-transfusion, with the peak reaction being at around 7
to 10 days. They are due to a secondary immune response, and in the majority of cas-
es there has been sensitization to red cell antigens through past transfusion or preg-
nancy. This response is often to Rhesus and Kidd antigens, and is seen clinically as
extravascular haemolysis with fever, jaundice, and anaemia. A serum sample should
be screened for antibodies and future transfusions proceeded by careful compatibility
testing.
Resorption of hematomas
Large hematomas, crush injury, and bleeding from major vessels result in large
pools of extravascular blood which, when reabsorbed, can result in an unconjugated
hyperbilirubinaemia. As these patients often have hepatocellular dysfunction due to
hypotension, hypoxia, and major surgery, as well as renal impairment, the severity
and duration of jaundice may be marked. In a similar way, massive pulmonary infarc-
tion can cause hyperbilirubinaemia.
Sepsis
A massive haemolysis can occur in association with Clostridium perfringens
(was C. welchii) infection 24 to 72 h after gastric, biliary tract, or colonic surgery.
The typical clinical picture is of a restless hypotensive patient, an acute rise in serum
bilirubin, and crepitus around the wound site. Several causes of liver dysfunction
probably occur simultaneously in these patients as the conjugated bilirubin level can
be greater than the unconjugated. As these cases can be fatal, prompt treatment with
massive doses of penicillin and hyperbaric oxygen are imperative. Meningococcal,
pneumococcal, and Gram-negative sepsis can cause haemolysis through disseminated
intravascular coagulation and secondary microangiopathic haemolysis. As sepsis can
also cause intrahepatic cholestasis, a combination of the two factors may cause
marked jaundice.
Open heart surgery
Early and late rises in bilirubin are seen after open-heart surgery. Early onset
jaundice may be seen in up to 23 per cent of such patients and the main contributing
factors are hypoxia, severity of right-heart failure preoperatively, and number of units
of blood transfused. Although it has been suggested that cardiopulmonary bypass and
prosthetic valves cause haemolysis, these are probably not significant contributors to
the increased bilirubin. Late jaundice due to an autoimmune haemolytic anaemia has
been reported where anaemia and jaundice, exacerbated by repeat transfusion, occur a
few weeks after surgery. The presence of antiglobulin antibodies confirms the diag-
nosis; steroids are the treatment of choice.
Gilbert's syndrome
Gilbert's syndrome is a benign, familial, mild, unconjugated hyperbilirubinaemia
that affects 2 to 5 per cent of the population. It is more common in men than women
and presents in the second or third decade. It is usually diagnosed incidentally at a
routine medical examination or when blood tests have been taken for another reason.
The serum bilirubin does not exceed 100μmol/l and is usually less than 50 μmol/l. Se-
rum liver biochemical tests and hepatic histology are normal. There are no abnormal
physical signs. Jaundice is usually mild but deepens with fasting and therefore may
occur in the postoperative period.
Gilbert's syndrome has an autosomal recessive mode of inheritance. The pro-
moter region of the gene encoding UGT1*1 is lengthened in patients with this condi-
tion resulting in reduced UGT1 (bilirubin uridine-diphosphate glucuronosyl transfer-
ase) enzyme production. Hepatic clearance of bilirubin is thus reduced. The diagnosis
can be confirmed by: a two- to threefold increase in bilirubin induced by a 48-h fast
and reversed by the resumption of a normal diet; the fall in bilirubin on taking pheno-
barbitone, which induces the hepatic conjugating enzyme; and the increase following
intravenous injection of nicotinic acid, which raises the fragility of red blood calls.
There is reduced bilirubin UDP-glucuronosyl transferase activity in liver biopsies;
however Gilbert's syndrome can usually be diagnosed without recourse to biopsy. A
diagnosis of Gilbert's syndrome must be considered if an isolated raised level of un-
conjugated bilirubin is noted after fasting or episodes of vomiting postoperatively.

Circulatory failure
Circulatory failure/ surgical shock
Circulatory failure contributes to hepatic dysfunction in many surgical situa-
tions, although it is rarely the sole cause of the liver abnormality. Major trauma,
burns, sepsis, massive blood loss, and surgery can be precipitants of ‘shock', and
these factors often occur together. In particular, gastrointestinal blood loss and septi-
caemia increase the risk of liver dysfunction when associated with hypotension. Cho-
lestasis is the most common pattern of injury following hypotension, and this is a be-
nign complication with a good prognosis. Prolonged hypotension, which is often as-
sociated with increased right atrial pressure, results in ischaemic hepatitis, for exam-
ple in open-heart surgery. There is an initial striking elevation of serum aminotrans-
ferases up to 200 times the normal level, a marked decrease in prothrombin time, and
a typically delayed bilirubin rise. These dramatic changes are seen within hours of
surgery, and where no severe liver damage has occurred they revert rapidly to normal
with restoration of liver blood flow and oxygenation. However, massive centrilobular
hepatic necrosis can occur, and the ischaemic hepatitis can progress to fulminant he-
patic failure, which has a high mortality rate. The clinical manifestation of hypoxic
liver cell necrosis inevitably postdates the hypoxic event, and other causes, especially
a viral hepatitis, must be considered.
Massive haemorrhage in combination with massive transfusion (for example,
more than 20 units of blood) puts the liver particularly at risk of damage, should the
patient survive. Patients with major trauma are particularly at risk of this form of liver
damage as well as that due to direct liver injury. In one study, 2 per cent of patients
with major trauma and shock developed significant jaundice. Patients with major
burns form another group in which circulatory failure is an important factor in the
etiology of the associated hepatic dysfunction. Haemolysis often adds to the bilirubin
load on the liver.
Hepatic artery ligation
The normal liver usually tolerates hepatic artery ligation without significant sequel
unless the flow of portal-vein blood is inadequate because of vascular stricture and
sepsis. Minimal derangements of bilirubin and alkaline phosphatase levels occur, and
moderate increases in the aminotransferase levels in the first week may be the only
consequence. Hepatic arterial collateral vessels develop very rapidly and this, in
combination with the portal circulation, reduces the ischaemic insult. Extensive mobi-
lization of the liver can involve division of the ligamentum and triangular ligament
and if this precedes hepatic artery ligation, massive liver necrosis may result. If in-
farction occurs, the amounts of bilirubin and aminotransferases rise rapidly to high
levels.
Post-transfusion hepatitis
The incidence of post-transfusion hepatitis has declined dramatically over the
past 50 years by the identification of the viral agents responsible for the majority of
cases and the development of antibody assays to screen donated blood for their pres-
ence. In countries where volunteer blood donors are used and hepatitis B surface an-
tigen and hepatitis C antibody screening are routine, clinical post-transfusion hepatitis
has been virtually eradicated. Although hepatitis G virus is certainly a transfusion
transmissible agent, there is currently no evidence to suggest a causal relationship be-
tween HGV infection and hepatitis. HGV screening is therefore not routinely under-
taken.
Drugs
Many drugs used in the pre- and postoperative period have been associated
with liver dysfunction. Almost every naturally occurring liver disease that affects
humans can be mimicked by the toxic effects of drugs on the liver and this occurs
through a wide range of mechanisms. Drugs can affect bilirubin metabolism at any
stage causing hyperbilirubinaemia. The drug or its metabolite can be hepatotoxic or
can precipitate a hypersensitivity reaction. Hepatocellular dysfunction may be due to
cellular necrosis or intrahepatic cholestasis. Factors that increase the risk of drug-
induced hepatic injury include pre-existing liver disease, increasing age, female sex,
concurrent therapy, and genetic polymorphism. Early symptoms of drug-induced liver
injury are non-specific and include loss of appetite, lassitude, and occasionally right
upper quadrant discomfort. There may be few clinical signs however, even in a pa-
tient who has biochemical and histological evidence of considerable hepatobiliary
damage. Hypersensitivity reactions may be associated with a fever, rash, or eosino-
philia. Jaundice in drug-induced liver injury carries a poor prognosis with a fatal out-
come of approximately 10 per cent.
The list of potentially hepatotoxic agents is large and ever increasing. The gen-
eral anaesthetic drugs are discussed separately. A hepatitic serum biochemical pattern
must lead to exclusion of a viral etiology, and the differentiation of intrahepatic and
extrahepatic cholestasis is important and should be elucidated with ultrasound scan-
ning. Liver biopsy will only rarely give a diagnosis. Diagnostic challenge with a sus-
pected drug is not recommended as a severe or even fatal reaction can occur.
General anaesthetic drugs
Halothane is amongst the most important of the idiosyncratic hepatotoxins. It
was first introduced in 1956 and within 4 years there had been several reports of
postoperative liver necrosis. The National Halothane Study reported the incidence of
massive hepatic necrosis to be 1 in 35 000 halothane anaesthetics. Two subsequent
studies suggested that the incidence was even higher at 1 in 6000 and 1 in 20 000 us-
es. After acetaminophen, halothane is the second commonest drug cause of fulminant
hepatic failure. Nevertheless, it is still a commonly used general anaesthetic agent
with many favorable properties and few adverse effects. Two types of halothane-
induced liver injury appear to exist. Ten to thirty per cent of patients exposed to halo-
thane develop asymptomatic elevations of aminotransferase levels with no clinical
features of liver disease. This condition is benign and self-limiting. Its relationship to
the rare, severe syndrome of halothane hepatitis is unclear. The latter condition may
represent the severe end of a spectrum of liver injury associated with halothane expo-
sure or, more likely, is a separate idiosyncratic reaction. Multiple exposures are the
single most important risk factor for halothane hepatitis. Eighty per cent of patients
developing the condition have received halothane more than once, usually in the pre-
ceding 28 days. Women are more commonly affected, as is the case with many other
types of idiosyncratic hepatic drug reactions. Obesity is also a significant risk factor,
possibly due to increasing body stores of halothane or because of higher hepatic ac-
tivity of P450 2E1, an enzyme which catalyses the metabolism of halothane to reac-
tive metabolites. Concomitant drug therapy with microsomal enzyme inducing agents
may also predispose to halothane hepatitis, and there is evidence of a genetic predis-
position to developing the condition.
Fever is usually the initial symptom of halothane hepatitis and this occurs 7 to
14 days after a first exposure to the drug but earlier after multiple exposures. Symp-
toms of hepatitis occur 2 to 5 days later with anorexia, malaise, nausea, vomiting, and
right upper quadrant pain. In most cases, dark urine, pale stools, and jaundice follow,
although icteric cases of halothane hepatitis also occur. Liver biochemistry is typical
for acute hepatocellular necrosis with grossly elevated aminotransferase levels (e.g.
alanine aminotransferase raised to 10 times the normal level) and elevated serum bili-
rubin levels, whereas the alkaline phosphatase level is often less than twice normal.
Between 10 and 40 per cent of patients develop eosinophilia. The main histological
feature is centrilobular necrosis, varying from a multifocal spotty picture to confluent
massive necrosis. Ballooning degeneration of hepatocytes, inflammatory infiltrate,
stromal fibrosis, fatty change, and occasionally granulomatous aggregates are also
seen. Distinction from viral hepatitis may be difficult. A number of factors have been
postulated in the pathogenesis of halothane hepatotoxicity which include toxic prod-
ucts of metabolism, hypersensitivity, genetic predisposition, regional hepatic hypoxia,
and altered calcium homeostasis.
Management, as for all types of drug-induced acute hepatitis, is supportive. Pa-
tients with fulminant hepatic failure are best cared for in specialist centers and ideally
in proximity to a transplant unit. Patients who have had an adverse hepatic reaction to
halothane should be warned about the dangers of future exposure to the drug and ad-
vised to wear a Medic Alert bracelet. Up to 90 per cent of cases of halothane hepatitis
could be prevented by taking an appropriate history before administering anaesthesia
and by adhering to safety guidelines. Enflurane hepatitis has been described, but on
closer examination in many cases the alternative causes of liver injury had not been
adequately excluded. True cases are extremely rare. The difference in hepatotoxicity
between halothane and the other haloalkane anaesthetics is directly related to their
potential to undergo P450-mediated metabolism. Around 30 per cent of halothane is
metabolized, whereas the figures for enflurane and isoflurane are 2 per cent and less
than 1 per cent, respectively. When enflurane hepatitis does occur, it is similar to hal-
othane hepatitis in clinical presentation and histological features, and the two condi-
tions probably share the same pathogenesis. Case reports of hepatotoxicity associated
with isoflurane are extremely rare and so far sevoflurane and desflurane do not appear
to have any adverse effects on the liver.
Total parenteral nutrition (TPN)
Since its advent in the 1960s, parenteral nutrition has become safer, more relia-
ble, and progressively more efficient. However, complications still occur and hepato-
biliary abnormalities are second only to catheter sepsis in requiring cessation of par-
enteral feeding. A number of different patterns of liver dysfunction occur.
Short-term TPN
Hepatic steatosis (fatty change) is the earliest and most benign hepatic lesion. It
occurs within the first 14 days of TPN administration and is often, but not necessari-
ly, paralleled by a rise in the serum aminotransferase levels. Patients receiving fat-
free TPN are much more likely to develop steatosis, and standard TPN regimens now
supply a proportion of calories as a lipid emulsion to minimize this problem. The ini-
tial change on histology is periportal fat infiltration but this may progress to pan- or
centrilobular infiltration. A number of factors influence the accumulation of fat with-
in the liver. Hepatic lipid metabolism is influenced by the balance between insulin
and glucagon. High concentration glucose infusions induce high insulin levels and
suppress glucagon production. Glycogenesis is therefore favored over lipolysis. Lipid
may also accumulate because of increased delivery from peripheral fat stores and
from defective production of lipoproteins that transport triglycerides from the liver.
Other proposed causes of a fatty liver include excess activity of endotoxins or tumor
necrosis factor, glutamine deficiency, a toxic effect of tryptophan metabolites, choline
and carnitine deficiency, and increased bacterial translocation from an atrophic gut.

Long-term TPN
Chronic progressive liver disease is rare but well described in patients receiving
long-term TPN. One study found 3 of 60 patients on home TPN developed clinically
severe liver disease. Of these, one died from hepatic encephalopathy and hepatorenal
syndrome after 11 years of TPN and another patient died postoperatively following a
cholecystectomy and duct exploration. Patients requiring parenteral nutrition are like-
ly to have multiple other risk factors for hepatic dysfunction such as hepatotoxic
drugs, multiple transfusions, and repeated surgery. It can therefore be difficult to iso-
late TPN as the cause. The histological picture may be similar to that seen in patients
on shorter-term TPN, but beyond 6-months therapy a cholestatic picture is the com-
mon finding. Cholestasis, hepatocyte necrosis, an alcoholic hepatitis-like picture,
steatonecrosis, and early cirrhosis may all occur. The pathogenesis is likely to multi-
factorial, involving any or all of the mechanisms discussed under short-term nutrition
and complicated by the underlying disease process. Patients with short bowel syn-
drome have the worst prognosis. Cholelithiasis becomes progressively more common
with increasing length of parenteral feeding. Biliary sludging has been found in 100
per cent of patients treated with more than 6 weeks of TPN and 23 per cent of 109 pa-
tients developed clinical cholecystitis during TPN treatment. Gallbladder stasis is the
most likely cause of gallstone disease in patients on TPN. Gallbladder contractions
are reduced by approximately two-thirds during exclusive parenteral nutrition.
Management of TPN-induced hepatobiliary disease
Once TPN has been identified as the most likely cause of deranged liver func-
tion, the optimal management is to restart enteral nutrition where possible. Liver
function tests will return to normal in most patients within 1 month of cessation of
TPN. If however continued parenteral nutrition is unavoidable and liver abnormalities
persist and worsen, a number of therapeutic measures can be attempted. First, a
change in the composition of the TPN may be helpful. Lipid emulsions should be
administered as approximately one-third of total calories. Patients should not receive
more than 3 g/kg per day of the lipid preparation, however, as this may predispose to
hepatic fat accumulation. Second, changing the timing of TPN administration (‘cy-
cling') may improve liver function. TPN solution is given for 8 to 12 h every 24 h ra-
ther than as a continuous infusion. This approach reduces the time during which the
serum glucose concentrations are high, thereby avoiding persistently high insulin lev-
els which may stimulate hepatic lipogenesis. An improvement in liver function tests
will take 2 to 3 weeks to manifest after changing from continuous infusion to cycling.
If neither of these two
approaches is effective, the total caloric intake will need to be reduced to prevent
progressive liver disease.
A number of other therapeutic options are currently under investigation. Met-
ronidazole appears to prevent the development of intrahepatic cholestasis in some
adult patients on TPN. The proposed mechanism for this action is the prevention of
the intestinal overgrowth of anaerobic bacteria allowing the bacterial 7à-
dehydroxylation of chenodeoxycholic acid to the potentially hepatotoxic lithocholic
acid. Ursodeoxycholic acid has been shown to improve the cholestatic liver function
tests of patients with intestinal failure treated with home TPN. Choline supplementa-
tion has been shown to reverse hepatic steatosis in a small number of patients on
long-term TPN who have low plasma concentrations of free choline. The most effec-
tive approach to the problem of TPN-induced biliary disease has yet to be established.
Cholecystokinin, chosen for its prokinetic effect on the gallbladder, has been used in
one small study, where it seemed to prevent the formation of biliary sludge in patients
treated with TPN. There is no evidence available yet, however, that it has any useful
effect on patients with established biliary disease. Ursodeoxycholic acid and che-
nodeoxycholic acid have been shown to prevent gallstone formation in animal studies
of TPN-induced biliary disease.
Fasting
Mild hyperbilirubinaemia can be precipitated by fasting and is due mainly to an
unconjugated bilirubin rise. The majority of patients showing this effect are probably
those with Gilbert's syndrome. Fatty change is also seen, particularly in acute weight
loss or starvation. This is related to the increase in serum fatty acids and increased
fatty acid turnover precipitated by decreased availability of glucose, a rise in gluca-
gon levels, and increased sympathetic nervous activity. Obese subjects who lose
weight rapidly may show a transient elevation of serum liver enzymes.
Obesity
Fatty change in the liver is seen in up to 50 per cent of subjects who are obese,
with occasional periportal inflammation and fibrosis. Steatonecrosis and cirrhosis
have been reported but this may be due to coexistent diabetes mellitus or alcoholic
liver disease. Fifty per cent of patients who are obese can be shown to be glucose in-
tolerant, and this and excess dietary fat and carbohydrate in relation to protein intake
may be involved in the etiology of steatosis. The fatty infiltrations are perivenular and
diffuse. Liver function tests may be abnormal and reflect more severe histological
change. The changes are, in general, benign and non progressive, and can be reversed
by weight loss.
Diabetes mellitus
Patients with diabetes also show fatty change in the liver; the majority of pa-
tients being non-dependent on insulin and also overweight. Steatosis is very rare in
patients with juvenile-onset insulin-dependent diabetes. Symptoms are rare; an en-
larged, slightly tender liver may be found on examination, and liver function tests
may be slightly deranged in about 20 per cent of patients with diabetes, but do not
correlate with histology. The fatty changes are centrilobular and diffuse. Weight loss
and good diabetic control will resolve these abnormalities. Steatonecrosis may also
occur and this is seen in the non-insulin-dependent group. It has been suggested that
the incidence of cirrhosis among patients with diabetes is twice that of the general
population. This suggestion is unproved and may originate in the number of patients
with cirrhosis who are glucose intolerant and have wrongly been classified as having
primary diabetes. Emergency biliary surgery in patients with diabetes has a higher
than expected mortality. This is due in part to the disruption of glucose control caused
by surgery, the increased risk of infection due to leukocyte dysfunction, and poorer
wound healing.
Sepsis
Hepatocellular dysfunction occurs early in sepsis despite a hyperdynamic cir-
culation and increased hepatic perfusion. This effect is mediated via Kupffer cell- or
macrophage-derived proinflammatory cytokines such as tumour necrosis factor and
interleukin 2. Sepsis can produce a deep jaundice, which may be cholestatic and oc-
curs 2 to 4 days after the onset of bacteraemia. Pneumonia, Gram-negative bacterae-
mia, intra-abdominal abscess, and pyelonephritis can all cause a raised bilirubin.
Gram-negative infection in infants frequently causes cholestasis. As in most cases of
hepatic dysfunction discussed here, sepsis may be only one element in a multifactori-
al etiology. Biochemically and histologically, the changes are very similar to those
observed with circulatory failure, with a moderate rise in conjugated bilirubin, ami-
notransferases, and alkaline phosphatase levels. However, an increase in the unconju-
gated bilirubin level also occurs, giving a rise in total bilirubin out of keeping with
the increase in liver enzymes. Hepatic histological changes include biliary stasis, fatty
change, and periportal inflammation. Extrahepatic biliary obstruction must be exclud-
ed. Pneumococcal, meningococcal, and Gram-negative sepsis may cause haemolysis
by disseminated intravascular coagulation or a secondary microangiopathic haemoly-
sis, and in these conditions the rise in unconjugated bilirubin will be prominent.
Benign postoperative intrahepatic cholestasis
‘Benign postoperative intrahepatic cholestasis' is unlikely to be a specific enti-
ty. It occurs in situations where blood loss is a prominent problem and is probably
due to a combination of hypotension and multiple blood transfusions. Caroli in 1950
was the first to describe the occurrence of postoperative cholestatic jaundice. Benign
postoperative intrahepatic cholestasis has been included in all lists of causes of post-
operative jaundice since about this time. The etiology of postoperative cholestasis is
discussed within this section, and the majority of cases given this label in the past
now have a definable cause.
Extrahepatic obstruction
Bile duct injury
Bile duct injury can follow cholecystectomy, common bile duct exploration, or
any upper abdominal operation. If unrecognized at operation, jaundice, biliary fistula,
or biliary peritonitis will occur in the early postoperative period. ERCP plays an im-
portant role in the diagnosis and treatment of postoperative biliary injury.
Common bile duct stones
Retained common bile duct stones after cholecystectomy and/or exploration of
the common bile duct are uncommon. In the majority of cases, ERCP will both diag-
nose and treat this problem by sphincterotomy. Reoperation is required if ERCP fails
or is not available. Some practitioners advocate visualization and, if necessary, clear-
ance of the bile duct at ERCP prior to cholecystectomy. Occasionally, blood may col-
lect in the common bile duct and cause obstruction.
Postoperative pancreatitis
Acute postoperative pancreatitis is uncommon and the cause is unknown. Thir-
ty per cent of patients may be jaundiced, and oedema of the head of the pancreas is
thought to result in some degree of obstruction and a low-grade hyperbilirubinaemia.
Acalculous cholecystitis
Acute non-calculous cholecystitis can occur after major trauma, burns, surgery
that does not involve the upper abdomen, and in patients receiving long-term total
parenteral nutrition, especially infants. It accounts for about 1 per cent of all cases of
cholecystitis. A Japanese series of acalculous cholecystitis after gastrectomy demon-
strated an incidence of 0.64 per cent. The etiology is unknown, but it has been sug-
gested that biliary stasis is important. Postoperative cholecystitis occurs most com-
monly in the fifth to seventh decade, but in patients with trauma or burns this form of
cholecystitis is seen most frequently in the second to fourth decade. The sex ratio is
also different in these two groups; females predominate in the former, males in the
latter. The postoperative form tends to follow a major surgical procedure. This form
of cholecystitis can occur up to 1 month after the operation. Right upper quadrant
pain and tenderness is usually accompanied by nausea, vomiting, and fever. The ob-
served bilirubin rise is variable but may be up to 85 μmol/l; levels of aminotransfer-
ases and alkaline phosphatase are only mildly raised. Ultrasound may show enlarge-
ment of the gallbladder and, by definition, no gallstones are seen. ERCP is often nec-
essary to exclude other causes of obstruction, although cholecystectomy should not
be delayed in these already seriously ill patients. Histologically, the gallbladder
shows vascular dilatation, congestion, and edema in all layers, without fibrosis. Ab-
scesses of varying size may be seen in the gallbladder wall and the mucosal surface is
necrotic and ulcerated. Perforation is frequent.
Acalculous cholecystitis occurs in patients receiving total parenteral nutrition for
more than 3 months with an incidence of 4 per cent. The level of mortality has been
given as between 33 and 75 per cent. However, this may pertain only to patients with
major trauma and reflects the already much increased mortality in this group.
Basic literatures:
35. Oxford Textbook of Surgery (3-Volume Set) 2nd edition (January 15, 2000):
by Peter J. Morris (Editor), William C. Wood (Editor) By Oxford Press
36. Sabiston Textbook of Surgery 17th edition by Courtney M. Townsend Jr.,
Kenneth L. Mattox, B. Mark, MD Evers, Kenneth L., MD Mattox, Courtney Town-
send, Daniel Beauchamp, B. Mark Evers, Kenneth Mattox W.B. Saunders Company
(June, 2004)
37. Schwartz´s Principles of Surgery 8th Edition F.Charles Brunicardi. Copyright
©2007 the McGraw-Hill Companies.
38. Hospital surgery/ Edited by L.Kovalchuk et al. - Ternopil: Ukrmedkny-
ha,2004.- 472 p.

Additional literatures:

1. Klein AS, Lillemoe KD, Yeo CJ, et al: Liver, biliary tract, and pancreas, in
O'Leary JP (ed): Physiologic Basis of Surgery. Baltimore: Williams & Wilkins, 1996,
p 441.
2. Scott-Conner CEH, Dawson DL: Operative Anatomy. Philadelphia: JB Lip-
pincott Company, 1993, p 388.
3. Lee HJ, Choi BI, Han JK, et al: Three-dimensional ultrasonography using the
minimum transparent mode in obstructive biliary diseases: Early experience. J
Ultrasound Med 21:443, 2002. [PMID: 11934101]
4. Strasberg SM: The pathogenesis of cholesterol gallstones a review. J
Gastrointest Surg 2:109, 1998. [PMID: 9925435]
5. Chikamori F, Kuniyoshi N, Shibuya S, et al: Early scheduled laparoscopic chol-
ecystectomy following percutaneous transhepatic gallbladder drainage for patients
with acute cholecystitis. Surg Endosc 16:1704, 2002. [PMID: 12209324]
6. Grobmyer SR, Lieberman MD, Daly JM: Gallbladder cancer in the twentieth
century: Single institution's experience. World J Surg 28:47, 2004. [PMID:
14639492]

Tests for initial level of knowledge, keys for tests:


1. Complains of acute cholangitis are:
A. High temperature, pain in the right subcostal area, jaundice.
B. Nausea, vomiting.
C. Ground.
D. Labile and liquid stools.
E. Zoster pain.
2. Patient R., female 63 year old, complaining of severe acute pain in the right half of
the abdomen, which irradiates to the right supraclavicular area, an increase in body
temperature, dryness and bitterness in the mouth. There have been repeated vomiting,
which does not bring relief. The beginning of pain is linked with consumption of fatty
and fried foods. The patient lies on the right side. Pale. Tachycardia. Tongue is dry.
Painful palpation in the right half area of the abdomen and somewhat strained in the
right subcostal area. Give probable diagnosis.
A. Acute appendicitis
B. Perforated ulcer of the stomach.
C. Acute cholecystitis.
D. Right renal colic.
E. Acute intestinal obstruction.

3. A 48 old-year patient, male, sick for two weeks when he noticed icterus of the scle-
ra. He notes on slight pain in the right subcostal area. The pain is not acute. After two
days of onset, appeared gray stools, jaundice of skin which gradually increased. Gen-
eral condition is satisfactory. On sonography revealed enlargement intrahepatic and
extrahepatic bile ducts and main pancreatic duct. What is the most likely diagnosis?
A. Major duodenal papilla tumor.
B. Choledocholithiasis.
C. Norm cholestatic hepatitis.
D. Cirrhosis.
E. Total liver tumor.

4. The female patient operated on of chronic calculous cholecystitis and choledocho-


lithiasis. Three years after surgery patient has a recurrent choledocholithiasis. Which
of the listed causes of non-core, leading to relapse in extrahepatic bile duct calculus
after a cholecystectomy?
A. Failure to comply with the recommendations of diet.
B. Availability impede outflow of bile.
C. Narrowing of the terminal part of the CBD and associated chol-
angitis.
D. Long-cystic duct stump.
E. In-seam threads education CBD not resolves.

5. The patient 52 year-age after the attack of pain in right subcostal area, had yellow-
ness of skin and mucous membranes. On ultrasound investigation, calculi were found
in the gallbladder, choledoch was extended to 1.5 cm, and choledoch content is not
clearly visualized. Which test will confirm of choledocholithiasis?
A. Fibrogastroduodenosсopy.
B. Endoscopic retrograde сholangiography.
C. Ultrasound re-examination after training.
D. Computed tomography.
E. Blood tests for bilirubin and liver test.
6. During operation with acute destructive cholecystitis presence of purulent cholan-
gitis. While there were no choledoch calculus and duodenal stenosis, symptom nipple
was present. Gallbladder is removed. How to finish the operation?
A. Supraduodenal choledochoduodenoanastomois.
B. External drainage of the CBD.
C. Transduodenal choledochoduodenostomy.
D. Choledochojejunoanastomosis.
E. Drainage of abdominal cavity.

7. The 38 old-years patient, after a cholecystectomy two years ago, had attacks of he-
patic colic with yellowing of the skin. Postcholecystectomy syndrome is diagnosed.
Which of the following is often the cause of relapse?
A. Abandoned during the first operation calculus.
B. Stenos papilitis.
C. Duodenostasis.
D. Dyskinesia of the biliary duct.
E. Cholestasia.

8. The female patient 32 year-old, had a cholecystectomy two years ago, connected
with acute calculous cholecystitis.. In gallbladder calculus was much smaller. During
six months, she has three times repeated attacks of hepatic colic. Two days ago, after
the attack, appeared yellowing of the skin and sclera. Bilirubin is 90 µmol /L. What
research can help to diagnose this patient effectively?
A. Ultrasound of pancreatoduodenal zone.
B. Ray examination of gastrointestinal tract.
C. Gastroduodenoscopy.
D. Oral-cholangiography.
E. Angiography.

9. The patient 32 year-old, female, after cholecystectomy 6 months ago, regenerates a


liver colic attacks accompanied yellowing of skin. On ultrasonography revealed left 2
x 1 cm calculus in distal part of choledoch. What treatment is best for the patient?
A. Endoscopic + duodenocholedochoscopy with papilosphincterot-
omy.
B. Conservative therapy and antispasmodic drugs.
C. Lithotripsy.
D. Choledocholithotomy.
E. Choledochoduodenoanastomosis.

10. A male patient, 62 years old, complains of presence of jaundice, itching of skin,
light stool and dark urine. OBJECTIVE: scleras and skin are yellow, the traces of
wound on skin. Gallbladder is enlarged and painless. Blood tests: Total bilirubin 85
µmol / L. What method of research is optimally used to clarify the diagnosis?
A. Infusion cholegraphy.
B. Duodenal-sensing.
C. X-ray examination of the abdomen.
D. Oral-cholecystography.
E. Retrograde cholangiopancreatography.
Keys for tests

1 2 3 4 5 6 7 8 9 10
A C A A B B A A A E

Tests for final level of knowledge, keys for tests:

1. 60 years old patient, male, has jaundice for over 3 weeks, began without pain and
pain is getting more intensive. Abdomen palpation is soft. Symptom Courvoisier's is
positive. Ultrasound marked biliary tract and gallbladder enlargement choledoch.
What is the origin of these changes?
A. Hepatitis.
B. Calculous biliary disease.
C. Chronic pancreatitis.
D. Cancer head of the pancreas.
E. Cancer of the liver.

2. Female 68-years old complaining on yellowness of skin and sclera, grey stool, dark
colour of the urine. Body temperature increased to 37.4 degrees. Jaundice appeared a
day after attack of pain in the right subcostal area. Abdomen soft, painful in the right
subcostal area, liver is not increased. Symptom Murphy is positive. The gallbladder is
increased. What is the most probable diagnosis?
A. Acute cholecystitis, obstructive jaundice.
B. Cholestatic hepatitis.
C. Residual choledocholithiasis.
D. Cholecystolythiasis.
E. Cancer head of the pancreas.

3. Female 53-year old was operated with cholecystectomy for acute calculous chole-
cystitis one month ago. The operation finished outer drainage of choledoch. Cholan-
giography was not made. Drainage removed on 7 days after the operation. Three days
ago, the temperature increased to 38.8, had chills and sweating. Scleras are yellow,
stool is grey color. Abdominal palpation reveals pain in the right subcostal area. For-
mulate a preliminary diagnosis?

A. Residual choledocholithiasis, cholangitis, obstructive jaundice.


B. Postcholecystectomy syndrome.
C. Abscess of the liver.
D. Stenosis papillitis.
E. Stricture of the choledoch.

4. Indications to choledochotomy:
A. Empyema of the gallbladder.
B. Cholangitis, obstructive jaundice, stones in the duct.
C. Acute pancreatitis.
D. Hepatitis.
E. Perforated ulcer.

5. 43 years old patient complains of having night pain in the right subcostal area, gen-
eral weakness, bloating, and yellowness of the scleras. ALT 1.9., AST 2.4., bilirubin
88 µmol/L (direct 40, indirect 48). What is your recommendation for treatment?
A. Bile stimulated.
B. Antispasmodic.
C. Vitamins.
D. H2-blockers.
E. Ice on the abdomen wall.

6. Child 9-years old complaints of abdominal pain, more from the right, subfebrile
temperature, vomiting that does not bring relief. In the blood, clotting speed of eryth-
rocytes was found. He has been already ill for three years. What disease caused this
clinical picture?
A. Chronic cholecystitis
B. Chronic enterocolitis
C. Ulcerative colitis
D. Dyskinesia of the bill duct.
E. Acute intestinal infection

7. Female 53-year old came to the surgery department complaining of pain in the
right subcostal area that radiates to the right shoulder nausea, vomiting, fever 37.8 C.
Her belly is swollen and stiff during palpation, tenderness in the right subcostal, de-
termined in the same dense, painful movements. Positive symptoms by Orthner and
Mussy-Georgievsky are positive. In the blood - leukocytes are 12.6·109/l. Formulate a
preliminary diagnosis:
A. Tumor of the liver
B. Acute pancreatitis.
C. Acute gastroduodenitis.
D. Calculous biliary disease. .
E. Perforated ulcer.

8. Which of these operations used for postcholecystectomy syndrome and stenosis of


large duodenal papilla?
A. Choledochojejunoanastomosis.
B. Hepaticotomy.
C. Resection of the major duodenal nipple.
D. Transduodenal choledochoduodenostomy.
E. Endoscopic papillosphincterotomy.

9. What type of operation is used for removing stones in ampoule of the major duo-
denal papilla?
A. Transduodenal papillosphincterotomy.
B. Cholecystectomy.
C. Choledochotomy.
D. Segmental resection of choledochal.
E. External drainage of choledochal.
10. What operation results in syndrome of “blend bag”?
A. Choledochoduodenostomy.
B. Cholecystectomy.
C. External drainage of choledochal.
D. Choledochotomy.
E. Papillosphincterotomy.

Keys for tests

1 2 3 4 5 6 7 8 9 10

D A A B A A D E A A

Tasks for the final level of knowledge.

1. A 32-year-old woman undergoes a cholecystectomy for acute cholecystitis and is


discharged home on the sixth postoperative day. She returns to the clinic 8 mo after
the operation for a routine visit and is noted by the surgeon to be jaundiced. Laborato-
ry values on readmission show total bilirubin 5.6 mg/dL; direct bilirubin 4.8 mg/dL;
alkaline phosphatase 250 IU (normal 21–91 IU); SGOT 52 KU (normal 10–40 KU);
SGPT 51 KU (normal 10–40 KU). An ultrasonogram shows dilated intrahepatic
ducts. The patient undergoes the transhepatic cholangiogram (seen below). The ap-
propriate management is:

Answer: Roux-en-Y choledochojejunostomy

2. A 19-year-old college student returns from a trip to Cancun, and 2 weeks lat-
er develops malaise, weakness, and anorexia. A week later he notices jaundice.
When he presents for evaluation his total bilirubin is 12, with 7 indirect and 5
direct. His alkaline phosphatase is mildly elevated, and the SGOT and SGPT
(transaminases) are very high.

Answer: hepatocellular jaundice

3. A patient with progressive jaundice that has been present for 4 weeks is
found to have a total bilirubin of 22, with 16 direct and 6 indirect, and minimal-
ly elevated SGOT. The alkaline phosphatase was twice normal value of a cou-
ple of weeks ago, and now is about six times the upper limit of normal.

Answer: A generic example of obstructive jaundice

4. A 40-year-old obese mother of five children presents with progressive jaun-


dice, which she first noticed 4 weeks ago. She has a total bilirubin of 22, with
16 direct and 6 indirect, and minimally elevated SGOT. The alkaline phospha-
tase is about six times the upper limit of normal. She gives a history of multiple
episodes of colicky right upper quadrant abdominal pain, brought about by in-
gestion of fatty food.

Answer: obstructive jaundice, with a good chance of being caused by stones

5. A 66-year-old man presents with progressive jaundice, which he first noticed


6 weeks ago. He has total bilirubin of 22, with 16 direct and 6 indirect, and
minimally elevated SGOT. The alkaline phosphatase is about six times the up-
per limit of normal. He has lost 5 kg over the past 2 months, but is otherwise
asymptomatic. A sonogram shows dilated intrahepatic ducts, dilated extrahepat-
ic ducts, and a very distended, thin-walled gallbladder.

Answer: Malignant obstructive jaundice.

Materials for self-education

Main tasks Notes(instructions)


To repeat:
7. Anatomy of a gall bladder and hepato- - Represent methods of chronic cal-
duodenal ligament. culous cholecystitis diagnostics using
8. Pathophysiology of liver and pathophys- scheme.
iology extrahepatic biliary tract. - Make up a block diagram of symp-
9. Pathogenesis of calculous biliary disease toms of. calculous biliary disease and
and obstructive jaundice and complica- obstructive jaundice and complica-
tions. tions.

To study:
7. Age-specific features of calculous biliary -To do a differential diagnostic of
disease and obstructive jaundice and abdominal painful syndrome.
complications. -To do a differential diagnostic of
8. Tactic at complications of obstructive types of jaundice.
jaundice.
Study guide #8

“Peptic ulcer. Complications of gastric and duodenal ulcer: gastric outlet ob-
struction, malignization, penetration. Conservative and operative treatments,
types of operations. Diseases of the operated stomach”

Overview.
Gastroduodenal ulcer is one of the most widespread diseases of organs of ab-
dominal region. About 3,5 – 12% of population suffers from this pathology, thus 35 –
47% are people of working age. The morbidity of this disease in stomach and duode-
num has increased morbidity for 38,4% for 10 years in the end of the ХХ century has
grown in Ukraine.
Gastric outlet obstruction is the least frequent ulcer complication. Most cases
are associated with duodenal or pyloric ulceration, with gastric ulceration accounting
for only 5 percent of cases. Ulcer penetration refers to penetration of the ulcer into the
bowel wall without free perforation and leakage of luminal contents into the ab-
dominal organs. Surgical series suggest that penetration occurs in 20 percent of ul-
cers, but only a small proportion of penetrating ulcers will be clinically evident. The
incidence of malignancy in benign gastric ulcer is approximately about 1% to 2%.
The best method for surgical treatment of gastroduodenal ulcer is not decided
completely. Today the operation for gastric ulcer is a classic resection of stomach as a
Billroth I (preferable) or Billroth II. The risk of malignancy gave surgeons all
grounds for urgent interference on an ulcer taking into account all rules of ablastics.
However the most of postgastroresection complications predetermined disability in
20 – 30% patients are during the first year after operation after an operation but 10 –
15% patients develop these complications in more late terms in 6 – 80% cases. Ac-
cording to classification of О.О.Shalimov and V.F.Sayenko (1987), Postgatroresec-
tion complications are divided to three types: functional, organic and mixed. Vagot-
omy is the operation of choice for duodenal ulcer. Vagotomy is used for surgical
treatment of ulcer disease more than 30 years, the group of postvagotomic violations
is selected, and all complications after surgical treatment of ulcer disease are incor-
porated under heading „diseases of operated stomach”. Analyzing the results from
remaining implementation of vagotomy, clinicians have discovered the development
of functional violations in 5 – 15% patients, and also the percent of recurrent ulcer
from vagotomy – highly selective vagotomy (HSV) was even higher, than after the
resection of stomach. This percent was about 3,5–60%. Ulcer penetration refers to
penetration of the ulcer through the bowel wall without free perforation and leakage
of luminal contents into the peritoneal cavity. Surgical series suggest that penetration
occurs in 20 percent of ulcers, but only a small proportion of penetrating ulcers will
be clinically evident.
Educational aims:
1. To understand the view and influence of ulcer disease on economic, social and
biological factors on the dynamics of amount of amount of patients suffer from
with complicated forms of ulcerous illness of stomach and duodenum.
2. On material of theme to develop the sense of responsibility for the timeliness of
exposure of disease and rightness of professional actions for stopping the dis-
ease development of the operated stomach using pathogenetic grounded choice
of method of surgical intervention in the complicated ulcerous illness of stom-
ach and duodenum.
A student must know:
15. Anatomo-physiological information about a stomach and duodenum, gastric
secretion phases.
16. Etiology and pathogenesis of gastroduodenal ulcer.
17. Clinical view of gastric outlet obstruction and penetration.
18. Modern methods of instrumental diagnosis of gastroduodenal ulcer.
19. Basic principles for conservative treatment of the uncomplicated ulcerous ill-
ness.
20. Basic principles for conservative treatment of the compensated pyloroduode-
nal stenosis, and preoperative preparation of patients with sub- and decompen-
sated stenosis.
21. Modern methods of surgical treatment for complicated peptic ulcer and duo-
denal ulcer.
22. Informations about prophylaxis for postoperative complications, rehabilitation
and health centre system of patients.
23. Classification of diseases of the operated stomach.
24. Causes of origin of dumping-syndrome, syndrome of abductive intestinal loop,
agastric asthenia, digestive allergy, peptic ulcer of anastomosis and recurrent
ulcer, gastro-intestinal fistula classification, pathological refluxes, clinics, di-
agnostics and methods of conservative and operative treatment.
A student must be able to:
1. Collect and estimate information of anamnesis for a patient with compli-
cated ulcer.
2. Use physical methods of inspection for diagnostics of stenosis.
3. Set the algorithm of patient’s inspection and estimate the results of labora-
tory, instrumental inspection of patients.
4. Determine optimum medical pathways (conservative, surgical), in the
case of decompensated stenosis to be able to prove necessity of pre-
operation preparation of patient.
5. Collect and estimate information of anamnesis for a patient with the dis-
eases of the operated stomach.
6. Knowing the complaints, anamnesis, physiological and instrumental
methods of inspection of patients to diagnose syndrome of dumping.
7. Perform appropriate tests for diagnostics of dumping-syndrome, syn-
drome of efferent intestinal loop, alkaline gastritis.
8. Set the algorithm of examination the patient with illness of the operated
stomach and estimate the results of laboratory, instrumental inspection of
patients.
9. Define optimum medical options for a certain patient (conservative, sur-
gical).

Terminology.

Term Definition
are focal defects in the gastric or duodenal mucosa which
Peptic ulcers
extend into the submucosa or deeper
Classification of gas-
according to classification gastric ulcers is divided on
tric ulcers by John-
three types
son
Gastric outlet results from fibrous scarring of chronic duodenal ulcer
obstruction disease
Malignization transformation of gastric epithelium in cancer
Ulcer penetration refers to penetration of the ulcer through the bowel wall
without free perforation and leakage of luminal contents
into the abdominal organs
Diseases of the oper- are the diseases which arise up after surgical treatment of
ated stomach peptic or duodenal ulcer or other pathologies of these or-
gans.
Content:

PEPTIC ULCER DISEASE

Peptic ulcers are focal defects in the gastric or duodenal mucosa which extend
into the submucosa or deeper (Fig. 1).

Fig. 1. Diagram of gastric erosions and ulcers.

They may be acute or chronic, and ultimately are caused by an imbalance be-
tween the action of peptic acid and mucosal defenses (Fig. 2).

Fig. 2. Balance of acid/peptic aggressive factors and mucosal defensive fac-


tors in the gastric mucosa.

Peptic ulcer remains a common outpatient diagnosis, but the number of physi-
cian visits, hospital admissions, and elective operations for peptic ulcer disease have
decreased steadily and dramatically over the past 3 decades. These trends all predated
the advent of fiberoptic endoscopy, highly selective vagotomy, and the use of H2-
blockers. However, the incidence of emergency surgery and the death rate associated
with peptic ulcers are fairly stable. These epidemiologic trends probably represent the
net effect of several factors, including decreasing prevalence of H. pylori infection,
better medical therapy, increases in outpatient management, and the use of NSAIDs
and aspirin (with and without ulcer prophylaxis).

These epidemiologic facts notwithstanding, it is important to reiterate that pep-


tic ulcer is a common disease in the U.S. In 2000, the total direct costs (hospital, phy-
sicians, and drugs) of peptic ulcer disease was about $3.3 billion, with indirect costs
(lost work and productivity) of over $6 billion. The prevalence of peptic ulcer in the
U.S. is about 2%, and the lifetime risk is about 10%. In 1998, the crude mortality rate
for peptic ulcer was 1.7 per 100,000 individuals. Gastric ulcer has a higher mortality
than duodenal ulcer because of its increased prevalence in the elderly. Recent studies
have shown an increase in the rates of hospitalization and mortality in elderly patients
for the peptic ulcer complications of bleeding and perforation. Presumably this is due
to the increasingly common use of NSAIDs and aspirin in this elderly cohort, many
of whom have H. pylori infection.

Classification of peptic ulcer

• acute or chronic,
• complicated or noncomplicated,
• gastric or duodenal ulcer

Classification of gastric ulcers by Johnson (1965)

• type I - ulcers of lesser curvature (3 cm higher than the pylorus);


• type II- double localization of ulcers simultaneously in the stomach and duode-
num;
• type III - ulcers of pyloric end of stomach (not farther than 3 cm from the pylo-
rus).

Pathophysiology and Etiology

A variety of factors may contribute to the development of peptic ulcer disease.


Although it is now recognized that the large majority of duodenal and gastric ulcers
are caused by H. pylori infection and/or NSAID use, the final common pathway to
ulcer formation is peptic acid injury of the gastroduodenal mucosal barrier. Thus the
adage "no acid, no ulcer" remains true even today. Acid suppression, either with med-
ication or surgery, remains a mainstay in healing both duodenal and gastric ulcers and
in preventing recurrence. It generally is thought that H. pylori predisposes to ulcera-
tion, both by acid hypersecretion, and by compromise of mucosal defense mecha-
nisms. NSAID use is thought to lead to peptic ulcer disease predominantly by com-
promise of mucosal defenses. Duodenal ulcer has typically been thought of as a dis-
ease of increased peptic acid action on the duodenal mucosa, whereas gastric ulcer
has been viewed as a disease of weakened mucosal defenses in the face of relatively
normal action of peptic acid. However, increased understanding of the pathophysiol-
ogy of peptic ulcer has blurred this distinction. Clearly, weakened mucosal defenses
play a role in many duodenal and most gastric ulcers (e.g., duodenal ulcer in an H.
pylori–negative patient on NSAIDs or a patient with a typical type I gastric ulcer with
acid hyposecretion), whereas increased aggressive activity of peptic acid may result
in a duodenal or gastric ulcer in the setting of normal mucosal defenses (e.g., a duo-
denal ulcer in a patient with Zollinger-Ellison syndrome, or a gastric ulcer in a patient
with gastric outlet obstruction, antral stasis, and acid hypersecretion).

Elimination of H. pylori infection or NSAID use is important for optimal ulcer


healing, and perhaps is even more important in preventing ulcer recurrence and/or
complications. A variety of other diseases are known to cause peptic ulcer, including
Zollinger-Ellison syndrome (gastrinoma), antral G-cell hyperfunction and/or hyper-
plasia, systemic mastocytosis, trauma, burns, and major physiologic stress. Other
causative agents include drugs (all NSAIDs, aspirin, and cocaine), smoking, alcohol,
and psychologic stress. In the U.S., probably more than 90% of serious peptic ulcer
complications can be attributed to H. pylori infection, NSAID use, or cigarette smok-
ing.

Epidemiologic studies suggest that smokers are about twice as likely to develop
peptic ulcer disease as nonsmokers. Smoking increases gastric acid secretion and du-
odenogastric reflux. Smoking decreases both gastroduodenal prostaglandin produc-
tion and pancreaticoduodenal bicarbonate production. These observations may be re-
lated, and any or all could explain the observed association between smoking and
peptic ulcer disease.

Although difficult to measure, both physiologic and psychologic stress un-


doubtedly play a role in the development of peptic ulcer in some patients. In 1842,
Curling described duodenal ulcer and/or duodenitis in burn patients. Decades later,
Cushing described the appearance of acute peptic ulceration in patients with head
trauma (Cushing ulcer). Even the ancients recognized the undeniable links between
peptic ulcer disease and stress. Patients still present with ulcer complications (bleed-
ing, perforation, and obstruction) that are seemingly exacerbated by stressful life
events. Recently, the use of crack cocaine has been linked to juxtapyloric peptic ul-
cers with a propensity to perforate. Alcohol is commonly mentioned as a risk factor
for peptic ulcer disease, but confirmatory data are lacking.

Clinical Manifestations

Over 90% of patients with peptic ulcer disease complain of abdominal pain.
The pain is located in the epigastrium and is typically nonradiating, and has burning
in quality. The mechanism of the pain is unclear. Patients with duodenal ulcer usually
experience pain 2 to 3 hours after a meal and at night. Two thirds of patients with du-
odenal ulcers have complaint of pain that awakens them from sleep. The pain of pep-
tic ulcer more commonly occurs with eating and is less likely to awaken the patient at
night. A history of peptic ulcer disease, use of NSAIDs, over-the-counter antacids, or
antisecretory drugs, is suggestive of the diagnosis. Other signs and symptoms include
nausea, bloating, weight loss, stool positive for occult blood, and anemia. Duodenal
ulcer is about twice as common in men than women, but the incidence of gastric ulcer
is similar in men and women. On average, gastric ulcer patients are 10 years older
than duodenal ulcer patients, and the incidence is increasing in the elderly, probably
because of increasing NSAID use in this cohort with a high incidence of H. pylori in-
fection.

Diagnosis
In the young patient with dyspepsia and/or epigastric pain, it may be appropri-
ate to initiate empiric therapy for peptic ulcer disease without confirmatory testing.
All patients over 45 with the above symptoms should have an upper endoscopy, and
all patients, regardless of age, should have this study if any alarm symptoms are pre-
sent. A double contrast upper GI x-ray study may be useful. All gastric ulcers should
be adequately biopsied, and any sites of gastritis should be biopsied to rule out H. py-
lori, and for histologic evaluation. Additional testing for H. pylori may be indicated.
Although somewhat controversial, it is not unreasonable to test all peptic ulcer pa-
tients for H. pylori. A baseline serum gastrin level is appropriate to rule out gastrino-
ma.

Medical Treatment

The conservative treatment of ulcer has the following objectives:


1.pain suppression and creation of a comfort state of the patient,
2.favoring a rapid epithelization in the ulcerous notch,
3.prevention of major complications – hemorrhage and perforation.

Patients with peptic ulcer disease should stop smoking and avoid alcohol and
NSAIDs (including aspirin). If H. pylori infection is documented, it should be treated
with one of several acceptable regimens (Table 1).

Table 1. Treatment Regimens for Helicobacter pylori Infections

Bismuth triple therapy


Bismuth, 2 tablets four times daily
plus
Metronidazole, 250 mg three times daily
plus
Tetracycline, 500 mg four times daily
PPI triple therapy
PPI twice daily
plus
Amoxicillin, 1000 mg two times daily
plus
Clarithromycin, 500 mg two times daily
or
Metronidazole, 500 mg two times daily
Quadruple therapy
PPI twice daily
plus
Bismuth, 2 tablets four times daily
plus
Metronidazole, 250 mg three times daily
plus
Tetracycline, 500 mg four times daily
NOTE: Treatment for 10–14 days is recommended. PPI = proton pump inhibitor.

Infectious disease consultation may be helpful in the compliant, symptomatic


patient with persistent H. pylori infection following treatment; or another regimen
could be tried (e.g., quadruple therapy). If initial H. pylori testing is negative, the ul-
cer patient may be treated with H2-receptor blockers or proton pump inhibitors. Su-
cralfate or misoprostol may also be effective. If ulcer symptoms persist, an empiric
trial of anti-H. pylori therapy is reasonable (false-negative H. pylori tests are com-
mon). Generally, antisecretory therapy can be stopped after 3 months if the ulcer-
ogenic stimulus (usually H. pylori, NSAIDs, or aspirin) has been removed.

However, long-term maintenance therapy for peptic ulcer should be considered


in all patients admitted to hospital with an ulcer complication, all high-risk patients
on NSAIDs or aspirin (the elderly or debilitated), and all patients with a history of re-
current ulcer or bleeding. Consideration should be given to maintenance therapy in
refractory smokers with a history of peptic ulcer. Misoprostol, sulcralfate, and acid
suppression may be quite comparable in many of these groups, but misoprostol may
cause diarrhea and cramps, and cannot be used in women of childbearing age because
of its abortifacient properties. Conservative treatment leads to ulcer cicatrisation
within 12-15 weeks in 80% of cases.

Surgical Treatment
The indications for surgery in peptic ulcer disease are bleeding, perforation,
obstruction, and intractability or nonhealing. Gastric cancer must always be consid-
ered in gastric ulcer, whereas malignancy is almost never an issue in duodenal ulcer.
Fundamentally, the vast majority of peptic ulcers are adequately treated by a variant
of one of the three basic operations: highly selective vagotomy, vagotomy and drain-
age, and vagotomy and distal gastrectomy.

Highly Selective Vagotomy (HSV), also called parietal cell vagotomy or prox-
imal gastric vagotomy, is safe (mortality risk <0.5%) and causes minimal side effects.
The operation severs the vagal nerve supply to the proximal two thirds of the stom-
ach, where essentially all the parietal cells are located. It preserves the vagal innerva-
tion to the antrum and pylorus, and the remaining abdominal viscera (Fig. 3).

Fig. 3. Highly selective vagotomy.

Inadequate denervation of the proximal stomach due to technical error may


lead to inadequate acid suppression and an unacceptably high incidence of ulcer re-
currence. HSV decreases total gastric acid secretion by about 65 to 75%, which is
quite comparable to the reduction seen with truncal vagotomy and acid-suppressive
medication. Gastric emptying of solids is typically normal in patients after parietal
cell vagotomy; liquid emptying may be normal or increased due to decreased compli-
ance associated with loss of receptive relaxation and accommodation. When applied
to uncomplicated duodenal ulcer, the recurrence rate is higher with HSV than with
vagotomy and antrectomy. However, our increased understanding of the pathophysio-
logic role of H. pylori and NSAIDs in the development of recurrent ulcer may miti-
gate this concern. HSV has not performed particularly well as a treatment for type II
(gastric and duodenal) and III (prepyloric) gastric ulcer, perhaps because of hypergas-
trinemia caused by gastric outlet obstruction and persistent antral stasis.

HSV was accepted into the surgical armamentarium largely as a treatment for
uncomplicated, intractable duodenal ulcer. Although the operation has been shown to
be effective in treating selected patients with complicated peptic ulcer, its usefulness
in this regard remains suspect for two reasons. First, many surgeons feel that compli-
cated ulcer disease may call for a more radical operation than uncomplicated disease
(a hypothesis that has not been proven). Second, HSV was conceived as an ulcer op-
eration that preserves the pylorus and does not involve opening the GI tract. Most pa-
tients with complicated peptic ulcer disease need an ancillary procedure that invali-
dates these two technical advantages of HSV (e.g., pyloroduodenotomy to oversew a
bleeding duodenal ulcer, or gastrojejunostomy to bypass an obstruction).

The Taylor procedure (anterior seromyotomy and posterior truncal vagotomy)


is an attractive and simple alternative to HSV. This operation appears to have a simi-
larly low incidence of side effects, and an acceptably low incidence of recurrent ul-
ceration. Effective acid suppression is achieved and normal gastric emptying is main-
tained. Posterior truncal and anterior HSV are comparable. Although formal HSV is
readily accomplished as a laparoscopic procedure, these shortcut operations are par-
ticularly attractive to the laparoscopic surgeon, and merit consideration.

Vagotomy and Drainage

Truncal vagotomy and pyloroplasty, and truncal vagotomy and gastrojejunos-


tomy are the paradigmatic vagotomy and drainage (V+D) procedures. However, se-
lective vagotomy and drainage, and HSV and gastrojejunostomy may be useful ulcer
operations in selected patients. The advantage of V+D is that it can be performed
safely and quickly by the experienced surgeon. The main disadvantages are the side
effect profile (10% of patients have significant dumping and/or diarrhea), and a 10%
recurrent ulcer rate. Whether the incidence of these postoperative problems (hereto-
fore determined by studies predominantly involving patients with intractable uncom-
plicated duodenal ulcer) will be different in the current era, with our improved
knowledge of complicated ulcer, H. pylori, and NSAIDs, is unknown. A serious at-
tempt should be made to perform a complete truncal vagotomy (Fig. 4), keeping in
mind that in many patients there are more than two vagal trunks at the esophageal
hiatus.

Fig. 4. Truncal vagotomy.

During truncal vagotomy, care must be taken not to perforate the esophagus, a
potentially lethal complication. Intraoperative frozen section confirmation of at least
two vagal trunks is prudent. Unlike HSV, V+D is widely accepted as a successful op-
eration for complicated peptic ulcer disease. It has been described as a useful part of
the operative treatment for bleeding duodenal and gastric ulcer, perforated duodenal
and gastric ulcer, and obstructing duodenal and gastric (type II and III) ulcer. When
applied to gastric ulcer, the ulcer should be excised or biopsied.

Truncal vagotomy denervates the antropyloric mechanism, and therefore some


sort of procedure is necessary to ablate or bypass the pylorus; otherwise gastric stasis
often results. Gastrojejunostomy is a good choice in patients with gastric outlet ob-
struction or a severely diseased proximal duodenum. The anastomosis is done be-
tween the proximal jejunum and the most dependent portion of the greater gastric
curvature (Fig. 5).
Fig. 5. Retrocolic gastrojejunostomy

On the other hand, gastroduodenoplasty (pyloroplasty) or duodenoplasty are


useful in some patients who require duodeno- or pyloroduodenotomy to deal with the
ulcer complication (e.g., posterior bleeding duodenal ulcer), in those with limited or
focal scarring in the pyloric region, or when gastrojejunostomy is technically diffi-
cult. The most commonly performed duodeno- or pyloroduodenotomy are the Hei-
neke-Mikulicz type, which closes a longitudinal transpyloric incision in a transverse
fashion (Fig. 6 A,B,C).

A B C

Fig. 6. Heineke-Mikulicz duodeno- or gastroduodenoplasty

Other occasionally useful techniques include the Finney (Fig. 7A) and the
Jaboulay pyloroplasties (Fig. 7B).
A B

Fig. 7. The Finney and the Jaboulay pyloroplasties

These more extensive pyloroplasty techniques may make subsequent distal gas-
tric resection more difficult and/or hazardous.

The advantages of vagotomy and antrectomy (V+A) are the extremely low ul-
cer recurrence rate and the applicability of the operation to many patients with com-
plicated peptic ulcer disease (e.g., bleeding duodenal and gastric ulcer, obstructing
peptic ulcer, nonhealing gastric ulcer, and recurrent ulcer). When applied to gastric
ulcer disease, the resection is usually extended far enough proximally to include the
ulcer. The disadvantage of V+A is the somewhat higher operative mortality rate when
compared with HSV or V+D. Following antrectomy, gastrointestinal continuity may
be re-established, either via a Billroth I gastroduodenostomy or a Billroth II loop gas-
trojejunostomy (Fig. 8). Since antrectomy routinely leaves a 60 to 70% gastric rem-
nant, reconstruction as a Roux-en-Y gastrojejunostomy should be avoided. The Roux-
en-Y operation is an excellent procedure for keeping duodenal contents out of the
stomach and esophagus. However, in the presence of a large gastric remnant, this re-
construction will predispose to marginal ulceration and/or gastric stasis.
Fig. 8. Different variants of gastrojejunostomy (Billroth II)

Distal gastrectomy without vagotomy (usually about a 50% gastrectomy to in-


clude the ulcer) has traditionally been the procedure of choice for type I gastric ulcer.
Reconstruction may be done as a Billroth I (preferable) or Billroth II.

Truncal vagotomy is added for type II and III gastric ulcers, or if the patient is
believed to be at increased risk for recurrent ulcer, and should be considered if Bill-
roth II reconstruction is contemplated. Though not routinely used today in the surgi-
cal treatment of peptic ulcer, subtotal gastrectomy (75% distal gastrectomy) without
vagotomy may be an appealing choice for an occasional ulcer patient. Periesophageal
dissection is avoided (vagotomy is unnecessary if 75% gastrectomy is performed),
and extensive periduodenal dissection is minimized (Billroth II is the reconstruction
of choice). Finally, concomitant gastric ulcers (type II or III) are resected. However,
subtotal gastrectomy is rarely the first operation of choice for any patient with duode-
nal ulcer, since it leaves an inadequate gastric reservoir, and since vagotomy and an-
trectomy has a lower recurrent ulcer rate, is at least as safe, and has a similar side ef-
fect profile.

Choice of Operation for Peptic Ulcer


The choice of operation for the individual patient with peptic ulcer disease de-
pends on a variety of factors, including the type of ulcer (duodenal, gastric, recurrent,
or marginal), the indication for operation, and the condition of the patient, among
others. Other important considerations are intra-abdominal factors (duodenal scar-
ring/inflammation, adhesions, or difficult exposure), the ulcer diathesis status of the
patient, the surgeon's experience and personal preference, whether H. pylori infection
is present, the need for NSAID therapy, previous treatment, and the likelihood of fu-
ture compliance with treatment. Table 2 shows the surgical options for managing var-
ious aspects of peptic ulcer disease. In general, resective procedures have a lower ul-
cer recurrence rate, but a higher operative morbidity and mortality rate when com-
pared to nonresective ulcer operations. Because ulcer recurrence is often related to H.
pylori and/or NSAIDs, it is usually managed adequately without reoperation. Thus,
gastric resection to minimize recurrence in duodenal ulcer disease is often not justi-
fied today; resection for gastric ulcer remains the standard because of the risk of can-
cer.

Table 2. Surgical Options in the Treatment of Duodenal and Gastric Ulcer Dis-
ease

Indication Duodenal Gastric

Bleeding 1. Oversewa 1. Oversew and biopsya


2. Oversew, V+D 2. Oversew, biopsy, V+D
3. V+A 3. Distal gastrectomyb

Perforation 1. Patcha 1. Biopsy and patcha


2. Patch, HSVb 2. Wedge excision, V+D
3. Patch, V+D 3. Distal gastrectomyb

Obstruction 1. HSV + GJ 1. Biopsy; HSV + GJ


2. V+A 2. Distal gastrectomyb

a
Unless the patient is in shock or moribund, a definitive procedure should be consid-
ered. bOperation of choice in low-risk patient. GJ = gastrojejunostomy; HSV = high-
ly-selective vagotomy; V+A = vagotomy and antrectomy; V+D = vagotomy and
drainage.

GASTRIC OUTLET OBSTRUCTION

Gastric outlet obstruction results from fibrous scarring of chronic duodenal


ulcer disease. Symptoms develop over a long period of time, occasionally more
acutely due to edema caused by acute exacerbation of ulceration. But, even in the
latter circumstance, preexisting scarring and stenosis are likely to be confirmed.
Gastric outlet obstruction occurs less frequently than the complications of peroration
and bleeding. It is likely that its incidence has decreased because of the advent of
potent acid-reducing drugs and identification and eradication of H. pylori. Essentials
of the management are listed in Table 3.

Table 3. Essentials: management of Gastric Outlet Obstruction


Clinical Presentation The symptoms of gastric outlet obstruction are usually
insidious and accompanied by a chronic history of duodenal ulcer. The initial
symptoms are early satiety, bloating, and halitosis. When vomiting eventually
develops, it is usually after the last meal of the day. The vomitus may contain
undigested food eaten 24 to 48 hours earlier. As the obstruction becomes more
complete, vomiting may occur after any meal. Chronic weight loss, even emaciation,
and chronic fatigue develop. Physical examination may show the presence of
“succussion splash” (i.e., a splashing sound in the epigastrium when the patient is
shaken from side to side). Infrequently, particularly in the emaciated patient, gastric
peristalsis may be visible in the epigastrium.

Investigation A barium meal confirms gastric outlet obstruction by showing a


dilated stomach and a small amount of barium entering the duodenum. The use of
upper GI endoscopy is necessary to exclude antral cancer as the cause. The endoscope
cannot be passed into the duodenum, and no gastric pathology may be found. Antral
biopsy for histology and H. pylori studies hould be obtained. The typical biochemical
findings when prolonged vomiting is present are of hypochloremic, hypokalemic, and
metabolic alkalosis. Vomiting results in loss of fluid, chlorides, and H+. Severe de-
hydration develops, and the kidneys attempt to compensate by retaining Na+. To ac-
complish this, potassium is initially exchanged, but as dehydration progresses and po-
tassium stores become depleted, H+ is exchanged for Na+ in the renal tubules. Early
in the evolution of biochemical derangements caused by gastric outlet obstruction, the
urine is alkaline; however, paradoxic aciduria soon develops as H+ is lost in the
urine, even as systemic metabolic alkalosis is developing. An electrocardiogram may
show the typical peaked T-waves of hypokalemia. Starvation leads to hypopro-
teinemia and potential vitamin K deficiency. Therefore, nutritional status and coagu-
lation factors need to be assessed.

Treatment The patient should be treated with naso-gastric suction to prevent


aspiration. Decompression of the stomach should also be started to restore gastric
muscle tone. Acid secretion can be suppressed by parenteral administration of H2-
receptor antagonists or proton-pump blockers. This is an important step that helps to
rapidly correct metabolic alkalosis. The extracellular space volume is severely con-
tracted and must be replenished by administration of normal saline (not lactated
Ringeris, which contains fewer chloride ions).A Foley catheter and central venous
pressure monitor may also be necessary.

Operative Management The following three considerations are paramount in


the timing of surgery for gastric outlet obstruction:

1. Correction of fluid and electrolyte imbalance and anemia;

2. Improvement of nutritional status by hyperalimenation for 7 to 10 days monitored


by measurement of serum albumin, ferritin, and so on;

3. Improvement of gastric tone by continuous naso-gastric suction for 7 to 10 days in


the totally atonic stomach. The clinical impression is that restoration of gastric tone
preoperatively lessens the incidence of prolonged failure of gastric emptying follow-
ing surgery.

The critical intraoperative step is to examine the duodenum for evidence of any
inflammatory mass (often indicating localized perforation) or advanced scarring that
would make closure of the duodenum unsafe. If inflammation or excessive scarring is
present, the pyloroduodenum should be left undisturbed, and truncal vagotomy and
gastrojejunostomy should be performed. If, on the other hand, pyloroplasty can be
performed safely, then truncal vagotomy and pyloroplasty are the treatment of choice.
In either case, a feeding jejunostomy and tube gastrostomy should be constructed in
anticipation of a protracted recovery period to restore adequate gastric emptying.
Some surgeons believe that vagotomy and antrectomy (V & A) is a superior option,
but there are no good data to support the contention.

Currently, gastric outlet obstruction is the least common indication for opera-
tion in peptic ulcer disease. Acute ulcers associated with obstruction due to edema
and/or motor dysfunction may respond to intensive antisecretory therapy and naso-
gastric suction. But most patients with significant obstruction from chronic ulceration
will require some sort of more substantial intervention.

Endoscopic balloon dilation can often transiently improve obstructive symp-


toms, but many of these patients ultimately fail and come to operation. Even in pa-
tients who have a successful initial dilation, at least half will require subsequent sur-
gery, usually for recurrent obstruction, although interval bleeding and perforation is
also a possibility. Some investigators have speculated that effective treatment of H.
pylori in patients with obstructing peptic ulcer disease will improve the results of bal-
loon dilation and decrease the ulcer recurrence rate. But again, a significant percent-
age of patients with ulcer-related gastric outlet obstruction do not have demonstrable
H. pylori infection. The most common operations for obstructing peptic ulcer disease
are V+A and V+D. HSV and drainage is comparable to vagotomy and antrectomy in
this setting. HSV and gastrojejunostomy is an appealing operation for obstruction,
both because it can be done as a laparoscopically assisted procedure, and because it
does not complicate future resection should this be needed. All gastric ulcers associ-
ated with obstruction should be adequately biopsied if not resected.
GASTRIC MALIGNANCY

Premalignant Conditions

Information regarding premalignant conditions and other factors of gastric ma-


lignancy is summarized in Table 4.

Helicobacter pylori. Patients with H. pylori infection have a six- to nine-fold


increased risk of gastric cancer. The pathogenesis is thought to proceed from gastritis
to dysplasia to cancer. The incidence of mucosa-associated lymphoid malignancy is
higher than adenocarcinoma.

Atrophic Gastritis and Pernicious Anemia. The risk of developing adenocar-


cinoma is increased nearly six-fold in patients with atrophic gastritis and pernicious
anemia. In a longitudinal prospective study,1 in 80 patients with pernicious anemia
developed cancer. The achlorhydria that accompanies this condition favors bacterial
proliferation, which generates carcinogenic nitrosamines from nitrates in food.

TABLE 4. Essentials: Gastric Malignancy


Gastric Polyps. Adenomatous polyps, which represent about 10% of all gastric
polyps, are significant risk for cancer. The cancer risk in small adenomatous polyps
(<2cm) is 2%, but the risk rises to 24% in polyps 2cm or larger.
Gastric Ulcer. The incidence of malignant degeneration of a benign gastric
ulcer is probably no higher than 1% to 2%.On the other hand, malignant lesions can
masquerade as benign ulcers more frequently. Ulcer’s malignization – characterized
by permanent pain unrelated to food intake, anorexia, constant weight loss, perma-
nent wasting, disappearance of localized pain, and installment of diffuse epigastric
pain.

Hypergastrinemia can be caused by gastrinoma, by prolonged achlorhydria


that occurs as a result of atrophic gastritis, and by long-term therapy with proton-
pump inhibitors. Hypergastrinemia results in hyperplasia of the ECL cells and a ten-
dency to cause carcinoid tumors. Gastric carcinoids occur more frequently in patients
with atrophic gastritis and the Zollinger-Ellison syndrome. Long-term therapy with
proton-pump inhibitor has caused carcinoid tumors in mice, but there has been no re-
ported incidence of carcinoid tumors in humans on long-term therapy.

PENETRATION

Ulcer penetration refers to penetration of the ulcer through the bowel wall
without free perforation and leakage of luminal contents into the peritoneal cavity.
Surgical series suggest that penetration occurs in 20 percent of ulcers, but only a
small proportion of penetrating ulcers become clinically evident. Penetration occurs
in descending order of frequency into the pancreas, hepatoduodenal ligament, biliary
tract, liver, greater omentum, mesocolon, colon, and vascular structures. Antral and
duodenal ulcers can penetrate into the pancreas. Penetration can also involve pyloric
or prepyloric ulcers penetrating the duodenum, eventually leading to a gastroduodenal
fistula evident as a "double" pylorus. A long-standing ulcer history is common but not
invariable in patients who develop penetration. Penetration often comes to attention
because of a change in symptoms or involvement of adjacent structures. The change
in symptom pattern may be gradual or sudden; it usually involves a loss of cyclicity
of the pain with meals, and loss of food and antacid relief. The pain typically be-
comes more intense, of longer duration, and is frequently referred to the lower thorac-
ic or upper lumbar region. The diagnosis of penetrating ulcer is suspected clinically
when an ulcer in the proper region is found. Mild hyperamylasemia can develop with
posterior penetration of either gastric or duodenal ulcer, but clinical pancreatitis is
uncommon. Penetration can be associated with a wide array of uncommon complica-
tions including perivisceral abscess (evident on CT or ultrasonography), erosion into
vascular structures leading to exsanguinating hemorrhage (aortoenteric fistula), or
erosion into the cystic artery. Rare biliary tract complications include erosion into the
biliary tree with choledochoduodenal fistula, extra hepatic obstruction, or hematobil-
ia. Fistulization into the pancreatic duct has also been reported with penetrating duo-
denal ulcer fistulae are seen with greater curvature gastric ulcers, particularly margin-
al ulcers. Typical features of this complication include pain, weight loss, and diar-
rhea; feculent vomiting is an uncommon, but diagnostic symptom. A duodenocolic
fistula can also occur. No rigorous studies are available to guide the management of
penetrating ulcers. One can assume that management should follow the intensive
measures outlined for refractory ulcers. CT or MRI is usually needed to confirm the
diagnosis. When therapy does not produce healing, surgery is required (duodenoplas-
ty with extraterritorialization of ulcer for duodenal ulcer and economy resection for
gastric ulcer).

DISEASES OF THE OPERATED STOMACH

Long-Term Sequelae of Ulcer Surgery With the exception of proximal gas-


tric vagotomy (PGV), any ulcer surgery may be associated with undesirable long-
term sequelae. These and their pathophysiologic bases are summarized in Table 5.

TABLE 5. Pathophysiologic Basis of Long-Term Complications of Ulcer Sur-


gery
The operation least likely to cause undesirable side effects, PGV, is associated
with the highest incidence of ulcer recurrence. On the other hand, the operation most
likely to cure the ulcer problem, V & A, can lead to long-term complications.

Ulcer Recurrence The causes of ulcer recurrence are incompleteness of vagot-


omy, inadequacy of gastric resection, or both. Occasionally, however, ulcer recur-
rence is due to an undiagnosed gastrinoma. Recurrence following distal gastrectomy
for gastric ulcer is uncommon.

Classification of diseases of the operated stomach


(by O.O. Shalimov and V.F. Saenko, 1987)
A. Postgastrectomy syndromes.
I. Functional disturbance.
 Dumping syndrome.
 Hypoglycemic syndrome.
 Postgastrectomy (agastric) asthenia.
 Syndrome of small stomach.
 Syndrome of afferent loop (functional origin).
 Gastroesophageal reflux.
 Alkaline reflux-gastritis.

II. Organic disturbances.


 Peptic ulcer of anastomosis.
 Gastro-colic fistula.
 Syndrome of afferent loop (mechanical variant).
 Cicatricial deformation and narrowing of anastomosis.
 Mistakes in the technique of operation.
 Postgastrectomy accompanying diseases (pancreatitis, enterocolitis, hepatitis).

III. Mixed disturbances (combination with dumping or postvagotomy diarrhea).

B. Postvagotomy syndromes.
• Relapse of ulcer.
• Diarrhea.
• Disturbance of function of esophagocardial transition.
• Disturbance of emptying of stomach.
• Dumping syndrome.
• Reflux-gastritis.
• Gallstone disease.

The incidence of recurrent ulcer after surgery for duodenal ulcer is higher than
other rates of recurrence and depends on the type of operative procedure used to treat
the primary ulcer (Table 6).
TABLE 6. Essentials: Recurrent Ulcer Following Surgical Therapy

Following PGV and truncal vagotomy and pyloroplasty (V & P),the site of ul-
cer recurrence is usually the duodenum, although it may also be the stomach. Follow-
ing truncal vagotomy and gastrojejunostomy (V & GJ), ulcer recurrence is nearly al-
ways in the jejunum, next to the stoma; hence, the names stomal and marginal ulcers.
The evaluation of ulcer recurrence includes endoscopy, measurement of plasma gas-
trin levels, assessment of completeness of vagotomy, and tests for presence of H. py-
lori.
Diagnosis with Upper Gastrointestinal Endoscopy. Barium meal studies are
usually not helpful in the diagnosis of recurrent ulcer. Upper GI endoscopy is the only
reliable method of diagnosis.
Diagnosis with Plasma Gastrin Levels. Following all types of vagotomy not as-
sociated with antral resection, both basal and postprandial hypergastrinemia develop.
Thus, elevated plasma gastrin levels following vagotomy must be interpreted with
caution. If there is concern that a gastrinoma may be present, a “secretin test” should
be performed to detect a paradoxical rise in plasma gastrin level following intrave-
nous injection of secretin. If hypergastrinemia is demonstrated in a patient who has
had antrectomy, either a gastrinoma or retained antrum syndrome is present. The lat-
ter syndrome develops after Bilroth II gastrectomy, when antral tissue is left in conti-
nuity with the duodenum. Chronic exposure of this tissue to alkaline secretion leads
to G-cell hyperplasia and hypergastrinemia. Again, the secretin test is needed to rule
out the diagnosis of gastrinoma.
Assessment of Completeness of Vagotomy Although a high basal acid output
(>5mEq/h) is suggestive of an incomplete vagotomy, the sham feeding (“chew and
spit”) test is more definitive. It evaluates the cephalic phase of acid secretion.
Diagnosis with Helicobacter pylori Testing Evaluation for the presence of H.
pylori may be accomplished with endoscopic biopsy, breath test, or serology. If H.
pylori infection is present, eradication therapy is needed.
Treatment If infection with the bacterium H. pylori is diagnosed, eradication
therapy should be started with the objective of effecting a permanent cure for the ul-
cer. Combination therapy with amoxicillin or with tetracycline, metronidazole and
omeprazole is effective. A histamine H2-receptor antagonist or proton-pump inhibitor
provides symptomatic relief but is unlikely to cure the recurrence of the ulcer. The
type of operation needed if medical therapy fails depends on the primary operation
that was performed.
Postvagotomy diarrhea The incidence of incapacitating diarrhea following
truncal vagotomy is 1% to 2%. The cause is unknown. Symptomatic treatment in-
cludes avoidance of certain foods and the use of bulk-forming agents (Kaopectate),
codeine, and Lomotil. Postvagotomy diarrhea has no satisfactory treatment and is best
avoided by performing PGV rather than truncal vagotomy as the primary procedure
of choice. Surgical therapy for postvagotomy diarrhea is a last resort. If pyloroplasty
was previously performed, pyloric sphincter reconstruction, which reverses the pylo-
roplasty, has had some success. If a gastrojejunostomy was performed, it can be taken
down. The most controversial procedure is the interposition of a 6-inch segment of
reversed jejunum between the stomach and duodenum or jejunum to slow intestinal
transit. The reported results are not very encouraging and the procedure is rarely, if
ever, recommended. Hence, the best form of treatment is prevention.
Dumping Syndrome Rapid entry of hyperosmolar chyme into the intestine as
a result of destruction, resection, or bypass of the pyloric sphincter is the main cause
of this side effect. Vagotomy, which interferes with gastric accommodation, contrib-
utes to rapid gastric emptying. Within 15 to 30 minutes of a meal, the patient experi-
ences epigastric distress, sweating, flushing, and profound fatigue. Exaggerated bow-
el sounds (borborygmi) and sudden diarrhea may also be experienced. As described
earlier, the underlying cause of the syndrome is the combination of fluid shift into the
intestine, which causes hypovolemia, and the release of vasoactive substances from
the intestine.
The best way to prevent dumping syndrome is to avoid, whenever possible,
performing operations that are likely to cause it, including gastrectomy and truncal
vagotomy and drainage. Pharmacologic and bacteriologic advances have nearly elim-
inated the need for elective ulcer surgery. In an emergency situation, the surgeon
must decide whether to perform the quickest and safest operation at that moment as
opposed to a lengthier operation with less undesirable side effects. Whenever the
condition of the patient allows, particularly in young patients and women, PGV is a
better choice than truncal vagotomy and drainage. In the setting of hemorrhage, con-
trol of bleeding is accomplished through duodenotomy, leaving the pyloric sphincter
intact. When perforation is the indication for emergent surgery, PGV is again pre-
ferred if an acid-reducing procedure is to be done.
Nonsurgical Management Dietary measures often effectively control dumping
syndrome. These include avoiding a high carbohydrate diet; eating small, frequent
meals; not ingesting fluids with the meals; and lying down for about 60 minutes after
eating. Patients with severe symptoms have been successfully treated with the long-
acting somatostatin analogue octreotide. The problem with this form of treatment is
cost and the long-term need for injection therapy.
Surgical Management As always, surgical treatment for the dumping syndrome
is a last resort. Some operative approaches are simple and have a chance to succeed.
These include pyloric sphincter reconstruction when a pyloroplasty is present, or
takedown of gastrojejunostomy when the stomach is otherwise intact. Other surgical
options are more complex and should be undertaken only in extreme cases. These in-
clude conversion of Bilroth II gastrectomy to Bilroth I, and interposition of jejunum
between the stomach and the duodenum. The latter procedures have had variable suc-
cess.
Reactive Hypoglycemia Patients may develop typical signs and symptoms of
hypoglycemia 90 to 120 minutes after a meal. In extreme cases, hypoglycemic crisis
may develop. This side effect used to be called the “late dumping syndrome.”The
pathophysiologic basis appears to be rapid absorption of glucose from the intestine,
which leads to excessive secretion of insulin due to release of the insulinotropic pep-
tide glucagon-like immunoactivity (GLI),which outlasts the hyperglycemic stimulus.
Avoidance of carbohydrates in the diet is helpful. The long-acting somatostatin ana-
logue octreotide is effective in controlling severe symptoms.
Gastroparesis A small percentage of patients may develop gastroparesis fol-
lowing vagotomy and/or gastric resection. The cause is unknown. Symptomatic ther-
apy with prokinetic agents (e.g.,dopamine antagonists,cisapride) may be helpful.
Some patients require repeated gastric resections, eventually necessitating total gas-
trectomy with Roux-en-Y esophagojejunostomy. In some patients with disabling
symptoms, total gastrectomy is the only definitive and successful treatment.
Bile Gastritis Regurgitation of bile into the stomach invariably occurs when
the pylorus is destroyed, resected, or bypassed. Some patients develop epigastric pain
and bilious vomiting presumably due to the resultant gastritis. Medical therapy in-
cludes bile salt antagonists and prokinetic agents. Bile reflux can be prevented or
minimized by inserting a 60-cm Roux-en-Y jejunal limb between the stomach and
upper jejunum. Unfortunately, the early encouraging results of this operation have not
been sustained over time.
Chronic Anemia Iron-deficiency anemia commonly occurs several years after
gastrectomy, but it can also develop following truncal vagotomy. The causes may in-
clude chronic occult blood loss from gastritis and poor absorption of dietary iron.
Megaloblastic anemia, due to vitamin B12 deficiency, may be seen after radical gas-
trectomy, indicating insufficient secretion of intrinsic factor. It can be successfully
treated with monthly vitamin B12 administration parenterally.
Malabsorption Postgastrectomy patients often undergo weight loss and some-
times show signs of malabsorption of fat, carbohydrates, vitamins, and metals. Mild
steatorrhea tends to occur after Bilroth II gastrectomy. Vitamin deficiencies may be
related to blind-loop syndrome. Lactose intolerance is unmasked in patients who have
a mild preoperative lactase deficiency. A significant long-term complication of gas-
tric surgery is calcium malabsorption, which over years may lead to osteoporosis,
particularly in women.
Postgastrectomy carcinoma A higher incidence of carcinoma of the stomach
is seen in patients who had gastrectomy 20 years or more previously. The cause is
unknown but may be related to hypoacidity favoring bacterial overgrowth and a gen-
eration of carcinogenic nitrosamines from food.
Basic literature:
39. Oxford Textbook of Surgery (3-Volume Set) 2nd edition (January 15,
2000): by Peter J. Morris (Editor), William C. Wood (Editor) By Oxford Press
40. Sabiston Textbook of Surgery 17th edition by Courtney M. Townsend
Jr., Kenneth L. Mattox, B. Mark, MD Evers, Kenneth L., MD Mattox, Courtney
Townsend, Daniel Beauchamp, B. Mark Evers, Kenneth Mattox W.B. Saunders
Company (June, 2004)
41. Schwartz´s Principles of Surgery 8th Edition F. Charles Brunicardi. Cop-
yright ©2007 the McGraw-Hill Companies.
42. Hospital surgery/ Edited by L. Kovalchuk et al. - Ternopil: Ukrmedkn-
yha, 2004. - 472 p.
Additional literature:
1. Allison M.C. et al. Gastrointestinal damage associated with the use of
nonsteroidal antiinflammatory drugs// N Engl J Med, 1992; 327:749-754.
2. Graham D.Y. NSAIDs,Helicobacter pylori,and Pandorais Box// N Engl
J Med, 2002;347:2162-2164.
3. Suerbaum S., Michetti P. Helicobacter pylori infection// N Engl J Med,
2002;347:1175-1186.
4. Dempsey D.T.: Reoperative gastric surgery and postgastrectomy syn-
dromes, in Zuidema GD, Yeo CJ (eds)// Shackelford's Surgery of the Alimentary
Tract, 5th ed., Vol. II. Philadelphia: Saunders, 2002, p. 161.
Tests for initial level of knowledge, keys for tests:

1. A 30-year-old man with a duodenal ulcer is being considered for surgery. His se-
rum gastrin level, however, is 150+10 pg/mL on three determinations. The surgeon
should perform
A. An arteriogram
B. A secretin stimulation test
C. A total gastrectomy
D. A subtotal gastrectomy
E. A highly selective vagotomy

2. Which of the following statements regarding stress ulceration is true?


A. It is true ulceration, extending into and through the muscularis mucosa
B. It classically involves the antrum
C. Increased secretion of gastric acid has been shown to play a causative role
D. It frequently involves multiple sites
E. It is seen following shock or sepsis, but for some unknown reason does not occur
following major surgery, trauma, or burns

3. A 72-year-old patient with an intractable type I ulcer along the incisura with a sig-
nificant amount of scarring along the entire length of the lesser curvature. Select the
appropriate surgical procedure for each patient.
A. Vagotomy and antrectomy
B. Antrectomy alone
C. Vagotomy and pyloroplasty
D. Vagotomy and gastrojejunostomy
E. Proximal gastric vagotomy

4. Which vessel is most commonly associated with a posterior duodenal ulcer?


A. Right gastroepiploic artery
B. Common hepatic artery
C. Gastroduodenal artery
D. Superior mesenteric artery
E. Middle colic artery

5. The lower esophageal sphincter pressure is increased by


A. Glucagon
B. Gastrin
C. Emptying of the stomach
D. Chocolate
E. Acid in the stomach

6. Gastroesophageal reflux is best characterized by which of the following state-


ments?
A. It is synonymous with hiatal hernia
B. It results from a higher than normal lower esophageal sphincter pressure
C. It may be associated with increased gastrin production
D. It is diagnosed by manometry and 24-hour monitoring of pH in the lower
esophagus
E. It is a relative contraindication to esophagoscopy
7. The typical carcinoma that develops in association with Barrett’s esophagus is
A. Epidermoid
B. Mucoepidermoid
C. Small cell
D. Adenocarcinoma
E. Squamous cell

8. The blood supply to the stomach and duodenum arises from all of the following ar-
teries EXCEPT
A. Gastroepiploic artery
B. Common hepatic artery
C. Splenic artery
D. Superior mesenteric artery
E. Inferior mesenteric artery

9. What substance is secreted by the C cells?


A. Gastrin
B. Pepsin
C. Pepsinogen
D. Gastric acid
E. Glucagon

10. Gastric acid production is altered by all of the following hormones or actions
EXCEPT
A. Cholecystokinin
B. Gastrin
C. Vagal stimulation
D. Secretin
E. Glucagon

Keys for tests


1 2 3 4 5 6 7 8 9 10
B D B C B D D E A E
Tests for final level of knowledge, keys for tests:

1. Omeprazole has been added to the H2-antagonists as a therapeutic approach to the


management of acute gastric and duodenal ulcers. It acts by
A. Blocking breakdown of mucosal-damaging metabolites of NSAIDs
B. Providing a direct cytoprotective effect
C. Buffering gastric acids
D. Inhibiting parietal cell hydrogen-potassium-ATPase
E. Inhibiting gastrin release and parietal cell acid production

2. A 45-year-old woman is explored for a perforated duodenal ulcer 6 h after onset of


symptoms. She has a history of chronic peptic ulcer disease treated medically with
minimal symptoms. The procedure of choice is
A. Simple closure with omental patch
B. Truncal vagotomy and pyloroplasty
C. Antrectomy and truncal vagotomy
D. Highly selective vagotomy
E. Hemigastrectomy

3. Six weeks after surgery, the patient returns complaining of postprandial weakness,
sweating, light-headedness, crampy abdominal pain, and diarrhea. The best manage-
ment would be
A. Antispasmodic medications (e.g., Lomotil)
B. Dietary advice and counseling that symptoms will probably abate within 3 mo of
surgery
C. Dietary advice and counseling that symptoms will probably not abate but are not
dangerous
D. Workup for neuroendocrine tumor (e.g., carcinoid)
E. Preparation for revision to Roux-en-Y gastrojejunostomy

4. A 55-year-old man complains of chronic intermittent epigastric pain, and gastros-


copy demonstrates a 2-cm ulcer of the distal lesser curvature. Endoscopic biopsy
yields no malignant tissue. After a 6-wk trial of H2-blockade and antacid therapy, the
ulcer is unchanged. Proper therapy at this point is
A. Repeat trial of medical therapy
B. Local excision of the ulcer
C. Billroth I partial gastrectomy
D. Billroth I partial gastrectomy with vagotomy
E. Vagotomy and pyloroplasty

5. A 42-year-old man with no history of use of nonsteroidal antiinflammatory drugs


(NSAIDs) presents with recurrent gastritis. Infection with Helicobacter pylori is sus-
pected. Which of the following statements is true?
A. Morphologically, the bacteria is a gram-positive, tennis-racket-shaped organism
B. Diagnosis can be made by serologic testing or urea breath tests
C. Diagnosis is most routinely achieved via culturing endoscopic scrapings
D. The most effective way to treat and prevent recurrence of this patients gastritis is
through the use of single-drug therapy aimed at eradicating H. pylori
E. The organism is easily eradicated
6. Intragastric pressure remains steady near 25 mm Hg during slow gastric filling, but
rises rapidly to high levels after reaching a volume of
A. 400-600 mL
B. 700-900 mL
C. 1000-1200 mL
D. 1300-1500 mL
E. 1600-1800 mL

7. Local stimuli that inhibit the release of gastrin from the gastric mucosa include
which of the following?
A. Small proteins
B. 20-proof alcohol
C. Caffeine
D. Acidic antral contents
E. Antral distention

8. A 30-year-old man with a duodenal ulcer is being considered for surgery because
of intractable pain and a previous bleeding episode. Serum gastrin levels are found to
be over 1000 pg/mL (normal 40-150) on three separate determinations. The patient
should be told that the operation of choice is
A. Vagotomy and pyloroplasty
B. Highly selective vagotomy and tumor resection
C.Subtotal gastrectomy
D.Total gastrectomy
E.Partial pancreatectomy

9. A 55-year-old man who is extremely obese reports weakness, sweating, tachycar-


dia, confusion, and headache whenever he fasts for more than a few hours. He has
prompt relief of symptoms when he eats. These symptoms are most suggestive of
which of the following disorders?
A. Diabetes mellitus
B. Insulinoma
C. Zollinger-Ellison syndrome
D. Carcinoid syndrome
E. Multiple endocrine neoplasia, type II

10. Which of the following organisms is most closely associated with gastric and du-
odenal ulcer disease?
A. Campylobacter
B. Cytomegalovirus
C. Helicobacter
D. Mycobacterium avium-intracellulare
E. Yersinia enterocolitica

Keys for tests


1 2 3 4 5 6 7 8 9 10
D C B C B C D B B C

Tasks for final level of knowledge

1. Eight days after a difficult hemigastrectomy and gastroduodenostomy for


gastric ulcer, a patient begins to leak 2-3 L of green fluid per day through the
right corner of his bilateral subcostal abdominal wound.

What is the management? If he is febrile, with an acute abdomen, and sick, he needs
to be explored. But if all the gastric and duodenal contents are leaking to the outside,
further surgery right away is not the answer. The problem is serious, though. Massive
fluid and electrolyte replacement will be needed, and nutritional support will have to
be provided with elemental nutrients delivered into the upper jejunum (total parenteral
nutrition is a poor second choice), hoping for eventual healing without having to oper-
ate again.

2. A patient who has had a subtotal gastrectomy for cancer, with a Billroth 2 recon-
struction, develops a "blowout" of the duodenal stump, and a subsequent duodenal
fistula. For the past 10 days he has been draining 750 to 1,500 ml/d of green fluid.
His serum electrolytes show a sodium of 132, a chloride of 104, and a bicarbonate of
15. The pH in his blood is 7.2, with a Pco2 of 35.

What is it? Again, metabolic acidosis, but now with a normal anion gap. He has been
losing lots of bicarbonate out of the fistula. The problem would not have developed if
his IV fluid replacement had contained lots of bicarbonate (or lactate, or acetate), but
the use of those agents is indicated now for the therapy of the existing abnormality.

3. A patient with severe peptic ulcer disease develops pyloric obstruction and has
protracted vomiting of clear gastric contents (i.e., without bile) for several days.
His serum electrolytes show sodium of 134, chloride 82, potassium 2.9, and
bicarbonate 34.

What is it? The classic hypochloremic, hypokalemic, metabolic alkalosis secondary


to loss of acid gastric juice. This man needs to be rehydrated (choose saline rather
than Ringer lactate), and he needs lots of potassium chloride (10 mEq/h will give him
plenty, and will be a safe rate). Very rarely is ammonium chloride (or diluted, buff-
ered hydrochloric acid) needed.
4. A 62-year-old man describes epigastric and substernal pain that he cannot
characterize well. At times his description sounds like gastroesophageal reflux,
at times it does not. Sonogram of the gallbladder, ECG, and cardiac enzymes
have been negative.

What is it? It is gastroesophageal reflux

5. A 72-year-old man has lost 40 pounds of weight over a 2- or 3-month period. He


gives a history of anorexia for several months, and of vague epigastric
discomfort for the past 3 weeks.

What is it? Cancer of the stomach.

Materials for the self-study of the students

Main tasks Notes(instructions)


Repeat: -To represent the methods of diagnos-
1. Topography and stomach tics of diseases of digestive tract
innervations -To make the flow diagram of gastric
2. Histological structure of stomach malignancy
3. Physiological functions of stomach
4.
Study: -To conduct differential diagnosis with
1. Types of duodenoplasty. the decompensated stenosis and cancer
2. Types of gastric resection. of gastric outlet
3. H. pylori and ulcer -To conduct differential diagnosis of
4. Pathogenesis of gastroesophageal pathologies of the stomach
reflux disease (GERD)
Study guide #9
“Bleeding from the digestive tract: causes of origin, diagnosis and differential
diagnosis, tactics of treatment.”

Overview.

Hemorrhage from the gastrointestinal tract is a common and serious clinical


problem. In the United States, 1% to 2% of acute hospital admissions are for patients
requiring evaluation and treatment of gastrointestinal hemorrhage. With an incidence
of 170 per 100,000 adults per year, gastrointestinal hemorrhage is a leading diagnosis
in patients admitted to intensive care units (ICUs). Although the overall mortality rate
for these patients ranges from 5% to 12%, the mortality rate in patients with persistent
or recurring hemorrhage approaches 40%. Mortality is linked not only to the degree
of hemorrhage but also, more importantly, to the coexisting medical conditions in the
patient with hemorrhage. Up to 85% of bleeding episodes cease spontaneously, al-
lowing a less urgent approach to identify the source of bleeding and to provide defini-
tive therapy; however, 15% of patients present with major, ongoing bleeding that re-
quire aggressive emergency diagnosis and management to allow successful clinical
outcomes. These high-risk patients are most likely to require surgical intervention and
to have poor outcomes.

Hemorrhage can arise in any area of the gastrointestinal tract: the esophagus,
stomach and duodenum, small bowel, and colon as well as organs that empty secre-
tions into the gastrointestinal tract, such as the liver through the biliary system and the
pancreas through the pancreatic duct. Although the spectrum of conditions giving rise
to acute hemorrhage varies, more than 85% of major bleeding episodes can be linked
to one of four diagnoses: peptic ulcer disease, variceal hemorrhage, colonic diverticu-
losis, or angiodysplasia. Other sources of hemorrhage are distinctly less common.
Gastrointestinal hemorrhage spans the socioeconomic strata and is equally common
in urban and rural environments. Only advancing age appears to be a risk factor for
hemorrhage that applies across the full spectrum of bleeding conditions of the intesti-
nal tract. Up to half of patients with acute gastrointestinal hemorrhage are older than
60 years of age.
Numerous advances in medical technology during the 1990s, particularly the
improved availability and application of diagnostic and therapeutic endoscopy, have
been instrumental in the evaluation and successful treatment of patients with major
bleeding. Although surgery is required for control of hemorrhage in only 5% to 10%
of patients hospitalized with gastrointestinal hemorrhage, it remains an essential
emergency intervention for those patients with severe or recurrent hemorrhage from
both the upper and lower gastrointestinal tract. Successful collaboration between the
surgeon and the gastroenterologist is essential for optimal management of these com-
plicated patients. In all patients, regardless of bleeding source, successful initial man-
agement requires that the treating physician be mindful of the potential severity of
gastrointestinal hemorrhage. Appropriate resuscitation to restore volume and red
blood cell deficits is critical in patients with major hemorrhage. This resuscitation
phase must be followed by rapid diagnosis of the source of bleeding. Subsequently,
establishment of appropriate specific therapies may be done to achieve successful
management.

Educational aims:
8. Interrogation and clinical inspection of patients with GI bleeding.
9. To determine the etiologic and pathogenic factors of GI bleeding.
10.To know the complications of peptic ulcer.
11.To develop a plan of examination of the patients with GI bleeding.
12.To estimate results of palpation, percussion of abdomen, X-ray, endosco-
py, laboratory data.
13.To make a differential diagnosis, substantiate and formulate a diagnosis
of GI bleeding.
14.To determine the indications for treatment of patients with GI bleeding.
15.To estimate efficiency of treatment and prognosis of disease.
A student must know:
25. Anatomo-physiological information about a stomach and duodenum, intestine
and colon.
26. Etiology and pathogenesis of GI bleeding.
27. Manifestations of GI bleeding.
28. Modern methods of instrumental diagnostics of GI bleeding.
29. Differential diagnostics of GI bleeding with other internal bleeding.
30. Basic principles of conservative treatment of GI bleeding.
31. Modern methods of surgical treatment of GI bleeding.
32. Question of prophylaxis of postoperative complications, rehabilitation and
health centre system of patients.

A student must be able to:


7. Collect and estimate anamnesis of the patients with GI bleeding.
8. Use methods of examination patients with GI bleeding.
9. Set the algorithm of examination patient.
10.Estimate the results of laboratory, instrumental inspection of patients.
11.Determine optimum medical tactics for a specific patient (conservative, surgi-
cal), and in the case of GI bleeding
12.To prove necessity of pre-operation preparation of patient and to define the op-
eration of choice depending on localization, age, sex.
Terminology.
Term Definition
Are due to bleeding in the gastrointestinal tract cavity as
Bleeding gastrodu-
a result of increasing necrosis in the ulcer area with
odenal ulcers
blood vessels, with the subsequent damage of their walls.
Upper gastrointestinal bleeding secondary to longitudinal
Mallory-Weiss
mucosal lacerations at the gastroesophageal junction or
syndrome
gastric cardia.
Dieulafoy's lesion is a medical condition characterized by a large tortuous
(exulceratio simple) arteriole in the stomach wall that erodes and bleeds
vomiting of blood or bloody gastric contents (“coffee
Hematemesis
grounds”)
Melena passage of dark tarry or maroon stool
Hematochezia passage of bright red blood from the rectum

Content:

Upper gastrointestinal hemorrhage is present in 85% of patients with acute gas-


trointestinal bleeding; lower gastrointestinal bleeding occurs in 10% to 15% of pa-
tients, with the small bowel as the source in only 1% to 5% of patients. Hemorrhage
from small bowel sources may be difficult to diagnose and is frequently referred to as
hemorrhage of obscure origin. The introduction of video capsule endoscopy 3 years
ago has enhanced the diagnosis and treatment of the uncommon small bowel lesion.

INITIAL EVALUATION AND TREATMENT OF PATIENTS WITH ACUTE


GASTROINTESTINAL HEMORRHAGE

Initial management of a patient with acute gastrointestinal hemorrhage has four


primary goals: (1) comprehensive patient assessment, with attention to hemodynamic
status and identification of significant medical co morbidities; (2) institution of ap-
propriate resuscitation and monitoring; (3) identification of the major source of gas-
trointestinal bleeding; and (4) institution of specific therapeutic interventions to stop
or control the bleeding. When the level of severity of the bleeding is clarified and ini-
tial assessment and resuscitation are complete, the patient may be triaged to the ap-
propriate level unit of care.

Goal 1: Initial Patient Assessment - Most patients with acute gastrointestinal


hemorrhage present for initial assessment in the emergency department. One quarter
of patients develops gastrointestinal hemorrhage during hospitalization for a concur-
rent illness; this group is particularly high risk for subsequent mortality. Initial as-
sessment in either case calls for a focused history and physical examination, with at-
tention to risk factors for gastrointestinal hemorrhage and laboratory evaluation.

History Except for patients in hemorrhagic shock, revealing information can be


obtained from the patient’s history. The essential elements to be ascertained are the
characteristics of the bleeding; the onset and duration of bleeding (hours or days ante-
cedent); the associated symptoms; the use of concurrent medications; and previous
significant medical conditions, particularly liver diseases.

Characteristics of Bleeding Acute gastrointestinal hemorrhage can present


with hematemesis (vomiting of blood or bloody gastric contents), melena (passage of
dark tarry or maroon stool), or hematochezia (passage of bright red blood from the
rectum). On initial evaluation of a patient with acute gastrointestinal hemorrhage, it is
important to determine whether the patient has experienced hematemesis, melena, or
hematochezia. Gastrointestinal bleeding that is slow or intermittent is usually not evi-
dent to the patient; hence, the term occult is associated with this pattern of blood loss.
Such patients present to primary care venues with secondary signs of slow blood loss,
such as anemia or fatigue. Hematemesis is diagnostic of upper gastrointestinal bleed-
ing, which is, bleeding from the esophagus, stomach, or duodenum. Rarely, hema-
temesis may result from brisk hemorrhage from the nasal passages or pharynx when
the patient swallows large volumes of blood. Melena can be indicative of either upper
or lower gastrointestinal hemorrhage. Dark, tarry stools are most commonly a sign of
an upper gastrointestinal source in which the blood has traversed the small bowel and
colon. Gastric acid degrades hemoglobin to hematin, and the actions of digestive en-
zymes and luminal bacteria further contribute to the appearance of melena. Melena
may also represent bleeding from lesions in the small bowel or right colon. Hemato-
chezia is the characteristic sign of colonic hemorrhage and reflects rapid elimination
of blood from the bowel. Ten percent of patients with very rapid upper gastrointesti-
nal hemorrhage may also have a history of hematochezia and syncope. It is essential
to determine the onset of bleeding and the frequency of episodes of hematemesis,
melena, or hematochezia and to make a rough estimate of the volume of blood loss.

Associated Symptoms Inquiry regarding associated symptoms is also of im-


portant value. A history of orthostatic dizziness or syncope indicates rapid and pro-
found blood loss. Antecedent dyspepsia is suggestive of peptic ulcer disease; crampy
abdominal pain is more consistent with upper gastrointestinal bleeding, whereas hem-
atochezia is usually painless. Antecedent vomiting may suggest Mallory-Weiss tears;
weight loss raises the possibility of malignancy.

Medications The risk for gastrointestinal ulceration and hemorrhage is elevated


in patients taking salicylates or nonsteroidal anti-inflammatory drugs (NSAIDs). Use
of these medications is linked not only to gastritis and gastric and duodenal ulcers but
also to much less commonly seen ulcerated lesions of the colon and small bowel. Fur-
ther, salicylates and NSAIDs impair platelet function and may contribute to poor co-
agulation in patients who develop the complication of hemorrhage. Use of other med-
ications that predisposes the patients to hemorrhage, such as warfarin and low-
molecular-weight heparin, should be elicited.
Past Medical History The past medical history should identify previous epi-
sodes of gastrointestinal bleeding or past history of conditions associated with acute
hemorrhage.

Physical Examination The major initial objective of the physical examination


is to determine the degree of blood loss and volume depletion. Patients in shock with
hypotension (systolic blood pressure <90 mm Hg in the supine position), tachycardia,
and cold extremities can be assumed to have a deficit of at least 40% of blood vol-
ume. Patients with less severe but substantial blood loss of 20% to 40% show hypo-
tension in the upright position. Orthostatic vital signs should be checked in all pa-
tients not in shock by allowing the patient to sit up with the legs dangling for a period
of 5 minutes. An elevation in pulse of more than 20 beats/min or a fall in blood pres-
sure of more than 10 mm Hg is a positive sign, indicative of at least a 20% blood vol-
ume loss. Signs of peripheral hypo perfusion, such as clammy, cool, pale extremities,
also reflect a volume loss of at least 20%. These signs are less reliable in elderly pa-
tients, who may show exaggerated postural changes or dull changes in heart rate, or
are more likely to be using β-blocker medication. All patients showing a volume defi-
cit of greater than 20% of blood volume require prompt and aggressive resuscitation.
The physical examination generally offers few specific signs relative to the source of
gastrointestinal hemorrhage. The oropharynx and nose should be examined to exclude
the rare unrecognized nasopharyngeal source of bleeding. Although epigastric ten-
derness may be elicited in patients with peptic ulcer conditions, this is not a reliable
sign. A rectal examination, noting the quality of the stool (i.e., brown, melena, or
hematochezia) should also be completed.

Initial Laboratory Assessment All patients with gastrointestinal hemorrhage


should have basic laboratory testing, including hemoglobin and hematocrit, coagula-
tion profile, liver function tests, serum electrolytes, and renal function. The initial
hematocrit may not reflect the actual degree of hemorrhage because intravascular
volume repletion from extracellular fluids may not have occurred. The finding of ini-
tial hemoglobin of less than 10 g/100 mL is associated with an increased risk for
morbidity and mortality.
Goal 2: Resuscitation Based on the estimated volume deficit, rapid restoration
of intravascular volume is indicated. All patients with gastrointestinal hemorrhage
should have two large-bore intravenous lines for administration of lactated Ringer’s
solution. Patients in shock should receive prompt transfusion of packed red blood
cells if immediate response to electrolyte solutions is not evident. Patients with major
hemorrhage, elderly patients, and patients with significant co morbidities (including
cardiac, pulmonary, hepatic, or renal insufficiency) should be monitored with central
venous or pulmonary artery catheters. Urine output should be monitored with a Foley
catheter. Ongoing hemorrhage requires continuous resuscitation with saline and red
blood cell transfusion. Coagulation defects should be corrected with component ther-
apy or fresh frozen plasma and platelets. Hemodynamically unstable patients should
have endotracheal intubation performed to protect the airways.

Goal 3: Identification of Source of Bleeding Successful management of a pa-


tient with acute gastrointestinal hemorrhage requires knowledge of the site of bleed-
ing. The specific aspects of diagnostic testing are considered in the detailed sections
that follow. The general considerations are reviewed here. Patients with hematemesis,
melena, or hematochezia require emergency upper endoscopy by an endoscopist ca-
pable of therapeutic intervention. Prior to this examination, a large-caliber orogastric
tube should be placed to lavage the gastric lumen to enhance visual examination.
Airways protection may require endotracheal intubation. Active volume resuscitation
must continue during the examination. If patients are hemodynamically stable and
show no signs of ongoing hemorrhage, endoscopic examination may be deferred to an
urgent status (within 12 hours) provided the patient can be carefully observed in the
meantime. Patients presenting with melena and hematochezia without a history of
hematemesis should have a nasogastric tube inserted to examine the gastric contents.
Findings of blood-tinged secretions, “coffee grounds,” or guaiac-positive fluid should
prompt upper endoscopy. Patients with melena and hematochezia with hemodynamic
instability should have initial emergency upper endoscopy. Bleeding peptic lesions in
the duodenum can elicit pyloric spasm precluding reflux of sufficient amounts of
blood into the gastric lake to cause hematemesis. Endoscopy is essential to examine
the duodenum in these patients. Even in stable patients, this examination should be
performed within 24 hours of the bleeding episode to optimize outcome. Hemody-
namically stable patients with hematochezia and patients with melena with a negative
upper gastrointestinal examination may be presumed to have acute lower gastrointes-
tinal hemorrhage. For these patients, the choice of initial diagnostic test remains con-
troversial. Mesenteric arteriography, colonoscopy, and labeled red blood cell scintig-
raphy are potentially valuable based on the clinical presentation. Diagnostic ap-
proaches are considered later in the section on lower gastrointestinal hemorrhage.

Goal 4: Institution of Specific Therapy After resuscitation and identification


of the source of bleeding, specific therapy can be instituted. For the 15% of patients
with ongoing gastrointestinal hemorrhage and hemodynamic instability, the time in-
terval until this intervention should be less than 2 hours, and all measures to provide
ongoing support to avoid shock should be employed during the interval. Fortunately,
bleeding stops spontaneously in most patients, allowing a more deliberate evaluation.
After the source of bleeding has been identified, specific intervention can be provid-
ed.

ACUTE UPPER GASTROINTESTINAL HEMORRHAGE

Upper gastrointestinal bleeding is defined as bleeding from a source proximal


to the ligament of Treitz. Acute upper gastrointestinal hemorrhage is a common and
potentially deadly condition accounting for approximately 85% of hospital admis-
sions for gastrointestinal bleeding. Despite the availability of effective antiulcer med-
ications and an improved understanding of the pathogenesis of ulcer disease, gastro-
duodenal ulcer disease remains the most common cause, responsible for half of bleed-
ing episodes. In urban environments, hemorrhage from esophageal and gastric varices
secondary to portal hypertension of alcoholic cirrhosis constitutes the next most fre-
quent source, identified in 10% to 20% of patients. Acute mucosal lesions, broadly
characterized as gastritis or duodenitis, are observed in 15% to 30% of patients with
hemorrhage in both urban and nonurban settings. Other causes have remained rela-
tively stable in frequency since the early 1970s, including Mallory-Weiss mucosal
tears at the gastroesophageal junction (8% to 10%), esophagitis (3% to 5%), malig-
nancy (3%), Dieulafoy’s lesion (1% to 3%), and more recently, “watermelon” stom-
ach (1% to 2%). A differential diagnosis for acute upper gastrointestinal hemorrhage
is shown in Table 1.
Table 1. A differential diagnosis for acute upper gastrointestinal hemorrhage

Clinical Presentations Hematemesis and melena are the most frequent clinical
findings in significant upper gastrointestinal bleeding. However, massive bleeding
from an upper source may be associated with hematochezia. Even in instances where
a lower gastrointestinal bleeding source is suspected, the passage of a nasogastric
tube is required to interrogate for the presence of blood in the stomach. Although all
sources of gastrointestinal bleeding have high associated morbidities, upper gastroin-
testinal bleeding has the highest risk for life-threatening hemorrhage.

BLEEDING PEPTIC ULCER


Peptic gastric and duodenal ulcers are the most common cause of acute hemor-
rhage in the upper gastrointestinal tract, each accounting for about 25% of cases.
Bleeding peptic ulcers account for about half of the clinically significant cases of up-
per GI bleeding at most medical centers. About 5% of patients with peptic ulcer dis-
ease have hemorrhage as the initial manifestation of the condition, and up to 20% of
patients with peptic ulcers develop bleeding at least once. Hemorrhage remains the
most lethal form of complicated ulcer disease; 80% of ulcer deaths in the elderly oc-
cur as a consequence of an episode of acute hemorrhage.

The actual appearance of the ulcer at endoscopy is the most important predictor
of rebleeding. Ulcers generally have one of five appearances: a clean ulcer base; a
flat, pigmented spot, which may be purple, brown, or black, on the ulcer surface; an
adherent clot; a visible vessel, which appears as a smooth surfaced or tubular protu-
berance on the smooth ulcer surface; or active bleeding with either spurting blood,
continuous oozing, or oozing around an adherent clot. Descriptive identification of
the ulcer characteristics has also been reported as the Forrest classification system,
where FI ulcers show active bleeding, FIIa represents an ulcer with a visible vessel or
pigmented protuberance, FIIb represents an ulcer with an adherent clot, FIIc repre-
sents an ulcer with a pigmented spot, and FIII shows a clean ulcer base without stig-
mata of bleeding. Rebleeding rates increase with ulcer size; ulcers greater than 2 cm
in diameter are high risk. As discussed later, endoscopic therapy is appropriate for ul-
cers with stigmata of bleeding. In contrast, active bleeding which is not controlled
with endoscopic measures mandates immediate surgical intervention. The transendo-
scopic Doppler device has been evaluated to assess blood flow beneath the ulcer sur-
face. A positive Doppler study indicating a blood vessel beneath the ulcer was a
strong predictor of rebleeding, although the value of this method of evaluation to pre-
dict rebleeding is yet to be demonstrated in a large cohort.

Therapeutic Interventions

Medical Management. Therapy is based on clinical presentation and endoscop-


ic findings. A patient with minimal bleeding and a clean ulcer base on endoscopy is at
very low risk for recurrent hemorrhage. Young patients may be discharged with spe-
cific antiulcer therapy: an antisecretory agent (either an H2 -receptor antagonist or
proton-pump inhibitor), cessation of NSAIDs if applicable, and H. pylori eradication
with antibiotics if H. pylori positive. Older patients with this clinical presentation
should be admitted for a brief period of in-hospital observation before discharge on a
similar regimen. Follow-up endoscopy is indicated at 6 weeks for patients with gas-
tric ulcer to ensure healing and to exclude malignancy but not for patients with duo-
denal ulcer. Patients with more significant hemorrhage and findings on endoscopy
with stigmata for lesions at increased risk for rebleeding should be admitted to the
hospital. Patients with clinical risk factors for adverse outcome should be admitted to
the ICU.

Endoscopic Therapy. Endoscopic therapy can be used to arrest active ulcer


bleeding and to prevent rebleeding in patients with ulcers at high risk for rebleeding
(FI, FIIa, and FIIb ulcers). Several endoscopic devices can deliver the thermal energy
required to achieve coagulation. Transendoscopic bipolar electrocoagulation and
heater probe therapy can decrease rebleeding rates and the need for surgical interven-
tion by up to 50%. In skilled hands, light amplification by stimulated emission of ra-
diation (LASER) coagulation offers similar results, although the risk for perforation
is higher. Injection therapy is an equally effective nonthermal method to secure he-
mostasis. Available sclerosing or vasoconstriction agents include absolute alcohol,
epinephrine, fibrin glue, and polidocanol. The choice of method and agent is accord-
ing to the preference of the endoscopist and equipment availability.

Endoscopic therapy fails in about 20% of patients, manifest as either failure to


control hemorrhage on initial presentation or as early recurrent hemorrhage. In the re-
cent past, rebleeding patients were treated surgically.

Surgical Therapy. Surgery is ultimately required in roughly 10% of patients


with bleeding ulcer. Surgery is indicated for patients with active hemorrhage not re-
sponsive to endoscopic measures, significant recurrent hemorrhage after endoscopic
treatment, an ongoing transfusion requirement, or transfusion requirements exceeding
6 units of packed red blood cells in a 24-hour interval. Now, the decision for surgery
is balanced by endoscopic expertise, patient characteristics, and transfusion require-
ments.
Choice of Operation. The goal of surgical intervention in bleeding peptic ulcer
is to control hemorrhage. This may be achieved by either direct suture ligation of the
bleeding vessel or, in the case of gastric ulcer, with gastric resection or ulcer excision.
The role for a definitive acid-reducing procedure is a secondary, but important, objec-
tive of the surgical procedure.

Bleeding Duodenal Ulcer

Operative intervention for bleeding duodenal ulcer requires direct exposure of


the ulcer in the duodenum by way of duodenoplasty. Provided the patient is stable
and free of life threatening, preoperative co morbid conditions, a definitive antisecre-
tory procedure is indicated. Postoperative antisecretory and eradicated therapy is
used. In case of negative compliance the parietal cell vagotomy with duodenoplasty
has been advocated by some surgeons. Duodenoplasty is successful in acutely con-
trolling hemorrhage in 96% of patients; duodenoplasty with PCV is successful in
acutely controlling hemorrhage in 90% of patients. Up to 10% of patients may devel-
op early rebleeding. Repeat surgical intervention is rarely required for these patients,
however, because bleeding frequently ceases with supportive measures. Reported op-
erative mortality rates range from less than 1% to 50% based on the patient’s co mor-
bid conditions.

Bleeding Gastric Ulcer

Again, the primary goal of surgical intervention for bleeding gastric ulcer dis-
ease is to stop hemorrhage. Unlike duodenal ulcer, there is a chance that a gastric ul-
cer may be malignant; up to 1% of gastric ulcers prove to be a gastric adenocarcino-
ma or lymphoma. Additionally, rebleeding rates for gastric ulcer treated with simple
ligation approach 30%. Ideally, therefore, the surgical procedure should include ulcer
excision (organ-saving (economy) resection).

BLEEDING CAUSED BY PORTAL HYPERTENSION

For completeness we briefly discuss bleeding related to portal hypertension, a


frequent cause of upper gastrointestinal bleeding. Bleeding from esophagogastric var-
ices is responsible for one third of all deaths in patients with cirrhosis and portal hy-
pertension. As many as 90% of cirrhotic patients develop esophageal varices, and
25% to 30% of these develop hemorrhage.

Treatment of Acute Variceal Hemorrhage Initial management calls for


prompt resuscitation with particular attention to correction of volume deficit, coag-
ulopathy, and airways management. Treatment in an ICU is imperative. Endoscopy is
necessary both to confirm the source of bleeding and to allow endoscopic therapy.
Both variceal sclerotherapy and rubber band ligation are effective endoscopic
measures. Complications of sclerotherapy include esophageal ulceration, bleeding
perforation, mediastinitis, pleural effusion, and pulmonary edema. Late stricture has
also been observed. Gastric varices are not effectively treated by sclerotherapy. In
good-risk patients with bleeding gastric varices, prompt surgical decompression
should be considered. Concomitant treatment with vasoactive drugs is indicated. The
somatostatin given by continuous intravenous infusion, offers the best efficacy and
safety profile. Vasopressin alone and vasopressin plus nitroglycerin by continuous
systemic intravenous infusion are also of benefit in decreasing splanchnic blood flow
and decreasing variceal bleeding.

ACUTE GASTRIC MUCOSAL LESIONS

Acute gastric mucosal lesions (AGMLs) include a broad category of acute ero-
sive mucosal conditions that develop in critically ill patients. Also known as stress
gastritis, acute mucosal ischemia, erosive gastritis, or stress ulceration, these condi-
tions share a common epidemiology and clinical presentation. Lesions resembling
AGMLs are also observed in patients on chronic NSAID therapy.

Treatment: Emergency evaluation with upper endoscopy, a procedure that


should be completed in the ICU, is indicated. Careful attention to prevent aspiration
and hypovolemia during the procedure is mandatory. If a solitary site of bleeding is
identified, endoscopic therapy, such as thermal or bipolar electrocoagulation, fibrin
glue application, or injection therapy, is appropriate. Frequently, however, bleeding is
too diffuse to allow endoscopic therapy. In this case, aggressive medical management
with transfusion and component therapy to correct coagulation defects and anemia is
indicated. The role of angiography in stress ulceration is limited to diffuse, unremit-
ting hemorrhage. Selective celiac catheterization may allow identification of the
bleeding arteries of origin. Embolization with coils or collagen gel or selective intra-
arterial infusion of vasopressin may arrest bleeding in up to 80% of patients provided
that selective bleeding vessels can be identified. Surgery is rarely used to treat
AGML. Only those patients who have failed aggressive medical management and en-
doscopic therapy and who have treatable critical illness are candidates for surgery.
Multiple sites are usually found, leaving only subtotal or near-total gastrectomy, with
Roux-en-Y gastrojejunostomy as the only viable option. Regardless of surgical pro-
cedure, the postoperative mortality rate is high, in excess of 50%. Death is usually
from multisystem organ failure.

MALLORY-WEISS TEARS

About 10% of cases of upper gastrointestinal hemorrhage are caused by Mallo-


ry-Weiss tears. The lesion is characterized by a tear in the proximal gastric mucosa
near the esophagogastric junction. Up to 90% of these lesions stop bleeding sponta-
neously without specific intervention. Patients with cirrhosis and portal hypertension
with coagulopathy are at greatest risk for mortality, which overall averages 3%. Ini-
tial assessment and treatment should include prompt history and physical examina-
tion, resuscitation, and endoscopic evaluation. Endoscopic therapy by either injection
or thermal energy is efficient in patients with active bleeding. Transfusion of packed
red blood cells is required in 40% to 70% of patients. Patients with active bleeding at
initial endoscopy and those with coagulation disorders are at greatest risk for rebleed-
ing; roughly 30% bleed again within the first 24 hours. Medical therapy includes acid
reduction with antisecretory agents. Surgery is rarely required for control of hemor-
rhage. If bleeding fails to stop after endoscopic therapy, laparotomy for oversewing
of the mucosal tear through a high gastrotomy is appropriate. An acid-reducing pro-
cedure is not required.

UNUSUAL CAUSES OF ACUTE UPPER GASTROINTESTINAL


HEMORRHAGE

Esophageal Sources The esophagus is the source of major hemorrhage in few-


er than 3% of patients admitted for evaluation of acute upper gastrointestinal hemor-
rhage. The most common causes are infectious esophagitis, gastroesophageal reflux
disease (GERD), Barrett’s esophagus, malignancy (including adenocarcinomas and
squamous carcinomas), medication-induced erosions, Crohn’s disease, and radiation.
Patients with human immunodeficiency virus (HIV) infection and other immunocom-
promised patients are at particular risk for infectious esophagitis, including erosive
esophagitis caused by Candida albicans, other fungi, herpes simplex virus, cytomeg-
alovirus, and mycobacterial infection. Bleeding may be massive, although episodes of
less severe hemorrhage are typical. Therapy is targeted to the cause of bleeding. If ac-
tive hemorrhage is identified, endoscopic electrocoagulation or heater probe therapy
is usually effective in stopping hemorrhage, at least temporarily while definitive
management is planned. Specific therapy is targeted to the etiology and includes ap-
propriate antibiotics for infectious causes, proton-pump inhibitors for reflux-
associated conditions, and definitive multimodality cancer therapies for malignant
tumors. Emergency surgery to control hemorrhage is rarely required. Treatment with
definitive surgical management, resection for tumors, or antireflux therapy for pa-
tients with reflux-induced esophagitis is dictated by the specific cause of the hemor-
rhage.

Dieulafoy’s Lesions are rare causes of acute upper gastrointestinal hemorrhage.


The lesions are unusually large submucosal or mucosal vessels found in the gastric
mucosa, most commonly along the lesser curvature in the mid-stomach. Bleeding oc-
curs when superficial erosion into the vessel occurs, resulting in brisk, voluminous
hemorrhage that ceases spontaneously. Endoscopic diagnosis is difficult because the
lesion is rarely associated with an obvious ulcerated lesion. If a lesion is definitively
identified, the site should be marked endoscopically with India ink injection to allow
precise surgical resection. Recurrent hemorrhage is common; often, several episodes
occur before accurate diagnosis. Efforts at endoscopic ablation with sclerotherapy and
electrocoagulation have not proved effective in the few reported series. Rather, ap-
propriate definitive management calls for wedge resection of the gastric wall. Precise
endoscopic localization of the lesion allows this limited resection in lieu of more ex-
tended blind gastric resection. Because the condition is not associated with peptic
mucosal injury, vagotomy is not indicated.
Aortoenteric Fistula is an uncommon condition in which an inflammatory tract
develops between the aorta and the gastrointestinal tract. The fistula may develop as a
primary process resulting from infectious aortitis, or inflammatory aortic aneurysm,
or as a secondary process following aortic replacement with a synthetic graft for
treatment of abdominal aortic aneurysm. The fistulas characteristically develop be-
tween the proximal anastomosis and the overlying small bowel (duodenum or jeju-
num), although communication to the colon has also been noted. It is thought that a
low-grade infection at the site of contact between the anastomosis and the bowel
leads to the fistula formation. This diagnosis must be considered in any patient with
acute gastrointestinal hemorrhage and a history of aortic surgery. The “herald” bleed,
an episode of acute hemorrhage that ceases spontaneously, occurs hours to days prior
to the inevitable exsanguinating hemorrhage that will ensue if the condition is not
recognized and treated. Emergency upper endoscopy is mandatory for all patients
with suspected aortoenteric fistula. If endoscopy is negative, computed tomography
(CT) to look for evidence of inflammation at the aortic anastomosis is indicated. Oth-
ers advocate emergency angiography, including lateral views, to identify the small
mycotic aneurysm that is frequently present. Angiography should be pursued in all
patients with negative CT scans. In patients with exsanguinating hemorrhage, emer-
gency laparotomy with control of the proximal aorta is indicated. Effective surgical
management calls for removal of the aortic graft and extra-anatomic vascular bypass
to restore distal aortic flow.

ACUTE LOWER GASTROINTESTINAL HEMORRHAGE

Acute lower gastrointestinal bleeding is hemorrhage arising distal to the liga-


ment of Treitz. The colon is the source of hemorrhage in more than 95% to 97% of
cases, with the remaining 3% to 5% arising in small bowel sites. Lower gastrointesti-
nal bleeding accounts for about 15% of major episodes of gastrointestinal hemorrhage
and hence is much less common than upper gastrointestinal bleeding. The incidence
of lower gastrointestinal bleeding increases with age, reflecting the parallel increase
in acquired lesions responsible for colonic bleeding: diverticulosis and angiodyspla-
sias. The differential diagnosis of acute lower gastrointestinal hemorrhage is shown in
Table 2.
Clinical Presentation The hallmark of acute lower gastrointestinal hemorrhage
is hematochezia; passage of bloody stool, blood, or blood clots per rectum. If bleed-
ing is slower and of lesser volume, melena may also be a presenting sign, although
this is more characteristic of an upper gastrointestinal source.

Table 2. Differential Diagnosis of Lower Gastrointestinal Hemorrhage

Similarly, up to 15% of patients with massive hemorrhage from an upper gas-


trointestinal source may present with hematochezia, which is indicative of at least
1000 mL of hemorrhage over a short interval from an upper gastrointestinal source.
Roughly half of patients present with both a decrease in hemoglobin and hematocrit
and hemodynamic instability; 30% have orthostatic changes, 10% syncope, and 19%
shock. Although lower gastrointestinal hemorrhage represents a genuine emergency,
it is generally less life-threatening than upper gastrointestinal hemorrhage. Patients
are less likely to present in shock, more likely to cease bleeding spontaneously, and
usually have a lower transfusion requirement.

Colonic diverticulosis represents the most common source of lower gastroin-


testinal hemorrhage, responsible for 40% to 55% of cases of hemorrhage in most se-
ries. Colonic diverticula are common acquired lesions of the abdominal colon. Alt-
hough 40% of patients in the 5th decade of life have diverticula, this incidence rises
to 80% by the 9th decade. Hemorrhage complicates 3% to 5% of patients with diver-
ticulosis. The anatomic basis for bleeding is thought to be asymmetrical rupture of
intramural branches (the vasa recta) of the marginal artery at the dome of the diver-
ticulum or at its antimesenteric margin. It appears likely that luminal traumatic fac-
tors, including impacted fecaliths with abrasion of the vessels, lead to hemorrhage.
Hemorrhage is rarely associated with the inflammation of clinical diverticulitis. Di-
verticular hemorrhage ceases spontaneously in up to 90% of patients. Transfusion of
greater than 4 units of packed red blood cells is rare. Although left colon diverticula
are more common, bleeding tends to be more common from right colon diverticular
sources. Hemorrhage from right colon lesions may also be of greater rate and volume
than that from left-sided diverticula. After an initial episode of hemorrhage, rebleed-
ing is likely to occur in 10% of patients in the first year; thereafter, the risk for re-
bleeding increases to 25% at 4 years. Given the prevalence of colonic diverticulosis,
and the fact that most episodes of hemorrhage tend to cease spontaneously, many epi-
sodes of lower intestinal hemorrhage are attributed to colonic diverticulosis as a pre-
sumptive rather than a definitive diagnosis.

Angiodysplasias are responsible for 3% to 20% of cases of acute lower intesti-


nal bleeding. Angiodysplasias, also referred to as arteriovenous malformations, are
small ectatic blood vessels in the submucosa of the gastrointestinal tract. The overly-
ing mucosa is often thin, and superficial erosion at the site of an angiodysplasia has
been observed on histologic examination of surgical or autopsy specimens. An-
giodysplasias are identified in 1% to 2% of autopsy evaluations and increase in fre-
quency with the age of the patient. Angiodysplasias may occur throughout the gastro-
intestinal tract and represent the most common cause of hemorrhage from the small
bowel in patients older than 50 years of age. Angiodysplasias are evident on colonos-
copy as red, flat lesions about 2 to 10 mm in diameter. Lesions may appear stellate,
oval, sharp, or indistinct. Colonoscopy is the most sensitive method to identify an-
giodysplasias, although angiography is also able to identify these lesions. The use of
meperidine during colonoscopy may decrease the ability to identify angiodysplasias
because of a reduction in mucosal blood flow. Another study has identified that the
use of a narcotic antagonist may increase the size of angiodysplasias and enhances the
detection rate. On angiography, angiodysplasias appear as ectatic, slowly emptying
veins or as arteriovenous malformations with brisk, early venous filling. More than
half of angiodysplasias are localized to the right colon, and bleeding from angiodys-
plasia correlates with this distribution. Angiodysplasias may be associated with many
medical conditions, including end-stage renal disease, aortic stenosis, von Wil-
lebrand’s disease, and others. It is not clear whether this association reflects the great-
er tendency of angiodysplasias to bleed in these conditions or whether, in fact, an-
giodysplasias are more common structural findings in them.

Colonic neoplasms, including adenomatous polyps, juvenile polyps, and car-


cinomas, present in a variety of manners. Typically, bleeding from these lesions is
slow, characterized by occult bleeding and secondary anemia. These neoplasms can
bleed briskly, however, and in some series, up to 20% of cases of acute hemorrhage
are ultimately found to arise from colonic polyps or cancers. Juvenile polyps are the
second most common cause of hemorrhage in patients younger than the age of 20
years.

A wide variety of inflammatory conditions can cause acute lower gastrointes-


tinal hemorrhage. Hemorrhage is rarely the presenting sign; rather, it develops in the
course of the disease, and the cause is suspected based on the patient’s history. Up to
20% of cases of acute lower gastrointestinal hemorrhage may be due to one of these
inflammatory conditions. Most episodes of bleeding cease spontaneously or with spe-
cific therapy directed at the cause. Hemorrhage complicates the course of ulcerative
colitis in up to 15% of cases. Emergency colectomy for persistent hemorrhage ac-
counts for 6% to 10% of emergency surgical colectomies in patients with this disease.
Crohn’s disease is less likely to cause massive colonic hemorrhage and occurs in
roughly 1% of patients with this condition. Infectious causes include Escherichia coli,
typhoid, Cytomegalovirus, and Clostridium difficile. Radiation injury is most com-
mon in the rectum after pelvic radiotherapy for prostate or gynecologic malignancies.
Bleeding is most common 1 year after radiation treatments but may occur up to 4
years later. Patients with immunosuppression or acquired immunodeficiency syn-
drome (AIDS) are at risk for acute lower intestinal hemorrhage from a unique set of
causes. Cytomegalovirus is the most common cause; Kaposi’s sarcoma, histoplasmo-
sis, and perianal fistulas and fissures are also problematic and are more likely to hem-
orrhage in patients with AIDS-induced thrombocytopenia.

Vascular causes of acute lower intestinal hemorrhage include the vasculitides


(polyarteritis nodosa, Wegener’s granulomatosis, rheumatoid arthritis, and others),
which are associated with punctate ulceration of the colon and small bowel. Colonic
ischemia with mucosal ulceration and friability may also result in acute hemorrhage,
often in the setting of acute abdominal pain and sepsis. Acute mesenteric ischemia
may be heralded by an episode of hematochezia in the context of severe abdominal
pain, preexisting vascular disease, arterial embolism risk, or hypercoagulability. Alt-
hough hemorrhage is an element in the clinical management of these patients, only
rarely does the control of hemorrhage become the major focus of therapy. Rather, res-
toration of visceral perfusion is the primary therapeutic objective.

Hemorrhoids are usually noted on physical examination in more than half of


patients with lower gastrointestinal hemorrhage. In fewer than 2% can the hemor-
rhage be attributed to these lesions, however. Unless unequivocal signs of bleeding
are evident on anoscopy, investigation of the patient for another source of lower in-
testinal bleeding should be pursued. Patients with portal hypertension may develop
massive hemorrhage from hemorrhoids, as can patients with HIV-associated throm-
bocytopenia with hemorrhoids.

Uncommon Causes Rare causes of lower gastrointestinal hemorrhage include


solitary rectal ulcer, Dieulafoy’s lesion of the colon, portal colopathy, NSAIDs, intus-
susceptions, or bleeding following colonoscopic biopsy or polypectomy.

The initial history and physical examination are directed to determining the po-
tential source of the hemorrhage and the severity of initial hemorrhage. Most cases
eventually are determined to result from angiodysplasia or diverticulosis, both of
which are usually asymptomatic before initial hemorrhage. Nonetheless, the initial
history should exclude other, less common causes of the bleeding. Specific inquiry
should be made regarding use of NSAIDs or anticoagulants. Abdominal pain or re-
cent diarrhea and fever may point to colitis, either infectious or ischemic. Patients
with prior aortic surgery should be considered to have an aortoenteric fistula until
proved otherwise. Prior radiation therapy for pelvic malignancy may indicate radia-
tion proctitis. Recent colonoscopy may suggest bleeding from a biopsy or polypecto-
my site. The cause of previous episodes of bleeding should be elicited, as should the
possibility of a history of inflammatory bowel disease. Family history of polyposis
syndromes or colonic malignancy may also be pertinent. Young patients—those less
than 30 years of age—are at greatest risk for bleeding from Meckel’s diverticulum or
intestinal polyps. Physical examination should include measurement of orthostatic vi-
tal signs in patients without evident shock. All patients should be resuscitated, as out-
lined in the previous section. Pertinent findings on physical examination may include
scars from previous abdominal incisions, the presence of abdominal masses, or skin
and oral lesions suggestive of polyposis syndromes. Stigmata of cirrhosis suggestive
of bleeding from hemorrhoids or varices secondary to portal hypertension should be
considered. The rectal examination is important to identify any anorectal pathology,
including tumors, ulcers, or polyps. The color of the rectal contents and the presence
of formed stool or blood clot should also be noted. Anoscopic examination to exclude
hemorrhage from hemorrhoids should be completed. A nasogastric tube should be in-
serted to look for blood or coffee ground–like material to exclude an upper gastroin-
testinal source. In patients with hematochezia and hemodynamic instability, emergen-
cy upper endoscopy is required.

Diagnosis

Emergency surgical intervention for ongoing massive hemorrhage is rarely


necessary before attempts are made to localize the precise source of bleeding. This
allows an orderly approach to identification of the bleeding site, which is essential for
appropriate therapy. After the patient has been resuscitated and stabilized, diagnostic
testing should begin. The choice of initial investigation remains controversial and is
dependent to some degree on local availability of procedures and expertise. The three
options for primary diagnostic testing are colonoscopy, selective visceral angi-
ography, and technetium 99m (99mTc)-labeled red blood cell scintigraphy.

Colonoscopy Recognition that most episodes of hemorrhage cease spontane-


ously and that stigmata of bleeding are subtle has led to efforts to perform colonosco-
py as early as possible in the course of evaluation. Urgent colonoscopy, completed
within 12 hours of admission, is indicated in patients who have ceased to have ongo-
ing significant hemorrhage and in whom resuscitation and hemodynamic stability
have been achieved. In this setting, colonoscopy can be completed after colonic purg-
ing. Positive findings on colonoscopy include identification of an active bleeding
site, identification of a nonbleeding visible vessel, clot adherent to a diverticular ul-
cerated orifice, clot adherent to a discrete focus of mucosa, or fresh blood localized to
a colonic segment. Hemorrhage can be attributed only to lesions with clear stigmata
of bleeding. Patients presenting with massive lower gastrointestinal hemorrhage are
poor candidates for emergency colonoscopy. The procedure is technically difficult
because of the inability to clear the mucosal surfaces of old or new hemorrhage. Find-
ing a discrete, actively bleeding vessel in the unprepared bowel is difficult even for
the most experienced endoscopist. Second, patients with massive hemorrhage have
hemodynamic instability, precluding the use of sedation and increasing the risk for
hypoxemia and complication. Further, resuscitation may be compromised during the
procedure. Hence, colonoscopy is most appropriately used as the initial diagnostic
procedure in patients presenting with acute hemorrhage that has ceased or in patients
with a more moderate degree of bleeding.

Selective Visceral Angiography Mesenteric arteriography has been widely


used in the evaluation and treatment of patients with lower gastrointestinal hemor-
rhage. Selective injection of radiographic contrast into the superior mesenteric or in-
ferior mesenteric arteries identifies hemorrhage in patients bleeding at a rate of 0.5
mL/min or greater. Given the characteristic intermittent bleeding seen in lower gas-
trointestinal hemorrhage associated with diverticulosis, arteriovenous malformations,
and other causes, bleeding may have ceased by the time of the study. Some radiolo-
gists have advocated evocative testing, including intra-arterial vasodilators, heparin,
and fibrinolytic agents, in an effort to identify a bleeding source accurately. This ap-
proach does not appear to be warranted except in patients with refractory intermittent
episodes of hemorrhage in a fully staffed suite. Because 90% of cases of hemorrhage
cease spontaneously, and only 10% rebleed, such evocative testing is inappropriate
for most patients. About 10% of patients develop a complication of angiography. Ma-
jor complications include stroke, renal failure, femoral artery thrombosis, lower ex-
tremity immobilization, and hematoma formation. Given that most patients with low-
er gastrointestinal hemorrhage are older than 60 years of age, medical co morbidities,
including vascular disease and renal insufficiency, may place these patients at high
risk for the procedure. Hence, angiography is reserved for patients with evidence of
significant ongoing hemorrhage.

Technetium 99m-Red Blood Cell Scintigraphy 99m


Tc-red blood cell scintig-
raphy has met with mixed success in the diagnosis of lower gastrointestinal hemor-
rhage. In this noninvasive nuclear medicine imaging procedure, the patient’s red
blood cells are labeled with a technetium isotope and reintroduced into the circula-
tion. With each bleeding episode, labeled blood is shed into the colonic lumen, creat-
ing an isotopic focus that can be imaged with whole abdominal scintigraphy. Rates of
bleeding as low as 0.1 mL/min can be detected. Images are obtained at distinct inter-
vals after injection, within the first 2 hours, and thereafter at 4- to 6-hour intervals, or
at the time of clinical evidence of rebleeding. After extravasation into the lumen, the
blood moves through the colonic lumen, generally from the right colon to the left, but
occasionally in retrograde fashion because of colonic contractions. If bleeding is pre-
sent at the time of injection and initial imaging, 99mTc-red blood cell scans can accu-
rately identify a source of bleeding in up to 85% of cases. If bleeding is not active at
the time of the initial study, or if delayed bleeding occurs, subsequent imaging to de-
tect the luminal isotope can be inaccurate because of the sporadic movement of the
tracer in the gut lumen. Patients in whom a surgical resection is anticipated to control
recurrent or persistent hemorrhage should have the bleeding confirmed with either a
positive angiogram or a positive colonoscopy. The red blood cell scans serve primari-
ly to target the subsequent confirmatory study.

Treatment

Endoscopic Treatment Endoscopic therapy includes the use of the same mo-
dalities available for upper gastrointestinal hemorrhage. Thermal heater probes, elec-
trocoagulation, and sclerotherapy have been used. Reports suggest that electro coagu-
lation can be successfully applied for bleeding colonic diverticula, although this ap-
proach has not been widely embraced. Efforts at endoscopic control of diverticular
hemorrhage may precipitate more significant bleeding. In contrast, angiodysplasias
are readily treated with endoscopic measures. Acute bleeding can be controlled in up
to 80% of patients with bleeding angiodysplasias, although rebleeding may develop
in up to 15%. Care must be taken to avoid precipitating massive hemorrhage when
treating angiodysplasias. Many endoscopists recommend approaching the lesion from
the perimeter, obliterating feeder vessels before cauterization of the central vessel.
Endoscopic therapy is also appropriate for patients with bleeding from a recent snare
polypectomy site. Bleeding develops in 1% to 2% of patients after polypectomy and
may occur up to 2 weeks after polypectomy. An endoscopic approach is recommend-
ed for these lesions.

Angiographic Treatment In patients whose bleeding source is identified by


angiography, a trial of angiographic therapy may be appropriate as a perioperative
temporizing measure or as a definitive measure for high-risk surgical candidates.
Provided selective catheterization of a mesenteric vessel leading directly to the bleed-
ing site can be completed, intra-arterial vasoconstrictor therapy with vasopressin can
temporarily achieve control of bleeding in up to 80% of patients. Rebleeding is com-
mon, however, after discontinuing the therapy. Complications are frequent and seri-
ous and include myocardial ischemia, pulmonary edema, mesenteric thrombosis, and
hyponatremia. Transarterial vasopressin should not be used in patients with coronary
artery disease or other vascular disease. The primary role of this therapy is to achieve
temporary control of bleeding before emergency definitive surgical resection.
Transcatheter embolization of massive bleeding may also be used for patients who
are poor candidates for surgical resection. Embolization of gelatin sponges or micro-
coils can achieve temporary control of bleeding from angiodysplasias and diverticula.
Given the lack of collateral blood supply to the colonic wall, these procedures may be
complicated by colonic infarction heralded by abdominal pain, fever, and sepsis.
Hence, like vasoconstrictive therapy, this procedure should be restricted to patients
who cannot tolerate surgery or as a temporizing measure in massive hemorrhage in
patients for whom a definitive surgical resection is imminent.

Surgery is indicated for patients with ongoing or recurrent hemorrhage. Trans-


fusion of more than 6 units of packed red blood cells, ongoing transfusion require-
ment, or persistent hemodynamic instability is an indication for colectomy in acute
hemorrhage. Patients who develop recurrent lower gastrointestinal hemorrhage are
also appropriately treated with colectomy because the risk for subsequent hemorrhage
increases with time. Segmental colectomy is indicated in patients with persistent or
recurrent colonic hemorrhage. Every effort should be made to localize the source of
bleeding so that a hemicolectomy can be performed rather than a blind subtotal ab-
dominal colectomy. Certainty of the site of bleeding is important; operation based on
a positive 99mTc-red blood cell scan alone can result in recurrent hemorrhage in up to
35% of patients. “Blind” total abdominal colectomy carries significantly higher peri-
operative morbidity, and associated mortality rates approach 25% in some series. Di-
arrhea and rapid transit after total abdominal colectomy can also be debilitating con-
ditions for elderly patients. There is no indication for a blind segmental colectomy,
for which rebleeding rates as high as 75% are seen. Mortality after colectomy for
acute lower gastrointestinal hemorrhage overall is less than 5%. As in upper gastroin-
testinal hemorrhage, bleeding is not the cause of death; rather, pneumonia, cardiovas-
cular events, and renal failure lead to poor outcomes, primarily in elderly patients
with recurrent hemorrhage. Thoughtful timely management can lead to a successful
outcome in most patients.

RARE CAUSES OF GASTROINTESTINAL HEMORRHAGE FROM


AN OBSCURE SOURCE

The small bowel is a rare source of acute hemorrhage. Only 2% to 5% of pa-


tients with acute gastrointestinal hemorrhage are ultimately determined to have bled
from a small intestinal source. This low frequency is fortunate because the small
bowel is a difficult organ to visualize and precise detection of the bleeding lesion is
characteristically delayed. Acute gastrointestinal hemorrhage from an obscure source
has been reported to occur from a variety of conditions. These include radiation enter-
itis, small intestinal varices, Crohn’s disease, tuberculosis, syphilis, typhoid, histo-
plasmosis, vasculitis, small bowel ulcerated lesions in patients with gastrin-secreting
tumors, and Dieulafoy’s lesions. Medical treatment is appropriate for most infectious
causes and in patients with Zollinger-Ellison syndrome. Enterectomy is required in
the other conditions. Disorders of the pancreas can cause acute gastrointestinal hem-
orrhage as blood is delivered into the duodenum through the pancreatic duct. Such
bleeding has been reported in the setting of acute pseudoaneurysms after pancreatec-
tomy and in pancreatic tumors. Bleeding is a rare complication of these disorders.
Angiography may confirm the presence of a pseudoaneurysm and allow angiographic
embolization for acute hemorrhage control. Pancreatic resection may be appropriate,
depending on the clinical condition. The liver may also be the source of presumed
acute gastrointestinal hemorrhage. Bleeding into the hepatic duct presents as gastroin-
testinal hemorrhage as blood enters the duodenum from the common bile duct, a con-
dition known as hemobilia. Hemobilia has been reported to occur secondary to hepat-
ic trauma with intrahepatic hematoma, hepatic aneurysms or other vascular malfor-
mations, hepatic tumors, hepatic abscess, or after hepatic resection or percutaneous
liver biopsy. This diagnosis is usually considered when endoscopic visualization dur-
ing acute hemorrhage shows blood entering the duodenum at the ampoule of Vater,
depending on the clinical scenario. Selective visceral angiography is usually required
to define the source and often allows definitive management by intra-arterial emboli-
zation.

Basic literature:
43.Oxford Textbook of Surgery (3-Volume Set) 2nd edition (January 15, 2000):
by Peter J. Morris (Editor), William C. Wood (Editor) By Oxford Press
44.Sabiston Textbook of Surgery 17th edition by Courtney M. Townsend Jr.,
Kenneth L. Mattox, B. Mark, MD Evers, Kenneth L., MD Mattox, Courtney
Townsend, Daniel Beauchamp, B. Mark Evers, Kenneth Mattox W.B. Saun-
ders Company (June, 2004)
45.Schwartz´s Principles of Surgery 8th Edition F. Charles Brunicardi. Copyright
©2007 the McGraw-Hill Companies.
46.Hospital surgery/ Edited by L. Kovalchuk et al. - Ternopil: Ukrmedknyha,
2004. - 472 p.

Additional literatures:

1. Church NI, Palmer KR: Ulcers and nonvariceal bleeding. Endoscopy


35:22–26, 2003.
2. Kollef MH, Canfield DA, Zuckerman G: Triage considerations for pa-
tients with acute gastrointestinal hemorrhage admitted to a medical intensive care
unit. Crit Care Med 23:1048–1054, 1995.
3. Vernava AM, Moore BA, Longo WE, et al: Lower gastrointestinal bleed-
ing. Dis Colon Rectum 40:846–858, 1997.
4. Wara P: Endoscopic prediction of major rebleeding: A prospective study
of stigmata of hemorrhage in bleeding ulcer. Gastroenterology 88:1209–1214, 1985.
5. Zuckerman GR, Prakash C, Askin MP, et al: AGA technical review on
the evaluation and management of occult and obscure gastrointestinal bleeding. Gas-
troenterology 118:201–221, 2000.

Tests for initial level of knowledge, keys for tests:


1. A 55-year-old man complains of chronic intermittent epigastric pain,
and gastroscopy demonstrates a 2-cm ulcer of the distal lesser curvature. Endoscopic
biopsy yields no malignant tissue. After a 6-wk trial of H2-blockade and antacid
therapy, the ulcer is unchanged. Proper therapy at this point is:

A. Repeat trial of medical therapy


B. Local excision of the ulcer
C. Billroth I partial gastrectomy
D. Billroth I partial gastrectomy with vagotomy
E. Vagotomy and pyloroplasty

2. A 55-year-old man complains of chronic intermittent epigastric pain, and


gastroscopy demonstrates a 2-cm ulcer of the distal lesser curvature. Endoscopic bi-
opsy yields no malignant tissue. After a 6-wk trial of H2-blockade and antacid thera-
py, the ulcer is unchanged. After initial resuscitation, this man should undergo:
A. Esophageal balloon tamponade
B. Barium swallow
C. Selective angiography
D. Esophagogastroscopy
E. Exploratory celiotomy

3. A diagnosis of bleeding esophageal varices is made in patient. The ap-


propriate initial therapy would be:
A. Intravenous vasopressin
B. Endoscopic sclerotherapy
C. Emergency portacaval shunt
D. Emergency esophageal transection
E. Esophageal balloon tamponade

4. Which of the following statements regarding stress ulceration is true?


A. It is true ulceration, extending into and through the muscularis mucosa
B. It classically involves the antrum
C. Increased secretion of gastric acid has been shown to play a causative role
D. It frequently involves multiple sites
E. It is seen following shock or sepsis, but for some unknown reason does not
occur following major surgery, trauma, or burns
5. Dieulafoys lesion of the stomach is characterized by
A. A large mucosal defect with underlying, friable vascular plexus
B. Frequent rebleeding after endoscopic treatment
C. Massive bleeding that requires subtotal gastrectomy
D. Location in the proximal stomach
E. Acid-peptic changes of the gastric mucosa

6. A 72-year-old man with severe coronary artery disease presents with pain-
less hematemesis following a prolonged bout of vomiting. Upper endoscopy reveals a
tear just below the gastroesophageal junction, which is actively bleeding. Select the
best course of action.
A. Administration of intravenous vasopressin
B. Administration of intraarterial vasopressin
C. Left thoracotomy, full-thickness suture ligation, and drainage of the pleural
cavity
D. Balloon tamponade
E. Insertion of a chest tube

7. A 56-year-old man complains of the onset of severe substernal pain after a


night of heavy drinking followed by uncontrolled retching. He states that there was a
small amount of blood in his vomit. A chest X-ray shows a moderate sized left pleu-
ral effusion. Select the best course of action.
A. Administration of intravenous vasopressin
B. Administration of intraarterial vasopressin
C. Left thoracotomy, full-thickness suture ligation, and drainage of the pleural
cavity
D. Balloon tamponade
E. Insertion of a chest tube
8. Which vessel is most commonly associated with a posterior duodenal ulcer?

F. Right gastroepiploic artery


G. Common hepatic artery
H. Gastroduodenal artery
I. Superior mesenteric artery
J. Middle colic artery

33. The blood supply to the stomach and duodenum arises from all of the
following arteries EXCEPT
F. Gastroepiploic artery
G. Common hepatic artery
H. Splenic artery
I. Superior mesenteric artery
J. Inferior mesenteric artery

10. Gastric acid production is altered by all of the following hormones or ac-
tions EXCEPT
F. Cholecystokinin
G. Gastrin
H. Vagal stimulation
I. Secretin
J. Glucagon

Keys for tests

1 2 3 4 5 6 7 8 9 10
C D B D D D C C A E

Tests for final level of knowledge, keys for tests:


1. The commonest cause of death in patients with alcoholic cirrhosis following
portosystemic shunting is:
A. Bleeding esophageal varices
B. Hepatic failure with encephalopaty
C. Malnutrition
D. Hepatocellular carcinoma
E. Cardiac failure with peripheral edema and ascites

2. A 30-year-old man with a duodenal ulcer is being considered for surgery be-
cause of intractable pain and a previous bleeding episode. Serum gastrin levels are
found to be over 1000 pg/mL (normal 40-150) on three separate determinations. The
patient should be told that the operation of choice is:
A. Vagotomy and pyloroplasty
B. Highly selective vagotomy and tumor resection
C.Subtotal gastrectomy
D.Total gastroctomy
E.Partial pancreatectomy

3. All of the following problems commonly occur with the use of balloon tam-
ponade for control of variceal bleeding EXCEPT
A. Pneumonia
B. Aspiration of nasopharyngeal secretion
C. Rebleeding following removal of the tube
D. Gastritis
E. Esophageal ulceration or perforation

4. Omeprazole has been added to the H2-antagonists as a therapeutic approach


to the management of acute bleeding gastric and duodenal ulcers. It acts by
A. Blocking breakdown of mucosal-damaging metabolites of NSAIDs
B. Providing a direct cytoprotective effect
C. Buffering gastric acids
D. Inhibiting parietal cell hydrogen-potassium-ATPase
E. Inhibiting gastrin release and parietal cell acid production
5. What is the most reliable method for precisely locating an upper gastrointes-
tinal lesion that is responsible for a bleeding?
A. Upper GI series
B. Exploratory laparotomy
C. Upper GI endoscopy
D. Arteriography
E. Radionuclide scanning

6. The most common cause of massive upper gastrointestinal bleeding is


A. Gastric ulcer
B. Erosive gastritis
C. Gastric carcinoma
D. Mallory-Weiss tear
E. Duodenal ulcer

7. A 50-year-old man is admitted with massive, bright red rectal bleeding.


He recently had a barium enema that demonstrated no diverticular or space-
occupying lesion. Nasogastric suction reveals no blood but does produce yellow bile.
The patient continues to bleed. What is the next diagnostic step?
A. Repeat barium enema.
B. Colonoscopy.
C. Upper GI series.
D. Mesenteric angiography.
E. Small bowel follow-through with barium.

8. All of the following statements regarding lower GI bleeding are true


EXCEPT
A. If bleeding is profuse, angiography may be useful.
B. The mortality rate is about 10%.
C. Persistent bleeding is an indication for surgery.
D. Only 10%-15% of patients stop bleeding spontaneously.
E. Blind total colectomy may be a necessary procedure

9. The correct surgical treatment for Mallory-Weiss tear of the esophagus is


A. Transthoracic ligation of varices.
B. Transthoracic antireflux procedure.
C. Laparotomy, gastrotomy, and oversewing the bleeding vessel.
D. Laparotomy and resection of the gastroesophageal junction.
E. Antibiotics and observation.

10. Massive upper GI bleeding occurs in an otherwise asymptomatic, nor-


mal man following a violent episode of retching and vomiting without blood. The
most likely cause of this man´s bleeding is:
A. Hiatal hernia.
B. Mallory-Weiss tear.
C. Carcinoma of the stomach.
D. Duodenal ulcer.
E. Gastritis.
Keys for tests

1 2 3 4 5 6 7 8 9 10
B B D D C E D D C B

Tasks for final level of knowledge

1. A 59-year-old man is referred for evaluation because he has been faint-


ing at his job where he operates heavy machinery. He is pale and emaciated, but oth-
erwise his physical examination is remarkable only for 4+ occult blood in the stool.
Lab shows a hemoglobin of 5 g/dl.
What is it? Cancer of the right colon.

2. A 56-year-old man has bloody bowel movements. The blood coats the
outside of the stool, and has been present on and off for several weeks. For the past
2 months he has been constipated, and his stools have become of narrow
caliber.
What is it? Cancer of the distal part, left side of the colon.

3. A 24-year-old man spends the night cruising bars and drinking heavily.
In the wee hours of the morning he is quite drunk, and he starts vomiting repeatedly.
He initially brings up gastric contents only, but eventually he vomits bright red blood.
What is it? Mallory-Weiss tear.

4. A 33-year-old man has had three large bowel movements that he de-
scribes as made up entirely of dark red blood. The last one was 20 minutes ago. He is
diaphoretic and pale, and has a blood pressure of 90 over 70 and a pulse rate
of 110. An NG tube returns copious amounts of bright red blood.
What is it? The area has been defined (tip of the nose to ligament of Treitz).
Proceed with endoscopy.
5. A 41-year-old man has been in the ICU for 2 weeks being treated for idi-
opathic hemorrhagic pancreatitis. He has had several percutaneous drainage proce-
dures for pancreatic abscesses, chest tubes for pleural effusions. He has been in and
out of septic shock and respiratory failure several times. Ten minutes ago he vomited
a large amountof bright red blood, and as you approach him he vomits again what
looks like another pint of blood.
What is it? In this setting it has to be stress ulcer.

Materials for the self-study of the students


Main tasks Notes (instructions)

Repeat:
-To represent the methods of diagnostics
1. Anatomy of GI tract
of diseases of alimentary tract
2. Physiology of GI tract
-To make the flow diagram of mecha-
3. Types of GI bleeding.
nisms and diagnosis of GI bleeding.

Study:
1. Stages of digestive bleeding.
2. Methods of endoscopic hemosta- -To conduct differential diagnosis with
sis. the inner bleeding of organs of ab-
3. Forrest classification dominal cavity
4. Non-peptic ulcer causes of upper
gastrointestinal bleeding.
Study guide #9.1
“Portal hypertension. Clinical picture, diagnosis, tactics of conservative and sur-
gical treatment.”

Overview

Portal hypertension may be classified as presinusoidal, sinusoidal, or postsi-


nusoidal. Sinusoidal causes are the most common in the Western Hemisphere due to
alcoholic cirrhosis that results from fibrous replacement in the space of Disse. Chron-
ic liver insufficiency is common. Postsinusoidal portal hypertension often has a vas-
cular etiology and also is associated with some degree of liver dysfunction. In con-
trast, patients with presinusoidal portal hypertension may have well-preserved hepatic
function. Etiologies include schistosomiasis, extrahepatic portal vein thrombosis, and
congenital hepatic fibrosis (most commonly seen in children). The sequelae (conse-
quences) of portal hypertension are varied, and the long-term outcomes are related
most strongly to the underlying degree of hepatic dysfunction. Upregulation of nitric
oxide synthase has been shown to play an important role in augmented blood flow in
portal hypertension. Although excessive splanchnic blood flow is rarely a sole cause
of portal hypertension, many cirrhotic patients have some increased splanchnic flow
that may be attributed to nitric oxide.

Educational aims:
18. Interrogation and clinical inspection of patients with portal hypertension.
19. To determine the etiologic and pathogenic factors of portal hypertension.
20. To find out the clinical features of portal hypertension.
21. To develop a plan of examination of the patients with portal hypertension.
22. To estimate results of physical examination, ultrasonography, X-ray, CT and ra-
diological examination in patients with portal hypertension.
23. To make differential diagnosis between different liver diseases.
24. To understand the pathogenesis of the complications of portal hypertension.
25.To determine the indications for treatment of patients with portal hypertension.
26.To compare the different methods of surgical treatment of portal hypertension.
27.To provide postoperative care after surgery.
A student must know:
19. Anatomy of the liver.
20. Classification of liver diseases.
21. Clinical presentations of uncomplicated cirrhosis.
22. Mechanisms of the development of the different forms of portal hypertension and
its complications.
23. Methods of diagnostics of portal hypertension.
24. Differential diagnosis between different liver diseases and lesions of the other or-
gans.
25. Surgical approaches to the treatment of portal hypertension.
26. Methods of the treatment of the patients with hepatic insufficiency.
27. Indications to liver transplantation.
A student must be able to:
13. To collect and estimate the complaints of patient with hepatic diseases, infor-
mation of anamnesis, to conduct physical examination and interpret the results
which have been gotten.
14. To define the rational quantity of laboratory and instrumental research methods.
15. Be able to examine patients with diseases of the liver.
16. To define the indications to surgical interventions and choose the appropriate sur-
gical method.
17. To prescribe preoperative preparation depending on patient’s state.
18. To provide the appropriate postoperative care.

Terminology
Term Definition
the diaphragmatic peritoneal duplications are referred to as the
Coronary liga-
coronary ligament whose lateral margins on either side are the
ment
right and left triangular ligaments
Falciform liga- a thin membrane that attaches the liver surface to the diaphragm,
ment abdominal wall, and umbilicus
runs along the inferior edge of the falciform ligament from the
Ligamentum teres
umbilicus to the umbilical fissure
Ligamentum extending from the left portal vein in the porta hepatis toward the
venosum left hepatic vein and the inferior vena cava (IVC)
is a consequence of chronic liver disease characterized by re-
Cirrhosis of the placement of liver tissue by fibrosis, scar tissue and regenerative
liver nodules (lumps that occur as a result of a process in which dam-
aged tissue is regenerated), leading to loss of liver function
is defined as an excess deposition of the components of extracel-
Fibrosis of the liv-
lular matrix (i.e., collagens, glycoproteins, proteoglycans) within
er
the liver
Esophageal vari-
are extremely dilated sub-mucosal veins in the lower esophagus
ces

Splenomegaly enlarged spleen

Content:
ANATOMY AND PHYSIOLOGY
Gross Anatomy
A precise knowledge of the anatomy of the liver is an absolute prerequisite to
performing surgery on the liver or biliary tree. With the development of hepatic sur-
gery over the past few decades a greater appreciation for the complex anatomy be-
yond the misleading minimal external markings has been realized. The days of utiliz-
ing the falciform ligament as the only marker of a left and right side of the liver are
over, and the anatomic contributions of Couinaud and the description of the segmen-
tal nature of the liver should be embraced and studied by students of hepatic surgery.

General Description and Topography


The liver is a solid gastrointestinal organ whose mass (1200 to 1600 g) largely
occupies the right upper quadrant of the abdomen. The costal margin coincides with
the lower margin, and the superior surface is draped over by the diaphragm. The large
majority of the right liver and most of the left liver is covered by the thoracic cage.
The liver extends superiorly to the height of the fifth rib on the right and the sixth rib
on the left. The posterior surface overlaps the inferior vena cava (IVC). A wedge of
liver extends to the left half of the abdomen across the epigastrium to lie above the
anterior surface of the stomach and under the central and left diaphragm. The superior
surface of the liver is convex and is molded to the diaphragm, whereas the inferior
surface is mildly concave and extends to a sharp anterior border.
The liver is situated in peritoneum except for the gallbladder, the porta hepatis,
and posteriorly on either side of the IVC in two wedge-shaped areas (called the bare
area of the liver to the right of the IVC). The peritoneal duplications on the liver sur-
face are referred to as ligaments. The diaphragmatic peritoneal duplications are re-
ferred to as the coronary ligament whose lateral margins on either side are the right
and left triangular ligaments. From the center of the coronary ligament emerges the
falciform ligament, which extends anteriorly as a thin membrane connecting the liver
surface to the diaphragm, abdominal wall, and umbilicus. The ligamentum teres (the
obliterated umbilical vein) runs along the inferior edge of the falciform ligament from
the umbilicus to the umbilical fissure. The umbilical fissure is on the inferior surface
of the left liver and contains the left portal triad. The falciform ligament, the most ob-
vious surface marking of the liver, historically was used to mark the division of the
right and left lobes of the liver in early descriptions of hepatic anatomy. However,
this description is inaccurate and of minimal utility to the hepatobiliary surgeon. On
the posterior surface of the left liver running from the left portal vein in the porta
hepatis toward the left hepatic vein and the IVC is the ligamentum venosum (oblite-
rated sinus venosus), which also runs in a fissure. Hepatic arterial and portal venous
blood flow enter the liver at the hilum and branch throughout the liver as a single unit
that also includes bile ducts (portal triad). This unit is enclosed in a peritoneal sheath
that originates at the hepatic hilum. Venous drainage is through hepatic veins that
empty directly into the IVC.

Portal Vein
The portal vein provides about 75% of hepatic blood flow; and although it is
postcapillary and largely deoxygenated, its large volume flow rate provides 50% to
70% of the liver’s oxygenation. The lack of valves in the portal venous system pro-
vides a system that can accommodate high flow at low pressure because of the low
resistance and allows measurement of portal venous pressure anywhere along the sys-
tem. The portal vein forms behind the neck of the pancreas at the confluence of the
superior mesenteric vein and the splenic vein at the height of the second lumbar ver-
tebra. The length of the main portal vein ranges from 5.5 to 8 cm, and its diameter is
usually about 1 cm. Cephalad to its formation behind the neck of the pancreas, the
portal vein runs behind the first portion of the duodenum and into the hepatoduodenal
ligament, where it runs along the right border of the lesser omentum usually posterior
to the bile duct and hepatic artery. The portal vein divides into main right and left
branches at the hilum of the liver. The left branch of the portal vein runs transversely
along the base of segment IV and into the umbilical fissure, where it gives off
branches to segments II and III and feeds back branches to segment IV. The left por-
tal vein also gives off posterior branches to the left side of the caudate lobe. The right
portal vein has a short extrahepatic course and usually enters the substance of the liv-
er, where it splits into anterior and posterior sectoral branches. These sectoral branch-
es can occasionally be seen extrahepatically and can come off the main portal vein
before its bifurcation. There is usually a small branch off the right portal vein or at the
bifurcation that comes off posteriorly to supply the caudate process. There are a num-
ber of connections between the portal venous system and the systemic venous system.
Under conditions of high portal venous pressure, these portosystemic connections
may enlarge secondary to collateral flow. The most significant portosystemic collat-
eral locations are (1) the submucosal veins of the proximal stomach and distal esoph-
agus, which receive portal flow from the short gastric veins and the left gastric vein
and can result in varices with the potential for intestinal hemorrhage; (2) umbilical
and abdominal wall veins, which recanalize from flow through the umbilical vein in
the ligamentum teres resulting in caput medusae; (3) the superior hemorrhoidal plex-
us, which receives portal flow from inferior mesenteric vein tributaries and yields
large hemorrhoids; and (4) other retroperitoneal communications yielding collaterals
that can make abdominal surgery hazardous. The anatomy of the portal vein and its
branches is relatively constant and has much less variation than the ductal and hepatic
arterial system. The portal vein is rarely found anterior to the neck of the pancreas
and the duodenum. Entrance of the portal vein directly into the vena cava has also
been described. Very rarely, a pulmonary vein may enter the portal vein. Lastly, there
may be a congenital absence of the left branch of the portal vein. In this situation, the
right branch courses through the right liver and curves around peripherally to supply
the left liver.

Assessment of Liver Function


A wide variety of tests are available to evaluate hepatic diseases. Screening for
hepatic disease, assessing hepatic function, diagnosing specific disorders, and prog-
nosticating are critical in the management of hepatic pathology. For the surgeon, as-
sessment of hepatic function and estimating the ability of a hepatic remnant to be suf-
ficient after liver resection is also of obvious importance. Unfortunately, most
measures of hepatic disease are gross and lack sensitivity, specificity, and accuracy.
We have divided these tests into the following categories: routine screening tests,
specific diagnostic tests, and quantitative tests of hepatic function.
Screening blood tests are often used to simply ask the question, is there a path-
ologic process in the hepatobiliary system? Standard liver function tests (LFTs) are
generally not tests of function and are not always specific to hepatic pathology. None-
theless they are valuable as a general screening method that can provide the basic
tools to recognize the presence of hepatic disease and give clues about the etiology of
that disease. Total bilirubin, direct bilirubin (conjugated), and indirect bilirubin (un-
conjugated) levels can be affected by a number of processes that are related to the
metabolism of bilirubin. Unconjugated hyperbilirubinemia can be a reflection of in-
creased bilirubin production (e.g., hemolysis), drug effects, inherited enzymatic dis-
orders, and the physiologic jaundice of the newborn. Conjugated hyperbilirubinemia
is generally a result of cholestasis or mechanical biliary obstruction, but it can also be
seen in some inherited disorders or hepatocellular disease. The transaminases alanine
aminotransferase (ALT) and aspartate aminotransferase (AST) are the most common
serum markers of hepatocellular necrosis, with subsequent leak of these intracellular
enzymes into the circulation. AST is found in a variety of other organs (heart, muscle,
and kidney), but ALT is liver specific. The level of elevation of these enzymes has
never been shown to be of prognostic value. Alkaline phosphatase (ALP) is expressed
in liver, bile ducts, bone, intestine, placenta, kidney, and leukocytes. Isoenzyme de-
terminations can sometimes be helpful in distinguishing the source of an elevated
ALP. Elevations of ALP in hepatobiliary diseases are generally secondary to choles-
tasis or biliary obstruction and are caused by increased production of the enzyme.
ALP can also be elevated in malignant disease of the liver. γ-Glutamyltranspeptidase
(GGT) is an enzyme in many organs aside from the liver (kidney, seminal vesicle,
spleen, pancreas, heart, and brain) and can be elevated in diseases affecting any of
these. It is induced by alcohol intake and is elevated in biliary obstruction. It is also a
nonspecific marker of liver disease, but it can be helpful in determining if an elevated
ALP level is from hepatic pathology. 5'-Nucleotidase is also found in a wide variety
of organs besides the liver, but increased levels are fairly specific to hepatic patholo-
gy. Like GGT, it can be helpful in determining whether an elevated ALP level is sec-
ondary to hepatic pathology. Albumin is synthesized exclusively in the liver and can
be used as a general measure of hepatic synthetic function. Because chronic malnutri-
tion and acute injury/inflammation can decrease albumin synthesis these factors must
be taken into account when evaluating a low serum albumin level. Because of the re-
markable protein synthetic capacity of the liver, hypoalbuminemia as a marker of liv-
er disease lacks sensitivity, and tremendous decreases in hepatic function are required
to be reflected in albumin levels. In general, it is most helpful in chronic liver disease.
Clotting factors are largely synthesized in the liver, and abnormalities of clotting can
be a marker of diminished hepatic synthetic function. Measures of specific clotting
factors such as factors V and VII have been used to evaluate hepatic function in the
transplant population. The prothrombin time is the best test to measure the effects of
hepatic disease on clotting and is usually a marker of advanced chronic liver disease.
Hepatic pathology can also affect clotting through intravascular coagulation and vit-
amin K malabsorption.
Once screening tests, along with clinical findings, have suggested liver disease,
specific tests can be used to help elucidate the etiology and guide treatment if neces-
sary. Serologic studies for hepatitis are important to determine the presence of viral
hepatitis. Autoimmune antibodies are used to diagnose primary biliary cirrhosis (an-
timitochondrial), primary sclerosing cholangitis (antineutrophil), and autoimmune
hepatitis. a1 -Antitrypsin and ceruloplasmin levels assist in the diagnosis of a1 -
antitrypsin deficiency and Wilson’s disease, respectively. Tumor markers such as
AFP carcandinoembryonic antigen (CEA) can be helpful in the diagnosis and man-
agement of primary and metastatic tumors of the liver. The LFTs discussed earlier, in
general, are gross, nonspecific, and contain little, if any, prognostic value. Many at-
tempts have been made to formulate dynamic and quantitative tests of hepatic func-
tion based on the liver’s ability to clear various exogenously administered substances.
Despite many years of research, it still remains unclear whether these tests of hepatic
function are any better than scoring systems derived from simple blood tests and clin-
ical observations. The aminopyrine breath test is based on the clearance, by the hepat-
ic p450 system, of radiolabeled aminopyrine. A breath test measuring radiolabeled
CO2 as a breakdown product of aminopyrine is performed after administration at a
specified time. The results largely depend on the functional hepatic mass, which is
generally not depleted until end-stage liver disease. There are varying results of stud-
ies comparing the aminopyrine breath test to standard LFTs and scoring systems. Its
main value appears to be prognosis in chronic liver disease, but it is clearly not an ef-
fective test to detect subclinical hepatic dysfunction. Substances such as antipyrine
and caffeine can evaluate liver function in a similar way with similar results. The li-
docaine clearance test yields similar information to the aminopyrine test because it is
based on its clearance by the hepatic p450 test. Lidocaine clearance is dependent on
blood flow and a complex distribution process, but measurement of one of its me-
tabolites, monoethylglycinexylidide (MEGX), has greatly simplified the test. This test
has been shown to have some prognostic value in the transplant population. The ga-
lactose elimination test is based on the liver’s role in phosphorylating galactose and
converting it to glucose. The rate at which galactose is eliminated from the blood-
stream can be a measure of hepatic function. Problems related to this test are that the
enzymes involved are genetically heterogeneous and considerable extrahepatic me-
tabolism occurs. Additionally, multiple blood samples are necessary, making the test
cumbersome. The value of this test has also largely been in assessing prognosis in
chronic liver disease rather than screening. Indocyanine green is a dye removed by
the liver by a carrier-mediated process and excreted into bile. This dye is rapidly
cleared from the bloodstream and is not metabolized. This is the only test that has
been shown to have some prognostic ability in cirrhotic patients undergoing liver re-
section, although this is not universally demonstrated in studies, nor is it universally
accepted.
Lastly, a large number of scoring systems based on clinical observation and
standard blood tests have been proposed. The most commonly used system is Pugh’s
modification of the Child score. Although all of these systems are less than perfect
and not universally accepted, the Child-Pugh score is commonly used in cirrhotic pa-
tients who require liver surgery. Mortality and survival rates after hepatectomy have
been shown to correlate with this score but are not always related to liver failure.
Child’s B and C patients generally do not fare well after partial hepatectomy as com-
pared with Child’s A patients.
Child-Pugh Classification
No. of Points*
Factor
1 2 3
Bilirubin (mg/dL) <2 2–3 >3

Albumin (g/dL) >3.5 2.8–3.5 <2.8


Prothrombin time 1–3 4–6 >6
(increased seconds)
<1.7 1.71-2.20 >2.20
INR
Ascites None Slight Moderate

Encephalopathy None Minimal Advanced


* Grade A, 5–6 points; grade B, 7–9 points; grade C, 10–15 points.

Chronic liver disease is classified into Child-Pugh class A to C, employing the


added score from above.

Points Class One year survival Two year survival

5-6 A 100% 85%

7-9 B 81% 57%

10-15 C 45% 35%

SURGICAL COMPLICATIONS OF CIRRHOSIS AND PORTAL


HYPERTENSION
Cirrhosis is the end result of a variety of mechanisms causing hepatocellular in-
jury, including toxins (alcohol), viruses (hepatitis B and hepatitis C), prolonged cho-
lestasis (extrahepatic and intrahepatic), autoimmunity (lupoid hepatitis), and metabol-
ic disorders (hemochromatosis, Wilson’s disease, a1 -antitrypsin deficiency). Alt-
hough the mechanisms are diverse, the pathologic response is uniform: hepatocellular
necrosis followed by fibrosis and nodular regeneration. Each of these elements may
exist alone (necrosis, uncomplicated hepatitis; fibrosis, congenital hepatic fibrosis;
nodular regeneration, partial nodular transformation), but all three are required for the
development of cirrhosis. Cirrhosis, always a diffuse process, may be classified either
morphologically or by etiology. Alcoholic cirrhosis, which is usually micronodular,
and posthepatitic cirrhosis, which is generally macronodular, are the two most com-
mon varieties in the United States. Because the pathologic responses to various
mechanisms of hepatocellular injury are so similar, occasionally the cause cannot be
ascertained (cryptogenic cirrhosis). Cirrhosis causes two major phenomena: hepato-
cellular failure and portal hypertension. Even after the noxious agent is removed
(e.g., abstinence from alcohol), the disease may progress. Although the mechanism is
not clear, both ischemia, secondary to extensive fibrosis and intrahepatic and extrahe-
patic shunts, and autoimmune factors may play roles. The altered hepatic architecture
and perisinusoidal fibrosis cause increased hepatic vascular resistance, resulting in
portal hypertension and its associated complications of variceal hemorrhage, en-
cephalopathy, ascites, and hypersplenism. Autopsy studies suggest an incidence of
cirrhosis of between 3.5% and 5%. Only 10% to 15% of heavy drinkers develop al-
coholic cirrhosis. Because of the large number of alcoholic people in the United
States, as well as a significant percentage of patients with nonalcoholic causes of
chronic liver disease, cirrhosis presently ranks as the sixth leading cause of death be-
tween the ages of 35 and 54 years. Hepatic failure and variceal hemorrhage are the
first and second most common causes of death, respectively, in patients with cirrho-
sis. Historically, the treatment of cirrhosis has been the treatment of the complications
of portal hypertension. Medical treatment of cirrhosis with antifibrogenesis drugs,
such as colchicine, has been ineffective. In contrast, since 1980, the surgical man-
agement of chronic liver disease with hepatic transplantation has been highly success-
ful, with long-term survival rates generally above 70%. A major challenge to the phy-
sician or surgeon managing patients with cirrhosis is to determine when definitive
treatment (transplantation) rather than palliative treatment (e.g., interventions to pre-
vent recurrent variceal hemorrhage) should be applied.
Epidemiology
Cirrhosis of the liver accounts for approximately 90 % of all cases of portal hy-
pertension presenting in the West. In Eastern and tropical countries non-cirrhotic
causes predominate. Distinct geographical distributions are found for non-cirrhotic
portal fibrosis, schistosomiasis, and idiopathic portal hypertension. In children, extra-
hepatic portal vein obstruction is the main cause of upper gastrointestinal bleeding:
this accounts for 40 to 50 per cent of all cases of portal hypertension in those under
17 years old.
The development of hepatic fibrosis reflects an alteration in the normally bal-
anced processes of extracellular matrix production and degradation. Extracellular ma-
trix, the normal scaffolding for hepatocytes, is composed of collagens (especially
types I, III, and V), glycoproteins, and proteoglycans. Stellate cells, located in the
perisinusoidal space, are essential for the production of extracellular matrix. Stellate
cells, which were once known as Ito cells, lipocytes, or perisinusoidal cells, may be-
come activated into collagen-forming cells by a variety of paracrine factors. Such fac-
tors may be released by hepatocytes, Kupffer cells, and sinusoidal endothelium fol-
lowing liver injury. As an example, increased levels of the cytokine transforming
growth factor beta1 (TGF-beta1) are observed in patients with chronic hepatitis C and
those with cirrhosis. TGF-beta1, in turn, stimulates activated stellate cells to produce
type I collagen. Increased collagen deposition in the space of Disse (the space be-
tween hepatocytes and sinusoids) and the diminution of the size of endothelial fenes-
trae lead to the capillarization of sinusoids. Activated stellate cells also have contrac-
tile properties. Both capillarization and constriction of sinusoids by stellate cells con-
tribute to the development of portal hypertension. Future drug strategies to prevent
fibrosis may focus on reducing hepatic inflammation, inhibiting stellate cell activa-
tion, inhibiting the fibrogenic activities of stellate cells, and stimulating matrix degra-
dation.
Portal hypertension is defined as a portal vein pressure above the normal range
of 5 to 8 mm Hg (68 – 109 mm H2O). Portal hypertension may also be defined by the
hepatic vein - portal vein pressure gradient, which is greater than 5 mm Hg in portal
hypertensive states.
The ‘backward' and ‘forward' theories of portal hypertension
It is currently believed that the principal and initial abnormality is increased
vascular resistance to portal flow and that portal hypertension is then maintained by
increased blood flow into the portal circulation, a phenomenon which has been con-
firmed conclusively both experimentally and clinically. Blood flow to the stomach,
spleen, and intestines is increased by 50 per cent in portal hypertension: this is
achieved largely by splanchnic vasodilatation and raised cardiac output.
Causes of increased resistance to flow are described as follows:
 Prehepatic
 Intrahepatic
 Predominantly presinusoidal
 Predominantly sinusoidal and/or postsinusoidal
 Posthepatic
 Mixed
The causes of prehepatic portal hypertension are:
 Portal vein thrombosis
 Splenic vein thrombosis
 Congenital atresia or stenosis of portal vein
 Extrinsic compression (tumors)
 Splanchnic arteriovenous fistula
The causes of intrahepatic portal hypertension are,
A. predominantly presinusoidal:
 Schistosomiasis (early stage)
 Primary biliary cirrhosis (early stage)
 Idiopathic portal hypertension (early stage)
 Nodular regenerative hyperplasia: Pathogenesis probably is obliterative
venopathy. The presence of nodules that press on the portal system also has
been postulated to play a role, although nodularity is present in most cases
without clinical evidence of portal hypertension.
 Myeloproliferative diseases: These act by direct infiltration by malignant cells.
 Polycystic disease
 Hepatic metastasis
 Granulomatous diseases (sarcoidosis, tuberculosis): Clinical liver dysfunction
is rare in sarcoidosis. Portal hypertension is an unusual, although well-
recognized manifestation of hepatic sarcoidosis. Sarcoid granulomas frequently
localize in the portal areas, resulting in injury to the portal veins.
B. predominantly sinusoidal and/or postsinusoidal:
 Hepatic cirrhosis
 Acute alcoholic hepatitis
 Schistosomiasis (advanced stage)
 Primary biliary cirrhosis (advanced stage)
 Idiopathic portal hypertension (advanced stage)
 Acute and fulminant hepatitis
 Congenital hepatic fibrosis
 Vitamin A toxicity: Noncirrhotic portal fibrosis is observed with various toxic
injuries, and one of these includes vitamin A toxicity. This probably is due to
vascular injury.
 Peliosis hepatitis
 Venoocclusive disease
The causes of posthepatic portal hypertension are:
 Inferior vena cava (IVC) obstruction
 Right heart failure
 Constrictive pericarditis
 Tricuspid regurgitation
 Budd-Chiari syndrome
The causes of hyperdynamic portal hypertension are:
 Arterial-portal venous fistula
 Increased portal blood flow
 Increased splenic flow

In patients with cirrhosis the overall mortality rate from oesophageal variceal
bleeding is about 40 %. If the patient recovers from the acute bleed the risk of recur-
rent haemorrhage during the same hospital stay is 60 %; this increases to more than
80 % at 2 years. The long-term survival of patients with cirrhosis following variceal
bleeding is poor, ranging from 6 to 35 % at 5 years. It is important to emphasize,
however, that only about 30 % of patients with cirrhosis will ever experience such
bleeding. The remaining die of liver failure, cachexia, and infection.

Presentation of portal hypertension


Portal hypertension per se has little in the way of clinical features, and usually
presents with complications such as:
 variceal haemorrhage (in only 30 % of these patients)
 ascites,
 hypersplenism.
A clinical history and findings indicative of liver disease, particularly cirrhosis.
The clinical history is therefore of major importance when portal hypertension is sus-
pected. It is very important careful questioning of the patient about alcoholism, past
jaundice or hepatitis, exposure to hepatotoxins, blood transfusion, drug abuse, neona-
tal or intra-abdominal sepsis, myeloproliferative disorder, the use of oral contracep-
tives or other sex hormone-containing medications, a history of myeloproliferative
disorder may indicate the Budd–Chiari syndrome.
Clinical findings
• the stigmata of liver disease:
– palmar erythema,
– spider naevi,
– white nails,
– jaundice,
– portasystemic encephalopathy.
• enlarged spleen – splenomegaly
• abdominal wall veins
– the caput medusae – cirrhosis
– at the flanks - Budd–Chiari syndrome
• venous hum
• the liver may be enlarged or shrunken
• a firm nodular liver indicates cirrhosis, while a smooth soft liver is suggestive
of extrahepatic portal obstruction
• ascites
• peripheral oedema
• rectal varices

Clinical features of hepatic encephalopathy


The symptoms of hepatic encephalopathy may range from mild to severe and
may be observed in as many as 70% of patients with cirrhosis. Symptoms are graded
on the following scale:

• Grade 0 - Subclinical; normal mental status, but minimal changes in memory,


concentration, intellectual function, coordination
• Grade 1 - Mild confusion, euphoria or depression, decreased attention, slowing
of ability to perform mental tasks, irritability, disorder of sleep pattern (i.e., in-
verted sleep cycle)

• Grade 2 - Drowsiness, lethargy, gross deficits in ability to perform mental


tasks, obvious personality changes, inappropriate behavior, intermittent disori-
entation (usually for time)

• Grade 3 - Somnolent but arousable, unable to perform mental tasks, disorienta-


tion to time and place, marked confusion, amnesia, occasional fits of rage,
speech is present but incomprehensible

• Grade 4 - Coma, with or without response to painful stimuli

Patients with mild and moderate hepatic encephalopathy demonstrate decreased


short-term memory and concentration on mental status testing. Findings upon
physical examination include asterixis and fetor hepaticus.

Investigation
 Laboratory investigations
 Endoscopy
 Plain radiographs of the abdomen and chest
 Barium swallow
 Ultrasound and duplex scanning
 Magnetic resonance imaging
 Computed axial tomography (CT scan)
 Diagnostic angiography
 Coeliac and superior mesenteric angiography
 Splenic venography
 Transhepatic venography
 Inferior vena cava and hepatic venography
 Carbon dioxide angiography
Endoscopy allows rapid and safe confirmation of the presence of gastroesopha-
geal varices and source of bleeding from the upper gastrointestinal tract.
Plain radiographs of the abdomen and chest are of limited value in the man-
agement of portal hypertension, but they may yield useful information. In an ab-
dominal radiograph the size of the liver and spleen may be assessed and rarely, gas
shadows in the portal circulation may be detected in patients with enterocolitis, intes-
tinal infection, or disseminated intravascular coagulation syndromes.
Percutaneous liver biopsy is a useful technique for establishing the cause of
cirrhosis and for assessing activity of the liver disease. Percutaneous liver biopsy
should not be done when either coagulopathy or moderate ascites is present. In these
situations, liver tissue can be obtained by means of a transjugular venous approach or
laparoscopy. Laparoscopic biopsy reduces the false-negative rate for diagnosing cir-
rhosis as compared with blind biopsy techniques.
Angiography no longer plays a major role in the investigation of portal hyper-
tension except where spiral CT or MRI facilities are not available. However, one ma-
jor advantage of this catheter-based technique remains the capacity to measure hepat-
ic blood flow, free and wedged hepatic pressures, and inferior vena cava pressures.
Also, in many parts of the world, visualization of the portal system, particularly
for identification of major portasystemic collaterals and provision of a map to allow
planning of surgical intervention, continues to be by classical angiography. Therefore
the various techniques of imaging of the portal circulation by angiography will be de-
scribed:
• Superior mesenteric angiogram. The venous phase shows a patency of portal
vein.
• Splenic venogram. The splenic injection gives the best definition of the portal
venous circulation.
• Inferior vena cavogram. The anteroposterior and lateral views of the inferior
vena cava may confirm the diagnosis of Budd–Chiari syndrome, permit meas-
uring of free and wedged hepatic pressures, and inferior vena cava pressures.

Hepatic Hemodynamic Assessment


In patients with alcoholic cirrhosis and many varieties of nonalcoholic cirrho-
sis, portal pressure can be indirectly estimated by measurement of hepatic venous
wedge pressure (HVWP). Because HVWP is normal in patients with presinusoidal
portal hypertension, portal pressure in these patients can be measured only directly by
transhepatic or umbilical venous cannulation of the portal venous system or by percu-
taneous puncture of the spleen. The portal pressure should be expressed as the portal
pressure gradient, which is the difference between the portal pressure and the inferior
vena cava pressure. It is an important measurement because a gradient in excess of 10
mm Hg is necessary for varices to form and a pressure higher than 12 mm Hg is re-
quired for varices to bleed. Because splanchnic venous thrombosis may be the cause
of portal hypertension or develop as a result of cirrhosis, portal venous anatomy
should be defined before performing a portosystemic shunt operation. Although se-
lective visceral angiography has been the most frequently used method for visualiza-
tion of the portal venous system and for qualitative estimation of hepatic portal perfu-
sion, this relatively invasive approach is presently being replaced in many institutions
by less invasive methods such as CT angiography, Doppler ultrasonography, and
magnetic resonance imaging. CT angiograms can delineate the location, size, and pa-
tency status of all veins (e.g., splenic vein and left renal vein) to be used in creation of
a portosystemic shunt. Magnetic resonance imaging has also been successfully used
to visualize the portal venous circulation. This technique is particularly appropriate
for patients with an allergy to radiopaque contrast agent. Doppler ultrasonography is a
noninvasive technique for assessment of portal venous patency, direction of portal
flow, and shunt patency status. Because of its noninvasiveness, Doppler ultrasonog-
raphy has become a standard for the evaluation of most patients with chronic liver
disease because direction of portal flow and its velocity can be diagnostic of associat-
ed portal hypertension. Ultrasound is also useful for assessing liver size, spleen size,
and the presence of liver masses. It can also detect ascites in its earliest stages (=100
mL). Doppler ultrasound is less accurate in assessing superior mesenteric and splenic
vein anatomy and flow characteristics. Likewise, Doppler ultrasonography usually
accurately assesses patency status of surgically constructed shunts unless there is
overlying bowel gas. Doppler ultrasound has also been used to evaluate patency and
narrowing of transjugular intrahepatic portosystemic shunts (TIPS), but it is less ac-
curate than direct cannulation via a systemic venous approach.
We should treat not portal hypertension but its complications!
Variceal hemorrhage
Bleeding from esophagogastric varices is the single most life-threatening com-
plication of portal hypertension, responsible for about one third of all deaths in pa-
tients with cirrhosis. Overall, acute variceal bleeding is associated with a mortality
rate of about 25% to 30%. Approximately one half of the deaths are due to uncon-
trolled bleeding. The risk for death from bleeding is mainly related to the underlying
hepatic functional reserve. Patients with extrahepatic portal venous obstruction and
normal hepatic function rarely die of bleeding varices, whereas those with decompen-
sated cirrhosis (Child-Pugh class C) may face a mortality rate in excess of 50%. The
greatest risk for rebleeding from varices is within the first few days after the onset of
hemorrhage; the risk declines rapidly between then and 6 weeks after hemorrhage on-
set, when it returns to the prehemorrhage risk level.
Treatment modalities
Therapy for portal hypertension and variceal bleeding has evolved during the
past 100 years. The many treatment modalities available suggest that no single thera-
py is entirely satisfactory for all patients or for all clinical situations. Sequential ther-
apies are often necessary. Nonoperative treatments are generally preferred for acutely
bleeding patients because they are often high operative risks because of decompen-
sated hepatic function. Therapies that are effective (a low rebleeding rate) and mini-
mally alter hepatic physiology are optimal for long-term prevention of recurrent
bleeding. Only treatments associated with minimal morbidity and mortality can be
considered for prophylaxis because many patients will be treated unnecessarily (only
one third to one half of patients with varices eventually bleed).
Treatment of the Acute Bleeding Episode
Because many patients with acute variceal bleeding have decompensated he-
patic function secondary to either recent alcoholism or hypotension, they are at high
risk for emergency surgical intervention. In addition, these patients often have other
complications of chronic liver disease, such as encephalopathy, ascites, coagulopathy,
and malnutrition. Therefore, emergency treatment should be nonoperative whenever
possible. Endoscopic treatment (sclerosis or ligation), which has become the mainstay
of nonoperative treatment of acute hemorrhage in most centers, controls bleeding in
more than 85% of patients, allowing an interval of medical management for im-
provement of hepatic function, resolution of ascites and encephalopathy, and en-
hancement of nutrition before definitive treatment for prevention of recurrent bleed-
ing. Pharmacotherapy can be initiated in any hospital, and some trials suggest that it
is just as effective as endoscopic treatment. Balloon tamponade, which is infrequently
used, can be life-saving in patients with exsanguinating hemorrhage and when the
other nonoperative methods are not successful. Transjugular intrahepatic portosys-
temic shunt (TIPS) has replaced operative shunts for managing acute variceal bleed-
ing when pharmacotherapy and endoscopic treatment fail to control bleeding. Emer-
gency surgical intervention in most centers is reserved for selected patients who are
not TIPS candidates.
A. Pharmacotherapy
Vasopressin, which is a potent splanchnic vasoconstrictor, has been the most
commonly used drug in the acute setting and controls hemorrhage in about half of pa-
tients. A meta-analysis of multiple trials has shown vasopressin to be more effective
than placebo. Vasopressin is usually administered intravenously as a bolus dose of 20
units over 20 minutes and then as a continuous infusion of 0.2 to 0.4 unit/minute. Be-
cause vasopressin also constricts systemic arterioles, it frequently causes hyperten-
sion, bradycardia, decreased cardiac output, and coronary vasoconstriction. There-
fore, the use of this drug should be confined to the intensive care unit, where the pa-
tient can be appropriately monitored. Because of the adverse systemic effects of vas-
opressin, nitroglycerin should be simultaneously infused at an initial rate of 40
μg/minute, which should then be titrated to achieve blood pressure control. The com-
bination of vasopressin and nitroglycerin may also be more effective than vasopressin
alone in controlling variceal hemorrhage. Randomized trials have shown that somato-
statin and its longer-acting analogue octreotide are as efficacious as endoscopic
treatment for control of acute variceal bleeding. These agents are also associated with
fewer adverse side effects than vasopressin. Because of their ease of administration
and effectiveness, these newer drugs may return pharmacotherapy to a more central
role in the treatment of acute portal hypertensive bleeding, especially when endoscop-
ic treatment is unlikely to be effective (failed chronic endoscopic therapy, gastric var-
ices, and PHG). Somatostatin is administered as a 250-μg intravenous bolus, followed
by a continuous infusion of 250 μg/hour for 2 to 4 days. Octreotide is given as an in-
travenous bolus of 50 μg followed by an infusion of 25 to 50 μg/hour for a similar
length of time. Because of the minimal adverse effects and ease of administration, oc-
treotide is now commonly used as an adjunct to endoscopic therapy.
B. Balloon Tamponade
The major advantages of variceal tamponade with the Sengstaken-Blakemore
tube are immediate cessation of bleeding in more than 85% of patients and wide-
spread availability of this device, including small community hospitals. Significant
disadvantages of balloon are frequent recurrent hemorrhage in up to 50% of patients
after balloon deflation, considerable discomfort for the patient, and a high incidence
of serious complications when the device is used by inexperienced personnel. The po-
tentially lethal complications of esophageal perforation secondary to intraesophageal
inflation of the gastric balloon, ischemic necrosis of the esophagus secondary to over-
inflation of the esophageal balloon, and aspiration can be avoided by using balloon
tamponade only in an intensive care unit and adhering to a strict protocol. Controlled
trials have demonstrated that balloon tamponade is as effective as pharmacotherapy
and endoscopic therapy in controlling acute variceal bleeding. Because of the effec-
tiveness of endoscopic treatment and pharmacotherapy for acute variceal bleeding,
balloon tamponade is infrequently required. It may be life-saving, however, when ex-
sanguinating hemorrhage prevents acute endoscopic treatment and in patients in
whom sclerotherapy has failed and who do not respond to pharmacotherapy. Because
balloon deflation is followed by a high rebleeding rate, definitive treatment, such as
endoscopic therapy, TIPS, or operation, should be planned for most patients in whom
the Sengstaken-Blakemore tube is used.
C. Endoscopic Treatment
Endoscopic treatment (variceal sclerosis or ligation) is the most commonly
used therapy for both management of the acute bleeding episode and prevention of
recurrent hemorrhage. In the acute setting, sclerotherapy and band ligation have been
shown to be equally efficacious. Both techniques require a skilled endoscopist and
stop bleeding in 80% to 90% of patients. Both intravariceal and paravariceal tech-
niques of sclerosant injection are used, and often these two techniques are purposeful-
ly or inadvertently combined. The most commonly used sclerosants in the United
States are sodium morrhuate and sodium tetradecyl sulfate. When experienced per-
sonnel are available, the initial sclerotherapy injections can often be done during the
endoscopy at which diagnosis of variceal bleeding is made. Each varix is usually in-
jected with 1 to 2 mL of sclerosant just above the esophagogastric junction and 5 cm
proximal to it. Alternatively, each varix can be ligated with a rubber band, as shown
in Figure 51–6. A subsequent treatment session is planned for 4 to 6 days later. Addi-
tional endoscopic treatments depend on the effectiveness of the initial treatments in
controlling bleeding and on whether endoscopic therapy has been selected as defini-
tive treatment for that patient. Minor complications of sclerotherapy, including ret-
rosternal chest pain, esophageal ulceration, and fever, occur commonly. More serious
complications, which account for the 1% to 3% mortality rate of this procedure, are
esophageal perforation, worsening of variceal hemorrhage, and aspiration pneumonit-
is. Failure of endoscopic treatment should be declared when two sessions fail to con-
trol hemorrhage. Unless urgent surgery is performed in such patients, the mortality
rate exceeds 60%.
D. Transjugular Intrahepatic Portosystemic Shunt
TIPS is a technique that accomplishes portal decompression without an opera-
tion. Because of the complexity of the procedure, an experienced interventional radi-
ologist is required. Access is gained to a major intrahepatic portal venous branch
through puncture through a hepatic vein. A parenchymal tract between hepatic and
portal veins is then created with a balloon catheter, and a 10-mm expandable metal
stent is inserted, thereby creating the shunt. In large series, the success rate of TIPS
has been more than 95%, but experience with this technique is limited in acutely
bleeding patients, who generally make up only a small fraction of patients receiving
TIPS. At the present time, TIPS should not be recommended as initial therapy for
acute variceal hemorrhage but should be used only after less invasive treatments, such
as endoscopic therapy and pharmacotherapy, have failed to control bleeding. TIPS is
effective in stopping bleeding in this setting. Mortality is related to the status of he-
patic function. One clear indication for TIPS is as a short-term bridge to liver trans-
plantation for patients in whom endoscopic treatment has failed. In addition to con-
trolling bleeding, advantages in this situation are that the lower portal pressure may
make the transplantation operation easier and that the shunt is removed when the re-
cipient liver is excised. Patients with advanced hepatic functional decompensation
(Child’s class C), even those who are not transplantation candidates, may be better
served by TIPS than by an emergency operation when less invasive approaches fail to
control bleeding. Hemodynamic studies suggest that TIPS is a nonselective shunt, and
several investigations have demonstrated a similar frequency of encephalopathy after
TIPS as has been previously reported after nonselective shunts. Another disadvantage
of the procedure is that shunt stenosis or occlusion develops in as many as half of pa-
tients within 1 year of TIPS insertion. This situation can often be remedied by repeat-
ed angiographic intervention, however. Absolute contraindications to TIPS include
right-sided heart failure and polycystic liver disease. Relative contraindications to the
procedure are portal vein thrombosis, hypervascular liver tumors, and encephalopa-
thy, which can be worsened by diversion of portal flow.
E. Emergency Surgery
Although nonoperative therapies are effective in most patients with acute vari-
ceal bleeding, emergency operation should be promptly done when less invasive
measures fail to control hemorrhage or are not indicated. The most common situa-
tions requiring either urgent or emergency surgery are failure of acute endoscopic
treatment, failure of long-term endoscopic therapy, hemorrhage from gastric varices
or PHG, and failure of TIPS placement. In most institutions, TIPS has become the
preferred treatment for acute variceal bleeding when pharmacotherapy and endoscop-
ic treatment have failed, with operative procedures being reserved for those situations
in which TIPS is not indicated or not available. Selection of the appropriate emergen-
cy operation should mainly be guided by the experience of the surgeon. Esophageal
transection with a stapling device is rapid and relatively simple, but rebleeding rates
after this procedure are high, and there is little evidence that operative mortality rates
are less than after surgical portal decompression. A commonly performed shunt oper-
ation in the emergency setting is the portacaval shunt because it rapidly and effective-
ly decompresses the portal venous circulation. Impressive results have been achieved
by Orloff and associates, but not by others, when the emergency portacaval shunt is
used as routine therapy for acute variceal bleeding. In patients who are not actively
bleeding at the time of surgery and in those in whom bleeding is temporarily con-
trolled by pharmacotherapy or balloon tamponade, a more complex operation, such as
the distal splenorenal shunt, may be appropriate. The major disadvantage of emergen-
cy surgery is that operative mortality rates exceed 25% in most reported series. Early
postoperative mortality is usually related to the status of hepatic functional reserve
rather than to the type of emergency operation selected.
F. Prevention of Recurrent Hemorrhage
After a patient has bled from varices, the likelihood of a repeat episode exceeds
70%. Because most patients with variceal hemorrhage have chronic liver disease, the
challenge of long-term management is both prevention of recurrent bleeding and
maintenance of satisfactory hepatic function. Options available for definitive treat-
ment include pharmacotherapy, chronic endoscopic treatment, TIPS, three hemody-
namic types of shunt operations (nonselective, selective, and partial), a variety of
nonshunt procedures, and hepatic transplantation. The most effective treatment regi-
men usually uses two or more of these therapies in sequence. In most institutions, ini-
tial treatment consists of pharmacotherapy or endoscopic therapy with portal decom-
pression by means of TIPS or an operative shunt reserved for failures of first-line
treatment. Hepatic transplantation is used for patients with end-stage liver disease.
Portosystemic Shunts
Portosystemic shunts are clearly the most effective means of preventing recur-
rent hemorrhage in patients with portal hypertension. These procedures are effective
because they all, to some degree, decompress the portal venous system by shunting
portal flow into the low-pressure systemic venous system. Diversion of portal blood,
however, that contains hepatotropic hormones, nutrients, and cerebral toxins is also
responsible for the adverse consequences of shunt operations, namely portosystemic
encephalopathy and accelerated hepatic failure. Depending on whether they com-
pletely decompress, compartmentalize, or partially decompress the portal venous cir-
culation, portosystemic shunts can be classified as nonselective, selective, or partial.
In addition to variceal decompression, the goal of selective and partial portosystemic
shunts is preservation of hepatic portal perfusion, thereby preventing or minimizing
the adverse consequences of these procedures.
Nonselective Shunts
Commonly used varieties of nonselective shunts, all of which completely divert
portal flow, include the end-to-side portacaval shunt (Eck’s fistula), the side-to-side
portacaval shunt, large-diameter interposition shunts, and the conventional
splenorenal shunt. The end-to-side portacaval shunt is the prototype of the combined
survival data from the four controlled investigations of the therapeutic portacaval
shunt (performed in patients with prior variceal hemorrhage). The most common
causes of death in medically treated and shunted patients were rebleeding and accel-
erated hepatic failure, respectively. Although no survival advantage could be demon-
strated for shunt patients, all of these studies had a crossover bias in favor of medical-
ly treated patients, several of whom received a shunt when they developed intractable
recurrent variceal hemorrhage. In addition, nearly all of the trial patients had alcohol-
ic cirrhosis; therefore, these results do not necessarily apply to other causes of portal
hypertension. Other important findings of these randomized trials include reliable
control of bleeding in shunted patients, variceal rebleeding in more than 70% of med-
ically treated patients, and spontaneous, often severe, encephalopathy in 20% to 40%
of shunted patients. All of the other nonselective shunts maintain continuity of the
portal vein, thereby connecting the portal and systemic venous systems in a side-to-
side fashion. Therefore, these procedures decompress both the splanchnic venous cir-
culation and the intrahepatic sinusoidal network. Because the liver and intestines are
both important contributors to ascites formation, side-to-side portosystemic shunts are
the most effective shunt procedures for relieving ascites as well as preventing recur-
rent variceal bleeding. Because they completely divert portal flow like the end-to-side
portacaval shunt, however, side-to-side shunts also accelerate hepatic failure and lead
to frequent postshunt encephalopathy. Synthetic grafts or autogenous vein may be in-
terposed between the portal and systemic venous circulations at a variety of locations.
A major disadvantage of prosthetic interposition shunts is a high graft thrombosis rate
that approaches 35% during the late postoperative interval. This problem can be
avoided by using autogenous vein (internal jugular vein) rather than a prosthetic graft.
On the other hand, advantages of these shunts are that they are relatively easy to con-
struct; the hepatic hilum is avoided, thereby making subsequent liver transplantation
less complicated; and they can be easily occluded if intractable postshunt encephalo-
pathy develops. The conventional splenorenal shunt consists of anastomosis of the
proximal splenic vein to the renal vein. Splenectomy is also done. Because the small-
er proximal rather than the larger distal end of the splenic vein is used, shunt throm-
bosis is more common after this procedure than after the distal splenorenal shunt.
Although early series noted that postshunt encephalopathy was less common after the
conventional splenorenal shunt than after the portacaval shunt, subsequent analyses
have suggested that this low frequency of encephalopathy was probably a result of
restoration of hepatic portal perfusion after shunt thrombosis developed in many pa-
tients. A conventional splenorenal shunt that is of sufficient caliber to remain patent
gradually dilates and eventually causes complete portal decompression and portal
flow diversion. A purported advantage of the procedure is that hypersplenism is elim-
inated by splenectomy. The thrombocytopenia and leukopenia that accompany portal
hypertension, however, are rarely of clinical significance, making splenectomy an
unnecessary procedure in most patients. In summary, nonselective shunts effectively
decompress varices. Because of complete portal flow diversion, however, they are
complicated by frequent postoperative encephalopathy and accelerated hepatic fail-
ure. Side-to-side nonselective shunts effectively relieve ascites and prevent variceal
hemorrhage. Presently, the only indications for nonselective shunts are in the emer-
gency setting when nonoperative means to control bleeding have failed, in patients
with both variceal hemorrhage and medically intractable ascites, and as a bridge to
hepatic transplantation in patients in whom bleeding is not controlled by endoscopic
treatment or TIPS.
Selective Shunts
The hemodynamic and clinical shortcomings of nonselective shunts stimulated
development of the concept of selective variceal decompression. In 1967, Warren and
colleagues introduced the distal splenorenal shunt; and in the following year, Inoku-
chi and associates reported their initial results with the left gastric vena caval shunt.
The latter procedure consists of interposition of a vein graft between the left gastric
(coronary) vein and the inferior vena cava and, thus, directly and selectively decom-
presses esophagogastric varices. Only a few patients with portal hypertension, how-
ever, have appropriate anatomy for this operation; experience with it has been limited
to Japan, and no controlled trials have been conducted. The distal splenorenal shunt
consists of anastomosis of the distal end of the splenic vein to the left renal vein and
interruption of all collateral vessels, such as the coronary and gastroepiploic veins,
connecting the superior mesenteric and gastrosplenic components of the splanchnic
venous circulation. This results in separation of the portal venous circulation into a
decompressed gastrosplenic venous circuit and a high-pressure superior mesenteric
venous system that continues to perfuse the liver. Although the procedure is techni-
cally demanding, it can be mastered by most well-trained surgeons who are knowl-
edgeable in the principles of vascular surgery. Not all patients are candidates for the
distal splenorenal shunt. Because sinusoidal and mesenteric hypertension is main-
tained and important lymphatic pathways are transected during dissection of the left
renal vein, the distal splenorenal shunt tends to aggravate rather than relieve ascites.
Thus, patients with medically intractable ascites should not undergo this procedure.
However, the larger population of patients who develop transient ascites after resusci-
tation from a variceal hemorrhage are candidates for a selective shunt. Another con-
traindication to a distal splenorenal shunt is prior splenectomy. A splenic vein diame-
ter of less than 7 mm is a relative contraindication to the procedure because the inci-
dence of shunt thrombosis is high when using a small-diameter vein. Although selec-
tive variceal decompression is a sound physiologic concept, the distal splenorenal
shunt remains controversial after an extensive clinical experience spanning more than
35 years. The key questions regarding this procedure are: How effective is it in pre-
serving hepatic portal perfusion? Is it superior to nonselective shunts with respect to
duration or quality of survival? Although the distal splenorenal shunt results in portal
flow preservation in more than 85% of patients during the early postoperative inter-
val, the high-pressure mesenteric venous system gradually collateralizes to the low-
pressure shunt, resulting in loss of portal flow in about half of patients by 1 year. The
degree and duration of portal flow preservation depend on both the cause of portal
hypertension and the technical details of the operation (extent to which mesenteric
and gastrosplenic venous circulations are separated). Henderson and coworkers have
shown that portal flow is maintained in most patients with nonalcoholic cirrhosis and
noncirrhotic portal hypertension (e.g., portal vein thrombosis). In contrast, portal flow
rapidly collateralizes to the shunt in patients with alcoholic cirrhosis. Modification of
the distal splenorenal shunt by purposeful or inadvertent omission of coronary vein
ligation results in early loss of portal flow. Even when all major collateral vessels are
interrupted, portal flow may be gradually diverted through a pancreatic collateral
network (pancreatic siphon). This pathway can be discouraged by dissecting the full
length of the splenic vein from the pancreas (splenopancreatic disconnection), which
results in better preservation of hepatic portal perfusion, especially in patients with
alcoholic cirrhosis. However, this extension of the procedure makes it technically
more challenging, which may be a significant disadvantage in an era when fewer
shunts are being done because of increased use of endoscopic therapy, TIPS, and he-
patic transplantation. Six of the seven controlled comparisons of the distal
splenorenal shunt with nonselective shunts have included predominantly alcoholic
cirrhotic patients. None of these trials has demonstrated an advantage to either proce-
dure with respect to long-term survival. Three of the studies have found a lower fre-
quency of encephalopathy after the distal splenorenal shunt, whereas the other trials
have shown no difference in the incidence of this postoperative complication. In con-
trast to survival, encephalopathy is a subjective endpoint that was assessed with a va-
riety of methods in the different trials. Another important endpoint in comparing
treatments for variceal hemorrhage is the effectiveness with which recurrent bleeding
is prevented. In nearly all uncontrolled and controlled series of the distal splenorenal
shunt, this procedure has been equivalent to nonselective shunts in preventing recur-
rent hemorrhage. Mainly because of these inconsistent results of the controlled trials,
there is no consensus as to which shunting procedure is superior in patients with al-
coholic cirrhosis. Because the quality of life (encephalopathy rate) was significantly
better in the distal splenorenal shunt group in three of the trials, however, there ap-
pears to be an advantage to selective variceal decompression even in this population.
Considerably fewer data are available regarding selective shunting in nonalcoholic
cirrhosis and in noncirrhotic portal hypertension. Because hepatic portal perfusion af-
ter the distal splenorenal shunt is better preserved in these disease categories, one
might expect improved results. A single controlled trial in patients with schistosomia-
sis (presinusoidal portal hypertension) demonstrated a lower frequency of encephalo-
pathy after the distal splenorenal shunt than after a conventional splenorenal shunt
(nonselective). The large Emory University series of the distal splenorenal shunt has
demonstrated better survival in patients with nonalcoholic cirrhosis than in those with
alcoholic cirrhosis. However, this has not been a consistent finding in all centers in
which the distal splenorenal shunt is performed. Several controlled trials have also
compared the distal splenorenal shunt with chronic endoscopic therapy. In these in-
vestigations, recurrent hemorrhage was more effectively prevented by selective
shunting than by sclerotherapy, but hepatic portal perfusion was maintained in a sig-
nificantly higher fraction of patients undergoing sclerotherapy. Despite this hemody-
namic advantage, encephalopathy rates have been similar after both therapies. The
two North American trials were dissimilar with respect to the effect of these treat-
ments on long-term survival. Sclerotherapy with surgical rescue for the one third of
sclerotherapy failures resulted in significantly better survival than selective shunt
alone in one study. In this investigation, 85% of sclerotherapy failures could be sal-
vaged by surgery. In contrast, a similar investigation conducted in a sparsely populat-
ed area (Intermountain West and Plains) showed superior survival after the distal
splenorenal shunt. Only 31% of sclerotherapy failures could be salvaged by surgery
in this trial. The survival results of these two studies suggest that endoscopic therapy
is a rational, initial treatment for patients who bleed from varices if sclerotherapy
failure is recognized and such patients promptly undergo surgery or TIPS. However,
patients living in remote areas are less likely to be salvaged by shunt surgery when
endoscopic treatment fails, and a selective shunt may be preferable initial treatment
for such patients. In a nonrandomized comparison to TIPS, the distal splenorenal
shunt had lower rates of recurrent bleeding, encephalopathy, and shunt thrombosis.
Ascites was less prevalent after TIPS. A multicenter randomized trial comparing
TIPS and the distal splenorenal shunt for the elective treatment of variceal bleeding in
good-risk cirrhotic patients is ongoing, but results are not yet available.
Partial Shunts
The objectives of partial and selective shunts are the same: (1) effective de-
compression of varices, (2) preservation of hepatic portal perfusion, and (3) mainte-
nance of some residual portal hypertension. Initial attempts at partial shunting con-
sisted of small diameter vein-to-vein anastomoses, but these generally either throm-
bosed or dilated with time, thereby becoming nonselective shunts. More recently, a
small-diameter interposition portacaval shunt using a polytetrafluoroethylene graft,
combined with ligation of the coronary vein and other collateral vessels, has been de-
scribed. When the prosthetic graft is 10 mm or less in diameter, hepatic portal perfu-
sion is preserved in most patients, at least during the early postoperative interval. Ear-
ly experience with this small-diameter prosthetic shunt is that fewer than 15% of
shunts have thrombosed, and most of these have been successfully opened by inter-
ventional radiologic techniques. A prospective, randomized trial of partial (8 mm in
diameter) and nonselective (16 mm in diameter) interposition portacaval shunts has
shown a lower frequency of encephalopathy after the partial shunt but similar surviv-
al after both types of shunts. The number of patients included in this investigation
was small, however, and further trials need to be done to confirm this finding. In an-
other controlled trial, the small-diameter interposition shunt was discovered to have a
lower overall failure rate than TIPS.
Nonshunt Operations
The objectives of nonshunt procedures are either ablation of varices or, more
commonly, extensive interruption of collateral vessels connecting the high-pressure
portal venous system with the varices. One exception is splenectomy, which is effec-
tive in left-sided portal hypertension caused by splenic vein thrombosis. The simplest
nonshunt operation is transection and reanastomosis of the distal esophagus with a
stapling device. This operation, which has generally been used in the emergency set-
ting, is frequently followed by recurrent hemorrhage. The most effective nonshunt
operation is extensive esophagogastric devascularization combined with esophageal
transection and splenectomy. The Sugiura procedure preserves the coronary and
paraesophageal veins to maintain a portosystemic collateral pathway and thus dis-
courage re-formation of varices. In Japan, the results with this operation have been
excellent, with rebleeding rates of less than 10%. Extensive devascularization proce-
dures, however, have generally been less successful in North American patients with
alcoholic cirrhosis. Long-term follow-up in American series has revealed rebleeding
rates of 35% to 55%, which are similar to the endoscopic therapy experience. In
many centers, esophagogastric devascularization procedures are mainly used for pa-
tients who are not suitable for shunts but with diffuse splanchnic venous thrombosis
and for patients with distal splenorenal shunt thrombosis.
Hepatic Transplantation
Liver transplantation is not a treatment for variceal bleeding, per se, but rather
needs to be considered for all patients who present with end-stage hepatic failure
whether or not it is accompanied by bleeding. Transplantation in patients who have
bled secondary to portal hypertension is the only therapy that addresses the underly-
ing liver disease in addition to providing reliable portal decompression. Because of
economic factors and a limited supply of donor organs, liver transplantation is not
available to all patients. Additionally, transplantation is not indicated for some of the
more common causes of variceal bleeding, such as schistosomiasis (normal liver
function) and active alcoholism (noncompliance). There is accumulating evidence
that variceal bleeders with well-compensated hepatic functional reserve (Child’s clas-
ses A and B+) are better served by nontransplantation strategies initially. The first-
line treatment for such patients should be pharmacologic and endoscopic therapy,
with portal decompression by means of an operative shunt or TIPS reserved for those
who fail first-line therapy and for circumstances in which pharmacologic or endo-
scopic treatment would be risky (e.g., patients with gastric varices and those geo-
graphically separated from tertiary medical care). Patients with variceal bleeding who
are transplantation candidates include nonalcoholic cirrhotic patients and abstinent
alcoholic cirrhotic patients with either limited hepatic functional reserve (Child’s
classes B and C) or a poor quality of life secondary to their disease (e.g., encephalo-
pathy, fatigue, or bone pain). In these patients, the acute hemorrhage should be treat-
ed with endoscopic therapy and the patient’s transplantation candidacy should be
immediately activated. If sclerotherapy is ineffective, a TIPS should be inserted as a
short-term bridge to transplantation. If a nontransplantation operation (e.g., shunt) is
performed initially, these patients should be carefully assessed at 6-month to 1-year
intervals and hepatic transplantation considered when other complications of cirrhosis
develop or hepatic functional decompensation is evident either clinically or by careful
assessment with quantitative tests of liver function.
Ascites and the hepatorenal syndrome
Ascites is usually an indicator of advanced cirrhosis and is associated with a 1-
year survival rate of approximately 50% compared to a 1-year survival rate of greater
than 90% for patients with cirrhosis but without ascites. Patients with ascites refracto-
ry to medical management, those who develop spontaneous bacterial peritonitis, and
those who evolve to the hepatorenal syndrome have a particularly poor prognosis.
Portal hypertensive ascites is initiated by altered hepatic and splanchnic hemodynam-
ics, which cause transudation of fluid into the interstitial space. When the rate of in-
terstitial fluid formation exceeds the lymph drainage capacity, ascites accumulates.
This pathophysiologic process results in an intravascular volume deficit, which initi-
ates compensatory mechanisms such as aldosterone secretion, to restore plasma vol-
ume. Both the liver and intestine are important sites of ascites formation, and clinical-
ly significant ascites is rare in patients with extrahepatic portal hypertension. The hy-
poalbuminemia that often accompanies advanced chronic liver disease may also con-
tribute to ascites formation. Since avid sodium retention by the kidneys is one of the
key mechanisms in the development of ascites, a central goal of treatment is to
achieve a negative sodium balance. A small percentage of patients with ascites can be
effectively treated by dietary salt restriction and bed rest alone. More commonly, diu-
retic therapy is required and will resolve this complication of portal hypertension in
greater than 90% of patients. Since secondary hyperaldosteronism is a key pathoge-
netic mechanism in the formation of ascites, a rational first-line diuretic is spironolac-
tone. A combination of salt restriction (2 g/day) and spironolactone in a dose of 100
to 400 mg/day results in effective diuresis in about two thirds of patients.
As a general guideline, patients with new-onset ascites that is barely detectable
on physical examination should be placed on salt restriction alone. However, patients
with more advanced or tense ascites usually require the combination of sodium re-
striction and diuretic therapy. The preferred initial spironolactone dose is 100 mg/day,
and this can be advanced to a maximum dose of 400 mg/day until effective diuresis is
achieved. If treatment with spironolactone alone is ineffective or results in hyper-
kalemia, furosemide in an initial dose of 40 mg/day should be added to the regimen.
During diuresis, body weight should be carefully monitored and not allowed to de-
crease at a rate of more than 1 lb/day in patients with ascites alone and no peripheral
edema. More aggressive diuresis usually results in contraction of the intravascular
volume and azotemia.
From 5% to 10% of patients with ascites are refractory to medical treatment
and require more invasive measures. The two mainstays of therapy in this group of
patients are large-volume paracentesis combined with intravenous albumin admin-
istration and TIPS. Because it can be done in the outpatient setting and is less inva-
sive, the generally preferred initial treatment for patients with ascites refractory to
medical treatment is large-volume paracentesis combined with intravenous albumin
infusion in a dose of 6 to 8 g/L of ascites removed. TIPS, which is more effective for
the long-term control of ascites than large-volume paracentesis, should be used in pa-
tients who require frequent paracentesis for management of their ascites. After large-
volume paracentesis, ascites is less likely to recur in patients treated with spironolac-
tone than in those not on a diuretic. Controlled trials have shown either complete or
partial resolution of ascites after placement of TIPS in more than 80% of patients
with medically intractable ascites. As in patients treated with TIPS for variceal bleed-
ing, major disadvantages of this therapy are a fairly high rate of encephalopathy and
eventual TIPS dysfunction in the majority of patients. Although initially effective in
the majority of patients, a surgically placed peritoneovenous shunt is seldom used in
the management of medically refractory ascites because of its associated complica-
tions such as occlusion, infection, and disseminated intravascular coagulation. Addi-
tionally, controlled trials have shown that this relatively simple operation, which can
be done under local anesthesia, is no more effective than medical management in pro-
longing patient survival. A surgically constructed side-to-side portal systemic shunt is
also effective in relieving ascites. However, because of the associated morbidity and
mortality, these operations are infrequently done and should be used only in ascitic
patients who have bled from esophagogastric varices and in whom TIPS is either not
indicated or has failed. Cirrhotic patients with ascites who develop fever, abdominal
tenderness, or worsening hepatic and/or renal function should undergo a diagnostic
paracentesis to rule out spontaneous bacterial peritonitis. This complication of ascites
is associated with a mortality rate of approximately 25% per episode. The diagnosis is
made with an ascitic fluid polymorphonuclear leukocyte count of greater than
250/mm3 or a positive ascites culture. The most common organisms causing sponta-
neous bacterial peritonitis are aerobic gram-negative ones, which likely come from
the bowel via bacterial translocation. Before culture results are available, antibiotic
therapy should be initiated when spontaneous bacterial peritonitis is suspected. A 5-
to 10-day course of either cefotaxime or a combination of amoxicillin and clavulanic
acid have been shown to be effective treatment. Since spontaneous bacterial peritoni-
tis recurs in more than 70% of patients, prophylactic therapy with oral norfloxacin
should be initiated as soon as intravenous therapy is completed and continued until
ascites is resolved. Another life-threatening complication of portal hypertension is the
hepatorenal syndrome that develops almost exclusively in patients with tense ascites
and declining hepatic function. When renal failure is rapidly progressive, the progno-
sis is poor, with a median survival of approximately 2 weeks. In other patients, renal
failure develops more gradually and the prognosis is somewhat better. The only relia-
ble treatment for the hepatorenal syndrome is liver transplantation. Because the renal
failure is functional rather than structural, once hepatic function is improved and por-
tal hypertension is relieved, the kidneys recover. A few small series have suggested
that renal function may improve in patients with the hepatorenal syndrome after in-
sertion of a TIPS. However, in this setting, TIPS should be regarded as a bridge to
liver transplantation in the near future.
Basic literature:
47.Oxford Textbook of Surgery (3-Volume Set) 2nd edition (January 15, 2000):
by Peter J. Morris (Editor), William C. Wood (Editor) By Oxford Press
48.Sabiston Textbook of Surgery 17th edition by Courtney M. Townsend Jr.,
Kenneth L. Mattox, B. Mark, MD Evers, Kenneth L., MD Mattox, Courtney
Townsend, Daniel Beauchamp, B. Mark Evers, Kenneth Mattox W.B. Saun-
ders Company (June, 2004)
49.Schwartz´s Principles of Surgery 8th Edition F.Charles Brunicardi. Copyright
©2007 The McGraw-Hill Companies.
50.Hospital surgery/ Edited by L.Kovalchuk et al. - Ternopil: Ukrmedknyha,
2004. - 472 p.
Additional literature:
1. Current Clinical Strategies. Surgery 2002 Edition / Red.: Samuel Eric Wilson. -
The University of California, Irvine, Manual of Surgery
2. Garcia-Tsao G., Sanyal A.J., Grace N.D., Carey W., Practice Guidelines Com-
mittee of the American Association for the Study of Liver Diseases, Practice
Parameters Committee of the American College of Gastroenterology. Preven-
tion and management of gastroesophageal varices and variceal hemorrhage in
cirrhosis / Hepatology. – 2007. – Vol.46, №3. – P.922-38.

Tests for initial level of knowledge, keys for tests:


1. Which statement regarding fat absorption is true?
A. Half of neutral fat can be absorbed in the complete absence of bile and pan-
creatic lipase
B. Fifty percent of the total bile salt pool is lost in the stool and replaced daily
by synthesis in the liver
C. Glycerol, short-chain fatty acids, and medium-chain triglycerides exit the
mucosal cell in chylomicrons
D. Conjugated bile salts are actively resorbed in the colon and returned to the
liver via the portal vein
E. Water-insoluble dietary lipid is rendered into soluble micelles through mix-
ing with pancreatic amylase

2. In the management of echinococcal liver cysts


A. A large cyst should be treated by percutaneous aspiration of its contents
B. Medical treatment with albendazole usually preempts the need for surgical
drainage
C. Negative serologic tests suggest that the cyst is chronic and inactive and that
no treatment is indicated
D. Leakage of cyst fluid puts the patient at risk for anaphylactic reaction
E. Coexistent extrahepatic cysts are uncommon

3. True statements regarding hemobilia include which of the following?


A. The classic presentation includes biliary colic, jaundice, and gastrointestinal
bleeding
B. Spontaneous bleeding secondary to hematologic disorders is the major cause
of this disorder
C. Percutaneous transhepatic catheter placement of an absorbable gelatin
sponge (Gelfoam) is the preferred treatment in cases of significant intrahepatic
bleeding
D. Angiography and endoscopy have no role in the treatment of intrahepatic
bleeding
E. Arterial embolization is advocated for hemobilia from the extrahepatic bile
ducts

4. True statements regarding cavernous hemangiomata of the liver in adults in-


clude
A. The majority become symptomatic
B. They may undergo malignant transformation
C. They enlarge under hormonal stimulation
D. They should be resected to avoid spontaneous rupture and life threatening
hemorrhage
E. A liver/spleen radionucleotide scan is the most sensitive and specific way to
make the diagnosis

5. Treatment of Choice for gastric varices is


A. Sclerotherapy
B. Band Ligation
C. Transjugular Intrahepatic Portosystemic Shunt
D. Application of cyanoacrylate glue
E. Surgery and suturing

6. Which of the following structures do not form the portal triad?


A. Portal Vein
B. Hepatic vein
C. Hepatic Artery
D. Bile Duct
E. Ligamentum venosum
7. Which is the most common cause of Hemobilia?
A. Blunt Trauma of Abdomen
B. Iatrogenic injury
C. Cholelithiasis
D. Hepatic artery aneurysm
E. Budd-Chiari syndrome

8. Which is not true regarding Haemangioendothelioma of liver?


A. It is a rare tumor
B. It is highly vascular
C. It involves both lobes of the liver
D. In 50% cases it has extrahepatic spread
E. Regularly the size is less than 5 cm

9. Most common cause of Budd Chiari syndrome in Asia is?


A. Polycythemia
B. IVC obstruction
C. Myeloproilferative disorders
D. Bechet’s disease
E. Tricuspid regurgitation

10. Most common site of origin of accessory Right Hepatic artery is?
A. Superior Mesenteric artery
B. Gastroduodenal artery
C. Coeliac artery
D. Right hepatic artery
E. Left gastric artery

Keys for tests


1 2 3 4 5 6 7 8 9 10
A D A C D B B D B A

Tests for final level of knowledge, keys for tests:

1. What percent of hepatic blood flow does portal vein provide?


A. 25%
B. 50%
C. 75%
D. 90%
E. 35%

2. The causes of increased vascular resistance to portal flow are the following
EXCEPT:
A. Intrahepatic
B. Posthepatic
C. Mixed
D. Retrocaval
E. Prehepatic

3. The prehepatic portal hypertension may be caused by the following entities


EXCEPT:
A. Splenic vein thrombosis
B. Portal vein thrombosis
C. congenital atresia or stenosis of portal vein
D. extrinsic compression
E. retroperitoneal fibrosis

4. The complications of portal hypertension include the following EXCEPT:


A. Variceal haemorrhage
B. Encephalopathy
C. Ascites
D. Hypersplenism
E. Caput medusa

5. The listed clinical findings are typical for portal hypertension EXCEPT:
A. Palmar erythema
B. Splenomegaly
C. The caput medusa
D. Ascites
E. Esophageal varices
6. For the confirmation of the presence of gastroesophageal varices and source
of bleeding from the upper gastrointestinal tract may be used:
A. Barium swallow
B. Plain radiographs of the abdomen
C. Upper endoscopy
D. Ultrasound and duplex scanning
E. Splenic venography

7. The following approaches may be utilized for the treatment of acute bleeding
episode EXCEPT:
A. Pharmacotherapy
B. Balloon Tamponade
C. Endoscopic sclerotherapy
D. Transjugular intrahepatic portosystemic shunt
E. Splenectomy

8. Ascites may be treated by the following ways EXCEPT:


A. Pharmacologically
B. Embolisation of splenic artery
C. TIPS
D. Peritoneovenous shunt
E. Sugiura procedure

9. Umbilical vein extends from the umbilicus to the


A. Bifurcation of the portal vein
B. Main right branch of the portal vein
C. Main left branch of the portal vein
D. IVC
E. Right hepatic vein
10. For the management of the hepatic hydatid cysts the following approaches
may be utilized:
A. Excision or pericystectomy
B. Marsupialization procedures
C. Drainage of the cyst
D. Regional chemotherapy
E. Liver resection

Keys for tests

1 2 3 4 5 6 7 8 9 10
C D E E A C E E C D

Tasks for final level of knowledge.

1. A 60-year-old male alcoholic is admitted to the hospital with hemateme-


sis. His blood pressure is 100/60 mm Hg, the physical examination reveals spleno-
megaly and ascites, and the initial hematocrit is 25%. Nasogastric suction yields 300
mL of fresh blood. After initial resuscitation, this man should undergo:

The answer is selective angiography

2. A 50-year-old alcoholic with known cirrhosis is noted to have a mass in the


right lobe of his liver and an elevated α-fetoprotein level. What is the most likely di-
agnosis?

The answer is hepatocellular carcinoma

3. A previously healthy 9-year-old child comes to the emergency room because


of fulminant upper gastrointestinal bleeding. The hemorrhage is most likely to be the
result of:
The answer is esophageal varices

4. A patient whose job involves worldwide travel presents with a complaints of


right upper quadrant pain. Examination reveals hepatomegaly and eosinophilia. The
most likely diagnosis is:

The answer is echinococcal cyst

5. A 45-year-old man is seen in the emergency room after vomiting bright red
blood. He has no previous symptoms. He drinks one alcoholic beverage a day. The
most reliable method for locating the lesion responsible for the bleeding is:

The answer is upper endoscopy

Materials for the self-study of the students

Main tasks Notes (instructions)


Repeat: 1. Causes of liver cirrhosis and fi-
6. Normal Anatomy of the liver and brosis.
portal system. 2. Structural changes of liver paren-
7. Normal Physiology of the liver chyma in cirrhosis.
and billiary tract. 3. Parameters of normal portal he-
8. Normal and pathologic portal he- modynamic and methods of their as-
modynamic. sessment.
9. Pathophysiology and etiology of 4. Indications to liver transplantation
the diffuse hepatic diseases. in case of advanced liver disease.
10. Etiology and pathogenesis
of parasitic and nonparasitic nodular
diseases of the liver.
11. Special investigations in
patients with hepatic diseases.
Study: 4. Make a scheme of portal circula-
4. Clinical symptoms and signs of acute tion.
appendicitis. 5. To prepare a scheme of hydatid
5. Clinical examination and special in- cyst covers.
vestigations in patients with acute 6. Surgical methods of liver echino-
appendicitis. coccus treatment.
6. Principles of surgical treatment of 7. Different types of porto-systemic
patients with complication's forms of shunts.
acute appendicitis.
Study guide #11
«Abdominal trauma. Classification, manifestation of the abdominal cavity and
retroperitoneal space organs lesions. Instrumental methods of diagnostics. Diag-
nostic and medical tactics.

Overview
Trauma is a major worldwide public health problem. It is one of the leading causes
of death and disability in both industrialized and developing countries. Globally, inju-
ry is the seventh leading cause of death, resulting in 5.8 million deaths in 2000.
Derived from the Greek word ‘tpavua', trauma refers to an injury (in + jus, ‘not
right') or wound. Either of these is characterized by a structural alteration and/or
physiologic imbalance that results when energy is imparted or vital functions com-
promised during interaction with physical or chemical agents. Trauma encompasses a
wide range of mechanisms of injury. Trauma is the main cause of death in the people
under the age 35 years. It constitutes up to 20% of surgical admission.
Educational aims:
16. Interrogation and clinical inspection of patients with trauma of abdomen.
17. To determine the etiologic and pathogenic factors of traumas of abdo-
men.
18. To find out the types of traumas of abdomen, the clinical features, differ-
ent variants of manifestation and complications.
19. To develop a plan of examination of the patients with trauma of abdo-
men.
20. To estimate laboratory data, results of laparoscopy, ECG, diagnostic lap-
arotomy, diagnostic puncture, CT of the abdomen cavity organs.
21. To draw a differential diagnosis, substantiate and formulate a diagnosis
of the trauma of abdomen.
22. To determine the indications for treatment of patients with the trauma of
abdomen.
23. To cure the patients with the traumas of abdomen after operations.
24. To estimate efficiency of treatment and prognosis of disease.
A student must know:
14.Аnatomo-physiological information about abdomen cavity organs.
15.Classifications of abdomen trauma.
16.Mechanisms of injury of abdomen.
17.Clinical picture of damages of abdomen.
18.Methods of diagnostics of abdomen damages.
19.Principles of treatment of abdomen trauma.
20.Emergent abdominal exploration.
21.Giving first aid for abdomen trauma.
22.Features of surgical interventions for the different traumas of abdomen.
23.General principles of operative management for abdomen trauma.

A student must be able to:


13.Collect and estimate the complaints of patient with abdominal trauma, infor-
mation of anamnesis, to conduct physical research and correctly interpret the
results obtained.
14.Define the rational volume of laboratory and instrumental methods of research.
15.Correctly interpret the results of clinical analyses, peritoneal lavage, ultrasound
diagnostics, computer tomography, X-ray.
16.Give first aid for traumas of the abdomen.
17.Define indications for operation and other methods of treatment of patients.
18.Perform pre-operative preparation of patients.
19.Conduct post-operative care.

Terminology.

Term Definition
Isolated trauma traumatic action on organs and tissues within the limits
of one anatomic area (thorax, abdomen, skull, extremi-
ties)
Combined trauma traumatic action within the limits of two and more of
(polytrauma) anatomic areas
Combination trauma combination two or more, different mechanism of action
of traumatic factors
Вlunt damages of the the mechanical damages of abdomen, when there is no
abdomen violation of the skin covering the area
Penetrating trauma is damage of abdomen, when there is violation of the
skin, muscles and get into abdominal cavity
Hypovolaemic shock Is pathologic condition due to decreased circulating
plasma volume
Mechanisms of injury
Blunt trauma
Victims of motor vehicle crashes, falls, or assaults are injured by impact force
and deformation related to deceleration and compression. Determinants of impact
force are magnitude (essentially, kinetic energy and area of application), duration of
application, and direction of application. The physical deformities that result from
impact force are known as strains. Strains are divided into those that are tensile
(stretching), shearing (opposing forces across an object), or compressive (crushing).
When the elasticity (tendency to regain original condition) or viscosity (resistance to
change in shape during motion) of a tissue or organ is exceeded by applied strains,
disruption results. In biomechanical terminology, disruption occurs at the elastic limit
or break point. Disruption causes injuries to the skin (abrasion, contusion, chop,
puncture, incision, and laceration), buckling or fracture of bones, and visceral or vas-
cular ruptures.
Other than impact force, factors that determine the magnitude of injury after
blunt trauma include gender, impact resistance of body parts, fixation of body parts
causing deformation during deceleration, and anatomic protection of body parts. The
impact of gender is not clear at this time, but the lighter body skeleton and smaller
muscular development of females may be important. In one study the fatality risk in
motor vehicle crashes for females aged 15 to 45 years was 25 per cent greater than for
males. Bones such as the first and second ribs, sternum, scapula, pelvis, and femur are
fractured only when significant impact forces are applied. Therefore, associated inju-
ries should always be suspected when such a fracture occurs. For example, associated
injuries to the head, chest, and abdomen occurred in 53, 64, and 33 per cent of pa-
tients, respectively, with fracture of the first rib in one review. Fixation of the de-
scending thoracic aorta to the ligamentum arteriosum, the liver to the falciform liga-
ment, and the small bowel to adhesions, the ligament of Trietz, and the retroperitone-
al caecum are thought to contribute to injuries occurring in these structures during
blunt deceleration trauma. Finally, the close associations of the brain and skull, the
right ventricle and the sternal area, and the spleen and ribs 9 to 11 are all related to
injuries occurring in these structures.
Penetrating trauma
The kinetic energy of stab wounds is low, and death occurs only if a critical or-
gan such as the heart or a major blood vessel is injured. Injuries from missile wounds
are caused by a combination of factors including the following: (1) missile (weight or
mass, shape, velocity, kinetic energy at impact); (2) medium (drag or resistance of the
medium and coefficient of drag); and (3) pattern of flight of missile upon impact
(whether yaw, tumbling, precession, or nutation is present). Direct mechanisms of in-
jury from missile wounds or fragments include cutting or laceration and transfer of
heat. Indirect mechanisms include longitudinal low-displacement shock or sonic pres-
sure waves and temporary cavitations from transverse high-displacement shear
waves. The low-displacement wave does not appear to cause damage to tissues, while
the high-displacement wave associated with medium- or high-velocity missiles signif-
icantly increases indirect damage by causing cavitations. This is thought to be due to
the transfer of kinetic energy causing alternating collapsing and reforming of the
cavity after the missile passes. Tissues that are relatively inelastic such as the brain,
liver, and spleen are damaged the most by cavitations. Other factors that increase the
magnitude of injury include fragmentation of the missile after striking the victim or
the creation of secondary missiles such as fragments of teeth or bone.

Injury to the abdomen


Blunt injury
Obvious indications for an emergency celiotomy in a patient who has suf-
fered blunt abdominal trauma include hypotension in combination with a rigid dis-
tended abdomen, peritonitis, or evisceration. The problem in evaluating many patients
with possible blunt abdominal injuries, however, is that the physical examination is
compromised by one of the following: (1) altered sensorial (injury to the brain, inges-
tion of alcohol or illicit drugs); (2) altered sensation (injury to the spinal cord); (3) in-
juries to adjacent structures (ribs, pelvis, thoracolumbar spine); or (4) physical exam-
ination is equivocal. In the groups described, additional diagnostic tests are necessary
to confirm a diagnosis of intra-abdominal injury.
Hypotensive patients with possible intra-abdominal hemorrhage undergo either
a diagnostic peritoneal lavage or surgeon-performed ultrasound. An open or closed
diagnostic peritoneal tap/lavage is performed through an infraumbilical midline site
(supraumbilical if there is pelvic fracture) after the insertion of a nasogastric tube and
bladder catheter. The return of 10 to 20 ml of gross blood or of bile, succus entericus,
stool, or food material is a positive ‘tap', and immediate celiotomy is indicated. In the
hypotensive patient with a grossly negative ‘tap', the value of a subsequent time-
consuming lavage with 1000 ml of normal saline solution is questionable.
Diagnostic peritoneal lavage is invasive, has an accuracy of 95 to 98 per cent
in detecting intraperitoneal hemorrhage, and has a complication rate of 0.5 to 1 per
cent. The previously described surgeon-performed ultrasound of the pericardium is
extended onto the right upper quadrant, left upper quadrant, and suprapubic area to
complete the ‘FAST' (focused assessment for the sonographic examination of
the trauma patient) in hypotensive (or stable) patients. As an alternative to diagnostic
peritoneal lavage, a surgeon-performed ultrasound is a noninvasive and rapidly-
performed assessment of the abdomen in the injured patient. An anechoic image (flu-
id or blood) in subphrenic area, Morison's pouch, the splenorenal fossa, or the pouch
of Douglas/pararectal areas in a hypotensive patient is essentially 100 per cent accu-
rate in confirming the need for an emergency celiotomy. Hemodynamically-stable pa-
tients in whom the physical examination is compromised by any of the previously de-
scribed factors are first evaluated by a surgeon-performed ultrasound. In the absence
of any intraperitoneal fluid on a first and subsequent examination, the need for a fol-
low-up examination of the abdomen with a CT scan is questionable – even in the in-
toxicated patient. The presence of intraperitoneal fluid on the ultrasound mandates a
follow-up abdominal CT to localize the source of hemorrhage. Findings on an ab-
dominal CT with contrast that indicate the need for an urgent celiotomy, even in a
hemodynamically stable patient.
When an abdominal contrast CT is not available to evaluate the hemodynam-
ically-stable patient, a chest radiograph, flat plate radiograph of the abdomen, or con-
trast study of the gastrointestinal or genitourinary tract will detect all of the injuries
except those to the liver, spleen, or pancreas. As previously noted under ‘Patterns of
injury', it is important to evaluate patients with marks of a restraint device across the
lower abdomen after blunt deceleration/compression trauma carefully. In a former
era, many of these patients would have died in a frontal collision. By surviving in the
modern era of restraints, death is exchanged for deceleration/compression injuries in
the abdomen. In one review of 61 children with a ‘linear ecchymosis across the ab-
domen' (seatbelt sign) after a motor vehicle crash, 14 children (23 per cent) injured a
hollow viscous, (21 per cent) had an injury to the lumbar spine, and five (8 per cent)
had injury to both.

Penetrating trauma
Approximately 25 to 33 per cent of patients with stab wounds of the anterior
abdominal wall (between anterior axillary lines) do not have penetration of the peri-
toneal cavity. Therefore, stable and cooperative patients without obvious indications
for a celiotomy (see below) should first undergo a wound exploration under local an-
esthesia in the emergency center. In the absence of penetration of the anterior fascia
or, if possible to determine, penetration of the peritoneum despite a deep track, the
patient is discharged after the stab wound site is irrigated and closed. Penetration of
the anterior fascia in large patients or the anterior peritoneum in thin patients man-
dates further evaluation. The most common option chosen around the world is serial
physical examinations for 24 h by a surgeon or senior resident. This noninvasive ap-
proach results in a delay to definitive operation in only 5 to 6 per cent of patients with
intra-abdominal injuries. A second option is to perform a standard diagnostic perito-
neal tap/lavage with positive results being the same as described for blunt abdominal
trauma. This invasive technique results in a certain number of false-positive results
(bleeding from the site of the stab wound), occasional false-negative results (early
lavage after small stab hole of midgut), and has an overall accuracy of 88 to 94 per
cent. In the 45 to 50 per cent of patients who are originally asymptomatic despite hav-
ing penetration of the anterior peritoneal cavity, 50 per cent of these will eventually
come to a celiotomy based on a changing physical examination or on a positive tap or
lavage.
Approximately 50 to 55 per cent of patients with anterior stab wounds pene-
trating the peritoneal cavity have the same obvious indications for an emergency celi-
otomy as do patients with blunt abdominal trauma. In addition, patients with the fol-
lowing should also undergo celiotomy: (1) new onset hematemesis, proctorrhagia, or
hematuria; (2) evidence of a left-sided diaphragmatic defect on finger palpation prior
to insertion of a thoracostomy tube; or (3) contrast radiography evidence of an injury
to the kidney (significant injury), ureter, or bladder.
The management of stab or gunshot wounds to the flank (between anterior
and posterior axillary lines from sixth intercostal space to iliac crest) or to the back
(posterior to posterior axillary line from tip of scapula to iliac crest) has changed over
the past 20 years. Because of the large bulk of muscles in this area in young males,
the routine celiotomies that were performed in the past were often negative. In pa-
tients in whom a local wound exploration does not reveal the end of a stab wound
track, either serial physical examinations or double (intravenous and oral) or triple
(add rectal and colon) contrast CT is performed. Serial examinations result in a false-
positive (unnecessary celiotomy)/ false-negative (delayed celiotomy) rate of 5 per
cent. Examination of most or all retroperitoneal viscera and vascular structures using
double or triple contrast CT has an overall accuracy rate of 96 to 97 per cent. Older
data documented that gunshot wounds traversing the peritoneal cavity resulted in vis-
ceral or vascular injuries needing surgical repair in 96 to 98 per cent of patients. In
recent years, it has become clear that 15 to 30 per cent of patients with gunshot
wounds in proximity to the peritoneal cavity or visceral–vascular retroperitoneum ac-
tually have missile tracks that pass through the body wall or anterior–lateral extra-
peritoneal area, only. In addition, some centers are observing isolated gunshot
wounds to the liver or kidney in stable patients in whom an emergency CT mostly
rules out associated injuries to the gastrointestinal tract. When the hemodynamically
stable patient without peritonitis presents with a possible extraperitoneal gunshot
wound, serial physical examinations rather than an emergency celiotomy are appro-
priate. If available, a surgeon-performed ultrasound documenting intraperitoneal fluid
(blood in the ‘asymptomatic patient') would be followed by celiotomy rather than se-
rial examinations.

Emergent Abdominal Exploration

ll abdominal explorations in adults are performed using a long midline incision


because of its versatility. For children under the age of 6, a transverse incision may be
advantageous. If the patient has been in shock or is currently unstable, no attempt
should be made to control bleeding from the abdominal wall until major sources of
hemorrhage have been identified and controlled. The incision should be made with a
scalpel rather than with an electrosurgical unit because it is faster. Liquid and clotted
blood is rapidly evacuated with multiple laparotomy pads and suction. Additional
pads are then placed in each quadrant to localize hemorrhage, and the aorta is palpat-
ed to estimate blood pressure.

If exsanguinating hemorrhage is encountered upon opening the abdomen, it is usu-


ally caused by injury to the liver, aorta, inferior vena cava, or iliac vessels. If the liver
is the source, the hepatic pedicle should be immediately clamped (a Pringle maneu-
ver) and the liver compressed posteriorly by tightly packing several laparotomy pads
between the hepatic injury and the underside of the right anterior chest wall. This
combination of maneuvers will temporarily control the hemorrhage from virtually all
survivable hepatic injuries.

If exsanguinating hemorrhage originates near the midline in the retroperitoneum,


direct manual pressure is applied with a laparotomy pad and the aorta is exposed at
the diaphragmatic hiatus and clamped. The same approach is used in the pelvis except
that the infrarenal aorta can be clamped, which is both easier and safer because
splanchnic and renal ischemia are avoided. Injuries of the iliac vessels pose a unique
problem for emergency vascular control. Because there are so many large vessels in
proximity, multiple vascular injuries are common. Furthermore, venous injuries are
not controlled with aortic clamping. A helpful maneuver in these instances is pelvic
vascular isolation. For stable patients with large midline hematomas, clamping the
aorta proximal to the hematoma is also a wise precaution. Many surgeons take a few
moments, once overt hemorrhage has been controlled, to identify obvious sources of
enteric contamination and minimize further spillage. This can be accomplished with a
running suture or with Babcock clamps.

Any organ can be injured by either blunt or penetrating trauma; however, certain
organs are injured more often depending on the mechanism. In blunt trauma, organs
that cannot yield to impact by elastic deformation are most likely to be injured. The
solid organs, liver, spleen, and kidneys, are representative of this group. For penetrat-
ing trauma, organs with the largest surface area when viewed from the front are most
prone to injury (i.e., the small bowel, liver, and colon). Since bullets and knives usu-
ally follow straight lines, adjacent structures are commonly injured (e.g., the pancreas
and duodenum). Penetrating trauma is not limited by the elastic properties of the tis-
sue, and vascular injuries are far more common. While these concepts simplify the
localization of injuries, unless the patient has exsanguinating hemorrhage, a methodi-
cal exploration should always be carried out.

All abdominal organs are systematically examined by visualization, palpation, or


both. Missed injuries are a serious problem with often fatal results. In penetrating
trauma missed injuries can occur if wound tracks are not followed their entire dis-
tance. A second common reason for missed injuries is failure to explore retroperito-
neal structures such as the ascending and descending colons, the second and third
portion of the duodenum, and ureters. Furthermore, injuries of the aorta or vena cava
may be temporarily tamponaded by overlying structures. If the retroperitoneum is
opened and the injury overlooked, delayed massive hemorrhage may occur following
abdominal closure. Blunt abdominal injuries are usually obvious, but injuries of the
pancreas, duodenum, bladder, and even the aorta can be overlooked.

Vascular Injuries

Injury to the major arteries and veins in the abdomen are a technical challenge to
the surgeon and are often fatal. All vessels are susceptible to injury with penetrating
trauma. Vascular injuries in blunt trauma are far less common and usually involve the
renal arteries and veins, though all other vessels, including the aorta, can be injured.
Several vessels are notoriously difficult to expose. These include the retrohepatic ve-
na cava; suprarenal aorta; the celiac axis; the proximal superior mesenteric artery; the
junction of the superior mesenteric, splenic, and portal veins; and the bifurcation of
the vena cava. Maneuvers have been described to aid in the exposure of all of these
vessels. The suprarenal aorta, celiac axis, proximal superior mesenteric, and left renal
arteries can all be exposed by left medial visceral rotation. This is accomplished by
incising the left lateral peritoneal reflection beginning at the distal descending colon
and extending the incision past the splenic flexure, around the posterior aspect of the
spleen, behind the gastric fundus, and ending at the esophagus. This incision permits
the left colon, spleen, pancreas, and stomach to be rotated toward the midline. Divi-
sion of the left crus of the diaphragm will permit access to the aorta above the celiac
axis. The maneuver is much more difficult and time-consuming than it first appears.
In contrast, mobilization of the right colon and a Kocher maneuver will expose the
entire vena cava except the retrohepatic portion, and it is technically simple. This is
referred to as a right medial visceral rotation. The kidney can be left in situ or mobi-
lized with the remaining viscera with both right and left medial rotations.

The junction of the superior mesenteric, splenic, and portal veins can be exposed
in elective surgery by dissecting the vessels from the pancreas as required when per-
forming a distal splenorenal shunt. However, in the presence of massive bleeding
from a venous injury, this may be impossible. Therefore in trauma surgery, the neck
of the pancreas is divided without hesitation. This provides excellent exposure of this
difficult area. Management of the transected pancreas will be discussed below. The
bifurcation of the vena cava is obscured by the right common iliac artery. This vessel
should be divided to expose extensive vena caval injuries of this area. The artery must
be repaired after the venous injury is treated or amputation occurs in as many as 50%
of patients.

Diaphragm

In blunt trauma the diaphragm is injured on the left in 75% of cases, presumably be-
cause the liver diffuses some of the energy on the right side. 70 For both blunt and
penetrating trauma, the diagnosis is suggested by an abnormality of the diaphragmatic
shadow on chest x-ray. Many of these are subtle, particularly with penetrating inju-
ries, and further diagnostic evaluation may be warranted. The typical injury from
blunt trauma is a tear in the central tendon, which may be large. Regardless of the eti-
ology, acute injuries are repaired through an abdominal incision. The laceration is
closed with a no. 1 monofilament permanent suture, using a simple running tech-
nique. Occasionally, large avulsions or GSWs with extensive tissue loss will require
polypropylene mesh to bridge the defect.

Spleen

Splenic injuries are treated nonoperatively, by splenic repair (splenorrhaphy), par-


tial splenectomy, or resection, depending on the extent of the injury and the condition
of the patient. Enthusiasm for splenic salvage has been driven by the evolving trend
toward nonoperative management of solid organ injuries and the rare but often fatal
complication of overwhelming postsplenectomy infection (OPSI). These infections
are caused by encapsulated bacteria (i.e., Streptococcus pneumoniae, Haemophilus
influenza, and Neisseria meningitidis) and are very resistant to treatment. OPSI oc-
curs most often in young children and immunocompromised adults. It is uncommon
in otherwise healthy adults. For this reason attempts to salvage the spleen are more
vigorous in children.

In order to safely remove or repair the spleen it should be mobilized to the extent
that it can be brought to the surface of the abdominal wall without tension. This re-
quires division of the attachments between the spleen and splenic flexure of the co-
lon. Next, an incision is made in the peritoneum and endoabdominal fascia beginning
at the inferior pole, a centimeter or two away from the spleen, and continuing posteri-
orly and superiorly until the esophagus is encountered, similarly to a left medial vis-
ceral rotation. Care must be taken not to pull on the posterior aspect of the spleen or it
will tear at the peritoneal reflection, causing significant hemorrhage. Rather, the
spleen should be rotated counterclockwise with posterior pressure applied to expose
the peritoneal reflection. It is often helpful to rotate the operating table 20° to the pa-
tient's right, so the weight of abdominal viscera aids in the retraction. A plane can
then be established between the spleen and pancreas and Gerota's fascia, which can
be extended to the aorta. This will complete mobilization and permit the spleen to be
repaired or removed without struggling for exposure.

Hilar injuries or a pulverized splenic parenchyma are usually treated by splenecto-


my. These authors have selectively reimplanted six pieces of the spleen (40 x 40 x 3
mm) within the leafs of the omentum. Technetium scans have confirmed their viabil-
ity, and immunoglobulin M (IgM) levels have normalized. However, the patient's re-
sponse to an antigenic challenge has not been evaluated. Splenectomy is also indicat-
ed for lesser splenic injuries in patients who have developed a coagulopathy and have
multiple abdominal injuries, and it is usually necessary in patients with failed splenic
salvage attempts. Partial splenectomy can be used in patients in whom only a portion
of the spleen has been destroyed, usually the superior or inferior half. Following re-
moval of the damaged portion, the same methods used to control hemorrhage from
hepatic parenchyma can be used for the spleen (see Liver section, above). When plac-
ing horizontal mattress sutures across a raw edge, gentle compression of the paren-
chyma by an assistant will facilitate hemostasis. Following ligation of the sutures and
releasing compression, the spleen will expand slightly and further tighten the sutures.
Drains are never used after completion of the repair or resection. If splenectomy is
performed, vaccines against the encapsulated bacteria are administered. The pneumo-
coccal vaccine is routinely given, and vaccines effective against Haemophilus influ-
enza and Neisseria meningitidis should be used if available.

Stomach and Small Intestine

Injuries of the stomach and small bowel pose no special problems or controversies.
Gastric injuries can occasionally be missed if a wound is located within the mesentery
of the lesser curvature or high in the posterior fundus. The stomach should be
clamped at the pylorus and inflated with air or methylene blue–colored saline if there
is any question. Patients with injuries that damage both nerves of Latarjet or both va-
gus should have a drainage procedure. If the distal antrum or pylorus is severely dam-
aged, it can be reconstructed with a Billroth I or II procedure. Although the authors
emphasize the single-layer running suture line, a running two-layer suture line is pre-
ferred for the stomach due to its rich blood supply and because postoperative hemor-
rhage has occurred when the single-layer technique has been used in the stomach.

With the almost universal use of CT for the diagnosis of blunt abdominal injury, inju-
ry to the small intestine can be missed. Wounds of the mesenteric border also can be
missed if the exploration is not comprehensive. Most injuries are treated with a lateral
single-layer running suture. Multiple penetrating injuries often occur close together.
Rather than performing many lateral repairs, judicious resections with end-to-end
anastomosis may save considerable time.

Duodenum

Duodenal hematomas are caused by a direct blow to the abdomen and occur more of-
ten in children than adults. Blood accumulates between the seromuscular and submu-
cosal layers, eventually causing obstruction. The diagnosis is suspected by the onset
of vomiting following blunt abdominal trauma; barium examination of the duodenum
reveals either the coiled spring sign or obstruction. Most duodenal hematomas in
children can be managed nonoperatively with nasogastric suction and parenteral nu-
trition. Resolution of the obstruction occurs in the majority of patients if this therapy
is continued for 7 to 14 days. If surgical intervention becomes necessary, evacuation
of the hematoma is associated with equal success but fewer complications than by-
pass procedures. Despite few existing data on adults, there is no reason to believe that
their hematomas should be treated differently from those of children. A new approach
is laparoscopic evacuation if the obstruction persists more than 7 days.

Duodenal perforations can be caused by both blunt and penetrating trauma. Blunt
injuries are difficult to diagnose because the contents of the duodenum have a neutral
pH, few bacteria, and are often contained by the retroperitoneum. Mortality may ex-
ceed 30% if the lesion is not identified and treated within 24 hours. The perforations
are not reliably identified by initial oral contrast CT examinations, therefore the au-
thors often obtain contrast x-rays with soluble contrast followed with barium if neces-
sary. Most perforations of the duodenum can be treated by primary repair. The au-
thors prefer to use a running single-layer suture of 3-0 monofilament. The wound
should be closed in a direction that results in the largest residual lumen. Occasionally,
penetrating injuries will damage only the pancreatic aspect of the second or third por-
tion. Because the duodenum cannot be adequately mobilized to repair the injury di-
rectly, the wound should be extended laterally or the duodenum divided so the pan-
creatic aspect can be sutured from the inside. As with other intestinal suture lines, du-
odenal repairs or anastomoses do not benefit from adjunctive external drainage.

Challenges arise when there is a substantial loss of duodenal tissue. Extensive inju-
ries of the first portion of the duodenum can be repaired by débridement and anasto-
mosis because of the mobility and rich blood supply of the distal gastric atrium and
pylorus. In contrast, the second portion is tethered to the head of the pancreas by its
blood supply and the ducts of Wirsung and Santorini, so the length of duodenum that
can be mobilized from the pancreas is limited to approximately 1 cm. Unlike the jeju-
num, ileum, or colon, this mobilization will yield little additional tissue to alleviate
tension on the suture line. As a result, suture repair of the second portion when tissue
is lost often results in an unacceptably narrow lumen, and end-to-end anastomosis is
virtually impossible; therefore more sophisticated repairs are required. For extensive
injuries proximal to the accessory papilla, débridement and end-to-end anastomosis is
appropriate. For lesions between the accessory papilla and the papilla of Vater, a vas-
cularized jejunal graft, either a patch or tubular interposition graft, may be required.
Experience with these procedures is limited. Duodenal injuries with tissue loss distal
to the papilla of Vater and proximal to the superior mesenteric vessels are best treated
by Roux-en-Y duodenojejunostomy. The distal portion of the duodenum is oversewn,
the jejunum is sutured end-to-end to the proximal duodenum, and the defunctional-
ized distal duodenum and proximal jejunum are drained into the jejunum. Alterna-
tively, the short defunctionalized duodenum can be resected; however, this is a rather
tedious dissection behind the superior mesenteric vessels that may not be tolerated by
a patient who has been in protracted shock.

Injuries to the third and fourth portions of the duodenum with tissue loss pose other
problems. Owing to the notoriously short mesentery of the third and fourth portions
of the duodenum, mobilization is limited because of the risk of ischemia. While end-
to-end duodenojejunal anastomoses are possible in these regions, the technique used
must resemble that of a hand-sewn, low anterior rectal anastomosis, with a posterior
row of interrupted sutures placed while the ends of the bowel are far apart. The jeju-
num is then parachuted down to the duodenum, and the anterior row is completed. In
the experience of these authors, duodenal fistulas are common when this method is
used. Therefore it is our preference to resect the third and fourth portions and perform
a duodenojejunostomy on the right side of the superior mesenteric vessels.

An important adjunct for high-risk or complex duodenal repairs is the pyloric ex-
clusion technique. By occluding the pylorus and performing a gastrojejunostomy, the
gastrointestinal stream can be diverted away from the duodenal repair. If a fistula
does develop, it is functionally an end fistula, which is easier to manage and more
likely to close than a lateral fistula, and the patient can take food by mouth to main-
tain nutritional status. To perform a pyloric exclusion, a gastrostomy is first made on
the greater curvature as close to the pylorus as possible. The pylorus is then grasped
with a Babcock clamp via the gastrostomy and oversewn with a 0 polypropylene su-
ture. A gastrojejunostomy restores gastrointestinal continuity. Vagotomy is not neces-
sary because marginal ulceration occurs at the same frequency (approximately 3%) as
duodenal ulceration occurs in the same patient population. Experience has shown that
the absorbable sutures do not last long enough to be effective, and even heavy poly-
propylene will give way in 3 to 4 weeks in most patients. A linear staple line across
the outside of the pylorus provides the most permanent pyloric closure.

Pancreas

Blunt pancreatic transection at the neck of the pancreas can occur with a direct blow
to the abdomen. As an isolated injury it is more difficult to detect than blunt duodenal
rupture; however, a missed pancreatic injury is more benign. Since the main pancreat-
ic duct is transected, the patient will develop a pseudocyst or pancreatic ascites, but
there is little inflammation since the pancreatic enzymes remain inactivated. The di-
agnosis can occasionally be made with CT using fine cuts through the pancreas.
However, CT will not identify a significant number of transections if performed with-
in 6 hours of injury.

Optimal management of pancreatic trauma is determined by the location of the in-


jury and whether or not the main pancreatic duct is injured. Pancreatic injuries in
which the pancreatic duct is not injured may be treated by drainage or left alone. In
contrast, pancreatic injuries associated with a ductal injury always require treatment
to prevent pancreatic ascites or a major external fistula. Direct exploration of perfora-
tions or lacerations will confirm the diagnosis of a ductal injury in most instances.
This leaves a small but significant percentage of patients in whom the diagnosis is in
doubt, and more invasive investigations may be required. One recommendation has
been to perform operative pancreatography. This procedure requires direct access to
the duct either by way of a duodenotomy or following resection of the tail of the pan-
creas. Five French pediatric feeding tubes are used for cannulating the duct. Two to
four milliliters of full-strength contrast material is slowly injected; injuries are identi-
fied by obstruction or extravasation. Great care must be taken to avoid overdistention
of the duct with contrast, which can produce pancreatitis. The obvious shortcoming of
this approach is the creation of a duodenal wound that must heal in a less-than-
optimal environment. While those who advocate transduodenal pancreatography have
had few duodenal fistulas, some have occurred. The problems associated with lateral
duodenal fistulas are sufficient to dampen any surgeon's enthusiasm for this approach.
If the patient already has a duodenal wound in the second portion, the above objec-
tions to pancreatography are mitigated. An expeditious alternative to pancreatography
is to pass a 1.5- to 2.0-mm coronary artery dilator into the main duct via the papilla
and observe the pancreatic wound. If the dilator is seen in the wound, a ductal injury
is confirmed. When inserted through the papilla of Vater, care must be taken to en-
sure that the dilator enters the pancreatic duct and not the bile duct. This can be de-
termined by palpation of the hepatic pedicle. The limitations of this approach are the
same as those for pancreatography.

A third method for identifying pancreatic ductal injuries is the use of endoscopic
retrograde pancreatography (ERP). This technique may be difficult to perform in an
anesthetized patient in the operating room, but the surgeon can assist by manipulating
the duodenum or occluding the distal portion to facilitate air insufflation. ERP is very
helpful in the delayed diagnosis of a ductal injury or in those patients who are too
sick to explore adequately during the initial operation. It is apparent based on the
above options that no ideal method exists for identifying pancreatic ductal injuries
that cannot be ruled out by direct exploration.

This dilemma tends to encourage aggressive local exploration, which may create a
ductal injury where none existed. For injuries involving the neck, body, or tail of the
pancreas, this is of minor consequence because a simple resection distal to the injury
cures the lesion. However, this is not the case for injuries to the head of the pancreas,
which cannot be treated with a simple resection. Rather than accepting the risks of
pancreatography or aggressive local exploration, a final option for identifying ductal
injuries in the head of the pancreas is to do nothing other than drain the pancreas.If
pancreatic fistula or pseudocyst develops, the diagnosis is confirmed. While this ap-
proach may not seem rational, the alternatives are also uninspiring. Fortunately, the
majority of pancreatic fistulas will close spontaneously with only supportive care.
The authors prefer this approach over operative pancreatography when the diagnosis
of ductal injury in the head of the pancreas is not apparent and ERP is not promptly
available.
Several options are available for treating injuries of the neck, body, and tail of the
pancreas when the main duct is transected. Historically, distal pancreatectomy with
splenectomy has been the preferred approach. However, during the past 15 years, in-
creasing interest in splenic preservation has stimulated the use of the splenic-
preserving distal pancreatectomy. This procedure is performed by dissecting the pan-
creas from the splenic vein. Another method for splenic preservation is to bury the
distal transected end of the pancreas in a Roux-en-Y limb. This technique also con-
serves the distal pancreas, but is seldom performed because of the added complexity
of the Roux-en-Y and the risks of pancreatojejunostomy.

For injuries to the head of the pancreas that involve the main pancreatic duct but
not the intrapancreatic bile duct, there are few options. Distal pancreatectomy alone is
rarely indicated because the risk of pancreatic insufficiency is significant if more than
85 to 90% of the gland is resected. A more limited resection from the site of the inju-
ry to the neck of the pancreas, with preservation of the pancreaticoduodenal vessels
and common duct, will allow for closure of the injured proximal pancreatic duct.
Pancreatic function can then be preserved by a Roux-en-Y pancreatojejunostomy
with the distal pancreas.

In contrast to injuries of the pancreatic duct, the diagnosis of injuries to the intra-
pancreatic common bile duct is simple. The first method is to squeeze the gallbladder
and observe the pancreatic wound. If bile is seen leaking from the pancreatic wound,
the presence of an injury is established. Operative cholangiography is diagnostic in
questionable cases. If a patient with an intrapancreatic bile duct injury is critically ill
from hemorrhage, external drainage can be utilized until the patient is fit for defini-
tive treatment. Small tangential perforations of the intrapancreatic bile duct may heal
with this treatment alone, although it is seldom recommended. Most authorities advo-
cate division of the common bile duct superior to the first portion of the duodenum,
ligation of the distal common duct, and reconstruction with a Roux-en-Y choledocho-
jejunostomy.

The use of drains has played an important role in the management of pancreatic
injuries. While many authorities advocate routine drainage of all pancreatic injuries, it
is not the practice of these authors to drain contusions, lacerations in which the prob-
ability of a major ductal injury is small or pancreatic anastomoses. However, draining
pancreatic injuries is recommended when there is a possible major ductal injury,
though it cannot be identified. If a drain is desirable, prospective studies have demon-
strated that closed-suction devices are associated with fewer infectious complications
than sump or Penrose drains. Almost all pancreatic fistulas will close spontaneously.
Nutritional support is important and electrolyte replacement may be necessary.

Pancreatoduodenal Injuries

Because the pancreas and duodenum are in physical contact, combined pancreati-
coduodenal injuries are common, particularly following penetrating trauma. These
lesions are dangerous because of the risk of duodenal suture line dehiscence and the
development of a lateral duodenal fistula. Each injury should be assessed as previous-
ly outlined. The simplest treatment is to repair the duodenal injury and drain the pan-
creatic injury. This method is appropriate for combined injuries without major duo-
denal tissue loss and without pancreatic or biliary ductal injuries. With more exten-
sive injuries, consideration should be given to providing additional protection for the
duodenal suture line. The authors prefer pyloric exclusion to other alternatives.

While most pancreatic and duodenal injuries can be treated with relatively simple
procedures, a few will require extensive operations such as pancreatoduodenectomy.
Examples of such injuries include transection of both the intrapancreatic bile duct and
the main pancreatic duct in the head of the pancreas, avulsion of the papilla of Vater
from the duodenum, and destruction of the entire second portion of the duodenum.
Most injuries of this nature are caused by high-energy gunshot wounds. In patients
with a pancreatoduodenal injury who also have an intrapancreatic bile duct injury, it
is possible to use the combination of pyloric exclusion and Roux-en-Y choledocho-
jejunostomy to avoid a pancreatoduodenectomy. However, the complexity and un-
predictable physiology of the combined procedures makes the pancreatoduodenecto-
my more attractive.

Hepatic trauma
Compressive injuries to the liver from the overlying ribs occur most frequently in
frontal motor vehicle crashes in which the victim has an impact with the lower rim of
the steering wheel or the dashboard. Compression against a shoulder belt restraint
may be a cause, as well, particularly if the device is worn improperly under the right
upper extremity. In ‘T-bone' side impacts, the front seat passenger is at significant
risk for a hepatic injury. Patients with penetrating wounds to the right thoracoab-
dominal area (nipples to costal margin and medial to right anterior axillary line) are at
risk of a hepatic injury if the diaphragm is penetrated. This occurs in approximately
15 per cent of patients with penetration of the body wall by a stab wound and in 45 to
48 per cent of those with gunshot wounds.
Diagnosis
In hypotensive patients who have suffered blunt abdominal or multisystem trauma,
either surgeon-performed ultrasound or a standard infraumbilical diagnostic peritone-
al lavage is appropriate. Using a 3.5 mHz transducer in the right midaxillary line be-
tween ribs 10 and 11, the visualization of fluid (blood unless ascites is present) in
Morison's pouch mandates a laparotomy in the absence of other overt sites of hemor-
rhage. An experienced surgeon-sonographer may visualize a hepatic injury, also.
When no fluid is present in Morison's pouch, the ultrasound probe is moved to image
the left subphrenic area/splenorenal recess and the pelvis. A diagnostic peritoneal tap
that yields 10 to 20 ml of gross blood or a formal lavage whose effluent is cloudy
enough to obscure the print on the bag of intravenous fluids mandates laparotomy in
the hypotensive patient, also. In any patient undergoing emergency laparotomy after
suffering blunt abdominal trauma, the most likely sources of hemorrhage are injuries
to the liver, spleen, or mesentery. A patient who is hemodynamically stable and with-
out peritonitis after suffering blunt abdominal trauma is evaluated by a spiral contrast
CT if the physical examination is equivocal or compromised or if there is intra-
abdominal fluid on the preliminary ultrasound. The volume of intraperitoneal fluid
(blood), magnitude of injury to the liver or other organ, and the presence or absence
of active hemorrhage on the contrast CT will determine whether nonoperative or op-
erative management is chosen in the stable patient.
Penetrating wounds to the abdomen in patients with peritonitis, hypotension, or
significant evisceration mandate laparotomy. Stab wounds to the right thoracoab-
dominal area in patients without fluid in the right subphrenic space or Morison's
pouch on ultrasound undergo in-hospital serial physical examinations for 24 h after
admission. An occasional stable patient with a gunshot wound to this area and mini-
mal tenderness may be evaluated by a contrast spiral CT to determine the magnitude
of hepatic and pulmonary injuries.
Nonoperative management
Approximately 80 to 85 per cent of all patients with hepatic trauma are stable upon
arrival in the emergency center, and, in the absence of other indications for an emer-
gency laparotomy, nonoperative management is appropriate after a contrast spiral CT.
Contraindications to nonoperative management include a period of hypotension in the
field or in the emergency center, persistent significant tachycardia despite aggressive
resuscitation, the presence of active hemorrhage from the liver, spleen, or kidney on
the contrast CT, or the presence of another organ injury mandating laparotomy. Pa-
tients are kept at bed rest, and their vital signs are monitored in the surgical intensive
care unit if a significant injury is present (American Association for the Surgery of
Trauma—Organ Injury Scale Grades III, IV, V). A falling hematocrit or continuing
need for transfusion during the nonoperative period should prompt an emergency he-
patic arteriogram or laparotomy. New onset peritonitis and hypotension are followed
by an emergency laparotomy. In stable patients a repeat spiral CT is appropriate at 5
to 7 days following injury to determine whether progression of the injury or some
healing has occurred. With some healing, discharge to a home situation in which a
family member is available to the patient is indicated if a Grade III, IV, or V injury
was present. Return to vigorous physical activity or contact sports are prohibited until
a late follow-up spiral CT shows healing.
Nonoperative management fails in approximately 2 to 7 per cent of patients with
blunt hepatic injuries. The hepatic injury, itself, will be the cause in 50 to 75 per cent
of the failures, and 65 to 85 per cent of the hepatic failures will be in patients with
Grade IV or V injuries on the original CT. Nonoperative management of gunshot
wounds of the liver is practiced in a similar fashion. The success rate is similar to that
described above for blunt trauma as missile tracks from civilian handguns are signifi-
cantly smaller than many of the Grade IV or V hepatic injuries presently undergoing
nonoperative management.
General principles of operative management
A midline incision is used, and blood and clots are evacuated manually or with a suc-
tion device. A vascular clamp is applied to the porta hepatis (Pringle maneuver) if a
significant (Grade III, IV, V) hepatic injury is present. The injured lobe is compressed
between laparotomy pads in the hands as the surgeon informs the anesthesiologist
about the need to contact the blood bank. Also, the surgeon should request that an up-
per hand retractor, various sizes of metal clips, O-chromic sutures on blunt needles,
and a 36–38 French thoracostomy tube be available in the operating room. When
blood and appropriate equipment is available in the operating room, the packs around
the liver are removed and the hepatic injury is inspected. Posterior lobar injuries or
Grade III, IV, or V injuries are best visualized by division of the ipsilateral triangular
ligament and the anterior coronary ligament at the edge of the liver. Folded dry lapa-
rotomy pads are then placed beneath the injured lobe to elevate it into the midline in-
cision. In obese patients or in those with a high likelihood of an injury to the extrahe-
patic veins or retrohepatic vena cava (dark venous hemorrhage as the injured lobe is
mobilized), a median sternotomy is also performed.
Simple techniques of hemostasis
Approximately 90 per cent of penetrating injuries and 60 per cent of blunt injuries
can be managed with 5 min of compression, the application of topical hemostatic
agents, or simple suture hepatorrhaphy. Currently available topical hemostatic agents
include oxidized regenerated cellulose, microfibrillar collagen hemostat, and fibrin
sealant. Fibrin sealant, only recently available in the United States, contains human
fibrinogen and thrombin, aprotinin, and calcium chloride. Five minutes of compres-
sion is performed after the application of a topical agent. After releasing compression,
the electrocautery is used for any remaining bleeders when only Grade I or Grade II
hepatic injuries are present. Suture hepatorrhaphy with O-chromic material is appro-
priate for Grade II and Grade III injuries. An interrupted or continuous suture tech-
nique is used, with the caveat that crushing sutures cause postoperative hepatic necro-
sis and ‘liver fever'. Drainage is not necessary in the absence of further hemorrhage or
obvious leakage of bile.
Advanced techniques of hemostasis
Advanced techniques are necessary in 10 per cent of penetrating wounds and in 40
per cent of blunt hepatic injuries. These patients have Grade III, IV, or V injuries that
will require the use of one or more of the following techniques: (1) extensive hepator-
rhaphy; (2) hepatotomy with selective vascular ligation; (3) viable omental
pack; (4) resectional debridement with selective vascular ligation; (5) absorbable
mesh compression; (6) formal resection; (7) selective hepatic artery ligation; (8)
intrahepatic balloon tamponade; (9) perihepatic packing; and (10) atriocaval shunt.
Extensive hepatorrhaphy
Extensive hepatorrhaphy is indicated in ‘damage control' situations in which in-
traoperative hypothermia (<34–35°C), metabolic acidosis (pH < 7.1–7.2), and/or a
coagulopathy (PT or PTT > 50 per cent normal) mandate a rapid operation. Large
figure-of-eight sutures or a continuous O-chromic suture is used to reapproximate the
sides of hepatic lacerations in the hope that hemorrhage from small hepatic arteries
and low pressure hepatic veins or portal veins will be controlled by compression. Ex-
tensive postoperative hepatic necrosis is likely when such sutures are tied too tight in
the presence of a prolonged Pringle maneuver.
Hepatotomy with selective vascular ligation
Gaining further exposure of a deep hepatic laceration or connecting the en-
trance and exit wounds of a penetrating wound with the finger fracture technique or
the electrocautery is known as hepatotomy. Once completed, large Deaver or Har-
rington retractors are used to maintain visibility in the depths of the hepatotomy as
selective vascular clipping or suture ligation of injured vessels is performed. This
technique should be utilized prior to the onset of hypothermia and only by surgeons
with sufficient experience in elective or traumatic hepatic surgery.
Viable omental pack
The gastrocolic omentum mobilized off the transverse colon with its blood
supply intact is used to fill Grade III, IV, or V hepatic injuries or hepatotomy sites.
Intrahepatic omentum is effective in controlling venous hemorrhage, managing
dead space, and in bringing mobile macrophages to the site of injury. While it does
not appear to aid healing, postoperative bleeding and drainage of bile are much de-
creased in the experience of most trauma surgeons. The viable omental pedicle is held
in place by compressing hepatic sutures tied under moderate tension.
Resectional debridement with selective vascular ligation
With disrupted hepatic tissue on the edge of an injured liver, the finger fracture
technique or the electrocautery should be used to create a new fresh edge of the liver
around the area of injury. Vessels and biliary ducts can then e clipped or suture ligat-
ed where they are intact, and all disrupted tissue outside this new line is then debrided
.The application of a viable omental pedicle to this new raw surface is controversial,
though this is appropriate when a coagulopathy makes hemostasis difficult.

Absorbable mesh compression


Wrapping an injured hepatic lobe in which all fragments are viable with a large
sheet of absorbable mesh tailored around the porta hepatis and inferior vena cava
hasbeen used in some centers. The technique is time-consuming, but eliminates the
need for reoperation as when perihepatic packs are used for compression.
Formal resection
Anatomic lobectomy is used in approximately 3 per cent of patients undergoing
operative management. No dissection is performed in the porta hepatis, and the lobec-
tomy is performed with a Pringle maneuver in place using finger fracture or electro-
cautery and metal clips. The large right hepatic vein can usually be controlled inside
the liver as the lobectomy is completed. Anatomic segmentectomy is much more
commonly utilized, especially with extensive lacerations beneath the falciform liga-
ment mandating resection of Couinaud's segments II and III (left lateral segment).
Selective hepatic artery ligation
Selective hepatic artery ligation is used in about 1 per cent of patients undergo-
ing operative management. It is indicated when arterial hemorrhage in a deep hepatic
laceration cannot be directly controlled, but stops whenever a Pringle maneuver is
applied. Extrahepatic ligation of the artery to the injured lobe in the porta hepatis will
fail to control hemorrhage when the wrong artery is ligated or when intrahepatic or
retrohepatic venous hemorrhage is present.
Intrahepatic balloon tamponade
The passage of a Foley or Fogarty balloon catheter into the hepatic track of a
knife or missile may allow for balloon compression of the site of parenchymal hemor-
rhage. This technique is particularly useful when the novice trauma surgeon has little
experience in completing an extensive hepatotomy through one or both lobes. The in-
flated balloon catheter is passed through the body wall away from the midline inci-
sion at the completion of the first laparotomy. After 48 to 72 h of balloon compres-
sion, the balloon is eflated and removed through the body wall in the surgical inten-
sive care unit. Rebleeding is extraordinarily rare when a parenchymal track has been
tamponaded for this period of time.
Perihepatic packing
The insertion of folded dry laparotomy pads over and, occasionally, below an
injured hepatic lobe is used in approximately 5 per cent of patients undergoing opera-
tive management. Packs should be used to tamponade minor hepatic injuries or sub-
capsular hematomas when a damage control procedure is performed. They are also
useful for any major hepatic parenchymal injury when advanced techniques of hemo-
stasis fail secondary to intraoperative hypothermia or a coagulopathy. The use of a
plastic sheet beneath the packs to prevent sticking to raw edges of parenchyma has
been useful in the author's experience. Packs are removed at a reoperation 48 to 72 h
after the original laparotomy when hypothermia, acidosis, and any coagulopathy are
corrected and the cardiovascular, respiratory, and renal systems are stable. Perihepatic
packs have also been used with success in patients with unruptured retrohepatic he-
matomas from presumed injuries to the retrohepatic vena cava.
Atriocaval shunt
A No.36 French thoracostomy tube or No.8 endotracheal tube inserted through
the right atrial appendage into the infrarenal inferior vena cava is an atriocaval shunt.
An extra hole at the level of the right atrium is cut before insertion. By pulling
circumferential umbilical tape tourniquets tight around the shunt at the suprarenal in-
ferior vena cava and intrapericardial inferior vena cava, venous return from the lower
body and renal veins is diverted into the shunt. This causes a 40 to 60 per cent de-
crease in hemorrhage from an injury in the retrohepatic vena cava and should allow
for a rapid repair. When there has not been a preoperative or intraoperative cardiac
arrest from exsanguination, use of the atrocaval shunt has resulted in a 33 to 50 per
cent survival in the modern era. Alternative approaches for injuries to the retrohepatic
vena cava include direct approach behind an injured lobe, total hepatic vascular isola-
tion, and deep hepatotomy.
Drainage
Closed suction drains above and below an injured lobe are used when an in-
traoperative coagulopathy or the extent of hepatic repair suggests that postoperative
drainage of blood and bile is likely.
Complications
Postoperative hyperpyrexia occurred in nearly two-thirds of patients with
Grade III, IV, or V injuries in one review. Early postoperative coagulopathies occur
in 15 per cent of patients, while reoperations for persistent or late hemorrhage used to
be necessary in 3 to 7 per cent of patients. Self-limited biliary fistulas occur in 8 to 10
per cent of patients, while intra-abdominal abscesses develop in 4 to 10 per cent. The
embolization of disrupted intrahepatic arteries or pseudoaneurysms to control postop-
erative bleeding or late hemobilia by the interventional radiologist has caused a sig-
nificant decrease in reoperations for hemorrhage. Reoperations for perihepatic ab-
scesses have essentially disappeared in the modern era for the same reason.
Mortality
The liver-related mortality for patients undergoing nonoperative management
has been 0 to 0.5 per cent in recent reports. Overall mortality in such patients is 8 to 9
per cent and is primarily due to associated intracranial injuries. In patients undergoing
operation for blunt hepatic injuries, the overall mortality is 15 to 20 per cent. The
mortality after operation for stab wounds and gunshot wounds to the liver is 2.5 and
10 per cent, respectively.

Gallbladder and Extrahepatic Bile Ducts

Injuries of the gallbladder are treated by lateral suture or cholecystectomy,


whichever is easier. If lateral suture is performed, absorbable suture should be used to
prevent the formation of calculi. Injuries of the extrahepatic bile ducts are a chal-
lenge. Because of the proximity of the portal vein, hepatic artery, and vena cava, as-
sociated vascular injuries are common and the patient's physiologic status is often
poor. Furthermore, the ducts are of normal size and texture (i.e., small in diameter
and thin walled). These factors usually preclude primary repairs except for the small-
est lacerations with no loss of tissue. These injuries can be treated by the insertion of
a T tube through the wound, or by lateral suture using 4-0 to 6-0 monofilament ab-
sorbable suture. Virtually all transections and any injury associated with significant
tissue loss will require a Roux-en-Y choledochojejunostomy. The anastomosis is per-
formed using a single-layer interrupted technique (because it is almost impossible to
do a running stitch) using 4-0 or 5-0 monofilament absorbable suture. A round patch
of seromuscular tissue the size of the common duct is removed from the jejunum at
the site of the anastomosis to inhibit wound contraction. The mucosa and submucosa
are punctured but not resected. Full-thickness bites of the duct and jejunum are taken.
Because of the small size of the duct, only 6 to 8 stitches can be used. T tubes are not
placed.

The jejunum is then sutured to the areolar tissue of the hepatic pedicle or
porta hepatis to take any tension off the anastomosis. Injuries of the hepatic ducts are
almost impossible to satisfactorily repair under emergency circumstances. One ap-
proach is to intubate the duct for external drainage and attempt a repair when the pa-
tient recovers. Alternatively, the duct can be ligated if the opposite lobe is normal and
uninjured. For patients who are critically ill, the common duct also can be treated by
intubation with external drainage.

Trauma damage control


Patients with shock from exsanguination related to abdominal injuries and mas-
sive transfusion develop pre- or intraoperative hypothermia, persistent metabolic aci-
dosis, and a coagulopathy. This so-called ‘vicious cycle of metabolic failure' is irre-
versible as long as the patient is in the operating room with the abdomen open during
a prolonged procedure. Pre- or intraoperative markers that confirm ‘metabolic failure'
and suggest that a ‘damage control' operation should be performed instead of a defini-
tive laparotomy. The concept of ‘damage control' operations has now been used for
over 15 years and includes three phases. Alternate closures of the abdominal incision
or coverage of the open abdomen are used in most patients undergoing a ‘damage
control' operation. Prospective randomized trials to confirm the value of ‘damage
control' are unlikely to be performed as the concept is now widely accepted. Also, it
appears to allow for the salvage of severely injured patients who died when prolonged
definitive first celiotomies were performed.

Colon
The treatment of injuries of the colon has been debated for nearly a century.
Finally, during the past decade, something resembling a consensus has been reached.
There are three conceptually different methods for treating colonic injuries: primary
repair, colostomy, and exteriorized repair. Primary repairs include lateral suture of
perforations and resection of the damaged colon with reconstruction by ileocolostomy
or colocolostomy. The advantage of primary repairs is that definitive treatment is car-
ried out at the initial operation. The disadvantage is that suture lines are created in
suboptimal conditions and leakage may occur. Several different styles of colostomies
have been used to manage colonic injuries. In some instances the injured colon can be
exteriorized like a loop colostomy. The injured area can be resected and an end colos-
tomy or ileostomy performed, and the distal colon can be brought to the abdominal
wall as a mucous fistula or oversewn and left in the abdominal cavity. Finally, a loop
colostomy can be created proximal to a suture line, which is left in the abdominal
cavity. The advantage of colostomy is avoiding an unprotected suture line in the ab-
domen. The disadvantage is that a second operation is required to close the colosto-
my. Often overlooked disadvantages are the complications associated with the crea-
tion of a colostomy, some of which may be fatal. Exteriorized repairs are created by
suspending a repaired perforation or anastomosis on the abdominal wall with an ap-
pliance after the fashion of a loop colostomy. If after 10 days the suture line does not
leak, it can be returned to the abdominal cavity under local anesthesia without subse-
quent risk of leakage. If the repair breaks down before 10 days, it is treated as a loop
colostomy. Healing is successful in 50 to 60% of cases. The advantage is avoidance
of an intraperitoneal suture line when it is at risk of leakage, and the disadvantage is
that 40 to 50% of patients will require colostomy closure. Stomal complications simi-
lar to those of colostomies also can occur with the exteriorization.

Numerous large retrospective and several prospective studies have now


clearly demonstrated that primary repair is safe and effective in the majority of pa-
tients with penetrating injuries. Colostomy is still appropriate in a few patients, but
the current dilemma is how to select them. Exteriorized repair is probably no longer
indicated since most patients who were once candidates for this treatment are now
successfully managed by primary repair. Two methods have been advocated that re-
sult in 75 to 90% of penetrating colonic injuries being safely treated by primary re-
pair. The first is to repair all perforations not requiring resection. If resection is re-
quired due to the local extent of the injury, and it is proximal to the middle colic ar-
tery, the proximal portion of the right colon up to and including the injury is resected
and an ileocolostomy performed. If resection is required distal to the middle colic ar-
tery, an end colostomy is created and the distal colon oversewn and left within the
abdomen. The theory behind this approach is that an ileocolostomy heals more relia-
bly than colocolostomy, because in the trauma patient who has suffered shock and
may be hypovolaemic, assessing the adequacy of the blood supply of the colon is
much less reliable than in elective procedures. The blood supply of the terminal ileum
is never a problem. The other approach is to repair all injuries regardless of the extent
and location (including colocolostomy), and reserve colostomy for patients with pro-
tracted shock and extensive contamination. The theory used to support this approach
is that systemic factors are more important than local factors in determining whether a
suture line will heal. Both of these approaches are reasonable and result in the majori-
ty of patients being treated by primary repairs. When a colostomy is required, regard-
less of the theory used to reach that conclusion, performing a loop colostomy proxi-
mal to a distal repair should be avoided because a proximal colostomy does not pro-
tect a distal suture line.

Complications related to the colonic injury and its treatment may include in-
tra-abdominal abscess, fecal fistula, wound infection, and stomal complications. In-
tra-abdominal abscess occurs in approximately 10% of patients, and most are man-
aged with percutaneous drainage. Fistulas occur in 1 to 3% of patients and usually
present as an abscess or wound infection, which after drainage is followed by contin-
uous fecal output. Most colonic fistulas will heal spontaneously. Wound infection can
be effectively avoided by leaving the skin and subcutaneous tissue open and relying
on healing by secondary intention. The skin can be closed primarily in approximately
60% of patients without developing an infection. However, this treatment should be
reserved for injuries with little contamination and in patients with minimal blood loss
and little subcutaneous fat.

Stomal complications include necrosis, stenosis, obstruction, and prolapse.


Taken together they occur in approximately 5% of patients, and most require reopera-
tion. Necrosis is a particularly serious complication which must be recognized and
treated promptly. Failure to do so can result in life-threatening septic complications
including necrotizing fasciitis.

Rectum

Rectal injuries are similar to colonic injuries with respect to the ecology of
the luminal contents, the structures and blood supply of the wall, and the nature and
frequency of complications. They differ in two important ways: mechanisms of injury
and accessibility. The rectum is often injured by GSWs, rarely by SWs, and frequent-
ly by acts of auto-eroticism and sexual misadventure. The rectum is also subject to
high-pressure injuries which can be caused by air guns or water under high pressure
as used in golf course irrigation systems. Access to the rectum is limited because of
the surrounding bony pelvis.

The diagnosis is suggested by the course of projectiles, the presence of


blood on digital examination of the rectum, and history. Patients in whom a rectal in-
jury is suspected should undergo proctoscopy. Hematomas, contusions, lacerations,
and gross blood may be seen. If the diagnosis is still in question, x-ray examinations
with soluble contrast enemas are indicated. At times it may be difficult to determine
whether an injury is present. These authors believe that these patients should be treat-
ed as if they do have an injury.

The portion of the rectum proximal to the peritoneal reflection is referred to


as the intraperitoneal segment and that distal to the reflection as the extraperitoneal
segment. This distinction is blurred somewhat because the broad posterior aspect of
the intraperitoneal portion could be considered as either. Injuries of the intraperitone-
al portion (including its posterior aspect) are treated as previously outlined in the sec-
tion on colonic injuries. Access to extraperitoneal injuries is so restricted, especially
in the narrow male pelvis, that indirect treatment is usually required. While colosto-
mies proximal to a suture line are avoided in patients with colonic injuries, there is
often no option in patients with extraperitoneal injuries, and sigmoid colostomies are
appropriate for most patients. 79 Properly constructed loop colostomies are preferred
because they are quick and easy to fashion and provide total fecal diversion. Essential
elements include: (1) adequate mobilization of the sigmoid colon so the loop will rest
on the abdominal wall without tension, (2) maintenance of the spur of the colostomy
(the common wall of the proximal and distal limbs after maturation) above the level
of the skin with a one-half-inch nylon rod or similar device, (3) longitudinal incision
in the tenia coli, and (4) immediate maturation in the OR using 3-0 braided absorba-
ble suture. A staple line can be applied across the distal limit to ensure complete di-
version, but it is not necessary and does complicate closure of the colostomy. A mu-
cous fistula is never required and should be avoided because of the risk of necrosis if
the inferior mesenteric or superior rectal arteries were injured or otherwise ligated.

If a perforation is inadvertently uncovered during dissection, it should be re-


paired as described above. Otherwise it is not necessary to explore the extraperitoneal
rectum to repair perforation. Furthermore, it may be extremely difficult or impossible
to accomplish this task. If the injury is so extensive that the surgeon feels it must be
repaired, the patient is better off treated by dividing the rectum at the level of injury,
oversewing or stapling the distal rectum, and creating an end colostomy (Hartmann's
procedure). In rare instances in which the anal sphincters have been destroyed, an ab-
dominoperineal resection may be necessary.

Extraperitoneal injuries of the rectum should be drained via a retroanal inci-


sion. Waldeyer's fascia is particularly tough at this level and may need to be sharply
incised. The drains, either Penrose or closed suction, should be placed close to the
perforation or suture line and should be left in until they fall out spontaneously or
drainage diminishes, which usually occurs within 7 to 10 days. Irrigation of the distal
rectum with various solutions is advocated by some authorities. It does not appear to
be either helpful or harmful in retrospective studies. It may be of benefit in a patient
whose rectum is loaded with feces. If it is done, the irrigation solution should be iso-
tonic and the anus should be mechanically dilated to avoid building up pressure that
might force feces out of an unrepaired perforation. If the patient has a concomitant
bladder injury and adjacent suture lines are created, a flap of viable omentum should
be placed between them to reduce the risk of a rectovesical fistula.

There have been a few reports of treating small extraperitoneal rectal inju-
ries by suture or drainage alone. The outcomes have been acceptable and colostomies
have been avoided. However, there has not been sufficient experience to recommend
this approach since pelvic sepsis associated with rectal injury is highly lethal. Com-
plications are similar in frequency and nature to colonic injuries. Pelvic osteomyelitis
may also occur. Bone biopsy should be performed to secure the diagnosis and bacte-
riology. Culture-specific intravenous antibiotics should be administered for 2 to 3
months. Débridement may be necessary.

Kidneys

There are several unique aspects to the diagnosis and treatment of renal inju-
ries. Three imaging techniques, CT, intravenous (IVP), and arteriography, can be
used to accurately evaluate the extent of a renal injury. However, the contrast material
required for each is nephrotoxic and limits the number of studies that can be per-
formed. The fact that there are two identical organs makes the sacrifice of one a via-
ble therapeutic option. Nearly 95% of all blunt renal injuries are treated nonoperative-
ly. The diagnosis is suspected by the finding of microscopic or gross hematuria and
confirmed by CT or IVP. Most cases of urinary extravasation and hematuria will re-
solve in a few days with bed rest. Persistent gross hematuria can be treated by embo-
lization. Persistent urinomas can be drained percutaneously. Operative treatment is
occasionally necessary for similar lesions that do not respond to these less invasive
measures. If a perinephric hematoma is encountered during laparotomy from blunt
trauma, exploration is indicated if it is expanding or pulsatile. Very large hematomas
should be explored because of the risk of a major vascular injury. Much has been
written about the need for vascular control at the junction of the renal vessels with the
aorta and vena cava prior to entering the hematoma. The authors have not found this
necessary or desirable. If emergent vascular control is necessary, a large curved vas-
cular clamp can easily be placed across the hilum from below, with the clamp parallel
to the vena cava and aorta. Hemostatic and reconstructive techniques used to
treat blunt renal injuries are similar to those used to treat the liver and spleen, alt-
hough two additional concepts are employed: the collecting system should be closed
separately, and the renal capsule preserved to close over the repair of the collecting
system (Fig. 6-59). Permanent sutures should be avoided because of the risk of calcu-
lus formation. The authors prefer absorbable monofilament sutures because of their
lack of abrasiveness. If nephrectomy is being considered and the status of the oppo-
site kidney is unknown, it should be palpated. The presence of a palpably normal op-
posite kidney is assurance that the patient will not be rendered anephric by a unilat-
eral nephrectomy. Unilateral renal agenesis occurs in 1 in 1000 patients.

Basic literature:
51.Oxford Textbook of Surgery (3-Volume Set) 2nd edition (January 15, 2000):
by Peter J. Morris (Editor), William C. Wood (Editor) By Oxford Press
52.Sabiston Textbook of Surgery 17th edition by Courtney M. Townsend Jr.,
Kenneth L. Mattox, B. Mark, MD Evers, Kenneth L., MD Mattox, Courtney
Townsend, Daniel Beauchamp, B. Mark Evers, Kenneth Mattox W.B. Saun-
ders Company (June, 2004)
53.Schwartz´s Principles of Surgery 8th Edition F. Charles Brunicardi. Copyright
©2007 the McGraw-Hill Companies.
54.Hospital surgery/ Edited by L. Kovalchuk et al. - Ternopil: Ukrmedknyha,
2004. - 472 p.

Additional literatures:
1. Poggetti RS, Moore EE, Moore FA, et al: Balloon tamponade for bilobar transfix-
ing hepatic gunshot wounds. J Trauma 33:694, 1992. [PMID: 1464918]
2. Cocanour CS, Moore FA, Ware DN, et al: Age should not be a consideration for
nonoperative management of blunt splenic injury. J Trauma 48:606, 2000. [PMID:
10780591]
3. Peitzman AB, Heil B, Rivera L, et al: Blunt splenic injury in adults: Multi-
institutional study of the Eastern Association for the Surgery of Trauma. J Trauma
49:177, 2000.
4. Bradley EL, Young PR, Chang MC, et al: Diagnosis and initial management of
blunt pancreatic trauma. Ann Surg 227:861, 1998.

Tests for initial level of knowledge, keys for tests:

1. Which of the following conditions is most likely to follow a compression-type ab-


dominal injury?
A. Renal vascular injury
B. Superior mesenteric thrombosis
C. Mesenteric vascular injury
D. Avulsion of the splenic pedicle
E. Diaphragmatic hernia

2. Blunt trauma to the abdomen most commonly injures which of the following or-
gans?
A. Liver
B. Kidney
C. Spleen
D. Intestine
E. Pancreas
3. Following blunt abdominal trauma, a 12-year-old girl develops upper abdominal
pain, nausea, and vomiting. An upper gastrointestinal series reveals a total obstruction
of the duodenum with a “coiled spring” appearance in the second and third portions.
Appropriate management is
A. Gastrojejunostomy
B. Nasogastric suction and observation
C. Duodenal resection
D. TPN to increase the size of the retroperitoneal fat pad
E. Duodenojejunostomy

4. An elderly pedestrian collides with a bicycle-riding pizza delivery man and suffers
a unilateral fracture of his pelvis through the obturator foramen. You would manage
this injury by
A. External pelvic fixation
B. Angiographic visualization of the obturator artery with surgical explo-
ration if the artery is injured or constricted
C. Direct surgical approach with internal fixation of the ischial ramus
D. Short-term bed rest with gradual ambulation as pain allows after 3 days
E. Hip spica

5. When operating to repair civilian colon injuries


A. A colostomy should be performed for colonic injury in the presence of
gross fecal contamination
B. The presence of shock on admission or more than two associated in-
traabdominal injuries is an absolute contraindication to primary colonic
repair
C. Distal sigmoidal injuries should not be repaired primarily
D. Right-sided colonic wounds should not be repaired primarily
E. Administration of intravenous antibiotics with aerobic and anaerobic
coverage has not been shown to decrease the incidence of wound infec-
tions after repair of colonic injuries

6. A 36-year-old man sustains a gunshot wound to the left buttock. He is hemody-


namically stable. There is no exit wound, and an xray of the abdomen shows the bul-
let to be located in the right lower quadrant. Correct management of a suspected rec-
tal injury would include
A. Barium studies of the colon and rectum
B. Barium studies of the bullet track
C. Endoscopy of the bullet track
D. Angiography
E. Sigmoidoscopy in the emergency room

7. Correct statements regarding blunt trauma to the liver include which of the follow-
ing?
A. Hepatic artery ligation for control of bleeding is associated with de-
creased morbidity and mortality
B. The incidence of intraabdominal infections is significantly lower in pa-
tients with abdominal drains
C. Intracaval shunting has dramatically improved survival among patients
with hepatic vein injuries
D. Nonanatomic hepatic debridement, with removal of the injured frag-
ments only, is preferable to resection along anatomic planes
E. Major hepatic lacerations that are sutured closed will result in intrahe-
patic hematomas, hemobilia, and bile fistulas

8. 18-year-old high school football player is kicked in the left flank. Three hours later
he develops hematuria. His vital signs are stable. The diagnostic tests performed re-
veal extravasation of contrast into the renal parenchyma. Treatment should consist of
A. Resumption of normal daily activity excluding sports
B. Exploration and suture of the laceration
C. Exploration and wedge resection of the left kidney
D. Nephrostomy
E. Antibiotics and serial monitoring of blood count and vital signs

9. An 18-year-old high school football player is kicked in the left flank. Three hours
later he develops hematuria. His vital signs are stable. Initial diagnostic tests in the
emergency room should include which of the following?
A. Retrograde urethrography
B. Retrograde cystography
C. Arteriography
D. Intravenous pyelogram
E. Diagnostic peritoneal lavage

10. True statements concerning penetrating pancreatic trauma include


A. Most injuries do not involve adjacent organs
B. Management of a ductal injury to the left of the mesenteric vessels is
Roux-en-Y pancreaticojejunostomy
C. Management of a ductal injury in the head of the pancreas is pancreati-
coduodenectomy
D. Small peripancreatic hematomas need not be explored to search for
pancreatic injury
E. The major cause of death is exsanguination from associated vascular in-
juries

Keys for tests


1 2 3 4 5 6 7 8 9 10
E C B D B E B E B E

Tests for final level of knowledge, keys for tests:


1. A 22-year-old man sustains a gunshot wound to the abdomen. At exploration, an
apparently solitary distal small-bowel injury is treated with resection and primary
anastomosis. On postoperative day 7, small-bowel fluid drains through the operative
incision. The fascia remains intact. The fistula output is 300 mL/day and there is no
evidence of intraabdominal sepsis. Correct treatment includes
A. Early reoperation to close the fistula tract
B. Broad-spectrum antibiotics
C. Total parenteral nutrition
D. Somatostatin to lower fistula output
E. Loperamide to inhibit gut motility

12. A 53-year-old man is involved in a high-speed automobile collision. He has


moderate respiratory distress. Physical examination shows no breath sounds over
the entire left chest. Percussion is unremarkable. Chest x-ray shows multiple air flu-
id levels in the left chest. What is it?

A. This one of the classic ruptured spleen

B. The classic for traumatic rupture of the aorta

C. Classic for traumatic diaphragmatic rupture


D. The classic for traumatic rupture of the liver
E. Hematoma of the rectus sheath
13. A 19-year-old gang member is shot in the abdomen with a .38-caliber revolv-
er. The entry wound is in the epigastrium, to the left of the midline. The bullet is
lodged in the psoas muscle on the right. He is hemodynamically stable, the ab-
domen is moderately tender. At exploratory laparotomy for the patient described
in the previous question, examination shows clean, punched-out entrance and ex-
it wound sin the transverse colon. Appropriate manage is:
A. Get a sygmostomy
B. Get a transversestomy
C. Drainage of the abdominal cavity
D. Get left-side gemicolectomy
E. Get a right-side gemicolectomy

14 A 19-year-old gang member is shot once with a .38-caliber revolver. The entry
wound is in the left mid-clavicular line, 2 inches below the nipple. The bullet is
lodged in the left paraspinal muscles. He is hemodynamically stable, but he is
drunk and combative and physical examination is difficult to perform. What is op-
timal tactic?
A. Ultrasound investigation
B. Get a laparotomy
C. Diagnostic peritoneal lavage
D. Computer tomography
E. X-ray chest examination
15 A 42-year-old man is stabbed in the belly by a jealous lover. The wound is
lateral to the umbilicus, on the left, and omentum can be seen protruding
through it. What is optimal tactics?
A. Computer tomography
B. Ultrasound investigation

C. Get a laparotomy
D. Diagnostic peritoneal lavage
E. Ultrasound investigation

16. In the course of a domestic fight, a 38-year-old obese woman is attacked


with a 4-inch-long Swiss blade. In addition to several superficial lacerations,
she was stabbed in the abdomen. She is hemodynamically stable, and does not
have any signs of peritoneal irritation. Correct treatment includes
A. Penetrating abdominal wounds have to be surgically explored
B. Diagnostic peritoneal lavage
C. Direct board-spectrum antibiotics
D. Total parenteral nutrition
E. X-ray abdomen examination

17. A 31-year-old woman smashes her car against a wall. She has multiple injuries
including upper and lower extremity fractures. Her blood pressure is 75 over 55,
with a pulse rate of 110, and a CVP of 0. On physical examination, she has a tender
abdomen, with guarding and rebound on all quadrants. What is it?

A. Acute diverticulitis

B. Hematoma of the rectus sheath

C. Solid organs are smashed

D. Injure of the kidney

E. injure of the abdomen wall


18. A 31-year-old woman smashes her car against a wall. She has multiple injuries
including upper and lower extremity fractures. Her blood pressure is 135 over 75,
with a pulse rate of 82. On physical examination she has a tender abdomen, with
guarding and rebound on all quadrants. What is it?
A. Solid organs will be bleed when smashed.
B. Acute peritonitis
C. Pancreatoduodenal injuries
D. Rectus-sheath hematoma
E. Traumatic shock

19. A 26-year-old woman has been involved in a car wreck. She has fractures
in both upper extremities, facial lacerations, and no other obvious injuries.
Chest x-ray is normal. Shortly thereafter she develops hypotension, tachycar-
dia, and dropping hematocrit. Her CVP is low. What is it?
A. Traumatic shock
B. The classic for traumatic diaphragm rupture
C. Acute pancreatitis
D. Acute mesenteric thrombosis
E. Obviously blood loss, it has to be in the abdomen.

20. A 27-year-old intoxicated man smashes his car against a tree. He is tender
over the left lower chest wall. Chest x-ray shows fractures of the 8th, 9th, and
10th ribs on the left. He has a blood pressure of 85 over 68 and a pulse rate of
128. What is it?
A. Hematoma of the rectus sheath
B. This one is a classic ruptured spleen
C. The classic for traumatic rupture of the aorta
D. The classic for traumatic diaphragm rupture
E. The classic for traumatic rupture of the hepar

Keys for tests

1 2 3 4 5 6 7 8 9 10
C C B B C A C A E B

Tasks for the final level of knowledge.

1. A multiple trauma patient is receiving massive blood transfusions as the


surgeons are attempting to repair many intraabdominal injuries. It is then
noted that blood is oozing from all dissected raw surfaces, as well as from his
IV line sites. His core temperature is normal. What is it?

The answer is signs of coagulopathy in this setting require a shotgun approach to


treatment.

2.. An exploratory laparotomy for multiple intraabdominal injuries has lasted 3.5
hours, during which time multiple blood transfusions have been given, and
several liters of Ringer lactate have been infused. When the surgeons are ready
to close the abdomen they find that the abdominal wall edges cannot be
pulled together without undue tension. Both the belly wall and the abdominal
contents seem to be swollen. What is it?
The answer is the so-called abdominal compartment syndrome

3. A 22-year-old gang member arrives in the ER with multiple gunshot wounds to the
chest and abdomen. He is diaphoretic, pale, cold, shivering, anxious, and asking for a
blanket and a drink of water. His blood pressure is 60 over 40. His pulse rate is 150,
barely perceptible. He has big distended veins in his neck and forehead. He is breathing
okay and has bilateral breath sounds and no tracheal deviation. What is it?

The answer is pericardial tamponade

4. A motorcycle daredevil attempts to jump over the 12 fountains in front of


Caesar's Palace Hotel in Las Vegas. As he leaves the ramp at very high speed,
his motorcycle turns sideways and he hits the retaining wall at the other end,
literally like a rag doll. At the ER he is found to be remarkably stable, although
he has multiple extremity fractures. A chest x-ray shows fracture of the left first
rib and widened mediastinum. What is it?

The answer is classic for traumatic rupture of the aorta

5. During a domestic dispute a young woman is stabbed in the chest with a 6-


inch-long butcher knife. On arrival at the ER she is found to have an entry
wound just to the left of the sternal border, at the fourth intercostal space. Her
blood pressure is 80 over 50, her pulse rate is 110, and she is cold, pale, and
perspiring heavily. She has big distended neck and facial veins, but she is
breathing normally and has bilateral breath sounds. What is it?

The answer is classical traumatic diaphragmatic rupture


Materials for the self-study of the students
Main tasks Notes (instructions)
Repeat:
1. Anatomy of organs of abdominal
cavity, structure of the peritoneum and
anatomy intestinum, features of blood circu-
-To represent the methods of diagnostics of dis-
lation of liver and spleen.
eases of abdomen and peritoneum as a table
2. Physiology of peritoneum and or-
-To make the flow diagram of mechanisms of
gans of abdominal cavity.
damage of abdomen
3. Pathogenesis of development of
complications from abdominal trauma.
4. Morphological changes in the organs
of abdominal cavity.
Study:
1. Techniques of hemostasis at injures
of the liver. -To conduct differential diagnosis with the
2. Types of accesses to the organs of damages of organs of abdomen
abdominal cavity. -To conduct differential diagnosis of patholo-
3. Tactics at the thoracoabdominal trauma. gies of the abdomen
4. Tactics at connections of head trauma and
trauma of abdomen.
Study guide #12
“Specific features of the acute abdominal viscus diseases in the pregnant. Diag-
nosis and differential diagnosis. Peculiarities of examination. Choice of surgical
tactics. Prevention of premature births. Manifestation of acute surgical diseases
of organs of abdominal cavity in elderly patients and children.”

Overview
Pregnant women are subject to the same surgical diseases as their nonpregnant coun-
terparts. An estimated 1% to 2% of pregnant women require surgical procedures, with
nonobstetric surgery necessary in up to 1% of pregnancies in the United States each
year. Most indications for surgical intervention are those that are common for the pa-
tient’s age group and unrelated to pregnancy, such as acute appendicitis, symptomatic
cholelithiasis, breast masses, or trauma. The pregnant patient offers unique challenges
to the surgeon. Changes in maternal anatomy and physiology and safety of the fetus
are among the issues of which the surgeon must be cognizant. The presentation of
surgical diseases in the pregnant patient may be atypical or may mimic signs and
symptoms associated with a normal pregnancy, and a standard evaluation may be un-
reliable due to pregnancy-associated changes in diagnostic tests or laboratory values.
Finally, many physicians may be more conservative in diagnostic evaluation and
treatment. Any of these factors may result in a delay in diagnosis and treatment, ad-
versely affecting maternal and fetal outcome. Although consultation with an obstetri-
cian is ideal when caring for a pregnant patient, the surgeon needs to be aware of cer-
tain fundamental principles when such a resource is unavailable. This chapter dis-
cusses the key points in caring for the pregnant patient who presents with nonobstet-
ric surgical disease.
The portion of the population older than age 65 years is expected to grow from
the present 12.7% to approximately 20% by the year 2030. The most rapidly growing
segment of this older population is persons older than age 85. Their number is ex-
pected to increase sixfold, reaching nearly 20 million by 2050. Social Security, Medi-
care, and Medicaid benefits to the elderly currently consume one third of U.S. spend-
ing and have the potential to consume the entire federal budget by 2012. Therefore,
the simple increase in number of older persons is going to stress the health care indus-
try. This will occur even though the actual cost for care of older persons is relatively
low when compared with younger counterparts. As the number of older patients in-
creases, it becomes increasingly important for every surgeon to have a clear under-
standing of the factors that influence the life expectancy of his or her older patients.
This is essential when weighting the risks of operation against the benefits of survival
time and quality of life. When comorbid disease is present, life expectancy decreases.
When making decisions about surgical treatment in older patients, it is important to
consider the actual life expectancy of the individual patient based on his or her over-
all health. Patients with serious comorbidity may not live long enough to gain the
benefit from surgery so the risk of surgery becomes an even greater concern.
Educational aims:
28.Interrogation and clinical inspection of pregnant and elderly patients with surgical
diseases.
29.To determine the clinical signs of surgical diseases in pregnant women and elderly
patients.
30.To develop a plan of examination of the pregnant women with suspected surgical
diseases.
31.To estimate the results of auscultation, palpation and abdominal percussion of
pregnant women.
32.To make a differential diagnosis of acute surgical diseases, in elderly patients and
pregnant women
33.To determine the indications for operative treatment of elderly patients with severe
comorbidity.
34.To estimate efficiency of treatment and prognosis of surgical disease in pregnant.

A student must know:


28.Anatomical and physiological peculiarities of pregnant women
29.Physiological changes in elderly patients.
30.What factors may influence the clinical course of the diseases in pregnant
women and elderly patients.
31.Clinical presentations of acute surgical diseases in pregnant women.
32.Diagnostic methods that can be used in pregnant patients.
33. Main principals of surgical interferences in elderly patients and pregnant
women.

A student must be able to:


19.To collect and estimate the complaints in elderly patients with suspected surgi-
cal pathology.
20.To define the rational quantity of laboratory and instrumental methods of in-
vestigation in pregnant patients with suspected acute surgical diseases.
21.To define the indications for surgical interventions and to choose the operation
method in pregnant women with acute surgical pathology.
22.Prescribe post-operative treatment and prevent premature birth.

Terminology.
Term Definition
Elderly patients The person of 65 years and more.
An aggregation of platelets, fibrin, clotting factors, and cel-
lular elements of the blood attached to the interior wall of
Thrombus
vein or artery, sometimes occluding the lumen of the ves-
sels.
Pulmonary Embolism The blockage of pulmonary artery by fat, air, tumor tissue,
(PE) or thrombus that usually arises from the peripheral vein
(most frequently one of deep veins of the legs).
Pulmonary Infarction Necrosis in part of a lung caused by an obstruction in
(PI) branch of a pulmonary artery.
Thrombophilias Hypercoagulable disorders
A surgical incision into an artery for the removal of an em-
Embolectomy bolus or clot, performed as emergency treatment for arterial
embolism.

Content
Appendicitis in pregnancy
The diagnosis of appendicitis in pregnancy is difficult because the appendix is
displaced by the gravid uterus. Early in the course of pregnancy the appendix remains
in its normal position, and diagnosis is routine. By the middle of the second trimester,
however, the appendix becomes displaced superiorly, attaining a position in the right
upper flank or epigastrium. Appendicitis may be easily mistaken for pyelonephritis or
cholecystitis. The abdominal wall is lifted from the appendix by the gravid uterus,
and muscular laxity occurs: the abdominal findings associated with peritoneal irrita-
tion by the inflamed appendix may therefore be fewer than one might expect in the
non-pregnant woman. Leukocytosis is a normal physiological response of pregnancy
(up to 12 500 leukocytes/mm3) and cannot be relied upon to help confirm the diagno-
sis of appendicitis. White blood-cell counts as high as 25 000 leukocytes/mm3 are not
unusual in pregnant women with appendicitis.
Rectus-sheath haematoma
This rare condition can mimic intra-abdominal pathology. The haematoma de-
velops from rupture of the inferior epigastric artery in the lower half of the abdomen.
Pregnant women are particularly affected; there is sometimes a history of injury, and
the right side is involved twice as commonly as the left. The onset of the pain is acute
and it is often accompanied by nausea and vomiting. There is marked tenderness in
the iliac fossa so that it is easy to misdiagnose appendicitis. Two physical signs will
reveal the true diagnosis. It may be possible to show that the tenderness and the
swelling if there is one, are confined to the abdominal wall. Secondly, bruising of the
skin may be visible. Sometimes this only appears a few days later and it is often at a
distance from the site of maximum tenderness. Ultrasonography and CT will both
make the diagnosis for certain. Once diagnosed the haematoma will slowly resolve
with rest; in patients who undergo surgery the diagnosis becomes apparent as the ab-
dominal incision is made. It is then worth tying off the bleeding vessel; this is also
needed in the rare patient in whom the haemorrhage does not stop. Similar bleeding
sometimes arises from spontaneous rupture of an intercostal artery. The dramatic
bruising of the abdominal wall spreading round from the lower thorax, often in a
segmental distribution, is unmistakable. The haematoma will normally resolve with
rest.

Endoscopic surgery in pregnancy


Experimental and clinical data have answered most concerns over the safety of
laparoscopic cholecystectomy or appendectomy during pregnancy. Studies on fetal
sheep have proved that maternal and fetal pH is linearly related, so fetal acidosis may
be avoided by maintaining respiratory alkalosis in the mother. The increased intra-
abdominal pressure caused by pneumoperitoneum is much less than the physiologic
pressures tolerated by the fetus during mid-pregnancy uterine contractions. A large
clinical experience of well over 100 laparoscopic cholecystectomies in pregnancy has
produced acceptable results. Nevertheless, all elective operations should be delayed
until after delivery. If delay puts the mother at undue risk, operations during the sec-
ond trimester are the next best option. During all procedures on pregnant patients, the
fetus should be carefully shielded from intraoperative X-rays and monitored via
transvaginal ultrasound. Initial access to the abdomen in the pregnant woman should
be well above the uterine fundus, which reaches the umbilicus at 20 weeks' gestation.
Most endoscopic surgeons favor the open (Hasson) approach over direct-puncture
laparoscopy to avoid the chance of uterine injury with the Veress needle.
Thromboprophylaxis in pregnancy
Clearly, low-dose aspirin is safe for use during pregnancy and is also an effec-
tive antiplatelet therapy in the prophylaxis of thromboembolism in a surgical setting.
It is the preferred treatment modality for low-risk patients. Without more rigorous
thromboprophylaxis, however, a pregnant woman who has already experienced a
thrombotic event is at high risk of recurrence. Therefore, subcutaneous heparin is
recommended in addition to low-dose aspirin; either 10 000 U of unfractionated hep-
arin can be self-administered subcutaneously twice a day, or a low-molecular-weight
heparin such as enoxaparin at a dosage of 40 mg can be self-administered subcutane-
ously once a day. Heparin does not cross the placenta or pass into breast milk, making
it safe for use in pregnancy as far as the fetus is concerned. Women taking heparin for
more than 6 weeks should be warned, however, that there is a risk of thrombocytope-
nia and osteopenia.
Crohn's disease in pregnancy
Female patients with Crohn's disease are as fertile, in general, as a control pop-
ulation. However, patients should avoid conceiving when the disease is active as
there tends to be a high incidence of miscarriage. Apart from this, pregnancy seems to
have little effect on the Crohn's disease and therefore patients should not be discour-
aged from having children if they wish. Should a relapse occur during pregnancy, it
should be treated as vigorously as if the patient was not pregnant. Corticosteroids and
mesalazine should be used as indicated, and should in no way be withheld. Patients
conceiving while on treatment should be reassured that the developing fetus is not at
risk and the drugs should only be withdrawn as dictated by the activity of the disease.
Some patients conceive while on azathioprine. Once again, there is no hard evidence
that this will harm the fetus. If the Crohn's disease has been difficult to control and
has only recently gone into remission with the introduction of azathioprine, the drug
should be continued. A recent audit on the outcome of pregnancies in women with
Crohn's disease who were receiving azathioprine has not shown an increased inci-
dence of congenital abnormalities. Methotrexate is absolutely contraindicated in
pregnancy.
Colic
Abnormal contraction of smooth muscle causes the regular and intermittent
pain that is called colic. Within the abdomen only the gut, the renal tract, the uterus,
and the biliary tract cause such a pain. The site and the distribution of the pain are dif-
ferent in each case and so they are easy to tell apart except, sometimes, for right-sided
renal colic and biliary pain. The most severe intestinal colic accompanies gastroenter-
itis, although it is a classic sign of small-bowel obstruction. Stones are the common
cause of renal and biliary colic, although blood clot and pus can cause ureteric colic
quite as severe as that due to a stone. Most, but not all, women know when they are
pregnant and are about to deliver a baby, but uterine colic due to a miscarriage, or
even severe dysmenorrhoea, sometimes present as acute abdominal pain.
Appendicitis in the elderly
Elderly patients tend to present late in the course of appendicitis and with less
well-defined symptoms; the incidence of perforation is therefore higher. Most elderly
patients with appendicitis have a history of several days of poorly defined abdominal
pain, anorexia, and fever. Rarely is the pain well localized to McBurney's point; ra-
ther, it is generally described as being in the right side of the abdomen. Most old pa-
tients have fever and abdominal tenderness at the time of presentation with appendici-
tis. Tenderness is either in the right lower quadrant, the right flank, or is diffuse from
the effects of free perforation. The psoas and obturator signs are unhelpful in old pa-
tients because almost every manoeuvre is equally uncomfortable.

Blunt trauma
As previously noted, compressive forces to the chest wall may cause fractured
ribs. Pain control is a primary goal in the elderly patient, the smoker, or in any patient
with multiple or bilateral fractured ribs. This is accomplished by intercostal nerve
blocks with 0.25 per cent bupivacaine, intrapleural instillation of 0.50 per cent bupi-
vacaine, or the insertion of an epidural catheter for continuous administration of fen-
tanyl citrate. An associated problem in patients with multiple rib fractures is blood
loss, which has long been reported to be 125 ml/rib (source unknown).
Confounding the clinical evaluation of shock is the variation in physiologic re-
sponse related to age and medical conditions. The elderly are more sensitive to blood
loss, in part because many are on medications which alter the normal physiologic re-
flexes. Evaluation of infants and children can be confusing because they have ex-
traordinary compensatory mechanisms. The pregnant patient suffering blood loss
from injury can herself have adequate perfusion while the fetus is subjected to an is-
chemic insult. The timely diagnosis of shock in injured patients often depends upon
repeated evaluations for signs of hypoperfusion.
Antivagal actions
The response to many noxious stimuli is the development of a vagally medicat-
ed bradycardia. Intramuscularly administered anticholinergics are ineffective in pre-
venting these responses. Routine use of antisialogogues has major disadvantages: the
patient suffers an unpleasant dry mouth during the preoperative period, and dry mu-
cous membranes are sticky and easily damaged during laryngoscopy and intubation.
Present use of these agents tends to be reserved for infants, and those situations where
a dry mouth is advantageous (such as for intraoral surgery). Hyoscine is the most po-
tent of the antisialogogues available and has the additional advantage of producing
amnesia and sedation. In the elderly patient, however, there is a significant incidence
of perioperative confused states.
Hypoxia and hypocapnia
Hypoxia and hypercapnia have a synergistic effect, increasing cerebral vasodi-
latation and hence intracerebral blood volume. Arterial oxygen tension should be
maintained above 100 mmHg and PaCO2 should not exceed 40 mmHg (5.3 kPa).
During anaesthesia, PaCO2 should be maintained in the range 30 to 35 mmHg (3.5–
4.5 kPa). The elderly patient is less tolerant of prolonged hypocapnia, which has been
shown to lead to postoperative memory impairment.
Fluid and electrolyte imbalance
Complications of fluid and electrolyte imbalance are often seen in the elderly
or debilitated patient, in the hypertensive patient treated with diuretics, and in diabetic
and neurosurgical patients. Hyponatraemia, hypocalcaemia, and hypermagnesaemia
have all been implicated in delayed return of consciousness.
Nervous system
Confusion is common in the perioperative period, especially in the elderly pa-
tient. Diagnosis is frequently difficult and management often suboptimal. Diagnosis
is frequently made by exclusion of possible causes and in many cases no obvious
cause for the acute brain syndrome is ever discovered. Relatively inexperienced
house staffs (interns) often have to manage patients with acute postoperative confused
states. Hypoxia must be excluded, either by oximetry or blood-gas estimation. Re-
view of the anaesthetic chart or recovery-room notes will often reveal a likely cause,
but in the majority none is ever ascertained. Management involves reassurance of the
patient and staff, combined with measures to prevent damage to suture lines, intrave-
nous equipment, and wound drains. Sedation should be used cautiously if at all. Ade-
quate analgesia must be ensured.
Comorbidity
It is important to take into account any comorbidity, the level of physiological
reserve, and the patient's biological age, which, individually or together, may make
recovery highly improbable. Elderly patients have a higher incidence of cardiovascu-
lar and respiratory disease; non-specific immune deficiency can be demonstrated in
them, due to poor nutrition, hormonal changes, or other mechanisms that reduce re-
sistance to hospital-acquired infection. The effects of a long, debilitating, critical ill-
ness in the elderly person often mean they may have little or no chance of returning to
independent life. Ideally, biological rather than chronological age should be consid-
ered and, consequently, relatively few intensive care units have an upper age limit for
admission. The nature and severity of specific premorbid medical conditions also
have a large effect on management and outcome in the intensive care unit. For exam-
ple, the patient with chronic obstructive pulmonary disease, severe exertional dysp-
noea, and grossly impaired lung function is less likely to tolerate long-term mechani-
cal ventilation without barotrauma and infectious complications. Weaning may be
protracted or even fail, with the possibility of chronic long-term ventilation support.
The presence of advanced, solid, or haematological malignancy may lead the patient,
the patient's family, and the doctors to consider intensive care to be inappropriate.
Anaesthesia and surgery carry increased risks in patients suffering from cardio-
vascular disease. This problem is accentuated in elderly individuals. Ischaemic heart
disease, chronic infection of the lower respiratory tract, and cardiac failure are the
disorders most commonly associated with postoperative deaths. Over the last decade,
better preoperative assessment and the introduction of more sophisticated monitoring
have increased the safety of surgery in patients with cardiovascular disease, as it has
become possible for the anaesthetist to detect changes in the circulation before life-
threatening complications occur. Nevertheless, myocardial infarction, progressive
myocardial ischaemia, dysrhythmias, congestive cardiac failure, and cerebrovascular
accidents continue to occur relatively frequently, reflecting the trend to undertake in-
vasive surgical procedures even in the severely ill patient. Indeed, almost one-third of
all surgical patients have either coronary heart disease or associated risk factors.
Moreover, despite advances in anaesthetic technique and postoperative pain relief, the
perioperative period imposes prolonged stress.
Hypertension in the elderly
Isolated systolic hypertension is due predominantly to the loss of elasticity of
the aorta and its major branches. The high pressure increases myocardial oxygen con-
sumption and may cause myocardial ischaemia. However, the treatment of isolated
systolic hypertension may be associated with subjective complaints, and on occasion,
objective deterioration of cardiac, cerebral, or renal function; in addition titration of
blood pressure is often difficult. Treatment of purely systolic hypertension in the el-
derly is probably not justified before surgery, but adequate cardiovascular monitoring
is essential to enable excessive hypo- or hypertension to be detected and treated im-
mediately.
When both systolic and diastolic pressures are elevated, the risks of cardiovas-
cular complications are increased in elderly as well as in younger patients. Antihyper-
tensive treatment reduces the risk of complications but it is important to achieve the
reduction gradually in order for cerebral autoregulation to return to normal limits.
It is often assumed that surgery of limb vessels is relatively well tolerated be-
cause it does not cause major haemodynamic instability. While peripheral vascular
surgery is better tolerated than aortic surgery, cardiovascular complications and fatali-
ty are still more frequent than in non-vascular surgery, reflecting the association of
peripheral vascular disease with hypertensive and coronary disease. Indeed, in elderly
patients, vascular surgery for limb salvage carries a mortality rate of up to 16 per
cent.
Valvular disease
The incidence of rheumatic fever has fallen in the Western world, but many el-
derly patients still suffer from rheumatic heart disease; others suffer from degenera-
tive valvular disease and the prevalence of valvular disease in people over the age of
65 may be as high as 4 per cent.
Minor tranquillizers in the benzodiazepine class are a useful adjunct to psycho-
logical support. Shorter acting agents (for example alprazolam, lorazepam, and oxa-
zepam) are preferable for use in patients who are elderly or debilitated. Oxazepam
and diphenhydramine are most easily metabolized in the liver and are the anxiolytic
and sedative hypnotic agents of choice in patients with impaired hepatic function. The
prescribing physician should be aware of the half-life and the potency of psychotropic
agents and of the patient's past psychopharmacologic history. Patients with debilitat-
ing anxiety should be referred for psychiatric assessment.
Sump ulcers
So-called sump ulcers may develop on the greater-curve aspect of the gastric
antrum and lower body of the stomach in patients, particularly the elderly individual,
taking analgesic medication. Such ulcers develop because of the combined effect of
gravity and the corrosive action of the drugs. Occasionally, these sump ulcers pene-
trate through the gastric wall and result in the formation of a gastrocolic fistula.
Pneumothorax
The tremendous success of VATS in the treatment of primary spontaneous
pneumothorax has led to earlier referral by physicians and increased acceptance by
patients for surgery. Stapled resection of apical bullas followed by mechanical pleu-
rodesis remains the most frequently used technique, although more cost-effective
means of eliminating the bullas (like suturing or looping) have been developed. While
cases of primary spontaneous pneumothorax are easily approachable by VATS,
treatment of secondary spontaneous pneumothorax (with established lung pathology
like emphysema or pneumoconiosis) requires more clinical judgment. Patients with
difficult adhesions to take down may be more suitable for thoracotomy, while those
who are elderly with multiple comorbidities may benefit more from a chemical pleu-
rodesis (we prefer talc slurry) if the lung can be fully re-expanded.
Acute cholecystitis
Extremely debilitated or elderly patients are often exceptions to the ‘equivalen-
cy rule.' While the most definitive management of choledocholithiasis might be chol-
ecystectomy and exploration of the bile duct (or endoscopic sphincterotomy), even
the laparoscopic approach may pose too great a physiologic challenge. In selected
cases, the surgeon may choose to provide a ‘less ideal' intervention to minimize iatro-
genic harm. For example, acute cholecystitis in a debilitated patient may be managed
by tube cholecystostomy, and cholangitis resolved by placing an endoscopic or percu-
taneous biliary stent. Although drainage is less durable than laparoscopic cholecystec-
tomy and bile-duct exploration, the reversal of life-threatening sepsis will permit
more definitive therapy (open or endoscopic) at a safer time.
Endoscopic surgery in elderly and infirm individuals
Laparoscopic cholecystectomy provides an alternative approach for debilitated
patients with symptomatic cholelithiasis. However, its minimally invasive nature does
not imply minimal physiologic impact. These individuals require great care in anes-
thesia, with Swan–Ganz and arterial catheters to monitor the physiologic derange-
ments created by the positive-pressure pneumoperitoneum. In some cases, intraopera-
tive management may be complicated by the laparoscopic access. Much of the ad-
vantage of minimally invasive surgery comes to bear after the operation, since a great
deal of the morbidity incurred by debilitated patients derives from postoperative pain
and impaired postoperative mobilization. Pulmonary complications, urinary-tract sep-
sis, deep venous thrombosis, pulmonary embolism, congestive heart failure, and my-
ocardial infarction are more likely if patients are unable to walk and reclaim perioper-
ative fluid from the interstitium. By allowing rapid and early mobilization, laparo-
scopic surgery has lessened the morbidity associated with procedures in elderly and
infirm patients. Patients in septic shock and those with severe cardiopulmonary com-
promise may benefit from procedures such as percutaneous cholecystostomy and en-
doscopic papillotomy. Such options may be safer than a laparoscopic operation, espe-
cially if there is a high likelihood for conversion to open surgery.
Aortic aneurysms
Aortic aneurysms are very rare in people under the age of 55 years and before
that age are virtually confined to patients with Marfan, Ehlers–Danlos, or arteria
magna syndromes. The common idiopathic abdominal aortic aneurysm is largely a
disease of elderly men. Comparison of fatalities from ruptured abdominal aortic aneu-
rysm by age and sex show that deaths are 13 times more common in men than in
women at age 60 to 65 years, but over 80 years of age are only four times more com-
mon in men than in women. At age 85 almost three times as many women as men are
still alive, so among the very elderly the numbers of men and women presenting with
ruptured aneurysms are similar. The changing pattern of presentation with age, com-
bined with an increase in the number of elderly people in the populations of most
wealthy nations, has led some surgeons to conclude erroneously that abdominal aortic
aneurysm has increased in incidence disproportionately in women. The common ab-
dominal aortic aneurysm of elderly men has been labelled as ‘atherosclerotic'. This
classification has little justification, has paralysed thinking, and needs to be re-
examined. It is interesting to note that aneurysmal disease is encumbered by more
than its fair share of unhelpful, or frankly misleading, descriptive terms, among which
are atheromatous, mycotic, inflammatory, dissecting, and arteriovenous aneurysms.
In the elderly individual the aorta, in common with every other artery, will have ob-
vious features of atherosclerosis but this is not enough evidence to make credible a
pathological diagnosis that does not fit with many known facts about the disease.
Complications of peptic ulcer
Complications of peptic ulcer account for 4500 deaths a year in the United
Kingdom. The surgeon, nowadays, is mainly concerned with the complications of
peptic ulcer rather than elective treatment. Unfortunately, so many of the patients are
elderly with other serious conditions, such as atrial fibrillation treated with anticoagu-
lants, which makes emergency surgery more hazardous.
Perforated duodenal ulcer, is there a place for non-operative treatment?
Although surgery is normally the correct treatment for perforated duodenal ul-
cer, the whole patient and the comorbidity need to be taken into account. Perforations
may seal themselves by adherence to liver, gallbladder, or omentum. If an elderly, un-
fit patient presents with a 3-day history suggesting perforation but does not show the
signs of generalized peritonitis, and is otherwise a very poor operative risk, then non-
operative treatment is entirely reasonable. This would mean nasogastric suction, in-
travenous fluids and antibiotics, and may be undertaken despite the presence of air
under the diaphragm on the radiograph. A careful watch must be kept to ensure the
patient does not deteriorate further. Abscesses caused by the perforation can often be
drained percutaneously later.

Haemorrhage
All haemorrhage from peptic ulcers is potentially lethal and temporary cessa-
tion of bleeding is no cause for complacency. Bleeding peptic ulcer accounts for 70
per cent of all upper gastrointestinal bleeds, and the general mortality is 5 to 10 per
cent. Thirty per cent of patients who bleed from peptic ulcers have no previous defi-
nite history or previous diagnosis of ulcer disease. The mortality among those who
need surgery is higher, around 20 per cent, because their bleeding is the most severe.
Sadly, the mortality has not improved over the last 50 years because, despite fibre op-
tic endoscopy and other advances, the patients who bleed are increasingly elderly
with many concurrent illnesses. Important points in their history include previous
peptic ulcer, alcohol abuse, drugs (particularly non-steroidal), and anticoagulants, and
cardiovascular and pulmonary disease. Important signs are the stigmata of liver dis-
ease, which point to oesophageal varices as the cause of the bleeding, but do not, it
must be emphasized, exclude peptic ulcer disease. Most elderly patients stand an op-
eration better than a recurrence of severe bleeding. Continued oozing, although not
dramatic, can be an unrecognized danger. Today, gastric surgeons have the challenge
of difficult decisions and difficult operations in the treatment of life-threatening
emergencies affecting an increasingly elderly population. Surgeons must also be alert
to early gastric malignancy associated with peptic ulcer, or its previous surgical
treatment, as well as to the long-term complications of peptic ulcer surgery, which
was performed frequently up to 10 to 15 years ago.
Diverticular disease
Diverticular disease is a disease of the modern elderly ‘Western' population.
Numerous studies have shown a progressive increase in prevalence throughout the
twentieth century. Diverticular disease occurs in about 5 to 10 per cent of people in
their 40s, increasing to about 70 per cent of people in their 80s. The prevalence is
much greater in the developed world than in less industrialized nations. Studies of
migrant groups, for example Japanese immigrants to Hawaii, have shown a dramatic
increase in diverticular disease with the shift to a ‘Western' environment. Acute diver-
ticulitis affects the sigmoid or descending colon in at least 85 per cent of cases. The
clinical features resemble ‘left-sided appendicitis'. Pain starts in the lower abdomen,
tending to localize in the left iliac fossa. Associated anorexia, nausea, fever, and al-
tered bowel habit are common features. If the inflamed segment of colon is adjacent
to the bladder, urinary symptoms occur. If the inflamed sigmoid is ‘flopped' over to
the right side, the clinical presentation is difficult to distinguish from appendicitis.
On examination, patients are flushed, pyrexic, and tachycardic. Abdominal ex-
amination reveals tenderness and guarding over the affected segment of colon, usual-
ly in the left iliac fossa. If the inflamed loop of sigmoid is situated deep in the pelvis,
tenderness may only be elicited by rectal or vaginal examination. In patients who
have had symptoms for several days, it is not uncommon to feel a tender mass, com-
prising the thickened, inflamed segment of colon and adherent omentum. Elderly and
immunocompromised patients tend to present with more non-specific features, with
few features to localize the condition to the abdomen
The diagnosis of generalized peritonitis is usually clinical. Laboratory investigations
are helpful in assessing the degree of systemic upset and comorbidity. An erect chest
radiograph usually demonstrates free gas, indicative of perforation. In unequivocal
cases more detailed radiological imaging is not required. In equivocal cases, particu-
larly the elderly or immunocompromised individual, CT may be helpful.
Large-bowel obstruction
When operating for acute large-bowel obstruction, the Hartmann's procedure is
often appropriate. A one-stage approach with on-table lavage and primary anastomo-
sis is favoured by some surgeons, particularly in the United Kingdom, although there
are no trials comparing these techniques. In the very frail or unstable patient, a pre-
liminary transverse colostomy (three-stage approach) may be appropriate. If the ile-
ocaecal valve is competent and the caecum critically distended, a subtotal colectomy
with ileosigmoid or ileorectal anastomosis may be undertaken. Although this usually
permits a primary anastomosis with no defunctioning stoma, the increased bowel fre-
quency afterwards may be troublesome, especially in the elderly individual.
Mesenteric ischemia
Obliterative atheromatous disease of the mesenteric arteries usually occurs in
elderly individuals who are medically debilitated with generalized atherosclerosis.
Despite recent progress in perioperative management and better understanding in
pathophysiology, mesenteric ischemia is one of the most lethal vascular disorders
with mortality rates ranging between 50 and 75 per cent. Delay in diagnosis and
treatment are the main contributing factors in its high mortality. It is estimated that
mesenteric ischemia accounts for 1 in every 1000 hospital admissions in the United
States. The prevalence is rising due in part to the increased awareness of this disease,
the advanced age of the population, and the significant comorbidity of these elderly
patients. Early recognition and prompt treatment before the onset of irreversible intes-
tinal ischemia are essential to improve the outcome. Abdominal pain is only present
in approximately 70 per cent of patients with non-occlusive mesenteric ischemia.
When present, the pain is usually severe but may vary in location, character, and in-
tensity. In the absence of abdominal pain, progressive abdominal distention with aci-
dosis may be an early sign of ischemia and impending bowel infarction. The diagno-
sis of non-occlusive mesenteric ischemia should be considered in elderly patients
with sudden abdominal pain who have any of the following risk factors: congestive
heart failure, acute myocardial infarction with cardiogenic shock, hypovolemic or
hemorrhagic shock, sepsis, pancreatitis, and administration of digitalis or vasocon-
strictor agents such as epinephrine.
Ischaemic colitis
Ischaemic colitis is an inflammatory condition produced by interruption of the
blood supply to the colon insufficient to cause full thickness tissue death. It most
commonly affects those in the sixth to the eighth decades of life and is thus being
seen with increasing frequency in our progressively elderly population. Ischaemic co-
litis may be caused by occlusion of a major artery, small vessel disease, venous ob-
struction, ‘low flow' states, or intestinal obstruction. In each case, the mucosa and
sub-mucosa are predominantly affected, the extent of injury being determined by the
severity and longevity of the insult. Ischaemia reduces the integrity of the mucosa and
allows invasion by pathogenic organisms such as clostridia, which are normal con-
stituents of colonic flora. These processes produce inflammation and mucosal ulcera-
tion which may resolve completely. Alternatively the insult can result in permanent
injury with healing by fibrosis and subsequent stricture formation. Rarely, necrotizing
colitis develops, which can spread to affect areas of the colon which are not ischaem-
ic. Although any part of the colon can be affected, the splenic flexure is particularly
susceptible to ischaemic injury because it is the site of the watershed between the su-
perior mesenteric artery, supplying the transverse colon, and the inferior mesenteric
artery which supplies the descending colon. These vessels are linked by a marginal
artery, but this is frequently absent or poorly developed at the splenic flexure. Occlu-
sion of either major artery or their feeding branches (middle colic artery from the su-
perior mesenteric artery and left colic artery from the inferior mesenteric artery) can
therefore result in ischaemia. This point is of particular relevance to aortic and colo-
rectal surgery if the inferior mesenteric artery is ligated. During aortic surgery it is
important to confirm pulsatile flow in the superior mesenteric and marginal arteries
prior to ligating a patent inferior mesenteric artery. If this is absent or if doubt exists
then the inferior mesenteric artery should be reimplanted in graft.
Clinical features
A typical patient is 50 years of age or more and complains of left-sided ab-
dominal pain which is acute in onset and started in the left iliac fossa. Loose stools,
which characteristically contain dark blood as well as clots, may be passed. There
may be a history of previous similar episodes, or of peripheral or cardiovascular dis-
ease collagen vascular disease, especially if the symptoms are atypical.
Examination
As ischaemic colitis is predominantly a disorder of colonic mucosa and submu-
cosa it is not usually associated with a major systemic upset, but a low grade pyrexia,
tachycardia should be expected. On abdominal examination, the affected colon is ten-
der and may be palpable. Dark blood will be present per rectum. Signs of peripheral
vascular disease or other associated conditions should be sought.
Investigation
It is important to first establish the diagnosis and then determine the presence
of any treatable aetiological factors.
Radiological investigations
A plain abdominal radiograph and a contrast enema are the most useful investi-
gations in the initial stages of this disorder. ‘Thumb-printing' is diagnostic and is
more often seen at the splenic flexure. It is present at an early stage (from 3 days) and
is the result of submucosal oedema and haemorrhage which produce swellings that
project into the bowel lumen. These are clearly seen in contrast studies. Later, muco-
sal ulceration and irregularity may develop and these can resemble the appearances of
ulcerative colitis or Crohn's disease. However, ulcerative colitis invariably affects the
rectum and there is loss of the normal colonic haustral pattern while in Crohn's dis-
ease deep ulcers resemble ‘rose thorns' and areas of affected colon are separated by
normal bowel. These features are not seen in ischaemic colitis. Although many of the
features of ischaemic colitis are reversible, stricture formation, if it occurs, is not and
causes further diagnostic problems. Ischaemic strictures a often long, uniform and
have smooth, gradual beginnings and ends, an appearance called ‘funnelling'. How-
ever, these findings do not exclude carcinoma; this diagnosis should be considered,
particularly if only a short segment of colon is affected. The role of angiography is
not established. Although it can be valuable in isolated cases where significant, symp-
tomatic occlusive lesions are revealed, there is generally no correlation between the
appearance of vessels at angiography and the integrity of the colonic blood supply.
Endoscopy
Ischaemic lesions are usually beyond the reach of the rigid sigmoidoscope, but
colonoscopy can be used to visualize and biopsy affected colon. In the early stages of
ischaemia, the mucosa will be heaped up, oedematous, and bluish purple (the ‘thumb-
prints' seen radiologically). It will bleed on contact with the endoscope or other in-
struments. Later, ulceration as well as strictures may be seen.
Differential diagnosis
It should be noted that some of these diagnoses, for example carcinoma, are al-
so possible aetiological factors for ischaemic colitis.
Treatment
This will be determined by the mode of presentation, which in turn reflects the
underlying stage of the ischaemic process. Conservative management is the mainstay
treatment for those seen with acute symptoms. The patient is rested in bed and given
intravenous fluids. Broad-spectrum antibiotics are often administered, although there
is no conclusive evidence to suggest that they influence outcome. There is no place
for anticoagulation or steroid administration unless this is indicated by an underlying
disorder such as vasculitis. It is very rare for ischaemic colitis to progress to frank co-
lonic gangrene, but all patients should be monitored frequently to assess progress. If
the injury is transient then resolution occurs after a few days to a week. More severe
insults lead to stricture formation. These require investigation and treatment if they
produce symptoms or if there is diagnostic doubt. Excision followed by end-to-end
anastomosis is safe, although it is essential to ensure the viability and vascularity of
the resection margins. If malignancy is excluded, then the resection can be limited to
the affected segment but a more radical excision should be performed if there is con-
tinuing diagnostic uncertainty.
Antibiotic colitis
This is caused by toxins produced by C. difficile. The organism is a Gram-
positive, anaerobic bacillus that can be found in otherwise healthy individuals but un-
der certain circumstances is able to multiply, produce toxin (toxins A and B), and
cause a pseudomembranous colitis. This form of colitis may occur spontaneously
without antibiotic exposure, in the elderly, associated with colonic obstruction, and in
immunosuppressed patients. However, the majority of cases occur 1 to 3 weeks fol-
lowing antibiotic usage. Virtually all antibiotics have been implicated but the com-
monest have been clindamycin, lincomycin, ampicillin, amoxicillin, and the cephalo-
sporins.
Intestinal obstruction involves the colon 20 to 40 per cent of the time. Colonic
obstruction is associated with potentially serious complications such as perforation,
and the timing and selection of appropriate operative procedures are important.
Symptoms can develop slowly and progressively or fulminantly. Among adults, el-
derly people are usually affected. The sigmoid colon is the usual site: this portion of
the intestine is thick walled, not particularly distensible, and comparatively narrow.
Angiodysplasia
Angiodysplasia is an acquired submucosal arteriovenous malformation which
may cause lower gastrointestinal bleeding in elderly patients. It is often difficult to
diagnose, and many aspects of the disease are unclear; reported statistics about inci-
dence and results of treatment differ widely. Angiodysplasia has been reported to ac-
count for between 2 and 60 per cent of adult patients with lower gastrointestinal
bleeding; it probably causes the majority of incidents of major lower gastrointestinal
bleeding in older patients, with diverticulosis accounting for most of the remainder.
There is also little agreement about the prevalence of angiodysplasia in adults; esti-
mates range from less than 1 to 30 per cent, but the true figure is probably 1 to 5 per
cent. The treatment of angiodysplasia depends on the amount of bleeding and the ex-
tent and site of the lesions. Asymptomatic cases do not need any treatment. At the
opposite end of the spectrum, massive haemorrhage requires resuscitation and thor-
ough investigation, including angiography. If an angiodysplastic lesion is identified
as the cause of massive bleeding, appropriate surgical resection is indicated. When
bleeding from angiodysplasia is slow or intermittent, selection of optimum treatment
is more difficult. A variety of options are available, although there are no comparative
trials. Surgical resection of the affected segment of bowel, usually the right colon, is
usually successful in stopping bleeding. Rebleeding occurs in 15 to 25 per cent of
surgically treated patients, usually because of overlooked arteriovenous malfor-
mations in other areas of the bowel, or an error in diagnosis. If preoperative evalua-
tion has reasonably convincingly demonstrated right-sided angiodysplasia, and left-
sided diverticulosis is found at laparotomy, it is still acceptable to limit the resection
to the right colon. If no specific treatment is undertaken, at least half of the patients
with angiodysplasia will rebleed. The major disadvantage of resection is the associat-
ed morbidity and mortality risk, particularly in an elderly population with coexisting
medical diseases. Endoscopic coagulation therapy, either laser photocoagulation (Fig.
4) or electrocoagulation, is an attractive alternative unless the lesions are large or nu-
merous. If a laser is to be used, argon is preferable to neodymium–yttrium aluminium
garnet (Nd:YAG). The light wavelength from an argon unit is preferentially absorbed
by red pigment and also penetrates more superficially than does light from an
Nd:YAG machine. Both characteristics are nicely suited to treatment of superficial
blood vessel malformations. Whether using laser photocoagulation or hot biopsy elec-
trocoagulation, it is best to start at the periphery of the lesion and progress toward the
centre. The mucosa should be cauterized until it is white, not black. Coagulation of
large angiodysplasias should be carried out in stages, several weeks apart, to mini-
mize the risk of perforation. The colon and terminal ileum should be searched care-
fully for synchronous lesions. Rebleeding occurs in only 10 to 30 per cent of patients,
and this is usually from lesions overlooked or incompletely coagulated, making early
follow-up colonoscopy prudent. Nevertheless, even in patients who rebleed after en-
doscopic treatment of arteriovenous malformation of the gastrointestinal tract, the
frequency of haemorrhage episodes and number of transfusions declines. Perforation
follows in up to 7 per cent of treatments, usually when an Nd:YAG laser is used.
Rectal prolapse
Rectal prolapse occurs at the extremes of life. Complete rectal prolapse is
found chiefly in elderly female patients: 85 per cent of adults with full-thickness rec-
tal prolapse are women and the incidence is maximal in the fifth decade and upwards.
Many patients are of very advanced age, being in their eighties or nineties. In men,
though the incidence is much lower, rectal prolapse presents throughout the age range
or may be more common in the second and third decades of life. Mucosal prolapse is
most common in young children.
Delorme's operation is becoming increasingly popular. With the bowel fully
prolapsed a circular incision is made through the mucosa of the prolapse 1 cm from
the dentate line. Infiltration with dilute adrenaline solution helps indicate the submu-
cosal plane. The mucosa is then dissected from the underlying muscle coat as a sleeve
until the apex of the prolapse is reached, and the dissection is carried on down into
the prolapse as far as possible. This leaves the outer aspect of the prolapse without
any mucosal covering. The underlying muscle coat is imbricated with a series of lon-
gitudinal sutures to bunch up or reef it and bring the edges of the mucosa together.
The mucosa is then sutured with absorbable sutures and the prolapse gently reduced.
The procedure has been used successfully in elderly patients. It has a particular place
in this group and can also be used in patients with rectal prolapse complicating chron-
ic ambulatory peritoneal dialysis.
The Thiersh operation Incisions are made in front of and behind the anal mar-
gin to allow the passage of wire, stout nylon, or even Silastic around the anal sphinc-
ter. It is usually overlapped and sewn together anteriorly to provide the right amount
of tension. The procedure can be carried out under regional or local anaesthesia in el-
derly frail patients. In principle, the technique works by supporting the reduced pro-
lapse and causing a local reaction which induces fibrosis and stenosis of the anal ca-
nal.
Acute cholecystitis
In elderly patients, acute cholecystitis may present more insidiously and the
frequent absence of typical physical signs results in a delay in diagnosis. In addition,
the incidence of complications is higher and the prevalence of intercurrent illness
combines to increase the mortality 10-fold. Acute cholecystitis is uncommon in chil-
dren, when it is usually associated with gallstones that develop as a complication of
haemolytic disease. Acalculous cholecystitis occurs in children, with severe sepsis.
Acute emphysematous cholecystitis is a severe and fulminant form of acute
cholecystitis that accounts for less than 1 per cent of cases. Stones are absent in 30 to
50 per cent. The patient is usually elderly and male, and 40 per cent have diabetes
mellitus. It is caused by a mixture of bacteria that includes gas-forming organisms,
and the pathognomonic diagnostic sign is gas within the wall or the lumen of the
gallbladder seen on a plain radiograph. The onset of the disease is abrupt and the
condition of the patient deteriorates rapidly. There is a high incidence of gangrene
and perforation, and emergency cholecystectomy is needed.
Acute cholecystitis can develop in the absence of stones in the gallbladder. It is
most often seen in patients in the intensive care unit and is associated with severe ill-
ness such as multiple trauma, extensive burns, major surgery and sepsis, often in an
elderly person. The cause is unknown but is thought to be related to gallbladder dis-
tension and bile stasis. The normal contraction of the gallbladder is inhibited in pa-
tients with sepsis and those on total parenteral nutrition, especially if opiate analge-
sics are given. This allows the development of biliary sludge, which may be demon-
strated in the gallbladder of many patients with major illness.
The mortality of open cholecystectomy is 1 per cent and the morbidity about 5
per cent. Pulmonary complications are the most common; wound infection, deep-vein
thrombosis, and cardiovascular problems account for the rest. The overall figure for
mortality conceals a considerable variation with age. Cardiac and respiratory diseases
are more frequent in elderly people, in whom it is more common to find complica-
tions from the stones themselves. The mortality rate in patients over the age of 70
years may reach 10 per cent. Some of these patients do, in fact, have minor versions
of recognized clinical conditions such as mesenteric adenitis, threadworm infestation,
gynaecological pain from ovulation, or torsion of a colonic appendix epiploicae.
Incomplete intestinal obstruction may not be clinically obvious and it may re-
solve before the diagnosis is made if the loop of bowel releases itself or the adhesion
tears. Obscure abdominal pain in the elderly person, which is uncommon, is often as-
sociated with cancer, particularly cancer of the colon. Social and psychological fac-
tors play a very important part in some patients. This is usually because of anxiety
about the minor abdominal pains that afflict everyone at some time or another rather
than being a primary cause in themselves.
A steadily increasing number of invasive cardiovascular procedures are per-
formed on the elderly patient. At the Emory Heart Center, the proportion of patients
over 75 years of age has increased from 6 per cent in 1985–87 to 14 per cent in 1994–
96. The explanation for this rapid emergence of coronary angioplasty as a therapeutic
option in this segment of our population is multifactorial. First, there is the dispropor-
tionate growth in the elderly population. Secondly, this population has the highest
prevalence of coronary atherosclerotic heart disease. Thirdly, coupled with this high
prevalence of disease is a steadily increasing life expectancy. Finally, and most im-
portantly, has been the reduction in morbidity and mortality with angioplasty. The
most recent studies on angioplasty in elderly people have demonstrated initial success
rates similar to those reported for younger age groups. The Emory data confirm this
equivalency in angiographic and clinical success rates until age 80 years, when the
success rates trend downward. In the Emory data, the major associated risks of cardi-
ac death and Q-wave myocardial infarction did not increase significantly until after
the age of 80 years. While most studies also noted this significant rise in complica-
tions occurring after the age of 75 or 80 years, the Washington Hospital Center results
noted that the increase began after the age of 65. Their results also noted an increase
in complications of vascular access beginning at the same age. Since the reclosure
rate after angioplasty in elderly people is similar to that in younger age groups, the
older patient must not tolerate acute occlusion after angioplasty as well as his or her
younger cohorts. Analysis of the Emory data suggests a confounding factor, namely,
the elderly patient is more likely to have his or her reclosure treated by medical
means, perhaps accounting for the increase in the postangioplasty myocardial infarc-
tion rate and possibly even the death rate. In other words, complications are more
likely to be accepted in the very elderly patient than to proceed to surgery and its in-
herent risks. Long-term results with coronary angioplasty show a higher cardiac death
rate and more frequent recurrence of angina in the patient over the age of 75. As
would be expected, the overall death rate is also higher. If complete revascularization
is achieved in the elderly patient, then the differences in the cardiac death rate and re-
currence of angina between the elderly and younger age groups disappear. Analysis
of the Emory data reveals that while coronary disease is more extensive in the elderly
patients the frequency of multisite angioplasty is the same as in younger patients, im-
plying that myocardial revascularization is more often incomplete in the elderly pa-
tient.
Basic literatures:
1. Townsend: Sabiston Textbook of Surgery, 18th ed.
2. Essentials of Surgery: Scientific Principles and Practice 2nd edition /
Greenfield L. J., Mulholland M. W., Oldham K. T., Zelenock G. B., Lillimoe K. D.,
Oldham K. – 1997. - Lippincott Williams & Wilkins Publishers.
3. Oxford Textbook of Surgery (3-Volume Set) 2nd edition / Morris P. J.,
Wood W. C. – 2000. - Oxford Press
4. Schwartz’s manual of surgery (8th edition) / Brunicardi F. - 2006 C.
McGRAW-HILL Medical Publishing Division New York, Chicago, San Francisco,
Lisbon, London, Madrid, Mexico City, Milan, New Delhi, San Juan, Seoul, Singa-
pore, Sydney, Toronto.
Additional literatures:
1. MAHMOODIAN S: Appendicitis complicating pregnancy. South Med. J.
85:19–24, 1992.

Tests for initial level of knowledge, keys for tests:

13. Which of the following statements regarding appendicitis during pregnancy is


correct?
K. Appendicitis is the most prevalent extrauterine indication for celiotomy during
pregnancy
L. Appendicitis occurs more commonly in pregnant women than in nonpregnant
women of comparable age
M. Suspected appendicitis in a pregnant woman should be managed with a period
of observation of due to the risks of laparotomy to the fetus
N. Noncomplicated appendicitis results in a 20% fetal mortality and premature labor
rate
O. The severity of appendicitis correlates with increased gestational age of the fetus

14. Diagnostic abdominal laparoscopy is contraindicated in which of the following


patients?
A. A patient with rebound tenderness following a tangential gunshot wound to the
abdomen
B. A stable patient with a stab wound to the lower chest wall
C. A patient with a mass in the head of the pancreas
D. A young female with pelvic pain and fever
E. An elderly patient in the intensive care unit suspected of having intestinal ische-
mia

15. A spry octogenarian who has never before been hospitalized is admitted with
signs and symptoms typical of a small bowel obstruction. Which of the following
clinical findings would give the most help in ascertaining the diagnosis?
A. Coffee-grounds aspirate from the stomach
B. Aerobilia
C. A leukocyte count of 40,000/μL
D. A pH of 7.5, PCO2 of 50 kPa, and paradoxically acid urine
E. A palpable mass in the pelvis

16. An 80-year-old man is admitted to the hospital complaining of nausea, abdominal


pain, distention, and diarrhea. A cautiously performed transanal contrast study re-
veals an “apple core” configuration in the rectosigmoid. Appropriate management
at this time would include
A. Colonoscopic decompression and rectal tube placement
B. Saline enemas and digital disimpaction of fecal matter from the rectum
C. Colon resection and proximal colostomy
D. Oral administration of metronidazole and checking a Clostridium difficile titer
E. Evaluation of an electrocardiogram and obtaining an angiogram to evaluate for
colonic mesenteric ischemia

17. A 65-year-old man who is hospitalized with pancreatic carcinoma develops ab-
dominal distention and obstipation. The abdominal radiograph revealed distended
loops of intestine. Appropriate management would best be achieved by:
A. Urgent colostomy or cecostomy
B. Discontinuation of anticholinergic medications and narcotics and correction of
metabolic disorders
C. Digital disimpaction of a fecal mass in the rectum
D. Diagnostic and therapeutic colonoscopy
E. Detorsion of the volvulus and colopexy or resection

18. Which statement concerning cholangitis is correct?


A. The most common infecting organism is Staphylococcus aureus
B. The diagnosis is suggested by the Charcot triad in elderly
C. The disease occurs primarily in young, immunocompromised patients
D. Cholecystostomy is the procedure of choice in affected patients
E. Surgery is indicated once the diagnosis of cholangitis is made

19. A 72-year-old patient with an intractable type I ulcer along the incisura with a
significant amount of scarring along the entire length of the lesser curvature. Se-
lect the appropriate surgical procedure for this patient.
A. Vagotomy and antrectomy
B. Antrectomy alone
C. Vagotomy and pyloroplasty
D. Vagotomy and gastrojejunostomy
E. Proximal gastric vagotomy

20. An 82-year-old nursing home patient presents to the emergency room with vomit-
ing, abdominal pain, and distention. A radiograph is obtained and demonstrates a
grossly dilated loop of intestine overlying the sacrum in the shape of an upside
down U. Select the likely diagnosis.
A. Perforated diverticulum
B. Perforated gastric ulcer
C. Ruptured spleen
D. Ruptured echinococcal liver cyst
E. Sigmoid volvulus

21.A 65-year-old previously healthy man presents with severe abdominal pain that
came on suddenly. He has abdominal tenderness and guarding in all four quad-
rants on physical examination. A radiograph is obtained and demonstrates a radio-
lucency under the right hemidiaphragm. What diagnosis is unlikely?
A. Perforated diverticulum
B. Perforated gastric ulcer
C. Ruptured echinococcal liver cyst
D. Perforated transverse colon carcinoma
E. Strangulated hernia with necrotic bowel

22.Causes of pain in the right lower quadrant include following except:


A. Appendicitis
B. Ruptured Meckel’s diverticulum
C. Ovarian cyst rupture
D. Ruptured ectopic pregnancy
E. Sigmoid volvulus

Keys for tests


1 2 3 4 5 6 7 8 9 10
A A B C D B B E C E

Tests for final level of knowledge, keys for tests:


1. A 30-year-old primigravida complains of headaches, restlessness, sweating, and
tachycardia. She is 18 wk pregnant and her blood pressure is 200/120 mm Hg.
Appropriate workup might include:
A. Exploratory laparotomy
B. Mesenteric angiography
C. Head CT scan
D. Abdominal CT scan
E. Abdominal ultrasonogram

2. Gynecologic and obstetric causes of left lower quadrant abdominal pain include
following except:
A. Ruptured ectopic pregnancy
B. Ovarian cyst rupture
C. Pelvic inflammatory disease
D. Tubo-ovarian abscess
E. Cholangitis

3. Physiologic alterations of laboratory values during pregnancy include the following


except:
A. Mild leukocytosis,
B. Physiologic anemia,
C. Mild hypernatremia
D. Mild dilutional hypoalbuminemia
E. Mildly increased alkaline phosphatase level

4. Following factor contribute to a mild hypercoagulopathy during pregnancy


A. Anemia
B. Hypernatremia
C. Leukocytosis
D. Estrogen
E. Progesterone

5. What diagnostic imaging is safe for the fetus?


A. Ultrasonography
B. Computerized tomography
C. Magnetic resonance imaging
D. Plane X-ray
E. Intravenous pyelography

6. Following disorders may cause diffuse abdominal pain may except:


A. Inflammatory bowel disease
B. Porphyria
C. Peritonitis
D. Hepatic abscess
E. Rib fracture

7. Choose the incorrect statement


A. A rigid abdomen with rebound tenderness remains a valid indicator of peritonitis
during pregnancy
B. The erythrocyte sedimentation rate is physiologically decreased
C. Abdominal wall laxity in late pregnancy might mask the classic signs of peritonitis
D. An abdominal mass may be missed on physical examination because of the pres-
ence of the enlarged gravid uterus
E. The location of maximal abdominal pain and tenderness from acute appendicitis
migrates superiorly and laterally as the appendix is displaced by the growing gravid
uterus

8. These are risk factors for ectopic pregnancy except:


A. Advanced maternal age
B. prior complicated abdominal surgery, such as ruptured appendicitis
C. Adnexal, or uterine pathology
D. Tubal surgery
E. Better PID therapy,

9. Obturator hernias typically are found in


A. Thin elderly women
B. Fat men
C. Pregnant
D. Young boys
E. Adolescents

10. Contraindications for Diagnostic Peritoneal Lavage


A. Equivocal pulmonary embolism
B. Unexplained shock or hypotension
C. Pregnancy
D. Altered sensorium (e.g., closed-head injury, drugs)
E. General anesthesia for extra-abdominal procedures

Keys for tests

1 2 3 4 5 6 7 8 9 10
E E C D A E B E A C

Tasks for the final level of knowledge.

1. An elderly diabetic woman with chronic steroid-dependent bronchospasm has an


ileocolectomy for a perforated cecum. She is taken to the ICU intubated and is main-
tained on broad-spectrum antibiotics, renal-dose dopamine, and a rapid steroid taper.
On postoperative day 2 she develops a fever of 39.2°C (102.5°F), hypotension, leth-
argy, and laboratory values remarkable for hypoglycemia and hyperkalemia. The
most likely diagnosis of this acute event is
The answer is adrenal insufficiency
2. A 25-year-old primigravida complains of headaches, restlessness, sweating, and
tachycardia. She is 19 wk pregnant and her blood pressure is 210/130 mm Hg. What
should include the appropriate treatment?
The answer is – this young pregnant woman presents with the symptoms of a
pheochromocytoma. Phenoxybenzamine and propranolol followed by a combined
vaginal delivery at term and excision of the tumor.

3. A 28-year-old who is 15 week pregnant has new onset of nausea, vomiting and
right-sided abdominal pain. She has been free of nausea since in her first trimester.
The pain has become worse over the past 6 h. What disease can be first of all suspect-
ed at this patient?

The answer is acute appendicitis as the most prevalent indication for celiotomy
during pregnancy.

4. The 25-year-old 37 week pregnant felt herself bad in supine position. Her AP de-
creased to 100/60 mm Hg, CVP – to 10 mm H20. Simply turning the patient to the left
side relieved these signs. What is underlying mechanism of the above mentioned dis-
orders?

The answer is the gravid uterus can compress the inferior vena cava in the supine
position and thereby compromise venous return.

23. A 32-old-year pregnant woman complained pain in right lower quadrant of the
abdomen for 3 days. The latter is severe and exacerbated by movements that require
muscular contraction of the abdominal wall. Physical examination demonstrates
tenderness over the rectus sheath, often with voluntary guarding. A diffuse mass, is
noted in the location of pain. The pain and tenderness associated with this process
permitted to suggest peritonitis. Ecchymosis is present in right lower quadrant. What
diseases should be differentiated and how?

The answer is appendicitis with rectus Sheath Hematoma. Ultrasonography or


CT will confirm the presence of the hematoma
Materials for self-education
Main tasks Notes(instructions)
Repeat:
10.Anatomy of pregnant uterus 1. Size of the uterus at different terms
of pregnancy
11.Physiological changes during pregnancy
12.Physiological changes in organisms of
elderly patients
Study:
9. The clinical signs of confusional states 1. Make the algorithm of the as-
10. The complications of pregnancy sessing of mental state of elderly pa-
tients
11. Hormonal changes in women’s or- 2. Make the table with clinical signs
ganisms during pregnancy of complications of pregnancy
Study guide #14

“Chest pain syndromes, respiratory and cardiac insufficiency in diagnosis and


differential diagnosis of chest diseases. Modern methods of diagnosis and treat-
ment of diseases of cardiovascular and respiratory systems.”

Few symptoms are more alarming than chest pain. In the minds of many peo-
ple, chest pain equals heart pain. While many other conditions can cause chest pain,
cardiac diseases are so common and so dangerous that the symptom of chest pain
should never be dismissed as being insignificant.
“Chest pain” is an inaccurate term. It is often used to describe any pain, pres-
sure, squeezing, choking, numbness or any other discomfort in the chest, neck, or up-
per abdomen, and is often associated with pain in the jaw, head, or arms. It can last
from less than a second to days or weeks, can occur frequently or rarely, and can oc-
cur intermittently or predictably. With such a broad definition, you can see why the
term “chest pain” is of little help to doctors.
It is important to keep in mind that chest pain is merely a symptom, not a diag-
nosis. And because it can be a symptom of anything from a catastrophe to a trivial
medical problem, when a person experiences chest pain it is important to try to char-
acterize that pain as quickly as possible as being either completely benign, or possibly
significant.
Chest pain is the presenting symptom in about 12% of emergency department
visits and has a one year mortality of about 5%.
Educational aims:
25. Interrogation and clinical inspection of patients with chest pain.
26. To determine the etiologic and pathogenic factors of chest pain.
27. To find out the types of chest pain, the clinical features, different variants of
manifestation and complications.
28. To develop a plan of examination of the patients with chest pain.
29. To estimate laboratory data end instrumental examination of the thorax.
30. To draw a differential diagnosis, substantiate and formulate a diagnosis for the
patients with chest pain.
31. To prescribe the treatment for patients with chest pain.
32. To determine the indications for operative treatment of patients with chest pain.
33. To cure the patients with chest pain.
34. To estimate efficiency of treatment and prognosis of disease.
A student must know:
24.Аnatomo-physiological information about thorax.
25.Classifications of chest pain.
26.Mechanisms of chest pain.
27.Clinical picture of chest pain.
28.Methods of diagnosis of chest pain.
29.Principles of treatment of chest pain.
30.Features of surgical interventions for the different types of chest pain.
A student must be able to:
20.Collect and estimate the complaints of patient with chest pain, information of
anamnesis, to conduct physical research and correctly interpret the results ob-
tained.
21.Define the rational volume of laboratory and instrumental methods of research.
22.Correctly interpret the results of clinical analyses, instrumental examinations
23.Define indications for operation and other methods of treatment of patients.
24.Perform pre-operative preparation of patients.
25.Conduct post-operative care.
Terminology.
Term Definition

Respiratory Insuffi-A condition where the lungs are unable to function


properly and maintain the normal processes of oxygen up-
ciency:
take and carbon dioxide elimination.
Chest pain follows a specific pattern, occurring when
someone engages in hard physical activity or experiences
Stable angina extreme emotion. Other situations that bring on angina
include smoking a cigarette or cigar, cold weather, a large
meal and straining in the bathroom. The pain usually goes
away when the pattern or trigger ends.
This chest pain occurs at rest, during sleep or very often
Unstable angina with minimal exertion. The discomfort may last and be
intense.
Pneumothorax (pleu- Is a collection of air or gas in the pleural cavity of the
ral pneumothorax) chest between the lung and the chest wall.
Hemothorax (or Is a condition that results from blood accumulating in the
haemothorax) pleural cavity.
(From Latin dyspnoea, from Greek dyspnoia from dysp-
Dyspnea or dyspnoea noos, shortness of breath) is a debilitating symptom that is
the experience of unpleasant or uncomfortable respiratory
sensations.

Content:
Chest pain may be a symptom of a number of serious conditions and is general-
ly considered a medical emergency. Even though it may be determined that the pain
is non-cardiac in origin, this is often a diagnosis of exclusion made after ruling out
more serious causes of the pain.
Differential diagnosis
The causes of chest pain range from non-serious to life threatening.
Cardiovascular
• Acute coronary syndrome
• Unstable Angina Pectoris - requiring emergency medical treatment but
not primary intervention as in a myocardial infarction
• Myocardial infarction ("heart attack")
• Aortic dissection
• Pericarditis and cardiac tamponade
Arrhythmia - atrial fibrillation and a number of other arrhythmias can cause
chest pain.
Stable angina pectoris - this can be treated medically and although it warrants
investigation, it is not an emergency in its strictest sense

Pulmonary
• Pulmonary embolism
• Pneumonia
• Hemothorax
• Pneumothorax and Tension pneumothorax
• Pleurisy - an inflammation which can cause painful respiration
GI
• Gastroesophageal reflux disease (GERD) and other causes of heartburn
• Hiatus hernia (which may not accompany GERD)
• Achalasia, nutcracker esophagus and other neuromuscular disorders of
the esophagus
• Functional dyspepsia
Chest wall
• Costochondritis or Tietze's syndrome - a benign and harmless form of
osteochondritis often mistaken for heart disease
• Spinal nerve problems
• Fibromyalgia
• Chest wall problems
• Radiculopathy
• Precordial catch syndrome
Chest conditions
• Herpes zoster commonly known as shingles
• Tuberculosis
Psychological
• Panic attack
• Anxiety
• Clinical depression
• Somatization disorder
• Hypochondria

Others
• Hyperventilation syndrome often presents with chest pain and a tingling
sensation of the fingertips and around the mouth
• Da Costa's syndrome
• Bornholm disease - a viral disease that can mimic many other conditions
• Precordial catch syndrome - another benign and harmless form of a
sharp, localized chest pain often mistaken for heart disease
• Carbon monoxide poisoning
• Sarcoidosis
• Lead poisoning
• High abdominal pain may also mimic chest pain
Diagnostic approach
In the emergency department the typical approach to chest pain involves ruling
out the most dangerous causes: heart attack, pulmonary embolism, thoracic aortic dis-
section, esophageal rupture, tension pneumothorax and cardiac tamponade. By elimi-
nation or confirmation the most serious causes, a diagnosis of the origin of the pain
maybe made. Often, no definite cause will be found and reassurance is then provided.
As in all medicine, a careful medical history and physical examination is essen-
tial in separating dangerous from trivial causes of disease. The management of chest
pain may be done in specialized units (termed medical assessment units) to concen-
trate on the investigations. A rapid diagnosis can be life-saving and often has to be
made without the help of X-rays or blood tests (e.g. aortic dissection). A focus on re-
cent health changes, family history (premature atherosclerosis, cholesterol disorders),
tobacco smoking, diabetes and other risk factors is useful.

Diagnostic Evaluation
The initial tests for patients with chest pain should include an electrocardio-
gram (ECG) and a chest x-ray.
The ECG may demonstrate regional ST segment depression/elevation indicat-
ing myocardial ischemia/ infarction, or may reveal the diffuse ST segment elevation
of pericarditis. A chest x-ray may reveal rib fractures, focal infiltrates of pneumonia,
wedge-shaped peripheral infiltrates of pulmonary emboli, or the radiolucency of a
pneumothorax. It may also suggest aortic dissection (widened mediastinum), or hiatal
hernia (stomach in the thoracic cavity).
If an acute coronary syndrome is suspected, medical therapy should be imme-
diately started and serial ECGs and cardiac enzymes (creatine kinase and troponin)
checked to confirm or exclude a myocardial infarction. For patients in whom the di-
agnosis remains uncertain but coronary artery disease (CAD) is suspected, a stress
test can be performed for clarification. Chest pain associated with ST segment de-
pression during a stress test is diagnostic of angina. Cardiac catheterization remains
the gold standard for the diagnosis of coronary artery disease and may be necessary to
rule out significant CAD in a subset of patients for whom other tests are unable to
confirm or exclude the diagnosis.
In patients with pulmonary emboli, arterial blood gases usually reveal hypoxia
and/or widened A-a gradient, and ventilation/perfusion (V/Q) scanning or spiral CT
scanning may confirm the diagnosis. Patients suspected of having an aortic dissection
should undergo an urgent transesophageal echocardiography, CT scanning with intra-
venous contrast, or magnetic resonance imaging (MRI). Patients suspected of having
a gastroesophageal cause of their chest pain may need a barium swallow (esophageal
reflux or rupture), endoscopy (esophagitis, gastritis, peptic ulcer disease), hepatobili-
ary hydroxyiminodiacetic acid (HIDA) scan or abdominal ultrasound (gall bladder
disease), esophageal manometry (esophageal spasm), or continuous esophageal pH
measurement (reflux) to confirm the diagnosis.
Depending on test results, patients may be referred to other specialized areas
for additional testing and therapy. Common referral areas include:
• Endocrinology
• Gastroenterology
• Hypertension
• Lipid management
• Nicotine dependence
• Patient and health education
• Pulmonary medicine
• Sleep disorders
• Vascular medicine
• Weight/nutrition counseling
Visits to other areas are coordinated by the patient's primary physician. Results
from these visits are returned to the primary physician to help develop a treatment
plan.
Angina Due to Coronary Artery Disease
Angina is caused by ischemia (oxygen starvation) of the cardiac muscle.
Coronary artery disease produces ischemia by narrowing the coronary arteries.
Subsequently, when the heart tries to perform at a high level (such as during
exercise), the narrowed artery is incapable of delivering the necessary blood volume
to the working muscle. Ischemia ensues, and the resultant pain is called angina.
Characteristics of pain: Angina can be quite variable, but classically is
described as a pressure-like, squeezing, crushing, or tight pain. Some patients with
angina deny pain at all, but agree when the term "discomfort" is described to them.
The discomfort often radiates to the jaw, shoulders or arms. It can be accompanied by
nausea, sweating, shortness of breath, dizziness, weakness, or fatigue. Classically,
angina is often provoked in a predictable manner by exercise, anger, or a large meal.
But it can also occur unpredictably, without any obvious triggering factor, and at rest.
Angina that is increasing in frequency or that occurs at rest is referred to as
"unstable." Unstable angina often precedes a heart attack, and should be treated as a
medical emergency.
Evaluation: If acute coronary syndrome ("unstable angina") is suspected,
many people are admitted briefly for observation, sequential ECGs, and determina-
tion of cardiac enzymes over time are done. On occasion, further tests on follow up
may determine the cause. TIMI score performed at time of admission may help strati-
fy risk.
Heartburn
Heartburn is caused by acidic fluid from the stomach washing up into the
esophagus.
Characteristics of pain: The pain of heartburn is often a burning discomfort
directly beneath the breastbone. It is often accompanied by belching, or symptoms of
bloating or gas. Sometimes an acid taste occurs in the mouth. Symptoms are often
worse after a large meal, or after using tobacco, alcohol, or caffeine. Symptoms tend
to improve with antacids.
Evaluation: The evaluation includes a careful medical history and physical
examination, which often point to the diagnosis. The ECG is generally normal.
Special swallowing tests are sometimes necessary to make the diagnosis.
Benign Chest Wall Pain
Benign chest wall pain is most likely a transient and fleeting irritation of the
pleura, the slippery membrane that lines and protects the lungs. The pleura are very
sensitive to pain, and for reasons that are usually not clear, some momentary irritation
causes a painful sensation that can be quite severe, but that quickly subsides. This
condition has no medical significance whatsoever. It is very common. Most people
will experience these symptoms at some point in their lives.
Characteristics of pain: The pain is usually a sharp "catch" that interrupts a
breath, and that returns with each breath for a few moments - then it subsides. It is not
related to exercise, and generally can be localized to a specific small area (smaller
than the palm of the hand) on the chest wall. It usually lasts for less than a minute, but
can come back on and off for an hour or so. Frequently patients will describe
recurrent symptoms every few weeks or months.
Evaluation: This condition can be evaluated by taking a careful history. The
condition is very common, and very easy to diagnose by history - as long as the
doctor is aware of it and understands it. The important thing to keep in mind is that
this condition is completely benign, and is not related to any medical problem or any
abnormality.
Anxiety or Panic Disorder
Anxiety or panic attacks are characterized by repeated episodes of intense
anxiety and fear that occur without warning and generally without an identifiable
cause. Panic attacks are often accompanied by chest pain, most likely caused by
muscle contractions in the chest wall.
Characteristics of pain: The pain is usually localized to the chest wall, and
can be fleeting and sharp, or can be a sharp "catch" that interrupts a breath. The chest
wall can remain "sore" for hours or days after a panic attack. The severity of the chest
pain is often magnified by the panic disorder itself.
Evaluation: This condition is usually apparent after taking a careful history.
However, if the patient has risk factors for coronary artery disease, a noninvasive
evaluation to rule out cardiac disease is often necessary. Anxiety disorders are often
highly disruptive to an individual's life, and are highly treatable. Unfortunately,
doctors all too often brush off patients presenting with chest pain who are found to
have anxiety disorders. Patients with this diagnosis should seek active medical
treatment from an understanding doctor.
Asthma, Bronchitis, Pneumonia, Pleuritis
Many varieties of lung problems can cause chest pain, including disorders of
the airways such as asthma or bronchitis, infection of the lungs (pneumonia), and
inflammation of the lining of the lungs (pleuritis, or pleurisy).
Characteristics of pain: Chest pain caused by lung problems can be localized
or diffuse, constant or increasing with breathing, and mild or severe. Pleurisy, in
particular, is characterized by chest pain that appears each time a person inhales, and
nearly disappears with exhalation. A severe coughing spell or two can sprain the
chest muscles, and leave them aching for several days.
Evaluation: Chest pain related to lung problems usually becomes quite
apparent after the doctor takes a complete medical history, and performs a physical
examination.
Mitral Valve Prolapse
In mitral valve prolapse (MVP), abnormalities of the mitral valve cause it to
flop backwards into the left atrium as the left ventricle contracts. This flopping, or
prolapse, is occasionally perceived as a form of chest pain.
Characteristics of pain: There is controversy as to how often MVP actually
causes chest pain - it probably does so far less often than is alleged. When MVP does
cause chest pain, the pain is most often described as a "catching" discomfort, and is
often positional in nature (i.e., it may occur, for instance, only when the patient is
lying on the left side.)
Evaluation: The evaluation of MVP consists of listening to the heart through
the stethoscope and doing an echocardiogram.
Pericarditis
Pericarditis is inflammation of the pericardium, the membranous sac that
encloses and protects the heart.
Characteristics of pain: Pericardial pain is usually localized to the area of the
chest, and often increases during breathing.
Evaluation: The diagnosis of pericarditis is usually made by taking a medical
history, doing a physical examination, and doing an ECG.
Recent Chest Trauma
Chest trauma of any type can cause chest pain by causing rib fracture or muscle
strain or contusion. Not infrequently, a seemingly mild chest trauma (e.g.: sliding into
second base during a weekend softball game) can be forgotten when the chest pain
becomes apparent a day or two later.
Characteristics of pain: Chest pain caused by trauma is almost invariably lo-
calized, is often described as soreness or a sharp pain. It usually worsens with breath-
ing, and often the chest wall is tender to touch or to pressure.
Evaluation: Chest pain caused by trauma usually becomes quite apparent after
the doctor takes a complete medical history, and performs a physical examination.
Sometimes a chest X-ray helps in making the diagnosis.
Peptic Ulcer
Peptic ulcer disease usually causes pain in the abdomen, but occasionally the
pain can be perceived in the chest.
Characteristics of pain: The pain of peptic ulcers is often described as a burn-
ing or gnawing sensation. It is often relieved by eating a meal, and often made worse
by drinking alcohol, smoking, or ingesting caffeine. It can be accompanied by symp-
toms of bloating or gas.
Evaluation: The evaluation includes a careful medical history and physical ex-
amination, which often point to the diagnosis. The ECG is generally normal, and ul-
cer disease is only rarely confused with heart disease. Endoscopy is the favored
method of diagnosis.
Coronary Artery Spasm
Angina is caused by ischemia (oxygen starvation) of the cardiac muscle. Coro-
nary artery spasm produces ischemia by narrowing the coronary arteries. The nar-
rowed artery is incapable of delivering the necessary blood volume to the working
muscle. Ischemia ensues, and the resultant pain is called angina.
Characteristics of pain: Angina can be quite variable, but classically is de-
scribed as a pressure-like, squeezing, crushing, or tight pain. Some patients with an-
gina deny pain at all, but agree when the term "discomfort" is described to them. The
discomfort often radiates to the jaw, shoulders or arms. It can be accompanied by
nausea, sweating, shortness of breath, dizziness, weakness, or fatigue. While classical
angina is often provoked in a predictable manner by exercise, anger, or a large meal,
coronary artery spasm can occur at any time, and often occurs at rest.
Evaluation: The evaluation of angina includes a careful medical history and
physical examination, which often point to the diagnosis. Testing always includes an
ECG, and often a stress test or a cardiac catheterization. Coronary artery spasm
should be suspected if the character of the pain and ECG changes are suggestive of
angina, but the coronary arteries are normal on catheterization. If spasm is being con-
sidered, special steps should be taken during the catheterization to attempt to provoke
spasm, thus documenting its presence.
Prinzmetals Syndrome or Gabriel’s Syndrome is chest pain caused by
spasms of coronary arteries. In most patients, there is coronary artery obstruction. In
cases where there is obstruction, spasms may occur near the narrowing of the artery.
With Prinzmetals Syndrome, chest pain may occur at rest. Some patients experience
palpitations. In others chest pain is triggered by exertion. Medications, stents or sur-
gery are treatment options. Once treated, the patient's prognosis is excellent and se-
vere complications such as arrhythmias, heart attack or sudden death are rare.
Coronary X Syndrome
Angina is caused by ischemia (oxygen starvation) of the cardiac muscle. In
Syndrome X, there are signs of ischemia on stress testing, but the coronary arteries
are normal on cardiac catheterization. The cause of Syndrome X is unclear, but most
authorities believe that patients with this syndrome have some sort of vascular ab-
normality involving the small branches of the coronary arteries - branches that are not
visualized on cardiac catheterization. Fortunately, the prognosis of patients with Syn-
drome X appears to be quite good.
Characteristics of pain: Angina can be quite variable, but classically is de-
scribed as a pressure-like, squeezing, crushing, or tight pain. Some patients with an-
gina deny pain at all, but agree when the term "discomfort" is described to them. The
discomfort often radiates to the jaw, shoulders or arms. It can be accompanied by
nausea, sweating, shortness of breath, dizziness, weakness, or fatigue.
Evaluation: The evaluation of angina includes a careful medical history and
physical examination, which often point to the diagnosis. Testing always includes an
ECG, and often a stress test or a cardiac catheterization. In Syndrome X, the charac-
teristics of the pain and the ECG are strongly suggestive of angina, but the coronary
arteries are normal on catheterization. Further, classic coronary artery spasm cannot
be provoked.
Aortic Dissection
Aortic dissection is a tearing of the lining of the aorta - the body's main artery
that comes directly off the heart. It often leads to heart attack or stroke. When aortic
dissection occurs acutely, it is often accompanied by sudden severe pain, sometimes
occurring in the chest.
Characteristics of pain: The pain of aortic dissection is often very sudden and
severe in onset. It is often described as a tearing pain, and is usually located beneath
the breastbone. When aortic dissection is acute, there is usually no question in the pa-
tient's mind that something quite catastrophic has just occurred, and medical help is
sought immediately. With subacute aortic dissection the tearing of the aorta occurs
more slowly and gradually, and the pain tends to be somewhat less severe.
Evaluation: The evaluation is begun by taking medical history and doing a
physical examination (which often reveals a new heart murmur). When dissection is
suspected, the diagnosis is made by echocardiogram, MRI scan, CT scan or cardiac
cath. If the dissection is mistaken as a "'routine" heart attack and thrombolytic drugs
are given, the dissection can become suddenly worse and death can ensue.
Treatment plans for chest pain are individualized depending on the underly-
ing cause, the presence of coexisting diseases, the age and medical history of the pa-
tient, and other factors. Treatment generally involves a multifaceted plan that ad-
dresses the cause, eases the pain, decreases the risk of developing serious complica-
tions, and helps a person to rest comfortably and live an active life.
If chest pain is suspected to be due to a respiratory, cardiac or serious gastroin-
testinal condition or disease, treatment generally involves supplemental oxygen.
There are a variety of devices that are worn on the face to deliver different concentra-
tions of supplemental oxygen. Oxygen is delivered to the lungs through a breathing
tube and mechanical ventilation in cases where chest pain accompanies respiratory
insufficiency, respiratory failure or cardiopulmonary arrest.
Chest pain caused by a heart attack is treated with supplemental oxygen, medi-
cations, such as nitroglycerin and thrombolytic drugs. Treatment also includes medi-
cation to dissolve a clot in the coronary artery that is causing the heart attack or a sur-
gical procedure that opens up the clogged coronary artery (angioplasty).
When chest pain is caused by a viral infection, such as viral pneumonia, the
treatment is rest, increased fluids and using a vaporizer. When chest pain is caused by
a bacterial infection, such as in bacterial pneumonia, antibiotics are prescribed as
well. Serious cases of pneumonia may require hospitalization.
When chest pain is caused by asthma, medications may include corticosteroids
and bronchodilators, which are breathed in via inhalers and nebulizers. Other respira-
tory diseases that cause chest pain may require additional intravenous medications,
hospitalization, intensive care and possibly life support measures. Chest pain caused
by the pain of such conditions as rib fractures may be helped by pain medications.
Chest pain caused by hyperventilation or and anxiety attack can be treated by breath-
ing into a paper bag and possibly anti-anxiety medication. Chest pain caused by
trauma to the chest that results in a collapsed lung condition, such as pneumothorax
or hemopneumothorax is treated by re-inflating the lung and draining the blood by
insertion of a chest tube. Intensive care is also required. Treatment of chest pain
caused by gastrointestinal diseases and conditions varies greatly depending on the
specific cause.
Basic literatures:
55.Oxford Textbook of Surgery (3-Volume Set) 2nd edition (January 15, 2000): by
Peter J. Morris (Editor), William C. Wood (Editor) By Oxford Press
56.Sabiston Textbook of Surgery 17th edition by Courtney M. Townsend Jr., Ken-
neth L. Mattox, B. Mark, MD Evers, Kenneth L., MD Mattox, Courtney Town-
send, Daniel Beauchamp, B. Mark Evers, Kenneth Mattox W.B. Saunders Com-
pany (June, 2004)
57.Schwartz´s Principles of Surgery 8th Edition F. Charles Brunicardi. Copyright
©2007 the McGraw-Hill Companies.
58.Hospital surgery/ Edited by L. Kovalchuk et al. - Ternopil: Ukrmedknyha, 2004.
- 472 p.

Additional literatures:
A. Richard N. Fogoros, M.D., About.com Guide/Updated October 05, 2006
B. Mallinson, T (2010). "Myocardial Infarction". Focus on First Aid (15): 15. Re-
trieved 2010-06-08.
C. Hess EP, Perry JJ, Ladouceur P, Wells GA, Stiell IG (March 2010). "Derivation
of a clinical decision rule for chest radiography in emergency department patients
with chest pain and possible acute coronary syndrome". CJEM 12 (2): 128–34.
D. Braunwald's Heart Disease: A Textbook of Cardiovascular Medicine, 7th ed.,
(October 20, 2004)

Tests for initial level of knowledge, keys for tests:

11. Tension pneumothorax has the following features, EXCEPT:


A. Displacement of the heart on the side opposite pneumothorax.
B. Distension of the neck veins.
C. Arterial hypotension.
D. All of the signs
E. The blunt percussion sound on the side of the lesion.

12. What is the first aid in a patient with tension pneumothorax?


A. Tube thoracostomy
B. Pleural puncture in the 7th intercostal space
C. Pleural puncture in 6th intercostal space
D. Thoracostomy
E. Bronchoscopy

13. A sharp left-sided chest pain appeared in a 35-year-old patient at


the time of intense physical activity. Objectively: the patient is covered with
cold sweat; breathing is difficult because of the pain. Auscultation: on the
right side the breath sound is vesicular, on the left side - weakened. Tachy-
cardia, heart rate -100. What is your diagnosis?
A. Spontaneous pneumothorax
B. Heart attack
C. Myocardial infarction
D. Intercostal Neuralgia
E. Pneumonia

14. 48-years-old woman with ischemic cardiomyopathy have progres-


sive attacks of stenocardia. On coronary angiography: anterior interventricu-
lar coronary artery is stenosed approximately 70% for 0, 7 sm. Other arteries
are without hemodynamic changes. It is an indication for:
A. Balloon angioplasty
B. Coronary artery bypass grafting of one artery
C. Coronary artery bypass grafting of 2-3 arteries
D. Transplantation of heart
E. Medicinal therapy

15. The best test for establishing the diagnosis and the degree of myo-
cardial dysfunction is
A. Serial ECGs
B. X-ray
C. Creatine phosphokinase (CPK-MB) fractionation
D. Echocardiography
E. Radionuclide angiography

16. What statement about heartburn is not correct?


A. Heartburn is caused by acidic fluid from the stomach washing up
into the esophagus.
B. The pain of heartburn is often a burning discomfort directly
beneath the breastbone.
C. It is often accompanied by burping, or symptoms of bloating or
gas.
D. Symptoms are often worse after a large meal, or after using
tobacco, alcohol, or caffeine.
E. Symptoms are often reduce after a large meal

17. What statement about pneumothorax is correct?


A. Chest pain occurs at rest, during sleep or very often with minimal
exertion.
B. It is often accompanied by burping, or symptoms of bloating or
gas.
C. Symptoms are often worse after a large meal, or after using
tobacco, alcohol, or caffeine.
D. Chest pain follows a specific pattern, occurring when someone en-
gages in hard physical activity or experiences extreme emotion.
E. It is a collection of air or gas in the pleural cavity of the chest be-
tween the lung and the chest wall.

18. What statement about unstable angina is correct?


A. Chest pain occurs at rest, during sleep or very often with minimal
exertion.
B. A condition where the lungs are unable to function properly and
maintain the normal processes of oxygen uptake and carbon dioxide remov-
al
C. The pain is often a burning discomfort directly beneath the
breastbone.
D. Chest pain follows a specific pattern, occurring when someone en-
gages in hard physical activity or experiences extreme emotion.
E. This is a condition that results from blood accumulating in the
pleural cavity.

19. What statement about stable angina is correct?


A. Сhest pain occurs at rest, during sleep or very often with minimal
exertion.
B. A condition where the lungs are unable to function properly and
maintain the normal processes of oxygen uptake and carbon dioxide remov-
al
C. Symptoms often reduce after a large meal.
D. Chest pain follows a specific pattern, occurring when someone en-
gages in hard physical activity or experiences extreme emotion.
E. Сondition that results from blood accumulating in the pleural cavi-
ty.

20. A 25-year-old woman arrives in the emergency room following an automobile


accident. She has acute dyspnea with a respiratory rate of 60 breaths/min.
Breathing sounds are markedly diminished on the right side. The first step in
managing the patient should be to:
A. Take a chest x-ray
B. Draw arterial blood for blood gas determination
C. Decompression of the right pleural space
D. Perform pericardiocentesis
E. Administer intravenous fluids

Keys for tests


1 2 3 4 5 6 7 8 9 10
E A A A A E E A D C

Tests for final level of knowledge, keys for tests:


1. The usual management of cardiac arrest should include all of the
following protocols EXCEPT
A. immediate resuscitation, as irreversible brain damage will result
after 3-4 minutes of diminished perfusion
B. establishment of an airway and ventilator support
C. open-chest cardiac massage
D. defibrillation, if cardiac arrest is due to ventricular fibrillation
E. administration of cardiotonic agents

2. What statement about pneumothorax is not true?


A. Is a collection of air or gas in the pleural cavity of the chest be-
tween the lung and the chest wall.
B. It may occur spontaneously in people without chronic lung condi-
tions ("primary") as well as in those with lung disease ("secondary").
C. Most pneumothorax occur after physical trauma to the chest, blast
injury, or as a complication of medical treatment.
D. The symptoms of a pneumothorax are determined by the size of
the air leak and the speed by which it occurs; they may include chest pain in
most cases and shortness of breath in many.
E. This disease is mainly diagnosed by the appearance of inflamma-
tion.
3. A 55-year-old man has a severe vomiting and retching spell punc-
tuated by a sharp substernal pain. The initial management plan should be:
A. Serial ECGs to rule out myocardial ischemia
B. Chest film
C. Left chest tube and spit fistula (cervical esophagostomy)
D. Flexible esophagogastroscopy to establish diagnosis.
E. Nasogastric tube, antibiotics

4. Several days following esophagectomy a patient complains of


dyspnea and chest tightness. A large pleural effusion is noted on chest radio-
graph and thoracocentesis yields milky fluid. What occur with patient?
A. Chylothorax
B. Hydrothorax
C. Pyothorax
D. Pleurisy
E. Failure of sutures

5. What statement about tension pneumothorax is not true?


A. Tension pneumothorax is characterized by rapid breathing, cyano-
sis, falling blood pressure (hypotension) and confusion.
B. The healthy side of the chest is not hyperexpanded but shows de-
creased movement, with increased movement on the affected side.
C. In very severe cases, the respiratory rate falls sharply, with shock
and coma.
D. Patients require hospital admission
E. Particular clinical signs may also be useful in the recognition of
tension pneumothorax, such as the presence of raised jugular venous pres-
sure (distended neck veins).

6. All of the following symptoms and signs are indicative of a ten-


sion pneumothorax EXCEPT
A. chest pain
B. shortness of breath
C. absent breath sounds unilaterally
D. shifting of the trachea towards the pneumothorax
E. hypotension

7. All of the following complications of myocardial infarction are


indications for surgical correction EXCEPT
A. ventricular premature beats
B. ventricular aneurysm
C. ruptured ventricle
D. ruptured intraventricular septum
E. papillary muscle dysfunction

8. A previously healthy 20-year-old man is admitted to a hospital


with acute onset of mild left-sided chest pain. General condition is normal.
The electrocardiographic findings are normal but chest x-ray shows a 20%
left pneumothorax. Patient needs :
A. Observation
B. Barium swallow
C. Thoracotomy
D. Tube thoracostomy
E. Thoracostomy and intubation

9. Several days following esophagectomy a patient complains of


dyspnea and chest tightness. A large pleural effusion is noted on chest radio-
graph and thoracocentesis yields milky fluid - chyle. Initial management of
this patient consists of which of the following procedure?
A. Immediate operation to repair the thoracic duct
B. Immediate operation to ligate the thoracic duct
C. Tube thoracostomy and low-fat diet
D. Observation and low-fat diet
E. Observation and antibiotics

10. Two weeks following the initial management of chylothorax after


esophagectomy there is persistent accumulation of chyle in the pleural space.
Appropriate management at this time includes which of the following proce-
dures?
A. Neck exploration and ligation of the thoracic duct
B. Subdiaphragmatic ligation of the thoracic duct
C. Thoracotomy and repair of the thoracic duct
D. Thoracotomy and ligation of the thoracic duct
E. Thoracotomy and abrasion of the pleural space

Keys for tests


1 2 3 4 5 6 7 8 9 10
C E B A B D E A C B

Tasks for final level of knowledge


1. A 43-year-old patient was diagnosed with mitral insufficiency, III stage of
disease. Calcification of valves was not discovered at the inspection. What method
of treatment is optimum?

The answer is anuloplasty.

2. During endoscopic biopsy of a distal esophageal stricture, perforation of the


esophagus is suspected when the patient complains of significant new substernal
pain. An immediate chest film reveals air in the mediastinum. You would recom-
mend:
The answer is perforation of the esophagus in the chest is a surgical catastro-
phe that requires aggressive intervention in virtually all circumstances. While
that intervention can usually consist of efforts to patch the perforation and drain the
mediastinum, concomitant obstructive esophageal disease, whether inflammatory
stenosis or cancer, mandates removal or bypass of the obstruction if control of the
leak and its consequent persisting mediastinal and pleural contamination is to be
accomplished.

3. A stockbroker in his mid forties consults you with complaints of episodes of


severe, often unbearable chest pain on swallowing. The diagnostic studies on the
esophagus you have ordered yield the following: endoscopic examination and bi-
opsy—mild inflammation distally; manometry—prolonged high-amplitude con-
tractions from the arch of the aorta distally, lower esophageal sphincter (LES) pres-
sure 20 mm Hg with relaxation on swallowing. You would recommend

The answer is the finding of prolonged high-amplitude contractions in the


body of the esophagus in a highly symptomatic patient is diagnostic of diffuse
esophageal spasm. The recommended treatment for this relatively rare disorder is
a long myotomy guided by the manometric evidence.

4. . A previously healthy 20-year-old man is admitted to a hospital with acute


onset of left-sided chest pain. The electrocardiographic findings are normal but
chest X-ray shows a 40% left pneumothorax. Treatment consists of :

The answer is pneumothorax of more than 25% requires placement of a chest


tube

5. A 50-year-old salesman is on a yacht with a client when he has a severe


vomiting and retching spell punctuated by a sharp substernal pain. He arrives in
your emergency room 4 hours later and has a chest film in which the left descend-
ing aorta is outlined by air density. Optimum strategy for care would be:

The answer is the presence of air in the mediastinum after an episode of vomit-
ing and retching is virtually pathognomonic of spontaneous rupture of the
esophagus (Boerhaave syndrome). Optimum strategy for care would be immedi-
ate thoracostomy
Study guide #14.1
“Diagnostic program in case of cardiac injuries. Triad of symptoms of cardiac
injuries, tactics of surgical treatment. Mediastinitis.”

Overview

Most cardiac injuries are the result of penetrating trauma. In civilian life the
rate of penetrating injuries of heart by cold steel arms (knife, daggers, etc.) consider-
ably predominate over the amount of gunshot wounds (bullet, fragmentation). If there
is a wound of the heart wall it is necessary to consider the damage of a cardiac muscle
without penetration into the heart cavity. In penetrating wound, the object which
causes the injury, penetrates into the cavity of heart. If the anterior and posterior walls
of the heart are damaged, it is called the perforating wound. Myocardial contusion
from direct myocardial impact occurs in approximately one third of patients sustain-
ing significant blunt chest trauma. Acute myocardial infarction is frequently the cause
of motor vehicle accidents or other trauma in older patients. While the diagnosis is
common, acute, life-threatening complications of ventricular arrhythmias and cardiac
pump failure occur in less than 5% and less than 1% of patients sustaining major
blunt chest trauma, respectively. Air embolism is a frequently overlooked lethal com-
plication of pulmonary injury.
Diseases of the mediastinum include mediastinitis, malignant and non-
malignant tumours, and cysts. Mediastinitis is the purulent inflammation of the medi-
astinum. It is a disease which threatens people’s life, has very high mortality at late
diagnosing or wrong treatment. More frequently it appears after operations on the
heart (approximately in 1% of operated patients). Increasing number of general
amount of patients with mediastinitis show large cardiologic surgical interferences.
During last 10 years optimal therapeutic measures of mediastinitis became more de-
termined. Practice has shown that only in 15-20% cases mediastinitis is diagnosed at
life of the patient. It is related to rare occurrence of disease, clear manifestations on
background of general severe infection and limited possibilities of treating patients
through rapid progress of process.
Educational aims
1. To acquaint the students with the prevalence of cardiac injuries, acute medias-
tinitis.
2. To understand the causes of mediastinitis origin, develop sense of responsibil-
ity of doctors for early diagnostics of mediastinitis, cardiac injuries.
3. According to the materials of the topic to form complex and rational approach
to the inspection of patients with the purpose of early diagnosis of cardiac inju-
ries.
4. To understand the influence of ecological, social and biological factors on the
development of cardiac injuries, mediastinitis.
5. According to the materials of the topic to develop the feeling of responsibility
for timely diagnosis and revealing of diseases and accuracy of professional ac-
tions with the purpose of its prophylaxis, treatment and prevention of compli-
cations.
A student must know:
1. Anatomy of mediastinum
2. Classification of tumours and cysts of mediastinum
3. Clinic, diagnostics and surgical tactics at acute mediastinitis
4. Methods of examination of patients with mediastinitis.
5. Modern possibilities of thoracoscopic surgery.
6. Principles of pre-operation preparation and postoperative period care.
7. Anatomo-physiological information about heart.
8. Modern ideas about etiology and pathogenesis of cardiac injuries.
9. Clinical picture of cardiac injuries.
10.Methods of diagnostics and differential diagnostics of cardiac injuries.
11.Principles of the treatment of cardiac injuries, indications to the different meth-
ods of treatment.
A student must be able to:
21.Take anamnesis carefully.
22.Make diagnosis correctly.
23.Order additional examination.
24.Perform differential diagnostics between the different diseases of mediastinum
25.Work with the results of roentgenologic examination of patients
26.Work with the information of ultrasonic diagnostic at pathology of mediasti-
num
27.Determine indications for the operation, to conduct pre-operation preparation
of patients.
28.Conduct pre- and post-operative period care for patients with pathology of me-
diastinum.
29.Take and work with complaints of patient with the cardiac injuries, information
of anamnesis, to conduct physical research and to interpret received results cor-
rectly.
30.Develop algorithm for the diagnostic search about cardiac injuries.
31.Interpret the results of endoscopy, ultrasonic diagnostics, computed tomogra-
phy, roentgenologic methods of research correctly.
32.Define indications for the operations and other methods of patients’ treatment.
33.Prescribe pre-operation preparation for patients and to perform a post-operation
period care.

Terminology.
Term Definition
is an emergency condition in which fluid accumu-
Pericardial tamponade lates in the pericardium (the sac in which the heart is
enclosed)
Myocardial contusion or Heart attack is the interruption of blood supply to
infarction part of the heart, causing heart cells to die.
is a pathological condition caused by gas bubbles in
Air embolism
a vascular system
A fulminant infectious process that spreads along the
Acute mediastinitis
fascial planes of the mediastinum.
hypotension, jugular venous distention, muffled
Beck's triad
heart sounds
is a procedure where fluid is aspirated from the peri-
Pericardiocentesis
cardium

Content
THE HEART DAMAGE

The spectrum of blunt cardiac injuries includes myocardial contusion, rupture,


and internal (chamber and septal) disruptions such as traumatic septal defects, papil-
lary muscle tears, and valvular tears. Myocardial contusions are by far the most
common of these injuries. They usually occur in persons who sustain a direct blow to
the sternum, as seen in a driver whose sternum is forcibly compressed by the steering
column in a deceleration injury.

The penetrating cardiac injuries are always characterized by the presence of the
orifice from injury, which is localized in "especially dangerous zone". The borders of
this zone are: from above - II rib, from below - left costal arch and epigastric region,
from the left anterior axillary line, from the right - parasternal line. The blood, which
flows into the cavity of pericardium squeezes cardiac muscle, disturbs cardiac activi-
ty, leads to discirculatory changes and can result in cardiac arrest. The latter usually
occurs at increasing of pressure in the pericardial cavity more, than 27 mm H20 which
relates to about 200 ml of volume of hemopericardium. In most cases the injuries of
pericardium, are associated with a pleural damage that results in the occurrence of
hemothorax.
In trauma patients the differential diagnosis of cardiogenic shock is a short list:
(1) tension pneumothorax, (2) pericardial tamponade, (3) myocardial contusion or in-
farction, and (4) air embolism.
Tension pneumothorax is the most frequent cause of cardiac failure and has
been discussed above.
Traumatic pericardial tamponade is most often associated with penetrating in-
jury to the heart. As blood leaks out of the injured heart, it accumulates in the peri-
cardial sac. Because the pericardium is not acutely distensible, the pressure in the per-
icardial sac will rise to match that of the injured chamber. Since this pressure is usual-
ly greater than that of the right atrium, right atrial filling is impaired and right ven-
tricular preload is reduced. This leads to decreased right ventricular output and in-
creased central venous pressure (CVP). Increased intrapericardial pressure also im-
pedes myocardial blood flow, which leads to subendocardial ischemia and a further
reduction in cardiac output. This vicious cycle may progress insidiously with injury
of the vena cava or atria, or precipitously with injury of either ventricle. With acute
tamponade, as little as 100 mL of blood within the pericardial sac can produce life-
threatening hemodynamic compromise.
Air embolism occurs when air from an injured bronchus enters an adjacent in-
jured pulmonary vein and returns to the left heart. Air accumulation in the left ventri-
cle impedes diastolic filling, and during systole it is pumped into the coronary arter-
ies, disrupting coronary perfusion. A typical scenario is a patient with a penetrating
chest injury who appears hemodynamically stable but suddenly arrests after being in-
tubated and placed on positive pressure ventilation. Air emboli also have been de-
scribed in conjunction with blunt thoracic trauma and can occur at any time when
manipulating a pulmonary venous injury.

Classification of cardiac injuries

1. Stab wounds of pericardium, heart, large vessels.


2. Isolated wounds of pericardium.
3. Isolated nonpenetrating cardiac injuries.
4. Wounds of coronary vessels (isolated and complicated by a penetrating
injury of cardiac wall).
5. Penetrating cardiac injuries.
6. Gunshot cardiac injuries.

Manifestation

Over 50% of patients with myocardial contusion demonstrate external signs of


thoracic trauma, including sternal tenderness, abrasions, ecchymosis, palpable crepi-
tus, rib fractures, or flail segments. Overall, fewer than 10% of patients have conduc-
tion abnormalities, dysrhythmias, or ischemic patterns on initial ECG.

The shock, as a rule, dominates among the clinical signs of a cardiac wound. It is
accompanied by loss of consciousness, caused by a massive hemorrhage or cardiac
tamponade (in 60-70 % of cases). Thus the majority of the patients enter the hospitals
without consciousness. The victims usually complain of weakness, dizziness, and
pain in the region of heart. The patients are frequently excited. Their skin is pale,
covered with cold, clammy sweat. Patients usually presents with a penetrating injury
in proximity to the heart, and they are hypotensive and have distended neck veins or
an elevated CVP.
The cardiac tamponade is characterized by Beck's triad:
• hypotension,
• jugular venous distention
• muffled heart sounds
On examination the attention is paid if the wound is located in "particular dan-
gerous zone". The apex beat in most cases is unsuccessfully found. Percussion reveals
the increase of heart borders, auscultation - a sharp weakening of cardiac sounds, and
sometimes a murmur "of a mill wheel”.
The data of additional investigation (a sharp reducing of voltage on ECG, and
enlargement of a cardiac shadow) enable to confirm the diagnosis of a cardiac wound.

Variants of clinical course and complications


Isolated nonpenetrating cardiac injuries. In these cases the injury is located
within the cardiac muscle. The complications of such damage depend on the depth
of the injury. A massive hemorrhage in pericardium, as a rule, is not present. How-
ever the situation sharply changes if there has been the injury of coronary vessels,
with predominant manifestations of cardiac ischemia.
Blunt cardiac injury can present in several ways. A sharp blow to the pericar-
dium can provoke ventricular fibrillation. This is referred to as commotio cordis and
is inevitably fatal unless it is recognized immediately and resuscitation implemented.
The heart also can be contused. Most of these patients present with new arrhythmias
(e.g., a bundle-branch block or premature ventricular contractions). Cardiac enzymes
have not been helpful in making the diagnosis. Finally, the heart can rupture; the right
atrium and right ventricle are most susceptible, but survival is possible provided the
diagnosis is suspected.
In penetrating wounds of heart and the left half of chest in some cases the
lung also could be damaged with the following development of pneumothorax and
pneumohemothorax. The pulmonary collapse and the respiratory disturbances ag-
gravate the state of the patients, and require immediate thoracotomy and cardiorrha-
phy.
Gunshot cardiac injuries (bullet, fragmentation), due to the injury of I both
anterior and posterior walls of the heart, are almost always fatal.
Diagnosis

Elevated cardiac isoenzyme levels are specific for myocardial injury, but they
lack clinical significance in patients without ECG abnormalities or hemodynamic in-
stability. First-pass radionuclide angiography (RNA) and echocardiography provide
sensitive assessment of ventricular wall motion and ejection fraction after blunt chest
trauma and are currently viewed as the gold standard for the diagnosis of myocardial
contusion. But while RNA and echocardiography sensitively detect small abnormali-
ties in myocardial function, they are poor predictors of the significant cardiac compli-
cations of pump failure and arrhythmia. Traditionally, management of patients with
myocardial contusion has included continuous ECG monitoring in an intensive care
unit for 48-72 h, even in hemodynamically stable patients without other injuries. Be-
cause of the large number of patients with blunt chest trauma from automobile acci-
dents, however, this policy has been scrutinized. Virtually all patients who develop
cardiac complications display ECG abnormalities on arrival in the emergency room
or within the first 24 h. Since an abnormal ECG is a good predictor of subsequent
complications, stable patients with possible myocardial contusions but with a normal
ECG tracing may be placed on telemetry for 24 h, rather than monitored in an inten-
sive care unit (ICU).
Ultrasonography (US) in the ED using a subxiphoid or parasternal view is ex-
tremely helpful if the findings are clearly positive; however, equivocal findings are
common for acute tamponade. The diagnostic criteria for myocardial contusion in-
clude some specific electrocardiographic abnormalities (i.e., ventricular dysrhythmi-
as, atrial fibrillation, sinus bradycardia, and bundle-branch block). Transient sinus
tachycardia is not indicative of contusion. Serial cardiac enzyme determinations (CK-
MB fraction (cardiac marker)) lack sensitivity, are not predictive of complications
under these conditions, and are not recommended.
Management
Early in the course of tamponade, blood pressure and cardiac output will tran-
siently improve with fluid administration. This may lead the surgeon to question the
diagnosis or be lulled into a false sense of security. Once the diagnosis of cardiac
tamponade is established, pericardiocentesis should be performed. Evacuation of as
little as 15 to 25 mL of blood may dramatically improve the patient's hemodynamic
profile. Pericardiocentesis should be done even if the patient appears to stabilize with
volume loading since subclinical myocardial ischemia can lead to sudden lethal ar-
rhythmias, and patients with tamponade may decompensate unpredictably. While per-
icardiocentesis is being performed, preparation should be made for emergent
transport to the OR. Emergent pericardiocentesis is successful in decompressing the
tamponade in approximately 80% of cases; most failures are due to clotted blood
within the pericardium. If pericardiocentesis is unsuccessful and the patient remains
severely hypotensive (SBP <70 mm Hg) or shows other signs of hemodynamic insta-
bility, ED thoracotomy should be performed. This is best accomplished using a left
anterolateral thoracotomy and a longitudinal pericardiotomy anterior to the phrenic
nerve, followed by evacuation of the pericardial sac and temporary control of the car-
diac injury. The patient is then transported to the OR for definitive repair.
Arrhythmias are treated by pharmacologic suppression. The management of
cardiogenic shock from cardiac pump failure includes early placement of a Swan-
Ganz pulmonary artery catheter to optimize fluid administration; inotropic support;
and urgent echocardiography to rule out septal or free wall rupture, valvular disrup-
tion, or pericardial tamponade. Patients with refractory cardiogenic shock may re-
quire placement of an intra-aortic balloon pump to decrease myocardial work and en-
hance coronary perfusion.
While the ideal initial management would be to provide optimal treatment for
the evolving infarction, decisions regarding lytic therapy and emergent angioplasty
must be individualized according to the patient's other injuries.
The patient should be placed in the Trendelenburg position to trap the air in the
apex of the left ventricle. Emergency thoracotomy is followed by cross-clamping the
pulmonary hilum on the side of the injury to prevent further introduction of air. Air is
aspirated from the apex of the left ventricle with an 18-gauge needle and 50-mL sy-
ringe. Vigorous open cardiac massage is used to force the air bubbles through the
coronary arteries. The highest point of the aortic root is also aspirated to prevent air
from entering the coronaries or embolizing to the brain. The patient should be kept in
the Trendelenburg position and the hilum clamped until the pulmonary venous injury
is controlled.
Most cardiac injuries are the result of penetrating trauma, and any part of the
heart is susceptible. Control of hemorrhage while the heart is being repaired is crucial
and several techniques can be used. The atria can be clamped with a Satinsky vascu-
lar clamp. Digital control and suturing beneath the finger is possible anywhere in the
heart, though the technique requires skill and a long, curved cardiovascular needle.
The reality of blood-borne viral infections raises the question of whether this method
should ever be used today. If the hole is small, a peanut sponge clamped in the tip of
a hemostat can be placed into the wound, or the blood loss may be accepted while su-
tures are being placed. For larger holes a 16F Foley catheter with a 30-mL balloon
can be inflated with 10 mL of saline. Gentle traction on the catheter will control hem-
orrhage from any cardiac wound since wounds too large for balloon tamponade are
incompatible with survival. Suture placement with the balloon inflated is problematic.
Usually the ends of the wound are closed progressively toward the middle until the
amount of blood loss is acceptable with the balloon removed. The use of skin staples
for the temporary control of hemorrhage has become popular, particularly when ED
thoracotomy has been performed. It has the advantages of reducing the risk of needle-
stick injury to the surgeon or assistant, and does not mandate the attention required by
a balloon catheter. In most instances, however, hemostasis is neither perfect nor de-
finitive. Inflow occlusion of the heart, by clamping the superior and inferior vena
cava can be performed for short periods, and this may be essential for the treatment of
extensive or multiple wounds, as well as for those that are difficult to expose. These
factors necessitate that the surgeon work very quickly.
Trauma surgeons accept the fact that interior structures of the heart may be
damaged, impairing cardiac output. However, immediate repair of valvular damage or
acute septal defects is rarely necessary and requires total cardiopulmonary bypass,
which has a high mortality in this situation. Most patients who survive to make it to
the hospital do well with only external repair. Following recovery, the heart can be
thoroughly evaluated, and if necessary, secondary repair can be performed under
more controlled conditions. Coronary artery injuries also pose difficult problems. Li-
gation leads to acute infarction distal to the tie; but again, reconstruction requires by-
pass. The right coronary artery can probably be ligated anywhere, but the resultant
arrhythmias may be extremely resistant to treatment. The left anterior descending and
circumflex cannot be ligated proximally without causing a large infarct. Fortunately,
such injuries are extremely rare as they usually produce death in the field.

MEDIASTINITIS
Mediastinum The boundaries of the mediastinum are the thoracic inlet, the di-
aphragm, the sternum, the vertebral column, and the pleura bilaterally. The mediasti-
num itself is divided into three portions delineated by the pericardial sac: the ante-
rosuperior and posterosuperior regions are in front of and behind the sac, respectively,
while the middle region designates the contents of the pericardium. Mediastinal
masses occur most frequently in the anterosuperior region (54%) and less often in the
posterosuperior (26%) and middle (20%) regions. Cysts (either pericardial, broncho-
genic, or enteric) are the most common tumors of the middle region; neurogenic tu-
mors are the most common (40%) of the primary tumors of the posterior mediasti-
num. The primary neoplasms of the mediastinum in the anteroposterior region are
thymomas (31%), lymphomas (23%), and germ-cell tumors (17%).

Injuries of mediastinum - There are open and closed injuries of mediastinum


and organs located in it. Clinical picture depends on the character of the trauma and
what organ of mediastinum is damaged and also on intensity of the internal or exter-
nal bleeding. Hemorrhage practically always arises at the closed trauma with forming
of the haematoma which can result in squeezing of vital organs (foremost the thin-
walled veins of mediastinum). Mediastinal emphysema and mediastinitis develop in
case of the damage of esophagus, trachea and primary bronchus. Clinically emphy-
sema is manifested by intensive pain behind a breastbone, characteristic crepitation in
the hypodermic cellular tissue of front surface of the neck, face, rarely pectoral wall.
The diagnosis is based on information of anamnesis (finding out the mechanism of
trauma), sequence of development of symptoms and the information of objective ex-
amination, exposure of the symptoms, characteristic for the damaged organ. At roent-
genologic research displacement of the mediastinum is visible in different sides, also
– the expansion of its shade, caused by hemorrhage. The considerable brightening of
the shade of the mediastinum is a roentgenologic symptom of mediastinal emphyse-
ma. The opened traumas usually combine with the damage of the organs of the medi-
astinum (that is accompanied by proper symptoms), and also bleeding, development
of the pneumomediastinum.

Descending necrotizing acute mediastinitis is acute suppurative inflammation


of cellular tissue of the mediastinum developing mostly as a necrotizing quickly pro-
gressive phlegmon. This form of the acute mediastinitis, arising out of acute suppura-
tive foci, is located on neck and head and is most common. Middle age of sick people
is 32—36 years, men are sick 6 times more frequent than women. The cause in more
than 50% cases is the odontogenic mixed aerobic-anaerobic infection, rarely an infec-
tion that comes from retropharyngeal abscesses, iatrogenic damages of esophagus,
lymphadenitis of lymphatic nodes of the neck and acute thyroiditis. An infection
quickly goes down along fascial layers of neck (mainly through visceral —
retroesophageal) in the mediastinum and causes severe necrotizing inflammation of
its tissues. Rapid dissemination of infection onto a mediastinum takes place by virtue
of gravitation and gradient of pressure, arising from pulling action of respiratory mo-
tions. Descending necrotizing mediastinitis differs from other forms of acute medias-
tinitis by extraordinarily fast growth of the inflammatory process and severe sepsis
(patient dies in 24-48 h). In spite of aggressive surgical interference and modern anti-
biotic therapy, mortality reaches 30%. Perforation of esophagus (damage by foreign
body, instrument, caused by diagnostic and treatment manipulations), postoperative
insufficiency of stitches are the sources of descending infection of mediastinum. This
mediastinitis necessary to be distinguished from necrotizing descending mediastinitis,
because it makes separate clinical unit and requires the special algorithm of treatment.
Manifestation and diagnostics The characteristic signs of descending ne-
crotizing mediastinitis are high temperature of body, chill, pain in the neck and face,
disorders of breathing. Sometimes there is reddening and slight swelling in the chin
area or the neck. The appearance of signs of inflammation in the oral cavity serves as
a signal to the beginning of immediate surgical treatment. Crepitation in this area can
be related to the anaerobic infection or emphysema, caused by trachea or esophagus
damage. Difficult breathing is the sign of threat of larynx edema, obstruction of the
respiratory tract. At roentgenologic research, increasing of retrovisceral (retroesopha-
geal) space, presence of liquid or edema is marked in this area, displacement of tra-
chea, emphysema of the mediastinum, smoothing of lordosis of the neck department
of the spine. For the confirmation of diagnosis it is necessary to make computed to-
mography immediately. Finding out the edema of tissue, accumulations of liquid in
the mediastinum and in the pleural cavity, emphysema of the mediastinum and the
neck allows determining the diagnosis and specifying the distribution borders of the
infection.
Acute postoperative mediastinitis is observed more frequent after longitudinal
sternotomy, used in the operations on a heart. International statistics show that its fre-
quency varies from 0,5 to 1,3%, and during transplantation of heart to 2,5 %. Mortali-
ty at postoperative mediastinitis reaches 35%. Causative agents in more than 50%
cases are Staphylococcus aureus, Staphylococcus epidermidis, rarely – Pseudomonas,
Enterobacter, Escherichia coli. The factors of risk of the development of the medias-
tinitis are obesity, heart operations, and cardiac insufficiency. Symptoms which allow
suspecting postoperative mediastinitis are increasing feeling of pain in the area of the
wound, displacement of the edges of the injured breastbone at cough or at palpation,
increase of the temperature of the body, shortness of breath, reddening and edema of
the edges of the wound. They appear usually on 5-10-th days after the operation, but
sometimes in a few weeks. Increase of temperature of the body, moderate leukocyto-
sis and slight mobility of breastbone can be observed at other diseases, which makes
diagnosis more difficult. Absence of changes at computed tomography does not elim-
inate mediastinitis. Usually in this research edema of tissues of the mediastinum and
small amount of liquid is found and it is a foundation for the hypothetical diagnosis of
mediastinitis in 75% cases. A chest CT scan can be particularly helpful in determin-
ing the extent of the spread and the best approach to surgical drainage.
Chronic (sclerosing) mediastinitis is often named fibrotic. Sclerosing or fi-
brosing mediastinitis is a result of chronic inflammation of the mediastinum, most
frequently in case of granulomatous infections such as histoplasmosis or tuberculosis.
The process begins in lymph nodes and continues as a chronic, low-grade inflamma-
tion leading to fibrosis and scarring. In many patients, the clinical manifestations are
silent. However, if the fibrosis is progressive and severe, it may lead to encasement of
the mediastinal structures, causing entrapment and compression of the low-pressure
veins (including the superior vena cava and innominate and azygos veins). This fi-
brotic process can compromise other structures such as the esophagus and pulmonary
arteries. The most frequent presentation of the illness - cough, shortness of breath,
difficulty of breathing, syndrome of squeezing of vena cava superior. More rarely
there is dysphagia, pain in the breast, secretion of blood with sputum. The diagnosis
of fibrotic mediastinitis often has to be set by the method of exception. At roentgeno-
logic research the changes of contours of the mediastinum, squeezing of pulmonary
artery and veins are visible. The most informing is computed-tomography examina-
tion, allowing defining prevalence of fibrosis, exposing granuloma and deposits of
calcium in it, to reveal squeezing of anatomic structures of mediastinum. Vascular
changes are easier diagnosed at computer tomography with the contrast studying. De-
pending on the symptoms of the disease for diagnostics we may require bronchosco-
py (narrowing, displacement of bronchial tubes, bronchitis), roentgenoscopy of
esophagus and esophagoscopy, ultrasonic examination of heart and other methods, as
a diagnosis of mediastinitis is quite often made by the method of exception.

Management

Acute mediastinitis is a true surgical emergency and treatment must be institut-


ed immediately and must be aimed at correcting the primary problem, such as the
esophageal perforation or oropharyngeal abscess. Another major concern is debride-
ment and drainage of the spreading infectious process within the mediastinum, neck,
pleura, and other tissue planes. Antibiotics, fluid resuscitation, and other supportive
measures are important, but surgical correction of the problem at its source and open
debridement of infected areas are critical measures. Surgical debridement may need
to be repeated, and other planes and cavities explored depending on the patient's clin-
ical status. Blood cell counts and serial CT scans may also be required. Persistent
sepsis or collections on CT scan may require further radical surgical debridement.
Sclerosing or fibrosing mediastinitis There is no definitive treatment. Surgery
is indicated only for diagnosis or in specific patients to relieve airway or esophageal
obstruction or to achieve vascular reconstruction. Reports of palliative success with
less invasive procedures (such as dilation and stenting of airways, the esophagus, or
the superior vena cava) are promising (30% died during long-term follow-up). Chron-
ic mediastinitis is similar to the fibrotic changes that occur in other sites including ret-
roperitoneal fibrosis, sclerosing cholangitis, and Riedel's thyroiditis.
Basic literatures:
59.Oxford Textbook of Surgery (3-Volume Set) 2nd edition (January 15, 2000):
by Peter J. Morris (Editor), William C. Wood (Editor) By Oxford Press
60.Sabiston Textbook of Surgery 17th edition by Courtney M. Townsend Jr.,
Kenneth L. Mattox, B. Mark, MD Evers, Kenneth L., MD Mattox, Courtney
Townsend, Daniel Beauchamp, B. Mark Evers, Kenneth Mattox W.B. Saun-
ders Company (June, 2004)
61.Schwartz´s Principles of Surgery 8th Edition F.Charles Brunicardi. Copyright
©2007 The McGraw-Hill Companies.
62.Hospital surgery/ Edited by L.Kovalchuk et al.- Ternopil: Ukrmedknyha,
2004.- 472 p.

Additional literatures:
1. Hoerbelt R, Keunecke L, Grimm H, et al: The value of a noninvasive diagnos-
tic approach to mediastinal masses. Ann Thorac Surg 75:1086, 2003.
2. Razzuk MA, Razzuk LM, Hoover SJ, et al: Infections of the mediastinum, in
Pearson FG, Cooper JD, Deslauriers J, et al (eds): Thoracic Surgery, 2nd ed.
New York: Churchill Livingstone, 2002, p 1599.
3. Wall MJ, Mattox KL, Chen C, et al: Acute management of complex cardiac in-
juries. J Trauma 42:905, 1997. [PMID: 9191673]

4. Maron BJ, Gohman TE, Kyle SB, et al: Clinical profile and spectrum of com-
motio cordis. JAMA 287:9, 2002.
5. Biffl WL, Moore FA, Moore EE, et al: Cardiac enzymes are irrelevant in the
patient with suspected myocardial contusion. Am J Surg 169:523, 1994.

Tests for initial level of knowledge, keys for tests:

1.Middle mediastinal masses include all of the following EXCEPT


A. Bronchogenic cyst
B. Ascending aortic aneurysm
C. Pericardial cyst
D. Ganglioneuroma
E. Lymphoma

2.Highly malignant tumors, arising from the sympathetic chain in the posterior
mediastinum
A.Neuroblastomas
B. Bronchogenic cyst
C. Thymoma
D. Pericardial cyst
E. Teratoma
3.Tumors occur in the anterior mediastinum and are associated with the clinical
syndrome of myasthenia gravis
A. Neuroblastomas
B. Bronchogenic cyst
C. Thymoma
D. Pericardial cyst
E. Teratoma

4. Tumors contain elements of all three germ cell layers and frequently calcify
A. Neuroblastomas
B. Bronchogenic cyst
C. Thymoma
D. Pericardial cyst
E. Teratoma

5.Cysts are the mesothelial origin and are located in the middle mediastinum, usu-
ally at the cardiophrenic angle
A. Neuroblastomas
B. Thymoma
C. Pericardial cyst
D. Teratoma
E. Bronchogenic cyst

6.Tumors primary to the thymus gland may be associated with which of the fol-
lowing disorders?
A. Lupus erythematosus
B. Scleroderma
C. Myasthenia gravis
D. Stricture of the esophagus
E. Pericarditis

7.Embriologically, the thymus gland is related to the


A. Fourth brachial pouch
B. Inferior parathyroid glands
C. Superior parathyroid glands
D. Third brachial pouch

8.Correct statements about thymic tumors (thymomas) include which of the fol-
lowing?
A. They are among the commonest mediastinal tumors
B. They have a female preponderance
C. Their pathology is best determined at surgical exploration
D. Most tumors in adults are malignant

9.Recommended treatment for malignant thymomas includes


A. Chemotherapy
B. Radiation therapy
C. Immunotherapy with monoclonal anti-T-cell antibody
D. Resection of the entire tumor, including areas of invasion

10. A 40-year-old woman with generalized myasthenia gravis is being treated with
pyridostigmine (Mestinon), a cholinesterase inhibitor. CT scan shows no evidence
of a thymoma. The next step in management of this patient should be:
A. Continued treatment with pyridostigmine
B. Serial CT scans and thymectomy if a thymoma is detected
C. High-dose steroid (prednisone) therapy
D. Plasmapheresis followed by thymectomy

Keys for tests

1 2 3 4 5 6 7 8 9 10
D A C E C C C B C D

Tests for final level of knowledge, keys for tests:

1. A 26-year-old man is brought to the emergency room after being re-


moved from the driver’s seat of a car involved in a head-on collision in which the pa-
tient was not wearing his seat belt. His ECG is shown below. The ECG is most con-
sistent with

A. Preexisting disease
B. Myocardial ischemia that caused the accident
C. Myocardial contusion that resulted from the accident
D. Chagas disease
E. Normal variant
2. A 26-year-old man is brought to the emergency room after being extri-
cated from the driver’s seat of a car involved in a head-on collision in which the pa-
tient was not wearing his seat belt. His ECG is shown above. The best test for estab-
lishing the diagnosis and the degree of myocardial dysfunction is
A. Serial ECGs
B. Creatine phosphokinase (CPK-MB) fractionation
C. Echocardiography
D. Radionuclide angiography
E. Coronary angiography

3. A 35-year-old man presents with a history of 4 days of severe substernal


pain and fever to 38.89°C (102°F). He has a past medical history of peptic ulcer dis-
ease that resulted in a Billroth II procedure 5 years earlier. On admission, the chest
film below is obtained. What is it?

A. Pericardial effusion is present


B. The condition may be managed with antibiotics and close observa-
tion if the patient remains hemodynamically stable
C. The condition could have resulted from recurrent peptic ulcer dis-
ease
D. Pneumopericardium
E. The previous Billroth II procedure effectively rules out peptic ulcer
as the cause of the condition

4. Select the proper intervention for pericardial tamponade


A. Endotracheal intubation
B. Cricothyroidotomy
C. Subxiphoid window
D. Tube thoracostomy
E. Occlusive dressing

5. Regarding myocardial contusion from blunt chest trauma, which of the


following statements is correct?
A. Elevated cardiac isoenzyme levels sensitively identify patients
at risk for life-threatening arrhythmias
B. The majority of patients have abnormalities on the initial ECG
post injury
C. First-pass radionuclide angiography (RNA) and echocardiog-
raphy are considered the gold standard for diagnosis
D. RNA and echocardiography are good predictors of subsequent
cardiac complications such as arrhythmias and pump failure
E. All patients diagnosed with myocardial contusion should be
monitored in an intensive care unit setting for 72 h

6. Which of the following situations would be an indication for performance of


a thoracotomy in the emergency room?
A. Massive hemothorax following blunt trauma to the chest
B. Blunt trauma to multiple organ systems with obtainable vital signs
in the field but none on arrival in the emergency room
C. Rapidly deteriorating patient with cardiac tamponade from pene-
trating thoracic trauma
D. Penetrating thoracic trauma and no signs of life in the field
E. Penetrating abdominal trauma and no signs of life in the field

7. The patient of 42 was taken in 3 hours after a trauma with the expressed
hypodermic emphysema of overhead half of a body, shortness of breath, tachycardia,
and pulse-120. Roentgenologic did not discover pneumotorax, mediastinum is
considerably extended in both sides. What is the first aid?
A. Draining of anterior mediastinum
B. Draining of posterior mediastinum
C. Draining of superior mediastinum
D. Draining of inferior mediastinum
E. Thoracotomy

8. The superior vena cava syndrome is most frequently seen in association with
Histoplasmosis (sclerosing mediastinitis)
A. Substernal thyroid
B. Thoracic aortic aneurysm
C. Constrictive pericarditis
D. Bronchogenic carcinoma
E. Bronchiectasis

9. Which part of mediastinum doesn’t exist


A. Anterior
B. Posterior
C. Middle
D. Front
E. Superior
F. Inferior

10. A 54-year-old woman crashes her car against a telephone pole at high
speed. On arrival at the ER she is breathing well. She has multiple bruises over the
chest, and is exquisitely tender over the sternum at a point where there is a gritty feel-
ing of bone grating on bone, elicited by palpation. What is it?
A. Sternal fracture with myocardial contusion
B. Traumatic rupture of the aorta.
C. Traumatic rupture of the gullet
D. Traumatic rupture of the stomach
E. Traumatic rupture of the lung

Keys for tests


1 2 3 4 5 6 7 8 9 10
C D D C C C A D C A

Tasks for final level of knowledge

1. A 22-year-old man is involved in a high-speed, head-on automo-


bile collision. He arrives in the ER in coma, with fixed, dilated pupils. He has
multiple obvious fractures in both upper extremities and in the right lower leg.
His blood pressure is 70 over 50, with a barely perceptible pulse rate of 140.

What is it? Shock in the trauma setting is caused by bleeding (the


most common source), pericardial tamponade.

2. A 22-year-old gang member arrives in the ER with multiple gun shot


wounds to the chest and abdomen. He is diaphoretic, pale, cold, shivering, anx-
ious, and asking for a blanket and a drink of water. His blood pressure is 60 over
40. His pulse rate is 150, barely perceptible. He has big distended veins in his
neck and forehead. He is breathing okay and has bilateral breath sounds and no
tracheal deviation.

What is it? Pericardial tamponade


3. During a domestic dispute a young woman is stabbed in the chest with a
6-inch-long butcher knife. On arrival at the ER she is found to have an entry wound
just to the left of the sternal border, at the fourth intercostals space. Her blood pres-
sure is 80 over 50, her pulse rate is 110, and she is cold, pale, and perspiring heavily.
She has big distended neck and facial veins, but she is breathing normally and has
bilateral breath sounds.

What is it? Pericardial tamponade

4. A 54-year-old woman crashes her car against a telephone pole at high speed.
On arrival at the ER she is breathing well. She has multiple bruises over the
chest, and is exquisitely tender over the sternum at a point where there is a
gritty feeling of bone grating on bone, elicited by palpation.

What is it? Obviously a sternal fracture (which a lateral chest x-ray will con-
firm), but the point is that she is at high risk for myocardial contusion and for trau-
matic rupture of the aorta.

5. A patient of 32 has the closed trauma of thorax. Hypodermic emphysema


appeared in the area of jugular pit, which quickly spread on a neck, head, upper part
of a body. What is the mechanism of such development of hypodermic emphysema?

The answer is affection of airways with the development of mediastinum


emphysema
Study guide #15
“Traumatic damages of the thorax. Clinical picture and diagnosis of pneumo-
thorax, hemothorax and chylothorax. Tactics of surgical treatment.”

Overview.

Modern traumatism represents an important social problem. Recent observation


shows an increase of major combined trauma, whose complications often result in
death. Approximately 30% of patients presenting with significant trauma have chest
wall injury. Trauma of the chest is usually accompanied by dysfunction of the vital
organs. Therefore it is necessary to constantly improve diagnostics and treatment of
the patients who suffer from trauma.
Thoracic organs are vitally important and considerably different from one an-
other in closeness, consistency, mobility and ability to withstand the impact of a
force/trauma. Life threatening situations that may develop rapidly such as valvular
pneumothorax, hemothorax, tamponade of heart and atelectasis of lungs requires a
doctor’s good knowledge of the clinics of this pathology, ability to quickly recognize
it and make the most rational decision in relation to the character and maintenance of
medical care. Closed trauma of the lungs accounts for 60% of cases, ribs (45, 4%),
heart (7, 7%), spine (4, 8%), diaphragm (3, 7%), large vessels (3, 0%) and large bron-
chial tubes (2, 6%).
Penetrating wounds of the chest are more life threatening for victims due to the
possible damage of intrapectoral organs and development of internal bleeding, em-
physema of mediastinum and increased lungs-cardiac insufficiency.
Educational aims:
35. Interrogation and clinical inspection of patients with trauma of thorax.
36. To determine the etiologic and pathogenic factors of traumas of thorax.
37. To find out the types of traumas of thorax, the clinical features, different
variants of manifestation and complications.
38. To develop a plan of examination of the patients with trauma of thorax.
39. To estimate laboratory data, results of thoracoscopy, ECG, bronchosco-
py, diagnostic puncture, sciagrams of the thoracic organs.
40. To draw a differential diagnosis, substantiate and formulate a diagnosis
of the trauma of thorax.
41. To determine the indications for treatment of patients with the trauma of
thorax.
42. To cure of the patients with the traumas of thorax after operations.
43. To estimate efficiency of treatment and prognosis of disease.
A student must know:
31.Аnatomo-physiological information about thorax and organs of mediastinum.
32.Classifications of chest damage.
33.Mechanisms of chest trauma.
34.Clinical picture of damages of thorax.
35.Methods of diagnostics of chest damages.
36.Principles of treatment of chest trauma.
37.Giving first aid for chest trauma.
38.Features of surgical interventions for the different traumas of thorax.
39.Technique of draining of pleural cavity.
A student must be able to:
26.Collect and estimate the complaints of patient with the trauma of thorax, in-
formation of anamnesis, to conduct physical research and correctly interpret the
results obtained.
27.Define the rational volume of laboratory and instrumental methods of research.
28.Correctly interpret the results of clinical analyses, pleural tapping, ultrasound
diagnostics, computed tomography, X-ray
29.Give first aid for traumas of the chest.
30.Define indications for operation and other methods of treatment of patients.
31.Perform pre-operative preparation of patients.
32.Conduct post-operative care.

Terminology.
Term Definition
traumatic action on organs and tissues within the limits of
Isolated trauma
one anatomic area (thorax, abdomen, skull, extremities)
Combined trauma traumatic action within the limits of two and more of ana-
(polytrauma) tomic areas
combination two or more, different mechanism of action
Combination trauma
of traumatic factors
Вlunt damages of the the mechanical damages of chest, when there is no viola-
chest tion of the skin covering the area
Pneumothorax the accumulation of air within the pleural space
Hemothorax the accumulation of blood in the pleural space
the accumulation of chyle in the pleural space after trau-
Chylothorax
ma to the thoracic duct or a major branch

Content:
The opened damages can be impenetrable or penetrable, blind or through,
without damage or with damage of internal organs or bones. As a rule, they are com-
plicated opened or valvular pneumothorax, hemothorax, hemopneumothorax and
hemopericardium.
The mechanical damage of chest is blunt, if there is no violation of safety of the
skin covering this area. In a peacetime such damages occur 9-10 times more frequent
than opened and accounts for 9% of the common traumas cases. It is known that 40-
45% lost because of mechanical traumas. During the last decade, these types of dam-
ages have become more frequent, due to an increased rate of movement of transports,
wide distribution of tall buildings and other reasons. Damages of the chest are direct
reason of injury in every quarter of cases and accounts for pain in half of the cases of
traffic accident.

Classification of blunt damages of the chest.


I. According to the injury of other organs:
 Isolated trauma.
 Combined trauma (cranio-cerebral, with the damage of abdominal organs, with
the damage of bones).
II. According to the mechanism of trauma:
 Contusion.
 Compression.
 Commotion.
 Fracture.
III. According to the character of the damage of chest organs:
 Without damage of organs.
 With damage of organs (lungs, trachea, bronchi, esophagus, heart, vessels, dia-
phragm etc.).
IV. According to the character of complications:
 Uncomplicated.
 Complicated:
1. Early (pneumothorax, hemothorax, subcutaneous, mediastinal emphysema
floatative rib fracture, traumatic shock, asphyxia);
2. Late (posttraumatic pneumonia, posttraumatic pleurisy, suppurative diseas-
es of lungs and pleura).
V. According to the state of cardiopulmonary system:
 Without phenomena of respiratory failure.
 Acute respiratory failure (of I, II, III degree).
 Without phenomena of cardiovascular failure.
 Acute cardiovascular failure (of I, II, III degree).
VI. According to the severity of trauma:
 Mild.
 Moderate.
 Severe

Trauma to the chest wall is common and can range from an isolated single rib
fracture to flail chest. Approximately 30% of patients presenting with significant
trauma have chest wall injury. Guidelines of the Advanced Trauma Life Support pro-
gram (Airway, Breathing, Circulation, Disability, and Exposure) always should be
followed in the preliminary assessment of these patients. This organized approach
helps to rule out injuries to the underlying viscera such as the lungs, heart, liver, and
spleen, all of which frequently are associated with chest wall injury.
Blunt chest wall trauma commonly results in contusion with localized tissue
swelling and hematoma formation. In severe cases, these injuries can progress to soft
tissue infections or necrosis. Initially, it often is difficult to distinguish between deep
muscle injury and bony fractures, given the pain that is caused by these injuries.
When subcutaneous emphysema is palpable on the chest wall, injury to the airway or
lung parenchyma leading to a pneumothorax or esophageal perforation should be sus-
pected.
Rib fractures are a common injury sustained after blunt chest wall trauma. A
higher incidence of fractures is observed in the elderly owing to the loss of chest wall
compliance from ossification of costal cartilage and osteoporosis. Symptoms include
pain during inspiration and localized tenderness. The management of rib fractures de-
pends on the number and location of the injuries. Upper thoracic rib fractures (T1-T5)
are uncommon because of the relatively protected position of these ribs below the up-
per girdle musculature. Fractures of the first two thoracic ribs usually are seen in
high-velocity injuries and can be associated with aortic disruption (6%). Similarly,
fractures of the lower thoracic ribs (T11-T12) are uncommon because the ribs are
short and less exposed. Frequently, fractures to ribs 11 and 12 are associated with in-
juries to underlying abdominal organs such as the spleen, liver, and diaphragm. Frac-
tures to thoracic ribs 5 to 10 are most commonly reported. Flail chest is a unique inju-
ry in which rib fractures lead to an unstable chest wall that result in a paradoxical mo-
tion during respiration. The injuries must occur along the same rib to produce the
free-floating segment. This injury arises from blunt chest wall trauma such as direct
impact from a steering wheel.
Pneumothorax is the accumulation of air within the pleural space. Pneumotho-
rax may be spontaneous or occur secondary to a traumatic, surgical, therapeutic, or
disease-related event. A pneumothorax compresses lung tissue and reduces pulmo-
nary compliance, ventilatory volumes, and diffusing capacity. These pathophysiolog-
ic consequences depend primarily on the size of the pneumothorax and condition of
the underlying lung. If air enters the pleural space repeatedly (as with inspiration) and
is unable to escape, positive pressure develops in the pleural space, causing compres-
sion of the entire lung, shifting of the mediastinum and heart away from the pneumo-
thorax, and severe respiratory compromise with hemodynamic collapse. This situa-
tion is called a tension pneumothorax.
Valvular pneumothorax is caused by the damage of a pulmonary tissue or chest
wall with formation of the valve, when the air during inspiration enters a pleural
space, and during expiration, due to valve closure, does not exit. It is the most dan-
gerous form of pneumothorax, which results in a complete pulmonary collapse, shift
of mediastinum, inflection of major vessels and cardiac arrest.
It may be the consequence of a pneumothorax from many causes. Pneumotho-
races may be classified as shown:
Spontaneous
 Primary
 Secondary
 Chronic obstructive pulmonary disease (COPD)
 Bullous disease
 Cystic fibrosis
 Pneumocystis-related congenital cysts
 Idiopathic pulmonary fibrosis (IPF)
 Pulmonary embolism
 Catamenial
 Neonatal
Traumatic
 Penetrating
 Blunt
Iatrogenic
 Mechanical ventilation
 Thoracentesis
 Lung biopsy
 Venous catheterization
 Postsurgical
Other

A primary spontaneous pneumothorax occurs without a known cause or evi-


dence of diffuse pulmonary disease or from subpleural blebs. A secondary spontane-
ous pneumothorax occurs as the result of an underlying pulmonary process that pre-
disposes to pneumothorax. Iatrogenic pneumothoraces are common and may be
caused by thoracentesis, central venous catheterization, surgery, mechanical ventila-
tion, or diagnostic lung biopsy. Patients with pneumothorax most commonly present
with chest pain. It is often sharp and pleuritic and may lead to severe respiratory em-
barrassment or become dull and persistent. Dyspnea is the second most common
symptom in patients with pneumothorax. Less common symptoms include nonpro-
ductive cough and orthopnea.

Hemothorax is the accumulation of blood in a pleural space. The cause of oc-


currence of this complication is the damage of vessels of the chest wall, pleura, lungs
and mediastinum.

Classification

I. According to extent:
1. Unilateral.
2. Bilateral.

II. According to the degree of hemorrhage:


1. Small (the loss is less than 10 % of volume of circulating blood).
2. Moderate (loss of 10-20 % of volume of circulating blood).
3. Great (loss of 20-40 % of volume of circulating blood).
4. Total (exceeds 40 % of volume of circulating blood).

III. According to the duration of bleeding:


1. With continued hemorrhage.
2. With the stopped bleeding.

IV. .According to the presence of clots in a pleural space:


1. Coagulated.
2. Non-coagulated.
V. According to the presence of infection:
1. Non-infected.
2. Infected (suppurative).

If hemothorax is the complication of blunt chest trauma, the clinical manifesta-


tions depend on the severity of the trauma and degree of hemorrhage. Also hemotho-
rax by itself results in pulmonary compression and shift of mediastinum.
In case of small hemothorax the clinical manifestations of hemorrhage are
slightly expressed or absent at all.
Dyspnea, cough, general malaise and dizziness are obvious in moderate hemo-
thorax. The skin is pale. The hemodynamic disturbances: tachycardia and decreased
arterial pressure are observed.
The great and total hemothorax are associated with extremely grave condition.
The patients are troubled with expressed general malaise, dizziness, dyspnea and dif-
ficult breathing. In some cases they enter medical hospitals in a terminal state. The
skin is extremely pale. The peripheral pulse is impaired or absent. Tachycardia, weak
cardiac tones, low arterial pressure is obvious.
The coagulated hemothorax. When patients delay in seeking medical aid or
when there is a major bleeding result in formation of clots in a pleural space and in
some cases all blood, which has accumulated in a pleural space, forms by itself a ma-
jor entire clot. Depending on degree of bleeding and, consequently, size of clot, the
patients complain of chest pain, which intensifies at respiration, dyspnea, general ma-
laise, and dizziness. As a rule, in 3-5 days a fever of 37, 5-38°C is observed.
Chylothorax develops most commonly after surgical trauma to the thoracic
duct or a major branch, but may be also associated with a number of other conditions
(Table 1). It is generally unilateral; for example, it may occur on the right after
esophagectomy where the duct is most frequently injured during dissection of the dis-
tal esophagus. The esophagus comes into close proximity to the thoracic duct as it en-
ters the chest from its origin in the abdomen at the cisterna chyli. If the mediastinal
pleura is disrupted on both sides, bilateral chylothoraces may occur. Left-sided chylo-
thoraces may develop after a left-sided neck dissection, especially in the region of the
confluence of the subclavian and internal jugular veins. Chylothorax may also follow
nonsurgical trauma, including penetrating or blunt injuries to the chest or neck area,
central line placements, and other surgical misadventures. It is also seen in neonates,
probably secondary to birth trauma. It may be seen in association with a variety of
benign and malignant diseases that generally involve the lymphatic system of the me-
diastinum or neck. Given the significant variability of the course of the thoracic duct
within the chest, some injuries are inevitable. The direct relationship of chylothorax
to a surgical procedure, traumatic event, or neoplastic process may not always be ob-
vious. Understanding the anatomy and course of the thoracic duct and some of its
more common variants is helpful.
Table 1 Etiology of Chylothorax
Congenital
• Atresia of thoracic duct
• Thoracic duct-pleural space fistula
• Birth trauma
Traumatic and/or latrogenic
• Blunt
• Penetrating
• Surgery
i. Cervical: excision of lymph nodes; radical neck dissection
ii. Thoracic
 Patent ductus arteriosus
 Coarctation of the aorta
 Vascular procedure reinvolving the origin of left subclavian artery
 Esophagectomy
 Sympathectomy
 Resection of thoracic aneurysm
 Resection of mediastinal tumors
 Left pneumonectomy
iii. Abdominal: sympathectomy; radical lymph node dissection
• Diagnostic procedures
i. Translumbar arteriography
ii. Subclavian vein catheterization
iii. Left-sided heart catheterization
• Neoplasms
• Infections
i. Tuberculous lymphadenitis
ii. Nonspecific mediastinitis
iii. Ascending lymphangitis
iv. Filariasis
• Miscellaneous
i. Venous thrombosis
ii. Left subclavian-jugular vein
iii. Superior vena cava
iv. Pulmonary lymphangiomatosis
SOURCE: Reproduced with permission from Cohen RG, et al: The pleura, in Sabiston DC, et al (eds):
Surgery of the Chest, 6th ed. Elsevier, 1995.

Most commonly, the thoracic duct originates in the abdomen from the cisterna chyli,
which is located in the midline, near the level of the second lumbar vertebra. From
this origin, the thoracic duct ascends into the chest through the aortic hiatus at the
level of T10 to T12, and courses just to the right of the aorta (see Fig. 18-40). As the
thoracic duct courses cephalad above the diaphragm, it most commonly remains in
the right chest, lying just behind the esophagus, between the aorta and azygos vein.
The duct continues superiorly, lying just to the right of the vertebral column. Then, at
about the level of the fifth or sixth thoracic vertebra, it crosses behind the aorta and
the aortic arch into the left posterior mediastinum. From this location, it again courses
superiorly, staying near the esophagus and mediastinal pleura as it exits the thoracic
inlet. As it exits the thoracic inlet, it passes just to the left, just behind the carotid
sheath and anterior to the inferior thyroid and vertebral bodies. Just medial to the an-
terior scalene muscle, it courses inferiorly and drains into the union of the internal
jugular and subclavian veins. Given the extreme variability in the main duct and its
branches, accumulation of chyle in the chest or flow from penetrating wounds may be
seen after a variety of traumatic and medical conditions.

The main function of the duct is to transport fat absorbed from the digestive
system. The composition of normal chyle is fat, with variable amounts of protein and
lymphatic material (Table 2). Given the high volumes of chyle that flow through the
thoracic duct, significant injuries can cause leaks in excess of 2 L per day; if left un-
treated, protein, volume, and lymphocyte depletion can lead to serious metabolic ef-
fects and death. The diagnosis generally requires thoracentesis, which may be grossly
suggestive; often the pleural fluid is milky and nonpurulent. However, if the patient is
nil per os (NPO, nothing by mouth), the pleural fluid may not be grossly abnormal.
Laboratory analysis of the pleural fluid shows a high lymphocyte count and high tri-
glyceride levels. If the triglyceride level is greater than 110 mg/100 mL, a chylotho-
rax is almost certainly present (a 99% accuracy rate). If the triglyceride level is less
than 50 mg/mL, there is only a 5% chance of chylothorax. In many clinical situations,
the accumulation of chyle may be slow, because of minimal digestive fat flowing
through the gastrointestinal tract after major trauma or surgery, so the diagnosis may
be more difficult to establish.

Table 2 Composition of Chyle


Component Amount (per 100 mL)
Total fat 0.4–5 g
Total cholesterol 65–220 mg
Total protein 2.21–5.9 g
Albumin 1.1–4.1 g
Globulin 1.1–3.1 g
Fibrinogen 16–24 g
Sugar 48–200 g
Electrolytes Similar to plasma
Cellular elements
Lymphocytes 400–6800/mm3

Erythrocytes 50–600/mm3

Antithrombin globulin >25% plasma concentrate


Prothrombin >25% plasma concentrate
Fibrinogen >25% plasma concentrate
SOURCE: Reproduced with permission from Miller JI: Diagnosis and management of chylothorax.
Chest Surg Clin North Am 6:139, 1996.

Manifestation and diagnosis.


After a severe mechanical trauma, patients usually complaint of severe pain on
the damaged side of chest, increased during movements, cough and deep breathing.
When there is damage of pulmonary tissues, shortness of breath, frequent cough and
hemoptysis occurs. When there is a massive loss of blood there are characteristic
symptoms in the pleural cavity: increased pulse decreased arterial pressure, pallor of
skin, vascular and respiratory insufficiency, and shock.
In area of damage of soft tissues of breast and broken ribs a haematoma is often
visible. At palpation of this area a sharp sickliness, especially expressed at the broken
ribs, is marked. Crepitating of splinters of bones is sometimes felt. At the damage of
pulmonary tissues it is possible to define hypodermic emphysema on characteristic
crepitation of air, saved in soft tissues of thorax.
When there is a severe pleural damage of the framework of thorax, lungs and
vessels the signs of loss of blood are distinctly determined, to respiratory and vascular
insufficiency, pneumo- and hemothorax. Their clinical symptoms will be shown be-
low.
A traumatic asphyxia is easily recognized by the characteristic navy blue color-
ing of overhead part of trunk, petechial spit of blood on a skin and shortness of
breath. Sometimes there is a temporal loss of eyesight and ear in connection with spit
of blood in these organs.
When subcutaneous emphysema is palpable on the chest wall, injury to the
airway or lung parenchyma leading to a pneumothorax or esophageal perforation
should be suspected. The diagnosis of flail chest is made on clinical examination.
On percussion, dullness is revealed in case of hemothorax. On auscultation -
the breathing over the site of hemothorax is sharply diminished or absent. The physi-
cal findings in case of the coagulated hemothorax (diminishing or absence of vocal
fremitus by palpation, dullness by percussion and sharply diminished or absent
breathing by auscultation) suggest the presence of pathological process in a pleural
space.
Chest radiography and chest CT can be helpful in case of chest trauma.
X-ray of the lungs
The lungs should be reviewed individually and in comparison, matching zone
by zone from apex to base. Before assessing the lung density the patient's position
should be noted. Rotation of the patient is identified by observing the distance be-
tween the medial end of the clavicle and the adjacent margin of the vertebral body. It
should be symmetrical in a correctly positioned patient. Rotation will alter the density
between the lungs, the side closer to the film appearing blacker. Any discrepancy in
density between the lungs in a patient who is not rotated must be regarded as patho-
logical. Serial radiographs may show a variation in lung density, in the same subject
depending upon the depth of respiration: the greater the degree of lung inflation, the
more translucent the lungs appear.
The normal linear markings are composed of the pulmonary blood vessels and
the fissures. An alteration in vasculature may be local or generalized, and should be
interpreted in conjunction with an assessment of the hilar pulmonary vessels and the
degree of lung inflation. Abnormal linear shadows may be identified from their com-
partment of origin, namely, the lung parenchyma, the interstitium, the bronchovascu-
lar bundles, or the pleura. Although it may be difficult to distinguish between paren-
chymal and interstitial origin, this method of evaluation is a useful aid to interpreta-
tion.
The silhouette sign is useful for identifying and localizing pulmonary disease.
The cardiac and diaphragmatic outline is entirely dependent on adjacent air-filled
lung for its visualization. If the lung opacifies, the outline will be obscured. Any in-
trathoracic abnormality that obliterates the contour of the heart or diaphragm is in an-
atomical continuity with these surfaces. Loss of the right heart border implies middle
lobe disease or, on the left, disease of the lingula. A collapsed and opaque lower lobe,
lying posteriorly, will not efface the cardiac border but will obscure the adjacent dia-
phragmatic contour. The pulmonary vessels should be examined from the periphery
towards the hilum. This method prevents the more prominent hilar vessels from di-
verting one's attention from the less conspicuous vessels of the periphery. The vessels
in comparable zones of the lung should be of similar size and number. The vessels
should be clearly defined: marginal haziness indicates an abnormality of the peri-
vascular connective tissue or lung parenchyma. The vessels in the upper zone of the
lungs are smaller than those of the lower zones at equivalent levels, because of pref-
erential perfusion of the lung bases due to gravity. In the outer periphery (1 to 2 cm)
of the lung, vessels are no longer visible. The pulmonary arterial system is invariably
related to the bronchial tree; they branch together as they run in the bronchovascular
bundles. The pulmonary veins lie within the interlobular septa and are thus separate
from the bronchoarterial pathways. These vessels terminate medially into two superi-
or and two inferior pulmonary veins. In the upper zones the veins lie lateral to the ar-
teries, while in the lower zones they have a more horizontal course. The apical, seg-
mental upper lobe bronchus may be visualized end-on, at the hilum. Comparison with
an adjacent opaque artery is a useful assessment of the calibre of the pulmonary cir-
culation: normally the artery is of a similar calibre to the bronchus. Redistribution of
blood flow resulting in the upper lobe vessels being larger than the lower lobe vessels
is a well recognized sign of increased pulmonary venous pressure.
Computed tomography of the chest
CT is used routinely in the assessment of benign and malignant lesions
throughout the chest wall, lung fields, or mediastinum. CT may also be used in pa-
tients with thoracic trauma, to define or confirm an abnormality detected or suspected
on the standard chest radiograph. Cross-sectional imaging overcomes the difficulties
of interpretation due to superimposition of structures on the chest film. CT is particu-
larly sensitive in detecting pleural abnormalities, air, and fluid collections, which may
pass unnoticed on the supine radiograph. It allows discrimination of blood from other
pleural fluids due to the differences in density and is valuable in the assessment of
mediastinal widening. Patients with high velocity deceleration injury and suspected
trauma to the aorta or brachiocephalic arteries must always be studied by angiography
to locate the site of injury precisely. However, in stable patients with mediastinal
widening in whom there is a low index of suspicion of aortic injury; CT with intrave-
nous contrast is advised. The mediastinal widening may be elucidated as being due to
a fractured sternum, unfolded thoracic aorta, congenital vascular anomaly, a para-
mediastinal pleural collection, or a paraspinal haematoma related to a vertebral frac-
ture. Pulmonary injury, including laceration, haematoma, and contusion is clearly de-
fined. CT contrast studies allow differentiation of contused lung from adjacent col-
lapsed lung or haematoma. Pulmonary abnormalities noted on CT are usually more
extensive than those recognized on the plain chest radiograph. Pleural lesions, haema-
toma, and fractures may be demonstrated by CT but are usually apparent on clinical
examination and on the plane radiograph. CT may be helpful in the evaluation of my-
ocardial injury. The ability to demonstrate tracheobronchial or esophageal disruption
is uncertain. Contrast examination of the esophagus remains the investigation of
choice in suspected esophageal rupture. In patients with severe lower thoracic injury,
CT scanning of the liver and spleen following intravenous contrast should be per-
formed.
Rib radiographs can help to confirm the diagnosis of rib fracture in an acute
setting but cannot completely rule out this injury. 3 to 6 weeks after injury, callus
formation around the fracture site is evident on repeat films.
The diagnosis of primary spontaneous pneumothorax usually is established by
history and physical examination and confirmed with chest radiography. Patients are
often tall, thin men from 25 to 40 years of age. Physical findings may be normal if the
pneumothorax is less than 25%. Characteristic physical findings include diminished
chest excursion and hyperresonance on percussion of the affected side. Breath sounds
are diminished to absent. Rarely, subcutaneous emphysema may be palpated or
pneumomediastinum auscultated on cardiac examination. A pneumothorax usually is
seen on the standard posteroanterior chest radiograph with displacement of the vis-
ceral pleura from the parietal pleura by air in the pleural space. The area appears hy-
perlucent with absent pulmonary markings. An end-expiratory chest radiograph may
appear to increase the size of the pneumothorax because of reduction in lung volume
during forced expiration. Recognition of a pneumothorax may be difficult on portable
supine or semirecumbent chest radiographs obtained in trauma or critically ill patients
because of both the location of the least dependent pleural spaces (anterior, subdia-
phragmatic) and associated radiographic findings. Patients with bullous disease also
may have chest radiographs that are difficult to interpret; chest CT may be useful in
these situations. The routine use of CT in patients with spontaneous primary pneumo-
thorax is not warranted because the confirmation of apical blebs does not change
treatment recommendations. The occurrence of apical blebs and bullae in these pa-
tients has been found to be greater than 85% in most recent surgical series.
The X-ray picture of hemothorax is rather specific. The intensive homogeneous
shadow on the side of the lesion with oblique upper contour (Damuaso' line) is ob-
served. The costal sinus is not visualized. In small hemothorax, depending on the de-
gree of intrapleural bleeding, the shadow observed only in the region of sinus. In
moderate hemothorax it reaches a scapular angle (on the back surface) or 5th rib on
anterior surface of the chest wall. In great hemothorax this shadow reaches the 3rd
rib, and total hemothorax characterized by complete shadow of a pleural space, and in
some cases - mediastinal shift to the healthy side. Chest roentgenogram of the coagu-
lated hemothorax reveals the intensive shadow, sometimes heterogeneous (with en-
lightenments and multiple levels).
At the small tears of bronchial tubes or trachea the state of patients some time
can remain satisfactory, and the first symptoms appear pretty lately because of for-
mation of scar, causing stenosis of the damaged bronchial tube. It is diagnosed on the
basis of information of bronchoscopy at which detects the rupture of the tracheal
wall.
Thoracoscopy
Conventional wisdom relates minimal access to limited exposure, but with the
advent of videoscopic surgery this is no longer true. The thoracoscope is attached to a
video camera unit and it provides a magnified view of the surgical field with high
resolution for details. The chest is the most suitable body cavity for the minimal ac-
cess approach not only because thoracotomy is a very painful incision but also be-
cause once the lung is collapsed (with selective one-lung ventilation) there is plenty
of room for instrument manoeuvring. The use of carbon dioxide insufflations, and
hence valved ports, is unnecessary. Conventional thoracic instruments can be placed
directly through small wounds into the chest.

Management

Soft tissue infections or necrosis require antibiotic therapy and debridement.


Circumferential burns to the chest wall require escharotomy to allow adequate chest
wall expansion.
Fractures to thoracic ribs 5 to 10 are most commonly reported. Injury to three
or more ribs often requires hospitalization for analgesia and monitoring of respiratory
status. Splinting from improperly controlled pain can lead to atelectasis, retained se-
cretions, and pneumonia. This is a particular problem in the elderly population. Anal-
gesia can be provided using oral, intravenous, or intramuscular opioid analgesics for
mild-to-moderate injuries, or epidural analgesia or intercostal nerve blocks for more
severe injuries. Pain relief in closed trauma of the chest is achieved by means of dif-
ferent blocks:
1. Vagosympathetic block.
2. Alcohol - novocaine block of the site of fracture.
3. Paravertebral block.
On the 2nd to 3rd day it is desirable to administrate electrophoresis with Novo-
caine. For the prophylaxis of congested phenomena in a pulmonary tissue used res-
piratory gymnastics, forced ventilation of lungs and inhalations.
Pulmonary contusion is the most commonly associated injury. Maintenance of
adequate ventilation is the goal of therapy. Stabilization of the chest wall has been at-
tempted using weights and rib binders, as well as fixation devices such as pins and
plates. The methods of renewal of the skeleton of the flail chest are divided onto three
groups:
1. External fixation of a movable segment by means of suturing for intercostal
muscles and traction during 2-3 weeks,
2. Intermedullary costal osteosynthesis;
3. Mechanical ventilation (often with positive end-expiratory pressure).
Mechanical ventilation with positive-pressure ventilation also occasionally is
used to treat injuries in the elderly or in those patients with underlying pulmonary
disease. Some centers report a more rapid wean from mechanical ventilation with the
use of internal fixation.
The treatment of a first-time spontaneous pneumothorax depends on the size of
pneumothorax, associated symptoms, and pulmonary history. Small pneumothoraces
(<20%) that are stable may be monitored if the patient has few symptoms. Follow-up
of a pneumothorax should include a chest radiograph to assess stability within 24 to
48 hours. An uncomplicated pneumothorax should reabsorb at a rate of approximate-
ly 1% per day. Indications for intervention include progressive pneumothorax, de-
layed pulmonary expansion, or development of symptoms. Moderate (20% to 40%)
and large (>40%) pneumothoraces nearly always are associated with persistent symp-
toms that cause physical limitations and require intervention. Simple needle aspira-
tion of a pneumothorax may relieve symptoms and can promote faster lung reexpan-
sion. It also may help to determine whether the initial fistula that caused the pneumo-
thorax has sealed or if there is an ongoing air leak that requires chest tube insertion.
This method is carried out using a standard thoracentesis kit and either an evacuated
bottle or hand aspiration via a three-way stopcock and syringe. The needle generally
is placed either anteriorly or laterally. The needle aspiration may be repeated, or a
chest tube or needle catheter/thoracic vent drainage system may be inserted. It pro-
vides excellent management of iatrogenic pneumothoraces after central venous access
or lung needle biopsy. This approach conservatively treats a sealed pneumothorax
and identifies those with an active air leak for chest tube insertion. Emergent needle
decompression for tension pneumothorax is carried out on the affected side by plac-
ing an 18-gauge needle or angiocatheter into the hemithorax at the midclavicular line
in the second anterior intercostal space. This emergency maneuver relieves the ten-
sion created within the thorax. It does not treat the pneumothorax; subsequent chest
tube insertion is required. Tube thoracostomy (chest tube insertion) and underwater
seal drainage are the mainstays of treatment for spontaneous pneumothorax.
Full re-expansion of the lung, even in the presence of a continuous leak, usual-
ly can be achieved with the application of suction to the thoracostomy drainage sys-
tem. The classic location for chest tube insertion is the same as for emergency needle
decompression because the tube can be inserted quickly and easily without the need
for patient positioning. The preferred approach is through the fourth, fifth, or sixth
intercostal space in the mid-to-anterior axillary line. This can be done under local an-
esthetic employing rib blocks or under intravenous procedural sedation. The chest
tube should be directed upward to the apex of the hemithorax. Care should be taken
to avoid the subcutaneous placement of a chest tube. Digital pleural dilatation is rec-
ommended to confirm entrance into the chest cavity, appreciate any adhesions, and
allow passage of the chest tube without need for a stylet, which can cause damage to
the lung or other intrathoracic structures. Needle catheter/thoracic vent drainage sys-
tems may be used for the treatment of pneumothorax. This system is comparable to a
chest tube and drainage system, although the tube is of much smaller diameter and is
inserted by means of the Seldinger technique or stylet. The end of the needle catheter
drain is modified to be completely compatible with the many underwater seal drain-
age systems available. Many kits also include a Heimlich valve (also available sepa-
rately), which can be used in conjunction with either a catheter drain or conventional
chest tube. The Heimlich valve and thoracic vent function as a one-way valve that lets
air escape from the hemithorax, similar to an underwater seal. Patients may be dis-
charged with these in place, to be removed at a later time after the leak has stopped.
Complications of chest tube insertion for pneumothorax are infrequent but in-
clude puncture of the underlying lung, with air leakage and pneumothorax; subdia-
phragmatic entry, with damage to the liver, spleen, or other intra-abdominal viscera;
bleeding secondary to intercostal vessel injury, or most commonly, larger vessel inju-
ry; and even cardiac puncture. Other technical complications include loss of a cathe-
ter, guidewire, or fragment in the pleural space, and infections. Occasionally, rapid
drainage of a large effusion can be followed by shortness of breath, clinical instabil-
ity, and a phenomenon referred to as postexpansion pulmonary edema. For this rea-
son, it is recommended to drain only up to 1 L initially. Most complications can be
avoided by consulting with a clinician experienced in pleural drainage techniques.

Re-expansion pulmonary edema is a rare complication that can be seen after


treatment of a pneumothorax. Risk factors for this complication have not been con-
sistently identified. Although re-expansion pulmonary edema is thought to be second-
ary to a sudden increase in capillary permeability, the exact mechanism of this in-
creased permeability is unknown. Most cases have been reported after rapid lung re-
expansion. An air leak may be present for a variable amount of time after tube thora-
costomy. Should the air leak persist for more than 72 hours or the lung does not com-
pletely re-expands, surgical intervention is compulsory. Primary spontaneous pneu-
mothorax tends to recur with increasing frequency after each episode. The risk of
first-time recurrence is about 25% to 30%.

Surgery is recommended for a recurrence or the development of a contralateral


pneumothorax. Surgical intervention for a first-time pneumothorax is recommended
in situations that include bilateral simultaneous pneumothoraces, complete (100%)
pneumothorax, pneumothorax associated with tension, and borderline cardiopulmo-
nary reserve and in patients in high-risk professions or activities involving significant
variations in atmospheric pressure, such as pilots or scuba divers. Surgery for compli-
cations of pneumothorax (empyema, hemothorax, or chronic pneumothorax) also is
recommended in patients with first-time spontaneous pneumothorax. Surgery for
primary spontaneous pneumothorax has evolved over recent years from open thora-
cotomy (axillary or posterolateral) to a minimally invasive video-assisted technique.
The surgery carried out is identical, despite the differences in approach. Apical blebs
are resected. The parietal pleura over the apex of the hemithorax can be removed
(pleurectomy), abraded (mechanical pleurodesis), or treated with talc or tetracycline-
like agents (chemical pleurodesis or poudrage). The recurrence rate for these proce-
dures, performed open or closed, is less than 5%.
Treatment options for primary and secondary spontaneous pneumothorax are
similar. However, patients with secondary pneumothorax generally are debilitated
from a respiratory standpoint and may have other significant comorbid diseases.
Treatment with tube thoracostomy alone has a high recurrence rate. Effective treat-
ment must be individualized but should include chemical or surgical pleurodesis in
combination with complete lung reexpansion and effective sealing of air leaks.
The tremendous success of video-assisted thoracic surgery in the treatment of
primary spontaneous pneumothorax has led to earlier referral by physicians and in-
creased acceptance by patients for surgery. Stapled resection of apical bullas followed
by mechanical pleurodesis remains the most frequently used technique, although
more cost-effective means of eliminating the bullas (like suturing or looping) have
been developed. While cases of primary spontaneous pneumothorax are easily ap-
proachable by video-assisted thoracic surgery, treatment of secondary spontaneous
pneumothorax (with established lung pathology like emphysema or pneumoconiosis)
requires more clinical judgment. Patients with difficult adhesions to take down may
be more suitable for thoracotomy, while those who are elderly with multiple comor-
bidities may benefit more from a chemical pleurodesis (we prefer talc slurry) if the
lung can be fully re-expanded.
The treatment of small hemothorax requires needle aspiration or drainage of
pleural space and elimination of blood at the mid-to-anterior axillary line in the sixth
or seventh intercostal space. The manipulation is carried out in VI-VII intercostal
spaces in the postaxillary or scapular lines. Total, great or moderate hemothorax with
persistent bleeding (positive Revilour-Greguar's test) requires thoracotomy for liqui-
dation of a bleeding source. The bleeding wounds of lungs are sewed up by twist su-
ture. If the pleural space contains liquid blood, the surgeon carries out its reinfusion.
The clots are removed from pleural space.
In case of the coagulated hemothorax the needle aspiration obtains small
amount of a liquid hemolyzed blood and small bloody thrombi (consequently to the
inner diameter of the needle).
The treatment plan for any chylothorax depends on its cause, the amount of
drainage, and the clinical status of the patient. In general, most patients are treated
with a short period of chest tube drainage, NPO orders, total parenteral nutrition
(TPN), and observation. Chest cavity drainage must be adequate to allow compete
lung expansion. Somatostatin has been advocated by some authors, with variable re-
sults. If significant chyle drainage (>500 mL per day in an adult, >100 mL in an in-
fant) continues despite TPN and good lung expansion, early surgical ligation of the
duct is recommended. Ligation can be approached best by right thoracotomy, and in
some experienced centers, by right VATS. Chylothoraces due to malignant conditions
often respond to radiation and/or chemotherapy, so less commonly require surgical
ligation. Untreated chylothoraces are associated with significant nutritional and im-
munologic depletion that leads to significant mortality. Before the introduction of
surgical ligation of the thoracic duct, the mortality rate from chylothorax exceeded
50%. With the availability of TPN for nutritional supplementation and surgical liga-
tion for persistent leaks, the mortality rate of chylothorax is less than 10%.
Basic literatures:
63.Oxford Textbook of Surgery (3-Volume Set) 2nd edition (January 15, 2000):
by Peter J. Morris (Editor), William C. Wood (Editor) By Oxford Press
64.Sabiston Textbook of Surgery 17th edition by Courtney M. Townsend Jr.,
Kenneth L. Mattox, B. Mark, MD Evers, Kenneth L., MD Mattox, Courtney
Townsend, Daniel Beauchamp, B. Mark Evers, Kenneth Mattox W.B. Saun-
ders Company (June, 2004)
65.Schwartz´s Principles of Surgery 8th Edition F. Charles Brunicardi. Copyright
©2007 the McGraw-Hill Companies.
66.Hospital surgery/ Edited by L. Kovalchuk et al. - Ternopil: Ukrmedknyha,
2004. - 472 p.
Additional literatures:
1. Shackford SR, Virgilio RW, Peters RM. Selective use of ventilator therapy
in flail chest injury. Journal of Thoracic Cardiovascular Surgery 1988; 81:
194–201.
2. Snow N, Richardson JD, Flint LM. Myocardial contusion: Implications for
patients with multiple traumatic injuries. Surgery 1982; 92: 744–50.
3. Spencer JA, Rogers CE, Westaby S. Clinico-radiological correlates in rup-
ture of the major airways. British Journal of Radiology 1991; 43: 371–6.
4. Westaby S. Blunt injuries to the chest. In: Westaby S, ed. Trauma: patho-
genesis and treatment. Heinemann Medical Books, London, 1989, pp. 118–
41.
5. Westaby S. Penetrating wounds of the chest and abdomen. In: Westaby S,
ed. Trauma: pathogenesis and treatment. Heinemann Medical Books, Lon-
don, 1989, pp. 159–80.

Tests for initial level of knowledge, keys for tests:

1. Chest X-ray of a 48 years old patient on the 4th day after a blunt trauma
of the chest shows non-homogenic shadow at the lower part of the thorax. Puncture -
small amount of the light-yellow exudate with the blood clots. What is the best treat-
ment for the patient?
A. Lung decortications
B. Drainage of pleural cavity
C. Daily punctures
D. Dissolving therapy
E. Antibacterial therapy

2. A 45-year-old patient complains of weakness after a blunt chest trauma


with sternal fracture. Physical examination reveals hypotension, cyanosis of the upper
part of the body, distension of the cervical veins. Pleural puncture is negative. Heart
rate 120, regular, pulse is weak. What is your diagnosis?
A. Lung thromboembolism
B. Heart tamponade
C. Heart shock
D. Heart infarction
E. Hemopericardium

3. All of the following symptoms and signs are indicative of a tension


pneumothorax EXCEPT:
A. Chest pain
B. Shortness of breath
C. Shifting of the trachea towards the pneumothorax
D. Absent of breath sounds unilaterally
E. Hypotension

4. A 24-year-old man involved in a motor vehicle accident presents with


blood pressure 80/60, heart rate 135, and respirations 45 and labored. Physical exam-
ination reveals agitation, sternal flail, and absent breath sound in the left hemithorax,
which is crepitate to auscultation and resonant to percussion. The initial maneuver
should be to:
A. Place a left chest tube
B. Obtain a chest x-ray
C. Intubate the patient
D. Begin two large-bore intravenous needles
E. Antishock therapy
5. Chest x-ray of a 55-year-old man involved in a high-speed motor vehicle
accident shows a widened mediastinum and pneumomediastinum, and electrocardio-
gram shows sinus tachycardia with frequent premature ventricular contractions. All of
the following maneuvers are appropriate at this time EXCEPT:
A. Aortography
B. Bronchoscopy
C. Continuous cardiac monitoring
D. Left thoracotomy
E. Endotracheal intubation

6. Chest x-ray of a 55-year-old man involved in a high-speed motor vehicle


accident shows a widened mediastinum and pneumomediastinum, and electrocardio-
gram shows sinus tachycardia with frequent premature ventricular contractions. Ex-
pected physiologic changes due to blunt chest trauma include all of the following
EXCEPT
A. Increased compliance
B. Elevated PaCO2
C. Elevated A gradient
D. Decreased ventricular contractions
E. Elevated shunt fractions

7. The usual management of cardiac arrest should include all of the follow-
ing protocols EXCEPT
A. Immediate resuscitation, as irreversible brain damage will result after 3-4
minutes of diminished perfusion
B. Establishment of an airway and ventilatory support
C. Defibrillation, if cardiac arrest is due to ventricular fibrillation
D. Administration of cardiotonic agent
E. Open-chest cardiac massage

8. A 32-year-old male suddenly felt suffocation, pain in the left hemithorax,


palpitations. Physical examination reveals grave condition, blood pressure - 100/70
mm Hg, heart rate - 100, respirations - 28, the left side of the chest lag behind in
breathing, tympanic sound at percussion on the left. The cause of this condition may
be?
A. Spontaneous pneumothorax
B. Myocardial infarction
C. Lung thromboembolism
D. Intercostal neuralgia
E. Hypotonic crisis

9. A male patient, 48-year-old came to the surgeon on duty with the com-
plaints of pain in the right hemithorax, suffocation, cough, giddiness. Anamnesis: one
hour ago, he fell down from a bike and struck the right side of his chest on the pave-
ment edge. Physical examination reveals: skin and mucosa are pale, depression, BP -
90/60 mm Hg, heart rate 100, weak breathing on the right, percussion - blunt sound in
lower part of the thorax. What pathologic process may be suspected?
A. Right sided closed pneumothorax
B. Right sided hemothorax
C. Fracture of the ribs on the right side
D. Right sided tension pneumothorax
E. Right sided pyopneumothorax

10. During an endoscopic biopsy of a distal esophageal cancer, perforation


of the esophagus is suspected when the patient complains of significant new subster-
nal pain. An immediate chest film reveals air in the mediastinum. You would recom-
mend:
A. Placement of a nasogastric tube to the level of perforation, antibiotics, close
observation
B. Spit fistula (cervical pharyngostomy), gastrostomy
C. Left thoracotomy, pleural patch over sewing of perforation, drainage of me-
diastinum
D. Esophagogastrectomy via celiotomy and right thoracotomy
E. Transhiatal esophagogastrectomy with cervical esophagogastrostomy

Keys for tests

1 2 3 4 5 6 7 8 9 10
A B C A D A E A B D

Tests for final level of knowledge, keys for tests:


11. A previously healthy 20-year old man is admitted to a hospital with acute
onset of left-sided chest pain. The electrocardiographic findings are normal but chest
X-ray shows a 40% left pneumothorax. Treatment consists of which of the following
procedures?
A. Observation
B. Barium swallow
C. Thoracotomy
D. Tube thoracostomy
E. Thoracostomy and intubation

12. A 50-year-old salesman is on a yacht with a client when he has a severe


vomiting and retching episode punctuated by a sharp substernal pain. He arrives in
your emergency room 4 hours later and has a chest film in which the left descending
aorta is outlined by air density. Optimum strategy for care would be:
A. Immediate thoracotomy
B. Serial ECGs and CPKs to rule out myocardial ischemia
C. Left chest tube and spit fistula (cervical esophagostomy)
D. Flexible esophagogastroscopy to establish diagnosis
E. Nasogastric tube, antibiotics, close monitoring

13. A sharp left-sided chest pain appeared in a 35-year-old patient at the time
of intense physical activity. Objectively: the patient is covered with cold sweat;
breathing is difficult because of the pain. Auscultation: on the right side the breath
sound is vesicular, on the left side - weakened. Tachycardia, heart rate -100. What is
your diagnosis?
A. Spontaneous pneumothorax
B. Heart attack
C. Myocardial infarction
D. Intercostal Neuralgia
E. Pneumonia

14. Tension pneumothorax has the following features, EXCEPT:


A. Displacement of the heart on the side opposite pneumothorax.
B. Distension of the neck veins.
C. Arterial hypotension.
D. All of the signs, without exception.
E. The blunt percussion sound on the side of the lesion.

15. What is the first aid in a patient with tension pneumothorax?


A. Tube thoracostomy
B. Pleural puncture in VII intercostal space
C. Occlusive bandage
D. Thoracotomy
E. Bronchoscopy

16. A 35-year-old patient has been admitted to a district hospital a week after
a vehicle accident with the clinical signs of a convoluted hemothorax. What is the
feasible therapeutic tactics in terms of prevention of acute pleural empyema in the pa-
tient?
A. Pleural puncture
B. Complex haemostatic conservative therapy
C. Surgical removal of a convolute hemothorax
D. Tube thoracostomy with passive drainage
E. Thoracostomy with active drainage

17. Which of the following situations would be an indication for perfor-


mance of a thoracotomy in the emergency room?
A. Massive hemothorax following blunt trauma to the chest
B. Blunt trauma to multiple organ systems with obtainable vital signs
in the field but none on arrival in the emergency room
C. Rapidly deteriorating patient with cardiac tamponade from penetrat-
ing thoracic trauma
D. Penetrating thoracic trauma and no signs of life in the field
E. Penetrating abdominal trauma and no signs of life in the field

18. A teenage boy falls from his bicycle and is run over by a truck. On arri-
val in the emergency room, he is awake and alert and appears frightened but in no
distress. The chest radiograph suggests an air-fluid level in the left lower lung field
and the nasogastric tube seems to coil upward into the left chest. The next best step in
management is:
A. Placement of a left chest tube
B. Immediate thoracotomy
C. Immediate celiotomy
D. Esophagogastroscopy
E. Removal and replacement of the nasogastric tube; diagnostic peri-
toneal lavage

19. A 65-year-old man who smokes cigarettes and has chronic obstructive
pulmonary disease falls and fractures the 7th, 8th, and 9th ribs in the left anterolateral
chest. Chest x ray is otherwise normal. Appropriate treatment might include:
A. Strapping the chest with adhesive tape
B. Immobilization with sandbags
C. Tube thoracostomy
D. Peritoneal lavage
E. Surgical fixation of the fractured ribs

20. A 25-year-old woman arrives in the emergency room following an auto-


mobile accident. She has acute dyspnea with a respiratory rate of 60 breaths/min.
Breath sounds are markedly diminished on the right side. The first step in managing
the patient should be to:
A. Take a chest x-ray
B. Draw arterial blood for blood gas determination
C. Decompress the right pleural space
D. Perform pericardiocentesis
E. Administer intravenous fluids

Keys for tests


1 2 3 4 5 6 7 8 9 10
D A A E A C E C D C

Tasks for final level of knowledge

1. A 34-year-old woman suffers severe blunt trauma in a car accident. She


has multiple injuries to her extremities, head trauma, and pneumothorax on the left
side. Shortly after initial examination it is noted that she is developing
progressive subcutaneous emphysema all over her upper chest and lower neck.

What is it? Thoracic subcutaneous emphysema.

2. A 22-year-old gang member arrives in the ER with multiple gunshot wounds


to the chest and abdomen. He has labored breathing and is cyanotic, diaphoretic, cold,
and shivering. His blood pressure is 60 over 40. His pulse rate is 150, barely percepti-
ble. He is in respiratory distress and has big distended veins in his neck and forehead,
his trachea is deviated to the left, and the right side of his chest is hyperresonant to
percussion, with no breath sounds.
The answer is tension pneumothorax.

3. A 25-year-old man is stabbed in the right chest. He is moderately short of


breath, has stable vital signs. No breath sounds at the base on the right chest, faint dis-
tant breath sounds at the apex. Dull to percussion at the base.

The answer is hemothorax.

4. A 25-year-old man is stabbed in the right chest. He is moderately short of


breath, has blood pressure of 95 /70, pulse rate of 100. No breath sounds are heard
over the right chest, which is dull to percussion. A chest tube placed at the right pleu-
ral base recovers 1250 ml of blood.

The answer is hemopneumothorax.

5. Several days following esophagectomy a patient complains of dyspnea and


chest tightness. A large pleural effusion is noted on chest radiograph and thoracen-
tesis yields milky fluid consistent with chyle.

The answer is chylothorax.

Materials for the self-study of the students

Main tasks Notes (instructions)


Repeat:
1. Anatomy of organs of medias-
tinum, lobar structure of lungs and
anatomy of pleural cavity, features of
blood circulation of lungs -To represent the methods of diagnostics
2. Physiology of pleura and lungs of diseases of lungs and pleura as a table
3. Pathogenesis of development -To make the flow diagram of mecha-
of complications from thoracic trau- nisms of damage of thorax
ma
4. Morphological changes in the
organs of mediastinum

Study:
1. Types of thoracoplastics.
2. Types of accesses to the organs -To conduct differential diagnosis with
of pectoral cavity. the damages of organs of mediastinum
3. Tactics in thoracoabdominal trau- -To conduct differential diagnosis of pa-
ma thologies of the pleura
4. Tactics at connections of head
trauma and trauma of thorax.
Study guide #16-17

“Diseases that lead to the syndrome of limb ischemia. Conservative and surgical
treatment of limb arterial occlusion. An estimation of pain syndrome for differ-
ential diagnosis of vascular diseases, lesions of the nervous system and pathology
of locomotive system.”

Overview.
Atherosclerosis is the most common cause of chronic arterial occlusive dis-
ease of the lower extremities. The arterial narrowing or obstruction that occurs as a
result of the atherosclerotic process reduces blood flow to the lower limb during ex-
ercise or at rest. Epidemiological studies indicate that up to 5% of men and 2.5% of
women 60 years of age or older have symptoms of intermittent claudication. The
prevalence is at least threefold higher when sensitive noninvasive tests are used to
make the diagnosis of arterial insufficiency in asymptomatic and symptomatic indi-
viduals. The symptoms of chronic arterial insufficiency of the lower extremities
progress rather slowly over time. Thus, after 5 to 10 years, more than 70% of pa-
tients report either no change or improvement in their symptoms, while 20% to 30%
have progressive symptoms and require intervention, and less than 10% need ampu-
tation. Despite the relatively benign prognosis for the affected limb, however, symp-
toms of intermittent claudication should be viewed as a sign of systemic atheroscle-
rosis. This explains why, compared with age-matched controls, patients with inter-
mittent claudication have a threefold increase in cardiovascular mortality
Acute arterial ischemia more frequent is an outcome of acute thrombosis of ex-
istent stenotic arterial segment (in 60%) or embolism (30%). Differential diagnostics
of these two pathological states is important, as these circumstances influence on tac-
tic of treatment and prognosis. Among other reasons of arterial ischemia is a trauma,
iatrogenic damage, aneurysm, defects of heart.
A risk of development of thromboses after surgical operations: Prosthetics of
hip joint - 60-65%. Amputation of hip 20%. Prosthetics of a knee-joint 25%.

Educational aims:
44. Interrogation and clinical inspection of patients with limb arterial occlusion,
lesions of the nervous system and pathology of locomotive system.
45. To determine the etiologic and pathogenic factors of limb arterial occlusion,
lesions of the nervous system and pathology of locomotive system.
46. To find out the types of limb arterial occlusion, lesions of the nervous system
and pathology of locomotive system, the clinical features, different variants
of manifestation and complications.
47. To develop a plan of examination of the patients with limb arterial occlusion,
lesions of the nervous system and pathology of locomotive system.
48. To estimate laboratory data end instrumental examination of the arteries.
49. To draw a differential diagnosis, substantiate and formulate a diagnosis for the
patients with limb arterial occlusion, lesions of the nervous system and pa-
thology of locomotive system.
50. To prescribe the treatment for patients with limb arterial occlusion, lesions of
the nervous system and pathology of locomotive system.
51. To determine the indications for operative treatment of patients with limb arte-
rial occlusion, lesions of the nervous system and pathology of locomotive
system.
52. To cure of the patients with limb arterial occlusion, lesions of the nervous sys-
tem and pathology of locomotive system.
53. To estimate efficiency of treatment and prognosis of disease.
A student must know:
40.Аnatomo-physiological information about arteries.
41.Classifications of of acute and chronic ischemia of extremities.
42.Mechanisms of limb arterial occlusion, lesions of the nervous system and pa-
thology of locomotive system.
43.Clinical picture of limb arterial occlusion, lesions of the nervous system and
pathology of locomotive system.
44.Methods of diagnosis of limb arterial occlusion, lesions of the nervous system
and pathology of locomotive system.
45.Principles of treatment of limb arterial occlusion, lesions of the nervous system
and pathology of locomotive system.
46.Features of surgical interventions for the different types of limb arterial occlu-
sion, lesions of the nervous system and pathology of locomotive system.
A student must be able to:
33.Collect and estimate the complaints of patient with limb arterial occlusion, le-
sions of the nervous system and pathology of locomotive system, information
of anamnesis, to conduct physical research and correctly interpret the results
obtained.
34.Define the rational volume of laboratory and instrumental methods of research.
35.Correctly interpret the results of clinical analyses, instrumental examinations
36.Define indications for operation and other methods of treatment of patients
with limb arterial occlusion, lesions of the nervous system and pathology of lo-
comotive system.
37.Perform pre-operative preparation of patients with limb arterial occlusion, le-
sions of the nervous system and pathology of locomotive system.
38.Conduct post-operative care.

Terminology.
Term Definition
Is the disease of vessels of neurohormonal genesis which
Obliterating endart-
begins from the affection of peripheral vessels, mainly ar-
eritis
teries, and results in the obliteration of their lumen.
Atherosclerosis oblit- Is a widespread disease, with a specific lesion of arteries
erans of the lower ex- of elastic and muscular types as a focal growth of con-
tremities necting tissue with a lipid infiltration of the tunica intima.
The sudden block of blood flow in a major artery caused
Acute arterial occlu-
by an embolism or thrombosis, which result from other
sion
diseases.

Content:
Endarteritis obliterans is a segmental, inflammatory, obliterating disease af-
fecting primarily small and medium-sized arteries and develops predominantly in
young adults, usually men, who smoke.
Etiology and pathogenesis
In etiopathogenesis of endarteritis obliterans is considerably played by the
consecutive or simultaneous influence of such factors like temperature, tobacco in-
toxication, mechanical traumas etc. The damage to the vascular walls results in auto-
immune processes, which considerably intensifies the proliferative processes of the
vascular intima. First of all, the vessels are affected. It results in intracapillary mal-
nutrition with a hypersensitivity of the vascular walls, which provokes occurrence of
pain. The latter causes the spasm of regional vessels, sluggish blood flow, hyperco-
agulability and finally necrosis of tissues.
Pathology
The morphological basis of endarteritis obliterans is the intimal hyperplasia
predominantly of medium-sized or small vessels of the extremities, which leads to
stenosis and obliteration of arterial lumen: latter causes hypoxia of the extremities,
which leads to necrosis.
Classification
In the course of endarteritis obliterans such stages are distinguished (according to A.
Shabanov, 1983):
1 - ischemic;
2 - of trophic changes;
3 - ulcerative-necrotic;
4 - Gangrenous.

Forms of clinical course and complications


The course of the disease and expressiveness of signs depends on its stage.
In ischemic stage fatigability of legs during walking, coldness, paresthesias
and muscular cramps are observed. The main signs are discoloration and tempera-
ture changes of the skin of feet, lability of vascular response, impaired pulsation and
blanching of capillaroscopy pattern. Angiogram is without pathological changes.
The stage of trophic changes is characterized by extremely fatigability and
coldness of legs, expressed paresthesias and appearance of pain when walking as
"intermittent claudication". Intermittent claudication (Latin: claudicatio intermittens)
is a clinical diagnosis given for muscle pain (ache, cramp, numbness or sense of fa-
tigue), classically in the calf muscle, which occurs during exercise and is relieved by
a short period of rest. Objective examination reveals blanching or cyanosis of feet,
thickened and deformed nails, skin atrophy and coldness, impaired or absent of arte-
rial pulsation on foot. On capillaroscopy in these patients it is possible to note patho-
logical changes of the pattern: decreasing of capillaries amount and spasm of arterial
part of a capillary loop. On arteriogram an occlusion of arteries of the leg is ob-
served.
For ulcerative-necrotic stage is characteristic the constant resting pain, which
amplifies when the patient is supine. The ulcers appear on toes and foot. The walk-
ing is limited, the sleep is disturbed. Rather frequently, the phenomena of accompa-
nying thrombophlebitis and lymphangitis join the ulcerative process. There are ex-
pressed atrophy of muscles, blanching of skin (cyanosis in the region of ulcers),
coldness of skin and absence of arterial pulsation on foot. The trophic changes exist
not only on skin, but also in bones (spotty osteoporosis). On arteriogram it is possi-
ble to note the occlusion of two or even of three arteries of the leg. On capillarosco-
py the blanched or cyanotic background, lack of capillaries and their deformity are
revealed.
The gangrenous stage is characterized, first of all, by the signs of toxemia
with its influence on mentality, cardiovascular system, kidneys and liver. The pa-
tients in this stage are sleepless either day, or night. The gangrene can be either wet
or dry. Thus the necrosis of soft tissues, and frequently of bones, edema of the leg,
ascending lymphangitis, thrombophlebitis and inguinal lymphadenitis takes place.
The temperature of skin in dry gangrene is usually decreased, in wet - can be nor-
mal. On angiogram observed the occlusion of arteries of foot, leg, and quite often
the femoral artery.
Among the complications of endarteritis obliterans most frequently: arterial
thrombosis and gangrene of extremity. The dry gangrene often develops in the re-
gions with lack or absence of muscles and subcutaneous fat. The demarcation line in
these cases is well defined with a slight expressed zone of inflammation on its edge.
The wet gangrene develops when the chronic ischemia is complicated by vascular
thrombosis. It can develop not only in arteries of foot, but also in arteries of the legs.
The extremity in such patients is swollen, with tense skin. The demarcation line in
such situations, as a rule, is expressed rather weakly. The leading sign of wet gan-
grene are the symptoms of general toxemia.
The diagnostic program
1. Complaints, anamnesis.
2.Examination of extremities.
3. Palpation, auscultation of vessels.
4. Rheovasography.
5. Dopplerography of vessels.
6. Aorto-arteriography.
7. Biochemical blood analysis.
8. Coagulogram.
Tactics and choice of treatment
Conservative therapy. The goal of the treatment of obliterating endarteritis
may be changed depending on the stage of disease; nevertheless the main purpose
should be the renewal or improvement of capillary circulation. This problem could
be solved by: 1) improving of blood rheology; 2) improving of peripheral macrohe-
modynamics, particularly by reducing of the arterio-venous dumping of blood (thus
the application of spasmolytics is categorically contraindicated); 3) normalization of
interaction between endothelium and formed elements of blood.
The most effective is the synthetic prostaglandin El - Vasaprostan. Its prompt thera-
peutic efficiency is caused by inhibition of free radicals of oxygen and lysosome en-
zymes of activated leukocytes in ischemic conditions, block of thrombocyte activity,
elevation of erythrocyte lability, decreasing of their aggregation and diminishing of
blood viscosity.
Among the other vasoactive drugs, pentoxyphillin is often of great value. Be-
sides the expressed block of thrombocytes, it stimulates decreasing of aggregation of
erythrocytes, elevating of their synthetic properties, and also inhibits anti-
inflammatory effect of various cytokines.
Desensitizing agents (dimedrol, pipolphen, diazolin, suprastin, tavegil) block
the influence of histamine and result in vasodilating effect, thus reducing the perme-
ability and fragility of the vascular wall.
The drugs, which influence on the blood rheology, are dextrans (rheopolyglu-
cin, polyglucin). They diminish blood viscosity, prevent aggregation of formed ele-
ments of blood, reduce peripheral resistance, and increase fibrinolytic activity of
blood. Anticoagulants (of direct and indirect action) and antiaggregants (aspirin) al-
so improve the blood rheology.
The stimulators of metabolism are also used in the therapy of arterial ische-
mia. These drugs are nicotinic acid, solcoseril, actovegin.
In the complex of treatment physiotherapeutic agents and oxygenotherapy are
also included.
The special attention thus should be paid to the blockade of ganglions, nerv-
ous trunks and plexuses. Taking into account a stage system of the regulation of
vascular tonus, such blockade can be performed on different levels of vegetative
nervous system: blockade of thoracic sympathetic ganglions and ganglions of a
lumbar part of sympathetic trunk. Particular application has also paranephral, epi-
dural and paraarterial blockade.
If conservative therapy is failed, the surgical treatment is necessary. A sympa-
thectomy is considered to be the most effective operation in the patients with endart-
eritis obliterans.
The ganglion sympathectomy solves such problems: completely removes vas-
omotor spasm, liquidates or reasonably relieves the pain. Operation is especially ef-
fective in onset of the disease. In later stages the sympathectomy loses its anesthetiz-
ing action. Operation is carried out on the background of conservative treatment,
which should last and in postoperative period. A lumbar sympathectomy requires
removing 1-3 sympathetic ganglions.
Contraindications for sympathectomy are: 1) atony of capillaries; 2) lack of
the effect at blocking ganglions; 3) complete obstruction of popliteal artery; 4) dura-
tion of reactive hyperemia exceeds 3 min; 5) anatomic and functional failure of col-
laterals.
Amputation should be deferred until conservative treatment has been given a
thorough trial. In some instances it may be best to allow digits that are entirely or
partially gangrenous or to slough spontaneously without amputation. It is almost
useless to delay amputation of the leg when gangrene extends well into the foot, and
it is inadvisable to delay amputation if pain is severe and cannot be controlled or if
severe infection or toxicity supervenes. When amputation of a leg is necessary, an
attempt should be always be made to carry out the procedure below the knee. Only
in rare instances it is necessary to do an amputation above the knee in patients with
endarteritis obliterans.
Atherosclerosis obliterans of the inferior extremities
The inner lining of arterial blood vessels is normally smooth, allowing blood
to flow easily. In lower extremity arterial disease, the lining becomes damaged,
leading to build up of cholesterol and other lipids, causing the arterial wall inner lin-
ing to become rough and thickened. This accumulation is called atherosclerosis, or
“hardening of the arteries.” As the atherosclerotic process of the lower extremity ar-
teries increases, the arteries become narrowed or blocked, causing blood flow to de-
crease. This can lead to discomfort, cramps, or pain in the hips, thighs or calves with
walking. This is called claudication.
Claudication typically occurs during physical activity such as walking and is
promptly relieved by a brief resting period (2-5 minutes).Normally; blood flow can
increase up to ten-fold to meet the increased need for additional oxygen in exercis-
ing muscles. However, when the leg arteries are blocked, blood flow cannot increase
in response to exercise and pain develops. Claudication pain always involves the
same muscle groups, usually the calves, and does not change from day to day. The
vascular surgeon relates the onset of claudication pain to a particular walking dis-
tance in terms of street blocks (e.g. “2-block claudication”) or distance travelled be-
fore the symptom occurs. This helps to provide a standard of measuring if there has
been any change before and after therapy has been initiated.
As atherosclerosis progresses and blockage becomes more severe, pain may
occur in the feet even when at rest. This pain, known as rest pain, occurs because the
arteries of the leg can no longer deliver adequate blood flow to the feet, even at rest.
Rest pain generally worsens when the legs are elevated, such as when lying in bed at
night. Relief from this pain may occur only when the feet are dangled. Gangrene or
“death of tissue” may occur when nutrition needed for normal growth and repair can
no longer be provided because of extensive arterial narrowing (stenosis) or complete
blockage (occlusion) of lower extremity arteries.
Currently, atherosclerosis affects up to 10% of the Western population 65
years or older. When claudication is used as an indicator of lower extremity arterial
disease, estimates are that 2% of the population aged 40 to 60 years and 6% older
than 70 years of age are affected. With the elderly population expected to increase to
22% by the year 2040, lower extremity arterial disease will be even more common.

Risk Factors
The risk factors for atherosclerosis affecting the lower extremities are the
same risk factors associated with coronary artery disease or cerebrovascular disease.
These risk factors include:
• Smoking
• High blood pressure (hypertension)
• High levels of blood cholesterol or triglycerides (hypercholesterolemia, hy-
perlipidemia)
• Obesity
• Sedentary lifestyle
• Diabetes
• Family history of heart disease or arterial disease
Smoking is the most influential of all the risk factors. Although the mecha-
nism by which smoking causes or worsens atherosclerosis is unclear, it is known
that the degree of damage to the arterial wall lining is directly related to the amount
of tobacco used. Quitting smoking is essential in the battle against atherosclerosis
progression.
Classification (according to A. Fountain, 1954)
I stage - complete compensation (coldness, fatigue, paresthesias);
II stage -functional circulatory insufficiency (a leading sign - intermittent claudica-
tion);
III stage - ischemia of extremity at rest (a leading sign - resting night pain);
IV stage - considerably expressed destruction of tissues of the distal parts of extrem-
ity (ulcers, necrosis, and gangrene).

Signs and Symptoms


Essential to the management of a patient with leg pain is a comprehensive
lower extremity examination including palpation of peripheral pulses. Signs and
symptoms that advanced lower extremity arterial disease is causing the leg pain in-
clude:
• Discoloration of the affected leg or foot when dangling (from pale to bluish-
red)
• Decreased hair growth on the legs and feet
• Diminished or absent pulses in the affected leg or foot
• Temperature difference in affected leg or foot (cooler than other extremity)
• Change in sensation (numbness, tingling, cramping, pain)
• Presence of non-healing wound on affected lower extremity
• Shrinking of calf muscles
• Presence of thickened toenails
• Development of gangrene
• Other diseases that must be considered

Other disorders that can cause leg pain are:


 Arthritis - Arthritic pain is variable from day to day and may be aggra-
vated by certain weather patterns or physical movements. Unlike clau-
dication, rest does not provide relief.
 Varicose veins - The pain associated with varicose veins is a dull ach-
ing, typically occurring at the end of the day or after prolonged periods
of standing. Pain from venous disease is not exacerbated by exercise.
 Venous thrombosis - Swelling and leg pain associated with venous
thrombosis typically occurs with walking and is relieved by extremity
elevation, unlike arterial disease.
 Spinal stenosis - narrowing of the spinal canal due to a ruptured disk or
arthritis of the back causes leg pain when standing and is not relieved
by brief resting periods. Relief of pain often occurs by leaning forward
against a stationary object (e.g. a tree) or sitting.
 Diabetic Neuropathy - Pain associated with this complication of diabe-
tes is usually present in both legs. It often presents with numbness and
diminished sensation in the lower extremities.
Diagnostic Testing
If you are suspected of having lower extremity arterial disease or your symp-
toms are worsening, the vascular surgeon will ask you certain questions, examine
you, and order either non-invasive or invasive diagnostic tests.
Non-invasive testing: These tests are performed in the clinic setting or in a
vascular laboratory, most often on an outpatient basis. They are virtually painless
methods of examining the blood flow to the extremities, with essentially no side ef-
fects or risks.
Pulse examination: This is the primary assessment of the circulation. Normal
findings by an experienced examiner make the likelihood of lower extremity arterial
disease remote.
Arterial blood pressures: Using an ultrasound stethoscope (Doppler), the
blood pressure in your arms and legs is measured, and the pressures are compared.
This test gives the vascular surgeon a generalized assessment of the severity of the
lower extremity arterial disease.
Duplex scanning: This test is useful for detecting blockages in an artery and
measuring the size of the artery. It may also be used to measure the size of a vein
that may be used as a bypass, a means of re-routing blood flow around a blocked or
occluded area.
Magnetic resonance angiography (MRA): This test is also useful for imag-
ing extremity vessels.

Invasive Testing: Tests in this category involve the injection of a contrast dye direct-
ly into your arteries under X-ray guidance.
Angiogram: This test is the most helpful imaging study used to direct treat-
ment of symptomatic lower extremity arterial disease. Dye is injected directly into
the artery and special x-rays are taken to reveal the exact location of the arterial
blockage. An angiogram is only necessary when interventional or surgical treatment
is being considered.

Medical Management
The treatment of leg pain due to claudication is primarily medical manage-
ment of the symptoms, with surgery reserved for severe exercise induced pain which
negatively impacts an individual’s lifestyle. While atherosclerosis cannot be totally
cured or prevented, the progression of the disease can be controlled through risk fac-
tor modification. This involves changing one’s lifestyle to include healthy habits.
Smoking: Tobacco in any form should be avoided. Continued cigarette smok-
ing is the most consistent adverse risk factor associated with progression of lower
extremity arterial disease in patients experiencing claudication. The nicotine found
in tobacco products causes the blood vessels to constrict, further narrowing them,
preventing blood from reaching the intended targets (e.g. body organs, extremities),
increasing the risk of atherosclerosis. In addition, smoking also decreases the
amount of oxygen in the blood and can cause the blood to clot more readily.
High blood pressure: Untreated or uncontrolled high blood pressure (hyper-
tension) causes the heart to work harder and creates additional stress on the arteries.
Blood pressure should be monitored regularly because often hypertension occurs
without symptoms. Take your medications as prescribed as long as your physician
instructs you to, even if it normalizes and you “feel good.”
Diet: The risk of atherosclerosis can also be reduced by carefully monitoring
cholesterol (found in organ and red meats, dairy products and egg yolk) and saturat-
ed fats (found in animal fat and plant oils) in the diet. Polyunsaturated fats (found in
corn, safflower and olive oils) are the fats to incorporate in your diet. In addition, a
salt-restricted diet will help control high blood pressure and fluid retention associat-
ed with weight gain. If you are overweight, a general weight reduction diet is advan-
tageous. Checkups with the doctor should include the monitoring of serum choles-
terol. If it remains high (>200) despite the above diet then medication to reduce the
cholesterol should be considered.
Exercise: Exercise plays a vital role in the treatment of atherosclerosis in pa-
tients with claudication. Patients with intermittent claudication often voluntarily re-
duce their daily walking because of pain and the fear of causing further tissue dam-
age. This leads to an increasingly sedentary lifestyle that complicates the picture
even more. Increase your walking distance gradually, stopping to rest when the leg
pain develops. When it disappears, begin walking again. Measurable improvement
ranged from 80-234% in controlled studies that incorporated a regular walking pro-
gram into the daily routine. A regular walking program of 45-60 minutes/day is rec-
ommended.
Diabetes: Due to the important role that diabetes mellitus plays in the earlier
onset and accelerated rate of atherosclerosis progression, it is crucial to follow the
advice of the health care team regarding diet, medications, and treatment. Early
treatment and meticulous management is paramount to controlling the effects of di-
abetes on arteries.
Foot care: When blood flow to the lower extremities is decreased, delayed
healing of sores, serious infections, and gangrene (tissue death) of the feet or toes
can occur after seemingly minor injuries (e.g. hang-nail, superficial laceration). Care
must be given to avoid any situation that might cause injury to the foot. Inspect your
feet daily. Immediately report to your physician the detection of any foot injuries or
sores.
Pharmacologic therapy: In addition to others prescribed by your physician,
the following medications may be added to your regimen:
• Anti-Platelet Agents These drugs decrease the overall risk of heart attacks
(angina, myocardial infarction) or strokes (cerebrovascular accidents or tran-
sient ischemic attacks) in persons with atherosclerosis. They also may im-
prove walking distance by enhancing blood flow and overall circulation. Two
examples of anti-platelet agents are:
i. Aspirin - One aspirin tablet (81-325 mg) daily is the main antiplatelet
agent utilized.
ii. Clopidogrel bisulfate (Plavix) – Plavix is a newer anti-platelet agent
that appears to be gaining in popularity. Studies are in the process of
determining which patient populations would most benefit from using
this agent.
• Anticoagulation Agents – These drugs inhibit clot formation. Examples of
these medications include:
i. Warfarin (Coumadin) – The amount of this pill that is prescribed is
based on specific blood test results.
ii. Lovenox – This medication, given as an injection, is used to achieve
adequate anticoagulation, often while waiting for the warfarin dose to
reach a therapeutic level.
• Other Agents – These medications improve the circulation in the lower ex-
tremity. Treatment for at least one month is required to produce noticeable re-
sults. Examples of these types of medications include:
i. Trental
ii. Pletal
Surgical Management
In cases where diligent medical therapy is not sufficient in resolving the
symptoms or the symptoms progress at a very fast rate and have become lifestyle
restricting, surgical therapy should be considered.
The first in providing surgical care is to determine exact location of the arteri-
al blockage in the leg. Although this can be performed with an ultrasound, the most
accurate test is called an arteriogram. An arteriogram is an outpatient procedure
where a small needle and catheter are inserted into your artery. It is similar to having
an IV started. A biologically safe dye is then injected into the arteries and pictures
are taken of the blood flowing down each leg. This provides a roadmap of all of the
normal and abnormal segments of arteries and allows the surgeon to identify the ar-
eas of concern.
Once the areas of arterial blockage is determined, two treatments options are
possible; angioplasty or open surgery. An angioplasty is when small balloon is used
to dilate a narrowed segment of an artery. Typically, the balloon is inserted into the
artery and placed exactly at the area of arterial narrowing. The balloon is then inflat-
ed; smashing the plaque that was inhibiting the blood flow. This procedure can be
performed at the same time as the arteriogram and usually requires less than a 24-
hour hospital stay.
If there is too much blockage in the arteries to treat with balloon angioplasty,
an open bypass operation must be undertaken. A bypass operation involves finding a
suitable blood vessel above and below the area of blockage and routing blood flow
between the two vessels with the aid of a bridge (graft) carry the blood between the
two vessels. The graft may be constructed from a vein in the leg or a synthetic mate-
rial. The procedure is rather tedious and usually requires 2 to 5 hours of surgery. A
hospital stay of 3 to 7 days after surgery can be expected. A blood transfusion is
needed in less than 10% of cases.
Angioplasty and open surgical repair are very safe procedures with excellent
results. Factors that may diminish the success of each procedure include:
• the amount of arterial blockage present,
• the overall health status of the patient and,
• Adherence to risk factor control after the intervention.
The most significant risk factor that contributes to early failure after interven-
tion is smoking; therefore lifelong cessation is paramount.

Conclusion
Patients with lower extremity arterial disease may present with symptoms
ranging from minor claudication to extensive gangrene and threatened limb loss
(amputation). While studies of large groups of patients with claudication reveal that
amputation is uncommon, it continues to be a significant fear. How rapidly arterial
disease progresses to limb loss largely depends on the number and severity of risk
factors (e.g. smoking, hypertension, obesity, diabetes). Timely and regular medical
evaluations and patient compliance with smoking cessation, diet and blood pressure
control, daily exercise, and adherence to prescribed treatment modalities can mark-
edly improve the claudication symptoms and ultimate outcome associated with low-
er extremity arterial disease.
Acute arterial occlusion can be the result of emboli from a distant source, acute
thrombosis of a previously patent artery, or direct trauma to an artery
Causes of Acute Arterial Occlusion
Embolus Thrombosis Trauma
Cardiac source Vascular grafts Blunt
Atrial fibrillation Atherosclerosis Penetrating
Myocardial infarction Thrombosis of aneurysm Iatrogenic
Endocarditis Entrapment syndrome
Valvular disease Hypercoagulable state
Atrial myxoma Low flow state
Prosthetic valves
Arterial source
Aneurysm
Atherosclerotic plaque
Paradoxical embolus

Arterial emboli Eighty percent of arterial emboli originate in the heart and
travel to the extremities; the lower extremities are affected much more frequently than
the upper extremities. The majority of these emboli occur in patients with significant
underlying cardiac disease; the severity of the patient's underlying cardiac condition
may increase the risk of surgery, and limit the options available for restoring blood
flow to the ischemic extremity. Potential sources of emboli from the heart include
ventricular thrombus formation following myocardial infarction, and atrial thrombus
in patients with atrial fibrillation. Up to 75 percent of patients with emboli to the low-
er extremities have a history of recent myocardial infarction or atrial fibrillation.
Arterial to arterial embolization of thrombus or plaque originating from aneu-
rysms or atherosclerotic lesions is another well described occurrence and accounts for
20 percent of peripheral emboli. Emboli typically lodge where there is an acute nar-
rowing of the artery, such as an atherosclerotic plaque or a point where the vessel
branches; the common femoral, common iliac, and popliteal artery bifurcations are
the most frequent locations. In a large series of arterial embolism, for example, the
following frequencies were noted:
• Femoral — 28 percent
• Arm — 20 percent
• Aortoiliac — 18 percent
• Popliteal — 17 percent
• Visceral and other — 9 percent each
In comparison to clot emboli, atheroemboli are less likely to produce symptoms
of acute arterial occlusion. Atheroemboli are typically nondistensible and irregularly
shaped; as a result, they tend to produce incomplete occlusion with secondary is-
chemic atrophy.
Arterial thrombosis — Thrombosis of a previously patent but stenotic artery is a well
known complication of atherosclerosis. Occlusion of atherosclerotic vessels may oc-
cur by two mechanisms:
• Progressive atherosclerotic narrowing of the artery, with resultant low flow, stasis,
and eventual thrombosis
• Intraplaque hemorrhage and local hypercoagulability
The ischemia resulting from arterial thrombosis in the face of underlying ather-
osclerosis is usually less severe than that following an acute embolus. This difference
is primarily due to the collateral circulation that develops over time in patients with
atherosclerosis and chronically narrowed vessels. Collaterals are frequently so exten-
sive that patients notice no change or only a mild increase in their symptoms of
chronic ischemia when a major atherosclerotic vessel becomes occluded.
Arteritides, ergotism, and hypercoagulable states can also result in arterial
thrombosis, occlusion, and acute extremity ischemia. While these conditions most
frequently affect the venous circulation, certain hypercoagulable states favor arterial
thrombosis (eg, antiphospholipid antibodies and hyperhomocysteinemia).
Arterial trauma - Acute arterial occlusion complicating vascular or cardiac
diagnostic and interventional procedures has become a more frequent cause of acute
extremity ischemia. The incidence of arterial complications following interventional
cardiac catheterization (including hematomas, arteriovenous fistulae, pseudoaneu-
rysms, arterial occlusion, and cholesterol emboli) has been reported to range from 1.5
to 9 percent. Although acute arterial occlusion occurs in less than one percent of in-
terventional catheterization procedures, this complication demands immediate surgi-
cal consultation. Intimal flaps and dissections are frequently the cause of the occlu-
sion, and operative repair of the vessel is required. Thromboemboli can also develop
at the sheath site or catheter tip, with embolization occurring during sheath removal.
Clinical evaluation A thorough history and physical examination is the first
step in the evaluation of the patient with acute extremity ischemia.
Pain associated with acute ischemia is usually located distally in the extremity,
gradually increases in severity, and progresses proximally as the length of ischemia
increases. Later, the pain may decrease in severity due to progressive ischemic senso-
ry loss.
It is essential to determine if the patient had symptoms of chronic ischemia be-
fore the acute event occurred. Patients with an embolus usually have no preexisting
ischemic symptoms, and can frequently pinpoint the exact time that symptoms began.
Thus, the sudden and dramatic development of ischemic symptoms in a previously
asymptomatic patient is most consistent with an embolus, while gradually increasing
symptoms in a patient with chronic ischemia is indicative of thrombosis.
Pulse. The quality and character of the peripheral pulses must be evaluated. If
pulses are not palpable, a hand held Doppler should be used. It is rare to have limb
threatening ischemia without a major pulse deficit. The status of the pulses in the con-
tralateral extremity is also important. The presence of a pulse deficit in an asympto-
matic contralateral extremity is an indication of underlying chronic arterial occlusive
disease and suggests that acute thrombosis of an already diseased vessel is the most
likely cause of the acute occlusion. By contrast, the presence of normal pulses in the
contralateral extremity suggests the absence of chronic occlusive disease, and in-
creases the likelihood that an embolus is the etiology of acute occlusion.
The skin of both the normal and affected extremity should be examined for
temperature, color, and capillary refill. The skin of the ischemic extremity is typically
cool and pale with delayed capillary filling. The level of arterial obstruction is usually
one joint above the line of demarcation between the normal and ischemic tissue. Both
extremities should also be examined for signs of chronic ischemia such as atrophy of
the skin, hair loss, and thickened nails.
Neurologic examination A careful neurologic examination must be performed.
Subjective sensory deficits such as numbness or paresthesias are signs of early nerve
dysfunction secondary to ischemia. Major loss of sensory or motor function is indica-
tive of advanced ischemia. The anterior compartment of the lower leg is most sensi-
tive to ischemia, and sensory deficits over the dorsum of the foot are often the earliest
neurologic sign of vascular insufficiency.

CLASSIFICATION OF ACUTE EXTREMITY ISCHEMIA


The Society of Vascular Surgeons (SVS) and International Society of Cardio-
vascular Surgeon (ISCVS) have developed a standardized method for categorizing
and reporting acute limb ischemia based upon clinical examination. Extremities are
placed in one of three categories based upon these clinical findings to help judge the
severity of ischemia:
SVS/ISCVC Classification of Acute Extremity Ischemia
Viable Threatened Nonviable
Pain Mild Severe Variable
Capillary refill Intact Delayed Absent
Motor deficit None Partial Complete
Sensory deficit None Partial Complete
Arterial Doppler Audible Inaudible Inaudible
Venous Doppler Audible Audible Inaudible
Treatment Urgent work-up Emergency surgery Amputation

• Viable limbs are under no immediate threat of tissue loss.


• Threatened limbs have reversible ischemia, but immediate relief of the arterial oc-
clusion is required if the extremity is to be salvaged and major amputation avoided.
• Nonviable extremities have irreversible ischemia and will require major amputation
regardless of the therapy that is instituted. Revascularization of the nonviable extrem-
ity may be required to allow healing of the amputation or to permit amputation at a
lower level.According to the clinical course of disease (V. Savelyev and all. (1973)).
IE- (ischemia at exertion) Lack of signs of ischemia at rest, (occurrence them only at
exertion).

Level of ischemia of extremity Main feature

IT (ischemia of tension) The absence of ischemia feature at rest


(They appear only at exercising)

IА The feeling of numb, cold, paresthesia

IB Pain

II А Parethis

II B Plegium
III А Subfascial swelling of muscles

III B Partial contraction

III C Total contraction

Diagnostic tests. Arteriography is the diagnostic procedure that provides the


most useful information in the setting of acute arterial occlusion. In addition to
demonstrating detailed arterial anatomy, arteriography can usually distinguish be-
tween thrombosis and embolism.
• An embolus will often demonstrate a sharp cut off with a rounded reverse meniscus
sign. The embolus may also be visible as an intraluminal filling defect if the vessel is
not completely occluded. Other findings which are most consistent with an embolus
include the presence of otherwise normal vessels, the absence of collateral circula-
tion, and the presence of multiple filling defects.
• Arterial thrombosis is usually visualized as a sharp or tapered, but not rounded, cut
off on arteriography. Diffuse atherosclerosis with well developed collateral circula-
tion is generally present.
Although all patients with acute extremity ischemia would benefit from the in-
formation obtained from arteriography, it is not possible to perform this test in every
case. Patients with a threatened extremity, for example, cannot tolerate the several
hour delays in revascularization while arteriography is being performed. Thus, pa-
tients with a viable extremity should generally undergo diagnostic arteriography,
while those with a threatened extremity should have immediate surgical revasculari-
zation with intraoperative arteriography as necessary.
Treatment. It is difficult to compare published results of the treatment of acute
extremity ischemia because of different methods used to describe the severity of is-
chemia and differences in the duration of ischemia. However, it is clear that acute ex-
tremity ischemia is associated with high hospital morbidity and mortality and high
rates of limb loss. Limb loss rates as high as 30 percent and hospital mortality as high
as 20 percent have been quoted in surgical series. Cardiopulmonary complications
account for the majority of the deaths, underscoring the severity of the baseline medi-
cal condition of these patients.
The best defence against limb loss is prompt initiation of therapy. Thus, once
the diagnosis of acute arterial occlusion has been made by history and physical exam-
ination, the Fifth ACCP Consensus Conference on Antithrombotic Therapy recom-
mends that the patient should immediately receive 10,000 units of intravenous hepa-
rin followed by a continuous heparin infusion. Anticoagulation will prevent further
propagation of thrombus, and inhibit thrombosis distally in the arterial and venous
systems due to low flow and stasis. Time is crucial; the decision to administer heparin
is based upon the clinical evaluation and should not be delayed while waiting for di-
agnostic procedures to be performed.
Following the initiation of heparin, treatment then varies depending upon the
viability of the limb. Options include surgery and thrombolytic therapy.
Patients with threatened extremities and patients with a threatened extremity
should undergo emergent surgical revascularization. The majority of these patients
have had an embolic event, and irreversible changes can occur within as little as four
to six hours of profound ischemia. While pharmacologic thrombolysis may success-
fully dissolve the embolus (see below), the time required is usually too long to allow
this to be an acceptable alternative to surgery.
At surgery, an embolus will be found in the majority of patients. Embolectomy
is usually all that is required to relieve the occlusion and provide adequate blood flow
to the extremity. Most surgeons perform an intraoperative completion arteriogram af-
ter the embolectomy to evaluate the adequacy of distal blood flow. Intraoperative
thrombolytic therapy may also be used if there are small emboli in the distal runoff
vessels. Depending upon the length and severity of the ischemia, a fasciotomy may be
required to prevent the development of a compartment syndrome.
Patients with viable extremities Intra-arterial thrombolysis has recently become
an alternative to surgical therapy in patients with ischemic but viable extremities.
Technical success rates in achieving thrombus dissolution approach 70 percent with
this technique, and limb salvage rates similar to those of surgical series have been re-
ported. However, the usefulness of thrombolytic therapy is limited by the severity of
the ischemia, and the length of time required achieving dissolution of the thrombus.
Recently, several randomized, prospective clinical trials have been published
comparing surgical revascularization to thrombolytic therapy in the treatment of acute
ischemic but viable lower extremities.
Thus, patients found to have an ischemic but viable extremity on clinical exam-
ination should undergo urgent arteriography in order to plan surgical or medical re-
vascularization. There are several findings on arteriography which are used to deter-
mine if thrombolytic therapy, PTA, or surgical revascularization is the most appropri-
ate treatment. These include:
• The presumed etiology (embolus versus thrombus)
• The location and length of the lesion
• The duration of symptoms
• The availability of autologous vein for bypass grafting
• The suitability of the patient for surgery
As an example, a proximal embolus at the bifurcation of the common femoral
artery is an ideal lesion for embolectomy. On the other hand, embolus to a distal ves-
sel (eg, to the tibial artery) may be best treated with a thrombolytic agent. The major
use of PTA is in the treatment of an underlying lesion after the clot has been lysed
with thrombolytic therapy.
Patients with nonviable extremities Patients with nonviable extremities should
undergo prompt amputation. (See "Treatment of limb threatening ischemia"). Arteri-
ography is usually not necessary, since the level of amputation is determined by clini-
cal findings and by the viability of tissues at the time of surgery. Every effort should
be made to preserve as many joints as possible, in order to decrease the work of am-
bulating with prosthesis and to improve the chances for successful rehabilitation. De-
lays in amputation of a nonviable extremity can result in infection, myoglobinuria,
acute renal failure, and hyperkalemia.

Basic literatures:
67.Oxford Textbook of Surgery (3-Volume Set) 2nd edition (January 15, 2000):
by Peter J. Morris (Editor), William C. Wood (Editor) By Oxford Press
68.Sabiston Textbook of Surgery 17th edition by Courtney M. Townsend Jr.,
Kenneth L. Mattox, B. Mark, MD Evers, Kenneth L., MD Mattox, Courtney
Townsend, Daniel Beauchamp, B. Mark Evers, Kenneth Mattox W.B. Saun-
ders Company (June, 2004)
69.Schwartz´s Principles of Surgery 8th Edition F. Charles Brunicardi. Copyright
©2007 the McGraw-Hill Companies.
70.Hospital surgery/ Edited by L. Kovalchuk et al. - Ternopil: Ukrmedknyha,
2004.

Additional literatures:
1. Yeager, RA, Moneta, GL, Taylor, LM, et al. Surgical management of se-
vere lower extremity ischemia. J Vasc Surg 1992; 15:385.
2. Quinones-Baldrich, WJ. Acute arterial and graft occlusion. In: Vascular
Surgery. A Comprehensive Review, Moore, WS (Ed), WB Saunders, Phila-
delphia, 1993, p. 648.
3. Abbott, WM, Randolph, DM, McCabe, CC, et al. Arterial embolism: A 44
year perspective. Am J Surg 1982; 143:460.
4. Nasser, TK, Mohler, ER, Wilensky, RL, et al. Peripheral vascular compli-
cations following coronary interventional procedures. Clin Cardiol 1995;
18:609.
5. Messina, LM, Brothers, TE, Wakefield, TW, et al. Clinical characteristics
and surgical management of vascular complications in patients undergoing
cardiac catheterization versus diagnostic procedures. J Vasc Surg 1991;
13:593.

Tests for initial level of knowledge, keys for tests:


1. All of the following complications may commonly follow lower extremity angi-
ography EXCEPT:
A. Renal failure
B. Dehydration
C. Arterial occlusion
D. Pseudoaneurysm formation
E. Intracerebral hemorrhage

2. True statements concerning acute arterial occlusion include all of the following
EXCEPT:
A. It is usually caused by emboli that originate from a cardiac source
B. It is in result in severe pain
C. It requires immediate heparinization
D. A fasciotomy is always required following restoration of blood flow
E. Balloon catheter embolectomy is the most commonly used surgical procedure

3. Invasive tests for assessment of peripheral arterial disease includes all of the fol-
lowing EXCEPT:
A Digital intravenous substraction angiography
B Arterial digital intravenous substraction angiography
C Duplex examination
D Convertional angiography
E Translumbal aortography
4. Noninvasive tests for assessment of peripheral arterial disease includes all of the
following EXCEPT:
A. Pulse volume recording
B. Segmental arterial blood pressure
C. Duplex examination
D. Translumbal aortography

5. Management of acute arterial occlusion includes all of the following EXCEPT:


A. Heparin therapy
B. Hydration therapy
C. Thrombolytic therapy
D. Haemostatic therapy

6. Patients with acute arterial occlusion should undergo surgery within how many
hours of onset?
A. 8-10 hours
B. 6-8 hours
C.10-12 hours
D. 12-24 hours

7. Patients with acute arterial occlusion should undergo thrombolytic therapy within
how many hours of onset?
A. 24-36 hours
B . 36-48 hours
C .48-72 hours
D. 12-24 hours

8. Virchow’s triad is:


A. Damage of vascular wall, change of a blood rheology and disturbance of circula-
tion
B. Necrotic processes, hemorrhage and gangrene
C. Atherosclerotic cardiosclerosis, acute myocardial infarction, rheumatic mitral de-
fects
D. None of the above

9. In 90 % of cases arterial thrombosis is caused by:


A. Acute myocardial infarction
B. Atherosclerosis
C. Rheumatic mitral defects
D. Varicosity
E. None of the above

10. According to the clinical course of disease (V.Savelyev and all., 1973), I A stage
is:
A. Sensation of numbness, coolness, paresthesia
B. Pain
C. Paresis
D. Plegia
E. Subfascial muscular edema

Keys for tests


1 2 3 4 5 6 7 8 9 10
E C C D D B D A B A

Tests for final level of knowledge, keys for tests:


1. According to the clinical course of disease (V.Savelyev and all., 1973), I B stage
is:
A. Sensation of numbness, coolness, paresthesia
B. Pain
C. Paresis
D. Plegia
E. Subfascial muscular edema

2. According to the clinical course of disease (V.Savelyev and all., 1973), II A stage
is:
A. Sensation of numbness, coolness, paresthesia
B. Pain
C. Paresis
D. Plegia
E. Subfascial muscular edema

3. According to the clinical course of disease (V.Savelyev and all., 1973), II B stage
is:
A. Sensation of numbness, coolness, paresthesia
B. Pain
C. Paresis
D. Plegia
E. Subfascial muscular edema

4. According to the clinical course of disease (V.Savelyev and all., 1973), III A stage
is:
A. Paresis
B. Plegia
C. Subfascial muscular edema
D. Partial muscular contracture
E. Total muscular contracture

5. According to the clinical course of disease (V.Savelyev and all., 1973), III B stage
is:
A. Paresis
B. Plegia
C. Subfascial muscular edema
D. Partial muscular contracture
E. Total muscular contracture

6. According to the clinical course of disease (V.Savelyev and all., 1973), III C stage
is:
A. Paresis
B. Plegia
C. Subfascial muscular edema
D. Partial muscular contracture
E. Total muscular contracture

7. Acute arterial occlusion is the indication for:


A. Conservative treatment
B. Urgent operative treatment
C. Elective operative treatment
D. None of the above
E .All of the statements is true

8. The absolute contraindications for operation in patients with acute arterial occlu-
sion include all of the following EXEPT:
A. An agonal state of the patient,
B. Total ischemic contracture of extremity (acute ischemia III С stage),
C. Grave condition of the patient with slight degree of ischemia (acute ischemia IA-
IB stage).
D. Acute myocardial infarction

9. The relative contraindications for operation in patients with acute arterial occlusion
include all of the following except
A. Acute myocardial infarction
B. Stroke
C. inoperable tumors
D. Slight degree of ischemia without signs of its progression
E. Grave condition of the patient with slight degree of ischemia (acute ischemia IA-
IB stage)

10. The commonest cause of death following arterial reconstruction of the lower ex-
tremity is
A. Graft infection
B. Cerebrovascular accident
C. Myocardial infarction
D. Systemic sepsis secondary to skin necrosis
E. None of the above

Keys for tests


1 2 3 4 5 6 7 8 9 10
B C D C D E B D E C
Tasks for final level of knowledge
1. A 65-year-old male cigarette smoker reports onset of claudication of his
right lower extremity approximately 3 weeks previously. His walking radius is lim-
ited to three blocks before the onset of claudication. Physical examination reveals
palpable pulses in the entire left lower extremity, but no pulses are palpable below
the right groin level. What investigations should be made?

The answer is Doppler ultrasonography, contrast angiography

2. A 27-year-old man sustains a single gunshot wound to the left thigh. In the
emergency room he is noted to have a large hematoma of his medial thigh. He com-
plains of paresthesias in his foot. On examination there are weak pulses palpable
distal to the injury and the patient is unable to move his foot. The appropriate initial
management of this patient would be

The answer is immediate exploration and repair

3. A 60-year-old man is admitted to the coronary care unit with a large anteri-
or wall myocardial infarction. On his second hospital day he begins to complain of
the sudden onset of numbness in his right foot and an inability to move his right
foot. On physical examination, the right femoral, popliteal, and pedal pulses are no
longer palpable. Vascular consultation is obtained. Diagnosis of acute arterial embo-
lus is made. The appropriate initial management of this patient would be

The answer is thrombolytic therapy

4. A 64-year-old man is admitted 14 mo following a femoropopliteal bypass


graft procedure with a cold foot and no graft pulse. Urokinase infusion is begun.
What management of this patient would be the best?

The answer is repeated bypass graft procedure

5. A 55-year-old man with recent onset of atrial fibrillation presents with a


cold, pulseless left lower extremity. He complains of left leg paresthesia and is una-
ble to dorsiflex his toes. Following a successful popliteal embolectomy, with resto-
ration of palpable pedal pulses, the patient is still unable to dorsiflexion his toes. The
next step in management should be:

The answer is immediate fasciotomy


Study guide #18
“Thromboses and embolisms of vessels, causes of development, diagnostics, dif-
ferential diagnostics, treatment and prevention.”

Overview
Hypercoagulation disorders, or thrombophilias, are most frequently encoun-
tered in surgical practice as deep venous thrombosis (DVT) or, less often, pulmonary
embolism (PE). Although a ready explanation based on Virchow's triad may be avail-
able for the majority of surgical patients, other potential causes must be considered.
Disseminated intravascular coagulation (DIC) is a syndrome rather than a spe-
cific disease. Many confusion and controversy surround its diagnosis and treatment.
Although DIC is generally considered a hemorrhagic disorder because of the obvious
bleeding problems encountered, it is important to recognize the sequelae resulting
from the microvascular (and sometimes large-vessel) thrombosis that accompanies
DIC and leads to end-organ failure and death.
The management of acute extremity ischemia remains a major surgical chal-
lenge. Even with optimal surgical management, acute lower extremity ischemia re-
sulting from thromboembolic disease continues to cause significant morbidity and
mortality. Limb loss rates of 8% to 22% and perioperative mortality rates of 10% to
17% continue to be reported. Maximization of limb salvage, although simultaneously
minimizing associated morbidity and mortality, requires expeditious diagnosis and
restoration of perfusion.
Acute DVT is a major cause of morbidity and mortality in the hospitalized pa-
tient, particularly in the surgical patient.
Pulmonary embolism (PE) is a relatively common cardiovascular emergency.
By occluding the pulmonary arterial bed it may lead to acute life-threatening but po-
tentially reversible right ventricular failure. PE is a difficult diagnosis that may be
missed because of non-specific clinical presentation. However, early diagnosis is
fundamental, since immediate treatment is highly effective. Acute pulmonary embo-
lism may occur in 25% of general surgery patients with great majority asymptomatic
and leads to 25% of postoperative deaths. Risk of fatal pulmonary embolism is 0.2 –
0.5% in moderate risk patient. Risk of fatal pulmonary embolism is 1 – 5 % in high
risk patient. Among patients with proximal deep vein thrombosis (DVT), about 50%
have an associated, usually clinically asymptomatic PE at lung scan. In about 70% of
patients with PE, DVT can be found in the lower limbs if sensitive diagnostic meth-
ods are used.
Educational aims:
35.Interrogation and clinical inspection of patients with the thromboembolic diseases.
36.To determine the etiologic and pathogenic factors of the thromboembolic diseases.
37.To find out the types of Pulmonary Embolism (PE).
38.To develop a plan of examination of the patients with arterial and venous throm-
bosis and embolism.
39.To estimate results of auscultation, ECG, echocardiogram, coronary angiography,
laboratory data, duplex investigation of the vessels.
40.To make a differential diagnosis, substantiate and formulate a diagnosis of the
thrombosis and embolism of arteries and veins.
41.To determine the indications for treatment of patients with the thromboembolic
diseases.
42.To cure of the patients with the thromboembolic diseases.
43.To estimate efficiency of treatment and prognosis of disease.
A student must know:
34. Anatomical and physiological data about arterial and venous systems and
lungs.
35. Main causes of arterial and vein thromboses.
36. Mechanism of development of Pulmonary Embolism (PE).
37. Mechanism of blood coagulation.
38. Clinical presentations of thromboembolic disorders
39. Methods of diagnosis of acute vessels’ diseases.
40. Main principals of surgical interferences at the different thrombotic and
embolic complications.
A student must be able to:
24. To collect and estimate the complaints of patient with the thromboem-
bolic diseases, gather anamnesis, to conduct physical research and interpret their re-
sults.
25. To define the rational quantity of laboratory and instrumental methods of
investigation.
26. Be able to do artificial pulmonary ventilation by methods “nose in nose”
or “mouth in mouth
27. To define the indications for surgical interventions and to choose the op-
eration method in case of thrombosis or embolism.
28. Prescribe post-operative treatment depending on patient’s state.

Terminology.
Term Definition
A condition in which a blood vessel is obstructed by
Thromboembolism a clot (thrombus) carried in the bloodstream from its site of
formation.
An aggregation of platelets, fibrin, clotting factors,
and cellular elements of the blood attached to the interior
Thrombus
wall of vein or artery, sometimes occluding the lumen of
the vessels.
Pulmonary Embolism The blockage of pulmonary artery by fat, air, tumor
(PE) tissue, or thrombus that usually arises from the peripheral
vein (most frequently one of deep veins of the legs).
Pulmonary Infarction Necrosis in part of a lung caused by an obstruction in
(PI) branch of a pulmonary artery.
Thrombophilias Hypercoagulation disorders
A surgical incision into an artery for the removal of
Embolectomy an embolus or clot, performed as emergency treatment for
arterial embolism.

Content:
THROMBOTIC DISORDERS
Thrombophilia may be caused by a decrease in antithrombotic proteins or an
increase in prothrombotic proteins. The former include antithrombin deficiency, pro-
tein C deficiency, and protein S deficiency. The latter include factor V Leiden, pro-
thrombin gene mutation (G20210A), and increased levels of factors VII, VIII, IX, and
XI or vWF.
Antithrombin deficiency is an autosomal dominant genetic disorder that affects
an estimated 1.1% of unselected patients with venous thromboembolism. Antithrom-
bin levels range from 40% to 70% of normal, with as many as 85% of affected pa-
tients suffering a thrombotic event by 50 years of age. These patients are generally
believed to be at greater risk than patients with other types of thrombophilia.
Protein C is a vitamin K–dependent glycoprotein synthesized by the liver that
inactivates factors Va and VIIIa. Protein C deficiency is also an autosomal dominant
genetic disorder that affects 3.2% of unselected patients with venous thromboembo-
lism. Up to 50% of affected patients will experience a thromboembolic event by the
age of 50.
Protein S is also a vitamin K–dependent glycoprotein that acts as a cofactor to
inactivate factors Va and VIIIa. Protein S deficiency affects 2.2% of unselected pa-
tients with thromboembolism, with up to 50% experiencing their first event by 25
years of age.
Factor V Leiden is a single–base pair mutation (Arg506➙Gln) of the factor V
gene that results in activated protein C resistance. Factor V Leiden is found in 4% to
6% of the general population and in 6% to 33% of unselected patients with venous
thromboembolism. There is a threefold to sevenfold increase risk for thromboembo-
lism, with 30% suffering an event by the age of 60.
The prothrombin gene mutation G20210A is a glycine-to-arginine mutation in
the factor II (prothrombin) gene. This mutation is identified in 6.2% of unselected pa-
tients with thrombotic events. A substantial number of these patients also carry the
factor V Leiden mutation.
Increased levels of factors VII, VIII, IX, and XI or vWF (>150 IU/dL) are as-
sociated with a 2.2- to 4-fold increase risk for thrombotic events and a 1.08 relative
risk for each 10-IU/dL increase. The causes of these elevations remain unclear but
may have a genetic basis.
In addition to these inherited disorders, acquired disease processes associated
with thrombotic events include pregnancy; cancer; sepsis; trauma; major operations,
particularly those involving the pelvis; nephrotic syndrome; myeloproliferative disor-
ders; drugs such as oral contraceptives, hormonal therapy, and chemotherapy; and
malnutrition, including folic acid and vitamin B12 and B6 deficiency. Arterial throm-
boembolic events are difficult to separate from underlying atherosclerotic disease.
Disorders include hypercysteinemia; paradoxical embolism through a patent foramen
ovale or an atrial or ventricular septal defect; and inherited thrombophilia. Treatment
of thrombotic events includes correction of the underlying process and anticoagula-
tion. The duration of anticoagulation remains controversial and depends on the mag-
nitude of the event, ongoing risk for thrombosis and treatment-associated risks, and
anticipated future circumstances. Prophylaxis for patients at risk is based on the un-
derlying disorder, magnitude of the risk, and anticipated requirements.

DISSEMINATED INTRAVASCULAR COAGULATION


DIC is a systemic thrombohemorrhagic disorder with evidence of coagulant ac-
tivation, deposition of fibrin, fibrinolytic activation, consumption of coagulation fac-
tor and platelets, and end-organ dysfunction that is seen in many clinical situations.
The disorder may have a spectrum of manifestations, from low-grade DIC, with min-
imal symptoms and minor laboratory abnormalities, to fulminant DIC, with life-
threatening bleeding and coagulation abnormalities producing end-organ dysfunction
and death. Conditions associated with DIC include hemolysis, massive transfusion,
amniotic fluid embolism, placental abruption, retained fetus, gram-negative and
gram-positive sepsis, viremia, burns, crush injury and tissue destruction, leukemia,
malignancy (especially metastatic), liver disease, and miscellaneous inflammatory
and autoimmune conditions, including vasculitis. Although the diagnosis of DIC is
often attached to patients who are undergoing massive transfusion, platelet dysfunc-
tion as a result of hypothermia or a specific factor deficiency is excluded before mak-
ing a diagnosis of DIC. In most cases, such patients will respond to rewarming and
replenishment of coagulation factors and platelets.
With activation of the coagulation and fibrinolytic systems, both thrombin and
plasmin are in circulation. Thrombin converts fibrinogen to fibrin monomers by
cleaving fibrinopeptide A and B from fibrinogen. These fibrin monomers form solu-
ble fibrin clots, which cause microvascular thrombosis with entrapment and depletion
of platelets. Simultaneous degradation of these factors by plasmin takes place. De-
pressed levels of fibrinogen and elevated levels of fibrinogen degradation products
(fibrin split products) result. These degradation products inhibit the normal coagula-
tion of blood by delaying polymerization of fibrin. Fibrin degradation products may
also interpose themselves between fibrin and polymers and form a weak fibrin clot.
Fibrin degradation products include X Y, D, and E fragments; platelet dysfunction is
attributable to the latter two fragments. Plasmin also degrades factors V, VIII, IX, and
XI and activates the complement system. These changes produce the clinically ob-
served alterations characteristic of DIC.
Laboratory abnormalities in DIC are variable and related to the many diseases
associated with this condition. Common abnormalities include an abnormal PT and
aPTT with depressed fibrinogen levels and abnormal platelet counts. Levels of fibrin
degradation products and d-dimer are elevated. The peripheral smear reveals frag-
mented RBCs, but this finding is not specific. Because of the continued activation of
coagulation, thrombin/antithrombin complexes will be formed. Levels of throm-
bin/antithrombin and AT-III are depressed. Fragments of coagulation factor degrada-
tion are elevated, including F1.2 and FpA. Because of activation of the fibrinolytic
system, plasminogen and α2-antiplasmin inhibitor levels are decreased.
Low-grade DIC generally responds to management of the underlying disorder.
The appropriate therapy for fulminant DIC remains controversial, compounded by the
lack of objective studies and many underlying causes. The assistance of a physician
experienced in managing DIC is valuable. Treatment of the underlying condition is
critical to successful management of DIC. Also important is treatment of the throm-
botic intravascular process that causes end-organ failure. Treatment with heparin has
never demonstrated beneficial effects in controlled trials. The most logical anticoagu-
lant would be directed against TF activity, such as recombinant TFPI, inactivated fac-
tor VIIa, and recombinant NAPc2. Antithrombin concentrates administered to attain a
serum level of 125% of normal have been useful in some patients. Continued bleed-
ing may be related to depletion of components, but random administration of blood
products, especially those containing fibrinogen, may exacerbate the syndrome.
Washed RBCs, platelets, AT-III concentrate, and crystalloid and colloid volume ex-
panders may be used. If other therapeutic measures are unsuccessful, inhibition of fi-
brinolysis may be attempted. ε-Aminocaproic acid may be given along with heparin.
Despite improved diagnostic and therapeutic modalities, mortality from DIC remains
high and closely related to the underlying disorder.

ACUTE THROMBOEMBOLIC DISEASE


Pathophysiology
Compared with other organs and tissues, the extremities are relatively resistant
to the effects of ischemia. Unlike the brain, which suffers infarction after only 4 to 8
minutes of ischemia, or the myocardium, which infarcts after 17 to 20 minutes, the
lower extremity may be salvaged after up to 5 to 6 hours of profound ischemia.
Evaluation of the effect of ischemia on the extremity is complicated by the fact
that the various tissues that comprise the extremity have different susceptibilities to
ischemic injury, and they manifest this injury in different fashions. Skin and bone are
relatively resistant to the effects of ischemia and may survive injuries that, by their
effect on other tissues, have rendered the limb painful and useless. Nervous tissue is
generally the most sensitive component of the extremity to the effects of ischemia.
Significant morbidity may therefore result from isolated ischemic nerve injury in an
otherwise intact limb.
Skeletal muscle is the major structural component of the extremity and, for a
variety of reasons, plays a key role in the pathophysiology of extremity ischemia.
Skeletal muscle constitutes more than 40% of the body mass and about 75% of the
lower extremity weight. Although skeletal muscle has a relatively slow resting meta-
bolic rate compared with other tissues, it accounts for 90% of the metabolic activity
of the lower extremity. Skeletal muscle receives 71% of the resting lower extremity
blood flow and a larger proportion during reperfusion hyperemia. Skeletal muscle
plays a pivotal role in the numerous local and systemic manifestations of extremity
ischemia-reperfusion injury.
Reperfusion Syndrome
The profound effects of revascularization of the ischemic lower extremity were
described as early as the 1950s by Haimovici. As ischemic skeletal muscles reper-
fuse, a variety of intracellular ions, structural proteins, enzymes, and other compo-
nents are released through the damaged sarcolemma into the circulation. The result-
ing myonephropathic syndrome, with its associated hemodynamic instability, lactic
acidosis, and hyperkalemia, is well recognized by surgeons. Myoglobin released from
injured muscle cells into the circulation is cleared through the kidneys, resulting in
dark urine (without red blood cells). Myoglobinuria may persist for 2 to 4 days after
reperfusion. Acute renal failure may ensue from myoglobin casts developing in the
renal tubules as well as direct toxic effects of the myoglobin on the tubules. Serum
creatine phosphokinase levels may increase dramatically (to >10,000 units) after
reperfusion of ischemic muscle. Myocardial contractility may become depressed; in-
creased cardiac irritability in the setting of electrolyte disturbances (typically hyper-
kalemia) may lead to life-threatening dysrhythmias.
When a lower extremity is subjected to severe ischemia, cellular membrane
dysfunction results. In this setting, the reperfusion phase is marked by the develop-
ment of both intracellular and interstitial edema. Intra-cellular edema results from
membrane damage and failure of the membrane-bound adenosine triphosphatase
(ATPase). Interstitial edema results from increased microvascular membrane permea-
bility to ions, water, and proteins. This edema may appear within minutes, progress-
ing significantly over the next 24 hours. The amount of edema is dependent on the
period of ischemia, the underlying occlusive disease, and the adequacy of revascular-
ization. When muscle edema occurs within the confines of an osseofascial compart-
ment, interstitial pressure continues to increase. Acute compartment syndrome results
as pressure increases beyond capillary perfusion pressure (30 mm Hg) and tissue per-
fusion is impaired. Unless recognized and decompressed by fasciotomy, compartment
syndromes will lead to prolonged tissue ischemia despite apparent successful revas-
cularization.
Occasionally, prolongation of the ischemic injury may also occur as a result of
microvascular obstruction to blood flow. Endothelial cell edema may predispose to
white blood cell and platelet sludging, leading to the so-called no-reflow phenome-
non. Similarly, prolonged vascular occlusion can lead to small vessel thrombosis in
the muscle and skin, which prevents tissue reperfusion when blood flow is restored to
the larger vessels.
Although current understanding of the systemic effects of limb revasculariza-
tion and compartment syndrome is well developed, understanding of ischemia-
reperfusion injury at the tissue level is only beginning to evolve. The pathophysiology
of the ischemic injury is complex and involves a variety of factors, including de-
creased cellular energy charge, inadequate oxygen and substrate delivery, altered ion
compartmentalization, and membrane permeability changes. More recently, attention
has focused on reperfusion injury (i.e., cellular injury that occurs or is manifested at
the time perfusion is restored to ischemic tissue). Most of this injury is believed to be
induced by oxygen-derived free radicals, which are formed as oxygen is reintroduced
into ischemic tissue. These oxygen-derived free radicals are generated by neutrophils
through an NADPH oxidase enzyme on the plasma membrane. These radicals are
highly reactive compounds that result from the univalent reduction of molecular oxy-
gen. The most important free radical species include the superoxide radical, hydrogen
peroxide, and the extremely reactive hydroxyl radical. These unstable compounds at-
tack the unsaturated bonds of fatty acids within the phospholipid membranes, causing
both mechanical and functional derangements within the reperfused tissue.
Etiology
Embolism
Embolic occlusion of a previously unobstructed vessel generally results in the
most severe forms of acute ischemia. The most common sources of arterial emboli
are:
I. Cardiogenic (80%)
a. Atrial fibrillation (50%)
b. Myocardial infarction (25%)
c. Other (5%)
II. Noncardiac (10%)
a. Aneurysmal disease (6%)
b. Proximal artery (3%)
c. Paradoxical emboli (1%)
III. Other or Idiopathic (10%)
About 80% of arterial emboli are cardiac in origin. In the past, most of these
emboli were due to complications of rheumatic heart disease, including emboli from
diseased valves as well as atrial fibrillation. More recently, atherosclerotic cardiovas-
cular disease has become the major contributor. About 70% of patients with cardio-
genic emboli have atrial fibrillation, with the emboli arising in atrial mural thrombus.
Atrial fibrillation is currently the most common source of cardiogenic emboli. Acute
myocardial infarction is the second most common cause of cardiogenic emboli, pre-
ceding about one third of peripheral embolic events. Ventricular mural thrombus,
which occurs after acute myocardial injury, is often the source of emboli. Mural
thrombus can form within hours of a myocardial infarction but may develop weeks
after myocardial infarction and ultimately occurs in more than one third of cases. Pe-
ripheral embolization may often be the first sign of a previously “silent” myocardial
infarction. Although readily available, standard echocardiographic techniques are of-
ten insensitive to the detection of thrombus within the atrium. Transesophageal echo-
cardiography is more sensitive in the detection of both atrial and ventricular mural
thrombus.
Although rheumatic valvular heart disease has declined in importance as a
source of emboli, emboli from prosthetic heart valves have become an increasingly
important source of embolization. Patients with prosthetic heart valves who have
sudden onset of lower extremity ischemia are suspected of having valvular emboliza-
tion, particularly in the setting of inadequate anticoagulation.
Bacterial or fungal endocarditis can result in peripheral embolization. Such
emboli tend to be more peripheral and result in septic complications rather than large
vessel occlusion and ischemia. IV drug abuse remains a major risk factor for endo-
carditis and subsequent embolic complications. These smaller emboli often result in
digital lesions and peripheral infected pseudoaneurysms. More rare forms of cardio-
genic emboli include embolization from intracardiac tumors, most commonly, atrial
myxomas. Pathologic examination of unusual appearing emboli is essential for mak-
ing the diagnosis in this situation. An unusual form of peripheral embolization,
termed paradoxical embolization, may occur in the setting of patients with intracardi-
ac defects (e.g., patent foramen ovale) and elevated right-sided cardiac pressures. In
such patients, venous emboli to the heart may gain access to the arterial circulation
through the patent foramen ovale and thereby embolize to the distal arterial circula-
tion. Although rare, such emboli need to be considered in patients with concurrent
deep venous thrombosis, pulmonary emboli, or appropriate cardiac defects.
Arterial-to-arterial embolization is another cause of extremity ischemia. Most
of these cases arise from atherosclerotic plaque of the aorta. In most situations, such
atheroembolization results in diffuse microembolization, resulting in the picture of
painful, bluish discoloration of the toes with cutaneous gangrene, livedo reticularis,
and often transient muscular pain. This so-called blue toe syndrome generally appears
in the setting of palpable peripheral pulses right down to the pedal level. Although
this syndrome most frequently occurs after intraluminal catheterization, it may occur
spontaneously. Such microembolization may also affect the renal and mesenteric cir-
culations, resulting in progressive renal failure and intestinal infarction. More rarely,
artery-to-artery emboli may be sufficiently large to result in macroembolization with
distal ischemia. This may result from mural thrombus or plaque from an atheroscle-
rotic aorta or from the mural thrombus lining an aneurysmal artery.
In 10% to 15% of cases, the source of embolization ultimately cannot be de-
termined. Such emboli are not designated as idiopathic until a detailed history and
physical examination, as well as complete cardiac and peripheral imaging, fails to
identify an embolic source.
The most common sites of embolization are:
1. Aortic bifurcation (10-15%)
2. Iliac bifurcation (15%)
3. Femoral bifurcation (40%)
4. Popliteal (10%)
5. Upper extremities (10%)
6. Cerebral (10-15%)
7. Mesenteric, visceral (5%)
Cardiogenic emboli are less likely to occur to the cerebral or visceral circula-
tion. Embolization to the cerebral circulation occurs in about 13% of cases with po-
tentially devastating results. Embolization to the mesenteric and renal vessels occurs
in about 5% of peripheral emboli. Although often clinically silent, such emboli can
result in acute catastrophes, such as mesenteric ischemia secondary to a superior mes-
enteric artery embolus.
Thrombosis
Acute thrombosis generally occurs in vessels affected by preexistent athero-
sclerosis. As such, there is generally some degree of collateral vessel development,
and the resultant ischemia is often less severe than with acute embolic disease. The
most common extremity vessel affected is the superficial femoral artery, which is of-
ten affected by long segments of atherosclerosis. Popliteal artery aneurysms are also
prone to thrombosis and may result in severe ischemia, particularly when associated
with embolization to the tibial vessels.
A particularly severe form of ischemia results from distal vascular thrombosis
of the extremities, which may occur in the setting of sepsis or with hypercoagulable
states. The most common hypercoagulable states associated with acute arterial
thrombosis are hyperhomocysteinemia, antithrombin III deficiency, lupus anticoagu-
lant (antiphospholipid antibody), and protein C deficiency. Although usually associ-
ated with venous thrombosis, activated protein C resistance caused by the spontane-
ous mutations of factor V Leiden may also cause arterial thrombosis. Exposure to
heparin may lead to heparin-induced thrombosis in patients with heparin-induced an-
tibodies. These antibodies cross-react with the heparin–platelet factor 4 complex on
the platelet surface, leading to granular, white, platelet-laden thrombi and thrombocy-
topenia.
Acute thrombosis of a previous arterial bypass graft may also lead to recurrent
ischemia. The degree of ischemia depends on the location of the graft and the original
indication for surgery. Early graft occlusions (within 2 months of surgery) are usually
caused by technical or judgmental errors. Intermediate graft occlusions (within 2
years) are generally attributable to the formation of intimal hyperplasia at the anasto-
moses or within the graft (for vein grafts).

Presentation and Evaluation


The classic presentation of patients with acute ischemia of the extremities may
be recalled by the “five Ps”: pain, pallor, pulselessness, paresthesias, and paralysis.
Pain is the most common complaint in alert patients. The degree of pain de-
pends on the severity of ischemia, which is generally determined by the location of
the occlusion and the degree of collateral flow. Ischemic pain can be severe and diffi-
cult to relieve with even large doses of narcotics. The sudden onset of severe ischem-
ic pain in a previously asymptomatic patient is most suggestive of an embolic occlu-
sion. Patients with spontaneous thrombosis often have had chronic symptoms of
claudication or various degrees of pain before the acute event. Obtunded patients may
have severe ischemia without complaints of pain. This is most commonly encoun-
tered in intubated, postoperative patients with spontaneous or iatrogenic arterial
thrombosis.
Pallor is a common but relative finding that depends on the degree of ischemia
and the underlying skin color. A sudden and complete embolic occlusion may result
in a cool, waxy-appearing white extremity with no signs of cutaneous blood flow.
Conversely, a partial occlusion may result in only delayed capillary refill with pallor
on elevation of the extremity and rubor on dependency.
The absence of arterial pulses on examination will alert the surgeon to both the
location of the arterial occlusion and the degree of ischemia. Patients with acute arte-
rial embolism generally have normal palpable pulses above the occlusion with a
complete absence below. The pulse immediately above the occlusion (e.g., the com-
mon femoral pulse in a patient with a femoral bifurcation embolus) may be particular-
ly prominent with a “water-hammer” quality that results from limited arterial outflow.
The presence of normal arterial pulses in the contralateral extremity is most sugges-
tive of an acute embolus because patients with acute thrombosis generally have some
degree of symmetrical pulse deficit, owing to long-standing atherosclerosis. A
handheld continuous-wave Doppler examination plays an important role in the initial
evaluation of patients with acute vessel occlusion. The presence of even monophasic
Doppler signals over the pedal vessels affirms distal vascular patency and at least
short-term viability of the distal tissues. Conversely, a complete absence of arterial
flow is most suggestive of profound ischemia and calls for immediate revasculariza-
tion.
As mentioned previously, within the extremities, the peripheral nerve is the tis-
sue that is most sensitive to ischemia. As such, the degree of neurologic dysfunction
is a sensitive barometer of the degree of ischemia. With mild ischemia, the findings
may be subjective and subtle. Early paresthesias may be characterized as a numbness
of the toes or a slight decrease of sensation of the foot compared with the contrala-
teral extremity to light touch or pinprick. With severe ischemia, however, profound
sensory loss may lead to complete anesthesia of the foot, indicative of impending tis-
sue loss without early revascularization. Paradoxically, these patients with the most
severe ischemia and complete anesthesia complain of less pain than those with milder
ischemia and intact sensation.
Weakness of the extremity is another important sign of neurologic ischemia of
the extremity. Mild ischemia results in a weakness or subjective stiffness of the toes
and foot, which may be easily appreciated on physical examination. As ischemia be-
comes more progressively severe, the weakness may progress to frank paralysis of the
effected extremity. It is not unusual for the patient with a sudden complete embolic
occlusion to suffer the immediate onset of paralysis of the affected extremity. Patients
with an aortic saddle embolus may have bilateral paralysis and anesthesia from the
waist down. In patients with severe ischemia characterized by anesthesia and paraly-
sis, it is important to distinguish reversible from irreversible ischemic changes. Pa-
tients with prolonged ischemia have palpable firmness to the extremity muscle and
stiffness to the extremity indicative of muscle rigor. Reperfusion of such an extremity
does not restore function and can result in severe systemic injury. Primary amputation
is the safest form of management in such cases.
The acute embolic occlusion of an extremity artery can be accurately diagnosed
by a careful history and physical examination in most cases; emboli to the cerebral
and visceral bed may be more difficult to identify and treat. The diagnosis in these
cases is based on a high index of suspicion based on the patient's complaints and his-
tory. For example, a patient with the sudden onset of severe aching abdominal pain
coupled with risk factors for cardiogenic emboli and a relative lack of physical find-
ings is presumed to have suffered a superior mesenteric artery embolus until proved
otherwise.
Because evaluation of patients with acute arterial occlusion generally differs
for patients who have suffered embolic versus thrombotic occlusion, it is important to
make the appropriate clinical distinction. As outlined previously, patients with emboli
tend to have risk factors (e.g., atrial fibrillation, recent myocardial infarction, pros-
thetic heart valve), a more sudden onset of symptoms (no prior claudication), and uni-
lateral findings (normal contralateral extremity). Despite these guidelines, the distinc-
tion can be difficult to make at times. Certainly, patients with recent myocardial in-
farctions or atrial fibrillation may have concurrent peripheral vascular occlusive dis-
ease and patients with peripheral vascular occlusive disease may suffer cardiogenic
emboli. Thus, the evaluation of each patient must be individualized to supply enough
information to effectively treat the patient without jeopardizing limb salvage or func-
tion by delaying revascularization.
When the history and physical examination implicates an embolus as the
source of occlusion, the subsequent evaluation is simple and direct. Routine preopera-
tive blood work and a chest radiograph are obtained, and a 12-lead electrocardiogram
is performed to document atrial fibrillation, cardiac ischemia, or a previous (and per-
haps unsuspected) myocardial infarction. Because arterial emboli are removed by di-
rect arterial cutting and removal of the embolus (as described later), there is generally
no need for preoperative arteriography. When the diagnosis of embolus is questiona-
ble or the site for simple arterial cutdown and arteriotomy unclear, a preoperative ar-
teriogram may be useful to define the anatomy and guide the revascularization proce-
dure. The management of distal extremity emboli (below the brachial or popliteal ar-
tery) may be assisted by detailed preoperative angiograms in selected cases. For ex-
ample, very distal emboli within the distal tibial or pedal vessels may be best treated
by catheter-based thrombolytic therapy (described later) rather than surgical extrac-
tion. Arteriograms of intra-arterial emboli often demonstrate an abrupt cutoff of the
artery with a rounded meniscus at the site of the embolus. Conversely, the embolus
may appear as an intraluminal defect with partial flow around it. Postoperatively,
when the embolus has been removed and the limb revascularized, and the patient is
stable, the evaluation is completed by documentation of the source of the embolus. In
most cases, this involves transesophageal echocardiography.
The surgical management of patients with acute arterial thrombosis is generally
more complex than the simple arteriotomy and clot extraction used for embolic oc-
clusion. Patients suffering thrombotic occlusion often have diffuse atherosclerotic
disease at multiple arterial levels, various degrees of collateral blood vessel develop-
ment, and unpredictable levels of distal arterial reconstitution. These patients are best
evaluated with complete arteriography to define the optimal approach to revasculari-
zation. Rarely, arterial thrombosis results in such profound ischemia that the patient is
taken immediately to the operating room and intraoperative arteriography is per-
formed after arterial exposure.
Management
Embolic Occlusion
Patients with acute arterial occlusion are anticoagulated with an IV heparin bo-
lus (5000-10,000 units) and begun on a continuous infusion at 1000 units/hour. The
primary goal of anticoagulation is to prevent thrombosis resulting from stagnant flow
beyond the primary embolic or thrombotic occlusion. Furthermore, short- and long-
term anticoagulation is indicated in patients with cardiogenic emboli to prevent addi-
tional embolic events. Recurrent embolization occurs in about 7% of patients who are
chronically anticoagulated versus 21% of those who are not.
The most important initial decision focuses on the viability and potential sal-
vage in the ischemic limb. Rarely, patients have such long-standing and severe is-
chemia that irreversible ischemic injury to the extremity (manifesting as rigor of the
muscles or frank gangrenous changes to the foot) has occurred. Such cases are best
treated with primary extremity amputation.
In most cases, however, early revascularization for restoration of limb function
is indicated. As mentioned earlier, immediate progression to the operating room after
minimal and expeditious evaluation is indicated for patients who have suffered occlu-
sive emboli to the extremities. For patients with emboli to the lower extremity, the
entire extremity is prepped and draped into the field. This allows immediate evalua-
tion of success of the revascularization as well as ready access for more distal arterial
exposure if necessary to complete the procedure. Emboli to the iliac and femoral ves-
sels are approached through a femoral incision. Femoral artery incision can be per-
formed under local anesthesia with IV sedation. Often, however, regional or general
anesthesia is preferable, particularly when popliteal artery exposure is necessary.
A standard longitudinal or oblique skin incision is made, with the common
femoral artery as well as profunda femoris and superficial femoral arteries all inde-
pendently controlled. Normal arteries, free of significant atherosclerosis, are best in-
cised transversely just above the femoral bifurcation. In patients with significant ath-
erosclerosis, however, longitudinal arteriotomies afford the best exposure and most
reliable closure (often with an arterial patch). Patients with iliac-level emboli often
have an absence of a femoral pulse. Inflow is restored by retrograde passage of a No.
4 or 5 balloon thrombectomy catheter. The catheter is passed in 10-cm increments.
Gentle inflation of the catheter as it is withdrawn engages and extracts the thrombus
without causing arterial wall injury. Each passage extends an additional 10 cm until
pulsatile arterial inflow is restored. Passages are continued until no additional throm-
bus is recovered. Attention is then turned to the outflow vessels. Antegrade passage
of a No. 3 Fogarty catheter, 3 to 5 cm down the profunda, generally extracts any im-
pacted thrombus and restores good backflow. The catheter is then passed down the
superficial femoral artery in increments until no further thrombus is extracted. This
may necessitate passage to the popliteal level or beyond. Blind passage of the Fogarty
catheter from the groin to below the popliteal artery almost always results in cannula-
tion of the peroneal artery. Specific cannulation of the anterior tibial and posterior tib-
ial arteries requires more distal, below-knee popliteal artery exposure. Alternatively,
selective passage of balloon catheters over guide wires using fluoroscopic control has
facilitated selective cannulation of distal vessels. When good inflow and reverse
bleeding have been restored, the transverse arteriotomy is closed with 5-0 polypro-
pylene sutures, and flow is restored.
For patients with a palpable popliteal pulse and distal emboli, exposure is best
obtained at the level of the below-knee popliteal artery through a standard medial in-
cision. The soleus muscle is carefully taken down off the tibia, exposing the tibioper-
oneal trunk. The anterior tibial veins may be divided to facilitate individual control of
the anterior tibial, posterior tibial and peroneal arteries. A distal popliteal arteriotomy
may then be performed and retrograde passage of a No. 3 catheter used to restore
normal inflow to the popliteal level. Selective cannulation of the anterior and posteri-
or tibial vessels, as well as the peroneal artery, may be performed with a No. 2 or 3
thrombectomy catheter, restoring good backflow. At the conclusion of the embolec-
tomy and closure of the arteriotomy, reperfusion of the extremity is assessed. In most
cases, restoration of palpable pedal pulses and pink, well-perfused toes reassures the
surgeon of a successful and complete embolectomy. In some cases, however, spasm
of the distal vessels secondary to passage of the Fogarty catheter may delay the return
of normal perfusion. Even in these cases, however, restoration of strong Doppler sig-
nals is present at the pedal level. After several minutes, palpable pulses and well-
perfused skin are reestablished. In any case in which there is a doubt as to the success
of the embolectomy, completion arteriography is performed to assess the distal vascu-
lature for spasm and retained thrombus. For those rare patients who have restoration
of patent vessels down to the mid-tibial level, with occlusions of the distal tibial and
pedal vessels, direct intraoperative arterial infusion of urokinase (50,000-250,000
units) may help lyse distal thrombus and restore perfusion to the foot. Occasionally,
direct cutdown and thrombectomy of the distal tibial and pedal levels is necessary to
remove distally impacted thrombus.
Patients with a saddle embolus to the aortic bifurcation with bilateral lower ex-
tremity ischemia are approached through simultaneous bilateral femoral artery inci-
sion. The femoral arteries are simultaneously clamped and opened, and retrograde
thrombectomies are performed to both limbs simultaneously to prevent fragmentation
of the thrombus and distal embolization to the extremity contralateral to the throm-
bectomy. After normal inflow is obtained, the remainder of the procedure is similar to
that for an iliac-level embolectomy.
Patients with upper extremity emboli are approached in a similar fashion. The
entire extremity is prepped and draped into the field. Usually the entire embolectomy
can be performed under local anesthesia through a longitudinal incision performed
just above the elbow. The brachial artery is carefully dissected from its companion
structures, and a transverse arteriotomy is performed. Most emboli impact just above
the elbow and are easily extracted by passage of a No. 3 Fogarty catheter both proxi-
mally and distally. Emboli that have lodged at the subclavian level are generally easy
to remove by retrograde passage of the No. 3 Fogarty catheter from the elbow proxi-
mally to the subclavian level. Arterial closure and assessment are similar to that per-
formed for the lower extremity.
Recently, several authors have reported on the adjunctive use of fluoroscopy
for accurate passage of the thrombectomy catheter. Similarly, others have found an-
gioscopy to be useful in the evaluation of completeness of the thrombectomy (par-
ticularly during thrombectomies of occluded prosthetic grafts).
Thrombotic Occlusion
As mentioned previously, patients with thrombotic arterial occlusion undergo
an initial arteriogram to delineate the arterial anatomy and define the best mode of re-
vascularization. In most cases, the site of thrombotic occlusion is well delineated. In
patients with a satisfactory inflow and outflow vessel with a long segment of occlud-
ed vessel, the best option is generally to proceed to surgery and perform a surgical
bypass procedure. Inflow to the femoral artery can be restored with either a direct
aortofemoral revascularization or various forms of extra-anatomic bypass (fem-
orofemoral bypass, axillofemoral bypass, or iliofemoral bypass), depending on the
patient's anatomy. Infrainguinal arterial occlusions are best treated with femoral-distal
bypass operations with autogenous vein. When short-segment thrombotic occlusions
are identified, a catheter-directed infusion of thrombolytic therapy may recanalize the
vessel, revealing an appropriate underlying lesion for balloon angioplasty. Recent
studies have suggested that aggressive thrombolytic therapy is most beneficial when
the ischemic interval is short and that initial thrombolysis may reduce the magnitude
of subsequent surgical procedures. When the arteriogram reveals no distal arterial re-
constitution appropriate for bypass, catheter-based thrombolysis may restore suffi-
cient perfusion to establish or reveal a distal vessel suitable for bypass.
Compartment Syndrome
As described previously, extremities subjected to prolonged periods of ische-
mia followed by reperfusion suffer reperfusion injury manifesting as both intracellu-
lar and interstitial edema. This reperfusion injury occurs regardless of the cause of ar-
terial occlusion (embolus or thrombosis) or mode of revascularization (balloon embo-
lectomy, surgical bypass, or catheter-based thrombolytic recanalization). When mus-
cular swelling occurs within the confines of an unyielding osseofascial bound space,
increased compartmental pressures occur. These increased pressures can occur in ei-
ther the arm or leg. In the lower extremity, the calf is the most frequently affected ar-
ea. The anterior compartment of the calf and then the lateral, deep posterior and su-
perficial posterior compartments are involved with decreasing frequency. Within the
thigh, the anterior quadriceps compartment is the most frequently involved area. In
the upper extremity, the anterior or volar forearm compartment is most frequently in-
volved, but the dorsal forearm, as well as hand and upper arm, may also be involved
by reperfusion edema.
When the intracompartmental pressures increase to more than the level of ca-
pillary perfusion pressure, both venous outflow and capillary perfusion may become
impaired. This may result in recurrent and prolonged ischemia unless the affected
compartment is decompressed. As with the initial ischemic injury, the nerve tissue is
most susceptible to the effects of compartment syndrome. The diagnosis of compart-
ment syndrome is based on a high degree of suspicion and careful evaluation for
signs and symptoms. Extremities that are revascularized after 4 to 6 hours of severe
ischemia are most at risk for development of compartment syndrome. The early signs
and symptoms must be carefully watched for, particularly in patients with decreased
sensorium. The typical clinical findings of early compartment syndrome are severe
pain that is disproportionate to the relative paucity of physical findings. Patients usu-
ally have marked tenderness on compression of the edematous calf and severe dis-
comfort on passive extension of the calf with dorsiflexion or plantar flexion of the
foot. Because the anterior compartment is the most commonly affected space within
the calf, the first neurologic findings are often numbness in the area of the great toe
web space, attributable to pressure on the deep peroneal nerve. Palpable pulses and
strong Doppler signals may be well preserved despite progressive compartment syn-
drome and must not lead to a false sense of security. When the diagnosis of compart-
ment syndrome is in question, direct measures of the pressure within the compartment
may be performed. A needle cannula placed directly into the compartment and at-
tached to a pressure transducer gives an accurate measure of the intracompartmental
pressure. There are also portable, handheld devices designed for measurement of in-
tracompartmental pressures. Although somewhat controversial, it is generally agreed
that, as compartmental pressure reaches 30 mm Hg, capillary perfusion is impaired,
and neurologic and muscular injury occurs.
Although significant calf swelling and increased compartment pressures can of-
ten be managed with extreme elevation of the extremity, marked elevation of com-
partment pressures with neurologic changes necessitates early and effective decom-
pression to prevent a permanent and disabling injury. Most vascular surgeons prefer a
two-incision, four-compartment fasciotomy. The medial-based longitudinal incision
is made just posterior to the tibia and is carried down through the fascia into the su-
perficial posterior space. The soleus muscle is then incised longitudinally, near its tib-
ial insertion, and the deep fascia is incised longitudinally to decompress the deep pos-
terior compartment. A second anterolateral calf incision is made longitudinally and
carried down through the fascia into the anterior compartment. A second longitudinal
fascial incision is made over the lateral compartment, decompressing the peroneus
muscles. Severe compartment syndrome manifests by an immediate pouting of the
muscles as they swell beyond the fascial incision. The skin incision needs to be long
enough to prevent any restriction on the underlying muscle.
When patients undergo revascularization after a prolonged interval (≥6 hours)
of severe ischemia, consideration for a prophylactic fasciotomy in anticipation of im-
pending compartment syndrome needs to be entertained. Such prophylactic fascioto-
mies may be performed through more limited skin incisions with blind extension of
the fascial incisions by sliding a slightly opened Metzenbaum scissors along the cut
edge of the fascia. The skin incisions can be extended later if full-blown compartment
syndrome develops.
An alternative surgical approach to four-compartment fasciotomy, favored by
many orthopedic surgeons, involves resection of a portion of the fibula or parafibular
dissection of all soft tissues away from a section of the fibula. Because the fascia of
all four compartments inserts on the fibula, disruption of these fibular attachments
may adequately decompress the various calf compartments.

DEEP VENOUS THROMBOSIS OF THE LOWER EXTREMITY


The triad of venous stasis, endothelial injury, and hypercoagulable state first
posited by Virchow in 1856 has held true a century and a half later.
Acute DVT poses several risks and has significant morbid consequences. The
thrombotic process initiating in a venous segment can, in the absence of anticoagula-
tion or in the presence of inadequate anticoagulation, propagate to involve more prox-
imal segments of the deep venous system, thus resulting in edema, pain, and immo-
bility. The most dreaded sequel to acute DVT is that of pulmonary embolism, a con-
dition of potentially lethal consequence. The late consequence of DVT, particularly of
the iliofemoral veins, can be CVI and ultimately post-thrombotic syndrome, as a re-
sult of valvular dysfunction in the presence of luminal obstruction.
For these reasons, understanding the pathophysiology, standardizing protocols
to prevent or reduce DVT, and instituting optimal treatment promptly all are critical
to reducing the incidence and morbidity of this unfortunately common condition.
Etiology
The triad of stasis, hypercoagulable state, and vessel injury all exist in most
surgical patients. It is also clear that increasing age places a patient at a greater risk,
with those older than 65 years representing a higher-risk population.
Stasis
Labeled fibrinogen studies in patients, as well as autopsy studies, have demon-
strated quite convincingly that the soleal sinuses are the most common sites of initia-
tion of venous thrombosis. The stasis may contribute to the endothelial cellular layer
contacting activated platelets and procoagulant factors, thereby leading to DVT. Sta-
sis, in and of itself, has never been shown to be a causative factor for DVT.
The Hypercoagulation State
Our knowledge of hypercoagulable conditions continues to improve, but it is
still undoubtedly embryonic. Should any of these conditions be identified, a treatment
regimen of anticoagulation is instituted for life, unless specific contraindications ex-
ist. It is generally appreciated that the postoperative patient, following major opera-
tive procedures, is predisposed to formation of DVT. After major operations, large
amounts of tissue factor may be released into the bloodstream from damaged tissues.
Tissue factor is a potent procoagulant expressed on the leukocyte cell surface as well
as in a soluble form in the bloodstream. Increases in platelet count, adhesiveness,
changes in coagulation cascade, and endogenous fibrinolytic activity all result from
physiologic stress, such as major operation or trauma, and have been associated with
an increased risk for thrombosis.
Venous Injury
It has been clearly established that venous thrombosis occurs in veins that are
distant from the site of operation; for instance, it is well known that patients undergo-
ing total hip replacement frequently develop contralateral lower extremity DVT.
In a set of elegant experiments, animal models of abdominal and total hip oper-
ations were used to study the possibility of venous endothelial damage distant from
the operative site. In these experiments, jugular veins were excised after the animals
were perfusion fixed. These experiments demonstrated that endothelial damage oc-
curred after abdominal operations and were much more severe after hip operations.
There were multiple microtears noted within the valve cusps that resulted in the expo-
sure of the subendothelial matrix. The exact mechanisms by which this injury at a dis-
tant site occurs and what mediators, whether cellular or humoral, are responsible are
not clearly understood, but that the injury occurs and occurs reliably is evident from
these and other studies.
Diagnosis
Incidence
Venous thromboembolism occurs for the first time in about 100 persons per
100,000 each year in the United States. This incidence increases with increasing age,
with an incidence of 0.5% per 100,000 at 80 years of age. More than two thirds of
these patients have DVT alone, and the rest have evidence of pulmonary embolism.
The recurrence rate with anticoagulation has been noted to be 6% to 7% in the ensu-
ing 6 months.
In the United States, pulmonary embolism causes 50,000 to 200,000 deaths per
year. A 28-day case fatality rate of 9.4% after first-time DVT and of 15.1% after first-
time pulmonary thromboembolism has been observed. Aside from pulmonary embo-
lism, secondary CVI (that resulting from DVT) is significant in terms of cost, morbid-
ity, and lifestyle limitation.
If the consequence of DVT, in terms of pulmonary embolism and CVI, is to be
prevented, the prevention, diagnosis, and treatment of DVT must be optimized.
Clinical Diagnosis
The diagnosis of DVT requires, to use an overused phrase, a high index of sus-
picion. Most are familiar with Homans' sign, which refers to pain in the calf on dorsi-
flexion of the foot. It is certainly true that although the absence of this sign is not a
reliable indicator of the absence of venous thrombus, the finding of a positive
Homans' sign prompts one to attempt to confirm the diagnosis. Certainly, the extent
of venous thrombosis in the lower extremity is an important factor in the manifesta-
tion of symptoms. For instance, most calf thrombi may be asymptomatic unless there
is proximal propagation. This is one of the reasons that radiolabeled fibrinogen test-
ing demonstrates a higher incidence of DVT than incidence studies using imaging
modalities. Only 40% of patients with venous thrombosis have any clinical manifes-
tations of the condition.
Major venous thrombosis involving the iliofemoral venous system results in a
massively swollen leg with pitting edema, pain, and blanching, a condition known as
phlegmasia alba dolens. With further progression of disease, there may be such mas-
sive edema that arterial inflow can be compromised. This condition results in a pain-
ful blue leg, the condition called phlegmasia cerulea dolens. With this evolution of
the condition, unless flow is restored, venous gangrene can develop.
Venography
Injection of contrast material into the venous system is obviously and under-
standably the most accurate method of confirming DVT and the location. The super-
ficial venous system has to be occluded with a tourniquet, and the veins in the foot
are injected for visualization of the deep venous system. Although this is a good test
for finding occlusive and nonocclusive thrombus, it is also invasive, subject to risks
of contrast, and requires interpretation with 5% to 10% error rate.

Impedance Plethysmography
Impedance plethysmography measures the change in venous capacitance and
rate of emptying of the venous volume on temporary occlusion and release of the oc-
clusion of the venous system. A cuff is inflated around the upper thigh until the elec-
trical signal has plateaued. When the cuff is deflated, there is usually rapid outflow
and reduction of volume. With a venous thrombosis, one notes a prolongation of the
outflow wave. It is not very useful clinically for the detection of calf venous throm-
bosis and in patients with prior venous thrombosis.
Fibrin, Fibrinogen Assays
The basis of fibrin or fibrinogen can be assayed by measuring the degradation
of intravascular fibrin. The D-dimer test measures cross-linked degradation products,
which is a surrogate of plasmin's activity on fibrin. It is shown that in combination
with clinical evaluation and assessment, the sensitivity exceeds 90% to 95%. The
negative predictive value is 99.3% for proximal evaluation and 98.6% for distal eval-
uation. In the postoperative patient, D-dimer is causally elevated due to surgery, and,
as such, a positive D-dimer assay for evaluating for DVT is of no use. However, a
negative D-dimer test in patients with suspected DVT has a high negative predictive
value, ranging from 97% to 99%.
Duplex Ultrasound
The modern diagnostic test of choice for the diagnosis of DVT is the duplex ul-
trasound, a modality that combines Doppler ultrasound and color-flow imaging. The
advantage of this test is that it is noninvasive, comprehensive, and without any risk of
contrast angiography. This test is also highly operator dependent, and this is one of
the potential drawbacks.
The Doppler ultrasound is based on the principle of the impairment of an ac-
celerated flow signal due to an intraluminal thrombus. A detailed interrogation begins
at the calf with imaging of the tibial veins and then proximally over the popliteal and
femoral veins. A properly done examination evaluates flow with distal compression
that results in augmentation of flow and with proximal compression that should inter-
rupt flow. If any segment of the venous system being examined fails to demonstrate
augmentation on compression, venous thrombosis is suspected.
Real-time B-mode ultrasonography with color-flow imaging has improved the
sensitivity and specificity of ultrasound scanning. With color-flow duplex imaging,
blood flow can be imaged in the presence of a partially occluding thrombus. The
probe is also used to compress the vein. A normal vein is easily compressed, whereas
in the presence of a thrombus, there is resistance to compression. In addition, the
chronicity of the thrombus can be evaluated based on its imaging characteristics,
namely, increased echogenicity and heterogeneity. Duplex imaging is significantly
more sensitive than indirect physiologic testing.
Magnetic Resonance Venography
With major advances in technology of imaging, magnetic resonance venogra-
phy has come to the forefront of imaging for proximal venous disease. The cost and
the issue of patient tolerance due to claustrophobia limit the widespread application,
but this is changing. It is a useful test for imaging the iliac veins and the IVC, an area
where duplex ultrasound is limited in its usefulness.
Prophylaxis
The patient who has undergone either major abdominal surgery or major ortho-
pedic surgery, has sustained major trauma, or has prolonged immobility (>3 days)
represents an elevated risk for the development of venous thromboembolism. The
specific risk factor analysis and epidemiologic studies dissecting the etiology of ve-
nous thromboembolism are beyond the scope of this chapter.
The methods of prophylaxis can be mechanical or pharmacologic. The simplest
method is for the patient to walk. Activation of the calf pump mechanism is an effec-
tive means of prophylaxis, as evidenced by the fact that few active people without
underlying risk factors develop venous thrombosis. A patient who is expected to be
up and walking within 24 to 48 hours is at low risk for developing venous throm-
bosis. The practice of having a patient “out of bed into a chair” is one of the most
thrombogenic positions that one could order a patient into. Sitting in a chair with the
legs in a dependent position causes venous pooling, which in the postoperative milieu
could easily be a predisposing factor in the development of thromboembolism.
The most common method of prophylaxis in the surgical universe has tradi-
tionally revolved around sequential compression devices, which periodically com-
press the calves and essentially replicate the calf bellows mechanism. This has clearly
reduced the incidence of venous thromboembolism in the surgical patient. The most
likely mechanism for the efficacy of this device is prevention of venous stasis. There
is some literature that suggests that fibrinolytic activity systemically is enhanced by a
sequential compression device. However, this is by no means established because
there are a considerable number of studies demonstrating no enhancement of fibrino-
lytic activity.
Another traditional method of thromboprophylaxis is the use of low-dose un-
fractionated heparin. The dose traditionally used was 5000 units of unfractionated
heparin every 12 hours. However, analysis of trials comparing placebo versus fixed-
dose heparin shows that the stated dose of 5000 units subcutaneously every 12 hours
is no more effective than placebo. When subcutaneous heparin is used on an every-8-
hour dosing, rather than every 12 hours, there is a reduction in the development of
venous thromboembolism.
More recently, a wealth of literature has revealed the efficacy of fractionated
low molecular weight heparin (LMWH) for prophylaxis and treatment of venous
thromboembolism. LMWH inhibits factor Xa and IIA activity, with the ratio of anti–
factor Xa to anti–factor IIA activity ranging from 1:1 to 4:1. LMWH has a longer
plasma half-life and has significantly higher bioavailability. There is much more pre-
dictable anticoagulant response than in fractionated heparin. No laboratory monitor-
ing is necessary because the partial thromboplastin time (PTT) is unaffected. A varie-
ty of analyses, including a major meta-analysis, have clearly shown that LMWH re-
sults in equivalent, if not better, efficacy with significantly less bleeding complica-
tions.
Comparison of LMWH with mechanical prophylaxis demonstrates superiority
of LMWH in reduction of the development of venous thromboembolic disease. Pro-
spective trials evaluating LMWH in head-injured and trauma patients have also
proved the safety of LMWH, with no increase in intracranial bleeding or major bleed-
ing at other sites. In addition, LMWH shows significant reduction in the development
of venous thromboembolism compared to other methods.
In short, LMWH is considered the optimal method of prophylaxis in moderate-
and high-risk patients. Even the traditional reluctance to use heparin in high-risk
groups such as the multiply injured trauma patient and the head-injured patient must
be re-examined, given the efficacy and safety profile of LMWH in multiple prospec-
tive trials.
Treatment
After a diagnosis of venous thrombosis has been established, a treatment plan
must be instituted. Complications of calf DVT include proximal propagation of
thrombus in up to one third of hospitalized patients and post-thrombotic syndrome. In
addition, untreated lower extremity DVT carries a 30% recurrence rate.
Any venous thrombosis involving the femoropopliteal system is treated with
full anticoagulation. Traditionally, the treatment of DVT centers on heparin treatment
to maintain the PTT at 60 to 80 seconds, followed by warfarin therapy to obtain an
International Normalized Ratio (INR) of 2.5 to 3.0. If unfractionated heparin is used,
it is important to use a nomogram-based dosing therapy. The incidence of recurrent
venous thromboembolism increases if the time to therapeutic anticoagulation is pro-
longed. For this reason, it is important to reach therapeutic levels within 24 hours. A
widely used regimen is 80 U/kg bolus of heparin, followed by a 15 U/kg infusion.
The PTT needs to be checked 6 hours after any change in heparin dosing. Warfarin is
started the same day. If warfarin is initiated without heparin, the risk for a transient
hypercoagulable state exists because protein C and S levels fall before the other vita-
min K–dependent factors are depleted. With the advent of LMWH, it is no longer
necessary to admit the patient for intravenous heparin therapy. It is now accepted
practice to administer LMWH to the patient as an outpatient, as a bridge to warfarin
therapy, which also is monitored on an outpatient basis.
The recommended duration of anticoagulant therapy continues to undergo evo-
lution. A minimum treatment time of 3 months is advocated in most cases. The recur-
rence rate is the same with 3 versus 6 months of warfarin therapy. If, however, the pa-
tient has a known hypercoagulable state or has experienced episodes of venous
thrombosis, then lifetime anticoagulation is required, in the absence of contraindica-
tions. The accepted INR range is 2.0 to 3.0; a recent randomized, double-blind study
confirmed that a goal INR of 2.0 to 3.0 was more effective in preventing recurrent
venous thromboembolism than a low-intensity regimen with a goal INR of 1.0 to 1.9.
Additionally, the low-intensity regimen did not reduce the risk for clinically im-
portant bleeding.
Oral anticoagulants are teratogenic and thus cannot be used during pregnancy.
In the case of the pregnant patient with venous thrombosis, LMWH is the treatment
of choice, and this is continued through delivery and can be continued postpartum as
indicated.
Thrombolysis
The advent of thrombolysis has resulted in increased interest in thrombolysis
for DVT. The purported benefit is preservation of valve function with subsequently
lesser chance of developing CVI. However, to date, few definitive, convincing data
exist to support the use of thrombolytic therapy for DVT.
One exception is the patient with phlegmasia in whom thrombolysis is advo-
cated for relief of significant venous obstruction. In this condition, thrombolytic ther-
apy probably results in better relief of symptoms and less long-term sequelae than
heparin anticoagulation alone. The alternative for this condition is surgical venous
thrombectomy. No matter which treatment is chosen, long-term anticoagulation is in-
dicated. The incidence of major bleeding is higher with lytic therapy.

DEEP VENOUS THROMBOSIS OF THE UPPER EXTREMITY


Upper extremity DVT is much less common than its lower extremity counter-
part, constituting only about 5% of all documented DVTs. Although not as common,
it is a serious problem; pulmonary embolism occurs in up to one third of all patients
with an upper extremity DVT. Upper extremity DVT usually refers to thrombosis of
the axillary or subclavian veins. The syndrome can be divided into two categories:
primary/idiopathic and secondary.
Primary etiologies include Paget-Schroetter syndrome and idiopathic upper ex-
tremity DVT. Patients with Paget-Schroetter syndrome develop effort thrombosis of
the extremity due to compression of the subclavian vein, the venous component of
thoracic outlet syndrome. A classic presentation involves a young athlete who uses
the upper extremity in a repetitive motion, such as swimming, which causes repetitive
extrinsic compression of the subclavian vein. In these patients, anatomic anomalies
such as a cervical rib or myofascial bands cause the venous compression. Plain films
are one of the first diagnostic tests used to confirm thoracic outlet syndrome. Treat-
ment with initial thrombolysis followed by first rib resection is standard of care. Idio-
pathic upper extremity DVT is sometimes eventually attributed to an occult malig-
nancy, and therefore a diagnosis of idiopathic upper extremity DVT warrants evalua-
tion for an undetected malignancy.
Secondary etiologies of upper extremity DVT are more common and include
indwelling central venous catheter, pacemaker, thrombophilia, or malignancy.
Classic findings on physical examination include unilateral swelling, pain, ex-
tremity discomfort, erythema, and a palpable cord. Diagnosis is confirmed by duplex
ultrasonography. Because the clavicle obscures the midportion of the subclavian vein,
venography or magnetic resonance venography may be required, and these constitute
second-line imaging modalities. Treatment of upper extremity DVT involves antico-
agulation therapy. Therapeutic dosing parameters are the same as for lower extremity
DVT. Long-term complications of upper extremity DVT include recurrence and post-
thrombotic syndrome. Post-thrombotic syndrome is treated with extremity elevation
and graduated elastic compression.
Superficial Thrombophlebitis
Superficial thrombophlebitis is a common disorder, diagnosed both in the hos-
pital and in the outpatient setting. In hospitalized patients, superficial thrombophlebi-
tis is usually due to an indwelling catheter. In the clinic, patients with thrombophlebi-
tis report common predisposing risk factors such as recent surgery, recent childbirth,
venous stasis, varicose veins, or intravenous drug use. Patients who deny any of the
aforementioned factors may be classified with idiopathic thrombophlebitis. In these
cases, care must be taken to ensure that the patient does not harbor an occult hyperco-
agulable state or an occult malignancy. Indeed, in 1876, Trousseau identified the phe-
nomenon of migratory thrombophlebitis and malignancy, particularly involving the
tail of the pancreas. Mondor's disease involves superficial thrombophlebitis of the su-
perficial veins of the breast. Diagnosis of superficial thrombophlebitis can be easily
made by physical examination of an erythematous palpable cord coursing along a su-
perficial vein, more commonly located along the lower extremities. Duplex ultraso-
nography is used if there is suspicion of proximal propagation into the deep venous
system. With this diagnosis of DVT, anticoagulation is indicated. If, however, throm-
bus abuts the saphenofemoral junction, treatment of this more elusive condition is
controversial. Some authors recommend serial ultrasound, others anticoagulation; an-
other alternative is operative ligation at the junction.
Treatment of localized, noncomplicated thrombophlebitis involves conserva-
tive therapy, which consists of anti-inflammatory medication and compression stock-
ings. When the thrombophlebitis involves clusters of varicosities, particularly in the
lower extremities, excision is indicated. Selective removal of the entire vein along its
course is only indicated in the rare instance of suppurative septic thrombophlebitis
after all other sources of sepsis have been excluded.

PULMONARY EMBOLISM
Pulmonary embolism (PE) is the blockage of pulmonary artery by fat, air, tu-
mor tissue, or thrombus that usually arises from the peripheral vein (most frequently
one of deep veins of the legs).
Aetiology and pathogenesis.
Thrombi most commonly develop in the veins of the lower leg from stasis and
a hypercoagulate state, and they propagate proximally to the deep veins of the leg and
pelvis. As these clots become larger and as the veins become larger, the propensity
for these clots to dislodge and embolized to the lungs increases. To reach the lungs,
thromboemboli travel through the right side of the heart. RA, right atrium; RV, right
ventricle; LA, left atrium; LV, left ventricle. When this occurs, a chain reaction of
events takes place:
 The pulmonary artery blood supply to those sections of the lung is occluded.
 Vasoactive agents are released with elevation of pulmonary vascular re-
sistance.
 A shunt develops as the pulmonary blood flow is redistributed.
 Pulmonary edema may occur.
 Alveolar dead space is increased, and gas exchange is impaired.
 Depending on the size of the thrombus or the patient's reaction to the embol-
ic event, right ventricular work is increased.
 With increased afterload, right ventricular dysfunction or failure may occur.
 Right ventricular hypokinesis with a normal arterial blood pressure is a poor
prognostic indicator.
 Paradoxical embolus from a patent foramen ovale may occur.

 Embolic burden of pulmonary embolism:


 Minor: <30% occlusion of pulmonary artery tree.
 Major: 30 – 50% occlusion of pulmonary artery tree.
 Massive: >50% occlusion of pulmonary artery tree.
Risk Factors
Although PE can occur in patients without any identifiable predisposing fac-
tors, one or more of these factors are usually identified (secondary PE). The propor-
tion of patients with idiopathic or unprovoked PE was about 20% in the International
Cooperative Pulmonary Embolism Registry (ICOPER). Venous thromboembolism
(VTE) is currently regarded as the result of the interaction between patient-related
and setting-related risk factors.
1) Patient-related predisposing factors are usually permanent:
a) Increasing age,
b) History of previous VTE,
c) Active cancer,
d) Paralytic stroke, extremity paresis,
e) Medical disorders causing prolonged bed rest,
f) Heart or acute respiratory failure,
g) Congenital or acquired thrombophilia
h) Hormone replacement therapy and
i) Oral contraceptive therapy
j) Varicose veins
k) Obesity
l) Pregnancy/antepartum
2) Setting-related predisposing factors are more often temporary:
a) Fracture (hip or leg)
b) Hip or knee replacement
c) Major general surgery
d) Major trauma
e) Spinal cord injury
f) Chemotherapy
g) Bed rest > 3 days
h) Immobility due to sitting (e.g. prolonged car or air travel)
i) Pregnancy/postpartum
j) Arthroscopic knee surgery
k) Central venous lines
Prevention
Prevention is considered in all patients having a major surgical procedure. All
hospitalized patients must be evaluated and stratified for their risk for pulmonary em-
bolism and the appropriate prophylaxis applied.
Unfractionated heparin is most commonly used for perioperative prophylaxis
and effectively reduces the rate of fatal pulmonary embolism. The dose is typically
5000 units three times daily and is continued until the patient is discharged and ambu-
latory.
LMWHs are an alternative to unfractionated heparin because of their character-
istics of improved bioavailability, improved absorption, once-daily injection, and re-
duced rates of heparin-induced thrombocytopenia.
Mechanical compression devices to stimulate fibrinolysis (from stimulation of
the venous endothelium) are effective in patients who are bed-bound; however, am-
bulatory patients are usually not compliant in their use within a general ward envi-
ronment.
Clinical Presentation
Clinical presentation may vary depending on degree of pulmonary artery ob-
struction. Signs and symptom range from mild anxiety with tachypnea and tachycar-
dia to profound dyspnea with cyanosis, hypotension and cardiac arrest.
Symptoms: dyspnoea, chest pain, cough, haemoptysis, syncope, systemic arte-
rial hypotension, oliguria, cold limb extremities, clinical signs of acute right heart
failure.
Signs: tachypnoea (≥ 20/ min), tachycardia (>100/min), sign of deep venous
thrombosis, fever (>38.5 C), cyanosis
Clinical signs, symptoms and routine laboratory tests do not allow the exclu-
sion or confirmation of acute PE, as they are neither sensitive nor specific, but in-
crease the index of its suspicion. The presence or absence of risk factors for VTE is
essential in the evaluation of the likelihood of PE. Moreover, it should be recognized
that the risk of PE increases with the number of risk factors present. However, PE
does occur frequently in individuals without any risk factors.
Physical Examination Findings
Physical examination finding include the sign of right ventricular dysfunction
such as: enlarged neck veins, an accentuated second pulmonary sound on cardiac ex-
amination, about 40% of patients with pulmonary embolism have right ventricular
dysfunction. The normal right ventricle with acute pulmonary embolism cannot toler-
ate a sustained mean pulmonary artery pressure of more than 40 mm Hg.
Diagnostic Studies
1. Chest X-Ray
The results are frequently normal. The most frequently encountered findings
(plate-like atelectasis, pleural effusion or elevation of a hemidiaphragm) are nonspe-
cific. Westermark's sign (decreased pulmonary vascular markings peripherally) or
Palla's sign (enlarged right descending pulmonary artery) may be present. However,
the chest X-ray is very useful in excluding other causes of dyspnoea and chest pain.
2. ECG
The result may be normal but will usually exhibit an abnormality such as sinus
tachycardia, an S1Q3T3 pattern, right ventricular hypertrophy with strain, right bun-
dle branch block, tachycardia, and T-wave inversion in the anterior chest leads (V1-
V4), and a QR pattern in lead V1. These may be helpful, particularly when of new
onset. Nevertheless, such changes are generally associated with the more severe
forms of PE and may be found in right ventricular strain of any cause.
3. Arterial Blood Gas with hypoxia and respiratory alkalosis suggestive of
pulmonary embolism. PE is generally associated with hypoxemia, but up to 20% of
patients with PE have a normal arterial oxygen pressure (PaO2) and a normal
alveolar-arterial oxygen gradient [D(A-a)O2].
4. D-dimer test highly sensitive but not specific, especially in postoperative
patient.
Plasma D-dimer, a degradation product of cross linked fibrin; D-dimer levels
are elevated in plasma in the presence of an acute clot because of simultaneous acti-
vation of coagulation and fibrinolysis. Hence, a normal D-dimer level renders acute
PE or DVT unlikely, i.e. the negative predictive value (NPV) of D-dimer is high. On
the other hand, although D-dimer is very specific for fibrin, the specificity of fibrin
for VTE is poor because fibrin is produced in a wide variety of conditions, such as
cancer, inflammation, infection, necrosis, dissection of the aorta, and the positive
predictive value (PPV) of D-dimer is low.
5. Ultrasound examination (Ultrasound study of leg veins, even if negative,
does not rule out pulmonary embolism).
6. Ventilation/perfusion (V/Q) scan:
Usually pivotal study in most hemodynamically stable patient with pulmonary
embolism. IV injection of technetium (Tc)-99 m labelled macroaggregated albumin
particles and ventilation studies, for which multiple tracers, such as xenon (Xe)-133
gas, Tc-99 m labelled aérosols or Tc-99 m-labelled carbon microparticles
(Technegas), can be used. If normal, the likelihood of pulmonary embolism is low. If
decreased perfusion is matched by normal ventilation, a high probability of pulmo-
nary embolism exists, and the patient receives treatment.
7. High resolution of Spiral CT Scan.
Uses of 120ml contrast and be completed in 30 sec. have a great specificity and
good sensitivity for central emboli but less accurate for peripheral emboli.
8. Echocardiography.
Hence, echocardiographic examination is not recommended as an element of
elective diagnostic strategy in haemodynamically stable, normotensive patients with
suspected PE. In patients with suspected high-risk PE presenting with shock or hypo-
tension, the absence of echocardiographic signs of RV overload or dysfunction prac-
tically excludes PE as a cause of haemodynamic instability.
9. Pulmonary Angiogram.

Medical Management
Treatment for pulmonary embolism is depends on embolic burden, hemody-
namic changes and patient’s baseline status. Patient should be placed on supplemental
oxygen and given fluids if no signs of congestive heart failure. Minor pulmonary em-
bolism and major pulmonary embolism with normal cardiopulmonary reserve usually
adequately treated with anticoagulant alone.
Patients with underlying cardiopulmonary disease may require caval interrup-
tion in addition. Patient with massive pulmonary embolism may require intubation
and inotropic support.
Thrombolysis or pulmonary embolectomy may be life saving for patients with
massive pulmonary embolism. Unless contraindicated, heparin may be initiated be-
fore diagnostic testing in patients with high index of suspicion was performed.
Anticoagulation:
 Unfractioned heparin:
o Usually IV bolus of 80 U/kg. Continuous infusion of 18 U/kg/h.
Check prothrombin time every 6 hour until stable in therapeutic
range (2.0 – 2.5 times control), then need daily or as needed.
 Warfarin (Coumadin):
o Can be started within first 24 hours once PTT therapeutic. Goal PT
time: In normal range 2.0 – 3.0. Heparin continued and overlapped
with warfarin until PT has been therapeutic for 2 conservative
days.
 Low molecular weight heparin (LWMH):
o Have similar antithrombotic effects as unfractionated heparin. Less
binding to plasma proteins yields more predictable dose response
and longer half-life.

Caval interruption:
The most widely used device is the Greenfield filter.
Indication:
1. Anticoagulation unsafe.
2. Major pulmonary embolism with hemodynamic instability.
3. Adjunctive procedure in patient undergoing pulmonary embolectomy.
4. Chronic pulmonary embolism in patient with cor pulmonale.
5. Prophylaxis for high risk patient (spinal cord injury or pelvic fracture).
Thrombolysis
Not often indicated because most do well with anticoagulant alone. May be as-
sociated with improved right ventricular function, pulmonary perfusion and function-
al status but this weighed against bleeding especially if recent surgery. If indicated,
thrombolytic agent maybe delivered peripherally or catheter – directed.
Pulmonary embolectomy
Surgical embolectomy sometimes indicated in patient with massive pulmonary
embolism and who are refractory to modalities described above. Catheter – based
techniques sometimes used but large series limited and specialized expertise neces-
sary.
Basic literatures:
1. Townsend: Sabiston Textbook of Surgery, 18th ed. – 500 p.
8. Oxford Textbook of Surgery (3-Volume Set) 2nd edition / Morris P. J., Wood
W. C. – 2000. - Oxford Press
9. Schwartz’s manual of surgery (8th edition) / Brunicardi F. - 2006 C. McGraw-
HILL Medical Publishing Division New York, Chicago, San Francisco, Lis-
bon, London, Madrid, Mexico City, Milan, New Delhi, San Juan, Seoul, Sin-
gapore, Sydney, Toronto.
10.Essentials of Surgery: Scientific Principles and Practice 2nd edition / Green-
field L. J., Mulholland M. W., Oldham K. T., Zelenock G. B., Lillimoe K. D.,
Oldham K. – 1997. - Lippincott Williams & Wilkins Publishers.

Additional literatures:
1. Torbicki A. et al. Guidelines on the diagnosis and management of acute pul-
monary embolism .The Task Force for the Diagnosis and Management of
Acute Pulmonary Embolism of the European Society of Cardiology (ESC) //
European Heart Journal – 2008. – Vol. 29, P. 2276–2315.

Tests for initial level of knowledge, keys for tests:


1. Earliest sign of deep vein thrombosis is:
A. Calf tenderness
B. Rise in temperature
C. Swelling of calf muscle
D. Homan’s sign
E. Varicose veins

2. A 64-year-old man is admitted 14 months after a femoropopliteal bypass


graft procedure with a cold foot and no graft pulse. Urokinase infusion was
started. Which of the following statements regarding management is true?
A. Clot lysis is accomplished in 25% of patients
B. After successful clot lysis, surgical revision of the opened graft should be consid-
ered only if early reocclusion occurs
C. Urokinase is less successful in lysing acute thromboses of prosthetic grafts than
those of vein grafts
D. Streptokinase is the preferred thrombolytic agent when treating graft occlusions
E. With optimal treatment, a 20% reocclusion rate is expected within 1 year

3. Cockett’s and Todd’s operation is for:


A. Burger’s disease
B. Lymphedema precoxa
C. AV fistula
D. Varicose veins
E. Deep veins thrombosis

4. The most reliable sign of deep vein thrombosis is:


A. Swelling of a limb distally
B. Positive Homan’s sign
C. Calf tenderness
D. Dilation of superficial veins
E. Teleangiectasia

5. Brodie-Trendelenberg test is for:


A. Testing sapheno-femoral junction incompetence
B. DVT
C. Mid thigh perforators in varicose veins
D. Calf Perforators
E. Thrombosis of superficial veins

6. White leg is due to:


A. Femoral vein thrombosis and lymphatic obstruction
B. Deep femoral vein thrombosis
C. Lymphatic obstruction only
D. None of the above
E. Superficial veins obstruction

7. Brodie-Trendlenburg test demonstrates:


A. Mid-thigh perforators
B. Deep vein thrombosis
C. Sapheno-femoral incompetence
D. Calf perforators
E. Superficial veins thrombosis

8. Most common source of pulmonary embolus:


A. Iliofemoral veins
B. Deep veins of leg
C. Deep veins of calf
D. Popliteal veins
E. Major saphenous vein

9. All of the following are correct about axillary vein thrombosis ex-
cept:
A. May be caused by a cervical rib
B. Treated with IV anticoagulant
C. Embolectomy is required in almost all cases
D. May occur following excessive exercise
E. Often is catheter-induced

10. Best method of diagnosing deep venous thrombosis is:


A. Venography
B. Duplex study
C. Plethysmography
D. Radionuclide scan
E. Rheovasography

Keys for tests


1 2 3 4 5 6 7 8 9 10
C E D C A B C C C B
Tests for final level of knowledge, keys for tests:

1. A 30-year-old woman in the last trimester of pregnancy suddenly develops


massive swelling of the left lower extremity from the inguinal ligament to the ankle.
The correct sequence of workup and treatment should be:
A. Venogram, bed rest, heparin
B. Impedance plethysmography, bed rest, heparin
C. Impedance plethysmography, bed rest, vena caval filter
D. Impedance plethysmography, bed rest, heparin, warfarin (Coumadin)
E. Clinical evaluation, bed rest, warfarin

2. A 68-year-old man is admitted to the coronary care unit with an acute my-
ocardial infarction. His post infarction course is marked by congestive heart failure
and intermittent hypotension. On the fourth hospital day, he develops severe
midabdominal pain. On physical examination, blood pressure is 90/60 mm Hg and
pulse is 110 beats/min and regular; the abdomen is soft with mild generalized tender-
ness and distention. Bowel sounds are hypoactive; stool occult blood test is positive.
The next step in this patient’s management should be:
A. Barium enema
B. Upper gastrointestinal series
C. Angiography
D. Ultrasonography
E. Celiotomy

3. Prophylactic regimens of documented benefit in decreasing the risk of


postoperative thromboembolism include:
A. Early ambulation
B. External pneumatic compression devices placed on the upper extremities
C. Elastic stockings
D. Leg elevation for 24 h postoperatively
E. Dipyridamole therapy for 48 h postoperatively

4. Ligation of injured major peripheral veins is rarely preferable to repair,


but may be justified for which reason?
A. In severe popliteal vascular injuries, venous ligation leads to a decreased amputa-
tion rate following successful arterial reconstruction when compared with com-
bined arterial and venous repair
B. Venous ligation leads to a decreased incidence of chronic venous insufficiency
when compared with venous repair
C. Venous ligation leads to a decreased operative time in patients with multiple inju-
ries or severe trauma when compared with venous repair
D. In the presence of extensive associated soft tissue injury, venous return is already
sufficiently impaired to render venous repair pointless
E. Even though ligated veins thrombose, they often recanalize
11. Which is true for patients with confirmed deep vein thrombosis of the
calf with phlebography?
A. Can expect asymptomatic recovery if treated promptly with anticoagulants
B. May be effectively treated with low dose heparin
C. May be effectively treated with pneumatic compression stockings
D. May be effectively treated with acetylsalicylic acid
E. Are at risk for significant pulmonary embolism

6. A 25-year-old woman presents to the emergency room complaining of


redness and pain in her right foot up to the level of the midcalf. She reports that her
right leg has been swollen for at least 15 years, but her left leg has been normal. On
physical examination she has a temperature of 39°C (102.2°F). The left leg is normal.
The right leg is not tender, but it is swollen from the inguinal ligament down and
there is an obvious cellulitis of the right foot. The patient’s underlying problem is
A. Popliteal entrapment syndrome
B. Acute arterial insufficiency
C. Primary lymphedema
D. Deep venous thrombosis
E. None of the above
7. Indications for placement of the Greenfield filter include:
A. Recurrent pulmonary embolus despite adequate anticoagulation therapy
B. Axillary vein thrombosis
C. Pulmonary embolus in a patient with a perforated duodenal ulcer
D. Pulmonary embolus due to deep vein thrombosis of the lower extremity that oc-
curs 2 wk postoperatively
E. Pulmonary embolus in a patient with metastatic pancreatic carcinoma

8. Which of the following statements concerning popliteal aneurysms is


true?
A. Surgery should be performed only if the patient is symptomatic
B. Limb loss is a definite risk in the untreated patient
C. The contralateral limb is affected in a similar fashion in over 75% of cases
D. Embolization is unlikely
E. Bleeding into the leg is the most common presentation

9. Which statement regarding contrast venography is true?


A. It is more accurate than Doppler analysis and B-mode ultrasound (duplex scan) at
detecting thrombi in the deep veins responsible for pulmonary emboli
B. It identifies incompetent deep, superficial, and perforating veins
C. It is totally noninvasive, painless, and safe
D. It is easily performed in a vascular laboratory or radiology suite or at the bedside
E. It is particularly sensitive in identifying the proximal extent of an iliofemoral
thrombus

10. An obese 50-year-old woman undergoes a laparoscopic cholecystectomy.


In the recovery room she is found to be hypotensive and tachycardic. Her arterial
blood gases reveal a pH of 7.29, partial pressure of oxygen of 60 kPa, and partial
pressure of CO2 of 54 kPa. The most likely cause of this woman’s problem is
A. Acute pulmonary embolism
B. CO2 absorption from induced pneumoperitoneum
C. Alveolar hypoventilation
D. Pulmonary edema
E. Atelectasis from high diaphragm

Keys for tests

1 2 3 4 5 6 7 8 9 10
B C B C E C A B B C

Tasks for final level of knowledge

1. A 45-year-old male having a long history of cigarette smoking presented with gan-
grene of left foot. An amputation of the left foot was done. Representative sections
from the specimen revealed presence of arterial thrombus with neutrophilic infiltrate
in the arterial wall. The inflammation also extended into the neighbouring veins and
nerves. What is the most probable diagnosis?
The answer is thromboangitis obliterans

2. A 60-year-old man is admitted to the intensive cardiac care unit with a large ante-
rior wall myocardial infarction. On his second hospital day he begins to complain of
the sudden onset of numbness in his right foot and an inability to move his right foot.
On physical examination the right femoral, popliteal, and pedal pulses are no longer
palpable. Vascular consultation is obtained. Diagnosis of acute arterial embolus is
made. What is the source of embolus?

The answer is the source of the embolus is most likely the left ventricle

3. A 25-year-old woman presents to the emergency room complaining of redness and


pain in her right foot up to the level of the midcalf. She reports that her right leg has
been swollen for at least 15 years, but her left leg has been normal. On physical ex-
amination she has a temperature of 39°C. The left leg is normal. The right leg is not
tender, but it is swollen from the inguinal ligament down and there is an obvious cel-
lulitis of the right foot. The patient’s underlying problem is
The answer is primary lymphedema

4. A 55-year-old man with recent onset of atrial fibrillation presents with a cold,
pulseless left lower extremity. He complains of left leg paresthesia and is unable to
dorsiflex his toes. Following a successful popliteal embolectomy, with restoration of
palpable pedal pulses, the patient is still unable to dorsiflex his toes. The next step in
management should be

The answer is immediate fasciotomy

5. Laura, a 59-year-old woman, has a left femoral venous thrombosis during pregnan-
cy 30 year ago. The left greater saphenous vein had been stripped at age 21. She now
presents with a large non-healing ulceration over the medial left calf, which has con-
tinuously progressed despite bed rest, elevation, and use of a support stocking. De-
scending phlebography of the left leg demonstrates a patent deep venous system, with
free flow of dye from the groin to foot. The first profunda femoris valve is competent.
Appropriate management must include:

The answer is division of the superficial femoral vein in the groin and transposi-
tion of its distal end into the profunda femoris vein below the level of the compe-
tent profunda valve
Study guide #19.1
“Diseases of veins. The vena cava superior syndrome. The vena cava inferior
syndrome. Causes, diagnostics, differential diagnostics, medical tactics. Lym-
phoedema of special sites.”

Overview
Superior vena cava syndrome (SVCS) was first described in l757 in a patient
with a syphilitic lesion of the aorta. The causes of SVCS have changed since that
time. In the 1950s, SVCS was primarily caused by aortic aneurysm and infections
such as tuberculosis and fibrous mediastinitis. In the 1980s and 1990s, malignant dis-
orders have become the dominant cause of SVCS. In most patients with SVCS, pri-
mary malignancies of the mediastinum are the causative factor. Benign disorders ac-
count for less than 10% of cases of SVCS. Modern antibiotic treatment of infectious
disorders is postulated to be the cause of the changing aetiologies of SVCS. This
study guide have analysed the anatomy of the superior vena cava and the pathophysi-
ology, malignant and benign causes, clinical presentation, and diagnosis of SVCS.
Treatment and prognosis are also discussed.
Inferior vena cava syndrome (IVCS) is a result of obstruction of the inferior vena
cava. It can be caused by invasion or compression by a pathological process or by
thrombosis in the vein itself. Epidemiological data is elusive owing to the wide varie-
ty of clinical presentation. In the U.S., incidence is estimated to be at 5-10 cases per
100'000 per year. Causes of the IVCS: obstruction by deep vein thrombosis or tu-
mors.
Venous Anatomy
Veins are part of a dynamic and complex system that returns venous blood to the
heart against the force of gravity in an upright individual. Venous blood flow is de-
pendent upon multiple factors such as gravity, venous valves, the cardiac and respira-
tory cycles, blood volume, and the calf muscle pump. Alterations in the intricate bal-
ance of these factors can result in venous pathology.
Structure of Veins
Veins are thin-walled, highly distensible, and collapsible structures. Their struc-
ture specifically supports their two primary functions of transporting blood toward the
heart and as a reservoir for preventing intravascular volume overload. The venous in-
tima is composed of a nonthrombogenic endothelium with an underlying basement
membrane and an elastic lamina. The endothelium produces endothelium-derived re-
laxing factor and prostacyclin, which help maintain a nonthrombogenic surface
through inhibition of platelet aggregation and by promoting platelet disaggregation. 1
Circumferential rings of elastic tissue and smooth muscle located in the media of the
vein allow for changes in vein caliber with minimal changes in venous pressure.
When an individual is upright and standing still, the veins are maximally distended
and their diameters may be several times greater than if the individual was in a hori-
zontal position.
Unidirectional blood flow is achieved with multiple venous valves. The number
of valves is greatest below the knee and decreases in number in the more proximal
veins. The inferior vena cava (IVC), the common iliac veins, the portal venous sys-
tem, and the cranial sinuses are valveless. Each valve is made of two thin cusps con-
sisting of a fine connective tissue skeleton covered by endothelium. Venous valves
close in response to cephalic-to-caudal blood flow at a velocity of at least 30 cm/s.
Upper Extremity Veins
As in the lower extremity, there are deep and superficial veins in the upper ex-
tremity. Deep veins of the upper extremity are paired and follow the named arteries in
the arm. Superficial veins of the upper extremity are the cephalic and basilic veins
and their tributaries. The cephalic vein originates at the lateral wrist and courses over
the ventral surface of the forearm. In the upper arm, the cephalic vein terminates in
the infraclavicular fossa, piercing the clavipectoral fascia to empty into the axillary
vein. The basilic vein runs medially along the forearm and penetrates the deep fascia
as it courses past the elbow in the upper arm. It then joins with the deep brachial
veins to become the axillary vein. The median cubital vein joins the cephalic and the
basilic veins on the ventral surface of the elbow.
The axillary vein becomes the subclavian vein at the lateral border of the first rib.
At the medial border of the scalenus anterior muscle, the subclavian vein joins with
the internal jugular vein to become the brachiocephalic vein. The left and right bra-
chiocephalic veins join to become the superior vena cava, which empties into the
right atrium.
Pathophysiology
Any pathology of the previously noted structures produces external pressure on
the superior vena cava or internally obstructs the vessel as a result of either throm-
bosis or direct invasion by the disease process. In addition, enlargement of the lymph
nodes may also compress the superior vena cava. In most cases, extrinsic compres-
sion develops gradually and the symptoms are initially mild because collateral circu-
lation has sufficiently developed. If the obstruction develops suddenly, as in the case
of a malignancy, the collateral circulation has not developed and the patient rapidly
becomes symptomatic. Thrombosis of the superior vena cava may progress to involve
all the major collateral vessels, and the resulting thrombosis eventually undergoes fi-
brosis that results in permanent occlusion of the superior vena cava. In this case,
thrombolytic therapy is of little or no benefit unless the treatment is directed at the
primary cause of SVCS.
Thrombosis develops when the balance between clotting and fibrinolysis is shift-
ed to favor coagulation. The thrombotic process can develop through either an intrin-
sic or extrinsic pathway. The extrinsic pathway begins with local cell injury which
leads to the release of tissue factor and exposure of the collagen cell matrix which
promotes platelet aggregation. Factor VII becomes activated as well as factors IX and
X. Coagulation proteins assemble on the platelet membrane surface. Adhesion of the
platelets is stimulated by von Willebrand's factor while platelets become adherent to
each other in the presence of fibrinogen. This results in the formation of a platelet
plug. In the presence of the prothrombinase complex (factors Xa and Va, calcium,
and prothrombin), thrombin is catalyzed resulting in the cleavage of fibrin peptides A
and B and the activation of factor XIII, which in turn catalyzes the cross-linking of
fibrin monomers. The net result is a firm clot in the presence of activated platelets
and factors Va and VIIIa.
Coagulation develops along the intrinsic pathway through contact activation,
when factor XI is converted to XIa, which in turn catalyzes the activation of factor IX
to IXa and activates the sequence converting factor X to Xa. Acting together on plate-
lets, factors VIII, IXa, X, and calcium catalyze the activation of factor X to Xa and
merge with the prothrombinase complex.
Several anticoagulant mechanisms balance the clotting. Antithrombin III stops
the cleavage of fibrinopeptides A and B, stops the activation of factors V and VIII,
and inhibits platelet aggregation and activation as well as factors IXa, Xa, and XIa.
Activated protein C inactivates factors Va and VIIIa and reduces the acceleration of
the rate of thrombin formation. Heparin cofactor II regulates thrombin formation. Ex-
trinsic pathway inhibitor is a rapid acting agent also known as tissue factor pathway
inhibitor which inactivates tissue factor VIIa activation of factor X; however, it does
not affect factor IX.
Plasmin is the major fibrinolytic enzyme, whose substrates are fibrin, fibrinogen,
and coagulation factors which act to interrupt platelet adhesion. Plasmin degrades
both circulating and clot-bound fibrin creating two fragments: E and D. It is the D
fragment that is measured in the D-dimer ELISA test and which serves as a marker of
fibrinolysis and thrombosis. Plasmin is the product of the interaction of plasminogen
and tissue plasminogen activator. Plasminogen is activated exogenously by streptoki-
nase or urokinase, indigenously by tissue plasminogen activator, and by the intrinsic
factors. The balance within this exquisite system depends on thrombus formation,
thrombus inhibition, and fibrinolysis.
From the historical perspective, Virchow described the risk of developing
throboembolism as due to some imbalance in these factors favouring coagulation.
Coon conducted an extensive autopsy study at the University of Michigan in the
1950s to identify premorbid factors that resulted in an increased risk of deep vein
thrombosis among patients admitted to hospital. This work was modified when the
study was repeated at a later time. Among the factors found to be associated with an
increased risk are heart disease, cancer, trauma, and obesity. While the risk increased
with age, other factors biased this finding and age was not considered to be an inde-
pendent factor. Surgery was found to have an additive effect on the risk. These find-
ings and their additive effects are also supported by the work of others.
The importance of understanding the factors resulting in an increased incidence
of deep vein thrombosis is the ability to provide cost-effective prophylactic interven-
tions for individual patients. However, a great deal of poorly developed data exists
regarding these factors. While the work cited earlier is carefully developed using
sound epidemiologic principles, not all work meets this standard. Many reports have
been drawn from limited populations and lack statistical power. Well validated lists
of risk factors for specific patient populations are sorely needed.
Risk factors for thromboembolic disease documented by consensus reports:
obesity, varicose veins, malignancy, trauma or operation > 2h, increasing age, prior
deep vein thrombosis, embolism, birth control pills, postpartum, immobilization, sep-
sis, pregnancy, stasis.
MALIGNANT AND BENIGN CAUSES
Malignant Causes
The most common malignancy associated with SVCS is lung cancer, followed by
lymphomas and metastatic tumors to the mediastinum (Table 1).
Table 1. Malignant Causes of Superior Vena Cava Syndrome
Malignancy Histological Subtypes
Lung cancer Small cell
Large cell
Adenocarcinoma
Undifferentiated
Lymphoma Lymphoblastic
Lymphocytic
Mixed
Nodular
Non-Hodgkin’s
Metastatic tumour Breast
Testicular
Other malignancy Kaposi’s sarcoma

Lung tumors. Approximately 5% to 15% of patients with bronchogenic carci-


noma develop SVCS. The syndrome is four times more likely to occur in patients
with right-sided tumors of the thoracic cavity. These lesions often cause obstructive
pneumonitis, which usually occurs with involvement of right hilar and mediastinal
lymph nodes. Among the types of lung cancers, all histologic cell types are associated
with SVCS. Lymphomas and metastatic tumors. The second group of malignancies
that commonly cause SVCS are lymphomas, especially non-Hodgkin’s lymphoma.
SVCS occurs in 3% to 8% of patients with lymphoma. The lymphoma usually origi-
nates in the anterior mediastinum and produces SVCS by external compression. Met-
astatic tumors, more commonly breast and testicular tumors, cause SVCS in approx-
imately 5% to 20% of patients.
Benign Causes
Many benign disorders can cause SVCS (Table 2).
Table 2. Benign Causes of Superior Vena Cava Syndrome

Infectious
Tuberculosis
Histoplasmosis
Actinomycosis
Syphilis
Pyogenic
Tumors
Cystic hygroma
Substernal goiter
Teratoma
Thymoma
Dermoid cyst
Cardiac
Atrial myxoma
Pericarditis
Intrapericardial band
Mitral stenosis
Complication of central catheter
Complication of congenital heart surgery
Complication of total parenteral nutrition line
Vascular
Aortic aneurysm
Arteriovenous fistula
Polycythemia
Idiopathic thrombosis
Other causes
Sarcoidosis
Post irradiation
Mediastinal hematoma
Pneumothorax
Behcet’s disease

As noted previously, however, benign disorders account for less than 10% of cas-
es of SVCS. Benign causes of SVCS include thymoma, cystic hygroma, benign cystic
teratoma, substernal thyroid goiter, dermoid cyst, and post irradiation therapy. Infec-
tions notable for causing SVCS are tuberculosis, histoplasmosis, actinomycosis,
syphilis, and pyogenic infections. In addition, the increased current use of invasive
monitoring devices, such as central lines, cardiac pacemakers, catheters for total par-
enteral nutrition, and Swan-Ganz monitoring devices, is associated with increasing
reports of thrombosis of the superior vena cava. Finally, aneurysms of the aorta and
aortic branches are occasionally responsible for causing SVCS.
Causes of the inferior vena cava syndrome (IVCS):
• Obstruction by deep vein thrombosis or tumors (most commonly renal cell
carcinoma)
• Compression through external pressure by neighbouring structures or tumors,
either by significantly compressing the vein or by promoting thrombosis by causing
turbulence by disturbing the blood flow. This is quite common during the third tri-
mester of pregnancy when the uterus compresses the vein in the right side position.
• Iatrogenic causes may be suspected in patients with a medical history of liver
transplantation, vascular catheters, dialysis and other invasive procedures in the vicin-
ity
• Budd-Chiari syndrome.
Essential properties of lower limb veins
Structure
The veins are specifically designed to allow flow in one direction. The presence
of numerous valves prevents venous reflux. The valves are supported by the vein wall
and their integrity depends on the vein having sufficient strength to prevent dilatation.
In the upright position the veins distend, but will collapse when the person lies flat.
The vein is lined with endothelium, providing a non-thrombogenic surface. The endo-
thelial layer may be damaged as a result of long periods of stasis, hypoxia, and an in-
teraction with the white cells. Such damage will enhance thrombogenesis, but the
veins themselves have a protective mechanism resulting in the production of fibrinol-
ysins capable of dissolving clot.
Arrangement of deep veins
The deep veins lie beneath the deep fascia; the superficial veins lie superficial to
the deep fascia. Some of the veins act as conduits and others, venous sinuses located
within the muscles, form an important part of the pumping mechanism
Veins as pumping chambers
Venous pumps are described in the foot, the calf, and the thigh. On walking and
on muscle contraction, the veins are compressed. The valves ensure the blood is
moved towards the heart and this mechanism is important in assisting venous return.
Damage to the joints, the muscles, or the valves will interfere with this mechanism.
With an impaired venous pump, normal venous return does not take place; venous
pressure, instead of falling, will remain the same during exercise. If there is evidence
of venous obstruction, then during exercise the venous pressure will rise. This has
pathophysiological consequences—the development of skin changes and ulceration.
Superficial and perforating veins
The superficial veins (superficial to the deep fascia) act as conduits taking blood
from the surface to the deep veins via perforating veins. Two clearly identifiable sys-
tems with free interconnection between them can be identified. One the long saphe-
nous vein draining the inner leg to the groin, the other the short saphenous vein drain-
ing the back of the calf to the popliteal vein behind the knee. It is dilatation of these
veins that plays an important part in temperature regulation. Gross dilatation will lead
to the development of varicose veins. Both systems have valves along their length,
becoming more numerous in the lower leg.
Perforating veins connect the superficial veins to the deep veins. In addition to
the connections between the long saphenous and common femoral vein, and the short
saphenous and the popliteal vein, there are estimated to be over 60 other sites of
communication. The perforating veins allow flow from the superficial to the deep
system. In some perforating veins, valves can be identified; in others the inward flow
seems to be regulated by muscle contraction. The role of perforating veins in the cau-
sation of venous problems remains controversial.
Collateral flow
Collateral veins will develop when other veins become obstructed; occlusion of
the femoral vein may result in dilatation of the superficial veins providing an alterna-
tive drainage mechanism. Patients who develop occlusion of the iliac vein following
thrombosis get dilated superficial veins in the region of the groin.
Venous physiology
Four-fifths of the blood in the circulation is in the veins. The veins have an im-
portant role in regulating the capacity of the circulation and contraction of the main
veins provides the body's initial response to blood loss. Paralysis of the veins result-
ing in venous pooling can lead to postural hypotension and fainting. Blood flows
through the veins as a result of arterial pressure across the capillary bed, muscular ve-
nous pumps, and the effect of gravity.
The pressure measured in the veins (mmHg (millimetres of mercury) will depend
on the position of the patient. With them lying and the foot slightly elevated, the
pressure in the veins will be zero. When standing, the pressure measured in the
veins is equivalent to the weight of a column of blood extending from the foot to the
heart. The pressure measured in the foot with a patient standing still may be equiva-
lent to 100 mmHg. If the patient then exercises, the pressure will fall to under 20
mmHg. This fall is caused by the venous muscle pumps and depends on normally
functioning veins, active muscles, and normal joints. Damage to the system will result
in no fall in venous pressure on exercise.
Venous occlusion following thrombosis of the major axial vein (femoral iliac
vein) may result in an increase in venous pressure on exercise, due to venous outflow
obstruction.

Educational aims
47. To collect the anamnesis and to spend clinical inspection sick vena cava supe-
rior syndrome and vena cava inferior syndrome.
48. To know an aetiology and pathogenesis vena cava superior syndrome and vena
cava inferior syndrome.
49. To know a clinical picture vena cava superior syndrome and vena cava inferior
syndrome.
50. To know complications, conditioned by the syndromes of vena cava superior
syndrome and by the vena cava inferior syndrome.
51. To make the plan of inspections of patients with vena cava superior syndrome
and vena cava inferior syndrome.
52. To make the plan to inspection of patients with syndrome of vena cava superior
and syndrome of vena cava inferior.
53. To define indications to surgical treatment of patients with syndrome of vena
cava superior and syndrome of vena cava inferior.
54. To define contraindications to surgical treatment of patients with syndrome of
vena cava superior and syndrome of vena cava inferior.
55. To appoint of conservative treatment of patients with syndrome of vena cava
superior and syndrome of vena cava inferior.
56. Methods of prophylaxis of veins cava syndromes.
57. To estimate work capacity of patients with syndrome of vena cava superior and
syndrome of vena cava inferior.
A student must know:
1. Surgical anatomy of the veins cava system, Normal venous function.
2. Pathophysiology and classification of syndrome of vena cava superior and syn-
drome of vena cava inferior illness.
4. Risk factors and aetiology of veins cava syndromes.
5. Methods of examination of patients with veins cava syndromes.
6. Symptoms and signs of veins cava syndromes.
7. Clinical examination and tests.
8. Venography and Duplex ultrasonography.
9. Comparison of diagnostic methods.
10. Conservative treatment and operative treatment.
11. Indications and contraindications to the surgical interference and choice of the
method of the operation of veins cava syndromes.
12. Planning of the operation for operative thrombectomy.
14. Other treatment approaches.
A student must be able to:
34.Take anamnesis carefully.
35.Make diagnosis correctly.
36.Order additional examination.
37.To establish pathogenetic factors of veins thrombosis.
38.To diagnose complications of venous disease.
39.To make the plan of inspection sick of venous disease.
40.To estimate data of tool methods of inspection at patients veins illness (ultrasonic
research, ultrasonic duplex scanning with colour mapping, phlebography).
41.To carry out differential diagnostics of venous disease.
42.To establish indications for surgical treatment and conservative treatment
43.To estimate efficiency of treatment of patients and work capacity of patients.
44. Prophylaxis.
Terminology
Term Definition
is characterized by gradual, insidious compression
Syndrome of vena cava superior
or obstruction of the superior vena cava.
Syndrome of vena cava inferior is a result of obstruction of the inferior vena cava.
is the end product of two interrelated processes, ac-
Thrombosis tivation of platelets and of the blood coagulation
pathway.
Virchow's triad are stasis, vessel wall injury, hypercoagulability

Content
Syndrome of vena cava superior is characterized by gradual, insidious com-
pression or obstruction of the superior vena cava. The most common etiology of
SVCS is related to malignancy such as bronchogenic cancer and lymphoma. Other
cases have a variety of causes such as chronic infections and chronic inflammations
that involve the mediastinum. Spontaneous SVC thrombosis is also known to occur
among rare causes.
Syndrome of vena cava inferior is a result of obstruction of the inferior vena
cava. It can be caused by invasion or compression by a pathological process or by
thrombosis in the vein itself. Causes of the IVCS: obstruction by deep vein throm-
bosis or tumors (renal cell carcinoma); compression through external pressure by
neighbouring structures or tumors, either by significantly compressing the vein or by
promoting thrombosis by causing turbulence by disturbing the blood flow; the third
trimester of pregnancy when the uterus compresses the vein in the right side position;
Iatrogenic causes in patients with a medical history of liver transplantion, vascular
catheters, Budd-Chiari syndrome.
Surgical anatomy of the veins cava
The superior vena cava is a thin-walled, readily compressible vessel that
transmits blood to the heart at low pressure. The superior vena cava is located in the
middle mediastinum and is encircled by rigid structures, including the trachea, right
bronchus, aorta, thymus, and pulmonary artery. The superior vena cava extends ap-
proximately 8 cm from the innominate vein to the right atrium. The distal 2 cm of the
superior vena cava are within the pericardial sac. The azygous vein enters the superior
vena cava posteriorly and is a significant venous collateral channel. Encircling the
superior vena cava are subcarinal, perihilar, and paratracheal lymph nodes. These
nodes drain the right lung and the lower lobe of the left lung.
The inferior venacava conveys blood to the right atrium from all the structures
below the diaphragm. It is formed by the union of common iliac veins at fifth lumbar
vertebral level. Then it ascends up anterior to the vertebral column, passing through
the posterior surface of the liver, pierces the diaphragm and ascends up and reaches
the inferoposterior part of the right atrium. It has got 2 parts - 1. Abdominal part; 2.
Thoracic part.
Lower extremity veins are divided into superficial, deep, and perforating veins.
The superficial venous system lies above the uppermost fascial layer of the leg and
thigh and consists of the greater saphenous vein (GSV) and lesser saphenous vein
(LSV) and their tributaries. The deep veins follow the course of major arteries in the
extremities. In the lower leg, paired veins parallel the course of the anterior and pos-
terior tibial and peroneal arteries and join behind the knee to form the popliteal vein.
Venous bridges connect the paired veins in the lower leg. The popliteal vein contin-
ues through the adductor hiatus to become the femoral vein. In the proximal thigh, the
femoral vein joins with the deep femoral vein to form the common femoral vein. In
the groin, the common femoral vein lies medial to the common femoral artery. The
common femoral vein becomes the external iliac vein at the inguinal ligament. Ve-
nous sinuses are thin-walled, large veins located within the substance of the soleus
and gastrocnemius muscles. These sinuses are valveless and are linked by valved,
small venous channels that prevent reflux. A large amount of blood can be stored in
the venous sinuses. With each contraction of the calf muscle bed, blood is pumped
out through the venous channels into the main conduit veins to return to the heart.
Etiology and pathogenesis
Etiological factors of SVCS have changed 1950s, SVCS was primarily caused by
aortic aneurysm and infections such as tuberculosis and fibrous mediastinitis. In the
1980s and 1990s, malignant disorders have become the dominant cause of SVCS. In
most patients with SVCS, primary malignancies of the mediastinum are the causative
factor. Benign disorders account for less than 10% of cases of SVCS. Modern antibi-
otic treatment of infectious disorders is postulated to be the cause of the changing eti-
ologies of SVCS.
Inferior vena cava syndrome (IVCS) is a result of obstruction of the inferior ve-
na cava. It can be caused by invasion or compression by a pathological process or by
thrombosis in the vein itself. Causes of the IVCS: compression through external pres-
sure by neighbouring structures or tumors, either by significantly compressing the
vein or by promoting thrombosis by causing turbulence by disturbing the blood flow
(this is quite common during the third trimester of pregnancy when the uterus com-
presses the vein in the right side position); iatrogenic causes may be suspected in pa-
tients with a medical history of liver transplantion, vascular catheters, dialysis and
other invasive procedures in the vicinity; Budd-Chiari syndrome.
Clinical symptoms and signs of vena cava superior syndrome
The typical symptoms of SVCS are most obvious when obstructive disease is
almost complete. Patients with SVCS most often present with complaints of facial
edema and erythema, swelling of the neck and/or arms, and visible dilatation of the
veins in the upper extremity. Patients with SVCS may also complain of dyspnea, per-
sistent cough, and orthopnea. As the disease progresses, the symptoms may include
hoarseness, periorbital edema, dysphagia, headaches, dizziness, syncope, lethargy,
and chest pain. Other findings may include confusion and laryngeal and/or glossal
edema. In some cases, the nerves that cross the superior mediastinum (i.e., vagus and
phrenic nerves) are affected by SVCS. This nerve involvement can lead to hoarseness
and paralysis of the diaphragm. These symptoms may be worsened by positional
changes such as bending forward, stooping, or lying down. Patients with SVCS and
vagus or phrenic nerve involvement find significant symptom relief when they are in
an upright position, and many of these patients sleep in a chair to avoid dyspnea.
The venous hypertension associated with SVCS can sometimes produce cerebral
vessel thrombosis and hemorrhage with dire results. Of all the symptoms of SVCS,
the most life-threatening complications are cerebral or laryngeal edema.
DIAGNOSIS
The diagnosis of SVCS can be made simply on physical examination. In cases in
which the extent of disease is minimal, the physical findings may not be prominent
and the diagnosis may be more difficult to establish. Today, establishing the underly-
ing diagnosis and etiology of SVCS has become more important because certain dis-
orders that cause SVCS may be more amenable to specific treatment regimens. For
example, small cell lung carcinoma and lymphoma respond dramatically to chemo-
therapy/irradiation, whereas thrombosis from a central line catheter does not respond
to this treatment.
Findings on chest radiography in patients with superior vena cava syn-
drome:
Mediastinal widening, pleural effusion(s), right hilar mass, bilateral lung infil-
trates, cardiomegaly, Calcified paratracheal lymph nodes, Anterior mediastinal mass
Laboratory Studies
Chest radiography. The initial diagnostic test for suspected SVCS is chest radi-
ography. Although this test is not specific for SVCS, chest radiography may be help-
ful in identifying the cause of the disorder. Findings on chest radiography that may be
helpful include widening of the superior mediastinum, pleural effusions, and a hilar
or mediastinal mass, usually on the right side. These radiologic findings usually sug-
gest an underlying malignancy, whereas calcified lymph nodes may be more predic-
tive of granulomatous disease. However, the results of chest radiography may appear
normal despite an obstruction in the superior vena cava. In the absence of previous
catheterization or surgery, a normal result on chest radiography in a patient with
SVCS is almost pathognomonic of chronic fibrous mediastinitis.
Contrast venography. The extent and site of obstruction as well as the nature of
obstruction must be identified when SVCS is diagnosed. Identification of these fea-
tures may be achieved by a number of radiologic imaging studies. Contrast venogra-
phy can provide information regarding the patency of the superior vena cava, the de-
gree of superior vena cava obstruction, and the differentiation between intrinsic and
extrinsic causitive factors responsible for the obstruction. Contrast venography also
provides assessment of collateral vessel formation, the degree of venous distension of
the neck and arms, measurement of actual venous pressure, and the presence of the
internal jugular vein reflux. Contrast venography is essential prior to planning any
surgical bypass operation. Surgical bypass operations are easier to accomplish when
the brachiocephalic veins are not involved. However, if all the intrathoracic veins are
obstructed, extrathoracic bypass operations can be undertaken, but the operation is
more technically difficult and the results are less favorable.Contrast venography is
also very helpful in documenting obstructions caused by thrombus formation. When
thrombosis is present, treatment with fibrinolytic agents (e.g., urokinase, streptoki-
nase) is pursued and repeat venography can be used to evaluate treatment efficacy. In
the rare cases in which fresh thrombosis is detected in the superior vena cava, throm-
boembolectomy may be an alternate method of treatment.
Radionuclide venography. Radionuclide venography can also be used to diag-
nose SVCS. This test is less invasive than contrast venography but is also less specif-
ic in defining patency and flow. Radionuclide venography may be of value in long-
term follow-up studies.
Computed tomography scanning. Computed tomography (CT) scanning pro-
vides an effective, noninvasive evaluation of the superior vena cava and its collateral
circulation. CT scanning provides anatomic details of the mediastinal and thoracic
organs, allows identification of the cause and extent of the obstruction, documents
collateral circulation, provides guidance for percutaneous biopsies, and guides the
formulation for radiotherapy.
Magnetic resonance imaging. Magnetic resonance imaging (MRI) is also used
extensively in the diagnosis of SVCS, and this test is often very important in deter-
mining the cause of SVCS. Although the collateral circulation is easier to detect by
CT scan, MRI, by virtue of its multidimensional capabilities, shows the relationships
of vessels, lymph nodes, and other mediastinal structures better than the information
provided by CT scanning.
Diagnostic surgery. When all other diagnostic procedures fail to provide infor-
mation about the cause of SVCS, surgery may be the last alternative. Exploratory
thoracotomy is successful in obtaining diagnostic tissue in patients with SVCS in vir-
tually every case. A surgical approach has several advantages—surgery allows direct
visualization of the underlying disease process, assessment of the extent of disease
involvement, and accessibility for tissue biopsy. However, compared to the previous-
ly described diagnostic methods, this procedure is the most invasive and is associated
with increased risks
Other diagnostic techniques. Other diagnostic techniques used in the evalua-
tion of SVCS include bronchoscopy, retinoscopy, cell cytology, and mediastinoscopy.
In each case, the risks of intervention, such as bleeding and perforation of the collat-
eral circulation, should be carefully weighed against the benefits for and safety of the
patient. Today, SVCS is seldom a medical emergency and all efforts should be made
to identify the etiology. Although the specific etiology of SVCS can be obtained by
tissue diagnosis in a few cases, this procedure may be difficult and even hazardous to
the patient.
TREATMENT
Depending on the underlying condition, multiple treatment options are available
for superior vena cava obstruction. The primary treatment options include radiation,
chemotherapy, thrombolytic therapy, anticoagulation, stents and balloon angioplasty,
and surgery.
Radiation Indications. The majority of cases of SVCS are caused by malignan-
cy; thus, most patients receive radiation treatment at some point in their illness.
Emergency radiation treatment has been administered to some patients with life-
threatening cerebral or laryngeal edema prior to a tissue diagnosis of malignancy. The
relief of obstructive symptoms by radiation therapy may provide sufficient time to
work up the cause of SVCS, thus allowing for more specific treatment. Radiotherapy
for the treatment of a thoracic malignancy or lymphoma may be appropriate, whereas
radiotherapy for the treatment of an underlying thrombosis or granulomatosis causing
the obstruction would be inappropriate. Therefore, delaying treatment for 1 to 2 days
if necessary to establish a firm tissue diagnosis is appropriate.
Dosage. Radiation treatment is initiated at high dose fractions daily for the first
few days. This treatment regimen is usually followed by conventional low daily dos-
es. The total dose is dependent on the underlying tumour histology. Lymphomas are
generally treated with 3000 to 4000 cGy, whereas carcinomas require 4000 to 5000
cGy or more to achieve control. Lower doses of radiation treatment may be consid-
ered in cases in which systemic disease is present and short-term palliation is the
goal. Because of the limited tolerance of the heart and spinal cord to radiation, short
duration, high-dose programs are used. Physicians must be aware of this dosage in-
tensity in treating patients who are receiving chemotherapeutic agents such as doxo-
rubicin, which can enhance radiation toxicity
Response to treatment. The response to radiation in most patients occurs within
3 to 4 days. Resolution of facial edema and venous distension of the upper extremities
in addition to radiographic improvement occur within 1 to 3 weeks. Radiation therapy
is usually not effective when thrombosis is causing the occlusion, which emphasizes
the importance of a complete and thorough evaluation of the venous system in the di-
agnostic workup of SVCS. When radiation therapy is successful, prolonged survival
has been reported, especially in cases in which full courses of treatment are complet-
ed. Of all patients with SVCS with malignancies, 10% to 20% survive more than 2
years.
Side effects. Radiation therapy is associated with a number of complications that
include persistent fever, bleeding or superior vena cava perforation at the site of tu-
mour invasion, nausea, vomiting, anorexia, leukopenia, haemoptysis, skin irritation,
and esophagitis. Pulmonary or mediastinal fibrosis may also occur as a late complica-
tion.
Chemotherapy. Chemotherapy may be used as a primary therapy or as an ad-
junct to radiotherapy for the treatment of SVCS, depending on the underlying etiolo-
gy of the obstruction. The treatment of choice for SVCS caused by mediastinal lym-
phoma is a combination of chemotherapy and radiotherapy.
Thrombolytic therapy. The role of thrombolytic therapy and subsequent anti-
coagulation for SVCS has become increasingly important within the past decade. Per-
icatheter thrombosis has been demonstrated by venography in approximately 50% of
non-anticoagulated patients with long term central venous catheters. Depending on
the acuteness or chronicity of the thrombosis, thrombolytic therapy can be used. In
patients with an acute occurrence, thrombolytic therapy can achieve excellent results.
Anticoagulation Patients with SVCS are at increased risk for deep vein throm-
bosis and pulmonary embolism. In patients for whom thrombosis is the cause of
SVCS, anticoagulation therapy should be administered after successful thrombolytic
treatment. Once the symptoms subside after thrombolytic therapy, anticoagulation
should be maintained as long as the central venous catheter is present. Recently, low
dose warfarin has been noted to significantly decrease thrombosis in patients with
central venous catheters
Stents and Balloon Angioplasty. Recent advances in interventional radiology
have contributed expandable wire stents and balloon angioplasty. These stents can be
placed across the stenotic portion of the vena cava. The stents have little thrombogen-
ic potential and usually remain widely patent without narrowing for months. Today,
placement and use of stents is limited when intraluminal thrombosis is present. How-
ever, after thrombolytic therapy, stent placement has been noted to be a more success-
ful approach. After stent placement, patients experience instantaneous relief of symp-
toms. The placement of stents is performed under local anaesthesia by radiologists.
The placement of a stent appears to be suitable therapy for the palliation of the symp-
toms of SVCS in cases for which other therapeutic modalities cannot be used or are
ineffective.11 For localized lesions, balloon angioplasty with or without stenting has
also been shown to significantly reduce the symptoms of SVCS.
Surgical Treatment. Surgical bypass is an additional alternative to relieve
SVCS. The surgical option is usually recommended to patients with benign disease
and to only a few patients with malignancy. Patients selected for surgery should have
the venographic sign of total superior vena cava obstruction associated with throm-
bosis of caval branches and distension of the veins of the upper extremity. Surgery in
cases of fibrosing mediastinitis can be extremely complicated. Because of the gradual
onset of this disorder, the collateral circulation is extensive and serious bleeding can
occur if any of these vessels is transacted. In addition, because of the associated ve-
nous hypertension, all the collateral circulation is under high pressure. The ad-
vantages of surgery are the expeditious and definitive removal of the obstruction and
the convenience of direct tissue diagnosis. Venous thrombectomy may be indicated in
select patients with catheter induced thrombosis of the superior vena cava when the
foreign material can be removed in addition to the obstructing catheter. However,
most data after surgical bypass are obtained from patients soon after surgery. Long-
term results after surgical bypass are lacking, chiefly because most of these patients
have a malignancy and their life expectancy is short.
Other Treatment Options
Additional measures used to treat SVCS include the administration of steroids or
diuretic agents and salt restriction. Diuretic agents may provide symptomatic relief of
edema; this relief is often immediate but not long term. Steroids are useful in the
presence of respiratory compromise but the long-term use of steroids may be consid-
ered harmful because of significant side effects.
PROGNOSIS
The prognosis of SVCS depends on the underlying obstruction. Malignancies of
the mediastinum are the most common cause of SVCS today, and the overall progno-
sis for these patients is poor. In past studies, the average survival time for patients
with SVCS caused by malignancies of the mediastinum has been approximately 6 to
9 months. Most patients with SVCS can be successfully managed with medical or ra-
diation therapy. For patients with severe unrelenting symptoms caused by malignant
disease, thrombolysis, balloon angioplasty, and stenting appear to be clinically ac-
ceptable forms of therapy. Surgical bypass is primarily reserved for the few patients
with persistent symptoms of SVCS secondary to a benign pathology.
CLINICAL SYMPTOMS AND SIGNS OF VENA CAVA INFERIOR
SYNDROME
Causes of the IVCS are: obstruction by deep vein thrombosis or tumors, uterus
compresses the vein at the third trimester of pregnancy; iatrogenic causes and Budd-
Chiari syndrome.
Deep vein thrombosis is a challenge for physicians of all disciplines. It usually
complicates the course of a disease, but it might also be encountered in the absence of
precipitating disorders. Thrombosis arises more often in the deep veins of the legs,
although it can take place in any section of the venous system. Genetic or acquired
causes are frequently associated, which makes it difficult to decide which patients
should be tested for hereditary thrombophilia and what diagnostic tests should be
performed. Thrombosis of the IVC has similar etiological factors to lower limb DVT.
Hypercoagulability (inherited or acquired thrombophilia), venous stasis secondary to
extraluminal pressure (e.g. from tumours or inflammatory processes) and vessel inju-
ry (due to trauma) have all been implicated as primary mechanisms in the pathophys-
iology of DVT.
Deep venous thrombosis of the lower extremity
Acute DVT is a major cause of morbidity and mortality in the hospitalized pa-
tient, particularly in the surgical patient. The triad of venous stasis, endothelial injury,
and hypercoagulable state first posited by Virchow in 1856 has held true a century
and a half later.
Acute DVT poses several risks and has significant morbid consequences. The
thrombotic process initiating in a venous segment can, in the absence of anticoagula-
tion or in the presence of inadequate anticoagulation, propagate to involve more prox-
imal segments of the deep venous system, thus resulting in edema, pain, and immo-
bility. The most dreaded sequel to acute DVT is that of pulmonary embolism, a con-
dition of potentially lethal consequence. The late consequence of DVT, particularly of
the iliofemoral veins, can be CVI and ultimately post-thrombotic syndrome, as a re-
sult of valvular dysfunction in the presence of luminal obstruction.
For these reasons, understanding the pathophysiology, standardizing protocols to
prevent or reduce DVT, and instituting optimal treatment promptly all are critical to
reducing the incidence and morbidity of this unfortunately common condition.
Etiology
The triad of stasis, hypercoagulable state, and vessel injury all exist in most sur-
gical patients. It is also clear that increasing age places a patient at a greater risk, with
those older than 65 years representing a higher-risk population.
Stasis
Labelled fibrinogen studies in patients, as well as autopsy studies, have demon-
strated quite convincingly that the soleal sinuses are the most common sites of initia-
tion of venous thrombosis. The stasis may contribute to the endothelial cellular layer
contacting activated platelets and procoagulant factors, thereby leading to DVT. Sta-
sis, in and of itself, has never been shown to be a causative factor for DVT.
The Hypercoagulable State
Our knowledge of hypercoagulable conditions continues to improve, but it is still
undoubtedly embryonic. The standard array of conditions screened for when search-
ing for a “hypercoagulable state” is listed in Table. 3 . Should any of these conditions
be identified, a treatment regimen of anticoagulation is instituted for life, unless spe-
cific contraindications exist. It is generally appreciated that the postoperative patient,
following major operative procedures, is predisposed to formation of DVT. After ma-
jor operations, large amounts of tissue factor may be released into the bloodstream
from damaged tissues. Tissue factor is a potent procoagulant expressed on the leuko-
cyte cell surface as well as in a soluble form in the bloodstream. Increases in platelet
count, adhesiveness, changes in coagulation cascade, and endogenous fibrinolytic ac-
tivity all result from physiologic stress, such as major operation or trauma, and have
been associated with an increased risk for thrombosis.
Risk factors for thromboembolic are listed in Table. 4.
Table. 3. Hypercoagulable States
1. Factor V Leiden mutation
2. Prothrombin gene mutation
3. Protein C deficiency
4. Protein S deficiency
5. Antithrombin III deficiency
6. Homocysteine
7. Antiphospholipid syndrome

Table. 4. Risk factors for thromboembolic


Obesity Increasing age Immobilization
Varicose veins Prior deep vein thrombosis/embolism Sepsis
Malignancy Thrombophilias Stasis
Stroke Birth control pills Pregnancy
Trauma or operation > Postpartum
2h

Venous Injury
It has been clearly established that venous thrombosis occurs in veins that are
distant from the site of operation; for instance, it is well known that patients undergo-
ing total hip replacement frequently develop contralateral lower extremity DVT.
In a set of elegant experiments, animal models of abdominal and total hip opera-
tions were used to study the possibility of venous endothelial damage distant from the
operative site. In these experiments, jugular veins were excised after the animals were
perfusion fixed. These experiments demonstrated that endothelial damage occurred
after abdominal operations and were much more severe after hip operations. There
were multiple microtears noted within the valve cusps that resulted in the exposure of
the subendothelial matrix. The exact mechanism by which this injury at a distant site
occurs and what mediators, whether cellular or humoral, are responsible are not clear-
ly understood, but that the injury occurs and occurs reliably is evident from these and
other studies.
Clinical Diagnosis
The diagnosis of DVT requires, to use an overused phrase, a high index of suspi-
cion. Most are familiar with Homans' sign, which refers to pain in the calf on dorsi-
flexion of the foot. It is certainly true that although the absence of this sign is not a
reliable indicator of the absence of venous thrombus, the finding of a positive
Homans' sign prompts one to attempt to confirm the diagnosis. Certainly, the extent
of venous thrombosis in the lower extremity is an important factor in the manifesta-
tion of symptoms. For instance, most calf thrombi may be asymptomatic unless there
is proximal propagation. This is one of the reasons that radiolabeled fibrinogen test-
ing demonstrates a higher incidence of DVT than incidence studies using imaging
modalities. Only 40% of patients with venous thrombosis have any clinical manifes-
tations of the condition.
Major venous thrombosis involving the iliofemoral venous system results in a
massively swollen leg with pitting edema, pain, and blanching, a condition known as
phlegmasia alba dolens. With further progression of disease, there may be such mas-
sive edema that arterial inflow can be compromised. This condition results in a pain-
ful blue leg, the condition called phlegmasia cerulea dolens. With this evolution of
the condition, unless flow is restored, venous gangrene can develop.
Venography
Injection of contrast material into the venous system is obviously and under-
standably the most accurate method of confirming DVT and the location. The super-
ficial venous system has to be occluded with a tourniquet, and the veins in the foot
are injected for visualization of the deep venous system. Although this is a good test
for finding occlusive and nonocclusive thrombus, it is also invasive, subject to risks
of contrast, and requires interpretation with 5% to 10% error rate.
Impedance Plethysmography
Impedance plethysmography measures the change in venous capacitance and
rate of emptying of the venous volume on temporary occlusion and release of the oc-
clusion of the venous system. A cuff is inflated around the upper thigh until the elec-
trical signal has plateau. When the cuff is deflated, there is usually rapid outflow and
reduction of volume. With a venous thrombosis, one notes a prolongation of the out-
flow wave. It is not very useful clinically for the detection of calf venous thrombosis
and in patients with prior venous thrombosis.
Fibrin, Fibrinogen Assays
The basis of fibrin or fibrinogen can be assayed by measuring the degradation of
intravascular fibrin. The D-dimer test measures cross-linked degradation products,
which is a surrogate of plasmin's activity on fibrin. It is shown that in combination
with clinical evaluation and assessment, the sensitivity exceeds 90% to 95%. The
negative predictive value is 99.3% for proximal evaluation and 98.6% for distal eval-
uation. In the postoperative patient, D-dimer is causally elevated due to surgery, and,
as such, a positive D-dimer assay for evaluating for DVT is of no use. However, a
negative D-dimer test in patients with suspected DVT has a high negative predictive
value, ranging from 97% to 99%.
Duplex Ultrasound
The modern diagnostic test of choice for the diagnosis of DVT is the duplex ul-
trasound, a modality that combines Doppler ultrasound and colour-flow imaging. The
advantage of this test is that it is noninvasive, comprehensive, and without any risk of
contrast angiography. This test is also highly operator dependent, and this is one of
the potential drawbacks.
The Doppler ultrasound is based on the principle of the impairment of an accel-
erated flow signal due to an intraluminal thrombus. A detailed interrogation begins at
the calf with imaging of the tibial veins and then proximally over the popliteal and
femoral veins. A properly done examination evaluates flow with distal compression
that results in augmentation of flow and with proximal compression that should inter-
rupt flow. If any segment of the venous system being examined fails to demonstrate
augmentation on compression, venous thrombosis is suspected.
Real-time B-mode ultrasonography with colour-flow imaging has improved the
sensitivity and specificity of ultrasound scanning. With colour-flow duplex imaging,
blood flow can be imaged in the presence of a partially occluding thrombus. The
probe is also used to compress the vein. A normal vein is easily compressed, whereas
in the presence of a thrombus, there is resistance to compression. In addition, the
chronicity of the thrombus can be evaluated based on its imaging characteristics,
namely, increased echogenicity and heterogeneity. Duplex imaging is significantly
more sensitive than indirect physiologic testing.
Magnetic Resonance Venography
With major advances in technology of imaging, magnetic resonance venography
has come to the forefront of imaging for proximal venous disease. The cost and the
issue of patient tolerance due to claustrophobia limit the widespread application, but
this is changing. It is a useful test for imaging the iliac veins and the IVC, an area
where duplex ultrasound is limited in its usefulness.
Prophylaxis
The patient who has undergone either major abdominal surgery or major ortho-
paedic surgery, has sustained major trauma, or has prolonged immobility (>3 days)
represents an elevated risk for the development of venous thromboembolism. The
specific risk factor analysis and epidemiologic studies dissecting the etiology of ve-
nous thromboembolism are beyond the scope of this chapter. The reader is referred to
more extensive analysis of this problem.
The methods of prophylaxis can be mechanical or pharmacologic. The simplest
method is for the patient to walk. Activation of the calf pump mechanism is an effec-
tive means of prophylaxis, as evidenced by the fact that few active people without
underlying risk factors develop venous thrombosis. A patient who is expected to be
up and walking within 24 to 48 hours is at low risk for developing venous throm-
bosis. The practice of having a patient “out of bed into a chair” is one of the most
thrombogenic positions that one could order a patient into. Sitting in a chair with the
legs in a dependent position causes venous pooling, which in the postoperative milieu
could easily be a predisposing factor in the development of thromboembolism.
The most common method of prophylaxis in the surgical universe has tradition-
ally revolved around sequential compression devices, which periodically compress
the calves and essentially replicate the calf bellows mechanism. This has clearly re-
duced the incidence of venous thromboembolism in the surgical patient. The most
likely mechanism for the efficacy of this device is prevention of venous stasis. There
is some literature that suggests that fibrinolytic activity systemically is enhanced by a
sequential compression device. However, this is by no means established because
there are a considerable number of studies demonstrating no enhancement of fibrino-
lytic activity.
Another traditional method of thromboprophylaxis is the use of low-dose unfrac-
tionated heparin. The dose traditionally used was 5000 units of unfractionated heparin
every 12 hours. However, analysis of trials comparing placebo versus fixed-dose hep-
arin shows that the stated dose of 5000 units subcutaneously every 12 hours is no
more effective than placebo. When subcutaneous heparin is used on an every-8-hour
dosing, rather than every 12 hours, there is a reduction in the development of venous
thromboembolism.
More recently, a wealth of literature has revealed the efficacy of fractionated
low-molecular-weight heparin (LMWH) for prophylaxis and treatment of venous
thromboembolism. LMWH inhibits factor Xa and IIA activity, with the ratio of anti–
factor Xa to anti–factor IIA activity ranging from 1:1 to 4:1. LMWH has a longer
plasma half-life and has significantly higher bioavailability. There is much more pre-
dictable anticoagulant response than in fractionated heparin. No laboratory monitor-
ing is necessary because the partial thromboplastin time (PTT) is unaffected. A varie-
ty of analyses, including a major meta-analysis, have clearly shown that LMWH re-
sults in equivalent, if not better, efficacy with significantly less bleeding complica-
tions.
Comparison of LMWH with mechanical prophylaxis demonstrates superiority of
LMWH in reduction of the development of venous thromboembolic disease. Prospec-
tive trials evaluating LMWH in head-injured and trauma patients have also proved
the safety of LMWH, with no increase in intracranial bleeding or major bleeding at
other sites. In addition, LMWH shows significant reduction in the development of
venous thromboembolism compared to other methods.
In short, LMWH is considered the optimal method of prophylaxis in moderate-
and high-risk patients. Even the traditional reluctance to use heparin in high-risk
groups such as the multiply injured trauma patient and the head-injured patient must
be re-examined, given the efficacy and safety profile of LMWH in multiple prospec-
tive trials.
Treatment
After a diagnosis of venous thrombosis has been established, a treatment plan
must be instituted. Complications of calf DVT include proximal propagation of
thrombus in up to one third of hospitalized patients and post-thrombotic syndrome. In
addition, untreated lower extremity DVT carries a 30% recurrence rate.
Any venous thrombosis involving the femoropopliteal system is treated with full
anticoagulation. Traditionally, the treatment of DVT centers on heparin treatment to
maintain the PTT at 60 to 80 seconds, followed by warfarin therapy to obtain an In-
ternational Normalized Ratio (INR) of 2.5 to 3.0. If unfractionated heparin is used, it
is important to use a nomogram-based dosing therapy. The incidence of recurrent ve-
nous thromboembolism increases if the time to therapeutic anticoagulation is pro-
longed. For this reason, it is important to reach therapeutic levels within 24 hours. A
widely used regimen is 80 U/kg bolus of heparin, followed by a 15 U/kg infusion.
The PTT needs to be checked 6 hours after any change in heparin dosing. Warfarin is
started the same day. If warfarin is initiated without heparin, the risk for a transient
hypercoagulable state exists because protein C and S levels fall before the other vita-
min K–dependent factors are depleted. With the advent of LMWH, it is no longer
necessary to admit the patient for intravenous heparin therapy. It is now accepted
practice to administer LMWH to the patient as an outpatient, as a bridge to warfarin
therapy, which also is monitored on an outpatient basis.
The recommended duration of anticoagulant therapy continues to undergo evolu-
tion. A minimum treatment time of 3 months is advocated in most cases. The recur-
rence rate is the same with 3 versus 6 months of warfarin therapy. If, however, the pa-
tient has a known hypercoagulable state or has experienced episodes of venous
thrombosis, then lifetime anticoagulation is required, in the absence of contraindica-
tions. The accepted INR range is 2.0 to 3.0; a recent randomized, double-blind study
confirmed that a goal INR of 2.0 to 3.0 was more effective in preventing recurrent
venous thromboembolism than a low-intensity regimen with a goal INR of 1.0 to 1.9.
Additionally, the low-intensity regimen did not reduce the risk for clinically im-
portant bleeding.
Oral anticoagulants are teratogenic and thus cannot be used during pregnancy. In
the case of the pregnant patient with venous thrombosis, LMWH is the treatment of
choice, and this is continued through delivery and can be continued postpartum as in-
dicated.
Thrombolysis
The advent of thrombolysis has resulted in increased interest in thrombolysis for
DVT. The purported benefit is preservation of valve function with subsequently less-
er chance of developing CVI. However, to date, few definitive, convincing data exist
to support the use of thrombolytic therapy for DVT.
One exception is the patient with phlegmasia in whom thrombolysis is advocated
for relief of significant venous obstruction. In this condition, thrombolytic therapy
probably results in better relief of symptoms and less long-term sequelae than heparin
anticoagulation alone. The alternative for this condition is surgical venous throm-
bectomy. No matter which treatment is chosen, long-term anticoagulation is indicat-
ed. The incidence of major bleeding is higher with lytic therapy.
Vena Caval Filter
The most worrisome and potentially lethal complication of DVT is pulmonary
embolism. The symptoms of pulmonary embolism, ranging from dyspnea, chest pain,
and hypoxia to acute cor pulmonale, are nonspecific and require a high index of sus-
picion. The gold standard remains the pulmonary angiogram, but increasingly, this is
being displaced by the computed tomographic angiogram.
Adequate anticoagulation is usually effective in stabilizing venous thrombosis,
but if a patient should develop a pulmonary embolism in the presence of adequate an-
ticoagulation, a vena cava filter is indicated. The general indications for a caval filter
are listed in Table. 5. The modern filters are placed percutaneously over a guidewire.
The Greenfield filter, with the most extensive use and data, has a 95% patency rate
and a 4% recurrent embolism rate. This high patency rate allows for safe suprarenal
placement if there is involvement of the IVC up to the renal veins or if it is placed in
a woman of childbearing potential.
Table. 5
Indications for a Vena Cava Filter
1. Recurrent thromboembolism despite adequate anticoagulation
2. Deep venous thrombosis in a patient with contraindications to anticoagulation
3. Chronic pulmonary embolism and resultant pulmonary hypertension
4. Complications of anticoagulation
5. Propagating iliofemoral venous thrombus in anticoagulation

The device-related complications are wound hematoma, migration of the device


into the pulmonary artery, and caval occlusion due to trapping of a large embolus. In
the latter situation, the dramatic hypotension that accompanies acute caval occlusion
can be mistaken for a massive pulmonary embolism. The distinction between the
hypovolemia of caval occlusion and the right heart failure from pulmonary embolism
can be made by measuring filling pressures of the right side of the heart. The treat-
ment of caval occlusion is volume resuscitation.
Retrievable Vena Caval Filters
Although generally safe, IVC filters are not without risk and significant morbidi-
ty. Therefore, permanent placement of a caval filter, particularly in a young patient
who may only require short-term caval protection, is not generally accepted. Retriev-
able filters entered the field as a potential solution for the patient with temporary in-
dications for pulmonary embolus prophylaxis. There are three retrievable IVC filters
that have U.S. Food and Drug Administration approval: the Recovery filter, the Op-
tEase filter, and the Gunther-Tulip filter. These filters vary slightly with respect to
shape and length. All can be deployed from the internal jugular vein or femoral vein
and retrieved from the right jugular vein (Gunther-Tulip and Recovery) or the right
femoral vein (OptEase). Before retrieval, a venogram is performed to ensure there is
no nidus of IVC thrombus in the filter. These filters can be placed either in an angi-
ography suite or at the bedside using intravascular ultrasound. A major advantage to
retrievable filters is that they may be removed when the patient either no longer re-
quires pulmonary embolism protection or is able to undergo anticoagulation. Patient
groups that may benefit from retrievable filters include multiple-trauma patients and
high-risk surgical patients. Insertion complications reported include vena cava perfo-
ration, filter migration, and venous thrombosis at the insertion site. Retrieval compli-
cations include failure to retrieve the filter, thrombus embolization from the filter,
vein retrieval site thrombus, and groin hematoma. However, the role of retrievable
filters continues to be a work in progress. Further investigation is required before de-
finitive practice guidelines are established - Table. 6
Table. 6
Indications for Placement of a Retrievable Inferior Vena Cava Filter
1. Prophylactic placement in a high-risk trauma patient (orthopaedic, spinal cord
patients)
2. Short-term duration contraindication of anticoagulation therapy
3. Protection during venous thrombolytic therapy
4. Extensive iliocaval thrombosis

The Budd-Chiari syndrome


Budd–Chiari syndrome encompasses a group of disorders caused by obstruction
to hepatic venous outflow. The range of presentations of the clinical syndrome is
from mild, non-specific, upper abdominal symptoms to a fulminant course with ex-
tensive hepatic necrosis. Budd originally described the pathologic features of hepatic
vein thrombosis in 1846 while the clinical syndrome of hepatomegaly, ascites, and
abdominal pain was described by Chiari in 1899. However, the syndrome now more
broadly includes any thrombotic or non-thrombotic occlusion at the hepatic veins or
the suprahepatic inferior vena cava leading to liver outflow obstruction. The venous
outflow obstruction results in marked elevation of sinusoidal pressure, intense con-
gestion of the liver, progressive necrosis of the hepatic parenchyma, and hepatocellu-
lar death.
The investigation and management of Budd–Chiari syndrome depends on a mul-
tidisciplinary approach with input from hepatology, hematology, radiology, and sur-
gery. The rarity of this syndrome often leads to a delay in diagnosis, but suspicion of
hepatic venous outflow obstruction mandates a careful work up with the priorities of
defining the following.
1. Is there a mechanical outflow obstruction?
2. Is there ongoing acute or chronic liver damage?
3. What is the underlying etiology for the syndrome?
4. What are the treatment options?
This chapter will define a method for working through these steps based on cur-
rent data.
Pathophysiology
Budd–Chiari syndrome is caused by obstruction to hepatic venous outflow. This
obstruction means that the liver sinusoids cannot drain while blood continues to flow
into the liver. This results in congestion of the sinusoids, increased sinusoidal pres-
sure, hepatocyte necrosis, and progressive liver damage. If this passes to the chronic
phase, centrilobular fibrosis results. Untreated, patients die from progressive liver
damage over several months to years.
The normal physiology of hepatic venous drainage is through the right, middle,
and left hepatic veins, with the caudate lobe having separate short veins draining into
the inferior vena cava. In addition, there are usually several small hepatic veins pass-
ing directly from the right lobe into the intrahepatic inferior vena cava. In Budd–
Chiari syndrome, if outflow obstruction only affects some of these veins, the seg-
ments of the liver that maintain venous drainage may be spared, will hypertrophy sig-
nificantly, and there is therefore a more chronic rate of progression of the total syn-
drome. The extent of hepatic venous thrombosis correlates with the clinical manifes-
tations of the syndrome and its prognosis. When there is total obstruction to all hepat-
ic venous outflow, the severe congestion and swelling will stretch Glisson's capsule
with severe pain and a fulminant course. On the other hand, the more common se-
quence is for one or two of the major hepatic veins to occlude and the symptoms may
be less severe leading to delay in diagnosis until there is further obstruction of other
major venous outflow.
Veno-occlusive disease of the liver presents with different pathophysiology but
has similar outcomes. Most common in patients undergoing bone marrow transplanta-
tion, this syndrome results in a non-thrombotic occlusion of small sublobular branch-
es of the hepatic veins causing a sinusoidal outflow obstruction. This in turn can lead
to hepatocyte necrosis and a full-blown syndrome.
Etiology. Etiologies are listed in Table 7
Table 7
Etiologies of Budd–Chiari syndrome (%)
1. Myeloproliferative disorder 50
2. Tumors 10
3. Estrogen use/pregnancy 10
4. Hypercoaguable states 5
5. Paroxysmal nocturnal hemoglobinuria 5
6. Caval or hepatic vein webs 5

Chronic myeloproliferative disorders


These are the most common etiologic factor in Budd–Chiari syndrome. Polycy-
themia rubra vera is the most common of these although other myeloproliferative dis-
orders such as myelofibrosis, essential thrombocythemia, and chronic lymphocytic
leukaemia may cause hepatic vein thrombosis. These disorders are characterized by a
common origin of a malignant stem cell change followed by proliferation. The out-
come of this process is an increased viscosity, a low-grade disseminated intravascular
coagulation, and intravascular thrombosis. The liver is the main site of clearance of
plasminogen activator, and a deficiency of hepatic antiplasmin may contribute to lo-
calization of thrombosis in the hepatic veins. Evaluation and detection of these hema-
tologic disorders is more difficult in patients with Budd–Chiari syndrome than in
their usual occurrence because of associated portal hypertension with increased plas-
ma volume and splenomegaly which may mask the increased red blood cell mass and
changes of thrombocythemia. Early detection of an underlying myeloproliferative
disorder can be made using spontaneous endogenous erythroid colony formation in
vitro and/or in bone marrow biopsies. Up to three-quarters of patients with 'idiopathic'
Budd–Chiari syndrome have been shown to fall into this category.
Webs
Webs in the suprahepatic inferior vena cava or hepatic venous outflow are a
common cause of Budd–Chiari syndrome in the Far East and South Africa. In the
United States, webs should be considered in immigrants from that region but are not
frequently seen in the native populations.
Tumors
Tumors as a cause of Budd–Chiari syndrome account for approximately 10 per
cent of cases. Most of these tumors are intrahepatic hepatocellular carcinomas that
directly invade the hepatic veins. However, other tumors such as renal and/or adrenal
tumors may involve the vena cava and/or the hepatic veins themselves. It is the me-
chanical obstruction caused by these tumors with intrinsic or extrinsic compression of
the hepatic veins that lead to Budd–Chiari syndrome.
Pregnancy and oral contraceptives
Pregnancy and oral contraceptives are cited in the etiology of Budd–Chiari syn-
drome and it is usually in the first 2 months after delivery that pregnancy is implicat-
ed as a risk factor. It has been postulated that increased levels of factors VII and VIII
along with increased fibrinogen may be etiologic. While the reported risk in patients
on the oral contraceptive pill is 2.4 over age-matched controls, other etiologic factors
should be sought as some of these patients may have an underlying hematologic dis-
order.
Paroxysmal nocturnal hemoglobinuria
Paroxysmal nocturnal hemoglobinuria may accompany other severe hematologic
disorders such as aplastic anemia and the acute leukemias. Characterized by comple-
ment-induced platelet activation, these patients are markedly hypercoagulable with
extensive thrombosis of all outflow tracts and a devastating fulminant course.
Hypercoagulable states
Hypercoagulable states account for a relatively low percentage of patients with
Budd–Chiari syndrome. However protein C, protein S, and antithrombin III deficien-
cies should all be sought in patients with hepatic venous thrombosis and no other de-
fined etiology. More recently, the factor V Leiden mutation has been shown to be a
frequent cause of hereditary thrombophilia. A recent analysis has documented a sig-
nificantly increased prevalence of this mutation in Budd–Chiari syndrome, so it is a
further factor which should be sought.
Clinical presentation
Budd–Chiari syndrome is relatively uncommon in Europe and the United States,
being much more common in India, South Africa, and the Orient. It can occur at any
age, but is most common in the third and fourth decades; it is slightly more common
in women than in men. Budd–Chiari syndrome may present in acute, subacute, or
chronic forms depending on the factors outlined in the pathophysiology above.
Acute fulminant Budd–Chiari syndrome
This is uncommon and only occurs when there is total obstruction of all venous
outflow with severe congestion, hepatocyte necrosis, and liver failure. This usually
implies a severe underlying hematologic disorder which must be fully defined in
making management decisions.
Subacute Budd–Chiari syndrome
This presents with symptoms over several weeks. Right upper quadrant pain and
ascites are the dominant symptoms, but the absence of other stigmas of liver disease
such as jaundice, muscle wasting, and spider angiomas often result in failure to make
the diagnosis. Hepatomegaly is present, but may be difficult to detect in the face of
significant ascites. Splenomegaly is not a feature in early Budd–Chiari syndrome, on-
ly occurring as portal hypertension develops later in the course. The most frequent
misdiagnoses are of a gynaecologic or gastrointestinal malignancy or an intra-
abdominal inflammatory process causing the ascites.
Chronic Budd–Chiari syndrome
This presents with portal hypertension and stigmas of chronic liver disease. Asci-
tes, variceal bleeding, and muscle wasting may all be present, and it is usually be-
cause 'something does not fit' in the work up that Budd–Chiari syndrome is consid-
ered and diagnosed as outlined below.
In all stages of the disease, non-specific symptoms such as general malaise,
tiredness, anorexia, and nausea are unhelpful. In considering the clinical presentation
of disorders in the differential diagnosis such as malignancies listed above, right-
sided cardiac problems, or early diagnosis of other liver diseases, these non-specific
symptoms will be similar. It cannot be overemphasized that the diagnosis of Budd–
Chiari syndrome depends on a high index of suspicion.
Diagnosis
Laboratory studies, as with the clinical presentation, are overall unhelpful in
making a diagnosis of Budd–Chiari syndrome. Liver tests usually show minimal ele-
vation of aminotransferases and bilirubin, a marginal decrease in serum albumin, and
a mild prolongation of prothrombin time. Considering the degree of hepatocellular
damage that is often seen at biopsy, it is surprising how minimal these laboratory
changes appear. The one exception is the unusual patient with fulminant Budd–Chiari
syndrome who may show marked laboratory abnormalities. Ascites fluid analysis
should be done but the findings are highly variable with total protein from 1.5 to 3 g.
The serum to ascites albumin gradient of greater than1.1 g/dl is usually found, but is
not diagnostic. The ascites cell count is variable with most patients having fewer than
100 white blood cells/mm3. Spontaneous bacterial peritonitis does not occur in
Budd–Chiari syndrome.
Diagnosis depends on a combination of appropriate radiologic imaging and liver
biopsy. If the diagnosis is confirmed, then full hematologic evaluation becomes im-
portant. The sequence to be followed can therefore be divided into the phases of: (I)
Is there a mechanical outflow obstruction? (II) Is there ongoing or chronic hepatic
damage? and (III) Is there an underlying hematologic disorder?
Radiologic investigation
Radiologic investigation must follow an appropriate sequence. Clinical suspicion
of Budd–Chiari syndrome should lead to hepatic ultrasound with Doppler flow stud-
ies of the main hepatic veins. In a normal subject, the three major hepatic veins can
be readily visualized and phasic flow documented within them. Inability to demon-
strate these veins or to only show small and stenotic vessels with no phasic flow rais-
es a high index of suspicion of hepatic vein occlusion. shows ultrasound imaging of
hepatic vein with appropriate phasic flow within it. If this can be demonstrated for all
major hepatic veins, Budd–Chiari syndrome is excluded. Concurrent with hepatic
vein imaging, the portal vein should be visualized to document patency and presence
of prograde flow to the liver within it. This is an important further piece of infor-
mation for management decisions.

Figure 1 Doppler ultrasound of hepatic veins. The patent hepatic vein can be
seen and the tracing shows normal phasic flow pattern. If these two components are
absent, the diagnosis of Budd–Chiari syndrome must be entertained.

Other radiologic imaging modalities such as hepatic scintigraphy, computed to-


mography, and magnetic resonance imaging have all been used both to assess liver
morphology and for vascular imaging in Budd–Chiari syndrome. While each of these
may provide some supportive evidence for Budd–Chiari syndrome, they are not first-
line diagnostic modalities and by and large are unnecessary.
The gold standard for radiologic diagnosis of Budd–Chiari syndrome is hepatic
venography and this should be performed if the Doppler ultrasound is equivocal or
strongly supports the diagnosis. Hepatic venography may be done either through a
transfemoral or a transjugular approach, and should attempt to cannulate each of the
major hepatic veins directly. Pressure measurements are of little value in making the
diagnosis as reliable pressures cannot be measured in the occluded or stenotic veins.
Contrast material should be injected at each site and will give the typical spider's web
appearance that is characteristic of Budd–Chiari syndrome Figure 2. The spider's web
represents contrast in recannulized vessels coursing on the surface of the liver which
are collaterals that are attempting to decompress the congested sinusoids. In involved
segments, no normal named hepatic vein can be identified. As indicated above, the
disease may only affect one or two of the three major hepatic veins, and it is for this
reason that all the major veins must be studied. Venography also includes visual and
pressure assessment of the inferior vena cava which may be compressed by the swol-
len liver (Figure 3) or thrombosed. This information helps plan decompressing thera-
py.

Figure 2 The typical spider's web of Budd–Chiari syndrome seen by hepatic ve-
nography. The interconnecting small branches represent collaterals attempting to de-
compress the sinusoids. No major hepatic vein is visualized.
Figure 3 Inferior vena caval compression. The swollen liver of Budd–Chiari
syndrome has compressed the inferior vena cava with a resultant pressure gradient
from the infrahepatic cava to the right atrium.

The role of celiac and/or superior mesenteric arteriography is more controversial


in the work up of patients with Budd–Chiari syndrome. These studies should be used
when Doppler ultrasound fails to demonstrate an open portal vein with flow. Arteri-
ography, followed through to the venous phase, is important to document if there is
thrombosis of the portal vein and its major tributaries as this may obviate some of the
surgical options.
Liver biopsy
This must be performed if the radiologic imaging studies confirm or are highly
suspicious for Budd–Chiari syndrome. This is the confirmatory test for a histologic
diagnosis in addition to assessing the degree of ongoing hepatocyte necrosis and liver
fibrosis. It is often said that the presence of massive ascites and a degree of coag-
ulopathy may make biopsy difficult, but if that is the case, ascites should be drained
and the coagulopathy corrected to allow this essential diagnostic step to be taken.
Separate biopsies of the right and left lobe should be done to evaluate the degree of
involvement of both sides of the liver. Transjugular liver biopsy is not a good option
in these patients unless the biopsy needle can be advanced some distance into the oc-
cluded hepatic veins to assure adequate tissue samples. The essential features on liver
biopsy to be evaluated by the pathologist are the degree of centrilobular congestion,
hepatocellular necrosis, degree of lobular collapse, and finally extent of fibrosis
and/or cirrhosis.
Laparoscopy has been suggested as a diagnostic modality in evaluating Budd–
Chiari syndrome. Its use should be limited to cases where there remains a diagnostic
dilemma although it does provide an alternative way for bilobar biopsies under direct
vision. Laparoscopy is an adjunctive rather than a primary diagnostic tool.
Hematologic evaluation
Hematologic evaluation is the third major diagnostic step. This is summarized in
Table and requires the input of an experienced haematologist who understands the
interrelation of the Budd–Chiari syndrome with the pathophysiologic changes of por-
tal hypertension and the impact of this on the hematologic diagnostic tests. The newer
modalities of spontaneous endogenous erythroid colony formation and measurement
of factor V gene for the Leiden mutation should be performed where available.
Table 8
Hematologic evaluation of patients with Budd–Chiari syndrome
1. Routine hematology with blood smear
2. Bone marrow to include reticulin stain and cytogenetics
3. Erythroid colony formation
4. Leukocyte alkaline phosphatase
5. Platelet aggregometry
6. Red blood cell mass
7. Sucrose hemolysis
8. Protein C, protein S, and antithrombin III levels
9. Factor V Leiden mutation

Management
The management of Budd–Chiari syndrome is divided into two phases, first the
treatment of the liver and second the treatment of the underlying etiology. Treatment
of the liver damage component is based on biopsy findings. Treatment of any under-
lying hematologic disorder is an ongoing process that includes management of the
underlying hematologic disorder and may include anticoagulation.
The three options in the management of the liver injury are:
• non-surgical management
• surgical decompression
• liver transplantation.
Figure 4. illustrates the main categories of biopsy findings and the associated
therapies. The decision on non-surgical treatment or surgical management is based on
the biopsy.
Figure 4
Congestion Ongoing necrosis Fibrosis /
No necrosis Mid/no fibrosis Cirrhosis

Treat ascites Side-to-Side shunt Orthotopic liver


Follow Up transplantation
- ultrasound
- biopsy
The three broad categories of liver biopsy findings in this figure dictate the man-
agement courses.
Non-surgical management must be used with caution as the disease may result in
progressive liver damage. This information shows a liver biopsy with mild Budd–
Chiari syndrome with some sinusoidal dilatation but minimal evidence of ongoing
necrosis. Management of ascites in patients such as this with salt restriction and diu-
retics is appropriate, but further follow-up with repeat ultrasound and repeat liver bi-
opsies in 3 to 6 months is mandatory. In the acute setting, providing the biopsy does
not show significant ongoing necrosis, thrombolytic therapy with streptokinase or
urokinase can be undertaken, but again requires careful invasive follow-up. A perito-
neal venous shunt does nothing to deal with the basic pathophysiology of Budd–
Chiari syndrome and is a purely palliative measure that should not be used unless all
other options have been eliminated. Web dilatation with angioplasty may play a role,
but again requires reassessment of liver pathology. When non-surgical management is
embarked upon, there must be a low index for moving forward to surgical decom-
pression if the follow-up biopsies show progression to necrosis or there is refractory
ascites.
Microscopy shows of acute Budd–Chiari syndrome with ongoing hepatocyte ne-
crosis under both high- and low-power views. This patient needs sinusoidal decom-
pression by side-to-side portal systemic shunt. The principle in the management of
this patient is that the sinusoids can be decompressed by using the portal vein as an
outflow track from the obstructed sinusoids and any side-to-side type of shunt will
suffice. The options are broad. A side-to-side portacaval shunt will be adequate if the
inferior vena cava is not obstructed. Equally, a mesocaval shunt is an infrahepatic
shunt that will decompress the sinusoids when there is no caval obstruction. Howev-
er, if there is severe caval obstruction, as shown by a cava to atrial pressure gradient
greater than 20 mmHg, a mesoatrial shunt may be required to decompress the sinus-
oids.
These shunts are illustrated in Figs 5 and 6. Other options are available to de-
compress the sinusoids. Transjugular intrahepatic portal systemic shunts (TIPS) have
been used in Budd–Chiari syndrome, applying the same principle of the portal vein
being the outflow track from the obstructed sinusoids. In the enlarged liver of Budd–
Chiari syndrome, TIPS need to be long, often with two or three stents, and if there is
an underlying thrombotic problem, careful monitoring is required to assure continued
patency. Are mesoatrial shunts still indicated? The issue is important in that this is a
complex procedure with a long synthetic graft. The availability of intravascular stents
to stent open an obstructed inferior vena cava may obviate the need for mesoatrial
shunts by combining an infrahepatic portacaval or mesocaval shunt with an intrahe-
patic inferior vena cava stent.

Fig 5.. Side-to-side infrahepatic shunts. Portacaval, mesocaval, and mesorenal


shunts are illustrated that all serve to use the portal vein as a decompressive route
from obstructed sinusoids in Budd–Chiari syndrome.
Fig 6. Mesoatrial shunt. A long synthetic poly(tetrafluoroethene) (PTFE) graft is
taken from the superior mesenteric vein in front of the liver, through the diaphragm,
and anastomosed to the right atrium for decompression of the liver using the portal
vein as an outflow tract from the sinusoids.

From the pathophysiologic point of view, any type of decompression of the si-
nusoids will halt the ongoing necrosis, stabilize the liver disease, and relieve the
symptoms of Budd–Chiari syndrome in the acute setting.
Figure 5 illustrates the third type of biopsy that may be seen in Budd–Chiari
syndrome with severe fibrosis and cirrhosis. There are only small islands of regenera-
tive nodules of hepatocytes in this densely fibrotic liver. This patient requires a liver
transplant. No attempt to decompress this liver will relieve the obstruction or alleviate
the symptoms. Progression to this degree of scarring is usually the result of recurrent
thrombotic episodes involving progressively more hepatic veins over several years.
The liver has progressively attempted to compensate for loss of hepatic segments by
hypertrophy of the remaining segments before they too become involved in the pro-
cess. Marked distortion of gross liver anatomy is therefore usually seen on ultra-
sound, CAT scan, or direct visualization of the liver. Transplant is often difficult in
these patients with the recipient hepatectomy being complicated by the many collat-
erals which have formed from the surface liver to the diaphragm in an attempt to de-
compress the sinusoids spontaneously. The transplant surgeon needs to be aware of
this and take appropriate intraoperative precautions to move through his phase.
Hematologic management
This must proceed in parallel with management of the liver disease. It is im-
portant that any underlying hematologic disease is identified preoperatively, therapy
initiated, and decisions made as to optimal timing for surgery considering the risk to
the liver and the risk of further vascular thrombosis. Management must be continued
long term, and, in addition to specific therapy aimed at any underlying blood disor-
der, may require long-term anticoagulation.
Outcomes
Budd–Chiari syndrome is rare, and there are no prospective, randomized, con-
trolled trials looking at outcomes. Large series amount to approximately 50 patients.
Reports have largely been based on local available expertise. However, in the last two
decades there has been increasing recognition of the need to use different therapies
and to tailor the management to the findings as outlined in this chapter. The reported
experience with non-surgical methods is largely anecdotal with case reports of
thrombolytic therapy, or web dilatations, and of conservative management of symp-
toms.
Experience with TIPS for Budd–Chiari syndrome is growing as exemplified by
the Freiburg experience. In this series of 12 patients managed by TIPS, the stent
could be placed successfully in all patients and alleviated symptoms in the 10 patients
with subacute and chronic disease. The two patients with fulminant disease pro-
gressed to liver failure and died. Long-term follow-up with TIPS is still awaited in the
series or other reports.
Scientifics summarizes reported experience of some of the larger series of surgi-
cal decompression by side-to-side shunts. These series include both infra- and su-
prahepatic shunts depending on the status of the inferior vena cava. However, some
have also included infrahepatic shunts combined with caval stent. Overall, reasonable
outcomes have been achieved with survival as indicated in this table, but it has been
increasingly recognized that decompression with a surgical shunt may not be optimal
therapy when there is significant fibrosis and cirrhosis.
Experience with liver transplantation for Budd–Chiari syndrome is summarized
from five centers in Table. 9. While early experience had fairly significant hospital
mortality, usually related to the preoperative treatments used and technically difficult
procedures as outlined above, the overall results have improved. The major issue in
facing a decision for transplantation is the severity of any underlying hematologic
disorder and the potential impact of post-transplant immunosuppression on that dis-
ease. Equally, the need for anticoagulation has to be carefully considered in these pa-
tients as the correct timing in initiating this is essential to avoid early graft throm-
bosis. Some of the transplants series have reported significant retransplant rates relat-
ed to vascular thrombosis following initial transplantation.
Table. 9.
Experience and outcome of orthotopic liver transplantation for Budd–
Chiari syndrome
Hospital mortality (%) Survival (%) (no. of years)
30 58 (5)
12 71 (2)
14 76 (3)
28 69 (5)
67 (5)

Summary
Clinical suspicion of Budd–Chiari syndrome should lead to a Doppler ultrasound
of the main hepatic veins. Failure to identify these vessels or lack of phasic flow
should lead to further evaluation with venography. Liver biopsy should be performed
at the same time to evaluate both lobes of the liver. Confirmation of a diagnosis of
Budd–Chiari syndrome should lead to definitive management based on the severity of
the findings on the biopsy. Treatment options range from minimal intervention for pa-
tients with sinusoidal congestion but no ongoing hepatocyte necrosis, through surgi-
cal decompression for those with acute Budd–Chiari syndrome and ongoing liver
damage, to liver transplantation for those with end stage liver disease.
Lymphoedema of special sites
Genital lymphoedema
Minor scrotal and penile lymphoedema can be tolerated without specific treat-
ment, although support garments may be helpful. Severe scrotal oedema is best treat-
ed by excisional reduction surgery in which a large central segment is excised from
the scrotum, preserving the spermatic cords and testicles. The flaps are then primarily
sutured using an absorbable material polydioxine sulphate and the scrotum is drained.
Mobilization of the testes with gentle abrasion of their surfaces may encourage adhe-
sions to form, allowing lymph to drain via the testicular lymphatics, aiding the scrotal
reduction.
The penis may be reduced by simple excisional procedures, combined with cir-
cumcision if necessary. Alternatively, the skin and subcutaneous tissue can be
stripped off the deep fascia and split skin grafts applied (a Charles operation of the
penis). Both scrotal and penile reduction operations produce gratifying results for the
surgeon and patient. Massive labial swelling can also be treated by excisional proce-
dures.
Eyelids and upper limb
Eyelid swelling can be treated by lid reduction. Arm swelling can be treated by a
Homans' type of limb reduction, which can be performed on both the inner and outer
sides of the upper limb. Patients with postmastectomy oedema must be assessed care-
fully to ensure that the venous drainage is satisfactory and to be certain that there is
no evidence of recurrent axillary nodal disease. Both venous obstruction and recur-
rent malignancy are contraindications to arm reduction. Postoperatively an elasticised
sleeve should be worn to try to prevent recurrent swelling.
Liposuction
Liposuction has been used to remove subcutaneous fat in patients with mild
lymphoedema. Anecdotal successes have been achieved but the cosmetic results are
variable and the procedure should be used with caution.
Chylous reflux
Some patients have dilated (almost varicose) valveless megalymphatics which
allow the reflux of lymph (often chyle) against the expected direction of flow. These
dilated lymphatics often end in cutaneous vesicles which are visible in the skin or
which may rupture into body cavities such as the pleura, peritoneum, kidney, bladder,
uterus, and vagina. Rupture results in the accumulation of lymph or chyle in the rele-
vant cavity (chylothorax, hydrothorax, chylous ascites, and chyluria) and chylous dis-
charge on to the skin surface or mucosa can also occur. Accumulation of chyle in the
pleural and peritoneal cavities produces severe symptoms, and patients often become
dyspnoeic and very distended. Patients with megalymphatics often also have a pro-
tein-losing enteropathy which can cause weight loss and exacerbate accumulation of
fluid in the body cavities and tissues. This results from leakage of lymph from the
mucosal surface of the bowel; associated lymphatic leakage from the serosal surface
may exacerbate the accumulation of ascites.
The diagnosis of chylous ascites or chylothorax must first be confirmed by aspi-
ration of the fluid, which is then tested for chylomicrons. The condition may be sus-
pected if there is pre-existing lymphoedema of the extremities and it is especially
likely if vesicles and lymphatic leakage are present. In quite a few patients, however,
the condition develops de novo. Chylothorax and chylous ascites must be distin-
guished from malignant ascites or a malignant effusion: cytological examination of
the aspirate may help to exclude or confirm the presence of malignant cells. CT scan
and ultrasound can demonstrate the presence of moderate or severe enlargement of
the abdominal or mediastinal lymph nodes which suggests the possibility of a lym-
phoma or secondary malignant spread. Guided biopsy, laparoscopy, or laparotomy
may be necessary to confirm these diagnoses. Contrast lymphography demonstrates
lymphadenopathy, filling defects, or the presence of megalymphatics and is indicated
if the diagnosis remains in doubt. Contrast lymphography may also demonstrate a
lymphatic leak which can be surgically sealed.
Lymphoedema associated with megalymphatics rarely requires reduction sur-
gery, but the complications of lymphatic vesicles, recurrent infections, and lymphatic
discharge on to the skin, chylous ascites, chyluria, and chylothorax often demand
treatment. Leakage of chyle or lymph may be prevented by ligating or under running
the dilated lymphatic channels, but this carries the risk of lymphatic obstruction
which will worsen the limb swelling. Despite this many patients benefit from ligation
of dilated lymphatics, and sealing off of any obvious site of fistulation.
If a patient with chylous ascites or chylothorax has no obvious leak on the lym-
phangiogram, chromium chloride studies and a barium study of the small bowel may
provide useful information before a laparotomy is performed. At laparotomy the pos-
terior abdominal wall over the main lymphatic pathways must be carefully inspected
for the presence of lymphatic leakage, and the whole of the intestine should be exam-
ined. If the surface of the small bowel is grossly abnormal and leaking lymph, the in-
volved or most abnormal segment should be resected. Consideration must be given to
shunting the ascites back into the venous system using a LeVeen or Denver shunt if
this simple approach fails. Although these shunts often work well in patients with re-
fractory ascites, chyle often blocks the plastic tubing, or the valve, and produces an
early occlusion of the shunt. Many patients improve with simple avoidance of fat and
prescription of medium-chain triglycerides combined with diuretics.
A chylothorax may respond to aspiration but often recurs and is best prevented
by surgical pleurodesis by pleural stripping. Some patients die from water- or lymph-
logged lungs after this procedure as the lymphatics draining the lung become ob-
structed when they are no longer able to empty into the pleural cavity. Nevertheless
many patients with severe problems as the result of megalymphatics can be helped by
some of the procedures outlined above. Cutaneous vesicles may be simply excised or
touched with the diathermy or cautery, but they tend to recur. Recurrent infections
should be treated by a prolonged course of broad-spectrum antibiotics.
Lymphangioma circumscriptum
These lesions are either considered as hamartomas or as localized abnormalities
of the cutaneous lymphatic drainage. They present as a number of clear or slightly
haemorrhagic cutaneous vesicles, often associated with subcutaneous thickening in
the underlying fat . Whimster thought that a lymphangioma circumscriptum was the
result of defective lymphatic drainage from the subcutaneous tissue where a number
of cisterns 'pump' lymph back into the overlying skin. These areas should be excised
if they are unsightly or painful. They often occur on the trunk and it is important to
excise a generous amount of subcutaneous tissue well beyond the ellipse of skin bear-
ing the vesicles in order to remove the subcutaneous bladders described by Whimster.
It is often quite difficult to excise all the skin lesions and they have a propensity to
recur: excisional surgery is only required if they are symptomatic.
Cystic hygroma
In this developmental abnormality of the lymphatic system, lymphatic fluid col-
lects in a cystic space which is often multilocular and situated in the base of the neck.
Cystic hygroma commonly appears in childhood and presents as a soft, brilliantly
translucent swelling in the base of the neck. Aspiration and injection of sclerosant
may be attempted, but the swellings often recur and may require excision. Cystic hy-
gromas must be dissected with great care as a number of important structures lie ad-
jacent to them.
Mesenteric cysts
These localized lymphatic cysts within the mesentery appear as well-
circumscribed mobile lumps within the abdomen. The diagnosis can be confirmed by
ultrasound or CT scanning. They are treated by resection, often in association with
the overlying area of small bowel. Although harmless, they may reach a considerable
size if left untreated.
Basic literatures:
71.Oxford Textbook of Surgery (3-Volume Set) 2nd edition (January 15, 2000):
by Peter J. Morris (Editor), William C. Wood (Editor) By Oxford Press
72.Sabiston Textbook of Surgery 17th edition by Courtney M. Townsend Jr.,
Kenneth L. Mattox, B. Mark, MD Evers, Kenneth L., MD Mattox, Courtney
Townsend, Daniel Beauchamp, B. Mark Evers, Kenneth Mattox W.B. Saun-
ders Company (June, 2004)
73.Schwartz´s Principles of Surgery 8th Edition F.Charles Brunicardi. Copyright
©2007 The McGraw-Hill Companies.
74.Hospital surgery/ Edited by L.Kovalchuk et al. - Ternopil: Ukrmedknyha,
2004. - 472 p.

Additional literatures:
4. .Greenfield's Surgery: Scientific principles and practice, 4th Edition, Editors:
Mulholland, Michael W.; Lillemoe, e.a., 2006, Lippincott Williams & Wilkins.
5. Fischer, Josef E., Mastery of Surgery, 5th Edition, 2007, Lippincott Williams
& Wilkins

Tests for initial level of knowledge, keys for tests:


1. A 45-year-old woman undergoes cardiac catheterization through a right femo-
ral approach. Two months later, she complains of right lower extremity swelling and
notes the appearance of multiple varicosities. On examination, a bruit is heard over
the right groin. What is the most likely diagnosis?
(A) Femoral artery thrombosis
(B) Superficial venous insufficiency
(C) Arteriovenous (AV) fistula
(D) Pseudoaneurysm
(E) Deep vein insufficiency

2. A young basketball player develops an acute onset of subclavian vein throm-


bosis (effort thrombosis) after heavy exercise. What is the next step in management?
(A) Active exercise of the limb
(B) Anti-inflammatory drugs
(C) Thrombolytic therapy
(D) Antibiotics
(E) First-rib resection

3. Four days after a hysterectomy, a 30-year-old woman develops phlegmasia ce-


rulean dolens over the right lower extremity. What is the most appropriate treatment?
(A) Bed rest and elevation
(B) Systemic heparinization
(C) Venous thrombectomy
(D) Prophylactic vena caval filter
(E) Local urokinase infusion

4. Four days after undergoing subtotal gastrectomy for stomach cancer, a 58-
year-old woman complains of right leg and thigh pain, swelling and redness, and has
tenderness on examination. The diagnosis of deep vein thrombosis is suspected. What
is the initial test to establish the diagnosis?
(A) Venography
(B) Venous duplex ultrasound
(C) Impedance plethysmography
(D) Radio-labelled fibrinogen
(E) Assay of fibrin/fibrinogen products

5. A middle-age woman has right leg and foot non pitting edema associated with
dermatitis and hyperpigmentation. The diagnosis of chronic venous insufficiency is
made. What is the treatment of choice?
(A) Vein stripping
(B) Pressure-gradient stockings
(C) Skin grafting
(D) Perforator vein ligation
(E) Valvuloplasty

6. A middle-aged man known to have peptic ulcer disease is admitted with upper
gastrointestinal (GI) bleeding. During his hospital stay, he develops DVT of the left
lower extremity. What is the most appropriate management?
(A) Anticoagulation
(B) Observation
(C) Thrombolytic therapy
(D) Inferior vena cava (IVC) filter
(E) Venous thrombectomy

7. Four days after undergoing hysterectomy, a 30-year-old woman develops


phlegmasia cerulea dolens over the right lower extremity. What is the most appropri-
ate treatment?
(A) Bed rest and elevation
(B) Systemic heparinization
(C) Venous thrombectomy
(D) Prophylactic vena caval filter
(E) Local urokinase infusion
8. A 45-year-old woman undergoes cardiac catheterization through a right femo-
ral approach. Two months later, she complains of right lower extremity swelling and
notes the appearance of multiple varicosities. On examination, a bruit is heard over
the right groin. What is the most likely diagnosis?
(A) Femoral artery thrombosis
(B) Superficial venous insufficiency
(C) Arteriovenous (AV) fistula
(D) Pseudoaneurysm
(E) Deep vein insufficiency

9. All these venous systems are present on lower extremity EXCEPT:


A. Long saphenous veins
B. Short saphenous vein
C. Veins arising from calf perforators
D. Popliteal vein
E. Deep veins

10. Eleven years after undergoing right modified radical mastectomy, a 61-year-
old woman develops raised red and purple nodules over the right arm. What is the
most likely diagnosis?
(A) Lymphangitis
(B) Lymphedema
(C) Hyperkeratosis
(D) Metastatic breast cancer
(E) Lymphangiosarcoma

Keys for tests

1 2 3 4 5 6 7 8 9 10
C C B B B D C C D E

Tests for final level of knowledge, keys for tests:

1. Four days after undergoing subtotal gastrectomy for stomach cancer, a 58-
year-old woman complains of right leg and thigh pain, swelling and redness, and has
tenderness on examination. The diagnosis of deep vein thrombosis is suspected. What
is the initial test to establish the diagnosis?
(A) Venography
(B) Venous duplex ultrasound
(C) Impedance plethysmography
(D) Radio-labelled fibrinogen
(E) Assay of fibrin/fibrinogen products

2. A middle-aged man known to have peptic ulcer disease is admitted with upper
gastrointestinal (GI) bleeding. During his hospital stay, he develops DVT of the left
lower extremity. What is the most appropriate management?
(A) Anticoagulation
(B) Observation
(C) Thrombolytic therapy
(D) Inferior vena cava (IVC) filter
(E) Venous thrombectomy

3. What from the venous systems of lower extremity is not indicated on this picture?

(A) Superficial veins


(B) Deep veins
(C) Perforating veins
(D) Leonardo’s vein

4. A newborn girl with family history of lymphedema is noted to have bilateral


lower extremity swelling. What is the diagnosis?
(A) Secondary lymphedema
(B) Lymphedema praecox
(C) Milroy disease
(D) Lymphedema tarda
(E) Meigs’s syndrome

5. Lymphedema results from:


(A) Mechanical insufficiency due to injury to the lymph vessels or impairment
of lymph flow due to paralysis, blockage or inadequacy of lymph
(B) Dynamic insufficiency in which the lymph flow exceeds the transport capac-
ity of the intact lymphatic system
(C) Under the influence of infection factors
(D) Under the influence of intestinal helminths

6. A young basketball player develops an acute onset of subclavian vein throm-


bosis (effort thrombosis) after heavy exercise. What is the next step in management?
(A) Active exercise of the limb
(B) Anti-inflammatory drugs
(C) Thrombolytic therapy
(D) Antibiotics
(E) First-rib resection
7. An 18-year-old man develops a painful, swollen leg while training for the
New York Marathon. There is tenderness in the calf and ecchymosis is present. What
is the most likely diagnosis?
(A) Cellulitis
(B) DVT
(C) Superficial thrombophlebitis
(D) Tear of the plantaris muscle
(E) Medical lemniscus tear

8. The manifestations of decompensation with trophic changes of soft tissues


such as indurations, hyperpigmentation, hemosiderosis, and dermatitis of the skin are
typical for:
A. Chronic venous insufficiency of the I-st degree
B. Chronic venous insufficiency of II-nd degree
C. Chronic venous insufficiency of the III-rd degree
D. The decompensated stage of varicosity
E. The compensated stage of varicosity

9. The contraindications for surgical treatment of varicosity include all of the


following EXCEPT
F. Obstruction of deep veins,
G. Decompensative heart failure,
H. Diseases of liver and kidneys with considerable disturbances of function,
I. Obesity of III degree.
J. Decompensation of varicosity with incompetence of valves of superficial and
perforating veins

10. Tests for estimation of deep veins patency


A. Hackenbruch's test.
B. Troyanov-Trendelenburg's test.
C. Sheinis' test.
D. Delbet-Perthes' test
E. Talman's test

Keys for tests

1 2 3 4 5 6 7 8 9 10
B D B C AB C D B E D

Tasks for final level of knowledge

1. A 25-year-old woman presents to the emergency room complaining of red-


ness and pain in her right foot up to the level of the midcalf. She reports that her right
leg has been swollen for at least 15 years, but her left leg has been normal. On physi-
cal examination she has a temperature of 39C0 (102.2F). The left leg is normal. The
right leg is not tender, but it is swollen from the inguinal ligament down and there is
an obvious cellulitis of the right foot. What is the patient's problem?

The answer is Deep venous thrombosis.

2. The patients complain usually of a heavy sensation and fatigability of legs,


their swelling or edema, burning pain in the region of varicosity and night cramps of
tibial muscles. During examination of the patient in standing position it is possible to
note a considerable varicosity of superficial veins of the inferior extremities. The skin
of lower legs more often is unchanged. The functional examination of the valves re-
veals valvular incompetence of superficial or perforating veins. What is the stage of
disease?

The answer is The decompensated stage

3. The patients complain of a constant gravity in legs, pain, prompt fatigue,


edema and cramps of tibial muscles. This is associated with pigmentation, indurations
and trophic ulcer with localization in the lower third of leg. Large protruding veins
are common for these patients. At functional examination it is possible to determine
valvular incompetence of superficial, perforating and deep veins. What is the stage of
disease?

The answer is The decompensated stage

4. This thrombosis is accompanied by progressing edema of the upper limb


(more often right), increasing pain and cyanosis of skin. The edema usually starts
from the hand and spreads upward reaches the neck. Veins of the extremity are over-
flowed, intense. In axillary fossa a dense cord (a thrombosed vein) is palpated.

The answer is Paget-Schrotter syndrome

5. The thrombosis of this location is extremely dangerous and without surgical


intervention most often results in a lethal outcome. Clinically it manifests by a back
pain, oliguria, anuria and vomiting. Microhematuria and proteinuria in urine is re-
vealed. The signs of renal failure are evident. The contents of urea, creatinine and po-
tassium in a blood plasma gradually elevate. In 2-3 days the uremia with the lethal
outcome develops. Very rarely the phenomena of uremia decreases, anuria overcomes
into polyuria and the patient recovers.

The answer is Thrombosis of renal veins

Materials for the self-study of the students

Main tasks Notes (instructions)


Repeat: 1. Represent methods of diagnostics
12. Normal Anatomy of the lower of Veins as pumping chambers.
extremity venous system 2. Causes of oedema in lower limbs.
13. Normal Venous Physiology. 3. Choose and write methods of
14. Pathophysiology and etiology conservative therapy of Trophic ulcer.
of the chronic venous disease.
15. Special investigations in pa-
tients with venous disease.
Causes of oedema in lower limbs.
Study: 8. Represent methods of diagnostics
7. Clinical symptoms and signs of venous of venous disease.
disease. 9. To make a differential diagnos-
8. Syndromes of valve deficiency and tics of veins diseases.
weak veins 10. Surgical technique in treatment of
9. Clinical examination and special inves- superficial vein incompetence.
tigations in patients with venous disease
10.Compression sclerotherapy in patients
with venous disease.
Principles of surgical treatment of venous
disease.
Study guide #19.2
“Postphlebitic syndrome of lower extremities (postthrombotic disease).”

Owerviev
Disorders of the vascular system can broadly be classified, in anatomic terms,
into arterial, venous, or lymphatic diseases. Although there is undoubtedly overlap
of these etiologies in some patients, for the most part, a clear understanding of the
symptoms, signs, clinical presentation, as well as the history, can usually classify
the problem discretely into one of those categories. The focus of this chapter is on
disorders of the venous system, which affect, according to some estimates, 40% of
the U.S. population. The significance of venous disease, in terms of scope, cost, and
implications, is not appreciated by most physicians since it is scarcely life or limb
threatening, except for the notable exception of pulmonary embolism. Disorders of
the venous system can be divided into thrombotic or thromboembolic disease and
venous insufficiency. Thrombotic disease of the veins can and does frequently lead
to venous insufficiency, the consequences of which are quite disabling. Another
factor in truly assessing the prevalence of venous disease, in particular venous in-
sufficiency, is that the range of venous insufficiency can span a vast array of mani-
festations, from mildly symptomatic varicose veins to severe chronic venous insuf-
ficiency (CVI) with ulceration. For accurate prevalence data, the essential require-
ment is that a uniform classification scheme be used, and,more important, the suc-
cess of various therapeutic options can be properly gauged only if the clinician is
knowledgeable about accurately classifying the disease, almost analogous to stag-
ing systems in oncology.

Educational aims:
1) Anatomo-physiological information about main veins, and lymphatic vessels.
2) Classification of postthrombotic illness.
3) Clinical picture of postthrombotic syndrome of lower extremities, and
lymphedema of extremities.
4) Methods of inspection of patients.
5) On material of theme to develop the sense of responsibility for the timeliness ex-
posure of disease and correct professional actions for achieving a medical effect.
6) Treatment program (conservative and surgical treatment) of patients with the
postthrombotic syndrome of lower extremities, and lymphedema of extremities.
7) The question of rehabilitations and health centre systems of patients with the
postthrombotic syndrome of lower extremities and lymphedema of extremities.
Student must know:
47. Аnatomo-physiological information about deep veins and organs of lower ex-
tremity
48.Classification of postthrombotic disease
49.Mechanism of postthrombotic disease
50.Clinical picture of postthrombotic disease
51.Methods of diagnostics of postthrombotic disease
52.Principles of treatment of postthrombotic disease.
53.Medical treatment of post-thrombotic disease
54.Features of surgical treatment are at the different form of postthrombotic disease
55.Technique of compression therapy.
Student be able to:

1) Collect and estimate information of anamnesis for patients with the post-thrombtic
syndrome of lower extremities and lymphedema of extremities.
2) Form the picture of role of ecological, social and biological factors in an origin and
development of postthrombotic syndrome of lower extremities
3) Conduct a system observation and make previous conclusions in relation to a diag-
nosis.
4) Appoint additional examinations.
5) Appoint conservative treatment and conduct a dynamic supervision after a patient.
6) Timely to define indications to operative intervention for patients with the
postthrombotic syndrome of lower extremities and lymphedema of extremities.
7) Define the adequate volume of operative intervention.
8) Provide the valuable care of patient after an operation.
Terminology

Term Definition
Postthrombotic dis- is a chronic disease of inferior extremities, which de-
ease velops due to a deep vein thrombosis and manifests by
the expressed edema, secondary varicosity of superfi-
cial veins and recurrent thrombosis.
inadequate drainage of venous blood from a part, re-
Chronic venous sulting in edema, dermatosis it occurs more in leg veins
insufficiency (CVI) because of partial vein blockage or blood leakage
around the valves of the veins
obstruction or closure of vessel or passageway in state
Oclussion
of being occluded
restoration of lumen in a blood vessel following
Recanalization thrombotic occlusion, by organization of thrombus
with formation of new channels
in recumbency, at a straight leg the maximal dorsal
Gomans test bending of foot is executed. If there is pain in a galf
muscle – a test is considered positive.
this procedure involves making an incision down to
Linton operation and including the fascia, reflecting it forwards, identi-
fying the perforating veins, and ligating them.
perforating veins can be interrupted either by a direct
SEPS surgical approach or indirectly with subfascial endo-
scopic perforator vein surgery
the bypass, from a great saphenous vein of a healthy
Palma operation extremity is formed, which connects the distal end of
the occluded segment.
by ligating the superficial femoral vein and diverting
blood from the superficial femoral vein via the saphe-
May–Husni operation
nous vein with competent valves, effects of deep ve-
nous insufficiency can be offset.

Content:
Thrombosis in the deep veins of the lower limb is a common complication of se-
rious illness, pregnancy, following surgical operations and after severe injuries, es-
pecially with fractures of the lower limb and pelvis. It may be localized to small areas
or extend massively throughout both lower limbs. In many instances there are no
signs or symptoms, the first manifestation of deep venous thrombosis being a pulmo-
nary embolism. With extensive iliofemoral deep venous thrombosis the classical
signs of deep vein thrombosis are more likely to occur. These signs include swelling
of the leg, dilatation of the superficial veins, warmth, pain and tenderness, often asso-
ciated with a low-grade pyrexia. Following a deep vein thrombosis the body's own
fibrolytic system will attempt to remove the clot. Over a period of time, dissolution of
the clot occurs, but there is a failure to restore valvular function and inevitable scar-
ring and stenosis of the deep veins. Although some patients make a complete recov-
ery from a deep venous thrombosis, others have marked changes in the deep vein giv-
ing rise to severe deep venous insufficiency.
In patients with severe deep venous changes, there is a failure of the calf muscle
pump, which results in the development of post-thrombotic changes. The post-
thrombotic changes result in sustained high venous pressure unrelieved by exercise.
This causes venous congestion, oedema, pigmentation and induration of the superfi-
cial tissues, lipodermatosclerosis, and eventually ulceration near the ankle. The long-
term consequences of deep vein thrombosis are known as the post-thrombotic or the
post-phlebitic syndrome. Although this syndrome is a common cause for chronic ve-
nous insufficiency and venous ulceration it must be remembered that gross incompe-
tence of the superficial veins alone, curable by surgery, can produce the same signs.
Etiology and pathogenesis
The cause of postphlebitic syndrome is the acute thrombosis of major veins of the
system of inferior vena cava. At once after thrombus formation under the influence of
the factors of a blood coagulating system there comes its retraction. Simultaneously
under the influence of fibrinolysis a spontaneous lysis of thrombus occurs. Depending
on activity of these systems there could be such further course of the disease: in one
cases a complete recanalization of the thrombus take place; and in others - complete
obliteration or partial patency of the vessel could be restored. Nevertheless, despite
the character, in this or that degree it always leads to incompetence of a valvular sys-
tem of affected veins. It also causes a venous hypertension more distal to the patho-
logical process.
Pathomorphology
In larger veins a considerably expressed sclerotic changes with the involvement of
valves in the process are observed. Histologically revealed a sclerosis of all venous
layers with the atrophy and necrobiosis of a functionally active elements (smooth
muscles and elastic membranes). The intima of the such vessels is sharply and uneven
thickened. Thus both sites of hypertrophy and zones of a sharp thinning up to the dis-
appearance of structural elements take place. Also revealed the lack of the valves in
such veins.
Classification:

1. According to the form of disease:


- Sclerotic;
- Varicose;
- Edematous;
- Ulcerative.
2. According to localization of the lesion:
- Inferior vena cava;
- Iliac vein;
- Femoral vein;
- Popliteal vein;
- Tibial veins.
3. According to the spread o/ the Lesion:
- Isolated;
- Combined;
- Wide-spread.
4. According to the character of the lesion:
- Occlusion;
- Partial recanalization;
- Complete recanalization.
5. According to the degree of a venous insufficiency:
- Compensation;
- Subcompensation;
- Decompensation.

Symptomatology and clinical course

The postphlebitic syndrome clinically manifests by a dull ache in the affected ex-
tremity, expressed edema and secondary varicosity. In the course of disease with the
change of stages a lot of new signs, such as induration of tissues, hyperpigmentation
of the skin, stasis dermatitis and recurrent trophic ulcers may occur. The general state
of the patient usually remains satisfactory. The clinical manifestations basically de-
pend on pathological changes in the venous system. Due to periphlebitis all the vas-
cular-nervous fascicle in the region of thrombosis is involved in cicatrical tissues. It
also causes the edema on legs, which is mostly expressed after prolonged standing of
the patient or in the evening. Some manifestations of postphlebitic syndrome caused
by venous insufficiency (arching pain, heaviness in legs) after recumbence or eleva-
tion of legs may considerably decrease or disappear.
The varicose veins is commonly not a permanent sign of postphlebitic syndrome.
As a rule, it develops in the system of great saphenous vein, particularly in sites of
perforating veins. Sometimes the dilation of superficial veins of inferior part of ante-
rior abdominal wall is observed. Its degree frequently directly depends on the level of
localization of pathological process. The most discomfort manifestation of postphle-
bitic disease for the patients is necessary to consider trophic ulcers. They are formed
mostly in the lower third of the leg, on its inner surface. The skin of stasis area is cy-
anotic or hyperpigmentative, a subcutaneous fat is firmed. The ulcer may be of vari-
ous size, always forms on the background of edema, slowly heals and has predisposi-
tion to recurrences. Sometimes it circulatory covers the inferior third of leg.
Despite that the pain sensation in affected extremity is a permanent sign of the
disease, its intensity is almost always unmeasureable. Pain syndrome is the most ex-
pressed in phase of occlusion. In majority of patients with a posphflebitic syndrome
the pain increases after prolonged standing or in the evening.

Heaviness in legs and dull ache caused by the venous stasis are localized, at
first, in the distal parts of affected extremity. The degree of expressiveness of
these signs always directly depends on the degree of decompensation of the
venous outflow. Posthrombotic syndrome is characterized by a stage course of the
disease.
I stage manifests by edema of legs without the phenomena of skin hyperpigmen-
tation and induration of a subcutaneous fat. In these patients it is possible to note a
cyanosis of the skin. After recumbence or night rest the edema completely disappears.
II stage of the disease is characterized by the edema, which decreases in 24 hours
of staying in bed. It might be connected with initial disturbances of a lymph flow.
Further there may be a slight hyperpigmentation of the skin and indurative changes of
a subcutaneous fat.
III stage is accompanied by decompensation of a lymph outflow from the ex-
tremity and transformation of a venous edema in a lymphovenous. It results in the de-
velopment of a subcutaneous fibrosis, which extends around inferior third of the leg.
Variants of clinical course and complications

Sclerotic form. Characteristic for this form is the absence of varicose veins of the
extremity in I stage of the disease and moderate dilation of the tributaries of a super-
ficial veins in the sites of location of perforating veins in II and III stages. The main
trunks of a great and small saphenous veins are without the signs of pathological dila-
tion. The induration of a subcutaneous fat and hyperpigmentation of the leg more ex-
pressed in comparison with the other forms of postphlebitic disease. In sclerotic form
there is a so-called subcutaneous fibrosis of the fat, which as a rule, is localized in in-
ferior third of the leg.
Varicose form. The skin of legs in patients is of usual colour, the edema ex-
pressed insignificantly, but the superficial veins are varicose changed. Their localiza-
tion usually assigns the level of venous lesion. So, the involvement in the process of
iliofemoral segment manifests by the simultaneous varicosity of a great and small sa-
phenous veins and incompetence of perforating veins of lower leg. In complete oc-
clusion of a major veins of pelvis or restricted occlusion of iliac vein the varicosity of
superficial veins is localized mainly in the upper third of thigh and lower part of ante-
rior abdominal wall. In occlusion of the distal parts of inferior vena cava the varicose
veins may observed on both legs and lateral surface of the abdomen and chest.
Edematous form. This form develops at once after the acute phenomena of a
deep venous thrombosis and is characterized by pain, edema am moderate cyanosis.
Pain is usually of segmental character, and localized along the nerves, vascular
fascicle of the leg and thigh. Nevertheless, if the patient is recumbent with elevated
limb, the pain and heaviness in affected extremity gradually disappears. The degree
of the edema directly depends on the severity of hemodynamic changes in larger
veins. Thus in restricted lesion of a femoral and popliteal segment, a moderate en-
largement of the leg and small edema of the lower third of thigh is observed. But the
spread of the process on the iliofemoral segment sharply enlarges the volume of en-
tire extremity, and results in edema on buttocks. In complete or restricted occlusion
of pelvic veins also detected a diffuse edema of the extremity. In case of the lesion of
distal parts of inferior vena cava the extremely expressed edema of both extremities
up to elephantiasis is observed. But in this form of the disease the varicosity is ab-
sent.
In satisfactory compensation of a venous outflow the edematous form of the dis-
ease sometimes disappears in several months. In a few patients the compensation of
venous outflow is so well developed, that there is no visual base for establishing of
the diagnosis of postflebitic disease, though in deep veins there is a segmental oblite-
ration.
Ulcerative form. In patients with this form of the disease it is possible to find out
all named above signs, which are rather expressed. The edema of leg, even after pro-
longed stay of the extremity at rest, does not disappear completely; the degree and
volume of varicose veins is enlarged, the hyperpigmentation and infiltration of a sub-
cutaneous fat with diffuse spreading on entire inferior half of leg occurs. The trophic
ulcer, which usually formed on a medial surface of inferior third of the leg, is accom-
panied by itching and trophic changes of skin and subcutaneous fat.
The diagnostic program
1. Anamnesis and physical examination.
2. Functional tests for the definition of valvular incompetence of
superficial, deep and of perforating veins.
3. General blood and urine analyses.
4.Coagulogram.
5.Sonography of vessels and dopplerography.
6.Phlebography.
Venous ulceration
Before concluding that an ulcer on the lower limb is venous, other causes of ul-
ceration need to be excluded. Some ulcers may have a mixed aetiological back-
ground, including an arterial and a venous component. Although there are many
causes of ulceration of the lower leg, most ulcers are venous, arterial or diabetic. Ex-
clusion of diabetic and arterial ulcers probably indicates an underlying venous cause.
Venous ulceration is due to sustained venous pressure that does not reduce on exer-
cise. Failure to reduce venous pressure on exercise indicates a failure of the calf mus-
cle pump, either due to direct damage to the deep valves or due to incompetence feed-
ing the superficial system of veins. Venous ulceration can be due to deep venous in-
sufficiency, superficial insufficiency, or a combination of both. Superficial insuffi-
ciency responds to surgical treatment and in those patients with superficial insuffi-
ciency alone, surgical treatment of the superficial veins will result in rapid ulcer heal-
ing and no recurrence. In patients with mixed superficial and deep venous insuffi-
ciency, improvement will occur with early ulcer healing, although the prognosis re-
mains guarded. In those patients with deep vein problems the ulcers remain difficult
to heal, with no realistic prospect of surgical intervention. As a result of maintained
venous pressure and a reduced pressure gradient across the capillary bed, white-cell
trapping and aggregation occurs. The white cells interact with the capillary endotheli-
um, resulting in their activation. Activation and the release of free radicals results in
local tissue destruction and ulceration. A number of markers of white-cell activation
(including CD11B) are elevated in patients with chronic venous insufficiency. It is
the venous stagnation and extravasation of cells, the breakdown of red blood cells and
the deposition of fibrin that lead to the classical changes of pigmentation (lipoder-
matosclerosis), leading to ulceration.
Management
It is important to determine the cause of the ulcer, excluding arterial and diabet-
ic causes. Having determined that an ulcer is venous, those patients with superficial
venous insufficiency should be identified and treated surgically. Those with primarily
deep venous insufficiency or a combination of deep and superficial venous insuffi-
ciency are treated by surgical debridement and elastic compression. Superficial infec-
tion results in pain. Effective debridement, with antibiotics reserved for those patients
with a spreading cellulitis, should be employed. Compression can be applied by
bandaging the limb or by the application of compression stockings. Bandaging using
four-layer techniques to achieve good levels of compression results in ulcer healing
between 6 and 12 weeks. Rarely is it necessary to admit a patient to hospital to un-
dergo surgical debridement and split-skin grafting. Although split-skin grafting is ef-
fective in reducing the area of ulceration and on occasions achieving complete heal-
ing, the underlying pathophysiology remains unchanged and recurrence is inevitable.
Conservative therapy is applied in: a) a grave state of the patient caused by
concomitant diseases; b) in edematous form of the disease; c) in expressed incompe-
tence of the venous outflow.
The following agents should be included into a complex of conservative treat-
ment:
- Direct (Heparin), and indirect anticoagulants (Phenilin, Pelentan);
- Antiaggregants (Aspirin, Curantyl, Rheopolyglucin);
- The agents, which improve the microcirculation (Niacin, Xantinol, Trental, Sermi-
on);
- The agents, which raise the tonus of a venous wall (Indomethacin,
Methindolum, Troxevasin, Venoruton);
- Antiinflammatory therapy (Antibiotics, Nonsteroid antiinflammatory agents).
The presence of trophic ulcer requires:
a) Sanation of the surface of trophic ulcer: bactericidal and bacteriostatic agents of a
local action (Furacilin, Polymyxin, UVR, Photoradiotherapy);
b) Stimulation of regenerative processes in the wound by Methyluracil, Lorinden,
Flucinar).
During conservative therapy it is always necessary to take into account, that the
patients who take anticoagulants and fibrinolytic agents, should be under the dynamic
monitoring of a blood coagulating system.
The special attention is paid to the application of elastic bandage and organiza-
tion of a rational regimen of physical exertion. The appropriate regimen of workload
manifests by decreasing of edema of the extremity.
For maintaining of obtained effect it is desirable to use the sanatorium treat-
ment.
Taking into account, that the clinical course of the disease always has the pro-
gressing character, by method of choice should be the surgical treatment. The re-
canalization of thrombosed veins and the development of collateral blood flow usual-
ly ends in 6-8 months after the acute thrombosis. This period, as a rule, determined
by the form of the disease. Therefore the surgical treatment for such patient should be
applied after 6-8 months from the beginning of the disease. The purpose of such op-
erative approach is the partial or complete liquidation of incompetence of venous
flow in deep veins.
In I stage of postphlebitic disease the operative treatment is rarely applied. In
sclerotic form of this stage due to the absence of veins that are suitable for bypass
grafting, the surgical treatment is impossible, nevertheless in the varicose form there
is actual opportunity for performance of operation. For this purpose is used a great
saphenous vein of a healthy extremity. The latter is exposed from saphenofemoral
junction down to inferior third of thigh. The distal end of the vein is ligated, and the
proximal one is provided above pubis and anastomosed with femoral vein below the
site of its occlusion.
It is necessary to mean, that the postthrombotic occlusion of femoral vein in I
stage of the disease is the contraindication for removing of a great saphenous vein of
the affected extremity. Nevertheless the operation of cross-bypass may be applied by
means of great saphenous vein of healthy extremity.

The II stage of the development of postphlebitic disease usually requires the


removing of dilated veins, and ligating of perforating veins. In this stage there is an
actual possibility of a wide application of plastic and reconstructive operations.

In III stage of the disease the surgical treatment is indicated even more often,
nevertheless in this situation it is much more hardly to receive satisfactory results be-
cause of the pathological changes of lymphatic system and subcutaneous fat.

Following venous thrombosis there may be complete resolution, leaving valvu-


lar damage scarring and stenosis of the veins, or resolution may be partial, leaving a
complete obstruction to the more proximal deep veins.

Surgical treatment to replace damaged valves has not proved successful when
the valvular damage is related to thrombosis. In primary valvular insufficiency, valve
repair procedures have been described and shown to be effective (Kistner operation).
Surgical bypass procedures designed to overcome occlusions or divert blood from
the deep veins through competent superficial veins are more effective.
In patients with iliofemoral thrombosis and complete occlusion, blood from the
deep veins can be diverted into the deep veins of the opposite leg by routing the long
saphenous vein from the opposite leg, suprapubically, and then anastomosing it into
the common femoral vein on the affected side. As an alternative to the saphenous
vein a polytetrafluoroethylene, externally supported, vascular graft can be used.
The main goal of the operative treatment is to improve a venous hemodynamic
by means of creating additional pathways of the outflow. The Palma's operation is
performed in segmental lesion of iliofemoral segment of the venous system. The by-
pass, from a great saphenous vein of a healthy extremity is formed, which connects
the distal end of the occluded segment.
In restricted occlusion of the iliofemoral segment the anastomosis of the poplit-
eal vein with a great saphenous vein of the thigh is formed.
Popliteal femoral vein bypass (May–Husni operation. By ligating the superfi-
cial femoral vein and diverting blood from the superficial femoral vein via the saphe-
nous vein with competent valves, effects of deep venous insufficiency can be offset.
The operations, which improve the blood flow in deep veins include thrombin-
timectomy, plastics of a deep fascia of the leg (Ascor's operation). The Ascor's opera-
tion is performed as follows: a deep fascia of a back surface of the leg is cut by a lon-
gitudin al incision and sutured as duplicature.
There are lot of operations, which goal is to form the artificial valves in veins.
The most popular are: the creation of the valves by means of a fold suturing of a ve-
nous wall inside the lumen; and by invaginating of a tributary stump inside the vein.
For the extravascular correction of the valvular incompetence the lavsan spirals
are applied, which after the arrangement on the vein narrow the walls of incompetent
valve and compensate its function.
In order to make external muscle valve a tendon of gentle muscle of thigh is
provided in the transverse direction between popliteal artery and vein and sutured to
the tendon of biceps muscle of a thigh (Psattakis operation).
Also possible the replacement of the segment of a valvular incom-
petence recanalized femoral vein by the graft of a great saphenous vein with func-
tioning valve.
Operations, which reduce hydrostatic pressure in different levels of a venous
system of inferior extremity:
- resection of popliteal vein;
- resection of femoral vein;
- resection of posterior tibial vein;
Perforating veins connect the superficial to the deep veins. The role of these
veins is to convey blood from the superficial to the deep system. In patients with su-
perficial venous insufficiency the perforator veins may enlarge to transfer more blood
from the superficial to the deep system. They only become incompetent when blood
is transferred from the deep to the superficial system.
Some perforating veins have valves to prevent flow from deep to superficial;
other perforating veins do not appear to have valves but remain competent due to
muscular activity.
The role of a perforating vein in ulceration remains controversial. Perforating
veins can become incompetent following deep venous thrombosis and perforating
veins are often seen in association with ulceration. The transmission of high deep ve-
nous pressure to the subcutaneous tissues is undoubtedly an important factor in the
changes that take place in the skin and subcutaneous tissues resulting in skin changes
followed by ulceration.
The perforators can now be identified using Duplex ultrasound imaging. Perfo-
rating veins can be interrupted either by a direct surgical approach or indirectly with
subfascial endoscopic perforator vein surgery (SEPS). The direct approach to perfo-
rating veins using the posterior or posteromedial incision described by Dodd and Lin-
ton can rarely be justified. This procedure involves making an incision down to and
including the fascia, reflecting it forwards, identifying the perforating veins, and li-
gating them.
The problems with wound healing and extensive scarring have made this an
unpopular surgical procedure with little evidence that it results in maintained ulcer
healing. More recently, an endoscopic technique using an endoscope to enter the sub-
fascial space to identify the perforator veins and direct surgical ligation has been de-
veloped. Studies are being undertaken to assess the efficacy of this procedure in terms
of ulcer healing and recurrence. The technique involves making a small incision be-
neath the knee, inserting an endoscope, and creating a space using gas, saline or,
more recently, inflatable devices. An inflatable balloon placed in this space when in-
flated would create a space through which the perforating veins can be seen to trav-
erse. These veins are then ligated under direct vision with clips, the balloon and endo-
scope retrieved, and the small scar in normal skin closed.
Basic literature:
75.Oxford Textbook of Surgery (3-Volume Set) 2nd edition (January 15, 2000):
by Peter J. Morris (Editor), William C. Wood (Editor) By Oxford Press
76.Sabiston Textbook of Surgery 17th edition by Courtney M. Townsend Jr.,
Kenneth L. Mattox, B. Mark, MD Evers, Kenneth L., MD Mattox, Courtney
Townsend, Daniel Beauchamp, B. Mark Evers, Kenneth Mattox W.B. Saun-
ders Company (June, 2004)
77.Schwartz´s Principles of Surgery 8th Edition F.Charles Brunicardi. Copyright
©2007 The McGraw-Hill Companies.
78.Hospital surgery/ Edited by L.Kovalchuk et al.- Ternopil: Ukrmedknyha,
2004.- 472 p.
Additional literature:
1. Bradbury AW, Murie JA, Ruckley CV. Role of the leucocyte in the pathogene-
sis of vascular disease. British Journal of Surgery 1993; 80: 1503–12.
2. Nicolaides AN, Sumner DS. Investigation of patients with deep vein throm-
bosis and chronic venous insufficiency. Med-Orion, London, 1991.
3. Tibbs DJ, Scurr JH, Sabiston DC, Davies MG, Mortimer PS. Varicose veins,
venous disorders, and lymphatic problems in the lower limbs. Oxford Universi-
ty Press, Oxford, 1997.
4. MacKenzie RK, Paisley A, Allan PL, et al: The effect of long saphenous vein
stripping on quality of life. J Vasc Surg 35:1197, 2002. [PMID: 12042731]

Tests for initial level of knowledge, keys for tests:

1. Indirect anticoagulants demonstrate its effect after administration


1. 12-48 hours.
2. After 4-hr.
3. After 72-hr.
4. After 56-hr.

2. Search for nonpharmacological methods of correction coagulation system.


A. Haemodilution.
B. Haemodialysis.
C. Haemosorption.
D. Artificial [forced] diuresis.
E. Lymphosorbtion.

3. Indirect anticoagulants action unclude:


A. Drugs of group alkaloid
B. Fibrinolytics drugs
C. Drugs of group 4-oxykrinin, phenylindonrian
D. Thrombolytic drugs

4. Absolute contraindications for the administration anticoagulant stipulates:


A. Bleeding any localization, hemorrhagic diathesis
B. Aplastic anemia
C. Respiratory compromise
D. Cardiac decompensation
E. Cerebral circulatory [cerebrovascular] insufficiency

5. To activator fibrinolysin refer:


A. Fenilin, omefin
B. Gastrocepin, venter
C. Nicotinic acid, complamin
D. Group B-vitamins

6. Preparations, which inhibit antiaggregatory effect:


A. Reopoliglucin, trental, aspirin, curantil
B. Omefin, fenilin, pelentan
C. Cinarisin, sermion, solcoseril
D. Fentolamin, thropaphen

7. Preparations thrombolytic therapy:


A. Corglicon, strophanthin
B. Celanid, lantosid
C. Fibrinolisin, celiasa, streptokinase
D. Reopolyglucin, polyglucin

8. Methods of control of coagulation and fibrinolytic systems:


A. Coagulogram, thromboelastogram
B. Determination of local speed of blood flow
C. Haemogram
D. Biochemical analysis of blood

9. When bleeding caused by anticoagulants of indirect action it is necessary to inject:


A. Pipolfen
B. Vikasol
C. Digitoxin
D. Hydrocortyson

10. In fibrinolytic bleeding appoint:


A. Hydrokortyzon, prednizolon
B. 0.1% Nicotinic acid solution
C. 1% Amben solution, 5% aminocaproic acid solution
D. 5% Ascorbic acid solution

Keys for tests

1 2 3 4 5 6 7 8 9 10
A A C A C A C A B C

Tests for final level of knowledge, keys for tests:

1. The operation of Palm-Experon is executed for:


A. Creation of artificial valves
B. Improvement of haemodynamic.
C. Hyperfunction of adrenals.
D. Bandaging of veins.
E. Electrocoagulation of veins.

2. Thrombosis of deep veins system leads to:


A. Post-thrombophlebetic syndrome.
B. Thrombophlebitis.
C. Anuria.
D. Cardiomegalya.

3. The operation of choice in patients with insufficiency of perforating veins leg is:
A. Linton, Cocket
B. Bebcock, Narat
C. Troyanov-Trendelenburg
D. Madelung

4. A male 40-years old had 3 ears ago the thrombosis of deep veins of femoral
segment on the right. Now he is overweight, swelling of the right lower extremity.At
an observation there was a moderate edema of shin, bore of induration of skin on
lower third of shin, superficial varicose of shin. What is the possible diagnosis?

A. Post-thrombophlebetic syndrome, varicose form


B. Acute thrombophlebitis of veins of right lower extremity
C. Chronic venous insufficiency
D. Thrombosis of deep veins of lower extremities
E. Gangrene of right lower extremity
79.Patients with phlebographically confirmed deep vein thrombosis of the calf

A. Can expect asymptomatic recovery if treated promptly with anticoagulants


B. May be effectively treated with lowdose heparin
C. May be effectively treated with pneumatic compression stockings
D. May be effectively treated with acetylsalicylic acid
E. Are at risk for significant pulmonary embolism

6. A 49 year old patient complains about pain, feeling of heavyness in the left
lower extremity, presence of trophic ulcer on the internal surface of lower third of the
left shin. Pulsation of arteries of thigh and popliteal is preserved, but on feet –it is
weakened. What is the possible diagnosis?

A. Chronic venous insufficiency


B. Acute thrombosis
C. Post-thrombophlebetic syndrome of legs of II-B degree
D. Thrombosis of deep veins of lower extremities
E. Gangrene of lower extremity

7. Characteristic for this form is the absence of varicose veins of the extremity
in I stage of the disease and moderate dilation of the tributaries of a superficial veins
in the sites of location of perforating veins in II and III stages. The main trunks of a
great and small saphenous veins are without the signs of pathological dilation. The
induration of a subcutaneous fat and hyperpigmentation of the leg more expressed in
comparison with the other forms of postphebitic disease. Subcutaneous fibrosis of the
fat, which as a rule, is localized in inferior third of the leg. What is it?
A. Sclerotic form of postphlebitic syndrome
B. Varicose form of postphlebitic syndrome
C. Edematous form of postphlebitic syndrome
D. Ulcerative form of postphlebitic syndrome

8. The skin of legs in patient is usual colour, the edema expressed insignificant-
ly, but the superficial veins are varicosed. Their localization assigns the level of ve-
nous lesion. The involvement in the process of iliofemoral segment manifests by the
simultaneous varicosity of great and small saphenous veins and incompetence of per-
forating veins of lower leg. What is that?
A. Sclerotic form of postphlebitic syndrome
B. Varicose form of postphlebitic syndrome
C. Edematous form of postphlebitic syndrome
D. Ulcerative form of postphlebitic syndrome

9. This form develops at once after the acute phenomena of a deep venous
thrombosis and is characterized by pain, edema and moderate cyanosis. Pain has
segmental character, and localized along the nerves, vascular fascicle of the leg and
thigh.The degree of the edema directly depends on the severity of hemodynamic
changes in larger veins. In afffected lesion of femoral and popliteal segment, moder-
ate enlargement of the leg and small edema of the lower third of thigh is observed.
What is that?
Sclerotic form of postphlebitic syndrome
Varicose form of postphlebitic syndrome
Edematousform of postphlebitic syndrome
Ulcerative form of postphlebitic syndrome.

56.Which statement regarding contrast venography is true?

A. It is more accurate than Doppler analysis and B-mode ultrasound (duplex scan)

at detecting thrombi in the deep veins responsible for pulmonary emboli

B. It identifies incompetent deep, superficial, and perforating veins


C. It is totally noninvasive, painless, and safe
D. It is easily performed in a vascular laboratory or radiology suite or at the
bedside
E. It is particularly sensitive in identifying the proximal extent of an iliofemo-
ral thrombus

Keys for tests

1 2 3 4 5 6 7 8 9 10
B A A A E C A B C B

Tasks for final level of knowledge

1. In patient with this form of the disease it is possible to find out all named
above signs, which are rather expressed. Edema of leg, even after prolonged stay of
the extremity at rest, does not disappear completely, the degree and volume of vari-
cose veins is enlarged, the hyperpigmentation and infiltration of a subcutaneous fat
with diffuse spreading into entire inferior half of leg occurs. The trophic ulcer, which
usually formed on medial surface of inferior third of the leg, is accompanied by itch-
ing and trophic changes of skin and subcutaneous fat. What is that?

The answer is ulcerative form of postphlebitic syndrome

2. A 50 years old patient has a pain in the left lower extremity, which increases at
the physical loading, he complains about holding apart, slight swelling in the area of
shin and foot. On skin inspection lower part of shin was indurated, bronze tint, hypo-
dermic veins are extended, there is an ulcer with necrotic masses. What is the possi-
ble diagnosis?

The answer is postphlebitic syndrome

3. The female 47 old years patient complains in the presence of edema on a foot,
feeling of weight, holding apart in feet, rapid fatigue during standing and walking,
which disappear in position lying. Objectively: expansion of superficial veins of the
left shin and thigh is with an edema, pigmentation and trophic disorders of skin, in
n/3 shins. Formulate a previous diagnosis:
The answer is postphlebitic syndrome.

4. The female 49 old years patient grumbles about feeling of weight, holding apart,
in feet, rapid fatigue during standing and walking, which disappear in position, lying.
Objectively: expansion of superficial veins of the left shin and thigh is with pigmenta-
tion and trophic disorders of skin. From what functional test it is needed to begin in-
spection of skin?

The answer is Trojanov Trendelenburg test.

5. A victim, 25 years, days ago got the gun wound of soft fabrics of the left shin. A
pressing bandage was imposed, to the doctor did not apply. The state became worse.
Complaints about holding apart pain in lower extremity, sharp oedema of lower ex-
tremity. Temperature of body - 39°S, propulsion tension is 80/50 mm of rt.st.. What is
diagnosis?

The answer is gas gangrene.

You might also like